07.09.2016 Views

Engineering Economy 2012

Create successful ePaper yourself

Turn your PDF publications into a flip-book with our unique Google optimized e-Paper software.

Seventh Edition<br />

ENGINEERING<br />

ECONOMY


Seventh Edition<br />

ENGINEERING<br />

ECONOMY<br />

Leland Blank , P. E.<br />

Texas A & M University<br />

American University of Sharjah, United Arab Emirates<br />

Anthony Tarquin , P. E.<br />

University of Texas at El Paso<br />

TM


TM<br />

ENGINEERING ECONOMY: SEVENTH EDITION<br />

Published by McGraw-Hill, a business unit of The McGraw-Hill Companies, Inc., 1221 Avenue of the Americas, New<br />

York, NY 10020. Copyright © <strong>2012</strong> by The McGraw-Hill Companies, Inc. All rights reserved. Previous editions<br />

© 2005, 2002, and 1998. No part of this publication may be reproduced or distributed in any form or by any means, or<br />

stored in a database or retrieval system, without the prior written consent of The McGraw-Hill Companies, Inc.,<br />

including, but not limited to, in any network or other electronic storage or transmission, or broadcast for distance<br />

learning.<br />

Some ancillaries, including electronic and print components, may not be available to customers outside the United<br />

States.<br />

This book is printed on recycled, acid-free paper containing 10% postconsumer waste.<br />

1 2 3 4 5 6 7 8 9 0 QDB/QDB 1 0 9 8 7 6 5 4 3 2 1<br />

ISBN 978-0-07-337630-1<br />

MHID 0-07-337630-2<br />

Vice President & Editor-in-Chief: Marty Lange<br />

Vice President EDP/Central Publishing Services: Kimberly Meriwether David<br />

Global Publisher: Raghothaman Srinivasan<br />

Sponsoring Editor: Peter E. Massar<br />

Senior Marketing Manager: Curt Reynolds<br />

Development Editor: Lorraine K. Buczek<br />

Senior Project Manager: Jane Mohr<br />

Design Coordinator: Brenda A. Rolwes<br />

Cover Designer: Studio Montage, St. Louis, Missouri<br />

Cover Image: © Brand X Pictures/PunchStock RF<br />

Buyer: Kara Kudronowicz<br />

Media Project Manager: Balaji Sundararaman<br />

Compositor: MPS Limited, a Macmillan Company<br />

Typeface: 10/12 Times<br />

Printer: Quad/Graphics-Dubuque<br />

All credits appearing on page or at the end of the book are considered to be an extension of the copyright page.<br />

Library of Congress Cataloging-in-Publication Data<br />

Blank, Leland T.<br />

<strong>Engineering</strong> economy / Leland Blank, Anthony Tarquin. — 7th ed.<br />

p. cm.<br />

Includes bibliographical references and index.<br />

ISBN-13: 978-0-07-337630-1 (alk. paper)<br />

ISBN-10: 0-07-337630-2<br />

1. <strong>Engineering</strong> economy. I. Tarquin, Anthony J. II. Title.<br />

TA177.4.B58 <strong>2012</strong><br />

658.15—dc22<br />

2010052297<br />

www.mhhe.com


This book is dedicated to Dr. Frank W. Sheppard, Jr. His lifelong<br />

commitment to education, fair financial practices, international<br />

outreach, and family values has been an inspiration to many—one<br />

person at a time.


MCGRAW-HILL DIGITAL OFFERINGS<br />

McGraw-Hill Create<br />

Craft your teaching resources to match the way you teach! With McGraw-Hill Create, www<br />

.mcgrawhillcreate.com, you can easily rearrange chapters, combine material from other content<br />

sources, and quickly upload content you have written like your course syllabus or teaching notes.<br />

Find the content you need in Create by searching through thousands of leading McGraw-Hill<br />

textbooks. Arrange your book to fit your teaching style. Create even allows you to personalize<br />

your book’s appearance by selecting the cover and adding your name, school, and course information.<br />

Order a Create book and you’ll receive a complimentary print review copy in 3–5 business<br />

days or a complimentary electronic review copy (eComp) via email in minutes. Go to www.<br />

mcgrawhillcreate.com today and register to experience how McGraw-Hill Create empowers<br />

you to teach your students your way.<br />

McGraw-Hill Higher Education and Blackboard Have<br />

Teamed Up<br />

Blackboard, the Web-based course-management system, has partnered with McGraw-Hill to better<br />

allow students and faculty to use online materials and activities to complement face-to-face<br />

teaching. Blackboard features exciting social learning and teaching tools that foster more logical,<br />

visually impactful and active learning opportunities for students. You’ll transform your closeddoor<br />

classrooms into communities where students remain connected to their educational experience<br />

24 hours a day.<br />

This partnership allows you and your students access to McGraw-Hill’s Create right from<br />

within your Blackboard course – all with one single sign-on. McGraw-Hill and Blackboard can<br />

now offer you easy access to industry leading technology and content, whether your campus<br />

hosts it, or we do. Be sure to ask your local McGraw-Hill representative for details.<br />

Electronic Textbook Options<br />

This text is offered through CourseSmart for both instructors and students. CourseSmart is an<br />

online resource where students can purchase the complete text online at almost half the cost of a<br />

traditional text. Purchasing the eTextbook allows students to take advantage of CourseSmart’s<br />

web tools for learning, which include full text search, notes and highlighting, and email tools for<br />

sharing notes between classmates. To learn more about CourseSmart options, contact your sales<br />

representative or visit www.CourseSmart.com.


CONTENTS<br />

Preface to Seventh Edition xiii<br />

LEARNING<br />

STAGE 1<br />

THE FUNDAMENTALS<br />

Chapter 1 Foundations of <strong>Engineering</strong> <strong>Economy</strong> 2<br />

1.1 <strong>Engineering</strong> Economics: Description and<br />

Role in Decision Making 3<br />

1.2 Performing an <strong>Engineering</strong> <strong>Economy</strong> Study 4<br />

1.3 Professional Ethics and Economic Decisions 7<br />

1.4 Interest Rate and Rate of Return 10<br />

1.5 Terminology and Symbols 13<br />

1.6 Cash Flows: Estimation and Diagramming 15<br />

1.7 Economic Equivalence 19<br />

1.8 Simple and Compound Interest 21<br />

1.9 Minimum Attractive Rate of Return 25<br />

1.10 Introduction to Spreadsheet Use 27<br />

Chapter Summary 31<br />

Problems 31<br />

Additional Problems and FE Exam Review Questions 35<br />

Case Study—Renewable Energy Sources for Electricity Generation 36<br />

Case Study—Refrigerator Shells 37<br />

Chapter 2 Factors: How Time and Interest Affect Money 38<br />

PE Progressive Example—The Cement Factory Case 39<br />

2.1 Single-Amount Factors (FP and PF ) 39<br />

2.2 Uniform Series Present Worth Factor and Capital Recovery Factor (PA and AP) 43<br />

2.3 Sinking Fund Factor and Uniform Series Compound Amount Factor (AF and FA) 46<br />

2.4 Factor Values for Untabulated i or n Values 48<br />

2.5 Arithmetic Gradient Factors (PG and AG) 50<br />

2.6 Geometric Gradient Series Factors 58<br />

2.7 Determining i or n for Known Cash Flow Values 61<br />

Chapter Summary 64<br />

Problems 64<br />

Additional Problems and FE Exam Review Questions 69<br />

Case Study—Time Marches On; So Does the Interest Rate 70<br />

Chapter 3 Combining Factors and Spreadsheet Functions 72<br />

3.1 Calculations for Uniform Series That Are Shifted 73<br />

3.2 Calculations Involving Uniform Series and Randomly Placed Single Amounts 76<br />

3.3 Calculations for Shifted Gradients 80<br />

Chapter Summary 86<br />

Problems 86<br />

Additional Problems and FE Exam Review Questions 92<br />

Case Study—Preserving Land for Public Use 93<br />

Chapter 4 Nominal and Effective Interest Rates 94<br />

PE Progressive Example—The Credit Card Offer Case 95<br />

4.1 Nominal and Effective Interest Rate Statements 96<br />

4.2 Effective Annual Interest Rates 99<br />

4.3 Effective Interest Rates for Any Time Period 105<br />

4.4 Equivalence Relations: Payment Period and Compounding Period 106<br />

4.5 Equivalence Relations: Single Amounts with PP CP 107


viii<br />

Contents<br />

4.6 Equivalence Relations: Series with PP CP 109<br />

4.7 Equivalence Relations: Single Amounts and Series with PP CP 112<br />

4.8 Effective Interest Rate for Continuous Compounding 114<br />

4.9 Interest Rates That Vary over Time 116<br />

Chapter Summary 117<br />

Problems 118<br />

Additional Problems and FE Exam Review Questions 122<br />

Case Study—Is Owning a Home a Net Gain or Net Loss over Time? 124<br />

LEARNING<br />

STAGE 2<br />

BASIC ANALYSIS TOOLS<br />

Chapter 5 Present Worth Analysis 128<br />

PE Progressive Example—Water for Semiconductor Manufacturing Case 129<br />

5.1 Formulating Alternatives 129<br />

5.2 Present Worth Analysis of Equal-Life Alternatives 131<br />

5.3 Present Worth Analysis of Different-Life Alternatives 133<br />

5.4 Future Worth Analysis 137<br />

5.5 Capitalized Cost Analysis 138<br />

Chapter Summary 142<br />

Problems 142<br />

Additional Problems and FE Exam Review Questions 147<br />

Case Study—Comparing Social Security Benefits 149<br />

Chapter 6 Annual Worth Analysis 150<br />

6.1 Advantages and Uses of Annual Worth Analysis 151<br />

6.2 Calculation of Capital Recovery and AW Values 153<br />

6.3 Evaluating Alternatives by Annual Worth Analysis 155<br />

6.4 AW of a Permanent Investment 157<br />

6.5 Life-Cycle Cost Analysis 160<br />

Chapter Summary 164<br />

Problems 164<br />

Additional Problems and FE Exam Review Questions 169<br />

Case Study—The Changing Scene of an Annual Worth Analysis 171<br />

Chapter 7 Rate of Return Analysis: One Project 172<br />

7.1 Interpretation of a Rate of Return Value 173<br />

7.2 Rate of Return Calculation Using a PW or AW Relation 175<br />

7.3 Special Considerations When Using the ROR Method 179<br />

7.4 Multiple Rate of Return Values 180<br />

7.5 Techniques to Remove Multiple Rates of Return 184<br />

7.6 Rate of Return of a Bond Investment 190<br />

Chapter Summary 193<br />

Problems 193<br />

Additional Problems and FE Exam Review Questions 198<br />

Case Study—Developing and Selling an Innovative Idea 200<br />

Chapter 8 Rate of Return Analysis: Multiple Alternatives 202<br />

8.1 Why Incremental Analysis Is Necessary 203<br />

8.2 Calculation of Incremental Cash Flows for ROR Analysis 203<br />

8.3 Interpretation of Rate of Return on the Extra Investment 206<br />

8.4 Rate of Return Evaluation Using PW: Incremental and Breakeven 207<br />

8.5 Rate of Return Evaluation Using AW 213<br />

8.6 Incremental ROR Analysis of Multiple Alternatives 214


Contents ix<br />

8.7 All-in-One Spreadsheet Analysis (Optional) 218<br />

Chapter Summary 219<br />

Problems 220<br />

Additional Problems and FE Exam Review Questions 225<br />

Case Study—ROR Analysis with Estimated Lives That Vary 226<br />

Case Study—How a New <strong>Engineering</strong> Graduate Can Help His Father 227<br />

Chapter 9 Benefit/Cost Analysis and Public Sector Economics 228<br />

PE Progressive Example—Water Treatment Facility #3 Case 229<br />

9.1 Public Sector Projects 230<br />

9.2 Benefit/Cost Analysis of a Single Project 235<br />

9.3 Alternative Selection Using Incremental B/C Analysis 238<br />

9.4 Incremental B/C Analysis of Multiple, Mutually Exclusive Alternatives 242<br />

9.5 Service Sector Projects and Cost-Effectiveness Analysis 246<br />

9.6 Ethical Considerations in the Public Sector 250<br />

Chapter Summary 251<br />

Problems 252<br />

Additional Problems and FE Exam Review Questions 258<br />

Case Study—Comparing B/C Analysis and CEA of Traffic Accident Reduction 259<br />

LEARNING<br />

STAGE 2<br />

EPILOGUE: SELECTING THE BASIC ANALYSIS TOOL<br />

LEARNING<br />

STAGE 3<br />

MAKING BETTER DECISIONS<br />

Chapter 10 Project Financing and Noneconomic Attributes 266<br />

10.1 MARR Relative to the Cost of Capital 267<br />

10.2 Debt-Equity Mix and Weighted Average Cost of Capital 269<br />

10.3 Determination of the Cost of Debt Capital 271<br />

10.4 Determination of the Cost of Equity Capital and the MARR 273<br />

10.5 Effect of Debt-Equity Mix on Investment Risk 275<br />

10.6 Multiple Attribute Analysis: Identification and Importance of Each Attribute 278<br />

10.7 Evaluation Measure for Multiple Attributes 282<br />

Chapter Summary 283<br />

Problems 284<br />

Additional Problems and FE Exam Review Questions 289<br />

Case Study—Which Is Better—Debt or Equity Financing? 290<br />

Chapter 11 Replacement and Retention Decisions 292<br />

PE Progressive Example—Keep or Replace the Kiln Case 293<br />

11.1 Basics of a Replacement Study 294<br />

11.2 Economic Service Life 296<br />

11.3 Performing a Replacement Study 302<br />

11.4 Additional Considerations in a Replacement Study 306<br />

11.5 Replacement Study over a Specified Study Period 307<br />

11.6 Replacement Value 312<br />

Chapter Summary 312<br />

Problems 313<br />

Additional Problems and FE Exam Review Questions 319<br />

Case Study—Will the Correct ESL Please Stand? 321


x<br />

Contents<br />

Chapter 12 Independent Projects with Budget Limitation 322<br />

12.1 An Overview of Capital Rationing among Projects 323<br />

12.2 Capital Rationing Using PW Analysis of Equal-Life Projects 325<br />

12.3 Capital Rationing Using PW Analysis of Unequal-Life Projects 327<br />

12.4 Capital Budgeting Problem Formulation Using Linear Programming 329<br />

12.5 Additional Project Ranking Measures 332<br />

Chapter Summary 334<br />

Problems 334<br />

Additional Problems and FE Exam Review Questions 338<br />

Chapter 13 Breakeven and Payback Analysis 340<br />

13.1 Breakeven Analysis for a Single Project 341<br />

13.2 Breakeven Analysis Between Two Alternatives 345<br />

13.3 Payback Analysis 348<br />

13.4 More Breakeven and Payback Analysis on Spreadsheets 352<br />

Chapter Summary 355<br />

Problems 355<br />

Additional Problems and FE Exam Review Questions 361<br />

Case Study—Water Treatment Plant Process Costs 363<br />

LEARNING<br />

STAGE 4<br />

ROUNDING OUT THE STUDY<br />

Chapter 14 Effects of Inflation 366<br />

14.1 Understanding the Impact of Inflation 367<br />

14.2 Present Worth Calculations Adjusted for Inflation 369<br />

14.3 Future Worth Calculations Adjusted for Inflation 374<br />

14.4 Capital Recovery Calculations Adjusted for Inflation 377<br />

Chapter Summary 378<br />

Problems 379<br />

Additional Problems and FE Exam Review Questions 384<br />

Case Study—Inflation versus Stock and Bond Investments 385<br />

Chapter 15 Cost Estimation and Indirect Cost Allocation 386<br />

15.1 Understanding How Cost Estimation Is Accomplished 387<br />

15.2 Unit Method 390<br />

15.3 Cost Indexes 391<br />

15.4 Cost-Estimating Relationships: Cost-Capacity Equations 394<br />

15.5 Cost-Estimating Relationships: Factor Method 395<br />

15.6 Traditional Indirect Cost Rates and Allocation 397<br />

15.7 Activity-Based Costing (ABC) for Indirect Costs 401<br />

15.8 Making Estimates and Maintaining Ethical Practices 403<br />

Chapter Summary 404<br />

Problems 404<br />

Additional Problems and FE Exam Review Questions 410<br />

Case Study—Indirect Cost Analysis of Medical Equipment Manufacturing Costs 411<br />

Case Study—Deceptive Acts Can Get You in Trouble 412<br />

Chapter 16 Depreciation Methods 414<br />

16.1 Depreciation Terminology 415<br />

16.2 Straight Line (SL) Depreciation 418<br />

16.3 Declining Balance (DB) and Double Declining Balance (DDB) Depreciation 419<br />

16.4 Modified Accelerated Cost Recovery System (MACRS) 422<br />

16.5 Determining the MACRS Recovery Period 426


Contents xi<br />

16.6 Depletion Methods 427<br />

Chapter Summary 429<br />

Appendix 430<br />

16A.1 Sum-of-Years-Digits (SYD) and Unit-of-Production (UOP) Depreciation 430<br />

16A.2 Switching between Depreciation Methods 432<br />

16A.3 Determination of MACRS Rates 435<br />

Problems 438<br />

Additional Problems and FE Exam Review Questions 442<br />

Appendix Problems 443<br />

Chapter 17 After-Tax Economic Analysis 444<br />

17.1 Income Tax Terminology and Basic Relations 445<br />

17.2 Calculation of Cash Flow after Taxes 448<br />

17.3 Effect on Taxes of Different Depreciation Methods and Recovery Periods 450<br />

17.4 Depreciation Recapture and Capital Gains (Losses) 453<br />

17.5 After-Tax Evaluation 456<br />

17.6 After-Tax Replacement Study 462<br />

17.7 After-Tax Value-Added Analysis 465<br />

17.8 After-Tax Analysis for International Projects 468<br />

17.9 Value-Added Tax 470<br />

Chapter Summary 472<br />

Problems 473<br />

Additional Problems and FE Exam Review Questions 481<br />

Case Study—After-Tax Analysis for Business Expansion 482<br />

Chapter 18 Sensitivity Analysis and Staged Decisions 484<br />

18.1 Determining Sensitivity to Parameter Variation 485<br />

18.2 Sensitivity Analysis Using Three Estimates 490<br />

18.3 Estimate Variability and the Expected Value 491<br />

18.4 Expected Value Computations for Alternatives 492<br />

18.5 Staged Evaluation of Alternatives Using a Decision Tree 494<br />

18.6 Real Options in <strong>Engineering</strong> Economics 498<br />

Chapter Summary 503<br />

Problems 503<br />

Additional Problems and FE Exam Review Questions 509<br />

Case Study—Sensitivity to the Economic Environment 510<br />

Case Study—Sensitivity Analysis of Public Sector Projects—Water Supply Plans 511<br />

Chapter 19 More on Variation and Decision Making under Risk 514<br />

19.1 Interpretation of Certainty, Risk, and Uncertainty 515<br />

19.2 Elements Important to Decision Making under Risk 518<br />

19.3 Random Samples 523<br />

19.4 Expected Value and Standard Deviation 526<br />

19.5 Monte Carlo Sampling and Simulation Analysis 533<br />

Chapter Summary 540<br />

Problems 540<br />

Additional Problems and FE Exam Review Questions 543<br />

Case Study—Using Simulation and Three-Estimate Sensitivity Analysis 544<br />

Appendix A Using Spreadsheets and Microsoft Excel © 547<br />

A.1 Introduction to Using Excel 547<br />

A.2 Organization (Layout) of the Spreadsheet 549<br />

A.3 Excel Functions Important to <strong>Engineering</strong> <strong>Economy</strong> 550<br />

A.4 Goal Seek—A Tool for Breakeven and Sensitivity Analysis 558<br />

A.5 Solver—An Optimizing Tool for Capital Budgeting, Breakeven, and Sensitivity Analysis 559<br />

A.6 Error Messages 560


xii<br />

Contents<br />

Appendix B Basics of Accounting Reports and Business Ratios 561<br />

B.1 The Balance Sheet 561<br />

B.2 Income Statement and Cost of Goods Sold Statement 562<br />

B.3 Business Ratios 563<br />

Appendix C Code of Ethics for Engineers 566<br />

Appendix D Alternate Methods for Equivalence Calculations 569<br />

D.1 Using Programmable Calculators 569<br />

D.2 Using the Summation of a Geometric Series 570<br />

Appendix E Glossary of Concepts and Terms 573<br />

E.1 Important Concepts and Guidelines 573<br />

E.2 Symbols and Terms 576<br />

Reference Materials 579<br />

Factor Tables 581<br />

Photo Credits 610<br />

Index 611


PREFACE TO SEVENTH EDITION<br />

This edition includes the time-tested approach and topics of previous editions and introduces significantly<br />

new print and electronic features useful to learning about and successfully applying the exciting<br />

field of engineering economics. Money makes a huge difference in the life of a corporation, an<br />

individual, and a government. Learning to understand, analyze, and manage the money side of any<br />

project is vital to its success. To be professionally successful, every engineer must be able to deal with<br />

the time value of money, economic facts, inflation, cost estimation, tax considerations, as well as<br />

spreadsheet and calculator use. This book is a great help to the learner and the instructor in accomplishing<br />

these goals by using easy-to-understand language, simple graphics, and online features.<br />

What's New and What's Best<br />

This seventh edition is full of new information and features. Plus the supporting online materials<br />

are new and updated to enhance the teaching and learning experience.<br />

New topics:<br />

• Ethics and the economics of engineering<br />

• Service sector projects and their evaluation<br />

• Real options development and analysis<br />

• Value-added taxes and how they work<br />

• Multiple rates of return and ways to eliminate them using spreadsheets<br />

• No tabulated factors needed for equivalence computations (Appendix D)<br />

New features in print and online:<br />

• Totally new design to highlight important terms, concepts, and decision guidelines<br />

• Progressive examples that continue throughout a chapter<br />

• Downloadable online presentations featuring voice-over slides and animation<br />

• Vital concepts and guidelines identified in margins; brief descriptions available (Appendix E)<br />

• Fresh spreadsheet displays with on-image comments and function details<br />

• Case studies (21 of them) ranging in topics from ethics to energy to simulation<br />

Retained features:<br />

• Many end-of-chapter problems (over 90% are new or revised)<br />

• Easy-to-read language to enhance understanding in a variety of course environments<br />

• Fundamentals of <strong>Engineering</strong> (FE) Exam review questions that double as additional or<br />

review problems for quizzes and tests<br />

• Hand and spreadsheet solutions presented for many examples<br />

• Flexible chapter ordering after fundamental topics are understood<br />

• Complete solutions manual available online (with access approval for instructors)<br />

How to Use This Text<br />

This textbook is best suited for a one-semester or one-quarter undergraduate course. Students<br />

should be at the sophomore level or above with a basic understanding of engineering concepts<br />

and terminology. A course in calculus is not necessary; however, knowledge of the concepts in<br />

advanced mathematics and elementary probability will make the topics more meaningful.<br />

Practitioners and professional engineers who need a refresher in economic analysis and cost<br />

estimation will find this book very useful as a reference document as well as a learning medium.<br />

Chapter Organization and Coverage Options<br />

The textbook contains 19 chapters arranged into four learning stages, as indicated in the fl owchart<br />

on the next page, and five appendices. Each chapter starts with a statement of purpose and a specific<br />

learning outcome (ABET style) for each section. Chapters include a summary, numerous


xiv<br />

Preface to Seventh Edition<br />

CHAPTERS IN EACH LEARNING STAGE<br />

Composition by level<br />

Chapter 1<br />

Foundations of<br />

<strong>Engineering</strong> <strong>Economy</strong><br />

Learning<br />

Stage 1:<br />

The<br />

Fundamentals<br />

Chapter 2<br />

Factors: How Time and<br />

Interest Affect Money<br />

Chapter 3<br />

Combining Factors and<br />

Spreadsheet Functions<br />

Chapter 4<br />

Nominal and Effective<br />

Interest Rates<br />

Learning<br />

Stage 2:<br />

Basic<br />

Analysis<br />

Tools<br />

Chapter 5<br />

Present Worth<br />

Analysis<br />

Chapter 6<br />

Annual Worth<br />

Analysis<br />

Chapter 7<br />

Rate of Return<br />

Analysis:<br />

One Project<br />

Chapter 8<br />

Rate of Return<br />

Analysis: Multiple<br />

Alternatives<br />

Chapter 9<br />

Benefit/Cost Analysis<br />

and Public Sector<br />

Economics<br />

Learning Stage 2 Epilogue<br />

Selecting the Basic<br />

Analysis Tool<br />

Learning<br />

Stage 3:<br />

Making<br />

Better<br />

Decisions<br />

Chapter 10<br />

Project Financing and<br />

Noneconomic Attributes<br />

Chapter 11<br />

Replacement and<br />

Retention Decisions<br />

Chapter 12<br />

Independent Projects<br />

with Budget Limitation<br />

Chapter 13<br />

Breakeven and<br />

Payback Analysis<br />

Learning<br />

Stage 4:<br />

Rounding<br />

Out the<br />

Study<br />

Chapter 14<br />

Effects of<br />

Inflation<br />

Chapter 15<br />

Cost Estimation and<br />

Indirect Cost Allocation<br />

Chapter 16<br />

Depreciation<br />

Methods<br />

Chapter 17<br />

After-Tax Economic<br />

Analysis<br />

Chapter 18<br />

Sensitivity Analysis<br />

and Staged Decisions<br />

Chapter 19<br />

More on Variation<br />

and Decision Making<br />

under Risk<br />

end-of-chapter problems (essay and numerical), multiple-choice problems useful for course review<br />

and FE Exam preparation, and a case study.<br />

The appendices are important elements of learning for this text:<br />

Appendix A Spreadsheet layout and functions (Excel is featured)<br />

Appendix B Accounting reports and business ratios<br />

Appendix C Code of Ethics for Engineers (from NSPE)<br />

Appendix D Equivalence computations using calculators and geometric series; no tables<br />

Appendix E Concepts, guidelines, terms, and symbols for engineering economics<br />

There is considerable flexibility in the sequencing of topics and chapters once the first six<br />

chapters are covered, as shown in the progression graphic on the next page. If the course is designed<br />

to emphasize sensitivity and risk analysis, Chapters 18 and 19 can be covered immediately


Chapter Organization and Coverage Options xv<br />

CHAPTER AND TOPIC PROGRESSION OPTIONS<br />

Topics may be introduced at the point indicated or any point thereafter<br />

(Alternative entry points are indicated by )<br />

Numerical progression<br />

through chapters<br />

Inflation<br />

Cost<br />

Estimation<br />

Taxes and<br />

Depreciation<br />

Sensitivity, Staged<br />

Decisions, and Risk<br />

1. Foundations<br />

2. Factors<br />

3. More Factors<br />

4. Effective i<br />

5. Present Worth<br />

6. Annual Worth<br />

7. Rate of Return<br />

8. More ROR<br />

9. Benefit/Cost<br />

10. Financing and<br />

Noneconomic Attributes<br />

11. Replacement<br />

12. Capital Budgeting<br />

13. Breakeven and<br />

Payback<br />

14. Inflation<br />

15. Estimation<br />

16. Depreciation<br />

17. After-Tax<br />

18. Sensitivity, Decision<br />

Trees, and Real Options<br />

19. Risk and Simulation<br />

after Learning Stage 2 (Chapter 9) is completed. If depreciation and tax emphasis are vitally<br />

important to the goals of the course, Chapters 16 and 17 can be covered once Chapter 6 (annual<br />

worth) is completed. The progression graphic can help in the design of the course content and<br />

topic ordering.


SAMPLE OF RESOURCES FOR<br />

LEARNING OUTCOMES<br />

Each chapter begins with a purpose, list<br />

of topics, and learning outcomes<br />

(ABET style) for each corresponding<br />

section. This behavioral-based<br />

approach sensitizes the reader to what<br />

is ahead, leading to improved<br />

understanding and learning.<br />

LEARNING OUTCOMES<br />

Purpose: Use multiple factors and spreadsheet functions to find equivalent amounts for cash flows that have nonstandard<br />

placement.<br />

SECTION TOPIC LEARNING OUTCOME<br />

3.1 Shifted series • Determine the P , F or A values of a series<br />

starting at a time other than period 1.<br />

3.2 Shifted series and single cash<br />

flows<br />

• Determine the P , F , or A values of a shifted series<br />

and randomly placed single cash flows.<br />

3.3 Shifted gradients • Make equivalence calculations for shifted<br />

arithmetic or geometric gradient series that<br />

increase or decrease in size of cash flows.<br />

Time value of money<br />

It is a well-known fact that money makes money. The time value of money explains the change<br />

in the amount of money over time for funds that are owned (invested) or owed (borrowed).<br />

This is the most important concept in engineering economy.<br />

CONCEPTS AND GUIDELINES<br />

To highlight the fundamental building<br />

blocks of the course, a checkmark and title<br />

in the margin call attention to particularly<br />

important concepts and decision-making<br />

guidelines. Appendix E includes a brief<br />

description of each fundamental concept.<br />

IN-CHAPTER EXAMPLES<br />

Numerous in-chapter examples<br />

throughout the book reinforce the<br />

basic concepts and make<br />

understanding easier. Where<br />

appropriate, the example is solved<br />

using separately marked hand and<br />

spreadsheet solutions.<br />

EXAMPLE 4.6<br />

A dot-com company plans to place money in a new venture capital fund that currently returns<br />

18% per year, compounded daily. What effective rate is this ( a ) yearly and ( b ) semiannually?<br />

Solution<br />

( a) Use Equation [4.7], with r 0.18 and m 365.<br />

Effective i % per year ( 1 —— 0.18<br />

365 ) 365 1 19.716%<br />

(b) Here r 0.09 per 6 months and m 182 days.<br />

Effective i % per 6 months ( 1 —— 0.09<br />

182 ) 182 1 9.415%<br />

PROGRESSIVE EXAMPLES<br />

Water for Semiconductor Manufacturing<br />

Case: The worldwide contribution of<br />

semiconductor sales is about $250 billion<br />

per year, or about 10% of the world’s<br />

GDP (gross domestic product). This industry<br />

produces the microchips used in many<br />

of the communication, entertainment,<br />

transportation, and computing devices<br />

we use every day. Depending upon the<br />

type and size of fabrication plant (fab),<br />

the need for ultrapure water (UPW) to<br />

manufacture these tiny integrated circuits<br />

is high, ranging from 500 to 2000 gpm<br />

(gallons per minute). Ultrapure water is<br />

obtained by special processes that commonly<br />

include reverse osmosis deionizing<br />

resin bed technologies. Potable water<br />

obtained from purifying seawater or<br />

brackish groundwater may cost from<br />

$2 to $3 per 1000 gallons, but to obtain<br />

UPW on-site for semiconductor manufacturing<br />

may cost an additional $1 to $3 per<br />

1000 gallons.<br />

A fab costs upward of $2.5 billion to<br />

construct, with approximately 1% of this<br />

total, or $25 million, required to provide<br />

the ultrapure water needed, including<br />

the necessary wastewater and recycling<br />

equipment.<br />

A newcomer to the industry, Angular<br />

Enterprises, has estimated the cost profiles<br />

for two options to supply its anticipated<br />

fab with water. It is fortunate to<br />

PE<br />

have the option of desalinated seawater<br />

or purified groundwater sources in the<br />

location chosen for its new fab. The initial<br />

cost estimates for the UPW system are<br />

given below.<br />

Source<br />

Equipment first<br />

cost, $M<br />

Seawater<br />

(S)<br />

20<br />

Groundwater<br />

(G)<br />

22<br />

AOC, $M per year 0.5 0.3<br />

Salvage value, % of<br />

first cost<br />

Cost of UPW, $ per<br />

1000 gallons<br />

5 10<br />

4 5<br />

Angular has made some initial estimates<br />

for the UPW system.<br />

Life of UPW equipment 10 years<br />

UPW needs<br />

Operating time<br />

1500 gpm<br />

16 hours per<br />

day for 250 days<br />

per year<br />

This case is used in the following topics<br />

(Sections) and problems of this chapter:<br />

PW analysis of equal-life alternatives<br />

(Section 5.2)<br />

PW analysis of different-life alternatives<br />

(Section 5.3)<br />

Capitalized cost analysis (Section 5.5)<br />

Problems 5.20 and 5.34<br />

bla76302_ch04_094-126.indd 106<br />

Several chapters include a progressive<br />

example—a more detailed problem statement<br />

introduced at the beginning of the chapter and<br />

expanded upon throughout the chapter in<br />

specially marked examples. This approach<br />

illustrates different techniques and some<br />

increasingly complex aspects of a real-world<br />

problem.<br />

12/22/10 8:24 PM


INSTRUCTORS AND STUDENTS<br />

Contents xvii<br />

bla76302_ch08_202-227.indd 212 12/11/10 6:52 PM<br />

ONLINE PRESENTATIONS<br />

An icon in the margin indicates the<br />

availability of an animated voice-over slide<br />

presentation that summarizes the material in<br />

the section and provides a brief example for<br />

learners who need a review or prefer videobased<br />

materials. Presentations are keyed to<br />

the sections of the text.<br />

Breakeven ROR 17%<br />

EXAMPLE 13.8<br />

Breakeven<br />

Chris and her father just purchased Incremental a ROR small office 17% building for $160,000 that is in need of a lot<br />

of repairs, but is located in a prime commercial area of the city. The estimated costs each year<br />

for repairs, insurance, etc. are $18,000 the first year, increasing by $1000 per year thereafter.<br />

Figure At an 8–6 expected 8% per year return, use spreadsheet analysis to determine the payback period<br />

PW if versus the building i graph and is (a) PW kept versus for incremental 2 years and i graph, sold for Example $290,000 8.4 . sometime beyond year 2 or (b) kept<br />

for 3 years and sold for $370,000 sometime beyond 3 years.<br />

Solution<br />

Figure 13–11 shows the annual costs (column B) and the sales prices if the building is kept 2<br />

or 3 years (columns C and E, respectively). The NPV function is applied (columns D and F) to<br />

determine when the PW changes sign from plus to minus. These results bracket the payback<br />

period for each retention Figure period 7–12 and sales price. When PW 0, the 8% return is exceeded.<br />

(a) The 8% return payback Spreadsheet period is application between 3 and of 4 ROIC years method (column using D). If the Goal building Seek, is Example sold 7.6 .<br />

after exactly 3 years for $290,000, the payback period was not exceeded; but after 4 years<br />

it is exceeded.<br />

(b) At a sales price of $370,000, the 8% return payback period is between 5 and 6 years (column<br />

F). If the building is sold after 4 or 5 years, the payback is not exceeded; however, a<br />

sale after 6 years is beyond the 8%-return payback period.<br />

bla76302_ch07_172-201.indd 189<br />

MARR<br />

NPV(8%,$B$4:B7)+$B$3 PV(8%,A7,,290000)<br />

If kept 2 years and<br />

sold, payback is<br />

between 3 and 4<br />

If kept 3 years and<br />

sold, payback is<br />

between 5 and 6<br />

MARR<br />

Filter 1 ROR 25%<br />

Filter 2 ROR 23%<br />

3.1 Calculations for Uniform Series That Are Shifted<br />

When a uniform series begins at a time other than at the end of period 1, it is called a shifted<br />

series. In this case several methods can be used to find the equivalent present worth P . For<br />

example, P of the uniform series shown in Figure 3–1 could be determined by any of the<br />

following methods:<br />

• Use the P F factor to find the present worth of each disbursement at year 0 and add them.<br />

• Use the F P factor to find the future worth of each disbursement in year 13, add them, and<br />

then find the present worth of the total, using P F ( P F , i ,13).<br />

• Use the F A factor to find the future amount F A ( F A , i ,10), and then compute the present<br />

worth, using P F ( P F , i ,13).<br />

• Use the P A factor to compute the “present worth” P 3 A ( P A , i ,10) (which will be located<br />

in year 3, not year 0), and then find the present worth in year 0 by using the ( P F , i ,3) factor.<br />

12/11/10 4:32 PM<br />

SPREADSHEETS<br />

The text integrates spreadsheets to show<br />

how easy they are to use in solving virtually<br />

any type of engineering economic analysis<br />

problem. Cell tags or full cells detail<br />

built-in functions and relations developed<br />

to solve a specific problem.<br />

Figure 13–11<br />

Payback period analysis, Example 13.8<br />

bla76302_ch13_340-364.indd 354<br />

12/17/10 1:02 PM<br />

bla76302_ch03_072-093.indd 73<br />

12/7/10 7:26 AM<br />

FE EXAM AND COURSE<br />

REVIEWS<br />

Each chapter concludes with several<br />

multiple-choice, FE Exam–style<br />

problems that provide a simplified<br />

review of chapter material. Additionally,<br />

these problems cover topics for test<br />

reviews and homework assignments.<br />

ADDITIONAL PROBLEMS AND FE EXAM REVIEW QUESTIONS<br />

8.38 When conducting a rate of return (ROR) analysis<br />

involving multiple mutually exclusive alternatives,<br />

the first step is to:<br />

(a) Rank the alternatives according to decreasing<br />

initial investment cost<br />

(b) Rank the alternatives according to increasing<br />

initial investment cost<br />

(c) Calculate the present worth of each alternative<br />

using the MARR<br />

(d) Find the LCM between all of the alternatives<br />

8.39 In comparing mutually exclusive alternatives by<br />

the ROR method, you should:<br />

(a) Find the ROR of each alternative and pick<br />

the one with the highest ROR<br />

(b) S l h l i h i l<br />

8.43 For these alternatives, the sum of the incremental<br />

cash flows is:<br />

Year A B<br />

0 10,000 14,000<br />

1 2,500 4,000<br />

2 2,500 4,000<br />

3 2,500 4,000<br />

4 2,500 4,000<br />

5 2,500 4,000<br />

(a) $2500<br />

(b) $3500<br />

(c) $6000<br />

(d) $8000


CASE STUDY<br />

RENEWABLE ENERGY SOURCES FOR ELECTRICITY GENERATION<br />

Background<br />

Pedernales Electric Cooperative (PEC) is the largest<br />

member-owned electric co-op in the United States with over<br />

232,000 meters in 12 Central Texas counties. PEC has a capacity<br />

of approximately 1300 MW (megawatts) of power, of<br />

which 277 MW, or about 21%, is from renewable sources.<br />

The latest addition is 60 MW of power from a wind farm in<br />

south Texas close to the city of Corpus Christi. A constant<br />

question is how much of PEC’s generation capacity should be<br />

from renewable sources, especially given the environmental<br />

issues with coal-generated electricity and the rising costs of<br />

hydrocarbon fuels.<br />

Wind and nuclear sources are the current consideration for<br />

the PEC leadership as Texas is increasing its generation by<br />

nuclear power and the state is the national leader in wind<br />

farm–produced electricity.<br />

Consider yourself a member of the board of directors of<br />

PEC. You are an engineer who has been newly elected by the<br />

PEC membership to serve a 3-year term as a director-at-large.<br />

As such, you do not represent a specific district within the<br />

entire service area; all other directors do represent a specific<br />

district. You have many questions about the operations of<br />

PEC, plus you are interested in the economic and societal<br />

benefits of pursuing more renewable source generation<br />

capacity.<br />

Information<br />

Here are some data that you have obtained. The information<br />

is sketchy, as this point, and the numbers are very approximate.<br />

Electricity generation cost estimates are national<br />

in scope, not PEC-specific, and are provided in cents per<br />

kilowatt-hour (¢/kWh).<br />

Generation Cost, ¢/kWh<br />

Fuel Source Likely Range Reasonable Average<br />

Coal 4 to 9 7.4<br />

Natural gas 4 to 10.5 8.6<br />

Wind 4.8 to 9.1 8.2<br />

Solar 4.5 to 15.5 8.8<br />

National average cost of electricity to residential customers:<br />

11¢/kWh<br />

PEC average cost to residential customers: 10.27 ¢/kWh<br />

(from primary sources) and 10.92 ¢/kWh (renewable sources)<br />

Expected life of a generation facility: 20 to 40 years (it is<br />

likely closer to 20 than 40)<br />

Time to construct a facility: 2 to 5 years<br />

Capital cost to build a generation facility: $900 to $1500<br />

per kW<br />

You have also learned that the PEC staff uses the wellrecognized<br />

levelized energy cost (LEC) method to determine<br />

the price of electricity that must be charged to customers to<br />

break even. The formula takes into account the capital cost of<br />

the generation facilities, the cost of capital of borrowed<br />

money, annual maintenance and operation (M&O) costs, and<br />

the expected life of the facility. The LEC formula, expressed<br />

in dollars per kWh for ( t 1, 2, ... , n ), is<br />

tn<br />

P t A t C<br />

—————— t<br />

t1 (1 i)<br />

LEC <br />

t<br />

———————<br />

tn<br />

E<br />

——— t<br />

t1 (1 i) t<br />

where P t capital investments made in year t<br />

A t annual maintenance and operating (M&O) costs<br />

for year t<br />

C t fuel costs for year t<br />

E t amount of electricity generated in year t<br />

n expected life of facility<br />

i discount rate (cost of capital)<br />

Case Study Exercises<br />

1. If you wanted to know more about the new arrangement<br />

with the wind farm in south Texas for the additional<br />

60 MW per year, what types of questions would<br />

you ask of a staff member in your first meeting with<br />

him or her?<br />

2. Much of the current generation capacity of PEC facilities<br />

utilizes coal and natural gas as the primary fuel source.<br />

What about the ethical aspects of the government’s allowance<br />

for these plants to continue polluting the atmosphere<br />

with the emissions that may cause health problems for<br />

citizens and further the effects of global warming? What<br />

types of regulations, if any, should be developed for PEC<br />

(and other generators) to follow in the future?<br />

CASE STUDIES<br />

New and updated case studies at the<br />

end of most chapters present realworld,<br />

in-depth treatments and<br />

exercises in the engineering<br />

profession. Each case includes a<br />

background, relevant information,<br />

and an exercise section.


ACKNOWLEDGMENT OF CONTRIBUTORS<br />

It takes the input and efforts of many individuals to make significant improvements in a textbook.<br />

We wish to give special thanks to the following persons for their contributions to this edition.<br />

Paul Askenasy, Texas Commission on Environmental Quality<br />

Jack Beltran, Bristol-Myers Squibb<br />

Robert Lundquist, Ohio State University<br />

William Peet, Infrastructure Coordination, Government of Niue<br />

Sallie Sheppard, Texas A&M University<br />

We thank the following individuals for their comments, feedback, and review of material to assist<br />

in making this edition a real success.<br />

Ahmed Alim, University of Houston<br />

Alan Atalah, Bowling Green State University<br />

Fola Michael Ayokanmbi, Alabama A&M University<br />

William Brown, West Virginia University at Parkersburg<br />

Hector Carrasco, Colorado State University–Pueblo<br />

Robert Chiang, California State University, Pomona<br />

Ronald Cutwright, Florida State University<br />

John F. Dacquisto, Gonzaga University<br />

Houshang Darabi, University of Illinois at Chicago<br />

Freddie Davis, West Texas A&M University<br />

Edward Lester Dollar, Southern Polytechnic State University<br />

Ted Eschenbach, University of Alaska<br />

Clara Fang, University of Hartford<br />

Abel Fernandez, University of the Pacific<br />

Daniel A. Franchi, California Polytechnic State University, San Luis Obispo<br />

Mark Frascatore, Clarkson University<br />

Benjamin M. Fries, University of Central Florida<br />

Nathan Gartner, University of Massachusetts–Lowell<br />

Johnny R. Graham, University of North Carolina–Charlotte<br />

Liling Huang, Northern Virginia Community College<br />

David Jacobs, University of Hartford<br />

Adam Jannik, Northwestern State University<br />

Peter E. Johnson, Valparaiso University<br />

Justin W. Kile, University of Wisconsin–Platteville<br />

John Kushner, Lawrence Technological University<br />

Clifford D. Madrid, New Mexico State University<br />

Saeed Manafzadeh, University of Illinois at Chicago<br />

Quamrul Mazumder, University of Michigan–Flint<br />

Deb McAvoy, Ohio University<br />

Gene McGinnis, Christian Brothers University<br />

Bruce V. Mutter, Bluefield State College<br />

Hong Sioe Oey, University of Texas at El Paso<br />

Richard Palmer, University of Massachusetts<br />

Michael J. Rider, Ohio Northern University<br />

John Ristroph, University of Louisiana at Lafayette<br />

Saeid L. Sadri, Georgia Institute of Technology<br />

Scott Schultz, Mercer University<br />

Kyo D. Song, Norfolk State University<br />

James Stevens, University of Colorado at Colorado Springs<br />

John A. Stratton, Rochester Institute of Technology<br />

Mathias J. Sutton, Purdue University<br />

Pete Weiss, Valparaiso University


xx<br />

Acknowledgment of Contributors<br />

Greg Wiles, Southern Polytechnic State University<br />

Richard Youchak, University of Pittsburgh at Johnstown<br />

William A. Young, II, Ohio University<br />

If you discover errors that require correction in the next printing of the textbook or in updates of<br />

the online resources, please contact us. We hope you find the contents of this edition helpful in<br />

your academic and professional activities.<br />

Leland Blank<br />

Anthony Tarquin<br />

lelandblank@yahoo.com<br />

atarquin@utep.edu


LEARNING STAGE 1<br />

The Fundamentals<br />

LEARNING STAGE 1<br />

The Fundamentals<br />

CHAPTER 1<br />

Foundations of<br />

<strong>Engineering</strong> <strong>Economy</strong><br />

CHAPTER 2<br />

Factors: How Time<br />

and Interest Affect<br />

Money<br />

CHAPTER 3<br />

Combining Factors<br />

and Spreadsheet<br />

Functions<br />

CHAPTER 4<br />

Nominal and Effective<br />

Interest Rates<br />

The fundamentals of engineering economy are introduced in<br />

these chapters. When you have completed stage 1, you will be<br />

able to understand and work problems that account for the<br />

time value of money, cash flows occurring at different times with<br />

different amounts, and equivalence at different interest rates. The<br />

techniques you master here form the basis of how an engineer in<br />

any discipline can take economic value into account in virtually any<br />

project environment.<br />

The factors commonly used in all engineering economy computations<br />

are introduced and applied here. Combinations of these factors<br />

assist in moving monetary values forward and backward through<br />

time and at different interest rates. Also, after these chapters, you<br />

should be comfortable using many of the spreadsheet functions.<br />

Many of the terms common to economic decision making are<br />

introduced in learning stage 1 and used in later chapters. A checkmark<br />

icon in the margin indicates that a new concept or guideline<br />

is introduced at this point.


CHAPTER 1<br />

Foundations<br />

of <strong>Engineering</strong><br />

<strong>Economy</strong><br />

LEARNING OUTCOMES<br />

Purpose: Understand and apply fundamental concepts and use the terminology of engineering economics.<br />

SECTION TOPIC LEARNING OUTCOME<br />

1.1 Description and role • Define engineering economics and describe its<br />

role in decision making.<br />

1.2 <strong>Engineering</strong> economy study<br />

approach<br />

• Understand and identify the steps in an<br />

engineering economy study.<br />

1.3 Ethics and economics • Identify areas in which economic decisions can<br />

present questionable ethics.<br />

1.4 Interest rate • Perform calculations for interest rates and rates<br />

of return.<br />

1.5 Terms and symbols • Identify and use engineering economic<br />

terminology and symbols.<br />

1.6 Cash flows • Understand cash flows and how to graphically<br />

represent them.<br />

1.7 Economic equivalence • Describe and calculate economic equivalence.<br />

1.8 Simple and compound interest • Calculate simple and compound interest<br />

amounts for one or more time periods.<br />

1.9 MARR and opportunity cost • State the meaning and role of Minimum<br />

Attractive Rate of Return (MARR) and<br />

opportunity costs.<br />

1.10 Spreadsheet functions • Identify and use some Excel functions<br />

commonly applied in engineering economics.


T<br />

he need for engineering economy is primarily motivated by the work that engineers<br />

do in performing analyses, synthesizing, and coming to a conclusion as they work on<br />

projects of all sizes. In other words, engineering economy is at the heart of making<br />

decisions . These decisions involve the fundamental elements of cash flows of money, time,<br />

and interest rates. This chapter introduces the basic concepts and terminology necessary for<br />

an engineer to combine these three essential elements in organized, mathematically correct<br />

ways to solve problems that will lead to better decisions.<br />

1.1 <strong>Engineering</strong> Economics: Description and<br />

Role in Decision Making<br />

Decisions are made routinely to choose one alternative over another by individuals in everyday<br />

life; by engineers on the job; by managers who supervise the activities of others; by corporate<br />

presidents who operate a business; and by government officials who work for the public good.<br />

Most decisions involve money, called capital or capital funds , which is usually limited in<br />

amount. The decision of where and how to invest this limited capital is motivated by a primary<br />

goal of adding value as future, anticipated results of the selected alternative are realized.<br />

Engineers play a vital role in capital investment decisions based upon their ability and experience<br />

to design, analyze, and synthesize. The factors upon which a decision is based are commonly a<br />

combination of economic and noneconomic elements. <strong>Engineering</strong> economy deals with the<br />

economic factors. By definition,<br />

<strong>Engineering</strong> economy involves formulating, estimating, and evaluating the expected economic<br />

outcomes of alternatives designed to accomplish a defined purpose. Mathematical techniques<br />

simplify the economic evaluation of alternatives.<br />

Because the formulas and techniques used in engineering economics are applicable to all<br />

types of money matters, they are equally useful in business and government, as well as for<br />

individuals. Therefore, besides applications to projects in your future jobs, what you learn<br />

from this book and in this course may well offer you an economic analysis tool for making<br />

personal decisions such as car purchases, house purchases, major purchases on credit, e.g.,<br />

furniture, appliances, and electronics.<br />

Other terms that mean the same as engineering economy are engineering economic analysis,<br />

capital allocation study, economic analysis, and similar descriptors.<br />

People make decisions; computers, mathematics, concepts, and guidelines assist people in<br />

their decision-making process. Since most decisions affect what will be done, the time frame of<br />

engineering economy is primarily the future . Therefore, the numbers used in engineering economy<br />

are best estimates of what is expected to occur . The estimates and the decision usually<br />

involve four essential elements:<br />

Cash flows<br />

Times of occurrence of cash flows<br />

Interest rates for time value of money<br />

Measure of economic worth for selecting an alternative<br />

Since the estimates of cash flow amounts and timing are about the future, they will be somewhat<br />

different than what is actually observed, due to changing circumstances and unplanned<br />

events. In short, the variation between an amount or time estimated now and that observed<br />

in the future is caused by the stochastic (random) nature of all economic events. Sensitivity<br />

analysis is utilized to determine how a decision might change according to varying estimates,<br />

especially those expected to vary widely. Example 1.1 illustrates the fundamental<br />

nature of variation in estimates and how this variation may be included in the analysis at a<br />

very basic level.<br />

EXAMPLE 1.1<br />

An engineer is performing an analysis of warranty costs for drive train repairs within the first<br />

year of ownership of luxury cars purchased in the United States. He found the average cost (to<br />

the nearest dollar) to be $570 per repair from data taken over a 5-year period.


4 Chapter 1 Foundations of <strong>Engineering</strong> <strong>Economy</strong><br />

Year 2006 2007 2008 2009 2010<br />

Average Cost, $/repair 525 430 619 650 625<br />

What range of repair costs should the engineer use to ensure that the analysis is sensitive to<br />

changing warranty costs?<br />

Solution<br />

At first glance the range should be approximately –25% to 15% of the $570 average cost to<br />

include the low of $430 and high of $650. However, the last 3 years of costs are higher and<br />

more consistent with an average of $631. The observed values are approximately 3% of this<br />

more recent average.<br />

If the analysis is to use the most recent data and trends, a range of, say, 5% of $630 is recommended.<br />

If, however, the analysis is to be more inclusive of historical data and trends, a range<br />

of, say, 20% or 25% of $570 is recommended.<br />

The criterion used to select an alternative in engineering economy for a specific set of estimates<br />

is called a measure of worth . The measures developed and used in this text are<br />

Present worth (PW) Future worth (FW) Annual worth (AW)<br />

Rate of return (ROR) Benefit/cost (B/C) Capitalized cost (CC)<br />

Payback period Economic value added (EVA) Cost Effectiveness<br />

All these measures of worth account for the fact that money makes money over time. This is the<br />

concept of the time value of money.<br />

Time value of money<br />

It is a well-known fact that money makes money. The time value of money explains the change<br />

in the amount of money over time for funds that are owned (invested) or owed (borrowed).<br />

This is the most important concept in engineering economy.<br />

The time value of money is very obvious in the world of economics. If we decide to invest<br />

capital (money) in a project today, we inherently expect to have more money in the future than<br />

we invested. If we borrow money today, in one form or another, we expect to return the original<br />

amount plus some additional amount of money.<br />

<strong>Engineering</strong> economics is equally well suited for the future and for the analysis of past cash<br />

flows in order to determine if a specific criterion (measure of worth) was attained. For example,<br />

assume you invested $4975 exactly 3 years ago in 53 shares of IBM stock as traded on the New<br />

York Stock Exchange (NYSE) at $93.86 per share. You expect to make 8% per year appreciation,<br />

not considering any dividends that IBM may declare. A quick check of the share value shows it<br />

is currently worth $127.25 per share for a total of $6744.25. This increase in value represents a<br />

rate of return of 10.67% per year. (These type of calculations are explained later.) This past<br />

i nvestment has well exceeded the 8% per year criterion over the last 3 years.<br />

1.2 Performing an <strong>Engineering</strong> <strong>Economy</strong> Study<br />

An engineering economy study involves many elements: problem identification, definition of the<br />

objective, cash flow estimation, financial analysis, and decision making. Implementing a structured<br />

procedure is the best approach to select the best solution to the problem.<br />

The steps in an engineering economy study are as follows:<br />

1. Identify and understand the problem; identify the objective of the project.<br />

2. Collect relevant, available data and define viable solution alternatives.<br />

3. Make realistic cash flow estimates.<br />

4. Identify an economic measure of worth criterion for decision making.


1.2 Performing an <strong>Engineering</strong> <strong>Economy</strong> Study 5<br />

5. Evaluate each alternative; consider noneconomic factors; use sensitivity analysis as needed.<br />

6. Select the best alternative.<br />

7. Implement the solution and monitor the results.<br />

Technically, the last step is not part of the economy study, but it is, of course, a step needed to<br />

meet the project objective. There may be occasions when the best economic alternative<br />

requires more capital funds than are available, or significant noneconomic factors preclude the<br />

most economic alternative from being chosen. Accordingly, steps 5 and 6 may result in selection<br />

of an alternative different from the economically best one. Also, sometimes more than one project<br />

may be selected and implemented. This occurs when projects are independent of one another.<br />

In this case, steps 5 through 7 vary from those above. Figure 1–1 illustrates the steps above for<br />

one alternative. Descriptions of several of the elements in the steps are important to understand.<br />

Problem Description and Objective Statement A succinct statement of the problem and<br />

primary objective(s) is very important to the formation of an alternative solution. As an illustration,<br />

assume the problem is that a coal-fueled power plant must be shut down by 2015 due to the<br />

production of excessive sulfur dioxide. The objectives may be to generate the forecasted electricity<br />

Step in<br />

study<br />

1<br />

Problem<br />

description<br />

Objective<br />

statement<br />

2<br />

Available data<br />

Alternatives for<br />

solution<br />

One or more approaches<br />

to meet objective<br />

3<br />

Cash flows and<br />

other estimates<br />

Expected life<br />

Revenues<br />

Costs<br />

Taxes<br />

Project financing<br />

4<br />

Measure of worth<br />

criterion<br />

PW, ROR, B/C, etc.<br />

5<br />

6<br />

<strong>Engineering</strong><br />

economic analysis<br />

Best alternative<br />

selection<br />

Consider:<br />

• Noneconomic factors<br />

• Sensitivity analysis<br />

• Risk analysis<br />

7<br />

Implementation<br />

and monitoring<br />

Time<br />

passes<br />

1<br />

New problem<br />

description<br />

New engineering<br />

economy study<br />

begins<br />

Figure 1–1<br />

Steps in an engineering economy study.


6 Chapter 1 Foundations of <strong>Engineering</strong> <strong>Economy</strong><br />

needed for 2015 and beyond, plus to not exceed all the projected emission allowances in these<br />

future years.<br />

Alternatives These are stand-alone descriptions of viable solutions to problems that can meet<br />

the objectives. Words, pictures, graphs, equipment and service descriptions, simulations, etc.<br />

define each alternative. The best estimates for parameters are also part of the alternative. Some<br />

parameters include equipment first cost, expected life, salvage value (estimated trade-in, resale,<br />

or market value), and annual operating cost (AOC), which can also be termed maintenance and<br />

operating (M&O) cost, and subcontract cost for specific services. If changes in income (revenue)<br />

may occur, this parameter must be estimated.<br />

Detailing all viable alternatives at this stage is crucial. For example, if two alternatives are<br />

described and analyzed, one will likely be selected and implementation initiated. If a third, more<br />

attractive method that was available is later recognized, a wrong decision was made.<br />

Cash Flows All cash flows are estimated for each alternative. Since these are future expenditures<br />

and revenues, the results of step 3 usually prove to be inaccurate when an alternative is<br />

actually in place and operating. When cash flow estimates for specific parameters are expected to<br />

vary significantly from a point estimate made now, risk and sensitivity analyses (step 5) are<br />

needed to improve the chances of selecting the best alternative. Sizable variation is usually expected<br />

in estimates of revenues, AOC, salvage values, and subcontractor costs. Estimation of<br />

costs is discussed in Chapter 15, and the elements of variation (risk) and sensitivity analysis are<br />

included throughout the text.<br />

<strong>Engineering</strong> <strong>Economy</strong> Analysis The techniques and computations that you will learn and<br />

use throughout this text utilize the cash flow estimates, time value of money, and a selected<br />

measure of worth. The result of the analysis will be one or more numerical values; this can be<br />

in one of several terms, such as money, an interest rate, number of years, or a probability. In<br />

the end, a selected measure of worth mentioned in the previous section will be used to select<br />

the best alternative.<br />

Before an economic analysis technique is applied to the cash flows, some decisions about<br />

what to include in the analysis must be made. Two important possibilities are taxes and<br />

inflation. Federal, state or provincial, county, and city taxes will impact the costs of every<br />

alternative. An after-tax analysis includes some additional estimates and methods compared to<br />

a before-tax a nalysis. If taxes and inflation are expected to impact all alternatives equally, they<br />

may be disregarded in the analysis. However, if the size of these projected costs is important,<br />

taxes and inflation should be considered. Also, if the impact of inflation over time is important<br />

to the decision, an additional set of computations must be added to the analysis; Chapter 14<br />

covers the details.<br />

Selection of the Best Alternative The measure of worth is a primary basis for selecting<br />

the best economic alternative. For example, if alternative A has a rate of return (ROR) of<br />

15.2% per year and alternative B will result in an ROR of 16.9% per year, B is better economically.<br />

However, there can always be noneconomic or intangible factors that must be<br />

considered and that may alter the decision. There are many possible noneconomic factors;<br />

some typical ones are<br />

• Market pressures, such as need for an increased international presence<br />

• Availability of certain resources, e.g., skilled labor force, water, power, tax incentives<br />

• Government laws that dictate safety, environmental, legal, or other aspects<br />

• Corporate management’s or the board of director’s interest in a particular alternative<br />

• Goodwill offered by an alternative toward a group: employees, union, county, etc.<br />

As indicated in Figure 1–1 , once all the economic, noneconomic, and risk factors have been<br />

evaluated, a final decision of the “best” alternative is made.<br />

At times, only one viable alternative is identified. In this case, the do-nothing (DN) alternative<br />

may be chosen provided the measure of worth and other factors result in the alternative being<br />

a poor choice. The do-nothing alternative maintains the status quo.


1.3 Professional Ethics and Economic Decisions 7<br />

Whether we are aware of it or not, we use criteria every day to choose between alternatives.<br />

For example, when you drive to campus, you decide to take the “best” route. But how did you<br />

define best? Was the best route the safest, shortest, fastest, cheapest, most scenic, or what? Obviously,<br />

depending upon which criterion or combination of criteria is used to identify the best, a<br />

different route might be selected each time. In economic analysis, financial units (dollars or<br />

other currency) are generally used as the tangible basis for evaluation. Thus, when there are<br />

several ways of accomplishing a stated objective, the alternative with the lowest overall cost or<br />

highest overall net income is selected.<br />

1.3 Professional Ethics and Economic Decisions<br />

Many of the fundamentals of engineering ethics are intertwined with the roles of money and<br />

economics-based decisions in the making of professionally ethical judgments. Some of these<br />

integral connections are discussed here, plus sections in later chapters discuss additional aspects<br />

of ethics and economics. For example, Chapter 9, Benefit/Cost Analysis and Public Sector Economics,<br />

includes material on the ethics of public project contracts and public policy. Although it<br />

is very limited in scope and space, it is anticipated that this coverage of the important role of<br />

economics in engineering ethics will prompt further interest on the part of students and instructors<br />

of engineering economy.<br />

The terms morals and ethics are commonly used interchangeably, yet they have slightly<br />

different interpretations. Morals usually relate to the underlying tenets that form the character<br />

and conduct of a person in judging right and wrong. Ethical practices can be evaluated by<br />

using a code of morals or code of ethics that forms the standards to guide decisions and<br />

actions of individuals and organizations in a profession, for example, electrical, chemical,<br />

mechanical, industrial, or civil engineering. There are several different levels and types of<br />

morals and ethics.<br />

Universal or common morals These are fundamental moral beliefs held by virtually all people.<br />

Most people agree that to steal, murder, lie, or physically harm someone is wrong.<br />

It is possible for actions and intentions to come into conflict concerning a common moral.<br />

Consider the World Trade Center buildings in New York City. After their collapse on September 11,<br />

2001, it was apparent that the design was not sufficient to withstand the heat generated by the<br />

firestorm caused by the impact of an aircraft. The structural engineers who worked on the design<br />

surely did not have the intent to harm or kill occupants in the buildings. However, their design<br />

actions did not foresee this outcome as a measurable possibility. Did they violate the common<br />

moral belief of not doing harm to others or murdering?<br />

Individual or personal morals These are the moral beliefs that a person has and maintains<br />

over time. These usually parallel the common morals in that stealing, lying, murdering, etc. are<br />

immoral acts.<br />

It is quite possible that an individual strongly supports the common morals and has excellent<br />

personal morals, but these may conflict from time to time when decisions must be made. Consider<br />

the engineering student who genuinely believes that cheating is wrong. If he or she does not<br />

know how to work some test problems, but must make a certain minimum grade on the final<br />

exam to graduate, the decision to cheat or not on the final exam is an exercise in following or<br />

violating a personal moral.<br />

Professional or engineering ethics Professionals in a specific discipline are guided in their<br />

decision making and performance of work activities by a formal standard or code. The code<br />

states the commonly accepted standards of honesty and integrity that each individual is expected<br />

to demonstrate in her or his practice. There are codes of ethics for medical doctors, attorneys,<br />

and, of course, engineers.<br />

Although each engineering profession has its own code of ethics, the Code of Ethics for<br />

Engineers published by the National Society of Professional Engineers (NSPE) is very commonly<br />

used and quoted. This code, reprinted in its entirety in Appendix C, includes numerous<br />

sections that have direct or indirect economic and financial impact upon the designs, actions,


8 Chapter 1 Foundations of <strong>Engineering</strong> <strong>Economy</strong><br />

and decisions that engineers make in their professional dealings. Here are three examples from<br />

the Code:<br />

“Engineers, in the fulfillment of their duties, shall hold paramount the safety, health, and welfare<br />

of the public .” (section I.1)<br />

“Engineers shall not accept fi nancial or other considerations , including free engineering designs,<br />

from material or equipment suppliers for specifying their product.” (section III.5.a)<br />

“Engineers using designs supplied by a client recognize that the designs remain the property<br />

of the client and may not be duplicated by the engineer for others without express permission.”<br />

(section III.9.b)<br />

As with common and personal morals, conflicts can easily rise in the mind of an engineer<br />

between his or her own ethics and that of the employing corporation. Consider a manufacturing<br />

engineer who has recently come to firmly disagree morally with war and its negative effects on<br />

human beings. Suppose the engineer has worked for years in a military defense contractor’s<br />

facility and does the detailed cost estimations and economic evaluations of producing fighter<br />

jets for the Air Force. The Code of Ethics for Engineers is silent on the ethics of producing and<br />

using war materiel. Although the employer and the engineer are not violating any ethics code,<br />

the engineer, as an individual, is stressed in this position. Like many people during a declining<br />

national economy, retention of this job is of paramount importance to the family and the engineer.<br />

Conflicts such as this can place individuals in real dilemmas with no or mostly unsatisfactory<br />

alternatives.<br />

At first thought, it may not be apparent how activities related to engineering economics may<br />

present an ethical challenge to an individual, a company, or a public servant in government service.<br />

Many money-related situations, such as those that follow, can have ethical dimensions.<br />

In the design stage:<br />

• Safety factors are compromised to ensure that a price bid comes in as low as possible.<br />

• Family or personal connections with individuals in a company offer unfair or insider information<br />

that allows costs to be cut in strategic areas of a project.<br />

• A potential vendor offers specifications for company-specific equipment, and the design engineer<br />

does not have sufficient time to determine if this equipment will meet the needs of the<br />

project being designed and costed.<br />

While the system is operating:<br />

• Delayed or below-standard maintenance can be performed to save money when cost overruns<br />

exist in other segments of a project.<br />

• Opportunities to purchase cheaper repair parts can save money for a subcontractor working on<br />

a fixed-price contract.<br />

• Safety margins are compromised because of cost, personal inconvenience to workers, tight<br />

time schedules, etc.<br />

A good example of the last item—safety is compromised while operating the system—is the<br />

situation that arose in 1984 in Bhopal, India (Martin and Schinzinger 2005, pp. 245–8). A Union<br />

Carbide plant manufacturing the highly toxic pesticide chemical methyl isocyanate (MIC) experienced<br />

a large gas leak from high-pressure tanks. Some 500,000 persons were exposed to inhalation<br />

of this deadly gas that burns moist parts of the body. There were 2500 to 3000 deaths within<br />

days, and over the following 10-year period, some 12,000 death claims and 870,000 personal<br />

injury claims were recorded. Although Union Carbide owned the facility, the Indian government<br />

had only Indian workers in the plant. Safety practices clearly eroded due to cost-cutting measures,<br />

insufficient repair parts, and reduction in personnel to save salary money. However, one of<br />

the surprising practices that caused unnecessary harm to workers was the fact that masks, gloves,<br />

and other protective gear were not worn by workers in close proximity to the tanks containing<br />

MIC. Why? Unlike in plants in the United States and other countries, there was no air conditioning<br />

in the Indian plant, resulting in high ambient temperatures in the facility.<br />

Many ethical questions arise when corporations operate in international settings where the<br />

corporate rules, worker incentives, cultural practices, and costs in the home country differ from<br />

those in the host country. Often these ethical dilemmas are fundamentally based in the economics<br />

that provide cheaper labor, reduced raw material costs, less government oversight, and a host of


1.3 Professional Ethics and Economic Decisions 9<br />

other cost-reducing factors. When an engineering economy study is performed, it is important for<br />

the engineer performing the study to consider all ethically related matters to ensure that the cost<br />

and revenue estimates reflect what is likely to happen once the project or system is operating.<br />

It is important to understand that the translation from universal morals to personal morals and<br />

professional ethics does vary from one culture and country to another. As an example, consider the<br />

common belief (universal moral) that the awarding of contracts and financial arrangements for services<br />

to be performed (for government or business) should be accomplished in a fair and transparent<br />

fashion. In some societies and cultures, corruption in the process of contract making is common and<br />

often “overlooked” by the local authorities, who may also be involved in the affairs. Are these immoral<br />

or unethical practices? Most would say, “Yes, this should not be allowed. Find and punish the<br />

individuals involved.” Yet, such practices do continue, thus indicating the differences in interpretation<br />

of common morals as they are translated into the ethics of individuals and professionals.<br />

EXAMPLE 1.2<br />

Jamie is an engineer employed by Burris, a United States–based company that develops subway<br />

and surface transportation systems for medium-sized municipalities in the United States<br />

and Canada. He has been a registered professional engineer (PE) for the last 15 years. Last<br />

year, Carol, an engineer friend from university days who works as an individual consultant,<br />

asked Jamie to help her with some cost estimates on a metro train job. Carol offered to pay for<br />

his time and talent, but Jamie saw no reason to take money for helping with data commonly<br />

used by him in performing his job at Burris. The estimates took one weekend to complete, and<br />

once Jamie delivered them to Carol, he did not hear from her again; nor did he learn the identity<br />

of the company for which Carol was preparing the estimates.<br />

Yesterday, Jamie was called into his supervisor’s office and told that Burris had not received<br />

the contract award in Sharpstown, where a metro system is to be installed. The project estimates<br />

were prepared by Jamie and others at Burris over the past several months. This job was<br />

greatly needed by Burris, as the country and most municipalities were in a real economic<br />

slump, so much so that Burris was considering furloughing several engineers if the Sharpstown<br />

bid was not accepted. Jamie was told he was to be laid off immediately, not because the bid was<br />

rejected, but because he had been secretly working without management approval for a prime<br />

consultant of Burris’ main competitor. Jamie was astounded and angry. He knew he had done<br />

nothing to warrant firing, but the evidence was clearly there. The numbers used by the competitor<br />

to win the Sharpstown award were the same numbers that Jamie had prepared for Burris<br />

on this bid, and they closely matched the values that he gave Carol when he helped her.<br />

Jamie was told he was fortunate, because Burris’ president had decided to not legally charge<br />

Jamie with unethical behavior and to not request that his PE license be rescinded. As a result,<br />

Jamie was escorted out of his office and the building within one hour and told to not ask anyone<br />

at Burris for a reference letter if he attempted to get another engineering job.<br />

Discuss the ethical dimensions of this situation for Jamie, Carol, and Burris’ management.<br />

Refer to the NSPE Code of Ethics for Engineers (Appendix C) for specific points of concern.<br />

Solution<br />

There are several obvious errors and omissions present in the actions of Jamie, Carol, and<br />

B urris’ management in this situation. Some of these mistakes, oversights, and possible code<br />

violations are summarized here.<br />

Jamie<br />

• Did not learn identity of company Carol was working for and whether the company was to<br />

be a bidder on the Sharpstown project<br />

• Helped a friend with confidential data, probably innocently, without the knowledge or approval<br />

of his employer<br />

• Assisted a competitor, probably unknowingly, without the knowledge or approval of his<br />

employer<br />

• Likely violated, at least, Code of Ethics for Engineers section II.1.c, which reads, “Engineers<br />

shall not reveal facts, data, or information without the prior consent of the client or<br />

employer except as authorized or required by law or this Code.”


10 Chapter 1 Foundations of <strong>Engineering</strong> <strong>Economy</strong><br />

Carol<br />

• Did not share the intended use of Jamie’s work<br />

• Did not seek information from Jamie concerning his employer’s intention to bid on the<br />

same project as her client<br />

• Misled Jamie in that she did not seek approval from Jamie to use and quote his information<br />

and assistance<br />

• Did not inform her client that portions of her work originated from a source employed by a<br />

possible bid competitor<br />

• Likely violated, at least, Code of Ethics for Engineers section III.9.a, which reads, “Engineers<br />

shall, whenever possible, name the person or persons who may be individually responsible<br />

for designs, inventions, writings, or other accomplishments.”<br />

Burris’ management<br />

• Acted too fast in dismissing Jamie; they should have listened to Jamie and conducted an<br />

investigation<br />

• Did not put him on administrative leave during a review<br />

• Possibly did not take Jamie’s previous good work record into account<br />

These are not all ethical considerations; some are just plain good business practices for Jamie,<br />

Carol, and Burris.<br />

1.4 Interest Rate and Rate of Return<br />

Interest is the manifestation of the time value of money. Computationally, interest is the difference<br />

between an ending amount of money and the beginning amount. If the difference is zero or negative,<br />

there is no interest. There are always two perspectives to an amount of interest—interest paid<br />

and interest earned. These are illustrated in Figure 1–2 . Interest is paid when a person or organization<br />

borrowed money (obtained a loan) and repays a larger amount over time. Interest is earned<br />

when a person or organization saved, invested, or lent money and obtains a return of a larger<br />

amount over time. The numerical values and formulas used are the same for both perspectives, but<br />

the interpretations are different.<br />

Interest paid on borrowed funds (a loan) is determined using the original amount, also called<br />

the principal,<br />

Interest amount owed now principal [1.1]<br />

When interest paid over a specifi c time unit is expressed as a percentage of the principal, the result<br />

is called the interest rate.<br />

interest accrued per time unit<br />

Interest rate (%) ————————————— 100% [1.2]<br />

principal<br />

The time unit of the rate is called the interest period. By far the most common interest period<br />

used to state an interest rate is 1 year. Shorter time periods can be used, such as 1% per month.<br />

Thus, the interest period of the interest rate should always be included. If only the rate is stated,<br />

for example, 8.5%, a 1-year interest period is assumed.<br />

Loan<br />

Loan<br />

Bank<br />

Repayment<br />

interest<br />

Borrower<br />

Investor<br />

(a)<br />

Figure 1–2<br />

(a) Interest paid over time to lender. (b) Interest earned over time by investor.<br />

Repayment<br />

interest<br />

(b)<br />

Corporation


1.4 Interest Rate and Rate of Return 11<br />

EXAMPLE 1.3<br />

An employee at LaserKinetics.com borrows $10,000 on May 1 and must repay a total of<br />

$10,700 exactly 1 year later. Determine the interest amount and the interest rate paid.<br />

Solution<br />

The perspective here is that of the borrower since $10,700 repays a loan. Apply Equation [1.1]<br />

to determine the interest paid.<br />

Interest paid $10,700 10,000 $700<br />

Equation [1.2] determines the interest rate paid for 1 year.<br />

$700<br />

Percent interest rate ———— 100% 7% per year<br />

$10,000<br />

EXAMPLE 1.4<br />

Stereophonics, Inc., plans to borrow $20,000 from a bank for 1 year at 9% interest for new<br />

recording equipment. ( a ) Compute the interest and the total amount due after 1 year. ( b ) Construct<br />

a column graph that shows the original loan amount and total amount due after 1 year<br />

used to compute the loan interest rate of 9% per year.<br />

Solution<br />

(a) Compute the total interest accrued by solving Equation [1.2] for interest accrued.<br />

Interest $20,000(0.09) $1800<br />

The total amount due is the sum of principal and interest.<br />

Total due $20,000 1800 $21,800<br />

(b) Figure 1–3 shows the values used in Equation [1.2]: $1800 interest, $20,000 original loan<br />

principal, 1-year interest period.<br />

$<br />

$21,800<br />

$20,000<br />

Original<br />

loan<br />

amount<br />

Interest = $1800<br />

Interest rate<br />

$1800<br />

$20,000 100%<br />

= 9% per year<br />

Now<br />

1 year<br />

later<br />

Interest<br />

period is<br />

1 year<br />

Figure 1–3<br />

Values used to compute an interest rate of 9% per year. Example 1.4.<br />

Comment<br />

Note that in part ( a ), the total amount due may also be computed as<br />

Total due principal(1 interest rate) $20,000(1.09) $21,800<br />

Later we will use this method to determine future amounts for times longer than one interest<br />

period.


12 Chapter 1 Foundations of <strong>Engineering</strong> <strong>Economy</strong><br />

From the perspective of a saver, a lender, or an investor, interest earned (Figure 1–2 b ) is the<br />

final amount minus the initial amount, or principal.<br />

Interest earned total amount now principal [1.3]<br />

Interest earned over a specific period of time is expressed as a percentage of the original amount<br />

and is called rate of return (ROR).<br />

interest accrued per time unit<br />

Rate of return (%) ————————————— 100% [1.4]<br />

principal<br />

The time unit for rate of return is called the interest period, just as for the borrower’s perspective.<br />

Again, the most common period is 1 year.<br />

The term return on investment (ROI) is used equivalently with ROR in different industries and<br />

settings, especially where large capital funds are committed to engineering-oriented programs.<br />

The numerical values in Equations [1.2] and [1.4] are the same, but the term interest rate paid<br />

is more appropriate for the borrower’s perspective, while the rate of return earned is better for<br />

the investor’s perspective.<br />

EXAMPLE 1.5<br />

(a) Calculate the amount deposited 1 year ago to have $1000 now at an interest rate of 5%<br />

per year.<br />

(b) Calculate the amount of interest earned during this time period.<br />

Solution<br />

(a) The total amount accrued ($1000) is the sum of the original deposit and the earned interest.<br />

If X is the original deposit,<br />

Total accrued deposit deposit(interest rate)<br />

$1000 X X (0.05) X (1 0.05) 1.05 X<br />

The original deposit is<br />

X ——— 1000<br />

1.05 $952.38<br />

(b) Apply Equation [1.3] to determine the interest earned.<br />

Interest $1000 952.38 $47.62<br />

In Examples 1.3 to 1.5 the interest period was 1 year, and the interest amount was calculated<br />

at the end of one period. When more than one interest period is involved, e.g., the amount of interest<br />

after 3 years, it is necessary to state whether the interest is accrued on a simple or compound<br />

basis from one period to the next. This topic is covered later in this chapter.<br />

Since inflation can significantly increase an interest rate, some comments about the fundamentals<br />

of inflation are warranted at this early stage. By definition, inflation represents a decrease<br />

in the value of a given currency. That is, $10 now will not purchase the same amount of gasoline<br />

for your car (or most other things) as $10 did 10 years ago. The changing value of the currency<br />

affects market interest rates.<br />

Inflation<br />

In simple terms, interest rates reflect two things: a so-called real rate of return plus the expected<br />

inflation rate. The real rate of return allows the investor to purchase more than he or she could<br />

have purchased before the investment, while inflation raises the real rate to the market rate that<br />

we use on a daily basis.<br />

The safest investments (such as government bonds) typically have a 3% to 4% real rate of<br />

return built into their overall interest rates. Thus, a market interest rate of, say, 8% per year on a<br />

bond means that investors expect the inflation rate to be in the range of 4% to 5% per year.<br />

Clearly, inflation causes interest rates to rise.<br />

From the borrower’s perspective, the rate of inflation is another interest rate tacked on to the<br />

real interest rate . And from the vantage point of the saver or investor in a fixed-interest account,


1.5 Terminology and Symbols 13<br />

inflation reduces the real rate of return on the investment. Inflation means that cost and revenue<br />

cash flow estimates increase over time. This increase is due to the changing value of money that<br />

is forced upon a country’s currency by inflation, thus making a unit of currency (such as the dollar)<br />

worth less relative to its value at a previous time. We see the effect of inflation in that money<br />

purchases less now than it did at a previous time. Inflation contributes to<br />

• A reduction in purchasing power of the currency<br />

• An increase in the CPI (consumer price index)<br />

• An increase in the cost of equipment and its maintenance<br />

• An increase in the cost of salaried professionals and hourly employees<br />

• A reduction in the real rate of return on personal savings and certain corporate investments<br />

In other words, inflation can materially contribute to changes in corporate and personal economic<br />

analysis.<br />

Commonly, engineering economy studies assume that inflation affects all estimated values<br />

equally. Accordingly, an interest rate or rate of return, such as 8% per year, is applied throughout<br />

the analysis without accounting for an additional inflation rate. However, if inflation were explicitly<br />

taken into account, and it was reducing the value of money at, say, an average of 4% per year,<br />

then it would be necessary to perform the economic analysis using an inflated interest rate. (The<br />

rate is 12.32% per year using the relations derived in Chapter 14.)<br />

1.5 Terminology and Symbols<br />

The equations and procedures of engineering economy utilize the following terms and symbols.<br />

Sample units are indicated.<br />

P value or amount of money at a time designated as the present or time 0. Also P is<br />

referred to as present worth (PW), present value (PV), net present value (NPV), discounted<br />

cash flow (DCF), and capitalized cost (CC); monetary units, such as dollars<br />

F value or amount of money at some future time. Also F is called future worth (FW)<br />

and future value (FV); dollars<br />

A series of consecutive, equal, end-of-period amounts of money. Also A is called the<br />

annual worth (AW) and equivalent uniform annual worth (EUAW); dollars per<br />

year, euros per month<br />

n number of interest periods; years, months, days<br />

i interest rate per time period; percent per year, percent per month<br />

t time, stated in periods; years, months, days<br />

The symbols P and F represent one-time occurrences: A occurs with the same value in each interest<br />

period for a specified number of periods. It should be clear that a present value P represents a<br />

single sum of money at some time prior to a future value F or prior to the first occurrence of an<br />

equivalent series amount A .<br />

It is important to note that the symbol A always represents a uniform amount (i.e., the same<br />

amount each period) that extends through consecutive interest periods. Both conditions must<br />

exist before the series can be represented by A .<br />

The interest rate i is expressed in percent per interest period, for example, 12% per year. Unless<br />

stated otherwise, assume that the rate applies throughout the entire n years or interest periods.<br />

The decimal equivalent for i is always used in formulas and equations in engineering economy<br />

computations.<br />

All engineering economy problems involve the element of time expressed as n and interest<br />

rate i . In general, every problem will involve at least four of the symbols P , F , A , n , and i , with at<br />

least three of them estimated or known.<br />

Additional symbols used in engineering economy are defined in Appendix E.<br />

EXAMPLE 1.6<br />

Today, Julie borrowed $5000 to purchase furniture for her new house. She can repay the loan<br />

in either of the two ways described below. Determine the engineering economy symbols and<br />

their value for each option.


14 Chapter 1 Foundations of <strong>Engineering</strong> <strong>Economy</strong><br />

(a) Five equal annual installments with interest based on 5% per year.<br />

(b) One payment 3 years from now with interest based on 7% per year.<br />

Solution<br />

(a) The repayment schedule requires an equivalent annual amount A , which is unknown.<br />

P $5000 i 5% per year n 5 years A ?<br />

(b) Repayment requires a single future amount F, which is unknown.<br />

P $5000 i 7% per year n 3 years F ?<br />

EXAMPLE 1.7<br />

You plan to make a lump-sum deposit of $5000 now into an investment account that pays 6%<br />

per year, and you plan to withdraw an equal end-of-year amount of $1000 for 5 years, starting<br />

next year. At the end of the sixth year, you plan to close your account by withdrawing the remaining<br />

money. Define the engineering economy symbols involved.<br />

Solution<br />

All five symbols are present, but the future value in year 6 is the unknown.<br />

P $5000<br />

A $1000 per year for 5 years<br />

F ? at end of year 6<br />

i 6% per year<br />

n 5 years for the A series and 6 for the F value<br />

EXAMPLE 1.8<br />

Last year Jane’s grandmother offered to put enough money into a savings account to generate<br />

$5000 in interest this year to help pay Jane’s expenses at college. ( a ) Identify the symbols, and<br />

( b ) calculate the amount that had to be deposited exactly 1 year ago to earn $5000 in interest<br />

now, if the rate of return is 6% per year.<br />

Solution<br />

(a) Symbols P (last year is 1) and F (this year) are needed.<br />

P ?<br />

i 6% per year<br />

n 1 year<br />

F P interest ? $5000<br />

(b) Let F total amount now and P original amount. We know that F – P $5000 is<br />

accrued interest. Now we can determine P . Refer to Equations [1.1] through [1.4].<br />

F P Pi<br />

The $5000 interest can be expressed as<br />

Interest F – P ( P Pi ) – P<br />

Pi<br />

$5000 P (0.06)<br />

P $5000 ———<br />

0.06 $83,333.33


1.6 Cash Flows: Estimation and Diagramming 15<br />

1.6 Cash Flows: Estimation and Diagramming<br />

As mentioned in earlier sections, cash flows are the amounts of money estimated for future projects<br />

or observed for project events that have taken place. All cash flows occur during specific time periods,<br />

such as 1 month, every 6 months, or 1 year. Annual is the most common time period. For<br />

example, a payment of $10,000 once every year in December for 5 years is a series of 5 outgoing<br />

cash flows. And an estimated receipt of $500 every month for 2 years is a series of 24 incoming cash<br />

flows. <strong>Engineering</strong> economy bases its computations on the timing, size, and direction of cash flows.<br />

Cash inflows are the receipts, revenues, incomes, and savings generated by project and business<br />

activity. A plus sign indicates a cash inflow.<br />

Cash outflows are costs, disbursements, expenses, and taxes caused by projects and business<br />

activity. A negative or minus sign indicates a cash outflow. When a project involves only costs,<br />

the minus sign may be omitted for some techniques, such as benefit/cost analysis.<br />

Cash flow<br />

Of all the steps in Figure 1–1 that outline the engineering economy study, estimating cash flows<br />

(step 3) is the most difficult, primarily because it is an attempt to predict the future. Some examples<br />

of cash flow estimates are shown here. As you scan these, consider how the cash inflow<br />

or outflow may be estimated most accurately.<br />

Cash Inflow Estimates<br />

Income: $150,000 per year from sales of solar-powered watches<br />

Savings: $24,500 tax savings from capital loss on equipment salvage<br />

Receipt: $750,000 received on large business loan plus accrued interest<br />

Savings: $150,000 per year saved by installing more efficient air conditioning<br />

Revenue: $50,000 to $75,000 per month in sales for extended battery life iPhones<br />

Cash Outflow Estimates<br />

Operating costs: $230,000 per year annual operating costs for software services<br />

First cost: $800,000 next year to purchase replacement earthmoving equipment<br />

Expense: $20,000 per year for loan interest payment to bank<br />

Initial cost: $1 to $1.2 million in capital expenditures for a water recycling unit<br />

All of these are point estimates , that is, single-value estimates for cash flow elements of an<br />

alternative, except for the last revenue and cost estimates listed above. They provide a range estimate,<br />

because the persons estimating the revenue and cost do not have enough knowledge or experience<br />

with the systems to be more accurate. For the initial chapters, we will utilize point estimates. The use<br />

of risk and sensitivity analysis for range estimates is covered in the later chapters of this book.<br />

Once all cash inflows and outflows are estimated (or determined for a completed project), the<br />

net cash flow for each time period is calculated.<br />

Net cash flow cash inflows cash outflows [1.5]<br />

NCF R D [1.6]<br />

where NCF is net cash flow, R is receipts, and D is disbursements.<br />

At the beginning of this section, the timing, size, and direction of cash fl ows were mentioned<br />

as important. Because cash flows may take place at any time during an interest period, as a matter<br />

of convention, all cash flows are assumed to occur at the end of an interest period.<br />

The end-of-period convention means that all cash inflows and all cash outflows are assumed to<br />

take place at the end of the interest period in which they actually occur. When several inflows<br />

and outflows occur within the same period, the net cash flow is assumed to occur at the end of<br />

the period.<br />

End-of-period convention


16 Chapter 1 Foundations of <strong>Engineering</strong> <strong>Economy</strong><br />

Figure 1–4<br />

A typical cash flow time<br />

scale for 5 years.<br />

Year 1<br />

0 1<br />

2<br />

3 4<br />

Time scale<br />

Year 5<br />

5<br />

Figure 1–5<br />

Example of positive and<br />

negative cash flows.<br />

+<br />

i = 4% per year<br />

F =?<br />

Cash flow<br />

1<br />

2<br />

3<br />

4<br />

5<br />

Year<br />

–<br />

In assuming end-of-period cash flows, it is important to understand that future (F) and uniform<br />

annual (A) amounts are located at the end of the interest period, which is not necessarily<br />

December 31. If in Example 1.7 the lump-sum deposit took place on July 1, 2011, the withdrawals<br />

will take place on July 1 of each succeeding year for 6 years. Remember, end of the period<br />

means end of interest period, not end of calendar year.<br />

The cash flow diagram is a very important tool in an economic analysis, especially when the<br />

cash flow series is complex. It is a graphical representation of cash flows drawn on the y axis with<br />

a time scale on the x axis. The diagram includes what is known, what is estimated, and what is<br />

needed. That is, once the cash flow diagram is complete, another person should be able to work<br />

the problem by looking at the diagram.<br />

Cash flow diagram time t 0 is the present, and t 1 is the end of time period 1. We assume<br />

that the periods are in years for now. The time scale of Figure 1–4 is set up for 5 years. Since the<br />

end-of-year convention places cash flows at the ends of years, the “1” marks the end of year 1.<br />

While it is not necessary to use an exact scale on the cash flow diagram, you will probably<br />

avoid errors if you make a neat diagram to approximate scale for both time and relative cash flow<br />

magnitudes.<br />

The direction of the arrows on the diagram is important to differentiate income from outgo. A<br />

vertical arrow pointing up indicates a positive cash flow. Conversely, a down-pointing arrow indicates<br />

a negative cash flow. We will use a bold, colored arrow to indicate what is unknown<br />

and to be determined. For example, if a future value F is to be determined in year 5, a wide,<br />

colored arrow with F ? is shown in year 5. The interest rate is also indicated on the diagram.<br />

Figure 1–5 illustrates a cash inflow at the end of year 1, equal cash outflows at the end of years 2<br />

and 3, an interest rate of 4% per year, and the unknown future value F after 5 years. The arrow<br />

for the unknown value is generally drawn in the opposite direction from the other cash flows;<br />

however, the engineering economy computations will determine the actual sign on the F value.<br />

Before the diagramming of cash flows, a perspective or vantage point must be determined so<br />

that or – signs can be assigned and the economic analysis performed correctly. Assume you<br />

borrow $8500 from a bank today to purchase an $8000 used car for cash next week, and you plan<br />

to spend the remaining $500 on a new paint job for the car two weeks from now. There are several<br />

perspectives possible when developing the cash flow diagram—those of the borrower (that’s<br />

you), the banker, the car dealer, or the paint shop owner. The cash flow signs and amounts for<br />

these perspectives are as follows.<br />

Perspective Activity Cash flow with Sign, $ Time, week<br />

You Borrow 8500 0<br />

Buy car −8000 1<br />

Paint job −500 2<br />

Banker Lender −8500 0<br />

Car dealer Car sale 8000 1<br />

Painter Paint job 500 2


1.6 Cash Flows: Estimation and Diagramming 17<br />

$8500<br />

1<br />

2<br />

0<br />

$500<br />

Week<br />

$8000<br />

Figure 1–6<br />

Cash flows from perspective of borrower for loan and purchases.<br />

One, and only one, of the perspectives is selected to develop the diagram. For your perspective,<br />

all three cash flows are involved and the diagram appears as shown in Figure 1–6 with a time scale<br />

of weeks. Applying the end-of-period convention, you have a receipt of $8500 now (time 0) and<br />

cash outflows of $8000 at the end of week 1, followed by $500 at the end of week 2.<br />

EXAMPLE 1.9<br />

Each year Exxon-Mobil expends large amounts of funds for mechanical safety features<br />

throughout its worldwide operations. Carla Ramos, a lead engineer for Mexico and Central<br />

American operations, plans expenditures of $1 million now and each of the next 4 years just<br />

for the improvement of field-based pressure-release valves. Construct the cash flow diagram to<br />

find the equivalent value of these expenditures at the end of year 4, using a cost of capital estimate<br />

for safety-related funds of 12% per year.<br />

Solution<br />

Figure 1–7 indicates the uniform and negative cash flow series (expenditures) for five periods,<br />

and the unknown F value (positive cash flow equivalent) at exactly the same time as the fifth<br />

expenditure. Since the expenditures start immediately, the first $1 million is shown at time 0,<br />

not time 1. Therefore, the last negative cash flow occurs at the end of the fourth year, when F<br />

also occurs. To make this diagram have a full 5 years on the time scale, the addition of the<br />

year 1 completes the diagram. This addition demonstrates that year 0 is the end-of-period<br />

point for the year 1.<br />

i = 12%<br />

F =?<br />

1<br />

0 1 2<br />

3<br />

4<br />

Year<br />

A = $1,000,000<br />

Figure 1–7<br />

Cash flow diagram, Example 1.9.<br />

EXAMPLE 1.10<br />

An electrical engineer wants to deposit an amount P now such that she can withdraw an equal<br />

annual amount of A 1 $2000 per year for the first 5 years, starting 1 year after the deposit, and<br />

a different annual withdrawal of A 2 $3000 per year for the following 3 years. How would the<br />

cash flow diagram appear if i 8.5% per year?


18 Chapter 1 Foundations of <strong>Engineering</strong> <strong>Economy</strong><br />

Solution<br />

The cash flows are shown in Figure 1–8. The negative cash outflow P occurs now. The withdrawals<br />

(positive cash inflow) for the A 1 series occur at the end of years 1 through 5, and A 2<br />

occurs in years 6 through 8.<br />

A 1 = $2000<br />

A 2 = $3000<br />

0 1 2 3 4 5 6 7<br />

i = 8.5%<br />

8<br />

Year<br />

P =?<br />

Figure 1–8<br />

Cash flow diagram with two different A series, Example 1.10.<br />

EXAMPLE 1.11<br />

A rental company spent $2500 on a new air compressor 7 years ago. The annual rental income<br />

from the compressor has been $750. The $100 spent on maintenance the first year has increased<br />

each year by $25. The company plans to sell the compressor at the end of next year for<br />

$150. Construct the cash flow diagram from the company’s perspective and indicate where the<br />

present worth now is located.<br />

Solution<br />

Let now be time t 0. The incomes and costs for years 7 through 1 (next year) are tabulated<br />

below with net cash flow computed using Equation [1.5]. The net cash flows (one negative,<br />

eight positive) are diagrammed in Figure 1–9 . Present worth P is located at year 0.<br />

End of Year Income Cost Net Cash Flow<br />

−7 $ 0 $2500 $−2500<br />

−6 750 100 650<br />

−5 750 125 625<br />

−4 750 150 600<br />

−3 750 175 575<br />

−2 750 200 550<br />

−1 750 225 525<br />

0 750 250 500<br />

1 750 150 275 625<br />

$650<br />

$625<br />

$600<br />

$575<br />

$550<br />

$525<br />

P =?<br />

$500<br />

$625<br />

–7 –6 –5 –4 –3 –2 –1 0<br />

1<br />

Year<br />

$2500<br />

Figure 1–9<br />

Cash flow diagram, Example 1.11.


1.7 Economic Equivalence 19<br />

1.7 Economic Equivalence<br />

Economic equivalence is a fundamental concept upon which engineering economy computations<br />

are based. Before we delve into the economic aspects, think of the many types of equivalency we<br />

may utilize daily by transferring from one scale to another. Some example transfers between<br />

scales are as follows:<br />

Length:<br />

12 inches 1 foot 3 feet 1 yard 39.370 inches 1 meter<br />

100 centimeters 1 meter 1000 meters 1 kilometer 1 kilometer 0.621 mile<br />

Pressure:<br />

1 atmosphere 1 newton/meter 2 10 3 pascal 1 kilopascal<br />

Often equivalency involves two or more scales. Consider the equivalency of a speed of 110 kilometers<br />

per hour (kph) into miles per minute using conversions between distance and time scales<br />

with three-decimal accuracy.<br />

Speed:<br />

1 mile 1.609 kilometers 1 hour 60 minutes<br />

110 kph 68.365 miles per hour (mph) 68.365 mph 1.139 miles per minute<br />

Four scales—time in minutes, time in hours, length in miles, and length in kilometers—are<br />

combined to develop these equivalent statements on speed. Note that throughout these statements,<br />

the fundamental relations of 1 mile 1.609 kilometers and 1 hour 60 minutes are<br />

applied. If a fundamental relation changes, the entire equivalency is in error.<br />

Now we consider economic equivalency.<br />

Economic equivalence is a combination of interest rate and time value of money to determine<br />

the different amounts of money at different points in time that are equal in economic<br />

value.<br />

Economic equivalence<br />

As an illustration, if the interest rate is 6% per year, $100 today (present time) is equivalent to<br />

$106 one year from today.<br />

Amount accrued 100 100(0.06) 100(1 0.06) $106<br />

If someone offered you a gift of $100 today or $106 one year from today, it would make no difference<br />

which offer you accepted from an economic perspective. In either case you have $106<br />

one year from today. However, the two sums of money are equivalent to each other only when the<br />

interest rate is 6% per year. At a higher or lower interest rate, $100 today is not equivalent to $106<br />

one year from today.<br />

In addition to future equivalence, we can apply the same logic to determine equivalence for<br />

previous years. A total of $100 now is equivalent to $1001.06 $94.34 one year ago at an<br />

interest rate of 6% per year. From these illustrations, we can state the following: $94.34 last<br />

year, $100 now, and $106 one year from now are equivalent at an interest rate of 6% per year.<br />

The fact that these sums are equivalent can be verified by computing the two interest rates for<br />

1-year interest periods.<br />

$6<br />

——— 100% 6% per year<br />

$100<br />

and<br />

$5.66<br />

——— 100% 6% per year<br />

$94.34<br />

The cash flow diagram in Figure 1–10 indicates the amount of interest needed each year to make<br />

these three different amounts equivalent at 6% per year.


20 Chapter 1 Foundations of <strong>Engineering</strong> <strong>Economy</strong><br />

i = 6% per year<br />

100<br />

94.34<br />

$5.66 interest<br />

$6.00 interest<br />

Amount, $<br />

50<br />

0<br />

1<br />

Last<br />

year<br />

0 1 Time<br />

Now Next<br />

year<br />

Figure 1–10<br />

Equivalence of money at 6% per year interest.<br />

EXAMPLE 1.12<br />

Manufacturers make backup batteries for computer systems available to Batteries dealers<br />

through privately owned distributorships. In general, batteries are stored throughout the year,<br />

and a 5% cost increase is added each year to cover the inventory carrying charge for the distributorship<br />

owner. Assume you own the City Center Batteries outlet. Make the calculations<br />

necessary to show which of the following statements are true and which are false about battery<br />

costs.<br />

(a) The amount of $98 now is equivalent to a cost of $105.60 one year from now.<br />

(b) A truck battery cost of $200 one year ago is equivalent to $205 now.<br />

(c) A $38 cost now is equivalent to $39.90 one year from now.<br />

(d) A $3000 cost now is equivalent to $2887.14 one year earlier.<br />

(e) The carrying charge accumulated in 1 year on an investment of $20,000 worth of<br />

batteries is $1000.<br />

Solution<br />

(a) Total amount accrued 98(1.05) $102.90 $105.60; therefore, it is false. Another<br />

way to solve this is as follows: Required original cost is 105.601.05 $100.57 $98.<br />

(b) Equivalent cost 1 year ago is 205.001.05 $195.24 $200; therefore, it is false.<br />

(c) The cost 1 year from now is $38(1.05) $39.90; true.<br />

(d) Cost now is 2887.14(1.05) $3031.50 $3000; false.<br />

(e) The charge is 5% per year interest, or $20,000(0.05) $1000; true.<br />

Comparison of alternative cash flow series requires the use of equivalence to determine when<br />

the series are economically equal or if one is economically preferable to another. The keys to the<br />

analysis are the interest rate and the timing of the cash flows. Example 1.13 demonstrates how<br />

easy it is to be misled by the size and timing of cash flows.<br />

EXAMPLE 1.13<br />

Howard owns a small electronics repair shop. He wants to borrow $10,000 now and repay it<br />

over the next 1 or 2 years. He believes that new diagnostic test equipment will allow him to<br />

work on a wider variety of electronic items and increase his annual revenue. Howard received<br />

2-year repayment options from banks A and B.


1.8 Simple and Compound Interest 21<br />

Amount to pay, $ per year<br />

Year Bank A Bank B<br />

1 −5,378.05 −5,000.00<br />

2 −5,378.05 −5,775.00<br />

Total paid −10,756.10 −10,775.00<br />

After reviewing these plans, Howard decided that he wants to repay the $10,000 after only<br />

1 year based on the expected increased revenue. During a family conversation, Howard’s<br />

brother-in-law offered to lend him the $10,000 now and take $10,600 after exactly 1 year.<br />

Now Howard has three options and wonders which one to take. Which one is economically<br />

the best?<br />

Solution<br />

The repayment plans for both banks are economically equivalent at the interest rate of 5% per<br />

year. (This is determined by using computations that you will learn in Chapter 2.) Therefore,<br />

Howard can choose either plan even though the bank B plan requires a slightly larger sum of<br />

money over the 2 years.<br />

The brother-in-law repayment plan requires a total of $600 in interest 1 year later plus the<br />

principal of $10,000, which makes the interest rate 6% per year. Given the two 5% per year<br />

options from the banks, this 6% plan should not be chosen as it is not economically better than<br />

the other two. Even though the sum of money repaid is smaller, the timing of the cash flows<br />

and the interest rate make it less desirable. The point here is that cash flows themselves, or<br />

their sums, cannot be relied upon as the primary basis for an economic decision. The interest<br />

rate, timing, and economic equivalence must be considered.<br />

1.8 Simple and Compound Interest<br />

The terms interest, interest period, and interest rate (introduced in Section 1.4) are useful in calculating<br />

equivalent sums of money for one interest period in the past and one period in the future.<br />

However, for more than one interest period, the terms simple interest and compound interest become<br />

important.<br />

Simple interest is calculated using the principal only, ignoring any interest accrued in preceding<br />

interest periods. The total simple interest over several periods is computed as<br />

Simple interest (principal)(number of periods)(interest rate) [1.7]<br />

I Pni<br />

where I is the amount of interest earned or paid and the interest rate i is expressed in decimal form.<br />

EXAMPLE 1.14<br />

GreenTree Financing lent an engineering company $100,000 to retrofit an environmentally<br />

unfriendly building. The loan is for 3 years at 10% per year simple interest. How much money<br />

will the firm repay at the end of 3 years?<br />

Solution<br />

The interest for each of the 3 years is<br />

Interest per year $100,000(0.10) $10,000<br />

Total interest for 3 years from Equation [1.7] is<br />

Total interest $100,000(3)(0.10) $30,000


22 Chapter 1 Foundations of <strong>Engineering</strong> <strong>Economy</strong><br />

The amount due after 3 years is<br />

Total due $100,000 30,000 $130,000<br />

The interest accrued in the first year and in the second year does not earn interest. The interest<br />

due each year is $10,000 calculated only on the $100,000 loan principal.<br />

In most financial and economic analyses, we use compound interest calculations.<br />

For compound interest, the interest accrued for each interest period is calculated on the<br />

principal plus the total amount of interest accumulated in all previous periods. Thus,<br />

compound interest means interest on top of interest.<br />

Compound interest reflects the effect of the time value of money on the interest also. Now the<br />

interest for one period is calculated as<br />

Compound interest (principal all accrued interest)(interest rate) [1.8]<br />

In mathematical terms, the interest I t for time period t may be calculated using the relation.<br />

jt1<br />

I t <br />

(<br />

P I J )( i) [1.9]<br />

j1<br />

EXAMPLE 1.15<br />

Assume an engineering company borrows $100,000 at 10% per year compound interest and<br />

will pay the principal and all the interest after 3 years. Compute the annual interest and total<br />

amount due after 3 years. Graph the interest and total owed for each year, and compare with<br />

the previous example that involved simple interest.<br />

Solution<br />

To include compounding of interest, the annual interest and total owed each year are calculated<br />

by Equation [1.8].<br />

Interest, year 1: 100,000(0.10) $10,000<br />

Total due, year 1: 100,000 10,000 $110,000<br />

Interest, year 2: 110,000(0.10) $11,000<br />

Total due, year 2: 110,000 11,000 $121,000<br />

Interest, year 3: 121,000(0.10) $12,100<br />

Total due, year 3: 121,000 12,100 $133,100<br />

The repayment plan requires no payment until year 3 when all interest and the principal, a total<br />

of $133,100, are due. Figure 1–11 uses a cash flow diagram format to compare end-of-year<br />

( a ) simple and ( b ) compound interest and total amounts owed. The differences due to compounding<br />

are clear. An extra $133,100 – 130,000 $3100 in interest is due for the compounded<br />

interest loan.<br />

Note that while simple interest due each year is constant, the compounded interest due<br />

grows geometrically. Due to this geometric growth of compound interest, the difference between<br />

simple and compound interest accumulation increases rapidly as the time frame increases.<br />

For example, if the loan is for 10 years, not 3, the extra paid for compounding interest<br />

may be calculated to be $59,374.


1.8 Simple and Compound Interest 23<br />

0<br />

1 2 3 Year 0 1 2 3 Year<br />

10<br />

I I I<br />

I is constant 10<br />

I<br />

11<br />

12<br />

I<br />

I<br />

I increases<br />

geometrically<br />

100<br />

100<br />

Amount owed ( $1000)<br />

110<br />

120<br />

Amount owed ( $1000)<br />

110<br />

120<br />

Arithmetic<br />

increase<br />

Geometric<br />

increase<br />

130<br />

130<br />

140<br />

140<br />

(a)<br />

(b)<br />

Figure 1–11<br />

Interest I owed and total amount owed for ( a ) simple interest (Example 1.14) and ( b ) compound interest<br />

(Example 1.15).<br />

A more efficient way to calculate the total amount due after a number of years in Example 1.15<br />

is to utilize the fact that compound interest increases geometrically. This allows us to skip the<br />

year-by-year computation of interest. In this case, the total amount due at the end of each<br />

year is<br />

Year 1: $100,000(1.10)<br />

1<br />

$110,000<br />

Year 2: $100,000(1.10) 2 $121,000<br />

Year 3: $100,000(1.10) 3 $133,100<br />

This allows future totals owed to be calculated directly without intermediate steps. The general<br />

form of the equation is<br />

Total due after n years principal(1 interest rate) n years [1.10]<br />

P (1 i ) n<br />

where i is expressed in decimal form. Equation [1.10] was applied above to obtain the<br />

$133,100 due after 3 years. This fundamental relation will be used many times in the upcoming<br />

chapters.<br />

We can combine the concepts of interest rate, compound interest, and equivalence to demonstrate<br />

that different loan repayment plans may be equivalent, but differ substantially in amounts<br />

paid from one year to another and in the total repayment amount. This also shows that there are<br />

many ways to take into account the time value of money.


24 Chapter 1 Foundations of <strong>Engineering</strong> <strong>Economy</strong><br />

EXAMPLE 1.16<br />

Table 1–1 details four different loan repayment plans described below. Each plan repays a<br />

$5000 loan in 5 years at 8% per year compound interest.<br />

• Plan 1: Pay all at end. No interest or principal is paid until the end of year 5. Interest accumulates<br />

each year on the total of principal and all accrued interest.<br />

• Plan 2: Pay interest annually, principal repaid at end. The accrued interest is paid each<br />

year, and the entire principal is repaid at the end of year 5.<br />

• Plan 3: Pay interest and portion of principal annually. The accrued interest and one-fifth<br />

of the principal (or $1000) are repaid each year. The outstanding loan balance decreases<br />

each year, so the interest (column 2) for each year decreases.<br />

• Plan 4: Pay equal amount of interest and principal. Equal payments are made each year<br />

with a portion going toward principal repayment and the remainder covering the accrued<br />

interest. Since the loan balance decreases at a rate slower than that in plan 3 due to the equal<br />

end-of-year payments, the interest decreases, but at a slower rate.<br />

TABLE 1–1 Different Repayment Schedules Over 5 Years for $5000 at 8% Per Year<br />

Compound Interest<br />

(1)<br />

End of<br />

Year<br />

(2)<br />

Interest Owed<br />

for Year<br />

Plan 1: Pay All at End<br />

(3)<br />

Total Owed at<br />

End of Year<br />

(4)<br />

End-of-Year<br />

Payment<br />

(5)<br />

Total Owed<br />

After Payment<br />

0 $5000.00<br />

1 $400.00 $5400.00 — 5400.00<br />

2 432.00 5832.00 — 5832.00<br />

3 466.56 6298.56 — 6298.56<br />

4 503.88 6802.44 — 6802.44<br />

5 544.20 7346.64 $ – 7346.64<br />

Total $ – 7346.64<br />

Plan 2: Pay Interest Annually; Principal Repaid at End<br />

0 $5000.00<br />

1 $400.00 $5400.00 $400.00 5000.00<br />

2 400.00 5400.00 400.00 5000.00<br />

3 400.00 5400.00 400.00 5000.00<br />

4 400.00 5400.00 400.00 5000.00<br />

5 400.00 5400.00 – 5400.00<br />

Total<br />

Plan 3: Pay Interest and Portion of Principal Annually<br />

$7000.00<br />

0 $5000.00<br />

1 $400.00 $5400.00 $1400.00 4000.00<br />

2 320.00 4320.00 1320.00 3000.00<br />

3 240.00 3240.00 1240.00 2000.00<br />

4 160.00 2160.00 1160.00 1000.00<br />

5 80.00 1080.00 – 1080.00<br />

Total<br />

Plan 4: Pay Equal Annual Amount of Interest and Principal<br />

$6200.00<br />

0 $5000.00<br />

1 $400.00 $5400.00 $−1252.28 4147.72<br />

2 331.82 4479.54 −1252.28 3227.25<br />

3 258.18 3485.43 −1252.28 2233.15<br />

4 178.65 2411.80 −1252.28 1159.52<br />

5 92.76 1252.28 – 1252.28<br />

Total<br />

$−6261.40


1.9 Minimum Attractive Rate of Return 25<br />

(a) Make a statement about the equivalence of each plan at 8% compound interest.<br />

(b) Develop an 8% per year simple interest repayment plan for this loan using the same<br />

approach as plan 2. Comment on the total amounts repaid for the two plans.<br />

Solution<br />

(a) The amounts of the annual payments are different for each repayment schedule, and<br />

the total amounts repaid for most plans are different, even though each repayment<br />

plan requires exactly 5 years. The difference in the total amounts repaid can be explained<br />

by the time value of money and by the partial repayment of principal prior to<br />

year 5.<br />

A loan of $5000 at time 0 made at 8% per year compound interest is equivalent to each<br />

of the following:<br />

Plan 1 $7346.64 at the end of year 5<br />

Plan 2 $400 per year for 4 years and $5400 at the end of year 5<br />

Plan 3 Decreasing payments of interest and partial principal in years 1 ($1400)<br />

through 5 ($1080)<br />

Plan 4 $1252.28 per year for 5 years<br />

An engineering economy study typically uses plan 4; interest is compounded, and a constant<br />

amount is paid each period. This amount covers accrued interest and a partial<br />

amount of principal repayment.<br />

(b) The repayment schedule for 8% per year simple interest is detailed in Table 1–2. Since<br />

the annual accrued interest of $400 is paid each year and the principal of $5000 is repaid<br />

in year 5, the schedule is exactly the same as that for 8% per year compound interest, and<br />

the total amount repaid is the same at $7000. In this unusual case, simple and compound<br />

interest result in the same total repayment amount. Any deviation from this schedule will<br />

cause the two plans and amounts to differ.<br />

TABLE 1–2 A 5-Year Repayment Schedule of $5000 at 8% per Year Simple Interest<br />

End of<br />

Year<br />

Interest Owed<br />

for Year<br />

Total Owed at<br />

End of Year<br />

End-of-Year<br />

Payment<br />

Total Owed<br />

After Payment<br />

0 $5000<br />

1 $400 $5400 $400 5000<br />

2 400 5400 400 5000<br />

3 400 5400 400 5000<br />

4 400 5400 400 5000<br />

5 400 5400 – 5400 0<br />

Total $7000<br />

1.9 Minimum Attractive Rate of Return<br />

For any investment to be profitable, the investor (corporate or individual) expects to receive more<br />

money than the amount of capital invested. In other words, a fair rate of return, or return on investment,<br />

must be realizable. The definition of ROR in Equation [1.4] is used in this discussion,<br />

that is, amount earned divided by the principal.<br />

<strong>Engineering</strong> alternatives are evaluated upon the prognosis that a reasonable ROR can be<br />

expected. Therefore, some reasonable rate must be established for the selection criteria<br />

(step 4) of the engineering economy study ( Figure 1–1 ).


26 Chapter 1 Foundations of <strong>Engineering</strong> <strong>Economy</strong><br />

Minimum Attractive Rate<br />

of Return (MARR)<br />

The Minimum Attractive Rate of Return (MARR) is a reasonable rate of return established<br />

for the evaluation and selection of alternatives. A project is not economically viable unless<br />

it is expected to return at least the MARR. MARR is also referred to as the hurdle rate,<br />

cutoff rate, benchmark rate, and minimum acceptable rate of return.<br />

Cost of capital<br />

Figure 1–12 indicates the relations between different rate of return values. In the United<br />

States, the current U.S. Treasury Bill return is sometimes used as the benchmark safe rate. The<br />

MARR will always be higher than this, or a similar, safe rate. The MARR is not a rate that is<br />

calculated as a ROR. The MARR is established by (financial) managers and is used as a criterion<br />

against which an alternative’s ROR is measured, when making the accept/reject investment<br />

decision.<br />

To develop a foundation-level understanding of how a MARR value is established and used<br />

to make investment decisions, we return to the term capital introduced in Section 1.1. Although<br />

the MARR is used as a criterion to decide on investing in a project, the size of MARR is fundamentally<br />

connected to how much it costs to obtain the needed capital funds. It always costs<br />

money in the form of interest to raise capital. The interest, expressed as a percentage rate per<br />

year, is called the cost of capital. As an example on a personal level, if you want to purchase a<br />

new widescreen HDTV, but do not have sufficient money (capital), you could obtain a bank loan<br />

for, say, a cost of capital of 9% per year and pay for the TV in cash now. Alternatively, you might<br />

choose to use your credit card and pay off the balance on a monthly basis. This approach will<br />

probably cost you at least 15% per year. Or, you could use funds from your savings account that<br />

earns 5% per year and pay cash. This approach means that you also forgo future returns<br />

from these funds. The 9%, 15%, and 5% rates are your cost of capital estimates to raise the<br />

capital for the system by different methods of capital financing. In analogous ways, corporations<br />

estimate the cost of capital from different sources to raise funds for engineering projects and<br />

other types of projects.<br />

Rate of return,<br />

percent<br />

Expected rate of return on<br />

a new proposal<br />

Range for the rate of return on<br />

accepted proposals, if other<br />

proposals were rejected<br />

for some reason<br />

All proposals must offer<br />

at least MARR to<br />

be considered<br />

MARR<br />

Rate of return on<br />

“safe investment”<br />

Figure 1–12<br />

Size of MAAR relative to other rate of return values.


1.10 Introduction to Spreadsheet Use 27<br />

In general, capital is developed in two ways—equity financing and debt financing. A combination<br />

of these two is very common for most projects. Chapter 10 covers these in greater detail, but<br />

a snapshot description follows.<br />

Equity financing The corporation uses its own funds from cash on hand, stock sales, or retained<br />

earnings. Individuals can use their own cash, savings, or investments. In the example above, using<br />

money from the 5% savings account is equity financing.<br />

Debt financing The corporation borrows from outside sources and repays the principal and interest<br />

according to some schedule, much like the plans in Table 1–1. Sources of debt capital may be<br />

bonds, loans, mortgages, venture capital pools, and many others. Individuals, too, can utilize debt<br />

sources, such as the credit card (15% rate) and bank options (9% rate) described above.<br />

Combinations of debt-equity financing mean that a weighted average cost of capital (WACC)<br />

results. If the HDTV is purchased with 40% credit card money at 15% per year and 60% savings<br />

account funds earning 5% per year, the weighted average cost of capital is 0.4(15) 0.6(5) <br />

9% per year.<br />

For a corporation, the established MARR used as a criterion to accept or reject an investment<br />

alternative will usually be equal to or higher than the WACC that the corporation must bear to<br />

obtain the necessary capital funds. So the inequality<br />

ROR MARR WACC [1.11]<br />

must be correct for an accepted project. Exceptions may be government-regulated requirements<br />

(safety, security, environmental, legal, etc.), economically lucrative ventures expected to lead to<br />

other opportunities, etc.<br />

Often there are many alternatives that are expected to yield a ROR that exceeds the MARR as<br />

indicated in Figure 1–12 , but there may not be sufficient capital available for all, or the project’s<br />

risk may be estimated as too high to take the investment chance. Therefore, new projects that are<br />

undertaken usually have an expected return at least as great as the return on another alternative<br />

that is not funded. The expected rate of return on the unfunded project is called the opportunity<br />

cost.<br />

The opportunity cost is the rate of return of a forgone opportunity caused by the inability to<br />

pursue a project. Numerically, it is the largest rate of return of all the projects not accepted<br />

(forgone) due to the lack of capital funds or other resources. When no specific MARR is<br />

established, the de facto MARR is the opportunity cost, i.e., the ROR of the first project not<br />

undertaken due to unavailability of capital funds.<br />

Opportunity cost<br />

As an illustration of opportunity cost, refer to Figure 1–12 and assume a MARR of 12% per<br />

year. Further, assume that a proposal, call it A, with an expected ROR 13% is not funded due<br />

to a lack of capital. Meanwhile, proposal B has a ROR 14.5% and is funded from available<br />

capital. Since proposal A is not undertaken due to the lack of capital, its estimated ROR of 13%<br />

is the opportunity cost; that is, the opportunity to make an additional 13% return is forgone.<br />

1.10 Introduction to Spreadsheet Use<br />

The functions on a computer spreadsheet can greatly reduce the amount of hand work for equivalency<br />

computations involving compound interest and the terms P , F , A , i , and n . The use of a<br />

calculator to solve most simple problems is preferred by many students and professors as described<br />

in Appendix D. However, as cash flow series become more complex, the spreadsheet<br />

offers a good alternative. Microsoft Excel is used throughout this book because it is readily<br />

available and easy to use. Appendix A is a primer on using spreadsheets and Excel. The functions<br />

used in engineering economy are described there in detail, with explanations of all the


28 Chapter 1 Foundations of <strong>Engineering</strong> <strong>Economy</strong><br />

parameters. Appendix A also includes a section on spreadsheet layout that is useful when the<br />

economic analysis is presented to someone else—a coworker, a boss, or a professor.<br />

A total of seven Excel functions can perform most of the fundamental engineering economy<br />

calculations. The functions are great supplemental tools, but they do not replace the understanding<br />

of engineering economy relations, assumptions, and techniques. Using the symbols P , F , A ,<br />

i , and n defined in the previous section, the functions most used in engineering economic analysis<br />

are formulated as follows.<br />

To find the present value P : PV( i%, n, A, F )<br />

To find the future value F : FV( i%, n, A, P )<br />

To find the equal, periodic value A : PMT( i%, n, P, F )<br />

To find the number of periods n : NPER( i%, A, P, F )<br />

To find the compound interest rate i : RATE( n, A, P, F )<br />

To find the compound interest rate i : IRR(first_cell:last_cell)<br />

To find the present value P of any series: NPV( i %, second_cell:last_cell) first_cell<br />

If some of the parameters don’t apply to a particular problem, they can be omitted and zero is<br />

assumed. For readability, spaces can be inserted between parameters within parentheses. If the<br />

parameter omitted is an interior one, the comma must be entered. The last two functions require<br />

that a series of numbers be entered into contiguous spreadsheet cells, but the first five can be used<br />

with no supporting data. In all cases, the function must be preceded by an equals sign () in the<br />

cell where the answer is to be displayed.<br />

To understand how the spreadsheet functions work, look back at Example 1.6 a , where the<br />

equivalent annual amount A is unknown, as indicated by A ?. (In Chapter 2, we learn how<br />

engineering economy factors calculate A , given P , i , and n .) To find A using a spreadsheet<br />

function, simply enter the PMT function PMT(5%,5,5000). Figure 1–13 is a screen image<br />

of a spreadsheet with the PMT function entered into cell B4. The answer ($1154.87) is displayed.<br />

The answer may appear in red and in parentheses, or with a minus sign on your screen<br />

to indicate a negative amount from the perspective of a reduction in the account balance. The<br />

right side of Figure 1–13 presents the solution to Example 1.6 b. The future value F is determined<br />

by using the FV function. The FV function appears in the formula bar; and many examples<br />

throughout this text will include cell tags, as shown here, to indicate the format of<br />

important entries.<br />

The following example demonstrates the use of a spreadsheet to develop relations (not<br />

built-in functions) to calculate interest and cash flows. Once set up, the spreadsheet can be<br />

used to perform sensitivity analysis for estimates that are subject to change. We will illustrate<br />

the use of spreadsheets throughout the chapters. ( Note: The spreadsheet examples may<br />

be omitted, if spreadsheets are not used in the course. A solution by hand is included in virtually<br />

all examples.)<br />

PMT(5%,5,5000)<br />

FV(7%,3,,5000)<br />

Figure 1–13<br />

Use of spreadsheet functions PMT and FV, Example 1.6.


1.10 Introduction to Spreadsheet Use 29<br />

EXAMPLE 1.17<br />

A Japan-based architectural firm has asked a United States–based software engineering group<br />

to infuse GPS sensing capability via satellite into monitoring software for high-rise structures<br />

in order to detect greater than expected horizontal movements. This software could be very<br />

beneficial as an advance warning of serious tremors in earthquake-prone areas in Japan and the<br />

United States. The inclusion of accurate GPS data is estimated to increase annual revenue over<br />

that for the current software system by $200,000 for each of the next 2 years, and by $300,000<br />

for each of years 3 and 4. The planning horizon is only 4 years due to the rapid advances made<br />

internationally in building-monitoring software. Develop spreadsheets to answer the questions<br />

below.<br />

(a) Determine the total interest and total revenue after 4 years, using a compound rate of<br />

r eturn of 8% per year.<br />

(b) Repeat part (a) if estimated revenue increases from $300,000 to $600,000 in years 3 and 4.<br />

(c) Repeat part ( a ) if inflation is estimated to be 4% per year. This will decrease the real rate<br />

of return from 8% to 3.85% per year (Chapter 14 shows why).<br />

Solution by Spreadsheet<br />

Refer to Figure 1–14 a to d for the solutions. All the spreadsheets contain the same information,<br />

but some cell values are altered as required by the question. (Actually, all the questions can be<br />

answered on one spreadsheet by changing the numbers. Separate spreadsheets are shown here<br />

for explanation purposes only.)<br />

The Excel functions are constructed with reference to the cells, not the values themselves,<br />

so that sensitivity analysis can be performed without function changes. This approach<br />

treats the value in a cell as a global variable for the spreadsheet. For example, the<br />

8% rate in cell B2 will be referenced in all functions as B2, not 8%. Thus, a change in the<br />

rate requires only one alteration in the cell B2 entry, not in every relation where 8% is used.<br />

See Appendix A for additional information about using cell referencing and building<br />

spreadsheet relations.<br />

(a) Figure 1–14 a shows the results, and Figure 1–14 b presents all spreadsheet relations for<br />

estimated interest and revenue (yearly in columns C and E, cumulative in columns D<br />

and F). As an illustration, for year 3 the interest I 3 and revenue plus interest R 3 are<br />

I 3 (cumulative revenue through year 2)(rate of return)<br />

$416,000(0.08)<br />

$33,280<br />

R 3 revenue in year 3 I 3<br />

$300,000 33,280<br />

$333,280<br />

The detailed relations shown in Figure 1–14 b calculate these values in cells C8 and E8.<br />

Cell C8 relation for I 3 : F7*B2<br />

Cell E8 relation for CF 3 : B8 C8<br />

The equivalent amount after 4 years is $1,109,022, which is comprised of $1,000,000 in<br />

total revenue and $109,022 in interest compounded at 8% per year. The shaded cells in<br />

Figure 1–14 a and b indicate that the sum of the annual values and the last entry in the cumulative<br />

columns must be equal.<br />

(b) To determine the effect of increasing estimated revenue for years 3 and 4 to $600,000,<br />

use the same spreadsheet and change the entries in cells B8 and B9 as shown in<br />

Figure 1–14 c . Total interest increases 22%, or $24,000, from $109,222 to $133,222.<br />

(c) Figure 1–14 d shows the effect of changing the original i value from 8% to an inflationadjusted<br />

rate of 3.85% in cell B2 on the first spreadsheet. [Remember to return to the<br />

$300,000 revenue estimates for years 3 and 4 after working part ( b ).] Inflation has now<br />

reduced total interest by 53% from $109,222 to $51,247, as shown in cell C10.


30 Chapter 1 Foundations of <strong>Engineering</strong> <strong>Economy</strong><br />

(a) Total interest and revenue for base case, year 4<br />

(b) Spreadsheet relations for base case<br />

Revenue changed<br />

(c) Totals with increased revenue in years 3 and 4<br />

Rate of<br />

return<br />

changed<br />

(d) Totals with inflation of 4% per year considered<br />

Figure 1–14<br />

Spreadsheet solutions with sensitivity analysis, Example 1.17 a to c .<br />

Comment<br />

Later we will learn how to utilize the NPV and FV Excel financial functions to obtain the same<br />

answers determined in Figure 1–14 , where we developed each basic relation.<br />

When you are working with an Excel spreadsheet, it is possible to display all of the entries<br />

and functions on the screen as shown in Figure 1–14 b by simultaneously touching the <br />

and < `> keys, which may be in the upper left of the keyboard on the key with .


Problems 31<br />

CHAPTER SUMMARY<br />

<strong>Engineering</strong> economy is the application of economic factors and criteria to evaluate alternatives,<br />

considering the time value of money. The engineering economy study involves computing a<br />

specific economic measure of worth for estimated cash flows over a specific period of time.<br />

The concept of equivalence helps in understanding how different sums of money at different<br />

times are equal in economic terms. The differences between simple interest (based on principal<br />

only) and compound interest (based on principal and interest upon interest) have been described<br />

in formulas, tables, and graphs. This power of compounding is very noticeable, especially over<br />

extended periods of time, and for larger sums of money.<br />

The MARR is a reasonable rate of return established as a hurdle rate to determine if an alternative<br />

is economically viable. The MARR is always higher than the return from a safe investment<br />

and the cost to acquire needed capital.<br />

Also, we learned a lot about cash flows:<br />

End-of-year convention for cash flow location<br />

Net cash flow computation<br />

Different perspectives in determining the cash flow sign<br />

Construction of a cash flow diagram<br />

Difficulties with estimating future cash flows accurately<br />

PROBLEMS<br />

Basic Concepts<br />

1.1 List the four essential elements involved in decision<br />

making in engineering economic analysis.<br />

1.2 What is meant by ( a ) limited capital funds and<br />

( b ) sensitivity analysis?<br />

1.3 List three measures of worth that are used in engineering<br />

economic analysis.<br />

1.4 Identify the following factors as either economic<br />

(tangible) or noneconomic (intangible): first cost,<br />

leadership, taxes, salvage value, morale, dependability,<br />

inflation, profit, acceptance, ethics, interest<br />

rate.<br />

Ethics<br />

1.5 Stefanie is a design engineer with an international<br />

railroad locomotive manufacturing company in<br />

Illinois. Management wants to return some of the<br />

engineering design work to the United States<br />

rather than export all of it to India, where the primary<br />

design work has been accomplished for the<br />

last decade. This transfer will employ more people<br />

locally and could improve the economic conditions<br />

for families in and around Illinois.<br />

Stefanie and her design team were selected as a<br />

test case to determine the quality and speed of the<br />

design work they could demonstrate on a more<br />

fuel-efficient diesel locomotive. Neither she nor<br />

any of her team members have done such a significant<br />

design job, because their jobs had previously<br />

entailed only the interface with the subcontracted<br />

engineers in India. One of her team members had a<br />

great design idea on a key element that will improve<br />

fuel efficiency by approximately 15%. She<br />

told Stefanie it came from one of the Indiangenerated<br />

documents, but that it would probably be<br />

okay for the team to use it and remain silent as to its<br />

origin, since it was quite clear the U.S. management<br />

was about to cancel the foreign contract.<br />

Although reluctant at first, Stefanie did go forward<br />

with a design that included the efficiency improvement,<br />

and no mention of the origin of the idea was<br />

made at the time of the oral presentation or documentation<br />

delivery. As a result, the Indian contract<br />

was canceled and full design responsibility was<br />

transferred to Stefanie’s group.<br />

Consult the NSPE Code of Ethics for Engineers<br />

(Appendix C) and identify sections that are points<br />

of concern about Stefanie’s decisions and actions.<br />

1.6 Consider the common moral precept that stealing<br />

is wrong. Hector is with a group of friends in a<br />

local supermarket. One of Hector’s buddies takes a<br />

high-energy drink from a six-pack on the shelf,<br />

opens it, drinks it, and returns the empty can to the<br />

package, with no intention of paying for it. He then<br />

invites the others to do the same, saying, “It’s only


32 Chapter 1 Foundations of <strong>Engineering</strong> <strong>Economy</strong><br />

one drink. Others do it all the time.” All the others,<br />

except Hector, have now consumed a drink of their<br />

choice. Personally, Hector believes this is a form<br />

of stealing. State three actions that Hector can<br />

take, and evaluate them from the personal moral<br />

perspective.<br />

1.7 While going to work this morning off site from his<br />

office, an engineer accidently ran a stop sign and<br />

was in a car accident that resulted in the death of a<br />

5-year-old child. He has a strong belief in the universal<br />

moral that it is wrong to do serious harm to<br />

another person. Explain the conflict that can arise<br />

for him between the universal moral and his personal<br />

moral about doing serious harm, given the<br />

accident was deemed his fault.<br />

1.8 Claude is a fourth-year engineering university student<br />

who has just been informed by his instructor<br />

that he made a very low grade on his Spanish language<br />

final test for the year. Although he had a<br />

passing score prior to the final, his final grade was<br />

so low that he has now flunked the entire year and<br />

will likely have to extend his graduation another<br />

semester or two.<br />

Throughout the year, Claude, who hated the<br />

course and his instructor, has copied homework,<br />

cheated on tests, and never seriously studied for<br />

anything in the course. He did realize during the<br />

semester that he was doing something that even<br />

he consi dered wrong morally and ethically. He<br />

knew he had done badly on the final. The classroom<br />

was reconfigured for the final exam in a way<br />

that he could not get any answers from classmates,<br />

and cell phones were collected prior to the exam,<br />

thus removing texting possibilities to friends outside<br />

the classroom who might help him on the<br />

final exam. Claude is now face to face with the<br />

instructor in her office. The question to Claude is,<br />

“What have you been doing throughout this year<br />

to make passing scores repeatedly, but demonstrate<br />

such a poor command of Spanish on the<br />

final exam?”<br />

From an ethical viewpoint, what options does<br />

Claude have in his answer to this question? Also,<br />

discuss some of the possible effects that this experience<br />

may have upon Claude’s future actions and<br />

moral dilemmas.<br />

Interest Rate and Rate of Return<br />

1.9 RKI Instruments borrowed $3,500,000 from a private<br />

equity firm for expansion of its manufacturing<br />

facility for making carbon monoxide monitors/<br />

controllers. The company repaid the loan after<br />

1 year with a single payment of $3,885,000. What<br />

was the interest rate on the loan?<br />

1.10 Emerson Processing borrowed $900,000 for installing<br />

energy-efficient lighting and safety equipment<br />

in its La Grange manufacturing facility. The<br />

terms of the loan were such that the company could<br />

pay interest only at the end of each year for up to 5<br />

years, after which the company would have to pay<br />

the entire amount due. If the interest rate on the<br />

loan was 12% per year and the company paid only<br />

the interest for 4 years, determine the following:<br />

(a) The amount of each of the four interest<br />

payments<br />

(b) The amount of the final payment at the end of<br />

year 5<br />

1.11 Which of the following 1-year investments has the<br />

highest rate of return?<br />

(a) $12,500 that yields $1125 in interest,<br />

(b) $56,000 that yields $6160 in interest, or<br />

(c) $95,000 that yields $7600 in interest .<br />

1.12 A new engineering graduate who started a consulting<br />

business borrowed money for 1 year to furnish<br />

the office. The amount of the loan was $23,800,<br />

and it had an interest rate of 10% per year. However,<br />

because the new graduate had not built up a<br />

credit history, the bank made him buy loan-default<br />

insurance that cost 5% of the loan amount. In addition,<br />

the bank charged a loan setup fee of $300.<br />

What was the effective interest rate the engineer<br />

paid for the loan?<br />

1.13 When the inflation rate is expected to be 8% per<br />

year, what is the market interest rate likely to be?<br />

Terms and Symbols<br />

1.14 The symbol P represents an amount of money at a<br />

time designated as present. The following symbols<br />

also represent a present amount of money and require<br />

similar calculations. Explain what each symbol<br />

stands for: PW, PV, NPV, DCF, and CC .<br />

1.15 Identify the four engineering economy symbols<br />

and their values from the following problem statement.<br />

Use a question mark with the symbol whose<br />

value is to be determined.<br />

Thompson Mechanical Products is planning to<br />

set aside $150,000 now for possibly replacing its<br />

large synchronous refiner motors whenever it becomes<br />

necessary. If the replacement is not needed<br />

for 7 years, how much will the company have in its<br />

investment set-aside account, provided it achieves<br />

a rate of return of 11% per year?<br />

1.16 Identify the four engineering economy symbols and<br />

their values from the following problem statement.


Problems 33<br />

Use a question mark with the symbol whose value is<br />

to be determined.<br />

Atlas Long-Haul Transportation is considering<br />

installing Valutemp temperature loggers in all<br />

of its refrigerated trucks for monitoring temperatures<br />

during transit. If the systems will reduce<br />

insurance claims by $100,000 two years from<br />

now, how much should the company be willing to<br />

spend now, if it uses an interest rate of 12% per<br />

year?<br />

1.17 Identify the four engineering economy symbols<br />

and their values from the following problem statement.<br />

Use a question mark with the symbol whose<br />

value is to be determined.<br />

A green algae, Chlamydomonas reinhardtii,<br />

can produce hydrogen when temporarily deprived<br />

of sulfur for up to 2 days at a time. A small<br />

company needs to purchase equipment costing<br />

$3.4 million to commercialize the process. If the<br />

company wants to earn a rate of return of 10% per<br />

year and recover its investment in 8 years, what<br />

must be the net value of the hydrogen produced<br />

each year?<br />

1.18 Identify the four engineering economy symbols<br />

and their values from the following problem statement.<br />

Use a question mark with the symbol whose<br />

value is to be determined.<br />

Vision Technologies, Inc., is a small company<br />

that uses ultra-wideband technology to develop<br />

devices that can detect objects (including people)<br />

inside of buildings, behind walls, or below ground.<br />

The company expects to spend $100,000 per year<br />

for labor and $125,000 per year for supplies before<br />

a product can be marketed. At an interest rate of<br />

15% per year, what is the total equivalent future<br />

amount of the company’s expenses at the end of<br />

3 years?<br />

Cash Flows<br />

1.19 What is meant by end-of-period convention?<br />

1.20 Identify the following as cash inflows or outflows<br />

to commercial air carriers: fuel cost, pension plan<br />

contributions, fares, maintenance, freight revenue,<br />

cargo revenue, extra-bag charges, water and<br />

sodas, advertising, landing fees, seat preference<br />

fees.<br />

1.21 Many credit unions use semiannual interest periods<br />

to pay interest on customer savings accounts. For a<br />

credit union that uses June 30 and December 31 as<br />

its semiannual interest periods, determine the endof-period<br />

amounts that will be recorded for the deposits<br />

shown in the table.<br />

Month Deposit, $<br />

Jan 50<br />

Feb 70<br />

Mar —<br />

Apr 120<br />

May 20<br />

June —<br />

July 150<br />

Aug 90<br />

Sept —<br />

Oct —<br />

Nov 40<br />

Dec 110<br />

1.22 For a company that uses a year as its interest period,<br />

determine the net cash fl ow that will be recorded<br />

at the end of the year from the cash flows<br />

shown.<br />

Month<br />

Receipts,<br />

$1000<br />

Disbursements,<br />

$1000<br />

Jan 500 300<br />

Feb 800 500<br />

Mar 200 400<br />

Apr 120 400<br />

May 600 500<br />

June 900 600<br />

July 800 300<br />

Aug 700 300<br />

Sept 900 500<br />

Oct 500 400<br />

Nov 400 400<br />

Dec 1800 700<br />

1.23 Construct a cash flow diagram for the following<br />

cash flows: $25,000 outflow at time 0, $9000 per<br />

year inflow in years 1 through 5 at an interest rate<br />

of 10% per year, and an unknown future amount in<br />

year 5.<br />

1.24 Construct a cash flow diagram to find the present<br />

worth in year 0 at an interest rate of 15% per year<br />

for the following situation.<br />

Year Cash Flow, $<br />

0 19,000<br />

1–4 8,100<br />

1.25 Construct a cash flow diagram that represents the<br />

amount of money that will be accumulated in<br />

15 years from an investment of $40,000 now at an<br />

interest rate of 8% per year.<br />

Equivalence<br />

1.26 At an interest rate of 15% per year, an investment<br />

of $100,000 one year ago is equivalent to how<br />

much now?


34 Chapter 1 Foundations of <strong>Engineering</strong> <strong>Economy</strong><br />

1.27 During a recession, the price of goods and services<br />

goes down because of low demand. A company<br />

that makes Ethernet adapters is planning to expand<br />

its production facility at a cost of $1,000,000 one<br />

year from now. However, a contractor who needs<br />

work has offered to do the job for $790,000 if the<br />

company will do the expansion now instead of<br />

1 year from now. If the interest rate is 15% per year,<br />

how much of a discount is the company getting?<br />

1.28 As a principal in the consulting firm where you<br />

have worked for 20 years, you have accumulated<br />

5000 shares of company stock. One year ago, each<br />

share of stock was worth $40. The company has<br />

offered to buy back your shares for $225,000. At<br />

what interest rate would the firm’s offer be equivalent<br />

to the worth of the stock last year?<br />

1.29 A design/build engineering company that usually<br />

gives year-end bonuses in the amount of $8000 to<br />

each of its engineers is having cash flow problems.<br />

The company said that although it could not<br />

give bonuses this year, it would give each engineer<br />

two bonuses next year, the regular one of<br />

$8000 plus an amount equivalent to the $8000<br />

that each engineer should have gotten this year.<br />

If the interest rate is 8% per year, what will be the<br />

total amount of bonus money the engineers should<br />

get next year?<br />

1.30 University tuition and fees can be paid by using<br />

one of two plans.<br />

Early-bird: Pay total amount due 1 year in<br />

advance and get a 10% discount.<br />

On-time: Pay total amount due when classes start.<br />

The cost of tuition and fees is $10,000 per year.<br />

(a) How much is paid in the early-bird plan?<br />

(b) What is the equivalent amount of the savings<br />

compared to the on-time payment at the time<br />

that the on-time payment is made?<br />

Simple and Compound Interest<br />

1.31 If a company sets aside $1,000,000 now into a<br />

contingency fund, how much will the company<br />

have in 2 years, if it does not use any of the money<br />

and the account grows at a rate of 10% per year?<br />

1.32 Iselt Welding has extra funds to invest for future<br />

capital expansion. If the selected investment pays<br />

simple interest, what interest rate would be<br />

required for the amount to grow from $60,000 to<br />

$90,000 in 5 years?<br />

1.33 To finance a new product line, a company that<br />

makes high-temperature ball bearings borrowed<br />

$1.8 million at 10% per year interest. If the company<br />

repaid the loan in a lump sum amount after<br />

2 years, what was ( a ) the amount of the payment<br />

and ( b ) the amount of interest?<br />

1.34 Because market interest rates were near all-time<br />

lows at 4% per year, a hand tool company decided<br />

to call (i.e., pay off ) the high-interest bonds that it<br />

issued 3 years ago. If the interest rate on the bonds<br />

was 9% per year, how much does the company<br />

have to pay the bond holders? The face value<br />

(principal) of the bonds is $6,000,000.<br />

1.35 A solid waste disposal company borrowed money<br />

at 10% per year interest to purchase new haulers<br />

and other equipment needed at the companyowned<br />

landfill site. If the company got the loan<br />

2 years ago and paid it off with a single payment of<br />

$4,600,000, what was the principal amount P of<br />

the loan?<br />

1.36 If interest is compounded at 20% per year, how<br />

long will it take for $50,000 to accumulate to<br />

$86,400?<br />

1.37 To make CDs look more attractive than they really<br />

are, some banks advertise that their rates are higher<br />

than their competitors’ rates; however, the fine<br />

print says that the rate is a simple interest rate. If a<br />

person deposits $10,000 at 10% per year simple<br />

interest, what compound interest rate would yield<br />

the same amount of money in 3 years?<br />

MARR and Opportunity Cost<br />

1.38 Give three other names for minimum attractive<br />

rate of return.<br />

1.39 Identify the following as either equity or debt financing:<br />

bonds, stock sales, retained earnings,<br />

venture capital, short-term loan, capital advance<br />

from friend, cash on hand, credit card, home equity<br />

loan.<br />

1.40 What is the weighted average cost of capital for a<br />

corporation that finances an expansion project<br />

using 30% retained earnings and 70% venture capital?<br />

Assume the interest rates are 8% for the equity<br />

financing and 13% for the debt financing.<br />

1.41 Managers from different departments in Zenith<br />

Trading, a large multinational corporation, have offered<br />

six projects for consideration by the corporate<br />

office. A staff member for the chief financial officer<br />

used key words to identify the projects and then<br />

listed them in order of projected rate of return as<br />

shown below. If the company wants to grow rapidly<br />

through high leverage and uses only 10%<br />

equity financing that has a cost of equity capital of<br />

9% and 90% debt financing with a cost of debt


Additional Problems and FE Exam Review Questions 35<br />

capital of 16%, which projects should the company<br />

undertake?<br />

Project ID<br />

Projected ROR,<br />

% per year<br />

Inventory 30<br />

Technology 28.4<br />

Warehouse 19<br />

Products 13.1<br />

Energy 9.6<br />

Shipping 8.2<br />

Spreadsheet Functions<br />

1.42 State the purpose for each of the following built-in<br />

spreadsheet functions.<br />

( a) PV( i% , n , A , F )<br />

( b) FV( i %, n , A , P )<br />

( c) RATE( n , A , P , F )<br />

( d) IRR(first_cell:last_cell)<br />

( e) PMT( i %, n , P , F )<br />

( f ) NPER( i %, A , P , F )<br />

1.43 What are the values of the engineering economy<br />

symbols P , F , A , i , and n in the following functions?<br />

Use a question mark for the symbol that is to be<br />

determined.<br />

(a) NPER(8%,1500,8000,2000)<br />

( b ) FV(7%,102000,9000)<br />

(c) RATE(10,1000,12000,2000)<br />

(d ) PMT(11%,20,,14000)<br />

(e) PV(8%,15,1000,800)<br />

1.44 Write the engineering economy symbol that corresponds<br />

to each of the following spreadsheet<br />

functions.<br />

( a ) PMT ( b ) FV ( c ) NPER ( d ) PV ( e ) IRR<br />

1.45 In a built-in spreadsheet function, if a certain parameter<br />

is not present, ( a ) under what circumstances<br />

can it be left blank and ( b ) when must a<br />

comma be entered in its place?<br />

1.46 Sheryl and Marcelly both invest $1000 at 10% per<br />

year for 4 years. Sheryl receives simple interest and<br />

Marcelly gets compound interest. Use a spreadsheet<br />

and cell reference formats to develop relations that<br />

show a total of $64 more interest for Marcelly at the<br />

end of the 4 years. Assume no withdrawals or further<br />

deposits are made during the 4 years.<br />

ADDITIONAL PROBLEMS AND FE EXAM REVIEW QUESTIONS<br />

1.47 The concept that different sums of money at different<br />

points in time can be said to be equal to each<br />

other is known as:<br />

( a) Evaluation criterion<br />

( b) Equivalence<br />

( c) Cash flow<br />

( d ) Intangible factors<br />

1.48 The evaluation criterion that is usually used in an<br />

economic analysis is:<br />

( a) Time to completion<br />

( b) Technical feasibility<br />

( c) Sustainability<br />

( d ) Financial units (dollars or other currency)<br />

1.49 All of the following are examples of cash outflows,<br />

except :<br />

( a) Asset salvage value<br />

( b) Income taxes<br />

( c) Operating cost of asset<br />

( d ) First cost of asset<br />

1.50 In most engineering economy studies, the best alternative<br />

is the one that:<br />

( a) Will last the longest time<br />

( b) Is most politically correct<br />

( c) Is easiest to implement<br />

( d ) Has the lowest cost<br />

1.51 The following annual maintenance and operation<br />

(M&O) costs for a piece of equipment were collected<br />

over a 5-year period: $12,300, $8900,<br />

$9200, $11,000, and $12,100. The average is<br />

$10,700. In conducting a sensitivity analysis, the<br />

most reasonable range of costs to use (i.e., percent<br />

from the average) is:<br />

( a ) 5% ( b ) 11% ( c ) 17% ( d ) 25%<br />

1.52 At an interest rate of 10% per year, the equivalent<br />

amount of $10,000 one year ago is closest to:<br />

( a ) $8264 ( b ) $9091 ( c ) $11,000 ( d ) $12,000<br />

1.53 Assume that you and your best friend each have<br />

$1000 to invest. You invest your money in a fund<br />

that pays 10% per year compound interest. Your<br />

friend invests her money at a bank that pays 10%<br />

per year simple interest. At the end of 1 year, the<br />

difference in the total amount for each of you is:<br />

( a) You have $10 more than she does<br />

( b) You have $100 more than she does<br />

( c) You both have the same amount of money<br />

( d ) She has $10 more than you do<br />

1.54 The time it would take for a given sum of money to<br />

double at 4% per year simple interest is closest to:<br />

( a ) 30 years ( b ) 25 years<br />

( c ) 20 years ( d ) 10 years


36 Chapter 1 Foundations of <strong>Engineering</strong> <strong>Economy</strong><br />

1.55 All of the following are examples of equity financing,<br />

except :<br />

(a) Mortgage<br />

(b) Money from savings<br />

(c) Cash on hand<br />

(d ) Retained earnings<br />

1.56 To finance a new project costing $30 million, a<br />

company borrowed $21 million at 16% per year<br />

interest and used retained earnings valued at 12%<br />

per year for the remainder of the investment. The<br />

company’s weighted average cost of capital for the<br />

project was closest to:<br />

( a ) 12.5% ( b ) 13.6% ( c ) 14.8% ( d ) 15.6%<br />

CASE STUDY<br />

RENEWABLE ENERGY SOURCES FOR ELECTRICITY GENERATION<br />

Background<br />

Pedernales Electric Cooperative (PEC) is the largest<br />

member-owned electric co-op in the United States with over<br />

232,000 meters in 12 Central Texas counties. PEC has a capacity<br />

of approximately 1300 MW (megawatts) of power, of<br />

which 277 MW, or about 21%, is from renewable sources.<br />

The latest addition is 60 MW of power from a wind farm in<br />

south Texas close to the city of Corpus Christi. A constant<br />

question is how much of PEC’s generation capacity should be<br />

from renewable sources, especially given the environmental<br />

issues with coal-generated electricity and the rising costs of<br />

hydrocarbon fuels.<br />

Wind and nuclear sources are the current consideration for<br />

the PEC leadership as Texas is increasing its generation by<br />

nuclear power and the state is the national leader in wind<br />

farm–produced electricity.<br />

Consider yourself a member of the board of directors of<br />

PEC. You are an engineer who has been newly elected by the<br />

PEC membership to serve a 3-year term as a director-at-large.<br />

As such, you do not represent a specific district within the<br />

entire service area; all other directors do represent a specific<br />

district. You have many questions about the operations of<br />

PEC, plus you are interested in the economic and societal<br />

benefits of pursuing more renewable source generation<br />

capacity.<br />

Information<br />

Here are some data that you have obtained. The information<br />

is sketchy, as this point, and the numbers are very approximate.<br />

Electricity generation cost estimates are national<br />

in scope, not PEC-specific, and are provided in cents per<br />

kilowatt-hour (¢/kWh).<br />

Generation Cost, ¢/kWh<br />

Fuel Source Likely Range Reasonable Average<br />

Coal 4 to 9 7.4<br />

Natural gas 4 to 10.5 8.6<br />

Wind 4.8 to 9.1 8.2<br />

Solar 4.5 to 15.5 8.8<br />

National average cost of electricity to residential customers:<br />

11¢/kWh<br />

PEC average cost to residential customers: 10.27 ¢/kWh<br />

(from primary sources) and 10.92 ¢/kWh (renewable sources)<br />

Expected life of a generation facility: 20 to 40 years (it is<br />

likely closer to 20 than 40)<br />

Time to construct a facility: 2 to 5 years<br />

Capital cost to build a generation facility: $900 to $1500<br />

per kW<br />

You have also learned that the PEC staff uses the wellrecognized<br />

levelized energy cost (LEC) method to determine<br />

the price of electricity that must be charged to customers to<br />

break even. The formula takes into account the capital cost of<br />

the generation facilities, the cost of capital of borrowed<br />

money, annual maintenance and operation (M&O) costs, and<br />

the expected life of the facility. The LEC formula, expressed<br />

in dollars per kWh for ( t 1, 2, ... , n ), is<br />

tn<br />

P t A t C<br />

—————— t<br />

t1 (1 i)<br />

LEC <br />

t<br />

———————<br />

tn<br />

E<br />

——— t<br />

t1 (1 i) t<br />

where P t capital investments made in year t<br />

A t annual maintenance and operating (M&O) costs<br />

for year t<br />

C t fuel costs for year t<br />

E t amount of electricity generated in year t<br />

n expected life of facility<br />

i discount rate (cost of capital)<br />

Case Study Exercises<br />

1. If you wanted to know more about the new arrangement<br />

with the wind farm in south Texas for the additional<br />

60 MW per year, what types of questions would<br />

you ask of a staff member in your first meeting with<br />

him or her?<br />

2. Much of the current generation capacity of PEC facilities<br />

utilizes coal and natural gas as the primary fuel source.<br />

What about the ethical aspects of the government’s allowance<br />

for these plants to continue polluting the atmosphere<br />

with the emissions that may cause health problems for<br />

citizens and further the effects of global warming? What<br />

types of regulations, if any, should be developed for PEC<br />

(and other generators) to follow in the future?


Case Study 37<br />

3. You developed an interest in the LEC relation and<br />

the publicized cost of electricity of 10.27¢/kWh for<br />

this year. You wonder if the addition of 60 MW of<br />

wind-sourced electricity will make any difference in<br />

the LEC value for this next year. You did learn the<br />

following:<br />

This is year t 11 for LEC computation purposes<br />

n 25 years<br />

i 5% per year<br />

E 11 5.052 billion kWh<br />

LEC last year was 10.22 ¢/kWh (last year’s breakeven<br />

cost to customers)<br />

From these sketchy data, can you determine the value of unknowns<br />

in the LEC relation for this year? Is it possible to<br />

determine if the wind farm addition of 60 MW makes any<br />

difference in the electricity rate charged to customers? If not,<br />

what additional information is necessary to determine the<br />

LEC with the wind source included?<br />

CASE STUDY<br />

REFRIGERATOR SHELLS<br />

Background<br />

Large refrigerator manufacturers such as Whirlpool, General<br />

Electric, Frigidaire, and others may subcontract the molding of<br />

their plastic liners and door panels. One prime national subcontractor<br />

is Innovations Plastics. Because of improvements in mechanical<br />

properties, the molded plastic can sustain increased vertical<br />

and horizontal loading, thus significantly reducing the need<br />

for attached metal anchors for some shelving. However, improved<br />

molding equipment is needed to enter this market now.<br />

The company president wants a recommendation on whether<br />

Innovations should offer the new technology to the major manufacturers<br />

and an estimate of the necessary capital investment<br />

to enter this market.<br />

You work as an engineer for Innovations. At this stage,<br />

you are not expected to perform a complete engineering economic<br />

analysis, for not enough information is available. You<br />

are asked to formulate reasonable alternatives, determine<br />

what data and estimates are needed for each one, and ascertain<br />

what criteria (economic and noneconomic) should be<br />

utilized to make the final decision.<br />

Information<br />

Some information useful at this time is as follows:<br />

• The technology and equipment are expected to last about<br />

10 years before new methods are developed.<br />

• Inflation and income taxes will not be considered in the<br />

analysis.<br />

• The expected returns on capital investment used for the<br />

last three new technology projects were compound rates of<br />

15%, 5%, and 18%. The 5% rate was the criterion for<br />

enhancing an employee-safety system on an existing<br />

chemical-mixing process.<br />

• Equity capital financing beyond $5 million is not possible.<br />

The amount of debt financing and its cost are unknown.<br />

• Annual operating costs have been averaging 8% of first<br />

cost for major equipment.<br />

• Increased annual training costs and salary requirements<br />

for handling the new plastics and operating new equipment<br />

can range from $800,000 to $1.2 million.<br />

There are two manufacturers working on the new- generation<br />

equipment. You label these options as alternatives A and B.<br />

Case Study Exercises<br />

1. Use the first four steps of the decision-making process<br />

to generally describe the alternatives and identify what<br />

economic-related estimates you will need to complete<br />

an engineering economy analysis for the president.<br />

2. Identify any noneconomic factors and criteria to be considered<br />

in making the alternative selection.<br />

3. During your inquiries about alternative B from its manufacturer,<br />

you learn that this company has already produced<br />

a prototype molding machine and has sold it to a company<br />

in Germany for $3 million (U.S. dollars). Upon inquiry,<br />

you further discover that the German company already<br />

has unused capacity on the equipment for manufacturing<br />

plastic shells. The company is willing to sell time on the<br />

equipment to Innovations immediately to produce its own<br />

shells for U.S. delivery. This could allow immediate market<br />

entry into the United States. Consider this as alternative<br />

C, and develop the estimates necessary to evaluate C<br />

at the same time as alternatives A and B.


CHAPTER 2<br />

Factors: How<br />

Time and<br />

Interest Affect<br />

Money<br />

LEARNING OUTCOMES<br />

Purpose: Derive and use the engineering economy factors to account for the time value of money.<br />

SECTION TOPIC LEARNING OUTCOME<br />

2.1 FP and PF factors • Derive and use factors for single amounts—<br />

compound amount (FP) and present worth (PF)<br />

factor.<br />

2.2 PA and AP factors • Derive and use factors for uniform series—present<br />

worth (PA) and capital recovery (AP) factors.<br />

2.3 FA and AF factors • Derive and use factors for uniform series—<br />

compound amount (FA) and sinking fund (AF)<br />

factors.<br />

2.4 Factor values • Use linear interpolation in factor tables or<br />

spreadsheet functions to determine factor values.<br />

2.5 Arithmetic gradient • Use the present worth (PG) and uniform annual<br />

series (AG) factors for arithmetic gradients.<br />

2.6 Geometric gradient • Use the geometric gradient series factor (PA,g)<br />

to find present worth.<br />

2.7 Find i or n • Use equivalence relations to determine i (interest<br />

rate or rate of return) or n for a cash flow series.


T<br />

he cash flow is fundamental to every economic study. Cash flows occur in many<br />

configurations and amounts—isolated single values, series that are uniform, and<br />

series that increase or decrease by constant amounts or constant percentages.<br />

This chapter develops derivations for all the commonly used engineering economy factors<br />

that take the time value of money into account.<br />

The application of factors is illustrated using their mathematical forms and a standard notation<br />

format. Spreadsheet functions are used in order to rapidly work with cash flow series<br />

and to perform sensitivity analysis.<br />

If the derivation and use of factors are not covered in the course, alternate ways to perform<br />

time value of money calculations are summarized in Appendix D.<br />

The Cement Factory Case: Votorantim<br />

Cimentos North America, Inc., is a subsidiary<br />

of a Brazil-based company that<br />

recently announced plans to develop a<br />

new cement factory in Houston County<br />

in the state of Georgia. The plant will<br />

be called Houston American Cement, or<br />

HAC. The location is ideal for cement<br />

making because of the large deposit of<br />

limestone in the area.<br />

The plant investment, expected<br />

to amount to $200 million, has been<br />

planned for <strong>2012</strong>; however, it is currently<br />

delayed due to the economic downturn<br />

in construction. When the plant is completed<br />

and operating at full capacity,<br />

PE<br />

based upon the projected needs and<br />

cost per metric ton, it is possible that<br />

the plant could generate as much as<br />

$50,000,000 annually in revenue. All<br />

analysis will use a planning horizon of<br />

5 years commencing when the plant<br />

begins operation.<br />

This case is used in the following<br />

topics (and sections) of this chapter:<br />

Single-amount factors (2.1)<br />

Uniform series factors (2.2 and 2.3)<br />

Arithmetic gradient factors (2.5)<br />

Geometric gradient factors (2.6)<br />

Determining unknown n values (2.7)<br />

2.1 Single-Amount Factors ( F P and P F )<br />

The most fundamental factor in engineering economy is the one that determines the amount<br />

of money F accumulated after n years (or periods) from a single present worth P, with interest<br />

compounded one time per year (or period). Recall that compound interest refers to interest paid<br />

on top of interest. Therefore, if an amount P is invested at time t 0, the amount F 1 accumulated<br />

1 year hence at an interest rate of i percent per year will be<br />

F 1 P Pi<br />

P (1 i )<br />

where the interest rate is expressed in decimal form. At the end of the second year, the amount<br />

accumulated F 2 is the amount after year 1 plus the interest from the end of year 1 to the end of<br />

year 2 on the entire F 1 .<br />

The amount F 2 can be expressed as<br />

F 2 F 1 F 1 i<br />

P (1 i ) P (1 i ) i [2.1]<br />

F 2 P (1 i i i 2 )<br />

P (1 2 i i 2 )<br />

P (1 i )<br />

2<br />

Similarly, the amount of money accumulated at the end of year 3, using Equation [2.1], will be<br />

F 3 F 2 F 2 i


40 Chapter 2 Factors: How Time and Interest Affect Money<br />

F =?<br />

F = given<br />

i = given<br />

i = given<br />

0 1 2 n – 2 n – 1 n<br />

0 1 2 n – 2 n – 1 n<br />

P = given<br />

(a)<br />

P =?<br />

Figure 2–1<br />

Cash flow diagrams for single-payment factors: (a) find F, given P, and (b) find P, given F.<br />

(b)<br />

Substituting P (1 i ) 2 for F 2 and simplifying, we get<br />

F 3 P (1 i ) 3<br />

From the preceding values, it is evident by mathematical induction that the formula can be generalized<br />

for n years. To find F , given P ,<br />

F P(1 i) n [2.2]<br />

The factor (1 i ) n is called the single-payment compound amount factor (SPCAF), but it is usually<br />

referred to as the F P factor. This is the conversion factor that, when multiplied by P , yields<br />

the future amount F of an initial amount P after n years at interest rate i . The cash flow diagram<br />

is seen in Figure 2–1 a .<br />

Reverse the situation to determine the P value for a stated amount F that occurs n periods<br />

in the future. Simply solve Equation [2.2] for P .<br />

P F [<br />

1 ————<br />

(1 i) n ]<br />

F(1 i) n [2.3]<br />

The expression (1 i ) n is known as the single-payment present worth factor (SPPWF), or the<br />

P F factor. This expression determines the present worth P of a given future amount F after n<br />

years at interest rate i . The cash flow diagram is shown in Figure 2–1 b .<br />

Note that the two factors derived here are for single payments; that is, they are used to find the<br />

present or future amount when only one payment or receipt is involved.<br />

A standard notation has been adopted for all factors. The notation includes two cash flow symbols,<br />

the interest rate, and the number of periods. It is always in the general form ( X Y , i , n ). The<br />

letter X represents what is sought, while the letter Y represents what is given. For example, F P<br />

means fi nd F when given P. The i is the interest rate in percent, and n represents the number of<br />

periods involved.<br />

Using this notation, ( F P ,6%,20) represents the factor that is used to calculate the future<br />

amount F accumulated in 20 periods if the interest rate is 6% per period. The P is given. The<br />

standard notation, simpler to use than formulas and factor names, will be used hereafter.<br />

Table 2–1 summarizes the standard notation and equations for the F P and P F factors. This<br />

information is also included inside the front cover.<br />

TABLE 2–1<br />

FP and PF Factors: Notation and Equations<br />

Factor Standard Notation Equation Excel<br />

Notation Name Find/Given Equation with Factor Formula Function<br />

(FP,i,n)<br />

(PF,i,n)<br />

Single-payment<br />

compound amount<br />

Single-payment<br />

present worth<br />

FP F P(FP,i,n) F P(1 i) n FV(i%,n,,P)<br />

PF P F(PF,i,n) P F (1 i) n PV(i%,n,,F)


2.1 Single-Amount Factors (FP and PF ) 41<br />

To simplify routine engineering economy calculations, tables of factor values have been prepared<br />

for interest rates from 0.25% to 50% and time periods from 1 to large n values, depending<br />

on the i value. These tables, found at the rear of the book, have a colored edge for easy identification.<br />

They are arranged with factors across the top and the number of periods n down the left side.<br />

The word discrete in the title of each table emphasizes that these tables utilize the end-of-period<br />

convention and that interest is compounded once each interest period. For a given factor, interest<br />

rate, and time, the correct factor value is found at the intersection of the factor name and n . For<br />

example, the value of the factor ( P F ,5%,10) is found in the P F column of Table 10 at period 10<br />

as 0.6139. This value is determined by using Equation [2.3].<br />

(PF,5%,10) ———— 1<br />

(1 i) n<br />

———— 1<br />

(1.05) 10<br />

<br />

1 ———<br />

1.6289 0.6139<br />

For spreadsheets, a future value F is calculated by the FV function using the format<br />

FV(i%,n,,P) [2.4]<br />

A present amount P is determined using the PV function with the format<br />

PV(i%,n,,F) [2.5]<br />

These functions are included in Table 2–1. Refer to Appendix A or Excel online help for more<br />

information on the use of FV and PV functions.<br />

EXAMPLE 2.1<br />

Sandy, a manufacturing engineer, just received a year-end bonus of $10,000 that will be invested<br />

immediately. With the expectation of earning at the rate of 8% per year, Sandy hopes to take the<br />

entire amount out in exactly 20 years to pay for a family vacation when the oldest daughter is due<br />

to graduate from college. Find the amount of funds that will be available in 20 years by using<br />

(a) hand solution by applying the factor formula and tabulated value and (b) a spreadsheet function.<br />

Solution<br />

The cash flow diagram is the same as Figure 2–1a. The symbols and values are<br />

P $10,000 F ? i 8% per year n 20 years<br />

(a) Factor formula: Apply Equation [2.2] to find the future value F. Rounding to four decimals,<br />

we have<br />

F P(1 i) n 10,000(1.08) 20 10,000(4.6610)<br />

$46,610<br />

Standard notation and tabulated value: Notation for the FP factor is (FP,i%,n).<br />

F P(FP,8%,20) 10,000(4.6610)<br />

$46,610<br />

Table 13 provides the tabulated value. Round-off errors can cause a slight difference in<br />

the final answer between these two methods.<br />

(b) Spreadsheet: Use the FV function to find the amount 20 years in the future. The format is<br />

that shown in Equation [2.4]; the numerical entry is FV(8%,20,,10000). The spreadsheet<br />

will appear similar to that in the right side of Figure 1–13, with the answer<br />

($46,609.57) displayed. (You should try it on your own computer now.) The FV function<br />

has performed the computation in part (a) and displayed the result.<br />

The equivalency statement is: If Sandy invests $10,000 now and earns 8% per year every year<br />

for 20 years, $46,610 will be available for the family vacation.


42 Chapter 2 Factors: How Time and Interest Affect Money<br />

EXAMPLE 2.2 The Cement Factory Case<br />

As discussed in the introduction to this chapter, the Houston American Cement factory will<br />

require an investment of $200 million to construct. Delays beyond the anticipated implementation<br />

year of <strong>2012</strong> will require additional money to construct the factory. Assuming that the cost<br />

of money is 10% per year, compound interest, use both tabulated factor values and spreadsheet<br />

functions to determine the following for the board of directors of the Brazilian company<br />

that plans to develop the plant.<br />

(a) The equivalent investment needed if the plant is built in 2015.<br />

(b) The equivalent investment needed had the plant been constructed in the year 2008.<br />

Solution<br />

Figure 2–2 is a cash flow diagram showing the expected investment of $200 million ($200 M)<br />

in <strong>2012</strong>, which we will identify as time t 0. The required investments 3 years in the future<br />

and 4 years in the past are indicated by F 3 ? and P 4 ?, respectively.<br />

P −4 =?<br />

F 3 =?<br />

Figure 2–2<br />

Cash flow diagram for<br />

Example 2.2a and b.<br />

PE<br />

−4 −3 −2 −1 0 1 2 3 t<br />

2008 2009 2010 2011 <strong>2012</strong> 2013 2014 2015 Year<br />

$200 M<br />

(a) To find the equivalent investment required in 3 years, apply the FP factor. Use $1 million<br />

units and the tabulated value for 10% interest (Table 15).<br />

F 3 P(FP,i,n) 200(FP,10%,3) 200(1.3310)<br />

$266.2 ($266,200,000)<br />

Now, use the FV function on a spreadsheet to find the same answer, F 3 $266.20 million.<br />

(Refer to Figure 2–3, left side.)<br />

FV(10%,3,,200)<br />

PV(10%,4,,200)<br />

Figure 2–3<br />

Spreadsheet functions for Example 2.2.<br />

(b) The year 2008 is 4 years prior to the planned construction date of <strong>2012</strong>. To determine the<br />

equivalent cost 4 years earlier, consider the $200 M in <strong>2012</strong> (t 0) as the future value F<br />

and apply the PF factor for n 4 to find P 4 . (Refer to Figure 2–2.) Table 15 supplies<br />

the tabulated value.<br />

P 4 F(PF,i,n) 200(PF,10%,4) 200(0.6830)<br />

$136.6 ($136,600,000)<br />

The PV function PV(10%,4,,200) will display the same amount as shown in Figure<br />

2–3, right side.<br />

This equivalence analysis indicates that at $136.6 M in 2008, the plant would have cost about<br />

68% as much as in <strong>2012</strong>, and that waiting until 2015 will cause the price tag to increase about<br />

33% to $266 M.


2.2 Uniform Series Present Worth Factor and Capital Recovery Factor (PA and AP) 43<br />

2.2 Uniform Series Present Worth Factor and<br />

Capital Recovery Factor ( P A and A P )<br />

The equivalent present worth P of a uniform series A of end-of-period cash flows (investments)<br />

is shown in Figure 2–4 a . An expression for the present worth can be determined by considering<br />

each A value as a future worth F , calculating its present worth with the P F factor, Equation [2.3],<br />

and summing the results.<br />

P A [<br />

1 ————<br />

(1 i) ] A 1<br />

[ 1 ————<br />

(1 i) ] A 1<br />

[ 2<br />

(1 i) ] A 1<br />

[ n1 ————<br />

(1 i) ] n<br />

A [<br />

1 ————<br />

————<br />

(1 i) ] . . .<br />

3<br />

The terms in brackets are the PF factors for years 1 through n , respectively. Factor out A .<br />

P A [<br />

———— 1<br />

(1 i) 1<br />

1 ————<br />

(1 i) 1<br />

2 ————<br />

(1 i) . . . <br />

1<br />

3 ————<br />

(1 i) 1<br />

n1 ————<br />

(1 i) ] n [2.6]<br />

To simplify Equation [2.6] and obtain the PA factor, multiply the n -term geometric progression<br />

in brackets by the ( P F , i %,1) factor, which is 1(1 i ). This results in Equation [2.7]. Now<br />

subtract the two equations, [2.6] from [2.7], and simplify to obtain the expression for P when<br />

i 0 (Equation [2.8]).<br />

——— P<br />

1 i A ———— 1<br />

[(1 i) 1<br />

2 ————<br />

(1 i) 1<br />

3 ————<br />

(1 i) . . . 1<br />

4 ————<br />

(1 i) n <br />

——— 1<br />

1 i P A ———— 1<br />

[(1 i) 1<br />

2 ————<br />

(1 i) . . . 1<br />

3 ————<br />

(1 i) n ———— 1<br />

(1 i) ] n1<br />

P A [<br />

———— 1<br />

(1 i) 1<br />

1 ————<br />

(1 i) . . . <br />

1<br />

2 ————<br />

(1 i) 1<br />

n1 ————<br />

(1 i) ]<br />

n<br />

——— i<br />

1 i P A ———— 1<br />

[(1 i) 1<br />

n1 ————<br />

(1 i) ] 1<br />

P ——<br />

i A ———— 1<br />

[(1 i) n 1 ]<br />

———— 1<br />

(1 i ) ] [2.7]<br />

n1<br />

P A [ (1 i)n 1<br />

——————<br />

i(1 i) n ] i 0 [2.8]<br />

The term in brackets in Equation [2.8] is the conversion factor referred to as the uniform series<br />

present worth factor (USPWF). It is the PA factor used to calculate the equivalent P value in<br />

year 0 for a uniform end-of-period series of A values beginning at the end of period 1 and extending<br />

for n periods. The cash flow diagram is Figure 2–4 a .<br />

P =?<br />

i = given<br />

P = given<br />

i = given<br />

0 1 2 n – 2 n – 1 n<br />

0<br />

1 2<br />

n – 2 n – 1 n<br />

A = given<br />

(a)<br />

Figure 2–4<br />

Cash flow diagrams used to determine (a) P, given a uniform series A, and (b) A, given a present worth P.<br />

A =?<br />

(b)


44 Chapter 2 Factors: How Time and Interest Affect Money<br />

TABLE 2–2<br />

PA and AP Factors: Notation and Equations<br />

Factor<br />

Notation Name Find/Given<br />

Factor<br />

Formula<br />

(PA,i,n) Uniform series PA (1 i) n 1<br />

—————<br />

present worth<br />

i(1 i) n<br />

(AP,i,n) Capital recovery AP i(1 i ) n<br />

—————<br />

(1 i ) n − 1<br />

Standard<br />

Notation Equation<br />

P A(PA,i,n)<br />

A P(AP,i,n)<br />

Excel<br />

Function<br />

PV(i%,n,A)<br />

PMT(i%,n,P)<br />

To reverse the situation, the present worth P is known and the equivalent uniform series<br />

amount A is sought (Figure 2–4 b ). The first A value occurs at the end of period 1, that is, one<br />

period after P occurs. Solve Equation [2.8] for A to obtain<br />

A P [<br />

i(1 i ) n<br />

——————<br />

(1 i ) n 1 ] [2.9]<br />

The term in brackets is called the capital recovery factor (CRF), or AP factor. It calculates the<br />

equivalent uniform annual worth A over n years for a given P in year 0, when the interest<br />

rate is i.<br />

Placement of P<br />

The PA and AP factors are derived with the present worth P and the first uniform annual<br />

amount A one year (period) apart. That is, the present worth P must always be located one<br />

period prior to the first A .<br />

The factors and their use to find P and A are summarized in Table 2–2 and inside the front cover.<br />

The standard notations for these two factors are ( P A , i %, n ) and ( A P , i %, n ). Tables at the end of<br />

the text include the factor values. As an example, if i 15% and n 25 years, the P A factor<br />

value from Table 19 is ( P A ,15%,25) 6.4641. This will find the equivalent present worth at<br />

15% per year for any amount A that occurs uniformly from years 1 through 25.<br />

Spreadsheet functions can determine both P and A values in lieu of applying the PA and A P<br />

factors. The PV function calculates the P value for a given A over n years and a separate F value<br />

in year n , if it is given. The format, is<br />

PV ( i %, n , A , F ) [2.10]<br />

Similarly, the A value is determined by using the PMT function for a given P value in year 0 and<br />

a separate F , if given. The format is<br />

Table 2–2 includes the PV and PMT functions.<br />

PMT ( i %,n , P , F ) [2.11]<br />

EXAMPLE 2.3<br />

How much money should you be willing to pay now for a guaranteed $600 per year for 9 years<br />

starting next year, at a rate of return of 16% per year?<br />

Solution<br />

The cash flows follow the pattern of Figure 2–4 a , with A $600, i 16%, and n 9. The<br />

present worth is<br />

P 600( P A ,16%,9) 600(4.6065) $2763.90<br />

The PV function PV(16%,9,600) entered into a single spreadsheet cell will display the<br />

answer P ($2763.93).


2.2 Uniform Series Present Worth Factor and Capital Recovery Factor (PA and AP) 45<br />

EXAMPLE 2.4 The Cement Factory Case<br />

As mentioned in the chapter introduction of this case, the Houston American Cement plant<br />

may generate a revenue base of $50 million per year. The president of the Brazilian parent<br />

company Votorantim Cimentos may have reason to be quite pleased with this projection for<br />

the simple reason that over the 5-year planning horizon, the expected revenue would total<br />

$250 million, which is $50 million more than the initial investment. With money worth<br />

10% per year, address the following question from the president: Will the initial investment<br />

be recovered over the 5-year horizon with the time value of money considered? If so, by how<br />

much extra in present worth funds? If not, what is the equivalent annual revenue base required<br />

for the recovery plus the 10% return on money? Use both tabulated factor values and spreadsheet<br />

functions.<br />

PE<br />

Solution<br />

Tabulated value: Use the P A factor to determine whether A $50 million per year for<br />

n 5 years starting 1 year after the plant’s completion ( t 0) at i 10% per year is equivalently<br />

less or greater than $200 M. The cash flow diagram is similar to Figure 2–4 a , where the<br />

first A value occurs 1 year after P . Using $1 million units and Table 15 values,<br />

P 50( P A ,10%,5) 50(3.7908)<br />

$189.54 ($189,540,000)<br />

The present worth value is less than the investment plus a 10% per year return, so the president<br />

should not be satisfied with the projected annual revenue.<br />

To determine the minimum required to realize a 10% per year return, use the A P factor.<br />

The cash flow diagram is the same as Figure 2–4 b , where A starts 1 year after P at t 0 and<br />

n 5.<br />

A 200( A P ,10%,5) 200(0.26380)<br />

$52.76 per year<br />

The plant needs to generate $52,760,000 per year to realize a 10% per year return over<br />

5 years.<br />

Spreadsheet: Apply the PV and PMT functions to answer the question. Figure 2–5 shows<br />

the use of PV( i %, n , A , F ) on the left side to find the present worth and the use of<br />

PMT( i %, n , P , F ) on the right side to determine the minimum A of $52,760,000 per year.<br />

Because there are no F values, it is omitted from the functions. The minus sign placed before<br />

each function name forces the answer to be positive, since these two functions always display<br />

the answer with the opposite sign entered on the estimated cash flows.<br />

PV(10%,5,50)<br />

PMT(10%,5,200)<br />

Figure 2–5<br />

Spreadsheet functions to find P and A for the cement factory case, Example 2.4.


46 Chapter 2 Factors: How Time and Interest Affect Money<br />

2.3 Sinking Fund Factor and Uniform Series Compound<br />

Amount Factor ( A F and F A )<br />

The simplest way to derive the A F factor is to substitute into factors already developed. If P<br />

from Equation [2.3] is substituted into Equation [2.9], the following formula results.<br />

A F [<br />

1 ————<br />

—————<br />

(1 i ) n − 1 ]<br />

(1 i )<br />

n ] [<br />

i (1 i ) n<br />

A F [<br />

————— i<br />

(1 i ) n 1 ] [2.12]<br />

The expression in brackets in Equation [2.12] is the A F or sinking fund factor. It determines<br />

the uniform annual series A that is equivalent to a given future amount F . This is shown graphically<br />

in Figure 2–6 a , where A is a uniform annual investment.<br />

Placement of F<br />

The uniform series A begins at the end of year (period) 1 and continues through the year of<br />

the given F. The last A value and F occur at the same time.<br />

Equation [2.12] can be rearranged to find F for a stated A series in periods 1 through n (Figure<br />

2–6 b ).<br />

F A [ (1 i ) n 1<br />

——————<br />

i ] [2.13]<br />

The term in brackets is called the uniform series compound amount factor (USCAF), or F A factor.<br />

When multiplied by the given uniform annual amount A , it yields the future worth of the uniform<br />

series. It is important to remember that the future amount F occurs in the same period as the last A .<br />

Standard notation follows the same form as that of other factors. They are ( F A , i , n ) and<br />

( A F , i , n ). Table 2–3 summarizes the notations and equations, as does the inside front cover.<br />

As a matter of interest, the uniform series factors can be symbolically determined by using an<br />

abbreviated factor form. For example, F A ( F P )( P A ), where cancellation of the P is correct.<br />

Using the factor formulas, we have<br />

( F A , i , n ) [(1 i ) n ] [ (1 i ) n 1<br />

——————<br />

i (1 i ) ] n (1 i ) n 1<br />

——————<br />

i<br />

For solution by spreadsheet, the FV function calculates F for a stated A series over n years.<br />

The format is<br />

FV ( i %, n , A , P ) [2.14]<br />

The P may be omitted when no separate present worth value is given. The PMT function determines<br />

the A value for n years, given F in year n and possibly a separate P value in year 0. The<br />

format is<br />

PMT ( i %, n , P , F ) [2.15]<br />

If P is omitted, the comma must be entered so the function knows the last entry is an F value.<br />

F = given<br />

i = given<br />

0 1 2 n – 2 n – 1 n<br />

i = given<br />

F =?<br />

0 1 2 n – 2 n – 1 n<br />

A =?<br />

(a)<br />

Figure 2–6<br />

Cash flow diagrams to ( a ) find A, given F , and ( b ) find F, given A .<br />

A = given<br />

(b)


2.3 Sinking Fund Factor and Uniform Series Compound Amount Factor (AF and FA) 47<br />

TABLE 2–3<br />

FA and AF Factors: Notation and Equations<br />

Factor<br />

Notation Name Find/Given<br />

Factor<br />

Formula<br />

(FA,i,n) Uniform series FA (1 i) n 1<br />

—————<br />

compound amount<br />

i<br />

(AF,i,n) Sinking fund AF ————— i<br />

(1 i ) n − 1<br />

Standard Notation<br />

Equation<br />

F A(FA,i,n)<br />

A F(AF,i,n)<br />

Excel<br />

Functions<br />

FV(i%,n,A)<br />

PMT(i%,n,F)<br />

EXAMPLE 2.5<br />

The president of Ford Motor Company wants to know the equivalent future worth of a $1 million<br />

capital investment each year for 8 years, starting 1 year from now. Ford capital earns at a<br />

rate of 14% per year.<br />

Solution<br />

The cash flow diagram (Figure 2–7) shows the annual investments starting at the end of year 1<br />

and ending in the year the future worth is desired. In $1000 units, the F value in year 8 is found<br />

by using the FA factor.<br />

F 1000( F A, 14%,8) 1000(13.2328) $13,232.80<br />

i = 14%<br />

F =?<br />

0 1 2 3 4 5 6 7 8<br />

A = $1000<br />

Figure 2–7<br />

Diagram to find F for a uniform series, Example 2.5.<br />

EXAMPLE 2.6 The Cement Factory Case<br />

Once again, consider the HAC case presented at the outset of this chapter, in which a projected<br />

$200 million investment can generate $50 million per year in revenue for 5 years starting<br />

1 year after start-up. A 10% per year time value of money has been used previously to determine<br />

P , F , and A values. Now the president would like the answers to a couple of new questions<br />

about the estimated annual revenues. Use tabulated values, factor formulas, or spreadsheet<br />

functions to provide the answers.<br />

( a ) What is the equivalent future worth of the estimated revenues after 5 years at 10% per year?<br />

( b ) Assume that, due to the economic downturn, the president predicts that the corporation<br />

will earn only 4.5% per year on its money, not the previously anticipated 10% per year.<br />

What is the required amount of the annual revenue series over the 5-year period to be economically<br />

equivalent to the amount calculated in ( a )?<br />

PE<br />

Solution<br />

( a ) Figure 2–6 b is the cash flow diagram with A $50 million. Note that the last A value and<br />

F ? both occur at the end of year n 5. We use tabulated values and the spreadsheet<br />

function to find F in year 5.<br />

Tabulated value: Use the FA factor and 10% interest factor table. In $1 million units, the<br />

future worth of the revenue series is<br />

F 50( FA ,10%,5) 50(6.1051)<br />

$305.255 ($305,255,000)


48 Chapter 2 Factors: How Time and Interest Affect Money<br />

FV(10%,5,50)<br />

PMT(4.5%,5,,B5)<br />

Figure 2–8<br />

Spreadsheet functions to find F and A at i 4.5% for the cement factory case,<br />

Example 2.6.<br />

If the rate of return on the annual revenues were 0%, the total amount after 5 years would<br />

be $250,000,000. The 10% per year return is projected to grow this value by 22%.<br />

Spreadsheet: Apply the FV factor in the format FV(10%,5,50) to determine F <br />

$305.255 million. Because there is no present amount in this computation, P is omitted<br />

from the factor. See Figure 2–8, left side. (As before, the minus sign forces the FV function<br />

to result in a positive value.)<br />

( b ) The president of the Brazilian company planning to develop the cement plant in Georgia<br />

is getting worried about the international economy. He wants the revenue stream to generate<br />

the equivalent that it would at a 10% per year return, that is, $305.255 million, but<br />

thinks that only a 4.5% per year return is achievable.<br />

Factor formula: The AF factor will determine the required A for 5 years. Since the factor<br />

tables do not include 4.5%, use the formula to answer the question. In $1 million units,<br />

A 305.255( AF ,4.5%,5) 305.255 [<br />

0.045 ——————<br />

(1.045) 5 1 ] 305.255(0.18279)<br />

$55.798<br />

The annual revenue requirement grows from $50 million to nearly $55,800,000. This is a<br />

significant increase of 11.6% each year.<br />

Spreadsheet: It is easy to answer this question by using the PMT( i %, n ,,F) function with<br />

i 4.5% and F $305.255 found in part ( a ). We can use the cell reference method<br />

(described in Appendix A) for the future amount F . Figure 2–8, right side, displays the required<br />

A of $55.798 per year (in $1 million units).<br />

2.4 Factor Values for Untabulated i or n Values<br />

Often it is necessary to know the correct numerical value of a factor with an i or n value that is<br />

not listed in the compound interest tables in the rear of the book. Given specific values of i and<br />

n , there are several ways to obtain any factor value.<br />

• Use the formula listed in this chapter or the front cover of the book,<br />

• Use an Excel function with the corresponding P , F , or A value set to 1.<br />

• Use linear interpolation in the interest tables.<br />

When the formula is applied, the factor value is accurate since the specific i and n values are<br />

input. However, it is possible to make mistakes since the formulas are similar to each other, especially<br />

when uniform series are involved. Additionally, the formulas become more complex<br />

when gradients are introduced, as you will see in the following sections.<br />

A spreadsheet function determines the factor value if the corresponding P , A , or F argument<br />

in the function is set to 1 and the other parameters are omitted or set to zero. For example,<br />

the P F factor is determined using the PV function with A omitted (or set to 0) and<br />

F 1, that is, PV( i %,n,,1) or PV( i %, n ,0,1). A minus sign preceding the function identifier<br />

causes the factor to have a positive value. Functions to determine the six common factors are<br />

as follows.


2.4 Factor Values for Untabulated i or n Values 49<br />

Factor To Do This Excel Function<br />

PF Find P, given F. PV(i%,n,,1)<br />

FP Find F, given P. FV(i%,n,,1)<br />

PA Find P, given A. PV(i%,n,1)<br />

AP Find A, given P. PMT(i%,n,1)<br />

FA Find F, given A. FV(i%,n,1)<br />

AF Find A, given F. PMT(i%,n,,1)<br />

Figure 2–9 shows a spreadsheet developed explicitly to determine these factor values. When it is<br />

made live in Excel, entering any combination of i and n displays the exact value for all six factors. The<br />

values for i 3.25% and n 25 years are shown here. As we already know, these same functions will<br />

determine a final P , A , or F value when actual or estimated cash flow amounts are entered.<br />

Linear interpolation for an untabulated interest rate i or number of years n takes more time<br />

to complete than using the formula or spreadsheet function. Also interpolation introduces some<br />

level of inaccuracy, depending upon the distance between the two boundary values selected for<br />

i or n , as the formulas themselves are nonlinear functions. Interpolation is included here for individuals<br />

who wish to utilize it in solving problems. Refer to Figure 2–10 for a graphical description<br />

of the following explanation. First, select two tabulated values ( x 1 and x 2 ) of the parameter<br />

for which the factor is requested, that is, i or n , ensuring that the two values surround and are not<br />

too distant from the required value x . Second, find the corresponding tabulated factor values<br />

( f 1 and f 2 ). Third, solve for the unknown, linearly interpolated value f using the formulas below,<br />

where the differences in parentheses are indicated in Figure 2–10 as a through c .<br />

Enter requested i and n<br />

Figure 2–9<br />

Use of Excel functions to display<br />

factor values for any i and n<br />

values.<br />

Factor value<br />

axis<br />

f 2<br />

Table<br />

Figure 2–10<br />

Linear interpolation in factor<br />

value tables.<br />

c<br />

f 1<br />

f<br />

Unknown<br />

d<br />

Table<br />

Linear<br />

assumption<br />

a<br />

Known<br />

x 1<br />

Required<br />

x<br />

Known<br />

x 2<br />

i or n<br />

axis<br />

b


50 Chapter 2 Factors: How Time and Interest Affect Money<br />

f f 1 (x – x 1 ) ————<br />

(x 2 – x 1 ) (f 2 – f 1 ) [2.16]<br />

f f 1 a —<br />

b c f 1 d [2.17]<br />

The value of d will be positive or negative if the factor is increasing or decreasing, respectively,<br />

in value between x 1 and x 2 .<br />

EXAMPLE 2.7<br />

Determine the PA factor value for i 7.75% and n 10 years, using the three methods described<br />

previously.<br />

Solution<br />

Factor formula: Apply the formula from inside the front cover of the book for the PA factor.<br />

Showing 5-decimal accuracy,<br />

( P A ,7.75%,10) (1 i ) n 1<br />

—————<br />

i (1 i ) n (1.0775) 10 1<br />

———————<br />

0.0775(1.0775) 1.10947<br />

10 ————<br />

0.16348<br />

6.78641<br />

Spreadsheet: Utilize the spreadsheet function in Figure 2–9, that is, PV(7.75%,10,1), to<br />

display 6.78641.<br />

Linear interpolation: Use Figure 2–10 as a reference for this solution. Apply the Equation<br />

[2.16] and [2.17] sequence, where x is the interest rate i , the bounding interest rates are<br />

i 1 7% and i 2 8%, and the corresponding PA factor values are f 1 ( PA ,7%,10) 7.0236<br />

and f 2 ( PA ,8%,10) 6.7101. With 4-place accuracy,<br />

f f 1 (i i 1<br />

———<br />

)<br />

( i 2 – i 1 ) ( f 2 – f 1 ) 7.0236 —————<br />

(7.75 7)<br />

(8 7)<br />

7.0236 (0.75)(−0.3135) 7.0236 − 0.2351<br />

6.7885<br />

(6.7101 7.0236)<br />

Comment<br />

Note that since the P A factor value decreases as i increases, the linear adjustment is negative<br />

at 0.2351. As is apparent, linear interpolation provides an approximation to the correct factor<br />

value for 7.75% and 10 years, plus it takes more calculations than using the formula or spreadsheet<br />

function. It is possible to perform two-way linear interpolation for untabulated i and n<br />

values; however, the use of a spreadsheet or factor formula is recommended.<br />

2.5 Arithmetic Gradient Factors (PG and AG)<br />

Assume a manufacturing engineer predicts that the cost of maintaining a robot will increase by<br />

$5000 per year until the machine is retired. The cash flow series of maintenance costs involves a<br />

constant gradient, which is $5000 per year.<br />

An arithmetic gradient series is a cash flow series that either increases or decreases by a constant<br />

amount each period. The amount of change is called the gradient.<br />

Formulas previously developed for an A series have year-end amounts of equal value. In the<br />

case of a gradient, each year-end cash flow is different, so new formulas must be derived. First,<br />

assume that the cash flow at the end of year 1 is the base amount of the cash flow series and,<br />

therefore, not part of the gradient series. This is convenient because in actual applications, the<br />

base amount is usually significantly different in size compared to the gradient. For example, if<br />

you purchase a used car with a 1-year warranty, you might expect to pay the gasoline and insurance<br />

costs during the first year of operation. Assume these cost $2500; that is, $2500 is the base<br />

amount. After the first year, you absorb the cost of repairs, which can be expected to increase


2.5 Arithmetic Gradient Factors (PG and AG) 51<br />

0 1 2 3 4<br />

n –1 n<br />

Time<br />

Figure 2–11<br />

Cash flow diagram of an<br />

arithmetic gradient series.<br />

$2500<br />

$2700<br />

$2900<br />

$3100<br />

$2500<br />

+ (n – 2)200 $2500<br />

+ (n – 1)200<br />

0 1 2 3 4 5<br />

n –1<br />

G<br />

2G<br />

3G<br />

4G<br />

n<br />

Time<br />

Figure 2–12<br />

Conventional arithmetic<br />

gradient series without<br />

the base amount.<br />

(n –2)G<br />

(n –1)G<br />

each year. If you estimate that total costs will increase by $200 each year, the amount the second<br />

year is $2700, the third $2900, and so on to year n , when the total cost is 2500 ( n 1)200. The<br />

cash flow diagram is shown in Figure 2–11. Note that the gradient ($200) is first observed between<br />

year 1 and year 2, and the base amount ($2500 in year 1) is not equal to the gradient.<br />

Define the symbols G for gradient and CF n for cash flow in year n as follows.<br />

G constant arithmetic change in cash flows from one time period to the next; G may be positive<br />

or negative.<br />

CF n base amount ( n 1) G [2.18]<br />

It is important to realize that the base amount defines a uniform cash flow series of the size A that<br />

occurs eash time period. We will use this fact when calculating equivalent amounts that involve<br />

arithmetic gradients. If the base amount is ignored, a generalized arithmetic (increasing) gradient<br />

cash flow diagram is as shown in Figure 2–12. Note that the gradient begins between years<br />

1 and 2. This is called a conventional gradient .<br />

EXAMPLE 2.8<br />

A local university has initiated a logo-licensing program with the clothier Holister, Inc. Estimated<br />

fees (revenues) are $80,000 for the first year with uniform increases to a total of $200,000<br />

by the end of year 9. Determine the gradient and construct a cash flow diagram that identifies<br />

the base amount and the gradient series.<br />

Solution<br />

The year 1 base amount is CF 1 $80,000, and the total increase over 9 years is<br />

CF 9 CF 1 200,000 – 80,000 $120,000<br />

Equation [2.18], solved for G , determines the arithmetic gradient.<br />

G (CF 9 CF 1<br />

——————<br />

) ————<br />

120,000<br />

n 1 9 1<br />

$15,000 per year


52 Chapter 2 Factors: How Time and Interest Affect Money<br />

CF 9 =<br />

G = $15,000<br />

$200,000<br />

$185,000<br />

$170,000<br />

$155,000<br />

$140,000<br />

$125,000<br />

$110,000<br />

CF 1 = $95,000<br />

$80,000<br />

0 1 2 3 4 5 6 7 8 9<br />

Figure 2–13<br />

Diagram for gradient series, Example 2.8.<br />

Year<br />

The cash flow diagram (Figure 2–13) shows the base amount of $80,000 in years 1 through 9<br />

and the $15,000 gradient starting in year 2 and continuing through year 9.<br />

The total present worth P T for a series that includes a base amount A and conventional arithmetic<br />

gradient must consider the present worth of both the uniform series defined by A and the<br />

arithmetic gradient series. The addition of the two results in P T .<br />

P T P A P G [2.19]<br />

where P A is the present worth of the uniform series only, P G is the present worth of the gradient<br />

series only, and the or sign is used for an increasing ( G ) or decreasing ( G ) gradient,<br />

respectively.<br />

The corresponding equivalent annual worth A T is the sum of the base amount series annual<br />

worth A A and gradient series annual worth A G , that is,<br />

A T A A A G [2.20]<br />

Three factors are derived for arithmetic gradients: the P G factor for present worth, the A G<br />

factor for annual series, and the F G factor for future worth. There are several ways to derive<br />

them. We use the single-payment present worth factor ( P F , i , n ), but the same result can be obtained<br />

by using the F P , F A , or P A factor.<br />

In Figure 2–12, the present worth at year 0 of only the gradient is equal to the sum of the present<br />

worths of the individual cash flows, where each value is considered a future amount.<br />

Factor out G and use the P F formula.<br />

P G(PF,i,2) 2G(PF,i,3) 3G(PF,i,4) . . .<br />

[(n 2)G](PF,i,n 1) [(n 1)G](PF,i,n)<br />

P G [<br />

———— 1<br />

(1 i) 2<br />

2 ————<br />

(1 i) 3<br />

3 ————<br />

(1 i) . . . <br />

4<br />

Multiplying both sides of Equation [2.21] by (1 i ) 1 yields<br />

———— n 2<br />

(1 i) n 1<br />

n1 ————<br />

(1 i) ] n [2.21]<br />

P (1 i) 1 G [<br />

———— 1<br />

(1 i) 2<br />

1 ————<br />

(1 i) 3<br />

2 ————<br />

(1 i) . . . <br />

n 2<br />

3 ————<br />

(1 i) n 1<br />

n2 ————<br />

(1 i) ] [2.22]<br />

n1<br />

Subtract Equation [2.21] from Equation [2.22] and simplify.<br />

iP G [<br />

———— 1<br />

(1 i) 1<br />

1 ————<br />

(1 i) . . . <br />

2<br />

———— 1<br />

(1 i) 1<br />

n1 ————<br />

(1 i) n ]<br />

G<br />

[<br />

n<br />

————<br />

(1 i) ] n [2.23]<br />

The left bracketed expression is the same as that contained in Equation [2.6], where the<br />

P A factor was derived. Substitute the closed-end form of the P A factor from Equation [2.8]


2.5 Arithmetic Gradient Factors (PG and AG) 53<br />

i = given<br />

P G =?<br />

0 1 2 3 4 n –1<br />

n<br />

0 1 2 3 4<br />

n –1<br />

n<br />

G<br />

2G<br />

3G<br />

(a)<br />

(n –2)G<br />

(n –1)G<br />

(b)<br />

Figure 2–14<br />

Conversion diagram from an arithmetic gradient to a present worth.<br />

into Equation [2.23] and simplify to solve for P G , the present worth of the gradient series<br />

only.<br />

P G — G i [ (1 i)n 1<br />

——————<br />

i(1 i) n ———— n<br />

(1 i) ] n [2.24]<br />

Equation [2.24] is the general relation to convert an arithmetic gradient G (not including the<br />

base amount) for n years into a present worth at year 0 . Figure 2–14 a is converted into the<br />

equivalent cash flow in Figure 2–14 b . The arithmetic gradient present worth factor, or P G<br />

factor, may be expressed in two forms:<br />

(PG,i,n) — 1 i [ (1 + i)n 1<br />

—————<br />

i(1 i) n ———— n<br />

(1 i) ] n<br />

or (PG,i,n) (1 i)n in 1<br />

————————<br />

i 2 (1 i) n [2.25]<br />

Remember: The conventional arithmetic gradient starts in year 2, and P is located in year 0.<br />

Equation [2.24] expressed as an engineering economy relation is<br />

P G G(PG,i,n) [2.26]<br />

which is the rightmost term in Equation [2.19] to calculate total present worth. The G carries a<br />

minus sign for decreasing gradients.<br />

The equivalent uniform annual series A G for an arithmetic gradient G is found by multiplying<br />

the present worth in Equation [2.26] by the ( A P , i , n ) formula. In standard notation form, the<br />

equivalent of algebraic cancellation of P can be used.<br />

In equation form,<br />

A G G(PG,i,n)(AP,i,n)<br />

G(AG,i,n)<br />

A G — G i [ (1 i)n 1<br />

—————<br />

i(1 i) n ———— n<br />

——————<br />

(1 i) n 1 ]<br />

(1 i) n ] [<br />

i(1 i) n<br />

Placement of<br />

gradient P G<br />

A G G [<br />

— 1 i —————— n<br />

(1 i) n 1 ] [2.27]<br />

which is the rightmost term in Equation [2.20]. The expression in brackets in Equation [2.27] is<br />

called the arithmetic gradient uniform series factor and is identified by ( A G , i , n ). This factor<br />

converts Figure 2–15 a into Figure 2–15 b .<br />

The P G and A G factors and relations are summarized inside the front cover. Factor values<br />

are tabulated in the two rightmost columns of factor values at the rear of this text.


54 Chapter 2 Factors: How Time and Interest Affect Money<br />

i = given<br />

0 1 2 3 4 n –1<br />

n<br />

A G = ?<br />

0 1 2 3 4<br />

n –1<br />

n<br />

G<br />

2G<br />

(a)<br />

3G<br />

(n –2)G<br />

(n –1)G<br />

(b)<br />

Figure 2–15<br />

Conversion diagram of an arithmetic gradient series to an equivalent uniform annual series.<br />

There is no direct, single-cell spreadsheet function to calculate P G or A G for an arithmetic<br />

gradient. Use the NPV function to display P G and the PMT function to display A G after entering<br />

all cash flows (base and gradient amounts) into contiguous cells. General formats for these functions<br />

are<br />

NPV(i%, second_cell:last_cell) first_cell [2.28]<br />

PMT(i%, n, cell_with_P G ) [2.29]<br />

The word entries in italic are cell references, not the actual numerical values. (See Appendix A,<br />

Section A.2, for a description of cell reference formatting.) These functions are demonstrated in<br />

Example 2.10.<br />

An FG factor ( arithmetic gradient future worth factor ) to calculate the future worth F G of a<br />

gradient series can be derived by multiplying the P G and F P factors. The resulting factor,<br />

( F G , i , n ), in brackets, and engineering economy relation is<br />

F G G [ ( — 1 i ) ( (1 i)n – 1<br />

—————<br />

i ) n ]<br />

EXAMPLE 2.9<br />

Neighboring parishes in Louisiana have agreed to pool road tax resources already designated<br />

for bridge refurbishment. At a recent meeting, the engineers estimated that a total of<br />

$500,000 will be deposited at the end of next year into an account for the repair of old and<br />

safety-questionable bridges throughout the area. Further, they estimate that the deposits<br />

will increase by $100,000 per year for only 9 years thereafter, then cease. Determine the<br />

equivalent (a) present worth and (b) annual series amounts, if public funds earn at a rate<br />

of 5% per year.<br />

Solution<br />

(a) The cash flow diagram of this conventional arithmetic gradient series from the perspective<br />

of the parishes is shown in Figure 2–16. According to Equation [2.19], two computations<br />

must be made and added: the first for the present worth of the base amount P A<br />

and the second for the present worth of the gradient P G . The total present worth P T<br />

occurs in year 0. This is illustrated by the partitioned cash flow diagram in Figure 2–17.<br />

In $1000 units, the total present worth is<br />

P T 500(PA,5%,10) 100(PG,5%,10)<br />

500(7.7217) 100(31.6520)<br />

$7026.05 ($7,026,050)


2.5 Arithmetic Gradient Factors (PG and AG) 55<br />

0 1<br />

2<br />

3<br />

4<br />

5<br />

6<br />

7<br />

8<br />

9<br />

10<br />

$500<br />

$600<br />

$700<br />

$800<br />

$900<br />

$1000<br />

$1100<br />

$1200<br />

$1300<br />

$1400<br />

Figure 2–16<br />

Cash flow series with a conventional arithmetic gradient (in $1000 units),<br />

Example 2.9.<br />

P A =?<br />

A = $500<br />

1 2 9 10<br />

+<br />

P G =?<br />

G = $100<br />

1 2 9 10<br />

$100<br />

Base<br />

Gradient<br />

$900<br />

P T =?<br />

1<br />

2<br />

3<br />

4<br />

P T = P A + P G<br />

6<br />

5<br />

7<br />

8<br />

9<br />

10<br />

$500<br />

$600<br />

$700<br />

$800<br />

$900<br />

Figure 2–17<br />

Partitioned cash flow diagram (in $1000 units), Example 2.9.<br />

$1000<br />

$1100<br />

$1200<br />

$1300<br />

$1400<br />

(b) Here, too, it is necessary to consider the gradient and the base amount separately. The<br />

total annual series A T is found by Equation [2.20] and occurs in years 1 through 10.<br />

A T 500 100(AG,5%,10) 500 100(4.0991)<br />

$909.91 per year ($909,910)<br />

Comment<br />

Remember: The PG and AG factors determine the present worth and annual series of the<br />

gradient only. Any other cash flows must be considered separately.<br />

If the present worth is already calculated [as in part (a)], P T can be multiplied by an AP<br />

factor to get A T . In this case, considering round-off error,<br />

A T P T (AP,5%,10) 7026.05(0.12950)<br />

$909.873 ($909,873)


56 Chapter 2 Factors: How Time and Interest Affect Money<br />

EXAMPLE 2.10 The Cement Factory Case<br />

The announcement of the HAC cement factory states that the $200 million (M) investment is<br />

planned for <strong>2012</strong>. Most large investment commitments are actually spread out over several<br />

years as the plant is constructed and production is initiated. Further investigation may determine,<br />

for example, that the $200 M is a present worth in the year <strong>2012</strong> of anticipated investments<br />

during the next 4 years (2013 through 2016). Assume the amount planned for 2013 is<br />

$100 M with constant decreases of $25 M each year thereafter. As before, assume the time<br />

value of money for investment capital is 10% per year to answer the following questions using<br />

tabulated factors and spreadsheet functions, as requested below.<br />

(a) In equivalent present worth values, does the planned decreasing investment series equal<br />

the announced $200 M in <strong>2012</strong>? Use both tabulated factors and spreadsheet functions.<br />

(b) Given the planned investment series, what is the equivalent annual amount that will be<br />

invested from 2013 to 2016? Use both tabulated factors and spreadsheet functions.<br />

(c) (This optional question introduces Excel’s Goal Seek tool.) What must be the amount of<br />

yearly constant decrease through 2016 to have a present worth of exactly $200 M in<br />

<strong>2012</strong>, provided $100 M is expended in 2013? Use a spreadsheet.<br />

Solution<br />

(a) The investment series is a decreasing arithmetic gradient with a base amount of $100 M<br />

in year 1 (2013) and G $25 M through year 4 (2016). Figure 2–18 diagrams the cash<br />

flows with the shaded area showing the constantly declining investment each year. The P T<br />

value at time 0 at 10% per year is determined by using tables and a spreadsheet.<br />

Tabulated factors: Equation [2.19] with the minus sign for negative gradients determines<br />

the total present worth P T . Money is expressed in $1 million units.<br />

P T P A P G 100(PA,10%,4) 25(PG,10%,4) [2.30]<br />

100(3.1699) – 25(4.3781)<br />

$207.537 ($207,537,000)<br />

In present worth terms, the planned series will exceed the equivalent of $200 M in <strong>2012</strong><br />

by approximately $7.5 M.<br />

Spreadsheet: Since there is no spreadsheet function to directly display present worth for a<br />

gradient series, enter the cash flows in a sequence of cells (rows or columns) and use the<br />

NPV function to find present worth. Figure 2–19 shows the entries and function<br />

NPV(i%,second_cell:last_cell). There is no first_cell entry here, because there is no<br />

investment per se in year 0. The result displayed in cell C9, $207.534, is the total P T for<br />

the planned series. (Note that the NPV function does not consider two separate series of<br />

cash flows as is necessary when using tabulated factors.)<br />

The interpretation is the same as in part (a); the planned investment series exceeds the<br />

$200 M in present worth terms by approximately $7.5 M.<br />

(b) Tabulated factors: There are two equally correct ways to find A T . First, apply Equation<br />

[2.20] that utilizes the AG factor, and second, use the P T value obtained above and<br />

the AP factor. Both relations are illustrated here, in $1 million units,<br />

PE<br />

P T ?<br />

i = 10% per year<br />

Figure 2–18<br />

Cash flow diagram for decreasing<br />

gradient in $1 million<br />

units, Example 2.10.<br />

2013<br />

2014<br />

2015<br />

2016<br />

Year<br />

0<br />

1<br />

2<br />

3<br />

4<br />

Time<br />

Base<br />

A $100<br />

$25<br />

$50<br />

$100<br />

Gradient<br />

G $25


2.5 Arithmetic Gradient Factors (PG and AG) 57<br />

Figure 2–19<br />

Spreadsheet solution for<br />

Example 2.10a and b.<br />

Present worth of investments<br />

NPV(10%,C5:C8)<br />

Annual worth of investments<br />

PMT(10%,4,C9)<br />

Use Equation [2.20]:<br />

Use P T :<br />

A T 100 – 25(AG,10%,4) 100 25(1.3812)<br />

$65.471 ($65,471,000 per year)<br />

A T 207.537(AP,10%,4) 207.537(0.31547)<br />

$65.471 per year<br />

Spreadsheet: Apply the PMT function in Equation [2.29] to obtain the same A T <br />

$65.471 per year (Figure 2–19).<br />

(c) (Optional) The Goal Seek tool is described in Appendix A. It is an excellent tool to apply<br />

when one cell entry must equal a specific value and only one other cell can change. This<br />

is the case here; the NPV function (cell C9 in Figure 2–19) must equal $200, and the gradient<br />

G (cell C1) is unknown. This is the same as stating P T 200 in Equation [2.30] and<br />

solving for G. All other parameters retain their current value.<br />

Figure 2–20 (top) pictures the same spreadsheet used previously with the Goal Seek template<br />

added and loaded. When OK is clicked, the solution is displayed; G $26.721.<br />

Refer to Figure 2–20 again. This means that if the investment is decreased by a constant<br />

annual amount of $26.721 M, the equivalent total present worth invested over the 4 years<br />

will be exactly $200 M.<br />

Figure 2–20<br />

Solution for arithmetic<br />

gradient using Goal Seek,<br />

Example 2.10c.<br />

Present worth of investments:<br />

NPV(10%,C5:C8)<br />

Set up Goal Seek template<br />

Solution for G = $26.721 to make<br />

present worth exactly $200


58 Chapter 2 Factors: How Time and Interest Affect Money<br />

2.6 Geometric Gradient Series Factors<br />

It is common for annual revenues and annual costs such as maintenance, operations, and labor to<br />

go up or down by a constant percentage, for example, 5% or 3% per year. This change occurs<br />

every year on top of a starting amount in the first year of the project. A definition and description<br />

of new terms follow.<br />

A geometric gradient series is a cash flow series that either increases or decreases by a constant<br />

percentage each period. The uniform change is called the rate of change.<br />

g constant rate of change , in decimal form, by which cash flow values increase or decrease<br />

from one period to the next. The gradient g can be or .<br />

A 1 initial cash flow in year 1 of the geometric series<br />

P g<br />

present worth of the entire geometric gradient series, including the initial amount<br />

A 1<br />

Note that the initial cash flow A 1 is not considered separately when working with geometric<br />

gradients.<br />

Figure 2–21 shows increasing and decreasing geometric gradients starting at an amount A 1 in<br />

time period 1 with present worth P g located at time 0. The relation to determine the total present<br />

worth P g for the entire cash flow series may be derived by multiplying each cash flow in Figure<br />

2–21 a by the P F factor 1(1 i )<br />

n<br />

.<br />

A<br />

P g ———— 1<br />

(1 i) A 1 (1 g)<br />

1 ————<br />

(1 i) A 1 (1 g)2<br />

2 —————<br />

(1 i) . . . A 1 (1 g)n1<br />

3 ——————<br />

(1 i) n<br />

A 1 ——— 1<br />

[ 1 i 1 g (1 g)2<br />

———— ————<br />

2<br />

(1 i) (1 i) . . .<br />

(1 g)n1<br />

3 —————<br />

(1 i) ] n [2.31]<br />

Multiply both sides by (1 g )(1 i ), subtract Equation [2.31] from the result, factor out P g ,<br />

and obtain<br />

P g ( ———<br />

1 g<br />

1 i 1 ) A (1 g)n<br />

1 —————<br />

[ (1 i) ——— 1<br />

n1 1 i ]<br />

Solve for P g and simplify.<br />

P g A 1 [ 1 (———<br />

1 g<br />

1 i ) n<br />

———————<br />

i g ] g i [2.32]<br />

The term in brackets in Equation [2.32] is the ( P A , g , i , n ) or geometric gradient series present<br />

worth factor for values of g not equal to the interest rate i. When g i , substitute i for g in Equation<br />

[2.31] and observe that the term 1/(1 + i) appears n times.<br />

P g =?<br />

i = given<br />

g = given<br />

P g =?<br />

i = given<br />

g = given<br />

0 1 2<br />

A 1<br />

3<br />

A 1 (1 + g)<br />

A 1 (1 + g) 2<br />

4<br />

A 1 (1 + g) 3<br />

n<br />

0 1 2 3 4<br />

A 1 (1 – g)<br />

A n –1<br />

1 (1 – g) 3<br />

A 1 (1 – g) 2<br />

A 1 (1 – g)<br />

n<br />

(a)<br />

A 1 (1 + g) n –1<br />

A 1<br />

(b)<br />

Figure 2–21<br />

Cash flow diagram of (a) increasing and (b) decreasing geometric gradient series and present worth P g .


2.6 Geometric Gradient Series Factors 59<br />

P g A 1 (<br />

——— 1<br />

(1 i ) ——— 1<br />

(1 i ) ——— 1<br />

(1 i ) ... <br />

nA<br />

P g ——— 1<br />

(1 i)<br />

——— 1<br />

(1 i ) )<br />

[2.33]<br />

The (PA,g,i,n) factor calculates P g in period t 0 for a geometric gradient series starting in<br />

period 1 in the amount A 1 and increasing by a constant rate of g each period.<br />

The equation for P g and the ( P A , g , i , n ) factor formula are<br />

Placement of<br />

Gradient P g<br />

(PA,g,i,n) <br />

P g A 1 (PA,g,i,n) [2.34]<br />

1 ( ———<br />

1 g<br />

1 i ) n<br />

——————<br />

i − g<br />

n ———<br />

1 i<br />

g i<br />

g i<br />

[2.35]<br />

It is possible to derive factors for the equivalent A and F values; however, it is easier to determine<br />

the P g amount and then multiply by the A P or F P factor.<br />

As with the arithmetic gradient series, there are no direct spreadsheet functions for geometric<br />

gradient series. Once the cash flows are entered, P and A are determined using the NPV and PMT<br />

functions, respectively.<br />

EXAMPLE 2.11<br />

A coal-fired power plant has upgraded an emission control valve. The modification costs only<br />

$8000 and is expected to last 6 years with a $200 salvage value. The maintenance cost is expected<br />

to be high at $1700 the first year, increasing by 11% per year thereafter. Determine the<br />

equivalent present worth of the modification and maintenance cost by hand and by spreadsheet<br />

at 8% per year.<br />

Solution by Hand<br />

The cash flow diagram (Figure 2–22) shows the salvage value as a positive cash flow and all<br />

costs as negative. Use Equation [2.35] for g i to calculate P g . Total P T is the sum of three<br />

present worth components.<br />

P T =?<br />

P g =?<br />

$8000<br />

P T 8000 P g 200( P F ,8%,6)<br />

8000 1700 [ 1 (1.111.08) 6<br />

———————<br />

0.08 0.11 ] 200( P F ,8%,6)<br />

8000 1700(5.9559) 126 $17,999<br />

1<br />

$1700<br />

2<br />

i =8%<br />

g = 11%<br />

3<br />

4 5<br />

6<br />

$1700(1.11)<br />

$1700(1.11) 2<br />

$1700(1.11) 3<br />

$1700(1.11) 4<br />

Figure 2–22<br />

Cash flow diagram of a geometric<br />

gradient, Example 2.11.<br />

$200<br />

$1700(1.11) 5


60 Chapter 2 Factors: How Time and Interest Affect Money<br />

Solution by Spreadsheet<br />

Figure 2–23 details the spreadsheet operations to find the geometric gradient present worth P g<br />

and total present worth P T . To obtain P T $17,999, three components are summed—first<br />

cost, present worth of estimated salvage in year 6, and P g . Cell tags detail the relations for the<br />

second and third components; the first cost occurs at time 0.<br />

Comment<br />

The relation that calculates the ( PA,g,i%,n ) factor is rather complex, as shown in the cell tag<br />

and formula bar for C9. If this factor is used repeatedly, it is worthwhile using cell reference<br />

formatting so that A 1 , i , g , and n values can be changed and the correct value is always obtained.<br />

Try to write the relation for cell C9 in this format.<br />

Present worth of salvage<br />

PV(8%,6,,200)<br />

Present worth of maintenance costs, Eq. [2.35]<br />

1700 * ((1-((1.11)/(1.08))^6)/(0.08-0.11))<br />

Figure 2–23<br />

Geometric gradient and total present worth calculated via spreadsheet, Example 2.11.<br />

EXAMPLE 2.12 The Cement Factory Case<br />

Now let’s go back to the proposed Houston American Cement plant in Georgia. The revenue<br />

series estimate of $50 million annually is quite optimistic, especially since there are many<br />

other cement product plants operating in Florida and Georgia on the same limestone deposit.<br />

(The website for the HAC plant shows where they are located currently; it is clear that keen<br />

competition will be present.) Therefore, it is important to be sensitive in our analysis to possibly<br />

declining and increasing revenue series, depending upon the longer-term success of the<br />

plant’s marketing, quality, and reputation. Assume that revenue may start at $50 million by the<br />

end of the first year, but then decreases geometrically by 12% per year through year 5. Determine<br />

the present worth and future worth equivalents of all revenues during this 5-year time<br />

frame at the same rate used previously, that is, 10% per year.<br />

Solution<br />

The cash flow diagram appears much like Figure 2–21 b , except that the arrows go up for revenues.<br />

In year 1, A 1 $50 M and revenues decrease in year 5 to<br />

A 1 (1 g ) n 1<br />

50 M(1 0.12) 5–1 50 M(0.88) 4 $29.98 M<br />

First, we determine P g in year 0 using Eq. [2.35] with i 0.10 and g 0.12, then we calculate<br />

F in year 5. In $1 million units,<br />

1 <br />

P g 50 [ (<br />

—— 0.88<br />

$152.80<br />

1.10 ) 5<br />

———————<br />

0.10 (0.12) ] 50[3.0560]<br />

F 152.80( FP ,10%,5) 152.80(1.6105)<br />

$246.08<br />

This means that the decreasing revenue stream has a 5-year future equivalent worth of<br />

$246.080 M. If you look back to Example 2.6, we determined that the F in year 5 for the<br />

PE


2.7 Determining i or n for Known Cash Flow Values 61<br />

uniform revenue series of $50 M annually is $305.255 M. In conclusion, the 12% declining<br />

geometric gradient has lowered the future worth of revenue by $59.175 M, which is a sizable<br />

amount from the perspective of the owners of Votorantim Cimentos North America, Inc.<br />

2.7 Determining i or n for Known Cash Flow Values<br />

When all the cash flow values are known or have been estimated, the i value (interest rate or rate<br />

of return) or n value (number of years) is often the unknown. An example for which i is sought<br />

may be stated as follows: A company invested money to develop a new product. After the net<br />

annual income series is known following several years on the market, determine the rate of return<br />

i on the investment. There are several ways to find an unknown i or n value, depending upon the<br />

nature of the cash flow series and the method chosen to find the unknown. The simplest case involves<br />

only single amounts ( P and F ) and solution utilizing a spreadsheet function. The most<br />

difficult and complex involves finding i or n for irregular cash flows mixed with uniform and<br />

gradient series utilizing solution by hand and calculator. The solution approaches are summarized<br />

below, followed by examples.<br />

Single Amounts— P and F Only<br />

Hand or Calculator Solution Set up the equivalence relation and (1) solve for the variable<br />

using the factor formula, or (2) find the factor value and interpolate in the tables.<br />

Spreadsheet Solution Use the IRR or RATE function to find i or the NPER function to find<br />

n . (See below and Appendix A for details.)<br />

Uniform Series— A Series<br />

Hand or Calculator Solution Set up the equivalence relation using the appropriate factor<br />

( P/A , A/P , F/A , or A/F ), and use the second method mentioned above.<br />

Spreadsheet Solution Use the IRR or RATE function to find i or the NPER function to find n .<br />

Mixed A Series, Gradients, and/or Isolated Values<br />

Hand or Calculator Solution Set up the equivalence relation and use (1) trial and error or<br />

(2) the calculator functions.<br />

Spreadsheet Solution Use the IRR or RATE function to find i or the NPER function to find<br />

n . (This is the recommended approach.)<br />

Besides the PV, FV, and NPV functions, other spreadsheet functions useful in determining i<br />

are IRR (internal rate of return) and RATE, and NPER (number of periods) to find n . The formats<br />

are shown here and the inside front cover with a detailed explanation in Appendix A. In all three<br />

of these functions, at least one cash flow entry must have a sign opposite that of others in order<br />

to find a solution.<br />

IRR(first_cell:last_cell) [2.36]<br />

To use IRR to find i , enter all cash flows into contiguous cells, including zero values.<br />

RATE( n , A , P , F ) [2.37]<br />

The single-cell RATE function finds i when an A series and single P and/or F values are involved.<br />

NPER( i %, A , P , F ) [2.38]<br />

NPER is a single-cell function to find n for single P and F values, or with an A series.


62 Chapter 2 Factors: How Time and Interest Affect Money<br />

EXAMPLE 2.13<br />

If Laurel made a $30,000 investment in a friend’s business and received $50,000 5 years later,<br />

determine the rate of return.<br />

Solution<br />

Since only single amounts are involved, i can be determined directly from the P / F factor.<br />

P F ( P F , i , n ) F ———— 1<br />

(1 i ) n<br />

30,000 50,000 ———— 1<br />

(1 i ) 5<br />

0.600 ———— 1<br />

(1 i ) 5<br />

i ( 1 ——<br />

0.6 ) 0.2 1 0.1076 (10.76%)<br />

Alternatively, the interest rate can be found by setting up the standard P F relation, solving for<br />

the factor value, and interpolating in the tables.<br />

P F ( P F , i , n )<br />

30,000 50,000( P F , i ,5)<br />

( P F , i ,5) 0.60<br />

From the interest tables, a P / F factor of 0.6000 for n 5 lies between 10% and 11%. Interpolate<br />

between these two values to obtain i 10.76%.<br />

EXAMPLE 2.14<br />

Pyramid Energy requires that for each of its offshore wind power generators $5000 per year<br />

be placed into a capital reserve fund to cover unexpected major rework on field equipment.<br />

In one case, $5000 was deposited for 15 years and covered a rework costing $100,000 in<br />

year 15. What rate of return did this practice provide to the company? Solve by hand and<br />

spreadsheet.<br />

Solution by Hand<br />

The cash flow diagram is shown in Figure 2–24 . Either the A F or F A factor can be used.<br />

Using A F ,<br />

A F ( A F , i , n )<br />

5000 100,000( A F , i ,15)<br />

( A F , i ,15) 0.0500<br />

From the AF interest tables for 15 years, the value 0.0500 lies between 3% and 4%. By interpolation,<br />

i 3.98%.<br />

i = ?<br />

F = $100,000<br />

Figure 2–24<br />

Diagram to determine the rate<br />

of return, Example 2.14.<br />

0<br />

1<br />

2 3 4 5 6 7 8 9 10 11 12 13 14 15<br />

A = $5000


2.7 Determining i or n for Known Cash Flow Values 63<br />

Solution by Spreadsheet<br />

Refer to the cash flow diagram ( Figure 2–24 ) while completing the spreadsheet ( Figure 2–25 ).<br />

A single-cell solution using the RATE function can be applied since A $5000 occurs<br />

each year and F $100,000 takes place in the last year of the series. The function<br />

RATE(15,5000,,100000) displays the value i 3.98%. This function is fast, but it allows<br />

only limited sensitivity analysis because all the A values have to change by the same amount.<br />

The IRR function is much better for answering “what if ” questions.<br />

To apply the IRR function, enter the value 0 in a cell (for year 0), followed by –5000 for<br />

14 years and in year 15 enter 95,000 ( Figure 2–25 ). In any cell enter the IRR function. The<br />

answer i 3.98% is displayed. It is advisable to enter the year numbers 0 through n (15 in<br />

this example) in the column immediately to the left of the cash flow entries. The IRR function<br />

does not need these numbers, but it makes the cash flow entry activity easier and more<br />

accurate. Now any cash flow can be changed, and a new rate will be displayed immediately<br />

via IRR.<br />

i using RATE function<br />

RATE(15,-5000,,100000)<br />

Figure 2–25<br />

Use of RATE and IRR<br />

functions to determine<br />

i value for a uniform<br />

series, Example 2.14.<br />

i using IRR function<br />

IRR(E2:E17))<br />

EXAMPLE 2.15 The Cement Factory Case<br />

From the introductory comments about the HAC plant, the annual revenue is planned to be<br />

$50 million. All analysis thus far has taken place at 10% per year; however, the parent company<br />

has made it clear that its other international plants are able to show a 20% per year return<br />

on the initial investment. Determine the number of years required to generate 10%, 15%, and<br />

20% per year returns on the $200 million investment at the Georgia site.<br />

PE<br />

Solution<br />

If hand solution is utilized, the present worth relation can be established and the n values<br />

interpolated in the tables for each of the three rate of return values. In $1 million units, the<br />

relation is<br />

( PA , i %, n ) 4.00<br />

P 200 50( PA , i %, n ) ( i 10%, 15%, 20%)<br />

This is a good opportunity to utilize a spreadsheet and repeated NPER functions from<br />

Equation [2.38], since several i values are involved. Figure 2–26 shows the single-cell<br />

NPER( i %,50,200) function for each rate of return. The number of years (rounded up) to<br />

produce at least the required returns are<br />

Return, i% Years<br />

10 6<br />

15 7<br />

20 9


64 Chapter 2 Factors: How Time and Interest Affect Money<br />

Figure 2–26<br />

Use of NPER function to find n values for various rate of return requirements,<br />

Example 2.15.<br />

CHAPTER SUMMARY<br />

Formulas and factors derived and applied in this chapter perform equivalence calculations for<br />

present, future, annual, and gradient cash flows. Capability in using these formulas and their<br />

standard notation manually and with spreadsheets is critical to complete an engineering economy<br />

study. Using these formulas and spreadsheet functions, you can convert single cash flows into<br />

uniform cash flows, gradients into present worths, and much more. Additionally, you can solve<br />

for rate of return i or time n .<br />

PROBLEMS<br />

Use of Interest Tables<br />

2.1 Look up the numerical value for the following factors<br />

from the interest tables.<br />

1. ( PF ,6%,8)<br />

2. ( AP ,10%,10)<br />

3. ( AG ,15%,20)<br />

4. ( AF ,2%,30)<br />

5. ( PG ,35%,15)<br />

Determination of F , P , and A<br />

2.2 How much can Haydon Rheosystems, Inc., afford<br />

to spend now on an energy management system if<br />

the software will save the company $21,300 per<br />

year for the next 5 years? Use an interest rate of<br />

10% per year.<br />

2.3 A manufacturer of off-road vehicles is considering<br />

the purchase of dual-axis inclinometers for installation<br />

in a new line of tractors. The distributor of the<br />

inclinometers is temporarily overstocked and is offering<br />

them at a 40% discount from the regular cost<br />

of $142. If the purchaser gets them now instead of<br />

2 years from now, which is when they will be<br />

needed, what is the present worth of the savings per<br />

unit? The company would pay the regular price, if<br />

purchased in 2 years. Assume the interest rate is<br />

10% per year.<br />

2.4 The Moller Skycar M400 is a flying car known<br />

as a personal air vehicle (PAV) that is expected<br />

to be FAA-certified by December 31, 2011. The<br />

cost is $985,000, and a $100,000 deposit will hold<br />

one of the first 100 “cars.” Assume a buyer pays<br />

the $885,000 balance 3 years after making the<br />

$100,000 deposit. At an interest rate of 10% per<br />

year, what is the effective total cost of the PAV in<br />

year 3?<br />

2.5 A family that won a $100,000 prize on America’s<br />

Funniest Home Videos decided to put one-half of<br />

the money in a college fund for their child who<br />

was responsible for the prize. If the fund earned<br />

interest at 6% per year, how much was in the account<br />

14 years after it was started?<br />

2.6 One of the biggest vulnerabilities in a control system<br />

is network devices, such as Ethernet-based<br />

network switches that are located in unsecured<br />

locations and accessible to everyone. DeltaX<br />

switches, manufactured by Dahne Security, allow<br />

the user to automatically lock and unlock the port<br />

access to all switches in the network. The company<br />

is considering expanding its manufacturing<br />

lines now or doing it in 3 years. If the cost now<br />

would be $1.9 million, what equivalent amount<br />

could the company afford to spend in 3 years? The<br />

interest rate is 15% per year.<br />

2.7 A company that sells high-purity laboratory chemicals<br />

is considering investing in new equipment


Problems 65<br />

that will reduce cardboard costs by better matching<br />

the size of the products to be shipped to the<br />

size of the shipping container. If the new equipment<br />

will cost $220,000 to purchase and install,<br />

how much must the company save each year for<br />

3 years in order to justify the investment, if the<br />

interest rate is 10% per year?<br />

2.8 Red Valve Co. of Carnegie, Pennsylvania, makes a<br />

control pinch valve that provides accurate, repeatable<br />

control of abrasive and corrosive slurries, outlasting<br />

gate, plug, ball, and even satellite coated<br />

valves. How much can the company afford to<br />

spend now on new equipment in lieu of spending<br />

$75,000 four years from now? The company’s rate<br />

of return is 12% per year.<br />

2.9 If GHD Plastics purchases a new building now for<br />

$1.3 million for its corporate headquarters, what<br />

must the building be worth in 10 years? The company<br />

expects all expenditures to earn a rate of return<br />

of at least 18% per year.<br />

2.10 CGK Rheosystems makes high-performance rotational<br />

viscometers capable of steady shear and<br />

yield stress testing in a rugged, compact footprint.<br />

How much could the company afford to spend now<br />

on new equipment in lieu of spending $200,000<br />

one year from now and $300,000 three years from<br />

now, if the company uses an interest rate of 15%<br />

per year?<br />

2.11 Five years ago a consulting engineer purchased a<br />

building for company offices constructed of bricks<br />

that were not properly fired. As a result, some of<br />

the bricks were deteriorated from their exposure to<br />

rain and snow. Because of the problem with the<br />

bricks, the selling price of the building was 25%<br />

below the price of comparable, structurally sound<br />

buildings. The engineer repaired the damaged<br />

bricks and arrested further deterioration by applying<br />

an extra-strength solvent-based RTV elastomeric<br />

sealant. This resulted in restoring the building<br />

to its fair market value. If the depressed<br />

purchase price of the building was $600,000 and<br />

the cost of getting it repaired was $25,000, what is<br />

the equivalent value of the “forced appreciation”<br />

today, if the interest rate is 8% per year?<br />

2.12 Metso Automation, which manufactures addressable<br />

quarter-turn electric actuators, is planning to<br />

set aside $100,000 now and $150,000 one year<br />

from now for possible replacement of the heating<br />

and cooling systems in three of its larger manufacturing<br />

plants. If the replacement won’t be needed<br />

for 4 years, how much will the company have in<br />

the account, if it earns interest at a rate of 8% per<br />

year?<br />

2.13 Syringe pumps often fail because reagents adhere<br />

to the ceramic piston and deteriorate the seal. Trident<br />

Chemical developed an integrated polymer<br />

dynamic seal that provides a higher sealing force<br />

on the sealing lip, resulting in extended seal life.<br />

One of Trident’s customers expects to reduce<br />

downtime by 30% as a result of the new seal design.<br />

If lost production would have cost the company<br />

$110,000 per year for the next 4 years, how<br />

much could the company afford to spend now on<br />

the new seals, if it uses an interest rate of 12% per<br />

year?<br />

2.14 China spends an estimated $100,000 per year on<br />

cloud seeding efforts, which includes using antiaircraft<br />

guns and rocket launchers to fill the sky<br />

with silver iodide. In the United States, utilities<br />

that run hydroelectric dams are among the most<br />

active cloud seeders, because they believe it is a<br />

cost-effective way to increase limited water supplies<br />

by 10% or more. If the yields of cash crops<br />

will increase by 4% each year for the next 3 years<br />

because of extra irrigation water captured behind<br />

dams during cloud seeding, what is the maximum<br />

amount the farmers should spend now on the cloud<br />

seeding activity? The value of the cash crops without<br />

the extra irrigation water would be $600,000<br />

per year. Use an interest rate of 10% per year.<br />

2.15 The Public Service Board (PSB) awarded two contracts<br />

worth a combined $1.07 million to improve<br />

(i.e., deepen) a retention basin and reconstruct the<br />

spillway that was severely damaged in a flood<br />

2 years ago. The PSB said that, because of the weak<br />

economy, the bids came in $950,000 lower than engineers<br />

expected. If the projects are assumed to<br />

have a 20-year life, what is the annual worth of the<br />

savings at an interest rate of 6% per year?<br />

2.16 The National Highway Traffic Safety Administration<br />

raised the average fuel efficiency standard to<br />

35.5 miles per gallon for cars and light trucks by<br />

the year 2016. The rules will cost consumers an<br />

average of $434 extra per vehicle in the <strong>2012</strong><br />

model year. If a person purchases a new car in<br />

<strong>2012</strong> and keeps it for 5 years, how much must be<br />

saved in fuel costs each year to justify the extra<br />

cost? Use an interest rate of 8% per year.<br />

2.17 In an effort to reduce childhood obesity by reducing<br />

the consumption of sugared beverages,<br />

some states have imposed taxes on soda and<br />

other soft drinks. A survey by Roland Sturm of<br />

7300 fifth-graders revealed that if taxes averaged<br />

4 cents on each dollar’s worth of soda, no real<br />

difference in overall consumption was noticed.<br />

However, if taxes were increased to 18 cents on<br />

the dollar, Sturm calculated they would make a


66 Chapter 2 Factors: How Time and Interest Affect Money<br />

significant difference. For a student who consumes<br />

100 sodas per year, what is the future worth<br />

of the extra cost from 4 cents to 18 cents per<br />

soda? Assume the student consumes sodas from<br />

grade 5 through graduation in grade 12. Use an<br />

interest rate of 6% per year.<br />

2.18 The Texas Tomorrow Fund (TTF) is a program<br />

started in 1996 in Texas wherein parents could prepay<br />

their child's college tuition when the child was<br />

young. Actuaries set the price based on costs and<br />

investment earnings at that time. Later, the Texas<br />

legislature allowed universities to set their own tuition<br />

rates; tuition costs jumped dramatically. The<br />

cost for entering a newborn in 1996 was $10,500.<br />

If the TTF fund grew at a rate of 4% per year,<br />

while tuition costs increased at 7% per year, determine<br />

the state’s shortfall when a newborn enters<br />

college 18 years later.<br />

2.19 Henry Mueller Supply Co. sells tamperproof,<br />

normally open thermostats (i.e., thermostat closes<br />

as temperature rises). Annual cash flows are<br />

shown in the table below. Determine the future<br />

worth of the net cash flows at an interest rate of<br />

10% per year.<br />

Year 1 2 3 4 5 6 7 8<br />

Income, $1000 200 200 200 200 200 200 200 200<br />

Cost, $1000 90 90 90 90 90 90 90 90<br />

2.20 A company that makes self-clinching fasteners expects<br />

to purchase new production-line equipment<br />

in 3 years. If the new units will cost $350,000, how<br />

much should the company set aside each year, if<br />

the account earns 10% per year?<br />

Factor Values<br />

2.21 Find the numerical value of the following factors<br />

using ( a ) interpolation and ( b ) the formula.<br />

1. ( AP ,13%,15)<br />

2. ( PG ,27%,10)<br />

2.22 Find the numerical value of the following factors<br />

using ( a ) interpolation, ( b ) the formula, and ( c ) a<br />

spreadsheet function.<br />

1. ( FP ,14%,62)<br />

2. ( AF ,1%,45)<br />

2.23 For the factor ( FP ,10%,43), find the percent difference<br />

between the interpolated and formulacalculated<br />

values, assuming the formula- calculated<br />

value is the correct one.<br />

2.24 For the factor ( FA ,15%,52), find the percent difference<br />

between the interpolated and formula-<br />

calculated values, assuming the formula- calculated<br />

value is the correct one.<br />

Arithmetic Gradient<br />

2.25 Profits from recycling paper, cardboard, aluminum,<br />

and glass at a liberal arts college have increased<br />

at a constant rate of $1100 in each of the<br />

last 3 years. If this year’s profit (end of year 1) is<br />

expected to be $6000 and the profit trend continues<br />

through year 5, ( a ) what will the profit be at<br />

the end of year 5 and ( b ) what is the present<br />

worth of the profit at an interest rate of 8% per<br />

year?<br />

2.26 A report by the Government Accountability Office<br />

(GAO) shows that the GAO expects the<br />

U.S. Postal Service to lose a record $7 billion at<br />

the end of this year, and if the business model is<br />

not changed, the losses will total $241 billion by<br />

the end of year 10. If the losses increase uniformly<br />

over the 10-year period, determine the<br />

following:<br />

(a) The expected increase in losses each year<br />

(b)<br />

(c)<br />

The loss 5 years from now<br />

The equivalent uniform worth of the losses at<br />

an interest rate of 8% per year<br />

2.27 Rolled ball screws are suitable for high-precision<br />

applications such as water jet cutting. Their total<br />

manufacturing cost is expected to decrease because<br />

of increased productivity, as shown in the<br />

table. Determine the equivalent annual cost at an<br />

interest rate of 8% per year.<br />

Year 1 2 3 4 5 6 7 8<br />

Cost, $1000 200 195 190 185 180 175 170 165<br />

2.28 Western Hydra Systems makes a panel milling machine<br />

with a 2.7-m-diameter milling head that<br />

emits low vibration and processes stress-relieved<br />

aluminum panels measuring up to 6000 mm long.<br />

The company wants to borrow money for a new<br />

production/warehouse facility. If the company offers<br />

to repay the loan with $60,000 in year 1 and<br />

amounts increasing by $10,000 each year through<br />

year 5, how much can the company borrow at an<br />

interest rate of 10% per year?<br />

2.29 GKX Industries expects sales of its hydraulic seals<br />

(in inch and metric sizes) to increase according to<br />

the cash flow sequence $70 4 k , where k is in<br />

years and cash flow is in $1000.<br />

(a) What is the amount of the cash flow in year 3?<br />

(b) What is the future worth of the entire cash<br />

flow series in year 10? Let i 10% per year.


Problems 67<br />

2.30 For the cash flows below, determine the amount in year 1, if the annual worth in years 1 through 9 is $601.17 and<br />

the interest rate is 10% per year.<br />

Year 1 2 3 4 5 6 7 8 9<br />

Cost, $1000 A A 30 A 60 A 90 A 120 A 150 A 180 A 210 A 240<br />

2.31 Apple Computer wants to have $2.1 billion<br />

available 5 years from now to finance production<br />

of a handheld “electronic brain” that, based<br />

on your behavior, will learn how to control<br />

nearly all the electronic devices in your home,<br />

such as the thermostat, coffee pot, TV, and<br />

sprinkler system. The company expects to set<br />

aside uniformly increasing amounts of money<br />

each year to meet its goal. If the amount set<br />

aside at the end of year 1 is $50 million, how<br />

much will the constant increase G have to be<br />

each year? Assume the investment account<br />

grows at a rate of 18% per year.<br />

2.32 Tacozza Electric, which manufactures brush dc<br />

servomotors, budgeted $75,000 per year to pay for<br />

certain components over the next 5 years. If the<br />

company expects to spend $15,000 in year 1, how<br />

much of a uniform (arithmetic) increase each year<br />

is the company expecting in the cost of this part?<br />

Assume the company uses an interest rate of 10%<br />

per year.<br />

Geometric Gradient<br />

2.33 There are no tables in the back of your book for the<br />

geometric gradient series factors. Calculate the first<br />

two annual worth factor values, that is, A values for<br />

n 1 and 2, that would be in a 10% interest table<br />

for a growth rate of 4% per year.<br />

2.34 Determine the present worth of a geometric gradient<br />

series with a cash flow of $50,000 in year 1 and<br />

increases of 6% each year through year 8. The interest<br />

rate is 10% per year.<br />

2.35 Determine the difference in the present worth values<br />

of the following two commodity contracts at<br />

an interest rate of 8% per year.<br />

Contract 1 has a cost of $10,000 in year 1; costs<br />

will escalate at a rate of 4% per year for 10 years.<br />

Contract 2 has the same cost in year 1, but costs<br />

will escalate at 6% per year for 11 years.<br />

2.36 El Paso Water Utilities (EPWU) purchases surface<br />

water for treatment and distribution to EPWU customers<br />

from El Paso County Water Improvement<br />

District during the irrigation season. A new contract<br />

between the two entities resulted in a reduction in<br />

future price increases in the cost of the water from<br />

8% per year to 4% per year for the next 20 years. If<br />

the cost of water next year (which is year 1 of the<br />

new contract) will be $260 per acre-foot, what is<br />

the present worth of the savings (in $/acre-ft) to the<br />

utility between the old and the new contracts? Let<br />

the interest rate equal 6% per year.<br />

2.37 Determine the present worth of a maintenance<br />

contract that has a cost of $30,000 in year 1 and<br />

annual increases of 6% per year for 10 years. Use<br />

an interest rate of 6% per year.<br />

Interest Rate and Rate of Return<br />

2.38 Gesky Industrial Products manufactures brushless<br />

blowers for boilers, food service equipment, kilns,<br />

and fuel cells. The company borrowed $18,000,000<br />

for a plant expansion and repaid the loan in seven<br />

annual payments of $3,576,420, with the first payment<br />

made 1 year after the company received the<br />

money. What was the interest rate on the loan? Use<br />

hand and spreadsheet solutions.<br />

2.39 If the value of Jane’s retirement portfolio increased<br />

from $170,000 to $813,000 over a 1 5-year<br />

period, with no deposits made to the account over<br />

that period, what annual rate of return did she<br />

make?<br />

2.40 A person’s credit score is important in determining<br />

the interest rate on a home mortgage. According to<br />

Consumer Credit Counseling Service, a homeowner<br />

with a $100,000 mortgage and a 520 credit<br />

score will pay $110,325 more in interest charges<br />

over the life of a 30-year loan than a homeowner<br />

with the same mortgage and a credit score of 720.<br />

How much higher would the interest rate per year<br />

have to be in order to account for this much difference<br />

in interest charges, if the $100,000 loan is<br />

repaid in a single lump-sum payment at the end of<br />

30 years?<br />

2.41 During a period when the real estate market in<br />

Phoenix, Arizona, was undergoing a significant<br />

downturn, CSM Consulting Engineers made an<br />

agreement with a distressed seller to purchase an<br />

office building under the following terms: total<br />

price of $1.2 million with a down payment of<br />

$200,000 now and no payments for 4 years, after<br />

which the remaining balance of $1 million would


68 Chapter 2 Factors: How Time and Interest Affect Money<br />

be paid. CSM was able to make this deal because<br />

of poor market conditions at the time of purchase,<br />

and, at the same time, planning to sell the building<br />

in 4 years (when market conditions would probably<br />

be better) and move to a larger office building<br />

in Scottsdale, Arizona. If CSM was able to sell the<br />

building in exactly 4 years for $1.9 million, what<br />

rate of return per year did the company make on<br />

the investment?<br />

2.42 A start-up company that makes hydraulic seals<br />

borrowed $800,000 to expand its packaging and<br />

shipping facility. The contract required the company<br />

to repay the investors through an innovative<br />

mechanism called faux dividends, a series of<br />

uniform annual payments over a fixed period of<br />

time. If the company paid $250,000 per year for<br />

5 years, what was the interest rate on the loan?<br />

2.43 Bessimer Electronics manufactures addressable<br />

actuators in one of its Maquiladora plants in<br />

Mexico. The company believes that by investing<br />

$24,000 each year in years 1, 2, and 3, it will avoid<br />

spending $87,360 in year 3. If the company does<br />

make the annual investments, what rate of return<br />

will it realize?<br />

2.44 UV curable epoxy resins are used in sealing, in gap<br />

filling, and as a clear coating. Your boss saw a report<br />

submitted by the chief financial officer (CFO)<br />

that said the equivalent annual worth of maintaining<br />

the equipment used in producing the resins was<br />

$48,436 over the last 5 years. The report showed<br />

that the cost in year 1 was $42,000, and it increased<br />

arithmetically by $4000 each year. Your boss<br />

thought $48,436 was too high, so she asked you to<br />

determine what interest rate the CFO used in making<br />

the calculations. What was the interest rate?<br />

Number of Years<br />

2.45 Acme Bricks, a masonry products company, wants<br />

to have $600,000 on hand before it invests in new<br />

conveyors, trucks, and other equipment. If the<br />

company sets aside $80,000 per year in an account<br />

that increases in value at a rate of 15% per year,<br />

how many years will it be before Acme can purchase<br />

the equipment?<br />

2.46 An engineer who was contemplating retirement<br />

had $1.6 million in his investment portfolio. However,<br />

a severe recession caused his portfolio to decrease<br />

to only 55% of the original amount, so he<br />

kept working. If he was able to invest his money at<br />

a rate of return of 9% per year after the recession<br />

ended, how many years did it take for his account<br />

to get back to the $1.6 million value?<br />

2.47 You own a small engineering consulting company.<br />

If you invest $200,000 of the company’s money in<br />

a natural gas well that is expected to provide income<br />

of $29,000 per year, how long must the well<br />

produce at that rate in order to get the money back<br />

plus a rate of return of 10% per year?<br />

2.48 A perceptive engineer started saving for her retirement<br />

15 years ago by diligently saving $18,000<br />

each year through the present time. She invested in<br />

a stock fund that averaged a 12% rate of return over<br />

that period. If she makes the same annual investment<br />

and gets the same rate of return in the future,<br />

how long will it be from now (time zero) before she<br />

has $1,500,000 in her retirement fund?<br />

2.49 A mechanical engineering graduate who wanted<br />

to have his own business borrowed $350,000<br />

from his father as start-up money. Because he<br />

was family, his father charged interest at only 4%<br />

per year. If the engineer was able to pay his father<br />

$15,000 in year 1, $36,700 in year 2, and amounts<br />

increasing by $21,700 each year, how many<br />

years did it take for the engineer to repay the<br />

loan?<br />

2.50 The energy costs of a company involved in powder<br />

coating of outdoor furniture are expected to increase<br />

at a rate of $400 per year. The cost at the<br />

end of the next year (year 1) is expected to be<br />

$13,000. How many years will it be from now before<br />

the equivalent annual cost is $16,000 per year,<br />

if interest is 8% per year?<br />

2.51 In cleaning out some files that were left behind by<br />

the engineer who preceded you in your current job,<br />

you found an old report that had a calculation for<br />

the present worth of certain maintenance costs for<br />

state highways. The report contained the following<br />

equation (with cost in $1 million):<br />

12{1 [(1 0.03)(1 0.06)] x } (0.06 0.03)<br />

140<br />

The value of x that was used in the calculation was<br />

illegible. What is its value?<br />

2.52 The equivalent annual worth of an increasing<br />

arithmetic gradient is $135,300. If the cash flow in<br />

year 1 is $35,000 and the gradient amount is<br />

$19,000, what is the value of n at an interest rate of<br />

10% per year?<br />

2.53 You are told that the present worth of an increasing<br />

geometric gradient is $88,146. If the cash flow in<br />

year 1 is $25,000 and the gradient increase is<br />

18% per year, what is the value of n ? The interest<br />

rate is 10% per year.


Additional Problems and FE Exam Review Questions 69<br />

ADDITIONAL PROBLEMS AND FE EXAM REVIEW QUESTIONS<br />

2.54 The amount of money that Diamond Systems can<br />

spend now for improving productivity in lieu of<br />

spending $30,000 three years from now at an interest<br />

rate of 12% per year is closest to:<br />

( a) $15,700<br />

( b) $17,800<br />

(c) $19,300<br />

(d ) $21,350<br />

2.55 A manufacturing company spent $30,000 on a new<br />

conveyor belt. If the conveyor belt resulted in cost<br />

savings of $4200 per year, the length of time it<br />

would take for the company to recover its investment<br />

at 8% per year is closest to:<br />

( a) Less than 9 years<br />

( b) 9 to 10 years<br />

( c) 11 to 12 years<br />

( d ) Over 12 years<br />

2.56 Levi Strauss has some of its jeans stone-washed<br />

under a contract with independent U.S. Garment<br />

Corp. If U.S. Garment’s operating cost per machine<br />

is $22,000 for year 1 and increases by a<br />

constant $1000 per year through year 5, what is<br />

the equivalent uniform annual cost per machine<br />

for the 5 years at an interest rate of 8% per year?<br />

( a) $23,850<br />

( b) $24,650<br />

( c) $25,930<br />

( d ) Over $26,000<br />

2.57 The FG factor values can be derived by multiplying:<br />

( a) ( PF ) and ( AG ) factor values<br />

( b) ( FP ) and ( AG ) factor values<br />

( c) ( PF ) and ( PG ) factor values<br />

( d ) ( FP ) and ( PG ) factor values<br />

2.58 At i 4% per year, A for years 1 through 6 of the<br />

cash flows shown below is closest to:<br />

( a) $300<br />

( b) $560<br />

( c) $800<br />

( d ) $1040<br />

0 1<br />

$800<br />

2<br />

$700<br />

3<br />

$600<br />

4<br />

$500<br />

5<br />

$400<br />

6 Years<br />

$300<br />

2.59 The value of the factor ( PF , i ,10) can be found by<br />

getting the factor values for (PF,i,4) and ( PF , i ,6)<br />

and:<br />

(a) Adding the values for ( PF , i ,4) and ( PF , i ,6)<br />

(b) Multiplying the values for ( PF , i ,4) and<br />

( PF , i ,6)<br />

(c) Dividing the value for ( PF , i ,6) by the value<br />

for ( PF , i ,4)<br />

( d ) None of the above<br />

2.60 A small construction company is considering the<br />

purchase of a used bulldozer for $61,000. If the<br />

company purchases the dozer now, the equivalent<br />

future amount in year 4 that the company is<br />

paying for the dozer at 4% per year interest is<br />

closest to:<br />

(a) $52,143<br />

(b) $65,461<br />

(c) $71,365<br />

(d ) Over $72,000<br />

2.61 The cost of lighting and maintaining the tallest<br />

smokestack in the United States (at a shuttered<br />

ASARCO refinery) is $90,000 per year. At an<br />

interest rate of 10% per year, the present worth<br />

of maintaining the smokestack for 10 years is<br />

closest to:<br />

(a) $1,015,000<br />

(b) $894,000<br />

(c) $712,000<br />

(d ) $553,000<br />

2.62 An enthusiastic new engineering graduate plans to<br />

start a consulting firm by borrowing $100,000 at<br />

10% per year interest. The loan payment each year<br />

to pay off the loan in 7 years is closest to:<br />

(a) $18,745<br />

(b) $20,540<br />

(c) $22,960<br />

(d ) $23,450<br />

2.63 An engineer who believed in “save now and play<br />

later” wanted to retire in 20 years with $1.5 million.<br />

At 10% per year interest, to reach the $1.5 million<br />

goal, starting 1 year from now, the engineer must<br />

annually invest:<br />

( a) $26,190<br />

( b) $28,190<br />

( c) $49,350<br />

( d ) $89,680<br />

2.64 The cost of a border fence is $3 million per<br />

mile. If the life of such a fence is assumed to<br />

be 10 years, the equivalent annual cost of a


70 Chapter 2 Factors: How Time and Interest Affect Money<br />

10-mile-long fence at an interest rate of 10%<br />

per year is closest to:<br />

(a) $3.6 million<br />

(b) $4.2 million<br />

(c) $4.9 million<br />

(d) Over $5.0 million<br />

2.65 An investment of $75,000 in equipment that<br />

will reduce the time for machining self-locking<br />

fasteners will save $20,000 per year. At an interest<br />

rate of 10% per year, the number of years<br />

required to recover the initial investment is<br />

closest to:<br />

(a) 6 years<br />

(b) 5 years<br />

(c) 4 years<br />

(d) 3 years<br />

2.66 The number of years required for an account to accumulate<br />

$650,000 if Ralph deposits $50,000 each<br />

year and the account earns interest at a rate of 6%<br />

per year is closest to:<br />

(a) 13 years<br />

(b) 12 years<br />

(c) 11 years<br />

(d) 10 years<br />

2.67 Aero Serve, Inc., manufactures cleaning nozzles<br />

for reverse-pulse jet dust collectors. The company<br />

spent $40,000 on a production control system that<br />

will increase profits by $13,400 per year for<br />

5 years. The rate of return per year on the investment<br />

is closest to:<br />

(a) 20%<br />

(b) 18%<br />

(c) 16%<br />

(d) Less than 15%<br />

2.68 Energy costs for a green chemical treatment have<br />

been increasing uniformly for 5 years. If the cost in<br />

year 1 was $26,000 and it increased by $2000 per<br />

year through year 5, the present worth of the costs<br />

at an interest rate of 10% per year is closest to:<br />

(a) $102,900<br />

(b) $112,300<br />

( c) $122,100<br />

( d) $195,800<br />

2.69 In planning for your retirement, you expect to<br />

save $5000 in year 1, $6000 in year 2, and amounts<br />

increasing by $1000 each year through year 20. If<br />

your investments earn 10% per year, the amount<br />

you will have at the end of year 20 is closest to:<br />

( a) $242,568<br />

(b) $355,407<br />

(c) $597,975<br />

(d) $659,125<br />

2.70 Income from a precious metals mining operation<br />

has been decreasing uniformly for 5 years. If income<br />

in year 1 was $300,000 and it decreased by<br />

$30,000 per year through year 4, the annual worth<br />

of the income at 10% per year is closest to:<br />

(a) $310,500<br />

(b) $258,600<br />

( c) $203,900<br />

( d) $164,800<br />

2.71 If you are able to save $5000 in year 1, $5150 in<br />

year 2, and amounts increasing by 3% each year<br />

through year 20, the amount you will have at the<br />

end of year 20 at 10% per year interest is closest to:<br />

( a) $60,810<br />

( b) $102,250<br />

(c) $351,500<br />

(d) Over $410,000<br />

CASE STUDY<br />

TIME MARCHES ON; SO DOES THE INTEREST RATE<br />

Background<br />

Information<br />

During the last week, Sundara has read about different situations<br />

that involve money, interest rate, and different amounts<br />

of time. She has gotten interested in the major effects that<br />

time and interest rates have on the amount of money necessary<br />

to do things and the significant growth in the amount of<br />

money when a large number of years are considered. In all<br />

cases, the interest focuses on the amount of money at the end<br />

of the time period.<br />

The four situations are described here.<br />

A. Manhattan Island was purchased in 1626 for $24. After<br />

385 years in 2011, at 6% per year compounded interest,<br />

the current value must be very large.<br />

B. At the age of 22, if she saved only $2000 per year for<br />

the next 10 years (starting next year) and made a return<br />

of 6% per year, by today’s standards, she would have<br />

accumulated a nice sum at the age of 70.


Case Study 71<br />

C. A corporation invested $2 million in developing and<br />

marketing a new product in 1945 (just after World<br />

War II, this was a lot of money) and has made a steady<br />

net cash flow of $300,000 per year for some 65 years.<br />

Sundara estimated the annual rate of return must be<br />

quite good, especially given that she is lucky to earn 4%<br />

per year on her own investments these days.<br />

D. A friend who is not good with money, went to a pawn<br />

shop and borrowed $200 for one week and paid $30 in<br />

interest. Sundara thought this might be a pretty good<br />

deal, in case she ever ran low on cash. However, she did<br />

not know whether the interest was simple or compounded<br />

monthly, and how much may be owed were<br />

this loan not paid off for 1 year.<br />

Case Study Exercises<br />

1. What is the annual interest rate for each situation? Include<br />

both the annual simple and the compound rates<br />

for situation D.<br />

2. Calculate and observe the total amount of money involved<br />

in each situation at the end of the time periods<br />

compared to the starting amount. Is the ending amount<br />

larger or smaller than you would expect it to be prior to<br />

making any computations?<br />

3. Think of a situation for yourself that may be similar to<br />

any of those above. Determine the interest rate, the<br />

time period, and the starting and ending amounts of<br />

money.


CHAPTER 3<br />

Combining<br />

Factors and<br />

Spreadsheet<br />

Functions<br />

LEARNING OUTCOMES<br />

Purpose: Use multiple factors and spreadsheet functions to find equivalent amounts for cash flows that have nonstandard<br />

placement.<br />

SECTION TOPIC LEARNING OUTCOME<br />

3.1 Shifted series • Determine the P , F or A values of a series<br />

starting at a time other than period 1.<br />

3.2 Shifted series and single cash<br />

flows<br />

• Determine the P , F , or A values of a shifted series<br />

and randomly placed single cash flows.<br />

3.3 Shifted gradients • Make equivalence calculations for shifted<br />

arithmetic or geometric gradient series that<br />

increase or decrease in size of cash flows.


M<br />

ost estimated cash flow series do not fit exactly the series for which the factors,<br />

equations, and spreadsheet functions in Chapter 2 were developed. For<br />

a given sequence of cash flows, there are usually several correct ways to determine<br />

the equivalent present worth P , future worth F , or annual worth A . This chapter<br />

explains how to combine engineering economy factors and spreadsheet functions to<br />

address more complex situations involving shifted uniform series, gradient series, and<br />

single cash flows.<br />

3.1 Calculations for Uniform Series That Are Shifted<br />

When a uniform series begins at a time other than at the end of period 1, it is called a shifted<br />

series. In this case several methods can be used to find the equivalent present worth P . For<br />

example, P of the uniform series shown in Figure 3–1 could be determined by any of the<br />

following methods:<br />

• Use the P F factor to find the present worth of each disbursement at year 0 and add them.<br />

• Use the F P factor to find the future worth of each disbursement in year 13, add them, and<br />

then find the present worth of the total, using P F ( P F , i ,13).<br />

• Use the F A factor to find the future amount F A ( F A , i ,10), and then compute the present<br />

worth, using P F ( P F , i ,13).<br />

• Use the P A factor to compute the “present worth” P 3 A ( P A , i ,10) (which will be located<br />

in year 3, not year 0), and then find the present worth in year 0 by using the ( P F , i ,3) factor.<br />

Typically the last method is used for calculating the present worth of a uniform series that does<br />

not begin at the end of period 1. For Figure 3–1, the “present worth” obtained using the P A factor<br />

is located in year 3. This is shown as P 3 in Figure 3–2. Note that a P value is always located<br />

1 year or period prior to the beginning of the first series amount. Why? Because the P A factor<br />

was derived with P in time period 0 and A beginning at the end of period 1. The most common<br />

mistake made in working problems of this type is improper placement of P . Therefore, it is extremely<br />

important to remember:<br />

The present worth is always located one period prior to the first uniform series amount when<br />

using the P A factor.<br />

Placement of P<br />

To determine a future worth or F value, recall that the F A factor derived in Section 2.3 had<br />

the F located in the same period as the last uniform series amount. Figure 3–3 shows the location<br />

of the future worth when F A is used for Figure 3–1 cash flows.<br />

The future worth is always located in the same period as the last uniform series amount when<br />

using the F A factor.<br />

It is also important to remember that the number of periods n in the P A or F A factor is equal<br />

to the number of uniform series values. It may be helpful to renumber the cash flow diagram to<br />

avoid errors in counting. Figures 3–2 and 3–3 show Figure 3–1 renumbered to determine n 10.<br />

Placement of F<br />

0 1 2 3 4 5 6 7 8 9 10 11 12 13 Year<br />

Figure 3–1<br />

A uniform series that is<br />

shifted.<br />

A = $50<br />

P 3 =?<br />

0 1 2 3 4 5 6 7 8 9 10 11 12 13 Year<br />

1 2 3 4 5 6 7 8 9 10 n<br />

Figure 3–2<br />

Location of present worth<br />

and renumbering for n for<br />

the shifted uniform series<br />

in Figure 3–1.<br />

A = $50


74 Chapter 3 Combining Factors and Spreadsheet Functions<br />

Figure 3–3<br />

Placement of F and<br />

renumbering for n for the<br />

shifted uniform series of<br />

Figure 3–1.<br />

F =?<br />

0 1 2 3 4 5 6 7 8 9 10 11 12 13 Year<br />

1 2 3 4 5 6 7 8 9 10 n<br />

A = $50<br />

As stated above, several methods can be used to solve problems containing a uniform series<br />

that is shifted. However, it is generally more convenient to use the uniform series factors than the<br />

single-amount factors. Specific steps should be followed to avoid errors:<br />

1. Draw a diagram of the positive and negative cash flows.<br />

2. Locate the present worth or future worth of each series on the cash flow diagram.<br />

3. Determine n for each series by renumbering the cash flow diagram.<br />

4. Draw another cash flow diagram representing the desired equivalent cash flow.<br />

5. Set up and solve the equations.<br />

These steps are illustrated below.<br />

EXAMPLE 3.1<br />

The offshore design group at Bechtel just purchased upgraded CAD software for $5000 now<br />

and annual payments of $500 per year for 6 years starting 3 years from now for annual upgrades.<br />

What is the present worth in year 0 of the payments if the interest rate is 8% per year?<br />

Solution<br />

The cash flow diagram is shown in Figure 3–4. The symbol P A is used throughout this chapter<br />

to represent the present worth of a uniform annual series A , and P ' A represents the present worth<br />

at a time other than period 0. Similarly, P T represents the total present worth at time 0. The<br />

correct placement of P ' A and the diagram renumbering to obtain n are also indicated. Note that<br />

P ' A is located in actual year 2, not year 3. Also, n 6, not 8, for the P A factor. First find the<br />

value of P ' A of the shifted series.<br />

P ' A $500( P A ,8%,6)<br />

Since P ' A is located in year 2, now find P A in year 0.<br />

P A P ' A ( P F ,8%,2)<br />

P T =?<br />

P A =?<br />

1 2<br />

P A =?<br />

i = 8% per year<br />

3 4 5 6 7 8 Year<br />

0 1 2 3 4 5 6<br />

n<br />

A = $500<br />

P 0 = $5000<br />

Figure 3–4<br />

Cash flow diagram with placement of P values, Example 3.1.<br />

The total present worth is determined by adding P A and the initial payment P 0 in year 0.<br />

P T P 0 P A<br />

5000 500( P A ,8%,6)( P F ,8%,2)<br />

5000 500(4.6229)(0.8573)<br />

$6981.60


3.1 Calculations for Uniform Series That Are Shifted 75<br />

The more complex that cash flow series become, the more useful are the spreadsheet functions.<br />

When the uniform series A is shifted, the NPV function is used to determine P , and the<br />

PMT function finds the equivalent A value. The NPV function, like the PV function, determines<br />

the P values, but NPV can handle any combination of cash flows directly from the cells. As we<br />

learned in Chapter 2, enter the net cash flows in contiguous cells (column or row), making sure<br />

to enter “0” for all zero cash flows. Use the format<br />

NPV( i %, second_cell:last_cell) fi rst_cell<br />

First_cell contains the cash flow for year 0 and must be listed separately for NPV to correctly<br />

account for the time value of money. The cash flow in year 0 may be 0.<br />

The easiest way to find an equivalent A over n years for a shifted series is with the PMT function,<br />

where the P value is from the NPV function above. The format is the same as we learned<br />

earlier; the entry for P is a cell reference, not a number.<br />

PMT( i %, n , cell_with_ P , F )<br />

Alternatively, the same technique can be used when an F value was obtained using the FV function.<br />

Now the last entry in PMT is “cell_with_ F .”<br />

It is very fortunate that any parameter in a spreadsheet function can itself be a function. Thus,<br />

it is possible to write the PMT function in a single cell by embedding the NPV function (and FV<br />

function, if needed). The format is<br />

PMT( i %, n , NPV( i %,second_cell:last_cell) first_cell, F ) [3.1]<br />

Of course, the answer for A is the same for the two-cell operation or a single-cell, embedded<br />

function. All three of these functions are illustrated in Example 3.2.<br />

EXAMPLE 3.2<br />

Recalibration of sensitive measuring devices costs $8000 per year. If the machine will be recalibrated<br />

for each of 6 years starting 3 years after purchase, calculate the 8-year equivalent<br />

uniform series at 16% per year. Show hand and spreadsheet solutions.<br />

Solution by Hand<br />

Figure 3–5 a and b shows the original cash flows and the desired equivalent diagram. To convert<br />

the $8000 shifted series to an equivalent uniform series over all periods, first convert the<br />

uniform series into a present worth or future worth amount. Then either the A P factor or the<br />

A F factor can be used. Both methods are illustrated here.<br />

Present worth method. (Refer to Figure 3–5 a .) Calculate P ' A for the shifted series in year 2,<br />

followed by P T in year 0. There are 6 years in the A series.<br />

P ' A 8000( P A ,16%,6)<br />

P T P ' A ( P F ,16%,2) 8000( P A ,16%,6)( P F ,16%,2)<br />

8000(3.6847)(0.7432) $21,907.75<br />

The equivalent series A ' for 8 years can now be determined via the A P factor.<br />

A ' P T ( A P ,16%,8) $5043.60<br />

Future worth method. (Refer to Figure 3–5 a .) First calculate the future worth F in year 8.<br />

F 8000( F A ,16%,6) $71,820<br />

The A F factor is now used to obtain A ' over all 8 years.<br />

A ' F ( A F ,16%,8) $5043.20


76 Chapter 3 Combining Factors and Spreadsheet Functions<br />

P T =? P A =?<br />

F =? i = 16% per year<br />

0 1 2 3 4 5 6 7 8<br />

0 1 2 3 4 5 6 7 8<br />

A = $8000<br />

(a)<br />

A =?<br />

(b)<br />

PMT(16%,8,B12)<br />

PMT(16%,8,NPV(16%,B4:B11) B3<br />

NPV(16%,B4:B11) B3<br />

(c)<br />

Figure 3–5<br />

( a ) Original and ( b ) equivalent cash flow diagrams; and ( c ) spreadsheet functions to determine<br />

P and A , Example 3.2.<br />

Solution by Spreadsheet<br />

(Refer to Figure 3–5 c .) Enter the cash flows in B3 through B11 with entries of “0” in the first<br />

three cells. Use the NPV function to display P $21,906.87.<br />

There are two ways to obtain the equivalent A over 8 years. Of course, only one of these<br />

PMT functions needs to be entered. (1) Enter the PMT function making direct reference to<br />

the P value (see cell tag for DE5), or (2) use Equation [3.1] to embed the NPV function into<br />

the PMT function (see cell tag for DE8).<br />

3.2 Calculations Involving Uniform Series and Randomly<br />

Placed Single Amounts<br />

When a cash flow includes both a uniform series and randomly placed single amounts, the procedures<br />

of Section 3.1 are applied to the uniform series and the single-amount formulas are applied<br />

to the one-time cash flows. This approach, illustrated in Examples 3.3 and 3.4, is merely a<br />

combination of previous ones. For spreadsheet solutions, it is necessary to enter the net cash<br />

flows before using the NPV and other functions.<br />

EXAMPLE 3.3<br />

An engineering company in Wyoming that owns 50 hectares of valuable land has decided to<br />

lease the mineral rights to a mining company. The primary objective is to obtain long-term<br />

income to finance ongoing projects 6 and 16 years from the present time. The engineering<br />

company makes a proposal to the mining company that it pay $20,000 per year for 20 years<br />

beginning 1 year from now, plus $10,000 six years from now and $15,000 sixteen years from<br />

now. If the mining company wants to pay off its lease immediately, how much should it pay<br />

now if the investment is to make 16% per year?


3.2 Calculations Involving Uniform Series and Randomly Placed Single Amounts 77<br />

Solution<br />

The cash flow diagram is shown in Figure 3–6 from the owner’s perspective. Find the present<br />

worth of the 20-year uniform series and add it to the present worth of the two one-time amounts<br />

to determine P T' .<br />

P T 20,000( P A ,16%,20) 10,000( P F ,16%,6) 15,000( P F ,16%,16)<br />

$124,075<br />

Note that the $20,000 uniform series starts at the end of year 1, so the P A factor determines the<br />

present worth at year 0.<br />

$10,000<br />

$15,000<br />

A = $20,000<br />

0 1 2 3 4 5 6 7 15 16 17 18 19 20 Year<br />

P T = ?<br />

i = 16% per year<br />

Figure 3–6<br />

Diagram including a uniform series and single amounts, Example 3.3.<br />

When you calculate the A value for a cash flow series that includes randomly placed single<br />

amounts and uniform series, first convert everything to a present worth or a future worth.<br />

Then you obtain the A value by multiplying P or F by the appropriate A P or A F factor.<br />

Example 3.4 illustrates this procedure.<br />

EXAMPLE 3.4<br />

A design-build-operate engineering company in Texas that owns a sizable amount of land<br />

plans to lease the drilling rights (oil and gas only) to a mining and exploration company. The<br />

contract calls for the mining company to pay $20,000 per year for 20 years beginning 3 years<br />

from now (i.e., beginning at the end of year 3 and continuing through year 22) plus $10,000 six<br />

years from now and $15,000 sixteen years from now. Utilize engineering economy relations by<br />

hand and by spreadsheet to determine the five equivalent values listed below at 16% per year.<br />

1. Total present worth P T in year 0<br />

2. Future worth F in year 22<br />

3. Annual series over all 22 years<br />

4. Annual series over the first 10 years<br />

5. Annual series over the last 12 years<br />

Solution by Hand<br />

Figure 3–7 presents the cash flows with equivalent P and F values indicated in the correct years<br />

for the P A , P F , and F A factors.<br />

1. P T in year 0: First determine P' A of the series in year 2. Then P T is the sum of three<br />

P values: the series present worth value moved back to t 0 with the P F factor, and the<br />

two P values at t 0 for the two single amounts in years 6 and 16.


78 Chapter 3 Combining Factors and Spreadsheet Functions<br />

P ' A 20,000( P A ,16%,20)<br />

P T P ' A ( P F ,16%,2) 10,000( P F ,16%,6) 15,000( P F ,16%,16)<br />

20,000( P A ,16%,20)( P F ,16%,2) 10,000( P F ,16%,6)<br />

15,000( P F ,16%,16)<br />

$93,625 [3.2]<br />

2. F in year 22: To determine F in year 22 from the original cash flows (Figure 3–7), find<br />

F for the 20-year series and add the two F values for the two single amounts. Be sure to<br />

carefully determine the n values for the single amounts: n 22 6 16 for the $10,000<br />

amount and n 22 16 6 for the $15,000 amount.<br />

F 20,000( F A ,16%,20) 10,000( F P ,16%,16) 15,000( F P ,16%,6) [3.3]<br />

$2,451,626<br />

3. A over 22 years: Multiply P T $93,625 from (1) above by the A P factor to determine<br />

an equivalent 22-year A series, referred to as A 1–22 here.<br />

A 1–22 P T ( A P ,16%,22) 93,625(0.16635) $15,575 [3.4]<br />

An alternate way to determine the 22-year series uses the F value from (2) above. In this<br />

case, the computation is A 1–22 F ( A F ,16%,22) $15,575.<br />

$15,000<br />

$10,000<br />

$20,000<br />

0 1 2 3 4 5 13 20 n<br />

0 1 2 3 4 5 6 7 15 16 17 18 19 20 21 22 Year<br />

P T =?<br />

P A =?<br />

Figure 3–7<br />

Diagram for Example 3.4.<br />

i = 16% per year<br />

F =?<br />

4. A over years 1 to 10: This and (5), which follows, are special cases that often occur in<br />

engineering economy studies. The equivalent A series is calculated for a number of years<br />

different from that covered by the original cash flows. This occurs when a defined study<br />

period or planning horizon is preset for the analysis. (More is mentioned about study periods<br />

later.) To determine the equivalent A series for years 1 through 10 only (call it A 1–10 ),<br />

the P T value must be used with the A P factor for n 10. This computation transforms<br />

Figure 3–7 into the equivalent series A 1–10 in Figure 3–8 a .<br />

A 1–10 P T ( A P ,16%,10) 93,625(0.20690) $19,371 [3.5]<br />

5. A over years 11 to 22: For the equivalent 12-year series for years 11 through 22 (call it<br />

A 11–22 ), the F value must be used with the A F factor for 12 years. This transforms Figure<br />

3–7 into the 12-year series A 11–22 in Figure 3–8 b .<br />

A 11–22 F ( A F ,16%,12) 2,451,626(0.03241) $79,457 [3.6]<br />

Notice the huge difference of more than $60,000 in equivalent annual amounts that occurs<br />

when the present worth of $93,625 is allowed to compound at 16% per year for the first<br />

10 years. This is another demonstration of the time value of money.<br />

Solution by Spreadsheet<br />

Figure 3–9 is a spreadsheet image with answers for all five questions. The $20,000 series and<br />

the two single amounts have been entered into separate columns, B and C. The zero cash flow


P T = $93,625 A 1–10 = ? (a)<br />

3.2 Calculations Involving Uniform Series and Randomly Placed Single Amounts 79<br />

1 2 3 4 5 6 7 8 9 10 11 21 22<br />

Year<br />

Figure 3–8<br />

Cash flows of Figure 3–7<br />

converted to equivalent<br />

uniform series for<br />

( a ) years 1 to 10 and<br />

( b ) years 11 to 22.<br />

i = 16%<br />

A 11–22 = ? Figure 3–9<br />

0 1 9 10 11 12 13 14<br />

15 16 17 18 19 20 21 22 Year<br />

i = 16%<br />

(b)<br />

F = $2,451,626<br />

Spreadsheet using cell<br />

reference format.<br />

Example 3.4.<br />

values are all entered so that the functions will work correctly. This is an excellent example<br />

demonstrating the versatility of the NPV, FV, and PMT functions. To prepare for sensitivity<br />

analysis, the functions are developed using cell reference format or global variables, as indicated<br />

in the column E function. This means that virtually any number—the interest rate, any<br />

cash flow estimate in the series or the single amounts, and the timing within the 22-year time<br />

frame—can be changed and the new answers will be immediately displayed.<br />

1. Present worth values for the series and single amounts are determined in cells F6 and F10,<br />

respectively, using the NPV function. The sum of these in F14 is P T $93,622, which<br />

corresponds to the value in Equation [3.2]. Alternatively, P T can be determined directly<br />

via the sum of two NPV functions, shown in row 15.<br />

2. The FV function in row 18 uses the P value in F14 (preceded by a minus sign) to determine<br />

F twenty-two years later. This is significantly easier than Equation [3.3].<br />

3. To find the 22-year A series starting in year 1, the PMT function in row 21 references<br />

the P value in cell F14. This is effectively the same procedure used in Equation [3.4] to<br />

obtain A 1–22 .


80 Chapter 3 Combining Factors and Spreadsheet Functions<br />

For the spreadsheet enthusiast, it is possible to find the 22-year A series value directly<br />

by using the PMT function with embedded NPV functions. The cell reference format is<br />

PMT(D1,22,−(NPV(D1,B6:B27)B5 NPV(D1,C6:C27)C5)).<br />

4. and 5. It is quite simple to determine an equivalent uniform series over any number of<br />

periods using a spreadsheet, provided the series starts one period after the P value is located<br />

or ends in the same period that the F value is located. These are both true for the<br />

series requested here. The results in F24 and F27 are the same as A 1–10 and A 11–22 in Equations<br />

[3.5] and [3.6], respectively.<br />

Comment<br />

Remember that round-off error will always be present when comparing hand and spreadsheet<br />

results. The spreadsheet functions carry more decimal places than the tables during calculations.<br />

Also, be very careful when constructing spreadsheet functions. It is easy to miss a value,<br />

such as the P or F in PMT and FV functions, or a minus sign between entries. Always check<br />

your function entries carefully before touching .<br />

3.3 Calculations for Shifted Gradients<br />

In Section 2.5, we derived the relation P G ( P G , i , n ) to determine the present worth of the<br />

arithmetic gradient series. The P G factor, Equation [2.25], was derived for a present worth in<br />

year 0 with the gradient first appearing in year 2.<br />

Placement of gradient P<br />

The present worth of an arithmetic gradient will always be located two periods before the<br />

gradient starts.<br />

Refer to Figure 2–14 as a refresher for the cash flow diagrams.<br />

The relation A G ( A G , i , n ) was also derived in Section 2.5. The A G factor in Equation<br />

[2.27] performs the equivalence transformation of a gradient only into an A series from<br />

years 1 through n (Figure 2–15). Recall that the base amount must be treated separately. Then the<br />

equivalent P or A values can be summed to obtain the equivalent total present worth P T and total<br />

annual series A T .<br />

A conventional gradient series starts between periods 1 and 2 of the cash flow sequence. A<br />

gradient starting at any other time is called a shifted gradient . The n value in the P G and A G<br />

factors for a shifted gradient is determined by renumbering the time scale. The period in which<br />

the gradient fi rst appears is labeled period 2. The n value for the gradient factor is determined by<br />

the renumbered period where the last gradient increase occurs.<br />

Partitioning a cash flow series into the arithmetic gradient series and the remainder of the cash<br />

flows can make very clear what the gradient n value should be. Example 3.5 illustrates this partitioning.<br />

EXAMPLE 3.5<br />

Fujitsu, Inc. has tracked the average inspection cost on a robotics manufacturing line for<br />

8 years. Cost averages were steady at $100 per completed unit for the first 4 years, but have<br />

increased consistently by $50 per unit for each of the last 4 years. Analyze the gradient increase,<br />

using the P G factor. Where is the present worth located for the gradient? What is the<br />

general relation used to calculate total present worth in year 0?<br />

Solution<br />

The cash flow diagram in Figure 3–10 a shows the base amount A $100 and the arithmetic<br />

gradient G $50 starting between periods 4 and 5. Figure 3–10 b and c partitions these two<br />

series. Gradient year 2 is placed in actual year 5 of the entire sequence in Figure 3–10 c . It is<br />

clear that n 5 for the P G factor. The P G ? arrow is correctly placed in gradient year 0,<br />

which is year 3 in the cash flow series.


3.3 Calculations for Shifted Gradients 81<br />

0 1 2 3 4 5 6 7 8 Year<br />

A = $100<br />

G = $50<br />

(a)<br />

$150<br />

$200<br />

$250<br />

$300<br />

0 1 2 3 4 5 6 7 8 Year<br />

P G = ?<br />

A = $100<br />

(b)<br />

0 1 2<br />

3 4 5 6 7 8 Year<br />

Figure 3–10<br />

Partitioned cash flow, ( a ) ( b ) ( c ), Example 3.5.<br />

0 1 2 3 4 5<br />

G = $50<br />

(c)<br />

The general relation for P T is taken from Equation [2.19]. The uniform series A $100<br />

occurs for all 8 years, and the G $50 gradient present worth P G appears in year 3.<br />

$50<br />

$100<br />

$150<br />

P T P A P G 100( P A , i ,8) 50( P G , i ,5)( P F , i ,3)<br />

$200<br />

Gradient<br />

n<br />

It is important to note that the A G factor cannot be used to find an equivalent A value in periods<br />

1 through n for cash flows involving a shifted gradient. Consider the cash flow diagram of<br />

Figure 3–11. To find the equivalent annual series in years 1 through 10 for the gradient series<br />

only, first find the present worth P G of the gradient in actual year 5, take this present worth back<br />

to year 0, and annualize the present worth for 10 years with the A P factor. If you apply the annual<br />

series gradient factor ( A G , i ,5) directly, the gradient is converted into an equivalent annual<br />

series over years 6 through 10 only, not years 1 through 10, as requested.<br />

To find the equivalent A series of a shifted gradient through all the n periods, first find the<br />

present worth of the gradient at actual time 0, then apply the (AP, i, n) factor.<br />

P G =?<br />

0 1 2 3 4 5 6 7 8 9 10 Year<br />

$10<br />

$10<br />

$10<br />

$10<br />

0<br />

$10<br />

Figure 3–11<br />

Determination of G and n values used in factors for a shifted gradient.<br />

1<br />

$50<br />

2<br />

$65<br />

3<br />

$80<br />

4<br />

$95<br />

5 Gradient n<br />

$110<br />

G = $15


82 Chapter 3 Combining Factors and Spreadsheet Functions<br />

EXAMPLE 3.6<br />

Set up the engineering economy relations to compute the equivalent annual series in years 1<br />

through 7 for the cash flow estimates in Figure 3–12.<br />

0 1 2 3 4 5 6 7<br />

$50 $50 $50<br />

$70<br />

Figure 3–12<br />

Diagram of a shifted gradient, Example 3.6.<br />

$90<br />

$110<br />

$130<br />

Solution<br />

The base amount annual series is A B $50 for all 7 years (Figure 3–13). Find the present worth<br />

P G in actual year 2 of the $20 gradient that starts in actual year 4. The gradient n is 5.<br />

P G 20( P G , i ,5)<br />

Bring the gradient present worth back to actual year 0.<br />

P 0 P G ( P F , i ,2) 20( P G , i ,5)( P F , i ,2)<br />

Annualize the gradient present worth from year 1 through year 7 to obtain A G .<br />

A G P 0 ( A P , i ,7)<br />

Finally, add the base amount to the gradient annual series.<br />

A T 20( P G , i ,5)( P F , i ,2)( A P , i ,7) 50<br />

For a spreadsheet solution, enter the original cash flows into adjacent cells, say, B3 through<br />

B10, and use an embedded NPV function in the PMT function. The single-cell function is<br />

PMT( i %,7,−NPV( i %, B3:B10)).<br />

P 0 =? P G =?<br />

A T =?<br />

0 1 2 3 4 5 6 7<br />

0 1 2 3 4 5<br />

Year<br />

Gradient n<br />

$50 $50 $50<br />

A B = $50<br />

$70<br />

$90<br />

G = $20<br />

$110<br />

Figure 3–13<br />

Diagram used to determine A for a shifted gradient, Example 3.6.<br />

$130<br />

In Section 2.6, we derived the relation P g A 1 ( PA,g,i,n ) to determine the present worth of a<br />

geometric gradient series, including the initial amount A 1 . The factor was derived to find the<br />

present worth in year 0, with A 1 in year 1 and the first gradient appearing in year 2.<br />

Placement of<br />

gradient P<br />

The present worth of a geometric gradient series will always be located two periods before<br />

the gradient stars, and the initial amount is included in the resulting present worth. Refer<br />

to Figure 2–21 as a refresher for the cash flows.<br />

Equation [2.35] is the formula used for the factor. It is not tabulated.


3.3 Calculations for Shifted Gradients 83<br />

EXAMPLE 3.7<br />

Chemical engineers at a Coleman Industries plant in the Midwest have determined that a<br />

small amount of a newly available chemical additive will increase the water repellency of<br />

Coleman’s tent fabric by 20%. The plant superintendent has arranged to purchase the additive<br />

through a 5-year contract at $7000 per year, starting 1 year from now. He expects<br />

the annual price to increase by 12% per year thereafter for the next 8 years. Additionally,<br />

an initial investment of $35,000 was made now to prepare a site suitable for the contractor<br />

to deliver the additive. Use i 15% per year to determine the equivalent total present<br />

worth for all these cash flows.<br />

Solution<br />

Figure 3–14 presents the cash flows. The total present worth P T is found using g 0.12 and<br />

i 0.15. Equations [2.34] and [2.35] are used to determine the present worth P g for the entire<br />

geometric series at actual year 4, which is moved to year 0 using ( P F ,15%,4).<br />

P T 35,000 A ( P A ,15%,4) A 1 ( P A ,12%,15%,9)( P F ,15%,4)<br />

35,000 7000(2.8550) [ 7000 1 (1.121.15) 9<br />

———————<br />

0.15 0.12 ] (0.5718)<br />

35,000 19,985 28,247<br />

$83,232<br />

Note that n 4 in the ( P A ,15%,4) factor because the $7000 in year 5 is the cash flow of the<br />

initial amount A 1 .<br />

For solution by spreadsheet, enter the cash flows of Figure 3–14. If cells B1 through B14<br />

are used, the function to find P $83,230 is<br />

NPV(15%,B2:B14)B1<br />

The fastest way to enter the geometric series is to enter $7000 for year 5 (into cell B6) and set<br />

up each succeeding cell multiplied by 1.12 for the 12% increase.<br />

P T =? P g =? i = 15% per year<br />

0 1 2 3 4 5 6 7 8 9 10 11 12 13 Year<br />

$7000<br />

0 1 2 3 4 5 6 7 8 9<br />

$7840<br />

Geometric<br />

gradient n<br />

$35,000<br />

12% increase<br />

per year<br />

$17,331<br />

Figure 3–14<br />

Cash flow diagram including a geometric gradient with g 12%, Example 3.7.<br />

Decreasing arithmetic and geometric gradients are common, and they are often shifted gradient<br />

series . That is, the constant gradient is – G or the percentage change is – g from one period to<br />

the next, and the first appearance of the gradient is at some time period (year) other than year 2<br />

of the series. Equivalence computations for present worth P and annual worth A are basically the<br />

same as discussed in Chapter 2, except for the following.


84 Chapter 3 Combining Factors and Spreadsheet Functions<br />

$800<br />

$700<br />

$800<br />

$600<br />

$500<br />

$400<br />

0 1 2 3 4 5<br />

0 1 2 3 4 5 6<br />

Gradient n<br />

Year<br />

P T =?<br />

(a)<br />

$800<br />

A B = $800<br />

G = $100<br />

$400<br />

$300<br />

$200<br />

$100<br />

0 1 2 3 4 5 6<br />

Year<br />

–<br />

0 1 2 3 4 5 6 Year<br />

P A =?<br />

(b)<br />

P G =?<br />

(c)<br />

Figure 3–15<br />

Partitioned cash flow of a shifted arithmetic gradient, (a) (b) (c).<br />

For shifted, decreasing gradients:<br />

• The base amount A (arithmetic) or initial amount A 1 (geometric) is the largest amount in the<br />

first year of the series.<br />

• The gradient amount is subtracted from the previous year’s amount, not added to it.<br />

• The amount used in the factors is –G for arithmetic and –g for geometric gradient series.<br />

• The present worth P G or P g is located 2 years prior to the appearance of the first gradient;<br />

however, a PF factor is necessary to find the present worth in year 0.<br />

Figure 3–15 partitions a decreasing arithmetic gradient series with G $−100 that is shifted<br />

1 year forward in time. P G occurs in actual year 1, and P T is the sum of three components.<br />

P T $800( P F , i ,1) 800( P A , i ,5)( P F , i ,1) 100( P G , i ,5)( P F , i ,1)<br />

EXAMPLE 3.8<br />

Morris Glass Company has decided to invest funds for the next 5 years so that development of<br />

“smart” glass is well funded in the future. This type of new-technology glass uses electrochrome<br />

coating to allow rapid adjustment to sun and dark in building glass, as well as assisting<br />

with internal heating and cooling cost reduction. The financial plan is to invest first, allow appreciation<br />

to occur, and then use the available funds in the future. All cash flow estimates are<br />

in $1000 units, and the interest rate expectation is 8% per year.<br />

Years 1 through 5: Invest $7000 in year 1, decreasing by $1000 per year through year 5.<br />

Years 6 through 10: No new investment and no withdrawals.<br />

Years 11 through 15: Withdraw $20,000 in year 11, decreasing 20% per year through year 15.


3.3 Calculations for Shifted Gradients 85<br />

Determine if the anticipated withdrawals will be covered by the investment and appreciation<br />

plans. If the withdrawal series is over- or underfunded, what is the exact amount available in<br />

year 11, provided all other estimates remain the same?<br />

Solution by Hand<br />

Figure 3–16 presents the cash flow diagram and the placement of the equivalent P values used<br />

in the solution. Calculate the present worth of both series in actual year 0 and add them to determine<br />

if the investment series is adequate to fund the anticipated withdrawals.<br />

P g,0 =?<br />

P g,10 =?<br />

$20,000<br />

A 1 = $20,000<br />

g = 0.20<br />

i = 8% per year<br />

$8192<br />

0 1 2 3 4 5 0 1 2 3 4 5<br />

1 2 3 4 5 6 7 8 9 10 11 12 13 14 15<br />

Gradient n<br />

Year<br />

P G =?<br />

A = $7000<br />

G = $1000<br />

Figure 3–16<br />

Investment and withdrawal series, Example 3.8.<br />

Investment series: Decreasing, conventional arithmetic series starting in year 2 with A <br />

$7000, G $1000, and gradient n 5 years. The present worth in year 0 is<br />

P G [7000(PA,8%,5) 1000(PG,8%,5)]<br />

$20,577<br />

Withdrawal series: Decreasing, shifted geometric series starting in year 12 with A 1 $20,000,<br />

g 0.20, and gradient n 5 years. If the present worth in year 10 is identified as P g,10 , the<br />

present worth in year 0 is P g,0 . Use Equation [2.35] for the (PA,20%,8%,5) factor.<br />

The net total present worth is<br />

P g,0 P g,10 (PF,i,n) A 1 (PA,g,i,n)(PF,i,n) [3.7]<br />

20,000 { 1 [——————<br />

1 (0.20)<br />

1 0.08 ] 5<br />

————————<br />

0.08 (0.20)<br />

} (0.4632)<br />

20,000(2.7750)(0.4632)<br />

$25,707<br />

P T 20,577 25,707 $5130<br />

This means that more is withdrawn than the investment series earns. Either additional funds<br />

must be invested or less must be withdrawn to make the series equivalent at 8% per year.<br />

To find the exact amount of the initial withdrawal series to result in P T 0, let A 1 be an<br />

unknown in Equation [3.7] and set P g,0 P G 20,577.<br />

20,577 A 1 (2.7750)(0.4632)<br />

A 1 $16,009 in year 11<br />

The geometric series withdrawal would be 20% less each year.


86 Chapter 3 Combining Factors and Spreadsheet Functions<br />

NPV(8%,B4:B18) + B3<br />

(a)<br />

(b)<br />

Figure 3–17<br />

Spreadsheet solution, Example 3.8. (a) Cash flows and NPV function and (b) Goal Seek to determine initial withdrawal<br />

amount in year 11.<br />

Solution by Spreadsheet<br />

See Figure 3–17a. To determine if the investment series will cover the withdrawal series, enter<br />

the cash flows (in column B using the functions shown in column C) and apply the NPV function<br />

shown in the cell tag to display P T $5130 directly. As above, the sign indicates that,<br />

from a time value of money perspective, there is more withdrawn than invested and earned.<br />

The Goal Seek tool is very handy in determining the initial withdrawal amount that results<br />

in P T 0 (cell B19). Figure 3–17b shows the template and result A 1 $16,009 in year 11.<br />

Each succeeding withdrawal is 80% of the previous one.<br />

Comment<br />

If the withdrawal series is fixed as estimated initially and the investment series base amount A<br />

can be increased, the Goal Seek tool can be used to, again, set P T 0 (cell B19). However, now<br />

establish the entry in B4 as the changing cell. The response is A $8285, and, as before,<br />

succeeding investments are $1000 less.<br />

CHAPTER SUMMARY<br />

In Chapter 2, we derived the equations to calculate the present, future, or annual worth of specific<br />

cash flow series. In this chapter, we learned how to use these equations on cash flow series that<br />

are shifted in time from those for which the basic relations are derived. For example, when a<br />

uniform series does not begin in period 1, we still use the P A factor to find the “present worth”<br />

of the series, except the P value is located one period ahead of the first A value, not at time 0. For<br />

arithmetic and geometric gradients, the P value is two periods ahead of where the gradient starts.<br />

With this information, it is possible to solve for P , A , or F for any conceivable cash flow series.<br />

We have used the power of spreadsheet functions in determining P , F , and A values by singlecell<br />

entries and using cash flow estimates entered into a series of spreadsheet cells. Though<br />

spreadsheet solutions are fast, they do remove some of the understanding of how the time value<br />

of money and the factors change the equivalent value of money.<br />

PROBLEMS<br />

Present Worth Calculations<br />

3.1 Industrial Electric Services has a contract with the<br />

U.S. Embassy in Mexico to provide maintenance<br />

for scanners and other devices in the building. If<br />

the first payment of $12,000 is received now ,<br />

what is the present worth of the contract, provided<br />

the company will receive a total of 10 payments<br />

(i.e., years 0 through 9) and the interest rate is<br />

10% per year?


Problems 87<br />

3.2 Civil engineering consulting firms that provide services<br />

to outlying communities are vulnerable to a<br />

number of factors that affect the financial condition<br />

of the communities, such as bond issues and real<br />

estate developments. A small consulting firm entered<br />

into a fixed-price contract with a large developer,<br />

resulting in a stable income of $260,000 per<br />

year in years 1 through 3. At the end of that time, a<br />

mild recession slowed the development, so the parties<br />

signed another contract for $190,000 per year<br />

for 2 more years. Determine the present worth of the<br />

two contracts at an interest rate of 10% per year.<br />

3.3 The net cash flow associated with development<br />

and sale of a new product is shown. Determine the<br />

present worth at an interest rate of 12% per year.<br />

The cash flow is in $1000 units. Show ( a ) hand and<br />

( b ) spreadsheet solutions.<br />

Year 1 2 3 4 5 6 7 8 9<br />

Cash<br />

Flow, $ −120 −100 −40 50 50 80 80 80 80<br />

0<br />

3.4 Standby power for water utility pumps and other<br />

electrical devices is provided by diesel-powered<br />

generators. As an alternative, the utility can use<br />

natural gas to power the generators, but it will be a<br />

few years before the gas is available at remote<br />

sites. The utility estimates that by switching to gas,<br />

it will save $22,000 per year, starting 3 years from<br />

now. At an interest rate of 8% per year, determine<br />

the present worth in year 0 of the projected savings<br />

that will occur in years 3 through 10.<br />

3.5 Find the present worth at i 10% per year for the<br />

cash flow series shown below.<br />

1<br />

i = 10% per year<br />

2 3 4 5 6 7 8<br />

$200 $200<br />

$200<br />

$90 $90 $90<br />

Year<br />

3.6 The subsidized cost of producing water at the El<br />

Paso Water Utilities (EPWU) Kay Bailey Hutchison<br />

(KBH) desalting plant is $1.56 per 1000 gallons.<br />

Under a contract that EPWU has with Fort Bliss,<br />

the utility sells water to the Army post at a discounted<br />

price of $1.28 per 1000 gallons. (The<br />

KBH plant was built on Army property.) If Fort<br />

Bliss uses 2 billion gallons of water each year,<br />

what is the present worth of the discount for a 20-<br />

year period at an interest rate of 6% per year?<br />

3.7 The rising cost of athletic programs at major universities<br />

has induced college presidents and athletic<br />

directors to devise innovative ways to finance cashstrapped<br />

sports programs. One of the latest schemes<br />

for big-time athletics is the “sports mortgage.” At<br />

the University of Kansas, Jayhawk fans can sign up<br />

to pay $105,000 now, or over a 10-year period, for<br />

the right to buy top seats for football games during<br />

the next 30 years. In return, the seats themselves<br />

will stay locked in at current-year prices. Season<br />

tickets in tier 1 are currently selling for $350 each.<br />

A fan plans to purchase the sports mortgage along<br />

with a current-season ticket and pay for both now ,<br />

then buy a ticket each year for the next 30 years.<br />

What is the total present worth of the pricing plan at<br />

an interest rate of 10% per year?<br />

3.8 The cash flow associated with making self-locking<br />

fasteners is shown below. Determine the net present<br />

worth (year 0) at an interest rate of 10% per<br />

year.<br />

Year 0 1 2 3 4 5 6 7 8 9<br />

Income, $1000 20 20 20 20 30 30 30 30 30 30<br />

Cost, $1000 8 8 8 8 12 12 12 12 12 25<br />

3.9 Beckman Technologies, a relatively small manufacturer<br />

of precision laboratory equipment, borrowed<br />

$2 million to renovate one of its testing<br />

labs. In an effort to pay off the loan quickly, the<br />

company made four payments in years 1 through<br />

4, with each payment being twice as large as the<br />

preceding one. At an interest rate of 10% per year,<br />

what was the size of the first payment?<br />

Annual Worth Calculations<br />

3.10 Revenue from the sale of ergonomic hand tools<br />

was $300,000 in years 1 through 4 and $465,000 in<br />

years 5 through 9. Determine the equivalent annual<br />

revenue in years 1 through 9 at an interest rate<br />

of 10% per year.<br />

3.11 Two engineering graduates who recently got married<br />

are planning for their early retirement 20 years<br />

from now. They believe that they will need<br />

$2,000,000 in year 20. Their plan is to live on one<br />

of their salaries and invest the other. They already<br />

have $25,000 in their investment account. ( a ) How<br />

much will they have to invest each year if the account<br />

grows at a rate of 10% per year? ( b ) If the<br />

maximum they have available to invest each year<br />

is $40,000, will they reach their goal of $2 million<br />

by year 20?<br />

3.12 Costs associated with the manufacture of miniature<br />

high-sensitivity piezoresistive pressure transducers<br />

are $73,000 per year. A clever industrial<br />

engineer found that by spending $16,000 now to<br />

reconfigure the production line and reprogram


88 Chapter 3 Combining Factors and Spreadsheet Functions<br />

two of the robotic arms, the cost will go down to<br />

$58,000 next year and $52,000 in years 2 through 5.<br />

Using an interest rate of 10% per year, determine<br />

( a ) the equivalent annual cost of the manufacturing<br />

operations and (b) the equivalent annual savings in<br />

years 1 through 5.<br />

3.13 Calculate the equivalent annual cost in years 1<br />

through 9 of the following series of disbursements.<br />

Use an interest rate of 10% per year. Show ( a )<br />

hand and ( b ) spreadsheet solutions.<br />

Year Disbursement, $ Year Disbursement, $<br />

0 8000 5 4000<br />

1 4000 6 5000<br />

2 4000 7 5000<br />

3 4000 8 5000<br />

4 4000 9 5000<br />

3.14 For the cash flows below, find the value of x that<br />

makes the equivalent annual worth in years 1<br />

through 7 equal to $300 per year. Use an interest<br />

rate of 10% per year. Show solutions ( a ) by hand<br />

and ( b ) using the Goal Seek tool.<br />

Year Cash Flow, $ Year Cash Flow, $<br />

0 200 4 x<br />

1 200 5 200<br />

2 200 6 200<br />

3 200 7 200<br />

3.15 Precision Instruments, Inc. manufactures highsensitivity<br />

mini accelerometers designed for<br />

modal analysis testing. The company borrowed<br />

$10,000,000 with the understanding that it would<br />

make a $2,000,000 payment at the end of year 1<br />

and then make equal annual payments in years 2<br />

through 5 to pay off the loan. If the interest rate on<br />

the loan was 9% per year, how much was each<br />

payment in years 2 through 5?<br />

3.16 A construction management company is examining<br />

its cash flow requirements for the next 7 years. The<br />

company expects to replace office machines and<br />

computer equipment at various times over the<br />

7-year planning period. Specifically, the company<br />

expects to spend $6000 one year from now, $9000<br />

three years from now, and $10,000 six years from<br />

now. What is the annual worth (in years 1 through 7)<br />

of the planned expenditures, at an interest rate of<br />

10% per year?<br />

3.17 Find the equivalent annual worth for the cash<br />

flows shown, using an interest rate of 12% per<br />

year. Monetary units are $1000.<br />

Year 1 2 3 4 5 6 7 8 9<br />

Cash Flow, $ 20 20 20 20 60 60 60 60 60<br />

3.18 Calculate the net annual worth in years 1 through<br />

10 of the following series of incomes and expenses,<br />

if the interest rate is 10% per year.<br />

Year Income, $Year Expense, $Year<br />

0 0 2500<br />

1–4 700 200<br />

5–10 2000 300<br />

3.19 The city of El Paso gave the El Paso Tennis and<br />

Swim Club a 20-year lease to continue using a<br />

10-acre arroyo park for its facilities. The club<br />

will pay $1000 per year plus make $350,000 in<br />

improvements at the park. In addition, the club<br />

will open its tennis courts to the public from 1<br />

to 5 P.M. Monday through Thursday. If the club<br />

makes $100,000 worth of improvements now<br />

and then $50,000 worth each year for the next<br />

5 years, what is the equivalent annual cost of<br />

the lease to the club at an interest rate of 10%<br />

per year?<br />

3.20 Stadium Capital Financing Group is a Chicago<br />

company that conceived the so-called sports<br />

mortgage wherein fans agree to pay a relatively<br />

large amount of money over a 10- to 30-year<br />

period for the right to buy top seats for football<br />

games for up to the next 50 years. In return,<br />

season ticket prices stay locked in at currentyear<br />

prices, and the package can be sold in the<br />

secondary market, while taking a tax write-off<br />

for donating to a school. Assume a fan buys a<br />

sports mortgage at West Virginia University for<br />

$150,000 that is to be paid over a 10-year period<br />

with the right to buy two season tickets for<br />

$300 each for the next 30 years. The first<br />

payment is made now (i.e., beginning-of-year<br />

payment), and an additional nine payments are<br />

to be made at the end of each year for the next<br />

9 years. Assume the fan also purchases the two<br />

season tickets (also beginning-of-year payments).<br />

What is the total amount of the payment<br />

each year in years 0 through 9 ? Use an interest<br />

rate of 10% per year.<br />

Future Worth Calculations<br />

3.21 The expansion plans of Acme Granite, Stone &<br />

Brick call for the company to add capacity for a<br />

new product in 5 years. The company wants to<br />

have $360,000 available before it announces the<br />

product. If the company sets aside $55,000 now<br />

and $90,000 in year 2, what uniform annual<br />

amount will it have to put in an account in years 3<br />

through 5 to have the $360,000? Assume the account<br />

earns interest at 8% per year.


Problems 89<br />

3.22 For the cash flows shown, calculate the future<br />

worth in year 8 using i 10% per year.<br />

0<br />

1<br />

i = 10% per year<br />

2 3 4 5 6 7 8<br />

Year<br />

Year 0 1 2 3 4 5 6<br />

Cash Flow, $ 100 100 100 100 100 300 300<br />

3.23 Assume you plan to start an annuity plan by making<br />

your first deposit now . If you make annual<br />

deposits of a uniform amount A into the account<br />

that earns interest at a rate of 7% per year, how<br />

many years from now will it be until the value of<br />

the account is equal to 10 times the value of a<br />

single deposit?<br />

3.24 New actuator element technology enables engineers<br />

to simulate complex computer-controlled<br />

movements in any direction. If the technology results<br />

in cost savings in the design of amusement<br />

park rides, what is the future worth in year 5 of savings<br />

of $70,000 now and $20,000 per year in years<br />

1 through 3 at an interest rate of 10% per year?<br />

3.25 Austin Utilities is planning to install solar panels<br />

to provide some of the electricity for its groundwater<br />

desalting plant. The project would be done in<br />

two phases. The first phase will cost $4 million in<br />

year 1 and $5 million in year 2. This investment<br />

will result in energy savings (phase 2) of $540,000<br />

in year 3, $546,000 in year 4, and amounts increasing<br />

by $6000 each year through year 10. Let i <br />

10% per year.<br />

( a) What is the future worth of the savings ?<br />

( b) Is the cost of the solar project justified by the<br />

savings? (Hint: Calculate the difference between<br />

savings and cost).<br />

3.26 For the cash flow diagram shown, determine the<br />

value of W that will render the equivalent future<br />

worth in year 8 equal to $500 at an interest rate of<br />

10% per year.<br />

0<br />

1<br />

i = 10% per year<br />

2 3 4 5 6 7 8<br />

Year<br />

x<br />

x<br />

x<br />

x<br />

Random Single Amounts and Uniform Series<br />

3.28 A small oil company is planning to replace its<br />

Coriolis flowmeters with Emerson Hastelloy flowmeters.<br />

The replacement process will cost the<br />

company $50,000 three years from now. How<br />

much money must the company set aside each year<br />

beginning now (year 0) in order to have the total<br />

amount available immediately after making the<br />

last deposit at the end of year 3? Assume the company<br />

can invest its funds at 15% per year.<br />

3.29 A company that manufactures air-operated drain<br />

valve assemblies budgeted $74,000 per year to pay<br />

for plastic components over a 5-year period. If the<br />

company spent only $42,000 in year 1, what uniform<br />

annual amount should the company expect to<br />

spend in each of the next 4 years to expend the<br />

entire budget? Assume the company uses an interest<br />

rate of 10% per year.<br />

3.30 A recently hired chief executive officer wants to<br />

reduce future production costs to improve the company’s<br />

earnings, thereby increasing the value of the<br />

company’s stock. The plan is to invest $40,000<br />

now and $40,000 in each of the next 2 years to improve<br />

productivity. By how much must annual<br />

costs decrease in years 3 through 7 to recover the<br />

investment plus a return of 12% per year?<br />

3.31 Use the cash flow diagram to determine the single<br />

amount of money Q 4 in year 4 that is equivalent to<br />

all of the cash flows shown. Use i 10% per year.<br />

Q 4 =?<br />

x<br />

2x<br />

2x<br />

2x<br />

i = 10% per year<br />

$40 $40 $40 $40 $40<br />

$40 $40<br />

1<br />

0<br />

1<br />

2 3 4 5 6 7 8<br />

Year<br />

W<br />

3.27 For the cash flows shown in the diagram, determine<br />

the value of x that will make the future worth<br />

in year 8 equal to $70,000.<br />

$25 $25 $25 $25 $25<br />

$25<br />

$25<br />

$50<br />

$50 $50


90 Chapter 3 Combining Factors and Spreadsheet Functions<br />

3.32 For the following series of income and expenses,<br />

find the equivalent value in year 9 at an interest rate<br />

of 12% per year. Show ( a ) hand and ( b ) spreadsheet<br />

solutions.<br />

Years Income, $ Expense, $<br />

0 0 70,000<br />

1–6 9,000 13,000<br />

7–9 28,000 14,000<br />

10–16 38,000 19,000<br />

3.33 An investor just purchased property under a unique<br />

financing agreement with the seller. The contract<br />

price is $1.6 million. The payment plan is Z dollars<br />

now, 2 Z dollars in year 2, and 3 Z dollars in years 3<br />

through 5. If the interest rate on the transaction is<br />

10% per year, how much is the payment in year 2?<br />

3.34 A foursome of entrepreneurial electrical engineering<br />

graduates has a plan to start a new solar power<br />

equipment company based on STE (solar thermal<br />

electric) technology. They have recently approached<br />

a group of investors with their idea and asked for a<br />

loan of $5 million. Within the agreement, the loan is<br />

to be repaid by allocating 80% of the company’s<br />

profits each year for the first 4 years to the investors.<br />

In the fifth year, the company will pay the balance<br />

remaining on the loan in cash. The company’s business<br />

plan indicates that they expect to make no<br />

profit for the first year, but in years 2 through 4, they<br />

anticipate profits to be $1.5 million per year. If the<br />

investors accept the deal at an interest rate of 15%<br />

per year, and the business plan works to perfection,<br />

what is the expected amount of the last loan payment<br />

(in year 5 )?<br />

Shifted Gradients<br />

3.35 Find the present worth in year 0 for the cash flows<br />

shown. Let i 10% per year.<br />

0<br />

1<br />

$50<br />

$50<br />

i = 10% per year<br />

2 3 4 5 6 7 8<br />

$70<br />

$90<br />

$170<br />

$130<br />

$150<br />

$170<br />

Year<br />

3.36 For the cash flows shown, determine the present<br />

worth in year 0, if the interest rate is 12% per year.<br />

Year 1 2 3 4 5 6 7 8 9 10<br />

Cash Flow, $ 13 13 13 13 16 19 22 25 28 31<br />

3.37 A low-cost noncontact temperature measuring tool<br />

may be able to identify railroad car wheels that are in<br />

need of repair long before a costly structural failure<br />

occurs. If BNF Railroad saves $100,000 in years 1<br />

through 5, $110,000 in year 6, and constant amounts<br />

increasing by $10,000 each year through year 20,<br />

what is the equivalent annual worth over the 20 years<br />

of the savings? The interest rate is 10% per year.<br />

3.38 Determine the present worth in year 0 of the cash<br />

flows shown at an interest rate of 15% per year.<br />

Year 1 2 3 4 5 6 7 8 9 10<br />

Cash Flow, $ 90 90 90 85 80 75 70 65 60 55<br />

3.39 The Pedernales Electric Cooperative estimates that<br />

the present worth now of income from an investment<br />

in renewable energy sources is $12,475,000.<br />

There will be no income in years 1 and 2, but in<br />

year 3 income will be $250,000, and thereafter it<br />

will increase according to an arithmetic gradient<br />

through year 15. What is the required gradient, if<br />

the interest rate is 15% per year? Solve ( a ) by hand<br />

and ( b ) using a spreadsheet.<br />

3.40 Calculate the annual cost in years 1 through 9 of<br />

the following series of disbursements. Use an interest<br />

rate of 10% per year.<br />

Year Disbursement, $ Year Disbursement, $<br />

0 5000 5 7500<br />

1 5500 6 8000<br />

2 6000 7 8500<br />

3 6500 8 9000<br />

4 7000 9 9500<br />

3.41 The cost associated with manufacturing highperformance<br />

lubricants closely follows the cost of<br />

crude oil. For the last 10 years, a small, independent<br />

refiner had a cost of $3.4 million in<br />

years 1 through 3, after which the cost increased by<br />

3% per year through this year. Determine the current<br />

equivalent worth (i.e., now ) of the manufacturing<br />

cost, using an interest rate of 10% per year.<br />

Show both ( a ) hand and ( b ) spreadsheet solutions.<br />

3.42 Find the future worth in year 10 of $50,000 in<br />

year 0 and amounts increasing by 15% per year<br />

through year 10 at an interest rate of 10% per year.<br />

3.43 The cost of tuition at public universities has been<br />

steadily increasing for many years. One Midwestern<br />

university pledged to keep the tuition constant for


Problems 91<br />

4 years for all students who finished in the top 3%<br />

of their class. One such student who liked research<br />

planned to enroll at the university and continue<br />

there until earning a PhD degree (a total time of<br />

9 years). If the tuition for the first 4 years will be<br />

$7200 per year and it increases by 5% per year for<br />

the next 5 years, what is the present worth of the<br />

tuition cost at an interest rate of 8% per year?<br />

3.44 A private equity firm purchased a cable company<br />

and assumed the company’s debt as part of the<br />

transaction. The deal was structured such that the<br />

private equity firm received $3 million immediately<br />

after the deal was closed (in year 0) through<br />

the sale of some assets. This year (year 1) income<br />

was $3.36 million, and it is projected to increase<br />

by 12% each year through expansion of the customer<br />

base. What was the present worth in the year<br />

of purchase of the income stream over a 10-year<br />

period? The firm’s expected rate of return for any<br />

purchase is 15% per year.<br />

3.48 The pumping cost for delivering water from the<br />

Ohio River to Wheeling Steel for cooling hotrolled<br />

steel was $1.8 million for the first 3 years.<br />

An effective energy conservation program resulted<br />

in a reduced cost to $1.77 million in year 4,<br />

$1.74 million in year 5, and amounts decreasing by<br />

$30,000 each year through year 9. What is the<br />

equivalent annual worth of the pumping costs over<br />

the 9 years at an interest rate of 12% per year?<br />

3.49 Income from the mining of mineral deposits usually<br />

decreases as the resource becomes more difficult<br />

to extract. Determine the future worth in<br />

year 10 of a mineral lease that yielded income of<br />

$14,000 in years 1 through 4 and then amounts<br />

that decreased by 5% per year through year 10.<br />

Use an interest rate of 18% per year. Show ( a ) hand<br />

and ( b ) spreadsheet solutions.<br />

3.50 For the cash flows shown in the diagram, determine<br />

the future worth in year 8 at an interest rate of<br />

10% per year.<br />

i = 10% per year<br />

3.45 Calculate the present worth in year 0 of a series of<br />

cash flows that starts at $150,000 in year 0 and increases<br />

by 10% per year through year 5. Assume<br />

i 10% per year.<br />

0<br />

1<br />

2 3 4 5 6 7 8<br />

Year<br />

Shifted Decreasing Gradients<br />

3.46 For the cash flows shown, determine the value of<br />

G such that the present worth in year 0 equals<br />

$16,000 at an interest rate of 10% per year.<br />

Year 0 1 2 3 4 5<br />

Cash<br />

0 8000 8000 8000 G 80002 G 80003 G<br />

Flow, $<br />

3.47 For these cash flows, find the equivalent annual<br />

worth in years 1 through 7 at an interest rate of<br />

10% per year.<br />

Year Cash Flow, $ Year Cash Flow, $<br />

0 850 4 650<br />

1 800 5 600<br />

2 750 6 550<br />

3 700 7 500<br />

$370<br />

$320<br />

$270<br />

$220<br />

$420<br />

$470 $470<br />

3.51 Income from the sale of application software (apps)<br />

is usually constant for several years and then decreases<br />

quite rapidly as the market gets close to<br />

saturation. Income from one smart phone app was<br />

$38,000 in years 1 through 3 and then decreased<br />

geometrically by 15% per year through year 7. Determine<br />

the equivalent annual income in years 1<br />

through 7, using an interest rate of 10% per year.<br />

3.52 Determine the future worth in year 10 of a cash<br />

flow series that starts in year 0 at $100,000 and<br />

decreases by 12% per year. Use an interest rate of<br />

12% per year.


92 Chapter 3 Combining Factors and Spreadsheet Functions<br />

ADDITIONAL PROBLEMS AND FE EXAM REVIEW QUESTIONS<br />

3.53 A manufacturer of toilet flush valves wants to have<br />

$1,900,000 available 3 years from now so that a<br />

new product line can be initiated. If the company<br />

plans to deposit money each year, starting now , the<br />

equation that represents the deposit each year at<br />

8% per year interest is:<br />

(a) 1,900,000( AF ,8%,3)<br />

(b) 1,900,000( AF ,8%,4)<br />

(c) 1,900,000 1,900,000( AF ,8%,3)<br />

(d) 1,900,000 1,900,000( AF ,8%,2)<br />

3.54 The present worth in year 0 of a lease that requires<br />

a payment of $9000 now and amounts increasing<br />

by 5% per year through year 10 at 8% per year<br />

interest is closest to:<br />

( a ) $73,652<br />

( b ) $79,939<br />

( c ) $86,335<br />

( d ) Over $87,000<br />

3.55 For the diagram shown, the respective values of n<br />

for the following equation are:<br />

P 0 100( PA ,10%, n )( PF ,10%, n )<br />

( a ) 6 and 1<br />

( b ) 6 and 2<br />

( c ) 7 and 1<br />

( d ) 7 and 2<br />

P 0 =?<br />

0<br />

1<br />

i = 10% per year<br />

2 3 4 5 6 7<br />

Year<br />

3.58 Cindy wants to deposit money for 4 consecutive<br />

years starting 3 years from now so she can withdraw<br />

$50,000 twelve years from now. Assume the interest<br />

rate is 8% per year. The annual deposit is closest to:<br />

( a ) $6990<br />

( b ) $7670<br />

( c ) $8530<br />

( d ) $10,490<br />

3.59 The net present worth in year 0 of the following<br />

series of incomes and expenses at 8% per year is<br />

closest to:<br />

Years Income, $ Expenses, $<br />

0 12,000 1000<br />

1–6 700 100<br />

7–11 900 200<br />

( a ) $14,300 ( b ) $15,500<br />

( c ) $16,100 ( d ) $16,500<br />

3.60 For the cash flows shown, the equivalent annual<br />

worth in periods 1 through 5 at an interest rate of<br />

10% per year is closest to:<br />

Annual Period Amount, $<br />

0 1000<br />

1 1000<br />

2 1000<br />

3 1000<br />

4 1000<br />

5 1500<br />

( a ) $1120 ( b ) $1240<br />

( c ) $1350 ( d ) $1490<br />

$100 $100 $100 $100 $100 $100<br />

3.56 Summit Metals is planning to expand its Wichita,<br />

Kansas, manufacturing operation 5 years from now<br />

at a cost of $10,000,000. If the company plans to deposit<br />

money into an account each year for 4 years<br />

beginning 2 years from now (first deposit is in year 2)<br />

to pay for the expansion, the equation that represents<br />

the amount of the deposit at 9% per year interest is:<br />

(a) A 10,000,000( AF ,9%,5)<br />

(b) A 10,000,000( AF ,9%,4)<br />

(c) A 10,000,000( AP ,9%,4)<br />

(d) A 10,000,000( AF ,9%,4)( PF ,9%,1)<br />

3.57 The amount of money a person must deposit<br />

3 years from now in order to withdraw $10,000 per<br />

year for 10 years beginning 15 years from now at<br />

an interest rate of 10% per year is closest to:<br />

( a ) $15,500 ( b ) $17,200<br />

( c ) $19,300 ( d ) $21,500<br />

3.61 For the cash flows shown, the value of X that will<br />

make the present worth in year 0 equal to $5000 at<br />

an interest rate of 10% per year is closest to:<br />

Year 0 1 2 3 4 5 6 7 8 9<br />

Cash 200 300 400 500 600 700 800 900 1000 X<br />

Flow, $<br />

( a ) $2895 ( b ) $3125<br />

( c ) $3305 ( d ) $3765<br />

3.62 In order to have cash available for unforeseen<br />

emergencies, Baring Systems, a military contractor,<br />

wants to have $2,000,000 in a contingency<br />

fund 4 years from now. The amount the company<br />

must deposit each year in years 0 through 4 at an<br />

interest rate of 10% per year is closest to:<br />

( a ) $420,100 ( b ) $327,600<br />

( c ) $284,600 ( d ) $206,900


Case Study 93<br />

CASE STUDY<br />

PRESERVING LAND FOR PUBLIC USE<br />

Background and Information<br />

The Trust for Public Land (TPL) is a national organization<br />

that purchases and oversees the improvement of large land<br />

sites for government agencies at all levels. Its mission is to<br />

ensure the preservation of the natural resources, while providing<br />

necessary, but minimal, development for recreational<br />

use by the public. All TPL projects are evaluated at 7% per<br />

year, and TPL reserve funds earn 7% per year.<br />

A southern U.S. state, which has long-term groundwater<br />

problems, has asked the TPL to manage the purchase of<br />

10,000 acres of aquifer recharge land and the development of<br />

three parks of different use types on the land. The 10,000<br />

acres will be acquired in increments over the next 5 years<br />

with $4 million expended immediately on purchases. Total<br />

annual purchase amounts are expected to decrease 25% each<br />

year through year 5 and then cease for this particular project.<br />

A city with 1.5 million citizens immediately to the southeast<br />

of this acreage relies heavily on the aquifer’s water. Its<br />

citizens passed a bond issue last year, and the city government<br />

now has available $3 million for the purchase of land.<br />

The bond interest rate is an effective 7% per year.<br />

The engineers working on the park plan intend to complete<br />

all the development over a 3-year period starting in year<br />

4, when the amount budgeted is $550,000. Increases in construction<br />

costs are expected to be $100,000 each year through<br />

year 6.<br />

At a recent meeting, the following agreements were made:<br />

• Purchase the initial land increment now. Use the bond<br />

issue funds to assist with this purchase. Take the remaining<br />

amount from TPL reserves.<br />

• Raise the remaining project funds over the next 2 years in<br />

equal annual amounts.<br />

• Evaluate a financing alternative (suggested informally by<br />

one individual at the meeting) in which the TPL provides<br />

all funds, except the $3 million available now, until the<br />

parks development is initiated in year 4.<br />

Case Study Exercises<br />

1. For each of the 2 years, what is the equivalent annual<br />

amount necessary to supply the remaining project<br />

funds?<br />

2. If the TPL did agree to fund all costs except the $3 million<br />

bond proceeds now available, determine the equivalent<br />

annual amount that must be raised in years 4<br />

through 6 to supply all remaining project funds. Assume<br />

the TPL will not charge any extra interest over the 7% to<br />

the state or city on the borrowed funds.<br />

3. Review the TPL website (www.tpl.org). Identify some<br />

economic and noneconomic factors that you believe<br />

must be considered when the TPL is deciding to purchase<br />

land to protect it from real estate development.


CHAPTER 4<br />

Nominal and<br />

Effective<br />

Interest Rates<br />

LEARNING OUTCOMES<br />

Purpose: Make computations for interest rates and cash flows that are on a time basis other than a year.<br />

SECTION TOPIC LEARNING OUTCOME<br />

4.1 Statements • Understand interest rate statements that include<br />

nominal and effective rates.<br />

4.2 Effective annual rate • Derive and use the formula for an effective<br />

annual interest rate.<br />

4.3 Effective rate • Determine the effective interest rate for any<br />

stated time period.<br />

4.4 Payment period and<br />

compounding period<br />

• Determine the payment period (PP) and<br />

compounding period (CP) for equivalence<br />

computations.<br />

4.5 Single cash flows with PP CP • Perform equivalence calculations for singleamount<br />

cash flows and PP CP.<br />

4.6 Series cash flows with PP CP • Perform equivalence calculations for series and<br />

gradient cash flows and PP CP.<br />

4.7 Single amounts and series with<br />

PP CP<br />

• Perform equivalence calculations for cash flows<br />

with PP CP.<br />

4.8 Continuous compounding • Derive and use the effective interest rate formula<br />

for interest rates that are compounded<br />

continuously.<br />

4.9 Varying rates • Perform equivalency calculations for interest rates<br />

that vary from one time period to another.


I<br />

n all engineering economy relations developed thus far, the interest rate has been<br />

a constant, annual value. For a substantial percentage of the projects evaluated by<br />

professional engineers in practice, the interest rate is compounded more frequently<br />

than once a year; frequencies such as semiannually, quarterly, and monthly are common. In fact,<br />

weekly, daily, and even continuous compounding may be experienced in some project evaluations.<br />

Also, in our own personal lives, many of our financial considerations—loans of all types<br />

(home mortgages, credit cards, automobiles, boats), checking and savings accounts, investments,<br />

stock option plans, etc.—have interest rates compounded for a time period shorter than<br />

1 year. This requires the introduction of two new terms— nominal and effective interest rates.<br />

This chapter explains how to understand and use nominal and effective interest rates<br />

in engineering practice and in daily life situations. Equivalence calculations for any compounding<br />

frequency in combination with any cash flow frequency are presented.<br />

PE<br />

The Credit Card Offer Case: Today, Dave<br />

received a special offer of a new credit<br />

card from Chase Bank linked with the<br />

major airline that he flies frequently.<br />

It offers a generous bonus package<br />

for signing up by a specific date about<br />

60 days from now. The bonus package<br />

includes extra airline points (once the<br />

first purchase is made), priority airport<br />

check-in services (for 1 year), several<br />

free checked-bag allowances (for up<br />

to 10 check-ins), extra frequent-flyer<br />

points on the airline, access to airline<br />

lounges (provided he uses the card<br />

on a set time basis), plus several other<br />

rewards (rental car discounts, cruise trip<br />

amenities, and floral order discounts).<br />

The annual fee of $85 for membership<br />

does not start until the second year,<br />

and balance transfers from other credit<br />

cards have a low transfer fee, provided<br />

they are made at the time of initial<br />

membership.<br />

Dave has a credit card currently with<br />

a bank that he is planning to leave due<br />

to its poor customer service and high<br />

monthly fees. If he enrolls, he will transfer<br />

the $1000 balance on the current<br />

card to the new Chase Bank card.<br />

In the page that accompanies the<br />

offer letter, “pricing information” is<br />

included. This includes interest rates,<br />

interest charges, and fees. A summary of<br />

several of these rates and fees follows.<br />

APR (annual percentage rate) for purchases<br />

and balance transfers*<br />

APR for cash and overdraft advances*<br />

Penalty APR for late minimum payment,<br />

exceeding credit limit, and<br />

returned unpaid payments*†<br />

14.24% per year (sum of the current U.S.<br />

Government prime rate of 3.25% and<br />

10.99%, which is the APR added to determine<br />

the balance transfer APR for Chase Bank)<br />

19.24% per year<br />

29.99% per year (maximum penalty APR)<br />

Fees are listed as follows:<br />

Annual membership<br />

$85; free the first year<br />

Balance transfers<br />

$5 or 3% of each transfer, whichever is greater<br />

Cash advances<br />

$10 or 3% of each advance, whichever is greater<br />

Late payment $39 each occurrence, if balance exceeds $250<br />

Over the credit limit<br />

$39 each occurrence<br />

Returned check or payment $39 each occurrence<br />

*All APR rates are variable, based on a current 3.25% prime rate with 10.99% added to<br />

determine purchasebalance transfer APR; with 15.99% added to determine cashoverdraft<br />

APR; and with 26.99% added to determine penalty APR.<br />

†The penalty APR applies indefinitely to future transactions. If no minimum payment is received<br />

within 60 days, the penalty APR applies to all outstanding balances and all future transactions<br />

on the account.<br />

(Continued)


96 Chapter 4 Nominal and Effective Interest Rates<br />

This case is used in the following topics<br />

(and sections) of this chapter:<br />

Nominal and effective interest rate<br />

statements (4.1)<br />

Effective annual interest rates (4.2)<br />

Equivalence relations: Series with<br />

PP CP (4.6)<br />

4.1 Nominal and Effective Interest Rate Statements<br />

In Chapter 1, we learned that the primary difference between simple interest and compound<br />

interest is that compound interest includes interest on the interest earned in the previous period,<br />

while simple interest does not. Here we discuss nominal and effective interest rates, which<br />

have the same basic relationship. The difference here is that the concepts of nominal and effective<br />

must be used when interest is compounded more than once each year. For example, if an<br />

interest rate is expressed as 1% per month, the terms nominal and effective interest rates must<br />

be considered.<br />

To understand and correctly handle effective interest rates is important in engineering practice<br />

as well as for individual finances. The interest amounts for loans, mortgages, bonds, and stocks<br />

are commonly based upon interest rates compounded more frequently than annually. The engineering<br />

economy study must account for these effects. In our own personal finances, we manage<br />

most cash disbursements and receipts on a nonannual time basis. Again, the effect of compounding<br />

more frequently than once per year is present. First, consider a nominal interest rate.<br />

Nominal interest rate r<br />

A nominal interest rate r is an interest rate that does not account for compounding. By definition,<br />

r interest rate per time period number of periods [4.1]<br />

A nominal rate may be calculated for any time period longer than the time period stated by using<br />

Equation [4.1]. For example, the interest rate of 1.5% per month is the same as each of the following<br />

nominal rates.<br />

Time Period<br />

Nominal Rate by<br />

Equation [4.1]<br />

What This Is<br />

24 months 1.5 24 36% Nominal rate per 2 years<br />

12 months 1.5 12 18% Nominal rate per 1 year<br />

6 months 1.5 6 9% Nominal rate per 6 months<br />

3 months 1.5 3 4.5% Nominal rate per 3 months<br />

Note that none of these rates mention anything about compounding of interest; they are all of the<br />

form “ r % per time period.” These nominal rates are calculated in the same way that simple rates<br />

are calculated using Equation [1.7], that is, interest rate times number of periods.<br />

After the nominal rate has been calculated, the compounding period (CP) must be included<br />

in the interest rate statement. As an illustration, again consider the nominal rate of 1.5%<br />

per month. If we define the CP as 1 month, the nominal rate statement is 18% per year, compounded<br />

monthly, or 4.5% per quarter, compounded monthly . Now we can consider an effective<br />

interest rate.<br />

Effective interest rate i<br />

An effective interest rate i is a rate wherein the compounding of interest is taken into<br />

account. Effective rates are commonly expressed on an annual basis as an effective annual<br />

rate; however, any time basis may be used.<br />

The most common form of interest rate statement when compounding occurs over time periods<br />

shorter than 1 year is “% per time period, compounded CP-ly,” for example, 10% per year, compounded<br />

monthly, or 12% per year, compounded weekly. An effective rate may not always<br />

include the compounding period in the statement. If the CP is not mentioned, it is understood to


4.1 Nominal and Effective Interest Rate Statements 97<br />

be the same as the time period mentioned with the interest rate. For example, an interest rate of<br />

“1.5% per month” means that interest is compounded each month; that is, CP is 1 month. An<br />

equivalent effective rate statement, therefore, is 1.5% per month, compounded monthly.<br />

All of the following are effective interest rate statements because either they state they are<br />

effective or the compounding period is not mentioned. In the latter case, the CP is the same as<br />

the time period of the interest rate.<br />

Statement CP What This Is<br />

i 10% per year CP not stated; CP year Effective rate per year<br />

i effective 10% per year, CP stated; CP month Effective rate per year<br />

compounded monthly<br />

i 1<br />

1_ % per month<br />

2<br />

CP not stated; CP month Effective rate per month<br />

i effective 1 1_ % per month,<br />

2<br />

compounded monthly<br />

CP stated; CP month Effective rate per month; terms effective<br />

and compounded monthly are redundant<br />

i effective 3% per quarter,<br />

compounded daily<br />

CP stated; CP day Effective rate per quarter<br />

All nominal interest rates can be converted to effective rates. The formula to do this is discussed<br />

in the next section.<br />

All interest formulas, factors, tabulated values, and spreadsheet functions must use an effective<br />

interest rate to properly account for the time value of money.<br />

The term APR (Annual Percentage Rate) is often stated as the annual interest rate for credit<br />

cards, loans, and house mortgages. This is the same as the nominal rate . An APR of 15% is the<br />

same as a nominal 15% per year or a nominal 1.25% on a monthly basis. Also the term APY<br />

(Annual Percentage Yield) is a commonly stated annual rate of return for investments, certificates<br />

of deposit, and saving accounts. This is the same as an effective rate . The names are different,<br />

but the interpretations are identical. As we will learn in the following sections, the effective<br />

rate is always greater than or equal to the nominal rate, and similarly APY APR.<br />

Based on these descriptions, there are always three time-based units associated with an interest<br />

rate statement.<br />

Interest period ( t )—The period of time over which the interest is expressed. This is the t in<br />

the statement of r % per time period t , for example, 1% per month. The time unit of 1 year is<br />

by far the most common. It is assumed when not stated otherwise.<br />

Compounding period (CP)—The shortest time unit over which interest is charged or earned.<br />

This is defined by the compounding term in the interest rate statement, for example, 8% per<br />

year, compounded monthly. If CP is not stated, it is assumed to be the same as the interest<br />

period.<br />

Compounding frequency (m)—The number of times that compounding occurs within the<br />

interest period t. If the compounding period CP and the time period t are the same, the compounding<br />

frequency is 1, for example, 1% per month, compounded monthly.<br />

Consider the (nominal) rate of 8% per year, compounded monthly. It has an interest period t of<br />

1 year, a compounding period CP of 1 month, and a compounding frequency m of 12 times per<br />

year. A rate of 6% per year, compounded weekly, has t 1 year, CP 1 week, and m 52, based<br />

on the standard of 52 weeks per year.<br />

In previous chapters, all interest rates had t and CP values of 1 year, so the compounding frequency<br />

was always m 1. This made them all effective rates, because the interest period and<br />

compounding period were the same. Now, it will be necessary to express a nominal rate as an<br />

effective rate on the same time base as the compounding period.<br />

An effective rate can be determined from a nominal rate by using the relation<br />

Effective rate per CP <br />

r % per time period t<br />

—————————————<br />

m compounding periods per t r —<br />

m [4.2]<br />

As an illustration, assume r 9% per year, compounded monthly; then m 12. Equation<br />

[4.2] is used to obtain the effective rate of 9%12 0.75% per month, compounded monthly.


98 Chapter 4 Nominal and Effective Interest Rates<br />

Note that changing the interest period t does not alter the compounding period, which is 1 month<br />

in this illustration. Therefore, r 9% per year, compounded monthly, and r 4.5% per 6 months,<br />

compounded monthly, are two expression of the same interest rate.<br />

EXAMPLE 4.1<br />

Three different bank loan rates for electric generation equipment are listed below. Determine<br />

the effective rate on the basis of the compounding period for each rate.<br />

(a) 9% per year, compounded quarterly.<br />

(b) 9% per year, compounded monthly.<br />

(c) 4.5% per 6 months, compounded weekly.<br />

Solution<br />

Apply Equation [4.2] to determine the effective rate per CP for different compounding periods.<br />

The graphic in Figure 4–1 indicates the effective rate per CP and how the interest rate is distributed<br />

over time.<br />

Effective<br />

Nominal Compounding Compounding Rate per<br />

r% per t Period (CP) Frequency (m) CP ( —<br />

m r )<br />

Distribution over Time Period t<br />

(a) 9% per Quarter 4 2.25%<br />

year<br />

2.25%<br />

2.25% 2.25% 2.25%<br />

1 2 3 4<br />

Quarter<br />

(b) 9% per Month 12 0.75%<br />

year<br />

.75%<br />

.75%<br />

.75%<br />

.75%<br />

.75%<br />

.75%<br />

.75%<br />

.75%<br />

.75%<br />

.75%<br />

.75%<br />

.75%<br />

1 2 3 4 5 6 7 8 9 10 11 12<br />

0.173%<br />

Month<br />

(c) 4.5% per Week 26 0.173%<br />

6 months<br />

1 12 14 16<br />

26<br />

Week<br />

Figure 4–1<br />

Relations between interest period t, compounding period CP, and effective interest rate per CP.<br />

Sometimes it is not obvious whether a stated rate is a nominal or an effective rate. Basically<br />

there are three ways to express interest rates, as detailed in Table 4–1. The right column includes<br />

a statement about the effective rate. For the first format, a nominal interest rate is given and the<br />

compounding period is stated. The effective rate must be calculated (discussed in the next sections).<br />

In the second format, the stated rate is identified as effective (or APY could also be used),<br />

so the rate is used directly in computations.<br />

In the third format, no compounding period is identified, for example, 8% per year. This rate<br />

is effective over a compounding period equal to the stated interest period of 1 year in this case.<br />

The effective rate for any other time period must be calculated.<br />

TABLE 4–1<br />

Various Ways to Express Nominal and Effective Interest Rates<br />

Format of Rate Statement Example of Statement What about the Effective Rate?<br />

Nominal rate stated,<br />

compounding period stated<br />

Effective rate stated<br />

Interest rate stated, no<br />

compounding period stated<br />

8% per year, compounded<br />

quarterly<br />

Effective 8.243% per year,<br />

compounded quarterly<br />

Find effective rate for any time<br />

period (next two sections)<br />

Use effective rate of 8.243% per year<br />

directly for annual cash flows<br />

8% per year Rate is effective for CP equal to stated<br />

interest period of 1 year; find effective<br />

rate for all other time periods


4.2 Effective Annual Interest Rates 99<br />

EXAMPLE 4.2 The Credit Card Offer Case<br />

As described in the introduction to this case, Dave has been offered what is described as a credit<br />

card deal that should not be refused—at least that is what the Chase Bank offer letter implies. The<br />

balance transfer APR interest rate of 14.24% is an annual rate, with no compounding period mentioned.<br />

Therefore, it follows the format of the third entry in Table 4–1, that is, interest rate stated,<br />

no CP stated. Therefore, we should conclude that the CP is 1 year, the same as the annual interest<br />

period of the APR. However, as Dave and we all know, credit card payments are required monthly.<br />

(a) First, determine the effective interest rates for compounding periods of 1 year and<br />

1 month so Dave knows some effective rates he might be paying when he transfers the<br />

$1000 balance from his current card.<br />

(b) Second, assume that immediately after he accepts the card and completes the $1000 transfer,<br />

Dave gets a bill that is due 1 month later. What is the amount of the total balance he owes?<br />

PE<br />

Now, Dave looks a little closer at the fine print of the “pricing information” sheet and discovers<br />

a small-print statement that Chase Bank uses the daily balance method (including new transactions)<br />

to determine the balance used to calculate the interest due at payment time.<br />

(c) We will reserve the implication of this new finding until later, but for now help Dave by<br />

determining the effective daily interest rate that may be used to calculate interest due at<br />

the end of 1 month, provided the CP is 1 day.<br />

Solution<br />

(a) The interest period is 1 year. Apply Equation [4.2] for both CP values of 1 year ( m 1<br />

compounding period per year) and 1 month ( m 12 compounding periods per year).<br />

CP of year: Effective rate per year 14.241 14.24%<br />

CP of month: Effective rate per month 14.2412 1.187%<br />

(b) The interest will be at the monthly effective rate, plus the balance transfer fee of 3%.<br />

Amount owed after 1 month 1000 1000(0.01187) 0.03(1000)<br />

1000 11.87 30<br />

$1041.87<br />

Including the $30 fee, this represents an interest rate of (41.871000)(100%) 4.187%<br />

for only the 1-month period.<br />

(c) Again apply Equation [4.2], now with m 365 compounding periods per year.<br />

CP of day: Effective rate per day 14.24365 0.039%<br />

4.2 Effective Annual Interest Rates<br />

In this section, effective annual interest rates are calculated. Therefore, the year is used as the interest<br />

period t , and the compounding period CP can be any time unit less than 1 year. For example,<br />

we will learn that a nominal 18% per year, compounded quarterly is the same as an effective rate of<br />

19.252% per year.<br />

The symbols used for nominal and effective interest rates are<br />

r nominal interest rate per year<br />

CP time period for each compounding<br />

m number of compounding periods per year<br />

i effective interest rate per compounding period rm<br />

i a effective interest rate per year<br />

The relation i rm is exactly the same as Equation [4.2].


100 Chapter 4 Nominal and Effective Interest Rates<br />

Figure 4–2<br />

Future worth calculation<br />

at a rate i , compounded m<br />

times in a year.<br />

P(1 + i) m = P(1 + i a )<br />

P(1 + i) m –1<br />

P(1 + i) m –2<br />

P(1 + i) 3<br />

P(1 + i) 2 Future worth amounts<br />

P(1 + i)<br />

P<br />

1 2 3 m – 2 m – 1 m Number of CPs<br />

i i i i i i<br />

Effective i per<br />

compounding period<br />

1 2 3 m – 2 m – 1 m<br />

Compounding period<br />

As mentioned earlier, treatment for nominal and effective interest rates parallels that of<br />

simple and compound interest. Like compound interest, an effective interest rate at any point<br />

during the year includes (compounds) the interest rate for all previous compounding periods<br />

during the year. Therefore, the derivation of an effective interest rate formula directly parallels<br />

the logic used to develop the future worth relation F P (1 i ) n . We set P $1 for<br />

simplification.<br />

The future worth F at the end of 1 year is the principal P plus the interest P ( i ) through the<br />

year. Since interest may be compounded several times during the year, use the effective annual<br />

rate symbol i a to write the relation for F with P $1.<br />

F P Pi a 1 (1 i a )<br />

Now consider Figure 4–2. The effective rate i per CP must be compounded through all m periods<br />

to obtain the total effect of compounding by the end of the year. This means that F can also be<br />

written as<br />

F 1 (1 i )<br />

m<br />

Equate the two expressions for F and solve for i a . The effective annual interest rate formula<br />

for i a is<br />

i a (1 i )<br />

m<br />

1 [4.3]<br />

Equation [4.3] calculates the effective annual interest rate i a for any number of compounding<br />

periods per year when i is the rate for one compounding period.<br />

If the effective annual rate i a and compounding frequency m are known, Equation [4.3] can be<br />

solved for i to determine the effective interest rate per compounding period.<br />

i (1 i a ) 1 m<br />

1 [4.4]<br />

As an illustration, Table 4–2 utilizes the nominal rate of 18% per year for different compounding<br />

periods (year to week) to determine the effective annual interest rate. In each case, the effective<br />

rate i per CP is applied m times during the year. Table 4–3 summarizes the effective annual<br />

rate for frequently quoted nominal rates using Equation [4.3]. A standard of 52 weeks and<br />

365 days per year is used throughout. The values in the continuous-compounding column are<br />

discussed in Section 4.8.


TABLE 4–2 Effective Annual Interest Rates Using Equation [4.3]<br />

r 18% per year, compounded CP-ly<br />

Compounding<br />

Period, CP<br />

Times<br />

Compounded<br />

per Year, m<br />

Rate per<br />

Compound<br />

Period, i%<br />

Distribution of i over the Year of<br />

Compounding Periods<br />

Effective Annual<br />

Rate, i a (1 i ) m 1<br />

Year 1 18 18%<br />

(1.18) 1 1 18%<br />

1<br />

6 months 2 9 9%<br />

9%<br />

(1.09) 2 1 18.81%<br />

1 2<br />

Quarter 4 4.5 4.5% 4.5% 4.5% 4.5% (1.045) 4 1 19.252%<br />

1 2 3 4<br />

1.5% in each<br />

Month 12 1.5 (1.015) 12 1 19.562%<br />

1<br />

2 3 4 5 6 7 8 9 10 11 12<br />

0.34615% in each<br />

Week 52 0.34615 (1.0034615) 52 1 19.684%<br />

1 2 3 24 26 28 50 52<br />

101


102 Chapter 4 Nominal and Effective Interest Rates<br />

TABLE 4–3<br />

Nominal<br />

Rate r %<br />

Effective Annual Interest Rates for Selected Nominal Rates<br />

Semiannually<br />

(m 2)<br />

Quarterly<br />

(m 4)<br />

Monthly<br />

(m 12)<br />

Weekly<br />

(m 52)<br />

Daily<br />

(m 365)<br />

Continuously<br />

(m ; e r 1)<br />

0.25 0.250 0.250 0.250 0.250 0.250 0.250<br />

0.50 0.501 0.501 0.501 0.501 0.501 0.501<br />

1.00 1.003 1.004 1.005 1.005 1.005 1.005<br />

1.50 1.506 1.508 1.510 1.511 1.511 1.511<br />

2 2.010 2.015 2.018 2.020 2.020 2.020<br />

3 3.023 3.034 3.042 3.044 3.045 3.046<br />

4 4.040 4.060 4.074 4.079 4.081 4.081<br />

5 5.063 5.095 5.116 5.124 5.126 5.127<br />

6 6.090 6.136 6.168 6.180 6.180 6.184<br />

7 7.123 7.186 7.229 7.246 7.247 7.251<br />

8 8.160 8.243 8.300 8.322 8.328 8.329<br />

9 9.203 9.308 9.381 9.409 9.417 9.417<br />

10 10.250 10.381 10.471 10.506 10.516 10.517<br />

12 12.360 12.551 12.683 12.734 12.745 12.750<br />

15 15.563 15.865 16.076 16.158 16.177 16.183<br />

18 18.810 19.252 19.562 19.684 19.714 19.722<br />

20 21.000 21.551 21.939 22.093 22.132 22.140<br />

25 26.563 27.443 28.073 28.325 28.390 28.403<br />

30 32.250 33.547 34.489 34.869 34.968 34.986<br />

40 44.000 46.410 48.213 48.954 49.150 49.182<br />

50 56.250 60.181 63.209 64.479 64.816 64.872<br />

EXAMPLE 4.3<br />

Janice is an engineer with Southwest Airlines. She purchased Southwest stock for $6.90 per share<br />

and sold it exactly 1 year later for $13.14 per share. She was very pleased with her investment<br />

earnings. Help Janice understand exactly what she earned in terms of ( a ) effective annual rate and<br />

( b ) effective rate for quarterly compounding, and for monthly compounding. Neglect any commission<br />

fees for purchase and selling of stock and any quarterly dividends paid to stockholders.<br />

Solution<br />

(a) The effective annual rate of return i a has a compounding period of 1 year, since the<br />

stock purchase and sales dates are exactly 1 year apart. Based on the purchase price of<br />

$6.90 per share and using the definition of interest rate in Equation [1.2],<br />

amount of increase per 1 year<br />

i a ———————————— 100% —— 6.24 100% 90.43% per year<br />

original price<br />

6.90<br />

(b) For the effective annual rates of 90.43% per year, compounded quarterly, and 90.43%,<br />

compounded monthly, apply Equation [4.4] to find corresponding effective rates on the<br />

basis of each compounding period.<br />

Quarter: m 4 times per year i (1.9043) 14 1 1.17472 1 0.17472<br />

This is 17.472% per quarter, compounded quarterly.<br />

Month: m 12 times per year i (1.9043) 112 1 1.05514 1 0.05514<br />

This is 5.514% per month, compounded monthly.<br />

Comment<br />

Note that these quarterly and monthly rates are less than the effective annual rate divided by<br />

the number of quarters or months per year. In the case of months, this would be 90.43%12 <br />

7.54% per month. This computation is incorrect because it neglects the fact that compounding<br />

takes place 12 times during the year to result in the effective annual rate of 90.43%.<br />

The spreadsheet function that displays the result of Equation [4.3], that is, the effective annual<br />

rate i a , is the EFFECT function. The format is


4.2 Effective Annual Interest Rates 103<br />

EFFECT(nominal_rate_per_year, compounding_frequency)<br />

EFFECT( r %, m ) [4.5]<br />

Note that the rate entered in the EFFECT function is the nominal annual rate r% per year , not<br />

the effective rate i% per compounding period. The function automatically finds i for use in Equation<br />

[4.3]. As an example, assume the nominal annual rate is r 5.25% per year, compounded<br />

quarterly, and you want to find the effective annual rate i a . The correct input is EFFECT(5.25%,4)<br />

to display i a 5.354% per year. This is the spreadsheet equivalent of Equation [4.3] with<br />

i 5.254 1.3125% per quarter with m 4.<br />

i a (1 0.013125) 4 1 0.05354 (5.354%)<br />

The thing to remember about using the EFFECT function is that the nominal rate r entered<br />

must be expressed over the same period of time as that of the effective rate required, which is<br />

1 year here.<br />

The NOMINAL spreadsheet function finds the nominal annual rate r . The format is<br />

NOMINAL(effective_rate, compounding_frequency_per_year)<br />

NOMINAL( i a %, m ) [4.6]<br />

This function is designed to display only nominal annual rates. Accordingly, the m entered must<br />

be the number of times interest is compounded per year. For example, if the effective annual rate<br />

is 10.381% per year, compounded quarterly, and the nominal annual rate is requested, the function<br />

is NOMINAL(10.381%,4) to display r 10% per year, compounded quarterly. The<br />

nominal rates for shorter time periods than 1 year are determined by using Equation [4.1]. For<br />

example, the quarterly rate is 10%4 2.5%.<br />

The things to remember when using the NOMINAL function are that the answer is always a<br />

nominal annual rate, the rate entered must be an effective annual rate, and the m must equal the<br />

number of times interest is compounded annually.<br />

EXAMPLE 4.4 The Credit Card Offer Case<br />

In our Progressive Example, Dave is planning to accept the offer for a Chase Bank credit card<br />

that carries an APR (nominal rate) of 14.24% per year, or 1.187% per month. He will transfer<br />

a balance of $1000 and plans to pay it and the transfer fee of $30, due at the end of the first<br />

month. Let’s assume that Dave makes the transfer, and only days later his employer has a<br />

1-year assignment for him in the country of the Cameroon in northwestern Africa. Dave accepts<br />

the employment offer, and in his hurried, excited departure, he forgets to send the credit<br />

card service company a change of address. Since he is now out of mail touch, he does not pay<br />

his monthly balance due, which we calculated in Example 4.2 to be $1041.87.<br />

(a) If this situation continues for a total of 12 months, determine the total due after 12 months<br />

and the effective annual rate of interest Dave has accumulated. Remember, the fine print on<br />

the card’s interest and fee information states a penalty APR of 29.99% per year after one late<br />

payment of the minimum payment amount, plus a late payment fee of $39 per occurrence.<br />

(b) If there were no penalty APR and no late-payment fee, what effective annual interest rate<br />

would be charged for this year? Compare this rate with the answer in part ( a ).<br />

PE<br />

Solution<br />

(a) Because Dave did not pay the first month’s amount, the new balance of $1041.87 and all<br />

future monthly balances will accumulate interest at the higher monthly rate of<br />

29.99%12 2.499% per month<br />

Additionally, the $39 late-payment fee will be added each month, starting with the second<br />

month, and interest will be charged on these fees also each month thereafter. The first<br />

3 months and last 2 months are detailed below. Figure 4–3 details the interest and fees for<br />

all 12 months.


104 Chapter 4 Nominal and Effective Interest Rates<br />

SUM(B6:D6)<br />

J4/12<br />

K4/12<br />

Interest rate per month<br />

Month 1: 1.187%<br />

Months 2-12: penalty 2.499%<br />

Month 1: transfer fee<br />

Months 2-12: late-payment fee<br />

Figure 4–3<br />

Monthly amounts due for a credit card, Example 4.4.<br />

Month 1: 1000 1000(0.01187) 30 $1041.87<br />

Month 2: 1041.87 1041.87(0.02499) 39 $1106.91<br />

Month 3: 1106.91 1106.91(0.02499) 39 $1173.57<br />

. . .<br />

Month 11: 1689.25 1689.25(0.02499) 39 $1770.46<br />

Month 12: 1770.46 1770.46(0.02499) 39 $1853.71<br />

The effective monthly rate is determined by using the FP factor to find the i value at<br />

which $1000 now is equivalent to $1853.71 after 12 periods.<br />

1853.71 1000( FP, i ,12) 1000(1 i ) 12<br />

1 i (1.85371 ) 112 1.05278<br />

i 5.278% per month<br />

Since the compounding period is 1 month, use Equation [4.3] to determine the effective<br />

annual rate of 85.375% per year, compounded monthly.<br />

i a (1 i ) m 1 (1.05278 ) 12 1<br />

0.85375 (85.375%)<br />

(b) If there were no penalty fees for late payments and the nominal annual rate of 14.24% (or<br />

1.187% per month) were applied throughout the 12 months, the effective annual rate<br />

would be 15.207% per year, compounded monthly. By Equation [4.3], with a small<br />

rounding error included,<br />

i a (1 i ) m 1 (1.01187) 12 1 0.15207<br />

First, Dave will not pay at the stated rate of 14.24%, because this is the APR (nominal<br />

rate), not the APY (effective rate) of 15.207%. Second, and much more important, is the<br />

huge difference made by (1) the increase in rate to an APR of 29.99% and (2) the monthly<br />

fees of $39 for not making a payment. These large fees become part of the credit balance<br />

and accumulate interest at the penalty rate of 29.99% per year. The result is an effective<br />

annual rate jump from 15.207% to 85.375% per year, compounded monthly.<br />

Comment<br />

This is but one illustration of why the best advice to an individual or company in debt is to<br />

spend down the debt. The quoted APR by credit card, loan, and mortgage institutions can be<br />

quite deceiving; plus, the addition of penalty fees increases the effective rate very rapidly.


4.3 Effective Interest Rates for Any Time Period 105<br />

When Equation [4.3] is applied to find i a the result is usually not an integer. Therefore, the<br />

engineering economy factor cannot be obtained directly from the interest factor tables. There are<br />

alternative ways to find the factor value.<br />

• Use the factor formula with the ia rate substituted for i .<br />

• Use the spreadsheet function with i a (as discussed in Section 2.4).<br />

• Linearly interpolate between two tabulated rates (as discussed in Section 2.4).<br />

4.3 Effective Interest Rates for Any Time Period<br />

Equation [4.3] in Section 4.2 calculates an effective interest rate per year from any effective rate<br />

over a shorter time period. We can generalize this equation to determine the effective interest<br />

rate for any time period (shorter or longer than 1 year).<br />

Effective i per time period ( 1 r —<br />

m ) m 1 [4.7]<br />

where i effective rate for specified time period (say, semiannual)<br />

r nominal interest rate for same time period (semiannual)<br />

m number of times interest is compounded per stated time period (times per<br />

6 months)<br />

The term rm is always the effective interest rate over a compounding period CP, and m is<br />

always the number of times that interest is compounded per the time period on the left of the<br />

equals sign in Equation [4.7]. Instead of i a , this general expression uses i as the symbol for the<br />

effective interest rate, which conforms to the use of i throughout the remainder of this text.<br />

Examples 4.5 and 4.6 illustrate the use of this equation.<br />

EXAMPLE 4.5<br />

Tesla Motors manufactures high-performance battery electric vehicles. An engineer is on a<br />

Tesla committee to evaluate bids for new-generation coordinate-measuring machinery to be<br />

directly linked to the automated manufacturing of high-precision vehicle components. Three<br />

bids include the interest rates that vendors will charge on unpaid balances. To get a clear understanding<br />

of finance costs, Tesla management asked the engineer to determine the effective<br />

semiannual and annual interest rates for each bid. The bids are as follows:<br />

Bid 1:<br />

Bid 2:<br />

Bid 3:<br />

9% per year, compounded quarterly<br />

3% per quarter, compounded quarterly<br />

8.8% per year, compounded monthly<br />

(a) Determine the effective rate for each bid on the basis of semiannual periods.<br />

(b) What are the effective annual rates? These are to be a part of the final bid selection.<br />

(c) Which bid has the lowest effective annual rate?<br />

Solution<br />

(a) Convert the nominal rates to a semiannual basis, determine m, then use Equation [4.7] to<br />

calculate the effective semiannual interest rate i. For bid 1,<br />

r 9% per year 4.5% per 6 months<br />

m 2 quarters per 6 months<br />

Effective i% per 6 months ( 1 0.045 ———<br />

2 ) 2 1 1.0455 1 4.55%<br />

Table 4–4 (left section) summarizes the effective semiannual rates for all three bids.


106 Chapter 4 Nominal and Effective Interest Rates<br />

TABLE 4–4 Effective Semiannual and Annual Interest Rates for Three Bid Rates, Example 4.5<br />

Bid<br />

Nominal r per<br />

6 Months, %<br />

Semiannual Rates<br />

CP per<br />

6 Months, m<br />

Equation [4.7],<br />

Effective i, %<br />

Nominal r<br />

per Year, %<br />

Annual Rates<br />

CP per<br />

Year, m<br />

Equation [4.7],<br />

Effective i, %<br />

1 4.5 2 4.55 9 4 9.31<br />

2 6.0 2 6.09 12 4 12.55<br />

3 4.4 6 4.48 8.8 12 9.16<br />

(b) For the effective annual rate, the time basis in Equation [4.7] is 1 year. For bid 1,<br />

r 9% per year<br />

m 4 quarters per year<br />

Effective i% per year ( 1 0.09 ——<br />

4 ) 4 − 1 1.0931 1 9.31%<br />

The right section of Table 4–4 includes a summary of the effective annual rates.<br />

(c) Bid 3 includes the lowest effective annual rate of 9.16%, which is equivalent to an effective<br />

semiannual rate of 4.48% when interest is compounded monthly.<br />

EXAMPLE 4.6<br />

A dot-com company plans to place money in a new venture capital fund that currently returns<br />

18% per year, compounded daily. What effective rate is this ( a ) yearly and ( b ) semiannually?<br />

Solution<br />

(a) Use Equation [4.7], with r 0.18 and m 365.<br />

Effective i % per year ( 1 —— 0.18<br />

365 ) 365 1 19.716%<br />

(b) Here r 0.09 per 6 months and m 182 days.<br />

Effective i % per 6 months ( 1 —— 0.09<br />

182 ) 182 1 9.415%<br />

4.4 Equivalence Relations: Payment Period<br />

and Compounding Period<br />

Now that the procedures and formulas for determining effective interest rates with consideration<br />

of the compounding period are developed, it is necessary to consider the payment period .<br />

The payment period (PP) is the length of time between cash flows (inflows or outflows). It is<br />

common that the lengths of the payment period and the compounding period (CP) do not coincide.<br />

It is important to determine if PP CP, PP CP, or PP CP.<br />

If a company deposits money each month into an account that earns at the nominal rate of 8%<br />

per year, compounded semiannually, the cash flow deposits define a payment period of 1 month<br />

and the nominal interest rate defines a compounding period of 6 months. These time periods are<br />

shown in Figure 4–4. Similarly, if a person deposits a bonus check once a year into an account<br />

that compounds interest quarterly, PP 1 year and CP 3 months.<br />

r = nominal 8% per year, compounded semiannually<br />

CP<br />

6 months<br />

CP<br />

6 months<br />

Figure 4–4<br />

One-year cash flow diagram<br />

for a monthly payment<br />

period (PP) and<br />

semiannual compounding<br />

period (CP).<br />

0 1 2 3 4 5 6 7 8 9 10 11 12<br />

PP<br />

1 month<br />

Months


4.5 Equivalence Relations: Single Amounts with PP CP 107<br />

TABLE 4–5<br />

Length<br />

of Time<br />

Section References for Equivalence Calculations Based on<br />

Payment Period and Compounding Period Comparison<br />

Involves<br />

Single Amounts<br />

(P and F Only)<br />

PP CP Section 4.5 Section 4.6<br />

PP > CP Section 4.5 Section 4.6<br />

PP < CP Section 4.7 Section 4.7<br />

Involves Uniform Series<br />

or Gradient Series<br />

(A, G, or g)<br />

As we learned earlier, to correctly perform equivalence calculations, an effective interest rate<br />

is needed in the factors and spreadsheet functions. Therefore, it is essential that the time periods<br />

of the interest rate and the payment period be on the same time basis. The next three sections (4.5<br />

to 4.7) describe procedures to determine correct i and n values for engineering economy factors<br />

and spreadsheet functions. First, compare the length of PP and CP, then identify the cash flows as<br />

only single amounts ( P and F ) or as a series ( A , G , or g ). Table 4–5 provides the section reference.<br />

The section references are the same when PP CP and PP > CP, because the procedures to<br />

determine i and n are the same, as discussed in Sections 4.5 and 4.6.<br />

A general principle to remember throughout these equivalence computations is that when<br />

cash actually flows, it is necessary to account for the time value of money. For example, assume<br />

that cash flows occur every 6 months and that interest is compounded quarterly. After 3<br />

months there is no cash flow and no need to determine the effect of quarterly compounding.<br />

However, at the 6-month time point, it is necessary to consider the interest accrued during the<br />

previous two quarters.<br />

4.5 Equivalence Relations: Single Amounts with PP CP<br />

With only P and F estimates defined, the payment period is not specifically identified. In virtually<br />

all situations, PP will be equal to or greater than CP. The length of the PP is defined by the interest<br />

period in the stated interest rate. If the rate is 8% per year, for example, PP CP 1 year. However,<br />

if the rate is 10% per year, compounded quarterly, then PP is 1 year, CP is 1 quarter or<br />

3 months, and PP > CP. The procedures to perform equivalence computations are the same for<br />

both situations, as explained here.<br />

When only single-amount cash flows are involved, there are two equally correct ways to determine<br />

i and n for P F and F P factors. Method 1 is easier to apply, because the interest tables<br />

in the back of the text can usually provide the factor value. Method 2 likely requires a factor<br />

formula calculation, because the resulting effective interest rate is not an integer. For spreadsheets,<br />

either method is acceptable; however, method 1 is usually easier.<br />

Method 1: Determine the effective interest rate over the compounding period CP , and set n<br />

equal to the number of compounding periods between P and F . The relations to calculate P and<br />

F are<br />

P F ( P F , effective i % per CP, total number of periods n ) [4.8]<br />

F P ( F P , effective i % per CP, total number of periods n ) [4.9]<br />

For example, assume that the stated credit card rate is nominal 15% per year, compounded<br />

monthly. Here CP is 1 month. To find P or F over a 2-year span, calculate the effective monthly<br />

rate of 15%12 1.25% and the total months of 2(12) 24. Then 1.25% and 24 are used in the<br />

PF and FP factors.<br />

Any time period can be used to determine the effective interest rate; however, the interest rate<br />

that is associated with the CP is typically the best because it is usually a whole number. Therefore,<br />

the factor tables in the back of the text can be used.<br />

Method 2: Determine the effective interest rate for the time period t of the nominal rate, and<br />

set n equal to the total number of periods, using this same time period.


108 Chapter 4 Nominal and Effective Interest Rates<br />

The P and F relations are the same as in Equations [4.8] and [4.9] with the term effective i% per t<br />

substituted for the interest rate. For a credit card rate of 15% per year, compounded monthly, the<br />

time period t is 1 year. The effective rate over 1 year and the n values are<br />

Effective i % per year ( 1 0.15 ——<br />

12 ) 12 1 16.076%<br />

n 2 years<br />

The P F factor is the same by both methods: ( P F ,1.25%,24) 0.7422 using Table 5 in the rear<br />

of the text; and ( P F ,16.076%,2) 0.7422 using the P F factor formula.<br />

EXAMPLE 4.7<br />

Over the past 10 years, Gentrack has placed varying sums of money into a special capital<br />

accumulation fund. The company sells compost produced by garbage-to-compost plants in<br />

the United States and Vietnam. Figure 4–5 is the cash flow diagram in $1000 units. Find the<br />

amount in the account now (after 10 years) at an interest rate of 12% per year, compounded<br />

semiannually.<br />

Solution<br />

Only P and F values are involved. Both methods are illustrated to find F in year 10.<br />

Method 1: Use the semiannual CP to express the effective semiannual rate of 6% per 6-month<br />

period. There are n (2)(number of years) semiannual periods for each cash flow. Using tabulated<br />

factor values, the future worth by Equation [4.9] is<br />

F 1000(FP,6%,20) 3000(FP,6%,12) 1500(FP,6%,8)<br />

1000(3.2071) 3000(2.0122) 1500(1.5938)<br />

$11,634 ($11.634 million)<br />

Method 2: Express the effective annual rate, based on semiannual compounding.<br />

Effective i% per year ( 1 0.12 ——<br />

2 ) 2 1 12.36%<br />

The n value is the actual number of years. Use the factor formula (FP,i,n) (1.1236) n and<br />

Equation [4.9] to obtain the same answer as above.<br />

F 1000(FP,12.36%,10) 3000(FP,12.36%,6) 1500(FP,12.36%,4)<br />

1000(3.2071) 3000(2.0122) 1500(1.5938)<br />

$11,634 ($11.634 million)<br />

F =?<br />

0 1 2 3 4 5 6 7 8 9<br />

10 Year<br />

$1000<br />

Figure 4–5<br />

Cash flow diagram, Example 4.7.<br />

$3000<br />

$1500


4.6 Equivalence Relations: Series with PP CP 109<br />

TABLE 4–6 Examples of n and i Values Where PP CP or PP > CP<br />

Cash Flow Series<br />

$500 semiannually<br />

for 5 years<br />

$75 monthly for<br />

3 years<br />

$180 quarterly for<br />

15 years<br />

$25 per month<br />

increase for<br />

4 years<br />

$5000 per quarter<br />

for 6 years<br />

Interest Rate<br />

What to Find;<br />

What Is Given Standard Notation<br />

16% per year, Find P ; given A P 500( P A ,8%,10)<br />

compounded<br />

semiannually<br />

24% per year, Find F ; given A F 75( F A ,2%,36)<br />

compounded<br />

monthly<br />

5% per quarter Find F ; given A F 180( F A ,5%,60)<br />

1% per month Find P ; given G P 25( P G ,1%,48)<br />

1% per month Find A ; given P A 5000( A P ,3.03%,24)<br />

4.6 Equivalence Relations: Series with PP CP<br />

When uniform or gradient series are included in the cash flow sequence, the procedure is basically<br />

the same as method 2 above, except that PP is now defined by the length of time between<br />

cash flows. This also establishes the time unit of the effective interest rate. For example, if cash<br />

flows occur on a quarterly basis, PP is 1 quarter and the effective quarterly rate is necessary. The<br />

n value is the total number of quarters. If PP is a quarter, 5 years translates to an n value of<br />

20 quarters. This is a direct application of the following general guideline:<br />

When cash flows involve a series (i.e., A , G , g ) and the payment period equals or exceeds the<br />

compounding period in length:<br />

• Find the effective i per payment period.<br />

• Determine n as the total number of payment periods.<br />

In performing equivalence computations for series, only these values of i and n can be used in<br />

interest tables, factor formulas, and spreadsheet functions. In other words, there are no other<br />

combinations that give the correct answers, as there are for single-amount cash fl ows.<br />

Table 4–6 shows the correct formulation for several cash flow series and interest rates. Note<br />

that n is always equal to the total number of payment periods and i is an effective rate expressed<br />

over the same time period as n .<br />

EXAMPLE 4.8<br />

For the past 7 years, Excelon Energy has paid $500 every 6 months for a software maintenance<br />

contract. What is the equivalent total amount after the last payment, if these funds are taken<br />

from a pool that has been returning 8% per year, compounded quarterly?<br />

Solution<br />

The cash flow diagram is shown in Figure 4–6. The payment period (6 months) is longer than<br />

the compounding period (quarter); that is, PP > CP. Applying the guideline, we need to determine<br />

an effective semiannual interest rate. Use Equation [4.7] with r 4% per 6-month<br />

period and m 2 quarters per semiannual period.<br />

Effective i% per 6 months ( 1 0.04 ——<br />

2 ) 2 1 4.04%<br />

The effective semiannual interest rate can also be obtained from Table 4–3 by using the r value<br />

of 4% and m 2 to get i 4.04%.


110 Chapter 4 Nominal and Effective Interest Rates<br />

The value i 4.04% seems reasonable, since we expect the effective rate to be slightly<br />

higher than the nominal rate of 4% per 6-month period. The total number of semiannual payment<br />

periods is n 2(7) 14. The relation for F is<br />

F A(FA,4.04%,14)<br />

500(18.3422)<br />

$9171.09<br />

To determine the FA factor value 18.3422 using a spreadsheet, enter the FV function from<br />

Figure 2–9, that is, FV(4.04%,14,1). Alternatively, the final answer of $9171.09 can be<br />

displayed directly using the function FV(4.04%,14,500).<br />

i = 8% per year, compounded quarterly<br />

F =?<br />

0 1 2 3 4 5 6<br />

7<br />

Years<br />

A = $500<br />

Figure 4–6<br />

Diagram of semiannual deposits used to determine F, Example 4.8.<br />

EXAMPLE 4.9 Credit Card Offer Case<br />

In our continuing credit card saga of Dave and his job transfer to Africa, let’s assume he did<br />

remember that the total balance is $1030, including the $30 balance transfer fee, and he wants<br />

to set up a monthly automatic checking account transfer to pay off the entire amount in 2 years.<br />

Once he learned that the minimum payment is $25 per month, Dave decided to be sure the<br />

monthly transfer exceeds this amount to avoid any further penalty fees and the penalty APR of<br />

29.99% per year. What amount should he ask to be transferred by the due date each month?<br />

What is the APY he will pay, if this plan is followed exactly and Chase Bank does not change<br />

the APR during the 2-year period? Also, assume he left the credit card at home and will charge<br />

no more to it.<br />

Solution<br />

The monthly A series is needed for a total of n 2(12) 24 payments. In this case,<br />

PP CP 1 month, and the effective monthly rate is i 14.24%12 1.187% per month.<br />

Solution by hand:<br />

Use a calculator or hand computation to determine the A P factor value.<br />

A P ( A P , i , n ) 1030( A P ,1.187%,24) 1030(0.04813)<br />

$49.57 per month for 24 months<br />

Solution by Spreadsheet: Use the function PMT(1.187%,24,1) to determine the factor<br />

value 0.04813 to determine A for n 24 payments. Alternatively use the function<br />

PMT (1.187%,24,1030) to directly display the required monthly payment of A $49.57.<br />

The effective annual interest rate or APY is computed using Equation [4.7] with r 14.24%<br />

per year, compounded monthly, and m 12 times per year.<br />

Effective i per year ( 1 0.1424 ———<br />

12 ) 12 – 1 1.15207 1<br />

15.207% per year<br />

This is the same effective annual rate i a determined in Example 4.4 b .<br />

PE


4.6 Equivalence Relations: Series with PP CP 111<br />

EXAMPLE 4.10<br />

The Scott and White Health Plan (SWHP) has purchased a robotized prescription fulfillment<br />

system for faster and more accurate delivery to patients with stable, pill-form medication for<br />

chronic health problems, such as diabetes, thyroid, and high blood pressure. Assume this highvolume<br />

system costs $3 million to install and an estimated $200,000 per year for all materials,<br />

operating, personnel, and maintenance costs. The expected life is 10 years. An SWHP biomedical<br />

engineer wants to estimate the total revenue requirement for each 6-month period that<br />

is necessary to recover the investment, interest, and annual costs. Find this semiannual A value<br />

both by hand and by spreadsheet, if capital funds are evaluated at 8% per year, using two different<br />

compounding periods:<br />

Rate 1. 8% per year, compounded semiannually .<br />

Rate 2. 8% per year, compounded monthly .<br />

Solution<br />

Figure 4–7 shows the cash flow diagram. Throughout the 20 semiannual periods, the annual<br />

cost occurs every other period, and the capital recovery series is sought for every 6-month<br />

period. This pattern makes the solution by hand quite involved if the P F factor, not the P A<br />

factor, is used to find P for the 10 annual $200,000 costs. The spreadsheet solution is recommended<br />

in cases such as this.<br />

Solution by hand—rate 1: Steps to find the semiannual A value are summarized below:<br />

PP CP at 6 months; find the effective rate per semiannual period.<br />

Effective semiannual i 8%2 4% per 6 months, compounded semiannually.<br />

Number of semiannual periods n 2(10) 20.<br />

Calculate P , using the P F factor for n 2, 4, . . . , 20 periods because the costs are annual,<br />

not semiannual. Then use the A P factor over 20 periods to find the semiannual A .<br />

20<br />

P 3,000,000 200,000 [ ( P F ,4%, k ) ]<br />

k2,4<br />

3,000,000 200,000(6.6620) $4,332,400<br />

A $4,332,400( A P ,4%,20) $318,778<br />

Conclusion: Revenue of $318,778 is necessary every 6 months to cover all costs and interest<br />

at 8% per year, compounded semiannually.<br />

A per 6 months = ?<br />

2 4 6 8 10 12 14 16 18<br />

20<br />

0<br />

1 2 3 4 5 6 7 8 9<br />

10<br />

6 months<br />

Years<br />

$200,000 per year<br />

i 1 = 8%, compounded semiannually<br />

i 2 = 8%, compounded monthly<br />

P = $3 million<br />

Figure 4–7<br />

Cash flow diagram with two different compounding periods, Example 4.10.


112 Chapter 4 Nominal and Effective Interest Rates<br />

Solution by hand—rate 2: The PP is 6 months, but the CP is now monthly; therefore, PP CP.<br />

To find the effective semiannual rate, the effective interest rate Equation [4.7] is applied with<br />

r 4% and m 6 months per semiannual period.<br />

Effective semiannual i ( 1 0.04 ———<br />

6 ) 6 1 4.067%<br />

20<br />

P 3,000,000 200,000 [ <br />

k2,4<br />

( P F ,4.067%, k ) ]<br />

3,000,000 200,000(6.6204) $4,324,080<br />

A $4,324,080( A P ,4.067%,20) $320,064<br />

Now, $320,064, or $1286 more semiannually, is required to cover the more frequent compounding<br />

of the 8% per year interest. Note that all P F and A P factors must be calculated<br />

with factor formulas at 4.067%. This method is usually more calculation-intensive and errorprone<br />

than the spreadsheet solution.<br />

Solution by spreadsheet—rates 1 and 2: Figure 4–8 presents a general solution for the problem<br />

at both rates. (Several rows at the bottom of the spreadsheet are not printed. They continue the<br />

cash flow pattern of $200,000 every other 6 months through cell B32.) The functions in C8 and<br />

E8 are general expressions for the effective rate per PP, expressed in months. This allows some<br />

sensitivity analysis to be performed for different PP and CP values. Note the functions in C7<br />

and E7 to determine m for the effective rate relations. This technique works well for spreadsheets<br />

once PP and CP are entered in the time unit of the CP.<br />

Each 6-month period is included in the cash flows, including the $0 entries, so the NPV and<br />

PMT functions work correctly. The final A values in D14 ($318,784) and F14 ($320,069) are<br />

the same (except for rounding) as those above.<br />

C5/C6<br />

E5/E6<br />

+((1+((E2/(12/E5))/E7)) ∧ E7)1<br />

PMT(E8,E4,F9<br />

NPV(E8,B13:B32)+B12<br />

Figure 4–8<br />

Spreadsheet solution for semiannual A series for different compounding periods, Example 4.10.<br />

4.7 Equivalence Relations: Single Amounts<br />

and Series with PP CP<br />

If a person deposits money each month into a savings account where interest is compounded<br />

quarterly, do all the monthly deposits earn interest before the next quarterly compounding time?<br />

If a person's credit card payment is due with interest on the 15th of the month, and if the full payment<br />

is made on the 1st, does the financial institution reduce the interest owed, based on early<br />

payment? The usual answers are no. However, if a monthly payment on a $10 million, quarterly


4.7 Equivalence Relations: Single Amounts and Series with PP CP 113<br />

compounded, bank loan were made early by a large corporation, the corporate financial officer<br />

would likely insist that the bank reduce the amount of interest due, based on early payment.<br />

These are examples of PP < CP. The timing of cash flow transactions between compounding<br />

points introduces the question of how interperiod compounding is handled. Fundamentally,<br />

there are two policies: interperiod cash flows earn no interest, or they earn compound interest .<br />

For a no-interperiod-interest policy, negative cash flows (deposits or payments, depending on the<br />

perspective used for cash flows) are all regarded as made at the end of the compounding period,<br />

and positive cash flows (receipts or withdrawals) are all regarded as made at the beginning.<br />

As an illustration, when interest is compounded quarterly, all monthly deposits are moved to the end<br />

of the quarter (no interperiod interest is earned), and all withdrawals are moved to the beginning (no<br />

interest is paid for the entire quarter). This procedure can significantly alter the distribution of cash<br />

flows before the effective quarterly rate is applied to find P , F , or A . This effectively forces the cash<br />

flows into a PP CP situation, as discussed in Sections 4.5 and 4.6. Example 4.11 illustrates this<br />

procedure and the economic fact that, within a one-compounding-period time frame, there is no<br />

interest advantage to making payments early. Of course, noneconomic factors may be present.<br />

EXAMPLE 4.11<br />

Last year AllStar Venture Capital agreed to invest funds in Clean Air Now (CAN), a start-up<br />

company in Las Vegas that is an outgrowth of research conducted in mechanical engineering at<br />

the University of Nevada–Las Vegas. The product is a new filtration system used in the process<br />

of carbon capture and sequestration (CCS) for coal-fired power plants. The venture fund manager<br />

generated the cash flow diagram in Figure 4–9 a in $1000 units from AllStar’s perspective. Included<br />

are payments (outflows) to CAN made over the first year and receipts (inflows) from CAN<br />

to AllStar. The receipts were unexpected this first year; however, the product has great promise,<br />

and advance orders have come from eastern U.S. plants anxious to become zero-emission<br />

coal-fueled plants. The interest rate is 12% per year, compounded quarterly, and AllStar uses the<br />

no-interperiod-interest policy. How much is AllStar in the “red” at the end of the year?<br />

0<br />

$150<br />

$200<br />

Receipts from CAN<br />

$90<br />

$120<br />

0 1 2 3 4 5 6 7 8 9 10 11 12<br />

$50<br />

$75 $100<br />

$90<br />

Payments to CAN<br />

(a)<br />

$45<br />

1<br />

Year<br />

Month<br />

$180<br />

$165<br />

0<br />

0 1 2 3<br />

1 2 3 4<br />

4 5 6<br />

7 8 9 10 11 12<br />

$50<br />

Quarter<br />

Month<br />

$150<br />

$200<br />

$175<br />

(b)<br />

F = ?<br />

Figure 4–9<br />

( a ) Actual and ( b ) moved cash flows (in $1000) for quarterly compounding periods using no interperiod<br />

interest, Example 4.11.


114 Chapter 4 Nominal and Effective Interest Rates<br />

Solution<br />

With no interperiod interest considered, Figure 4–9 b reflects the moved cash flows. All negative<br />

cash flows (payments to CAN) are moved to the end of the respective quarter, and all<br />

positive cash flows (receipts) are moved to the beginning of the respective quarter. Calculate<br />

the F value at 12%4 3% per quarter.<br />

F 1000[150( F P ,3%,4) 200( F P ,3%,3) (175 180)( F P ,3%,2)<br />

165( F P ,3%,1) 50]<br />

$262,111<br />

AllStar has a net investment of $262,111 in CAN at the end of the year.<br />

If PP CP and interperiod compounding is earned, then the cash flows are not moved, and the<br />

equivalent P , F , or A values are determined using the effective interest rate per payment period.<br />

The engineering economy relations are determined in the same way as in the previous two sections<br />

for PP CP. The effective interest rate formula will have an m value less than 1, because there is<br />

only a fractional part of the CP within one PP. For example, weekly cash flows and quarterly compounding<br />

require that m 113 of a quarter. When the nominal rate is 12% per year, compounded<br />

quarterly (the same as 3% per quarter, compounded quarterly), the effective rate per PP is<br />

Effective weekly i % (1.03 ) 113 1 0.228% per week<br />

4.8 Effective Interest Rate for Continuous<br />

Compounding<br />

If we allow compounding to occur more and more frequently, the compounding period becomes<br />

shorter and shorter and m , the number of compounding periods per payment period, increases.<br />

Continuous compounding is present when the duration of CP, the compounding period, becomes<br />

infinitely small and m , the number of times interest is compounded per period, becomes infinite.<br />

Businesses with large numbers of cash flows each day consider the interest to be continuously<br />

compounded for all transactions.<br />

As m approaches infinity, the effective interest rate Equation [4.7] must be written in a new<br />

form. First, recall the definition of the natural logarithm base.<br />

lim<br />

h ( 1 1 —<br />

h ) h e 2.71828 [4.10]<br />

The limit of Equation [4.7] as m approaches infinity is found by using r m 1 h , which makes<br />

m hr .<br />

lim i lim ( 1 — r<br />

m m m ) m 1<br />

lim<br />

h ( 1 1 —<br />

h ) hr 1 lim<br />

h [ ( 1 1 —<br />

h ) h ] r 1<br />

i e r 1 [4.11]<br />

Equation [4.11] is used to compute the effective continuous interest rate, when the time periods<br />

on i and r are the same. As an illustration, if the nominal annual r 15% per year, the effective<br />

continuous rate per year is<br />

i % e 0.15 1 16.183%<br />

For convenience, Table 4–3 includes effective continuous rates for the nominal rates listed.<br />

To find an effective or nominal interest rate for continuous compounding using the spreadsheet<br />

functions EFFECT or NOMINAL, enter a very large value for the compounding frequency<br />

m in Equation [4.5] or [4.6], respectively. A value of 10,000 or higher provides sufficient<br />

accuracy. Both functions are illustrated in Example 4.12.


4.8 Effective Interest Rate for Continuous Compounding 115<br />

EXAMPLE 4.12<br />

(a) For an interest rate of 18% per year, compounded continuously, calculate the effective<br />

monthly and annual interest rates.<br />

(b) An investor requires an effective return of at least 15%. What is the minimum annual<br />

nominal rate that is acceptable for continuous compounding?<br />

Solution by Hand<br />

(a) The nominal monthly rate is r 18%12 1.5%, or 0.015 per month. By Equation<br />

[4.11], the effective monthly rate is<br />

i % per month e r − 1 e 0.015 − 1 1.511%<br />

Similarly, the effective annual rate using r 0.18 per year is<br />

i % per year e r − 1 e 0.18 − 1 19.722%<br />

(b) Solve Equation [4.11] for r by taking the natural logarithm.<br />

e r − 1 0.15<br />

e r 1.15<br />

ln e r ln 1.15<br />

r 0.13976<br />

Therefore, a rate of 13.976% per year, compounded continuously, will generate an effective<br />

15% per year return. The general formula to find the nominal rate, given the effective<br />

continuous rate i , is r ln(1 i ) .<br />

Solution by Spreadsheet<br />

(a) Use the EFFECT function with the nominal monthly rate r 1.5% and annual rate<br />

r 18% with a large m to display effective i values. The functions to enter on a spreadsheet<br />

and the responses are as follows:<br />

Monthly: EFFECT(1.5%,10000) effective i 1.511% per month<br />

Annual: EFFECT(18%,10000) effective i 19.722% per year<br />

(b) Use the function in Equation [4.6] in the format NOMINAL(15%,10000) to display<br />

the nominal rate of 13.976% per year, compounded continuously.<br />

EXAMPLE 4.13<br />

Engineers Marci and Suzanne both invest $5000 for 10 years at 10% per year. Compute the<br />

future worth for both individuals if Marci receives annual compounding and Suzanne receives<br />

continuous compounding.<br />

Solution<br />

Marci: For annual compounding the future worth is<br />

F P ( F P ,10%,10) 5000(2.5937) $12,969<br />

Suzanne: Using Equation [4.11], first find the effective i per year for use in the F P factor.<br />

Effective i % e 0.10 1 10.517%<br />

F P ( F P ,10.517%,10) 5000(2.7183) $13,591<br />

Continuous compounding causes a $622 increase in earnings. For comparison, daily compounding<br />

yields an effective rate of 10.516% ( F $13,590), only slightly less than the<br />

10.517% for continuous compounding.


116 Chapter 4 Nominal and Effective Interest Rates<br />

For some business activities, cash flows occur throughout the day. Examples of costs are energy<br />

and water costs, inventory costs, and labor costs. A realistic model for these activities is to<br />

increase the frequency of the cash flows to become continuous. In these cases, the economic<br />

analysis can be performed for continuous cash flow (also called continuous funds flow) and the<br />

continuous compounding of interest as discussed above. Different expressions must be derived<br />

for the factors for these cases. In fact, the monetary differences for continuous cash flows relative<br />

to the discrete cash flow and discrete compounding assumptions are usually not large. Accordingly,<br />

most engineering economy studies do not require the analyst to utilize these mathematical<br />

forms to make a sound economic decision.<br />

4.9 Interest Rates That Vary over Time<br />

Real-world interest rates for a corporation vary from year to year, depending upon the financial<br />

health of the corporation, its market sector, the national and international economies, forces of<br />

inflation, and many other elements. Loan rates may increase from one year to another. Home<br />

mortgages financed using ARM (adjustable-rate mortgage) interest is a good example. The mortgage<br />

rate is slightly adjusted annually to reflect the age of the loan, the current cost of mortgage<br />

money, etc.<br />

When P , F , and A values are calculated using a constant or average interest rate over the life<br />

of a project, rises and falls in i are neglected. If the variation in i is large, the equivalent values<br />

will vary considerably from those calculated using the constant rate. Although an engineering<br />

economy study can accommodate varying i values mathematically, it is more involved computationally<br />

to do so.<br />

To determine the P value for future cash flow values ( F t ) at different i values ( i t ) for each year t ,<br />

we will assume annual compounding. Define<br />

i t effective annual interest rate for year t ( t years 1 to n )<br />

To determine the present worth, calculate the P of each F t value, using the applicable i t , and sum<br />

the results. Using standard notation and the P F factor,<br />

P F 1 ( P F , i 1 ,1) F 2 ( P F , i 1 ,1)( P F , i 2 ,1) . . .<br />

F n ( P F , i 1 ,1)(P F , i 2 ,1) . . . ( P F , i n ,1) [4.12]<br />

When only single amounts are involved, that is, one P and one F in the final year n , the last term<br />

in Equation [4.12] is the expression for the present worth of the future cash flow.<br />

P F n ( P F , i 1 ,1)( P F , i 2 ,1) . . . ( P F , i n ,1) [4.13]<br />

If the equivalent uniform series A over all n years is needed, first find P , using either of the last<br />

two equations; then substitute the symbol A for each F t symbol. Since the equivalent P has been<br />

determined numerically using the varying rates, this new equation will have only one unknown,<br />

namely, A . Example 4.14 illustrates this procedure.<br />

EXAMPLE 4.14<br />

CE, Inc., leases large earth tunneling equipment. The net profit from the equipment for each of<br />

the last 4 years has been decreasing, as shown below. Also shown are the annual rates of return<br />

on invested capital. The return has been increasing. Determine the present worth P and equivalent<br />

uniform series A of the net profit series. Take the annual variation of rates of return into<br />

account.<br />

Year 1 2 3 4<br />

Net Profit $70,000 $70,000 $35,000 $25,000<br />

Annual Rate 7% 7% 9% 10%


Solution<br />

Figure 4–10 shows the cash flows, rates for each year, and the equivalent P and A . Equation<br />

[4.12] is used to calculate P . Since for both years 1 and 2 the net profit is $70,000 and the<br />

annual rate is 7%, the P A factor can be used for these 2 years only.<br />

$70,000<br />

P [70( P A ,7%,2) 35( P F ,7%,2)( P F ,9%,1)<br />

25( P F ,7%,2)( P F ,9%,1)( P F ,10%,1)](1000)<br />

[70(1.8080) 35(0.8013) 25(0.7284)](1000)<br />

$172,816 [4.14]<br />

Chapter Summary 117<br />

$35,000<br />

$25,000<br />

A = ?<br />

0<br />

1 2 3 4<br />

0<br />

1 2 3 4<br />

Year<br />

i = 7%<br />

i = 7%<br />

i = 9%<br />

i = 10%<br />

7% 7% 9% 10%<br />

P = ?<br />

$172,816<br />

Figure 4–10<br />

Equivalent P and A values for varying interest rates, Example 4.14.<br />

To determine an equivalent annual series, substitute the symbol A for all net profit values<br />

on the right side of Equation [4.14], set it equal to P $172,816, and solve for A . This<br />

equation accounts for the varying i values each year. See Figure 4–10 for the cash flow diagram<br />

transformation.<br />

$172,816 A [(1.8080) (0.8013) (0.7284)] A [3.3377]<br />

A $51,777 per year<br />

Comment<br />

If the average of the four annual rates, that is, 8.25% is used, the result is A $52,467. This is<br />

a $690 per year overestimate of the equivalent annual net profit.<br />

When there is a cash flow in year 0 and interest rates vary annually, this cash flow must be<br />

included to determine P . In the computation for the equivalent uniform series A over all years,<br />

including year 0, it is important to include this initial cash flow at t 0. This is accomplished by<br />

inserting the factor value for ( P F , i 0 ,0) into the relation for A . This factor value is always 1.00. It<br />

is equally correct to find the A value using a future worth relation for F in year n . In this case, the<br />

A value is determined using the F P factor, and the cash flow in year n is accounted for by including<br />

the factor ( F P , i n ,0) 1.00.<br />

CHAPTER SUMMARY<br />

Since many real-world situations involve cash flow frequencies and compounding periods other<br />

than 1 year, it is necessary to use nominal and effective interest rates. When a nominal rate r is<br />

stated, the effective interest rate i per payment period is determined by using the effective interest<br />

rate equation.<br />

Effective i ( 1 r —<br />

m ) m 1


118 Chapter 4 Nominal and Effective Interest Rates<br />

The m is the number of compounding periods (CP) per interest period. If interest compounding<br />

becomes more and more frequent, then the length of a CP approaches zero, continuous compounding<br />

results, and the effective i is e r 1.<br />

All engineering economy factors require the use of an effective interest rate. The i and n values<br />

placed in a factor depend upon the type of cash flow series. If only single amounts ( P and F )<br />

are present, there are several ways to perform equivalence calculations using the factors. However,<br />

when series cash flows ( A, G, and g ) are present, only one combination of the effective rate<br />

i and number of periods n is correct for the factors. This requires that the relative lengths of PP<br />

and CP be considered as i and n are determined. The interest rate and payment periods must have<br />

the same time unit for the factors to correctly account for the time value of money.<br />

From one year (or interest period) to the next, interest rates will vary. To accurately perform<br />

equivalence calculations for P and A when rates vary significantly, the applicable interest rate<br />

should be used, not an average or constant rate.<br />

PROBLEMS<br />

Nominal and Effective Rates<br />

4.1 From the interest statement 18% per year, compounded<br />

monthly, determine the values for interest<br />

period, compounding period, and compounding<br />

frequency.<br />

4.2 From the interest statement 1% per month, determine<br />

the values for interest period, compounding<br />

period, and compounding frequency.<br />

4.3 Determine the number of times interest would be<br />

compounded in 6 months from the interest statements<br />

( a ) 18% per year, compounded monthly,<br />

( b ) 1% per month, and ( c ) 2% per quarter.<br />

4.4 For an interest rate of 1% per 2 months, determine<br />

the number of times interest would be compounded<br />

in ( a ) 2 months, ( b ) two semiannual periods, and<br />

( c ) 3 years.<br />

4.5 Identify the compounding period for the following<br />

interest statements: ( a ) 3% per quarter; ( b ) 10%<br />

per year, compounded semiannually; ( c ) nominal<br />

7.2% per year, compounded monthly; ( d ) effective<br />

3.4% per quarter, compounded weekly; and ( e ) 2%<br />

per month, compounded continuously.<br />

4.6 Identify the following interest rate statements as<br />

either nominal or effective: ( a ) 1.5% per month,<br />

compounded daily; ( b ) 17% per year, compounded<br />

quarterly; ( c ) effective 15% per year, compounded<br />

monthly; ( d ) nominal 0.6% per month, compounded<br />

weekly; ( e ) 0.3% per week, compounded<br />

weekly; and ( f ) 8% per year.<br />

4.7 Convert the given interest rates in the left-hand<br />

column into the nominal rates listed in the righthand<br />

column. (Assume 4 weeksmonth.)<br />

Given Interest Rate Desired Interest Rate<br />

1% per month Nominal rate per year<br />

3% per quarter Nominal rate per 6 months<br />

2% per quarter Nominal rate per year<br />

0.28% per week Nominal rate per quarter<br />

6.1% per 6 months Nominal rate per 2 years<br />

4.8 What nominal interest rate per year is equivalent to<br />

11.5% per year, compounded monthly?<br />

4.9 For a Federal Credit Union that offers an interest<br />

rate of 8% per year, compounded quarterly, determine<br />

the nominal rate per 6 months.<br />

4.10 The Second National Bank of Fullertum advertises<br />

an APR of 14% compounded monthly for student<br />

loans. Determine the APY. Show hand and spreadsheet<br />

solutions.<br />

4.11 For an effective annual rate i a of 15.87% compounded<br />

quarterly, determine ( a ) the effective<br />

quarterly rate and ( b ) the nominal annual rate.<br />

( c ) What is the spreadsheet function to find the<br />

nominal annual rate above?<br />

4.12 High-tech companies such as IBM, AMD, and<br />

Intel have been using nanotechnology for several<br />

years to make microchips with faster speeds while<br />

using less power. A less well-known company in<br />

the chip business has been growing fast enough<br />

that the company uses a minimum attractive rate of<br />

return of 60% per year. If this MARR is an effective<br />

annual rate compounded monthly, determine<br />

the effective monthly rate.<br />

4.13 An interest rate of 21% per year, compounded<br />

every 4 months, is equivalent to what effective rate<br />

per year? Show hand and spreadsheet solutions.


Problems 119<br />

4.14 An interest rate of 8% per 6 months, compounded<br />

monthly, is equivalent to what effective rate per<br />

quarter?<br />

4.15 A small company that makes modular bevel gear<br />

drives with a tight swing ratio for optimizing pallet<br />

truck design was told that the interest rate on a<br />

mortgage loan would be an effective 4% per quarter,<br />

compounded monthly. The owner was confused<br />

by the terminology and asked you to help.<br />

What are ( a ) the APR and ( b ) the APY?<br />

4.16 In ‘N Out Payday Loans advertises that for a fee of<br />

only $10, you can immediately borrow up to $200<br />

for one month. If a person accepts the offer, what<br />

are ( a ) the nominal interest rate per year and<br />

( b ) the effective rate per year?<br />

4.17 A government-required truth-in-lending document<br />

showed that the APR was 21% and the APY was<br />

22.71%. Determine the compounding frequency at<br />

which the two rates are equivalent.<br />

4.18 Julie has a low credit rating, plus she was furloughed<br />

from her job 2 months ago. She has a new<br />

job starting next week and expects a salary to start<br />

again in a couple of weeks. Since she is a little<br />

short on money to pay her rent, she decided to borrow<br />

$100 from a loan company, which will charge<br />

her only $10 interest if the $110 is paid no more<br />

than 1 week after the loan is made. What are the<br />

( a ) nominal annual and ( b ) effective annual interest<br />

rates that she will pay on this loan?<br />

Equivalence When PP CP<br />

4.19 Assume you deposit 25% of your monthly check<br />

of $5500 into a savings account at a credit union<br />

that compounds interest semiannually. ( a ) What<br />

are the payment and compounding periods? ( b ) Is<br />

the payment period greater than or less than the<br />

compounding period?<br />

4.20 Interest is compounded quarterly, and singlepayment<br />

cash flows (that is, F and P ) are separated<br />

by 5 years. What must the compounding period be<br />

on the interest rate, if the value of n in the P F or<br />

F P equation is (a) n 5, ( b ) n 20, or ( c ) n 60?<br />

4.21 When interest is compounded quarterly and a<br />

uniform series cash flow of $4000 occurs every<br />

6 months, what time periods on i and n must be<br />

used?<br />

4.22 Pinpoint Laser Systems is planning to set aside<br />

$260,000 now for possibly replacing its 4-pole,<br />

38-MW synchronous motors when it becomes<br />

necessary. If the replacement is expected to take<br />

place in 3 years, how much will the company have<br />

in its investment set-aside account? Assume the<br />

company can achieve a rate of return of 12% per<br />

year, compounded quarterly.<br />

4.23 Wheeling-Pittsburgh Steel is investigating whether<br />

it should replace some of its basic oxygen furnace<br />

equipment now or wait to do it later. The cost<br />

later (i.e., 3 years from now) is estimated to be<br />

$1,700,000. How much can the company afford to<br />

spend now, if its minimum attractive rate of return<br />

is 1.5% per month?<br />

4.24 How much can Wells Fargo lend to a developer<br />

who will repay the loan by selling 6 view lots at<br />

$190,000 each 2 years from now? Assume the<br />

bank will lend at a nominal 14% per year, compounded<br />

semiannually.<br />

4.25 How much will be in a high-yield account at the<br />

National Bank of Arizona 12 years from now if<br />

you deposit $5000 now and $7000 five years from<br />

now? The account earns interest at a rate of 8% per<br />

year, compounded quarterly.<br />

4.26 Loadstar Sensors is a company that makes load<br />

force sensors based on capacitive sensing technology.<br />

The company wants to have $28 million for a<br />

plant expansion 4 years from now. If the company<br />

has already set aside $12 million in an investment<br />

account for the expansion, how much more must<br />

the company add to the account next year (i.e.,<br />

1 year from now) so that it will have the $28 million<br />

4 years from now? The account earns interest<br />

at 12% per year, compounded quarterly.<br />

4.27 A structural engineering consulting company is<br />

examining its cash flow requirements for the next<br />

6 years. The company expects to replace office<br />

machines and computer equipment at various<br />

times over the 6-year planning period. Specifically,<br />

the company expects to spend $21,000 two<br />

years from now, $24,000 three years from now,<br />

and $10,000 five years from now. What is the present<br />

worth of the planned expenditures at an interest<br />

rate of 10% per year, compounded semiannually?<br />

4.28 Irvin Aerospace of Santa Ana, California, was<br />

awarded a 5-year contract to develop an advanced<br />

space capsule airbag landing attenuation system<br />

for NASA’s Langley Research Center. The company’s<br />

computer system uses fluid structure interaction<br />

modeling to test and analyze each airbag<br />

design concept. What is the present worth of the<br />

contract at 16% per year, compounded quarterly, if<br />

the quarterly cost in years 1 through 5 is $2 million<br />

per quarter?


120 Chapter 4 Nominal and Effective Interest Rates<br />

4.29 Heyden Motion Solutions ordered $7 million<br />

worth of seamless tubes for its drill collars from<br />

the Timken Company of Canton, Ohio. (A drill<br />

collar is the heavy tubular connection between a<br />

drill pipe and a drill bit.) At 12% per year, compounded<br />

semiannually, what is the equivalent uniform<br />

cost per semiannual period over a 5-year<br />

amortization period?<br />

4.30 In 2010, the National Highway Traffic Safety Administration<br />

raised the average fuel efficiency standard<br />

to 35.5 miles per gallon (mpg) for cars and<br />

light trucks by the year 2016. The rules will cost<br />

consumers an average of $926 extra per vehicle in<br />

the 2016 model year. Assume Yolanda will purchase<br />

a new car in 2016 and plans to keep it for<br />

5 years. How much will the monthly savings in the<br />

cost of gasoline have to be to recover Yolanda’s<br />

extra cost? Use an interest rate of 0.75% per month.<br />

4.31 Fort Bliss, a U.S. Army military base, contributed<br />

$3.3 million of the $87 million capital cost for a<br />

desalting plant constructed and operated by El Paso<br />

Water Utilities (EPWU). In return, EPWU agreed<br />

to sell water to Fort Bliss at $0.85 per 1000 gallons<br />

for 20 years. If the army base uses 200 million gallons<br />

of water per month, what is the Army’s cost<br />

per month for water? The $3.3 million capital cost<br />

is amortized at an interest rate of 6% per year,<br />

compounded monthly.<br />

4.32 Beginning in 2011, city hall, administrative offices,<br />

and municipal courts in the city of El Paso,<br />

Texas, will go on a 10 hourday, 4-day workweek<br />

from the beginning of May through the end of September.<br />

The shortened workweek will affect 25%<br />

of the city’s 6100 employees and will save $42,600<br />

per month for those 5 months, because of reduced<br />

fuel, utility, and janitorial expenses. If this work<br />

schedule continues for the next 10 years, what is<br />

the future worth of the savings at the end of that<br />

time (i.e., end of year 2020)? Use an interest rate<br />

of 0.5% per month.<br />

4.33 In October 2009, Wal-Mart started selling caskets<br />

on its website that undercut many funeral homes.<br />

Prices ranged from $999 for steel models such as<br />

Dad Remembered to $3199 for the Sienna Bronze<br />

casket. Part of the business model is to get people<br />

to plan ahead, so the company is allowing people<br />

to pay for the caskets over a 12-month period with<br />

no interest. An individual purchased a Sienna<br />

Bronze casket and made 12 equal monthly payments<br />

(in months 1 through 12) at no interest. How<br />

much did this person save each month compared to<br />

another person who paid an interest rate of 6% per<br />

year, compounded monthly?<br />

4.34 NRG Energy plans to construct a giant solar plant<br />

in Santa Teresa, New Mexico, to supply electricity<br />

to West Texas Electric. The plant will have 390,000<br />

heliostats to concentrate sunlight onto 32 water<br />

towers to generate steam. It will provide enough<br />

power for 30,000 homes. If the company spends<br />

$28 million per month for 2 years in constructing<br />

the plant, how much will the company have to<br />

make each month in years 3 through 22 (that is,<br />

20 years) to recover its investment plus 18% per<br />

year, compounded monthly?<br />

4.35 Many college students have Visa credit cards that<br />

carry an interest rate of “simple 24% per year”<br />

(that is, 2% per month). When the balance on<br />

such a card is $5000, the minimum payment is<br />

$110.25.<br />

(a) What is the amount of interest in the first<br />

payment?<br />

(b) How long will it take, in months, to pay off<br />

the balance, if the cardholder continues to<br />

make payments of $110.25 per month and<br />

adds no other charges to the card?<br />

4.36 Bart is an engineering graduate who did not take<br />

the engineering economy elective during his B.S.<br />

degree course work. After working for a year or so,<br />

he found himself in financial trouble, and he borrowed<br />

$500 from a friend in the finance department<br />

at his office. Bart agreed to repay the loan<br />

principal plus $75 interest 1 month later. The two<br />

got separated doing different jobs, and 1 year went<br />

by. The friend e-mailed Bart after exactly 1 year<br />

and asked for the loan repayment plus the interest<br />

with monthly compounding, since Bart had made<br />

no effort to repay the loan during the year.<br />

( a ) What does Bart now owe his friend? ( b ) What<br />

effective annual interest rate did Bart pay on this<br />

$500 loan?<br />

4.37 AT&T announced that the early termination fee for<br />

smart phones will jump from $175 to $375. The<br />

fee will be reduced by $10 each month of the<br />

2-year contract. Wireless carriers justify the fees<br />

by pointing out that the cost of a new phone is<br />

heavily discounted from what the carrier pays the<br />

manufacturer. Assume AT&T pays $499 for an<br />

iPhone that it sells for $199. How much profit<br />

would the company have to make each month (i.e.,<br />

prior to the termination), if it wanted to make a rate<br />

of return of 1.5% per month on its $300 investment<br />

in a customer who terminates the contract<br />

after 12 months?<br />

4.38 What is the future worth of a present cost of<br />

$285,000 to Monsanto, Inc. 5 years from now at an<br />

interest rate of 2% per month?


Problems 121<br />

4.39 Environmental recovery company RexChem Partners<br />

plans to finance a site reclamation project<br />

that will require a 4-year cleanup period. The<br />

company plans to borrow $3.6 million now. How<br />

much will the company have to get in a lump-sum<br />

payment when the project is over in order to earn<br />

24% per year, compounded quarterly, on its investment?<br />

4.40 For the cash flows shown below, determine the<br />

equivalent uniform worth in years 1 through 5 at an<br />

interest rate of 18% per year, compounded monthly.<br />

Year 1 2 3 4 5<br />

Cash Flow, $ 200,000 0 350,000 0 400,000<br />

4.41 For the cash flow diagram shown, solve for F ,<br />

using an interest rate of 1% per month.<br />

0<br />

1<br />

i = 1% per month<br />

2 3 4 5 6 7 8<br />

$30 $30 $30 $30 $30 $30<br />

F =?<br />

$50 $50 $50<br />

Years<br />

4.42 According to the Government Accountability Office<br />

(GAO), if the U.S. Postal Service does not<br />

change its business model, it will lose $480 million<br />

next month and $500 million the month after that,<br />

and the losses will increase by $20 million per<br />

month for the next 10 years. At an interest rate of<br />

0.25% per month, what is the equivalent uniform<br />

amount per month of the losses through year 10?<br />

4.43 Equipment maintenance costs for manufacturing<br />

explosion-proof pressure switches are projected to<br />

be $100,000 in year 1 and increase by 4% each<br />

year through year 5. What is the present worth of<br />

the maintenance costs at an interest rate of 10%<br />

per year, compounded quarterly?<br />

Equivalence When PP CP<br />

4.44 If you deposit $1000 per month into an investment<br />

account that pays interest at a rate of 6% per year,<br />

compounded quarterly, how much will be in the<br />

account at the end of 5 years? There is no interperiod<br />

compounding.<br />

4.45 Freeport McMoran purchased two model MTVS<br />

peristaltic pumps (to inject sulfuric acid and King<br />

Lee antiscalant) for use at its nanofiltration water<br />

conditioning plant. The cost of the pumps was<br />

$950 each. If the chemical cost is $11 per day,<br />

determine the equivalent cost per month at an interest<br />

rate of 12% per year, compounded monthly.<br />

Assume 30 days per month and a 3-year pump<br />

life.<br />

4.46 Income from recycling paper and cardboard at the<br />

U.S. Army's Fort Benning Maneuver Center has<br />

averaged $3000 per month for 2½ years. What is<br />

the future worth of the income (after the 2½ years)<br />

at an interest rate of 6% per year, compounded<br />

quarterly? Assume there is no interperiod compounding.<br />

4.47 The Autocar E3 refuse truck has an energy recovery<br />

system developed by Parker Hannifin<br />

LLC that is expected to reduce fuel consumption<br />

by 50%. Pressurized fluid flows from carbon<br />

fi ber-reinforced accumulator tanks to two hydrostatic<br />

motors that propel the vehicle forward.<br />

(The truck recharges the accumulators when it<br />

brakes.) The fuel cost for a regular refuse truck is<br />

$900 per month. How much can a private wastehauling<br />

company afford to spend now on the<br />

recovery system, if it wants to recover its investment<br />

in 3 years plus a return of 14% per year,<br />

compounded semiannually? Assume no interperiod<br />

compounding.<br />

Continuous Compounding<br />

4.48 What effective interest rate per year is equal to<br />

1.2% per month, compounded continuously? Show<br />

hand and spreadsheet solutions.<br />

4.49 What effective interest rate per quarter is equal to<br />

a nominal 1.6% per month, compounded continuously?<br />

4.50 What nominal rate per month is equivalent to an<br />

effective 1.3% per month, compounded continuously?<br />

4.51 Companies such as GE that have huge amounts of<br />

cash flow every day base their financial calculations<br />

on continuous compounding. If the company wants<br />

to make an effective 25% per year, compounded<br />

continuously, what nominal daily rate of return has<br />

to be realized? Assume 365 days per year.<br />

4.52 U.S. Steel is planning a plant expansion that is expected<br />

to cost $13 million. How much money must<br />

the company set aside now in a lump-sum investment<br />

to have the money in 2 years? Capital funds<br />

earn interest at a rate of 12% per year, compounded<br />

continuously.


122 Chapter 4 Nominal and Effective Interest Rates<br />

4.53 Periodic outlays for inventory control software at<br />

Baron Chemicals are expected to be $150,000<br />

immediately, $200,000 in 1 year, and $350,000 in<br />

2 years. What is the present worth of the costs at<br />

an interest rate of 10% per year, compounded<br />

continuously?<br />

Varying Interest Rates<br />

4.54 Many small companies use accounts receivable as<br />

collateral to borrow money for continuing operations<br />

and meeting payrolls. If a company borrows<br />

$300,000 now at an interest rate of 1% per month,<br />

but the rate changes to 1.25% per month after<br />

4 months, how much will the company owe at the<br />

end of 1 year?<br />

4.55 The maintenance cost for furnaces at a copper<br />

smelting plant have been constant at $140,000 per<br />

year for the past 5 years. If the interest rate was 8%<br />

per year for the first 3 years and then it increased to<br />

10% in years 4 and 5, what is the equivalent future<br />

worth (in year 5) of the maintenance cost? Show<br />

hand and spreadsheet solutions.<br />

4.56 By filling carbon nanotubes with miniscule wires<br />

made of iron and iron carbide, incredibly thin<br />

nanowires can be extruded by blasting the carbon<br />

nanotubes with an electron beam. If Gentech Technologies<br />

plans to spend $1.7 million in year 1,<br />

$2.1 million in year 2, and $3.4 million in year 3 to<br />

develop the technology, determine the present<br />

worth of the investments in year 0, if the interest<br />

rate in year 1 is 10% and in years 2 and 3 it is 12%<br />

per year.<br />

4.57 Find ( a ) the present worth P , and ( b ) the equivalent<br />

uniform annual worth A for the cash flows<br />

shown below.<br />

P =?<br />

0<br />

1<br />

i = 10% i = 14%<br />

2 3 4 5 6 7 8<br />

$100 $100 $100 $100 $100<br />

$160 $160 $160<br />

Year<br />

ADDITIONAL PROBLEMS AND FE EXAM REVIEW QUESTIONS<br />

4.58 The term annual percentage rate is the same as:<br />

( c) 6.0% per quarter, compounded continuously<br />

(a) Effective rate<br />

(d) 9% per 6 months<br />

(b) Nominal rate<br />

(c) Annual percentage yield<br />

4.62 An effective 12.68% per year, compounded monthly,<br />

(d) All of the above<br />

is the closest to:<br />

(a) 12% per year<br />

4.59 An interest rate is an effective rate under all of the<br />

following conditions, except when:<br />

(b)<br />

(c)<br />

12% per year, compounded annually<br />

1% per month<br />

(a) The compounding period is not stated<br />

(d) 1% per month, compounded annually<br />

(b) The interest period and compounding period<br />

are the same<br />

(c) The interest statement says that the interest<br />

rate is effective<br />

(d) The interest period is shorter than the compounding<br />

period<br />

4.60 An interest rate of nominal 12% per year, compounded<br />

weekly, is:<br />

(a) An effective rate per year<br />

(b) An effective rate per week<br />

(c) A nominal rate per year<br />

(d) A nominal rate per week<br />

4.61 An interest rate of 1.5% per month, compounded<br />

continuously, is the same as:<br />

(a) An effective 1.5% per month<br />

( b) 4.5% per quarter, compounded continuously<br />

4.63 For an interest rate of 2% per month, the effective<br />

semiannual interest rate is:<br />

( a ) 2.02%<br />

( b ) 12.005%<br />

( c ) 12.31%<br />

( d ) 12.62%<br />

4.64 If you make quarterly deposits for 3 years into an<br />

account that compounds interest at 1% per month,<br />

the value of n in the F A factor that will determine<br />

F at the end of the 3-year period is:<br />

( a ) 3 ( b ) 12<br />

( c ) 36 ( d ) None of these


Additional Problems and FE Exam Review Questions 123<br />

4.65 Identify the following interest rates as nominal or<br />

effective.<br />

Rate 1: 1.5% per quarter<br />

Rate 2: 1.5% per quarter, compounded<br />

monthly<br />

( a) Both are nominal rates.<br />

( b) Rate 1 is nominal and rate 2 is effective.<br />

( c) Rate 1 is effective and rate 2 is nominal.<br />

( d) Both are effective.<br />

4.66 In solving uniform series problems, n in the standard<br />

factor notation equation is equal to the<br />

number of arrows (i.e., cash flows) in the original<br />

cash flow diagram when:<br />

( a) The payment period (PP) is longer than the<br />

compounding period (CP).<br />

( b) The compounding period is equal to the payment<br />

period.<br />

( c) Both ( a ) and ( b ) are correct.<br />

( d) The compounding period is longer than the<br />

payment period.<br />

4.67 An engineer who is saving for her retirement plans<br />

to deposit $500 every quarter, starting one quarter<br />

from now, into an investment account. If the account<br />

pays interest at 6% per year, compounded<br />

semiannually, the total she will have at the end of<br />

25 years is closest to:<br />

( a ) $50,000 ( b ) $56,400<br />

( c ) $79,700 ( d ) $112,800<br />

4.68 A company that makes flange-mount, motorized rotary<br />

potentiometers expects to spend $50,000 for a<br />

certain machine 4 years from now. At an interest rate<br />

of 12% per year, compounded quarterly, the present<br />

worth of the machine’s cost is represented by the following<br />

equation:<br />

( a) P 50,000( P F , 3%, 16)<br />

( b) P 50,000( P F , effective i 6 months, 8)<br />

( c) P 50,000( P F , effective i year, 4)<br />

( d) Any of the above<br />

4.69 For the cash flow diagram shown, the unit of the<br />

payment period (PP) is:<br />

( a ) Months ( b ) Quarters<br />

( c ) Semiannual ( d ) Years<br />

0<br />

1<br />

i = 1% per month<br />

2 3 4 5 6 7 8 9 10 11 12<br />

1 2<br />

$500<br />

$480<br />

$460<br />

$400 $400<br />

$420<br />

$440<br />

F =?<br />

3<br />

Quarter<br />

Year<br />

4.70 A small company plans to spend $10,000 in year 2<br />

and $10,000 in year 5. At an interest rate of effective<br />

10% per year, compounded semiannually, the equation<br />

that represents the equivalent annual worth A in<br />

years 1 through 5 is:<br />

( a) A 10,000( P F ,10%,2)(A P ,10%,5)<br />

10,000( A F ,10%,5)<br />

(b) A 10,000( A P ,10%,4) 10,000( A F ,10%,5)<br />

(c) A 10,000( P F ,5%,2)( A P ,5%,10)<br />

10,000( A F ,5%,10)<br />

(d) A [10,000( F P ,10%,5)<br />

10,000]( A F ,10%,5)<br />

4.71 Assume you make monthly deposits of $200 starting<br />

one month from now into an account that pays<br />

6% per year, compounded semiannually. If you<br />

want to know the total after 4 years , the value of n<br />

you should use in the F A factor is:<br />

( a ) 2 ( b ) 4 ( c ) 8 ( d ) 12<br />

4.72 The cost of replacing part of a cell phone videochip<br />

production line in 6 years is estimated to be<br />

$500,000. At an interest rate of 14% per year,<br />

compounded semiannually, the uniform amount<br />

that must be deposited into a sinking fund every<br />

6 months is closest to:<br />

( a ) $21,335 ( b ) $24,825<br />

( c ) $27,950 ( d ) $97,995


124 Chapter 4 Nominal and Effective Interest Rates<br />

CASE STUDY<br />

IS OWNING A HOME A NET GAIN OR NET LOSS OVER TIME?<br />

Background<br />

The Carroltons are deliberating whether to purchase a house<br />

or continue to rent for the next 10 years. They are assured by<br />

both of their employers that no transfers to new locations will<br />

occur for at least this number of years. Plus, the high school<br />

that their children attend is very good for their college prep<br />

education, and they all like the neighborhood where they<br />

live now.<br />

They have a total of $40,000 available now and estimate<br />

that they can afford up to $2850 per month for the total house<br />

payment.<br />

If the Carroltons do not buy a house, they will continue<br />

to rent the house they currently occupy for $2700 per<br />

month. They will also place the $40,000 into an investment<br />

instrument that is expected to earn at the rate of 6% per<br />

year. Additionally, they will add to this investment at the<br />

end of each year the same amount as the monthly 15-year<br />

mortgage payments. This alternative is called the rent–<br />

don’t buy plan.<br />

Information<br />

Two financing plans using fixed-rate mortgages are currently<br />

available. The details are as follows.<br />

Plan<br />

A<br />

B<br />

Description<br />

30-year fixed rate of 5.25% per year interest;<br />

10% down payment<br />

15-year fixed rate of 5.0% per year interest;<br />

10% down payment<br />

Other information:<br />

• Price of the house is $330,000 .<br />

• Taxes and insurance (T&I) are $500 per month.<br />

• Up-front fees (origination fee, survey fee, attorney’s fee,<br />

etc.) are $3000.<br />

Any money not spent on the down payment or monthly payment<br />

will be invested and return at a rate of 6% per year<br />

(0.5% per month).<br />

The Carroltons anticipate selling the house after 10 years<br />

and plan for a 10% increase in price, that is, $363,000 (after<br />

all selling expenses are paid)<br />

Case Study Exercises<br />

1. The 30-year fixed-rate mortgage (plan A) is analyzed<br />

below. No taxes are considered on proceeds from the<br />

savings or investments.<br />

Perform a similar analysis for the 15-year loan<br />

(plan B) and the rent–don’t buy plan. The Carroltons<br />

decided to use the largest future worth after 10 years to<br />

select the best of the plans. Do the analysis for them and<br />

select the best plan.<br />

Plan A analysis: 30-year fixed-rate loan<br />

Amount of money required for closing costs:<br />

Down payment (10% of $330,000) $33,000<br />

Up-front fees (origination fee, attorney’s<br />

fee, survey, filing fee, etc.) 3,000<br />

Total $36,000<br />

The amount of the loan is $297,000, and equivalent<br />

monthly principal and interest (P&I) is determined at<br />

5.25%12 0.4375% per month for 30(12) 360 months.<br />

A 297,000( A P ,0.4375%,360) 297,000(0.005522)<br />

$1640<br />

Add the T&I of $500 for a total monthly payment of<br />

Payment A $2140 per month<br />

The future worth of plan A is the sum of three future<br />

worth components: remainder of the $40,000 available<br />

for the closing costs ( F 1 A ); left-over money from that<br />

available for monthly payments ( F 2 A ); and increase in<br />

the house value when it is sold after 10 years ( F 3 A ).<br />

These are calculated here.<br />

F 1 A (40,000 36,000)( F P ,0.5%,120)<br />

$7278<br />

Money available each month to invest after the mortgage<br />

payment, and the future worth after 10 years is<br />

2850 2140 $710<br />

F 2 A 710( F A ,0.5%,120)<br />

$116,354


Case Study 125<br />

Net money from the sale in 10 years ( F 3 A ) is the difference<br />

between the net selling price ($363,000) and the<br />

remaining balance on the loan.<br />

Loan balance<br />

297,000( F P ,0.4375%,120)<br />

1640( F A ,0.4375%,120)<br />

297,000(1.6885) 1640(157.3770)<br />

$243,386<br />

F 3 A 363,000 243,386 $119,614<br />

Total future worth of plan A is<br />

F A F 1 A F 2 A F 3 A<br />

7278 116,354 119,614<br />

$243,246<br />

2. Perform this analysis if all estimates remain the same,<br />

except that when the house sells 10 years after purchase,<br />

the bottom has fallen out of the housing market and the<br />

net selling price is only 70% of the purchase price, that<br />

is, $231,000.


LEARNING STAGE 2<br />

Basic Analysis Tools<br />

LEARNING STAGE 2<br />

Basic Analysis<br />

Tools<br />

CHAPTER 5<br />

Present Worth<br />

Analysis<br />

CHAPTER 6<br />

Annual Worth<br />

Analysis<br />

CHAPTER 7<br />

Rate of Return<br />

Analysis: One Project<br />

CHAPTER 8<br />

Rate of Return<br />

Analysis: Multiple<br />

Alternatives<br />

CHAPTER 9<br />

BenefitCost Analysis<br />

and Public Sector<br />

Economics<br />

An engineering project or alternative is formulated to make or<br />

purchase a product, to develop a process, or to provide a<br />

service with specified results. An engineering economic<br />

analysis evaluates cash flow estimates for parameters such as initial<br />

cost, annual costs and revenues, nonrecurring costs, and possible<br />

salvage value over an estimated useful life of the product; process,<br />

or service. The chapters in this Learning Stage develop and demonstrate<br />

the basic tools and techniques to evaluate one or more alternatives<br />

using the factors, formulas, and spreadsheet functions<br />

learned in Stage 1.<br />

After completing these chapters, you will be able to evaluate<br />

most engineering project proposals using a well-accepted economic<br />

analysis technique, such as present worth, future worth, capitalized<br />

cost, life-cycle costing, annual worth, rate of return, or benefit /cost<br />

analysis.<br />

The epilogue to this stage provides an approach useful in selecting<br />

the engineering economic method that will provide the best<br />

analysis for the estimates and conditions present once the mutually<br />

exclusive alternatives are defined.<br />

Important note: If depreciation andor after-tax analysis is to be<br />

considered along with the evaluation methods in Chapters 5 through<br />

9, Chapter 16 and/or Chapter 17 should be covered, preferably after<br />

Chapter 6.


CHAPTER 5<br />

Present Worth<br />

Analysis<br />

LEARNING OUTCOMES<br />

Purpose: Utilize different present worth techniques to evaluate and select alternatives.<br />

SECTION TOPIC LEARNING OUTCOME<br />

5.1 Formulate alternatives • Identify mutually exclusive and independent<br />

projects; define revenue and cost alternatives.<br />

5.2 PW of equal-life alternatives • Select the best of equal-life alternatives using<br />

present worth analysis.<br />

5.3 PW of different-life alternatives • Select the best of different-life alternatives<br />

using present worth analysis.<br />

5.4 FW analysis • Select the best alternative using future worth<br />

analysis.<br />

5.5 CC analysis • Select the best alternative using capitalized cost<br />

(CC) analysis.


A<br />

future amount of money converted to its equivalent value now has a present worth<br />

(PW) that is always less than that of the future cash flow, because all P F factors have<br />

a value less than 1.0 for any interest rate greater than zero. For this reason, present<br />

worth values are often referred to as discounted cash flows (DCF), and the interest rate is referred<br />

to as the discount rate. Besides PW, two other terms frequently used are present value<br />

( PV ) and net present value ( NPV ). Up to this point, present worth computations have been<br />

made for one project or alternative. In this chapter, techniques for comparing two or more<br />

mutually exclusive alternatives by the present worth method are treated. Two additional applications<br />

are covered here—future worth and capitalized cost. Capitalized costs are used for<br />

projects with very long expected lives or long planning horizons.<br />

To understand how to organize an economic analysis, this chapter begins with a description<br />

of independent and mutually exclusive projects as well as revenue and cost<br />

alternatives.<br />

Water for Semiconductor Manufacturing<br />

Case: The worldwide contribution of<br />

semiconductor sales is about $250 billion<br />

per year, or about 10% of the world’s<br />

GDP (gross domestic product). This industry<br />

produces the microchips used in many<br />

of the communication, entertainment,<br />

transportation, and computing devices<br />

we use every day. Depending upon the<br />

type and size of fabrication plant (fab),<br />

the need for ultrapure water (UPW) to<br />

manufacture these tiny integrated circuits<br />

is high, ranging from 500 to 2000 gpm<br />

(gallons per minute). Ultrapure water is<br />

obtained by special processes that commonly<br />

include reverse osmosis deionizing<br />

resin bed technologies. Potable water<br />

obtained from purifying seawater or<br />

brackish groundwater may cost from<br />

$2 to $3 per 1000 gallons, but to obtain<br />

UPW on-site for semiconductor manufacturing<br />

may cost an additional $1 to $3 per<br />

1000 gallons.<br />

A fab costs upward of $2.5 billion to<br />

construct, with approximately 1% of this<br />

total, or $25 million, required to provide<br />

the ultrapure water needed, including<br />

the necessary wastewater and recycling<br />

equipment.<br />

A newcomer to the industry, Angular<br />

Enterprises, has estimated the cost profiles<br />

for two options to supply its anticipated<br />

fab with water. It is fortunate to<br />

PE<br />

have the option of desalinated seawater<br />

or purified groundwater sources in the<br />

location chosen for its new fab. The initial<br />

cost estimates for the UPW system are<br />

given below.<br />

Source<br />

Equipment first<br />

cost, $M<br />

Seawater<br />

(S)<br />

20<br />

Groundwater<br />

(G)<br />

22<br />

AOC, $M per year 0.5 0.3<br />

Salvage value, % of<br />

first cost<br />

Cost of UPW, $ per<br />

1000 gallons<br />

5 10<br />

4 5<br />

Angular has made some initial estimates<br />

for the UPW system.<br />

Life of UPW equipment 10 years<br />

UPW needs<br />

Operating time<br />

1500 gpm<br />

16 hours per<br />

day for 250 days<br />

per year<br />

This case is used in the following topics<br />

(Sections) and problems of this chapter:<br />

PW analysis of equal-life alternatives<br />

(Section 5.2)<br />

PW analysis of different-life alternatives<br />

(Section 5.3)<br />

Capitalized cost analysis (Section 5.5)<br />

Problems 5.20 and 5.34<br />

5.1 Formulating Alternatives<br />

The evaluation and selection of economic proposals require cash flow estimates over a stated<br />

period of time, mathematical techniques to calculate the measure of worth (review Example<br />

1.2 for possible measures), and a guideline for selecting the best proposal. From all the


130 Chapter 5 Present Worth Analysis<br />

Mandates<br />

Ideas<br />

Information<br />

Experience<br />

Estimates<br />

Plans<br />

Proposals<br />

Not viable<br />

Viable<br />

Not viable<br />

A<br />

B<br />

C<br />

1<br />

…<br />

m<br />

2<br />

D<br />

E<br />

Mutually<br />

exclusive<br />

alternatives<br />

Select<br />

only<br />

one<br />

1<br />

2<br />

…<br />

m<br />

+<br />

DN<br />

Either<br />

of these<br />

Nature<br />

of proposals<br />

Independent<br />

projects<br />

Select<br />

all<br />

justified<br />

DN<br />

1<br />

2<br />

…<br />

m<br />

DN = do nothing<br />

Types of cash flow estimates<br />

* Revenues and costs<br />

* Costs only<br />

Revenue alternative<br />

Cost alternative<br />

Perform evaluation and make selection<br />

Figure 5–1<br />

Progression from proposals to economic evaluation to selection.<br />

proposals that may accomplish a stated purpose, the alternatives are formulated. This progression<br />

is detailed in Figure 5–1. Up front, some proposals are viable from technological, economic,<br />

andor legal perspectives; others are not viable. Once the obviously nonviable ideas<br />

are eliminated, the remaining viable proposals are fleshed out to form the alternatives to be<br />

evaluated. Economic evaluation is one of the primary means used to select the best<br />

alternative(s) for implementation.<br />

The nature of the economic proposals is always one of two types:<br />

Mutually exclusive alternatives: Only one of the proposals can be selected. For terminology<br />

purposes, each viable proposal is called an alternative.<br />

Independent projects: More than one proposal can be selected. Each viable proposal is called a<br />

project.<br />

The do-nothing (DN) proposal is usually understood to be an option when the evaluation is<br />

performed.


5.2 Present Worth Analysis of Equal-Life Alternatives 131<br />

The DN alternative or project means that the current approach is maintained; nothing new<br />

is initiated. No new costs, revenues, or savings are generated.<br />

Do nothing<br />

If it is absolutely required that one or more of the defined alternatives be selected, do nothing is<br />

not considered. This may occur when a mandated function must be installed for safety, legal,<br />

government, or other purposes.<br />

Mutually exclusive alternatives and independent projects are selected in completely different<br />

ways. A mutually exclusive selection takes place, for example, when an engineer must select<br />

the best diesel-powered engine from several available models. Only one is chosen, and the<br />

rest are rejected. If none of the alternatives are economically justified, then all can be rejected<br />

and, by default, the DN alternative is selected. For independent projects one, two or more, in<br />

fact, all of the projects that are economically justified can be accepted, provided capital funds<br />

are available. This leads to the two following fundamentally different evaluation bases:<br />

Mutually exclusive alternatives compete with one another and are compared pairwise.<br />

Independent projects are evaluated one at a time and compete only with the DN project.<br />

Any of the techniques in Chapters 5 through 9 can be used to evaluate either type of proposal—<br />

mutually exclusive or independent. When performed correctly as described in each chapter, any<br />

of the techniques will reach the same conclusion of which alternative or alternatives to select.<br />

This chapter covers the present worth method.<br />

A parallel can be developed between independent and mutually exclusive evaluation. Assume<br />

there are m independent projects. Zero, one, two, or more may be selected. Since each project may<br />

be in or out of the selected group of projects, there are a total of 2 m mutually exclusive alternatives.<br />

This number includes the DN alternative, as shown in Figure 5–1. For example, if the engineer<br />

has three diesel engine models (A, B, and C) and may select any number of them, there are<br />

2 3 8 alternatives: DN, A, B, C, AB, AC, BC, ABC. Commonly, in real-world applications, there<br />

are restrictions, such as an upper budgetary limit, that eliminate many of the 2 m alternatives. Independent<br />

project analysis without budget limits is discussed in this chapter and through Chapter 9.<br />

Chapter 12 treats independent projects with a budget limitation; this is called capital budgeting.<br />

Finally, it is important to recognize the nature of the cash flow estimates before starting the<br />

computation of a measure of worth that leads to the final selection. Cash flow estimates determine<br />

whether the alternatives are revenue- or cost-based. All the alternatives or projects must be<br />

of the same type when the economic study is performed. Definitions for these types follow:<br />

Revenue: Each alternative generates cost (cash outflow) and revenue (cash inflow) estimates,<br />

and possibly savings, also considered cash inflows. Revenues can vary for each alternative.<br />

Cost: Each alternative has only cost cash flow estimates. Revenues or savings are assumed<br />

equal for all alternatives; thus they are not dependent upon the alternative selected. These are<br />

also referred to as service alternatives.<br />

Revenue or cost<br />

alternative<br />

Although the exact procedures vary slightly for revenue and cost cash flows, all techniques and<br />

guidelines covered through Chapter 9 apply to both. Differences in evaluation methodology are<br />

detailed in each chapter.<br />

5.2 Present Worth Analysis of Equal-Life Alternatives<br />

The PW comparison of alternatives with equal lives is straightforward. The present worth P is<br />

renamed PW of the alternative. The present worth method is quite popular in industry because all<br />

future costs and revenues are transformed to equivalent monetary units NOW; that is, all future<br />

cash flows are converted (discounted) to present amounts (e.g., dollars) at a specific rate of return,<br />

which is the MARR. This makes it very simple to determine which alternative has the best economic<br />

advantage. The required conditions and evaluation procedure are as follows:<br />

If the alternatives have the same capacities for the same time period (life), the equal-service<br />

requirement is met. Calculate the PW value at the stated MARR for each alternative.<br />

Equal-service<br />

requirement


132 Chapter 5 Present Worth Analysis<br />

For mutually exclusive (ME) alternatives, whether they are revenue or cost alternatives, the following<br />

guidelines are applied to justify a single project or to select one from several alternatives.<br />

Project evaluation<br />

ME alternative<br />

selection<br />

One alternative: If PW 0, the requested MARR is met or exceeded and the alternative is<br />

economically justified.<br />

Two or more alternatives: Select the alternative with the PW that is numerically largest,<br />

that is, less negative or more positive. This indicates a lower PW of cost for cost alternatives<br />

or a larger PW of net cash flows for revenue alternatives.<br />

Note that the guideline to select one alternative with the lowest cost or highest revenue uses<br />

the criterion of numerically largest. This is not the absolute value of the PW amount, because<br />

the sign matters. The selections below correctly apply the guideline for two alternatives A and B.<br />

PW A PW B Selected Alternative<br />

$2300 $1500 B<br />

500 1000 B<br />

2500 2000 A<br />

4800 400 A<br />

For independent projects, each PW is considered separately, that is, compared with the DN<br />

project, which always has PW 0. The selection guideline is as follows:<br />

Independent project<br />

selection<br />

One or more independent projects: Select all projects with PW 0 at the MARR.<br />

The independent projects must have positive and negative cash flows to obtain a PW value that<br />

can exceed zero; that is, they must be revenue projects.<br />

All PW analyses require a MARR for use as the i value in the PW relations. The bases used<br />

to establish a realistic MARR were summarized in Chapter 1 and are discussed in detail in<br />

Chapter 10.<br />

EXAMPLE 5.1<br />

A university lab is a research contractor to NASA for in-space fuel cell systems that are hydrogenand<br />

methanol-based. During lab research, three equal-service machines need to be evaluated<br />

economically. Perform the present worth analysis with the costs shown below. The MARR is<br />

10% per year.<br />

Electric-Powered Gas-Powered Solar-Powered<br />

First cost, $ 4500 3500 6000<br />

Annual operating cost (AOC), $/year 900 700 50<br />

Salvage value S, $ 200 350 100<br />

Life, years 8 8 8<br />

Solution<br />

These are cost alternatives. The salvage values are considered a “negative” cost, so a sign<br />

precedes them. (If it costs money to dispose of an asset, the estimated disposal cost has a sign.)<br />

The PW of each machine is calculated at i 10% for n 8 years. Use subscripts E , G , and S .<br />

PW E 4500 900( P A ,10%,8) 200( P F ,10%,8) $9208<br />

PW G 3500 700( P A ,10%,8) 350( P F ,10%,8) $7071<br />

PW S 6000 50( P A ,10%,8) 100( P F ,10%,8) $6220<br />

The solar-powered machine is selected since the PW of its costs is the lowest; it has the<br />

numerically largest PW value.


5.3 Present Worth Analysis of Different-Life Alternatives 133<br />

EXAMPLE 5.2 Water for Semiconductor Manufacturing Case<br />

As discussed in the introduction to this chapter, ultrapure water (UPW) is an expensive commodity<br />

for the semiconductor industry. With the options of seawater or groundwater sources,<br />

it is a good idea to determine if one system is more economical than the other. Use a MARR of<br />

12% per year and the present worth method to select one of the systems.<br />

PE<br />

Solution<br />

An important first calculation is the cost of UPW per year. The general relation and estimated<br />

costs for the two options are as follows:<br />

$<br />

UPW cost relation: ——<br />

year cost in $<br />

——————<br />

(<br />

———<br />

1000 gallons ) ( gallons ————<br />

minute ) ( minutes<br />

hour<br />

Seawater: (41000)(1500)(60)(16)(250) $1.44 M per year<br />

Groundwater: (51000)(1500)(60)(16)(250) $1.80 M per year<br />

) ( hours ———<br />

day ) (——<br />

days<br />

year )<br />

Calculate the PW at i 12% per year and select the option with the lower cost (larger PW<br />

value). In $1 million units:<br />

PW relation: PW first cost PW of AOC PW of UPW PW of salvage value<br />

PW S 20 0.5(PA,12%,10) 1.44(PA,12%,10) 0.05(20)(PF,12%,10)<br />

20 0.5(5.6502) 1.44(5.6502) 1(0.3220)<br />

$30.64<br />

PW G 22 0.3(PA,12%,10) 1.80(PA,12%,10) 0.10(22)(PF,12%,10)<br />

22 0.3(5.6502) 1.80(5.6502) 2.2(0.3220)<br />

$33.16<br />

Based on this present worth analysis, the seawater option is cheaper by $2.52 M.<br />

5.3 Present Worth Analysis of Different-Life<br />

Alternatives<br />

When the present worth method is used to compare mutually exclusive alternatives that have<br />

different lives, the equal-service requirement must be met. The procedure of Section 5.2 is followed,<br />

with one exception:<br />

The PW of the alternatives must be compared over the same number of years and must end<br />

at the same time to satisfy the equal-service requirement.<br />

This is necessary, since the present worth comparison involves calculating the equivalent PW of<br />

all future cash flows for each alternative. A fair comparison requires that PW values represent<br />

cash flows associated with equal service. For cost alternatives, failure to compare equal service<br />

will always favor the shorter-lived mutually exclusive alternative, even if it is not the more economical<br />

choice, because fewer periods of costs are involved. The equal-service requirement is<br />

satisfied by using either of two approaches:<br />

LCM: Compare the PW of alternatives over a period of time equal to the least common<br />

multiple (LCM) of their estimated lives.<br />

Study period: Compare the PW of alternatives using a specified study period of n years.<br />

This approach does not necessarily consider the useful life of an alternative. The study period<br />

is also called the planning horizon.<br />

Equal-service<br />

requirement<br />

LCM or study<br />

period<br />

For either approach, calculate the PW at the MARR and use the same selection guideline as<br />

that for equal-life alternatives. The LCM approach makes the cash flow estimates extend to the<br />

same period, as required. For example, lives of 3 and 4 years are compared over a 12-year period.


134 Chapter 5 Present Worth Analysis<br />

The first cost of an alternative is reinvested at the beginning of each life cycle, and the estimated<br />

salvage value is accounted for at the end of each life cycle when calculating the PW values over<br />

the LCM period. Additionally, the LCM approach requires that some assumptions be made about<br />

subsequent life cycles.<br />

The assumptions when using the LCM approach are that<br />

1. The service provided will be needed over the entire LCM years or more.<br />

2. The selected alternative can be repeated over each life cycle of the LCM in exactly the same<br />

manner.<br />

3. Cash flow estimates are the same for each life cycle.<br />

As will be shown in Chapter 14, the third assumption is valid only when the cash flows are<br />

expected to change by exactly the inflation (or deflation) rate that is applicable through the LCM<br />

time period. If the cash flows are expected to change by any other rate, then the PW analysis must<br />

be conducted using constant-value dollars, which considers inflation (Chapter 14).<br />

A study period analysis is necessary if the first assumption about the length of time the alternatives<br />

are needed cannot be made. For the study period approach, a time horizon is chosen over<br />

which the economic analysis is conducted, and only those cash flows which occur during that time<br />

period are considered relevant to the analysis. All cash flows occurring beyond the study period are<br />

ignored. An estimated market value at the end of the study period must be made. The time horizon<br />

chosen might be relatively short, especially when short-term business goals are very important. The<br />

study period approach is often used in replacement analysis (Chapter 11). It is also useful when the<br />

LCM of alternatives yields an unrealistic evaluation period, for example, 5 and 9 years.<br />

EXAMPLE 5.3<br />

National Homebuilders, Inc., plans to purchase new cut-and-finish equipment. Two manufacturers<br />

offered the estimates below.<br />

Vendor A<br />

Vendor B<br />

First cost, $ 15,000 18,000<br />

Annual M&O cost, $ per year 3,500 3,100<br />

Salvage value, $ 1,000 2,000<br />

Life, years 6 9<br />

(a) Determine which vendor should be selected on the basis of a present worth comparison,<br />

if the MARR is 15% per year.<br />

(b) National Homebuilders has a standard practice of evaluating all options over a 5-year<br />

period. If a study period of 5 years is used and the salvage values are not expected to<br />

change, which vendor should be selected?<br />

Solution<br />

(a) Since the equipment has different lives, compare them over the LCM of 18 years. For life<br />

cycles after the first, the first cost is repeated in year 0 of each new cycle, which is the last<br />

year of the previous cycle. These are years 6 and 12 for vendor A and year 9 for B. The<br />

cash flow diagram is shown in Figure 5–2. Calculate PW at 15% over 18 years.<br />

PW A 15,000 15,000(PF,15%,6) 1000(PF,15%,6)<br />

15,000(PF,15%,12) 1000(PF,15%,12) 1000(PF,15%,18)<br />

3,500(PA,15%,18)<br />

$45,036<br />

PW B 18,000 18,000(PF,15%,9) 2000(PF,15%,9)<br />

2000(PF,15%,18) 3100(PA,15%,18)<br />

$41,384


5.3 Present Worth Analysis of Different-Life Alternatives 135<br />

PW A =?<br />

$1000 $1000<br />

$1000<br />

1 2 6 12 16 17 18<br />

Year<br />

$3500<br />

$15,000<br />

$15,000<br />

Vendor A<br />

$15,000<br />

PW B =?<br />

$2000<br />

$2000<br />

1 2 9<br />

16 17 18<br />

Year<br />

$3100<br />

$18,000<br />

$18,000<br />

Vendor B<br />

Figure 5–2<br />

Cash flow diagram for different-life alternatives, Example 5.3 a .<br />

Vendor B is selected, since it costs less in PW terms; that is, the PW B value is numerically<br />

larger than PW A .<br />

(b) For a 5-year study period, no cycle repeats are necessary. The PW analysis is<br />

PW A 15,000 3500(PA,15%,5) 1000(PF,15%,5)<br />

$26,236<br />

PW B 18,000 3100(PA,15%,5) 2000(PF,15%,5)<br />

$27,397<br />

Vendor A is now selected based on its smaller PW value. This means that the shortened<br />

study period of 5 years has caused a switch in the economic decision. In situations such as<br />

this, the standard practice of using a fixed study period should be carefully examined to<br />

ensure that the appropriate approach, that is, LCM or fixed study period, is used to satisfy<br />

the equal-service requirement.<br />

EXAMPLE 5.4 Water for Semiconductor Manufacturing Case<br />

PE<br />

When we discussed this case in the introduction, we learned that the initial estimates of equipment<br />

life were 10 years for both options of UPW (ultrapure water)—seawater and groundwater.<br />

As you might guess, a little research indicates that seawater is more corrosive and the equipment<br />

life is shorter—5 years rather than 10. However, it is expected that, instead of complete replacement,<br />

a total refurbishment of the equipment for $10 M after 5 years will extend the life through<br />

the anticipated 10 th year of service.<br />

With all other estimates remaining the same, it is important to determine if this 50%<br />

reduction in expected usable life and the refurbishment expense may alter the decision to go<br />

with the seawater option, as determined in Example 5.2. For a complete analysis, consider<br />

both a 10-year and a 5-year option for the expected use of the equipment, regardless of the<br />

source of UPW.


136 Chapter 5 Present Worth Analysis<br />

Seawater option<br />

OC UPW cost/year<br />

0.5 1.44<br />

Groundwater option<br />

OC UPW cost/year<br />

0.3 1.80<br />

Both options<br />

Salvage values<br />

included here<br />

Salvage values are the<br />

same after 5 and 10 years<br />

Figure 5–3<br />

PW analyses using LCM and study period approaches for water for semiconductor manufacturing case,<br />

Example 5.4.<br />

Solution<br />

A spreadsheet and the NPV function are a quick and easy way to perform this dual analysis.<br />

The details are presented in Figure 5–3.<br />

LCM of 10 years: In the top part of the spreadsheet, the LCM of 10 years is necessary to satisfy<br />

the equal-service requirement; however, the first cost in year 5 is the refurbishment cost of<br />

$10 M, not the $20 M expended in year 0. Each year’s cash flow is entered in consecutive<br />

cells; the $11.94 M in year 5 accounts for the continuing AOC and annual UPW cost of<br />

$1.94 M, plus the $10 M refurbishment cost. The NPV functions shown on the spreadsheet<br />

determine the 12% per year PW values in $1 million units.<br />

PW S $36.31<br />

PW G $33.16<br />

Now, the groundwater option is cheaper ; the economic decision is reversed with this new<br />

e stimate of life and year 5 refurbishment expense.<br />

Study period of 5 years: The lower portion of Figure 5–3 details a PW analysis using the<br />

second approach to evaluating different-life alternatives, that is, a specific study period, which<br />

is 5 years in this case study. Therefore, all cash flows after 5 years are neglected.<br />

Again the economic decision is reversed as the 12% per year PW values favor the seawater<br />

option.<br />

PW S $26.43<br />

PW G $28.32<br />

Comments<br />

The decision switched between the LCM and study period approaches. Both are correct answers<br />

given the decision of how the equal-service requirement is met. This analysis demonstrates<br />

how important it is to compare mutually exclusive alternatives over time periods that<br />

are believable and to take the time necessary to make the most accurate cost, life, and MARR<br />

estimates when the evaluation is performed.


5.4 Future Worth Analysis 137<br />

If the PW evaluation is incorrectly performed using the respective lives of the two options,<br />

the equal-service requirement is violated, and PW values favor the shorter-lived option, that is,<br />

seawater. The PW values are<br />

Option S: n 5 years, PW S $26.43 M, from the bottom left calculation in Figure 5–3.<br />

Option G: n 10 years, PW G $33.16 M, from the top right calculation in Figure 5–3.<br />

For independent projects , use of the LCM approach is unnecessary since each project is compared<br />

to the do-nothing alternative, not to each other, and satisfying the equal-service requirement<br />

is not a problem. Simply use the MARR to determine the PW over the respective life of<br />

each project, and select all projects with a PW 0.<br />

5.4 Future Worth Analysis<br />

The future worth (FW) of an alternative may be determined directly from the cash flows, or by<br />

multiplying the PW value by the F P factor, at the established MARR. The n value in the F P<br />

factor is either the LCM value or a specified study period. Analysis of alternatives using FW<br />

values is especially applicable to large capital investment decisions when a prime goal is to<br />

maximize the future wealth of a corporation’s stockholders.<br />

Future worth analysis over a specified study period is often utilized if the asset (equipment, a<br />

building, etc.) might be sold or traded at some time before the expected life is reached. Suppose<br />

an entrepreneur is planning to buy a company and expects to trade it within 3 years. FW analysis<br />

is the best method to help with the decision to sell or keep it 3 years hence. Example 5.5 illustrates<br />

this use of FW analysis. Another excellent application of FW analysis is for projects that will come<br />

online at the end of a multiyear investment period, such as electric generation facilities, toll roads,<br />

airports, and the like. They are analyzed using the FW value of investment commitments made<br />

during construction.<br />

The selection guidelines for FW analysis are the same as for PW analysis; FW 0 means the<br />

MARR is met or exceeded. For two or more mutually exclusive alternatives, select the one<br />

with the numerically largest FW value.<br />

ME alternative<br />

selection<br />

EXAMPLE 5.5<br />

A British food distribution conglomerate purchased a Canadian food store chain for £75 million<br />

3 years ago. There was a net loss of £10 million at the end of year 1 of ownership. Net cash<br />

flow is increasing with an arithmetic gradient of £5 million per year starting the second year,<br />

and this pattern is expected to continue for the foreseeable future. This means that breakeven<br />

net cash flow was achieved this year. Because of the heavy debt financing used to purchase the<br />

Canadian chain, the international board of directors expects a MARR of 25% per year from<br />

any sale.<br />

(a) The British conglomerate has just been offered £159.5 million by a French company<br />

wishing to get a foothold in Canada. Use FW analysis to determine if the MARR will be<br />

realized at this selling price.<br />

(b) If the British conglomerate continues to own the chain, what selling price must be obtained<br />

at the end of 5 years of ownership to just make the MARR?<br />

Solution<br />

(a) Set up the future worth relation in year 3 (FW 3 ) at i 25% per year and an offer price of<br />

£159.5 million. Figure 5–4 a presents the cash flow diagram in million £ units.<br />

FW 3 75(FP,25%,3) 10(FP,25%,2) 5(FP,25%,1) 159.5<br />

168.36 159.5 £8.86 million<br />

No, the MARR of 25% will not be realized if the £159.5 million offer is accepted.


138 Chapter 5 Present Worth Analysis<br />

£159.5<br />

FW = ?<br />

i 25%?<br />

i = 25%<br />

£5<br />

£10<br />

0 1 2 3<br />

Year<br />

0 1 2 3 4 5<br />

Year<br />

£10<br />

£5<br />

£10<br />

£5<br />

£75<br />

(a)<br />

£75<br />

Figure 5–4<br />

Cash flow diagrams for Example 5.5. (a) i ?; (b) FW ?.<br />

(b)<br />

(b) Determine the future worth 5 years from now at 25% per year. Figure 5–4 b presents the<br />

cash flow diagram. The A G and F A factors are applied to the arithmetic gradient.<br />

FW 5 75(FP,25%,5) 10(FA,25%,5) 5(AG,25%,5)(FA,25%,5)<br />

£246.81 million<br />

The offer must be for at least £246.81 million to make the MARR. This is approximately<br />

3.3 times the purchase price only 5 years earlier, in large part based on the required<br />

MARR of 25%.<br />

5.5 Capitalized Cost Analysis<br />

Many public sector projects such as bridges, dams, highways and toll roads, railroads, and hydroelectric<br />

and other power generation facilities have very long expected useful lives. A perpetual<br />

or infinite life is the effective planning horizon. Permanent endowments for charitable organizations<br />

and universities also have perpetual lives. The economic worth of these types of projects or<br />

endowments is evaluated using the present worth of the cash flows.<br />

Capitalized Cost (CC) is the present worth of a project that has a very long life (more than, say,<br />

35 or 40 years) or when the planning horizon is considered very long or infinite.<br />

The formula to calculate CC is derived from the PW relation P A ( P A , i %, n ), where n <br />

time periods. Take the equation for P using the P A factor and divide the numerator and denominator<br />

by (1 i ) to obtain<br />

n<br />

P A<br />

[ 1 ———— 1<br />

(1 i) n<br />

————— ]<br />

i<br />

As n approaches , the bracketed term becomes 1 i . We replace the symbols P and PW with<br />

CC as a reminder that this is a capitalized cost equivalence. Since the A value can also be termed<br />

AW for annual worth, the capitalized cost formula is simply<br />

CC A —<br />

i<br />

or<br />

CC AW ——<br />

i<br />

[5.1]<br />

Solving for A or AW, the amount of new money that is generated each year by a capitalization<br />

of an amount CC is<br />

AW CC (i) [5.2]<br />

This is the same as the calculation A P (i ) for an infinite number of time periods. Equation [5.2]<br />

can be explained by considering the time value of money. If $20,000 is invested now (this is the


5.5 Capitalized Cost Analysis 139<br />

capitalization) at 10% per year, the maximum amount of money that can be withdrawn at the end<br />

of every year for eternity is $2000, which is the interest accumulated each year. This leaves the<br />

original $20,000 to earn interest so that another $2000 will be accumulated the next year.<br />

The cash flows (costs, revenues, and savings) in a capitalized cost calculation are usually of<br />

two types: recurring, also called periodic, and nonrecurring. An annual operating cost of $50,000<br />

and a rework cost estimated at $40,000 every 12 years are examples of recurring cash flows.<br />

Examples of nonrecurring cash flows are the initial investment amount in year 0 and one-time<br />

cash flow estimates at future times, for example, $500,000 in fees 2 years hence.<br />

The procedure to determine the CC for an infinite sequence of cash flows is as follows:<br />

1. Draw a cash flow diagram showing all nonrecurring (one-time) cash flows and at least two<br />

cycles of all recurring (periodic) cash flows.<br />

2. Find the present worth of all nonrecurring amounts. This is their CC value.<br />

3. Find the A value through one life cycle of all recurring amounts. (This is the same value in<br />

all succeeding life cycles, as explained in Chapter 6.) Add this to all other uniform<br />

amounts (A) occurring in years 1 through infinity. The result is the total equivalent uniform<br />

annual worth (AW).<br />

4. Divide the AW obtained in step 3 by the interest rate i to obtain a CC value. This is an<br />

application of Equation [5.1].<br />

5. Add the CC values obtained in steps 2 and 4.<br />

Drawing the cash flow diagram (step 1) is more important in CC calculations than elsewhere,<br />

because it helps separate nonrecurring and recurring amounts. In step 5 the present worths of all<br />

component cash flows have been obtained; the total capitalized cost is simply their sum.<br />

EXAMPLE 5.6<br />

The Haverty County Transportation Authority (HCTA) has just installed new software to charge<br />

and track toll fees. The director wants to know the total equivalent cost of all future costs incurred<br />

to purchase the software system. If the new system will be used for the indefinite future,<br />

find the equivalent cost ( a ) now, a CC value, and ( b ) for each year hereafter, an AW value.<br />

The system has an installed cost of $150,000 and an additional cost of $50,000 after<br />

10 years. The annual software maintenance contract cost is $5000 for the first 4 years and<br />

$8000 thereafter. In addition, there is expected to be a recurring major upgrade cost of $15,000<br />

every 13 years. Assume that i 5% per year for county funds.<br />

Solution<br />

(a) The five-step procedure to find CC now is applied.<br />

1. Draw a cash flow diagram for two cycles (Figure 5–5).<br />

2. Find the present worth of the nonrecurring costs of $150,000 now and $50,000 in year<br />

10 at i 5%. Label this CC 1 .<br />

CC 1 150,000 50,000(PF,5%,10) $180,695<br />

0 2 4 6 8 10 12 14 20 26<br />

Year<br />

$5000<br />

$8000<br />

i = 5% per year<br />

$50,000<br />

$15,000 $15,000<br />

$150,000<br />

Figure 5–5<br />

Cash flows for two cycles of recurring costs and all nonrecurring amounts, Example 5.6.


140 Chapter 5 Present Worth Analysis<br />

3 and 4. Convert the $15,000 recurring cost to an A value over the first cycle of 13 years, and<br />

find the capitalized cost CC 2 at 5% per year using Equation [5.1].<br />

A 15,000(AF,5%,13) $847<br />

CC 2 8470.05 $16,940<br />

There are several ways to convert the annual software maintenance cost series to A<br />

and CC values. A straightforward method is to, first, consider the $−5000 an A series<br />

with a capitalized cost of<br />

CC 3 50000.05 $100,000<br />

Second, convert the step-up maintenance cost series of $−3000 to a capitalized cost<br />

CC 4 in year 4, and find the present worth in year 0. (Refer to Figure 5–5 for cash flow<br />

timings.)<br />

CC 4 ————<br />

3,000 (PF,5%,4) $49,362<br />

0.05<br />

5. The total capitalized cost CC T for Haverty County Transportation Authority is the sum<br />

of the four component CC values.<br />

CC T 180,695 16,940 100,000 49,362<br />

$346,997<br />

(b) Equation [5.2] determines the AW value forever.<br />

AW Pi CC T (i) $346,997(0.05) $17,350<br />

Correctly interpreted, this means Haverty County officials have committed the equivalent<br />

of $17,350 forever to operate and maintain the toll management software.<br />

For the comparison of two alternatives on the basis of capitalized cost, use the procedure<br />

above to find the A value and CC T for each alternative. Since the capitalized cost represents the<br />

total present worth of financing and maintaining a given alternative forever, the alternatives will<br />

automatically be compared for the same number of years (i.e., infinity). The alternative with the<br />

smaller capitalized cost will represent the more economical one. This evaluation is illustrated in<br />

Example 5.7 using the progressive example for this chapter.<br />

EXAMPLE 5.7 Water for Semiconductor Manufacturing Case<br />

Our case study has progressed (in Example 5.4) to the point that the life of the seawater option<br />

can be extended to 10 years with a major refurbishment cost after 5 years. This extension is<br />

possible only one time, after which a new life cycle would commence. In $1 million units, the<br />

estimates and PW values (from Figure 5–3) are as follows:<br />

Seawater: P S $20; AOC S $1.94; n S 10 years; refurbishment, year 5 $10;<br />

S S 0.05(20) $1.00; PW S $36.31<br />

Groundwater: P G $22; AOC G $2.10; n G 10 years; S G 0.10(22) $2.2;<br />

PW G $33.16<br />

PE<br />

If we assume that the UPW (ultrapure water) requirement will continue for the foreseeable<br />

future, a good number to know is the present worth of the long-term options at the selected<br />

MARR of 12% per year. What are these capitalized costs for the two options using the estimates<br />

made thus far?<br />

Solution<br />

Find the equivalent A value for each option over its respective life, then determine the CC<br />

value using the relation CC A i . Select the option with the lower CC. This approach satisfies<br />

the equal-service requirement because the time horizon is infinity when the CC is determined.


5.5 Capitalized Cost Analysis 141<br />

Seawater: A S PW S (AP,12%,10) 36.31(0.17698) $6.43<br />

CC S 6.430.12 $53.58<br />

Groundwater: A G PW G (AP,12%,10) 33.16(0.17698) $5.87<br />

CC G 5.870.12 $48.91<br />

In terms of capitalized cost, the groundwater alternative is cheaper.<br />

Comment<br />

If the seawater-life extension is not considered a viable option, the original alternative of<br />

5 years could be used in this analysis. In this case, the equivalent A value and CC computations<br />

in $1 million units are as follows.<br />

A S,5 years 20(AP,12%,5) 1.94 0.05(20)(AF,12%,5)<br />

$7.33<br />

CC S, 5 years 7.33/0.12 $61.08<br />

Now, the economic advantage of the groundwater option is even larger.<br />

If a finite-life alternative (for example, 5 years) is compared to one with an indefinite or very<br />

long life, capitalized costs can be used. To determine capitalized cost for the finite life alternative,<br />

calculate the equivalent A value for one life cycle and divide by the interest rate (Equation [5.1]).<br />

This procedure is illustrated in Example 5.8 using a spreadsheet.<br />

EXAMPLE 5.8<br />

The State Legislature has mandated a statewide recycling program to include all types of<br />

plastic, paper, metal, and glass refuse. The goal is zero landfill by 2020. Two options for the<br />

materials separation equipment are outlined below. The interest rate for state-mandated projects<br />

is 5% per year.<br />

Contractor option (C): $8 million now and $25,000 per year will provide separation services<br />

at a maximum of 15 sites. No contract period is stated; thus the contract and services are offered<br />

for as long as the State needs them.<br />

Purchase option (P): Purchase equipment at each site for $275,000 per site and expend an<br />

estimated $12,000 in annual operating costs (AOC). Expected life of the equipment is 5 years<br />

with no salvage value.<br />

(a) Perform a capitalized cost analysis for a total of 10 recycling sites.<br />

(b) Determine the maximum number of sites at which the equipment can be purchased and<br />

still have a capitalized cost less than that of the contractor option.<br />

Solution<br />

(a) Figure 5–6, column B, details the solution. The contract, as proposed, has a long life.<br />

Therefore, the $8 million is already a capitalized cost. The annual charge of A $25,000<br />

is divided by i 0.05 to determine its CC value. Summing the two values results in<br />

CC C $8.5 million.<br />

For the finite, 5-year purchase alternative, column B shows the first cost ($275,000 per<br />

site), AOC ($12,000), and equivalent A value of $−755,181, which is determined via<br />

the PMT function (cell tag). Divide A by the interest rate of 5% to determine CC P <br />

$15.1 million.<br />

The contractor option is by far more economical for the anticipated 10 sites.<br />

(b) A quick way to find the maximum number of sites for which CC P CC C is to use Excel’s<br />

Goal Seek tool, introduced in Chapter 2, Example 2.10. (See Appendix A for details<br />

on how to use this tool.) The template is set up in Figure 5–6 to make the two CC values<br />

equal as the number of sites is altered (decreased). The result, shown in column C, indicates<br />

that 5.63 sites make the options economically equivalent. Since the number of sites


142 Chapter 5 Present Worth Analysis<br />

PMT($B$1,B13,B12*B4) B14*B4<br />

Figure 5–6<br />

Spreadsheet solution of Example 5.8 using capitalized cost ( a ) for 10 recycling sites and ( b ) to determine the<br />

number of sites to make the alternatives economically equal.<br />

must be an integer, 5 or fewer sites will favor purchasing the equipment and 6 or more<br />

sites will favor contracting the separation services.<br />

This approach to problem solution will be called breakeven analysis in later chapters<br />

of the text. By the way, another way to determine the number of sites is by trial<br />

and error. Enter different values in cell B4 until the CC values favor the purchase<br />

alternative.<br />

CHAPTER SUMMARY<br />

The present worth method of comparing alternatives involves converting all cash flows to present<br />

dollars at the MARR. The alternative with the numerically larger (or largest) PW value is selected.<br />

When the alternatives have different lives, the comparison must be made for equal-service<br />

periods. This is done by performing the comparison over either the LCM of lives or a specific<br />

study period. Both approaches compare alternatives in accordance with the equal-service requirement.<br />

When a study period is used, any remaining value in an alternative is recognized<br />

through the estimated future market value.<br />

If the life of the alternatives is considered to be very long or infinite, capitalized cost is the<br />

comparison method. The CC value is calculated as A i , because the P A factor reduces to 1 i in<br />

the limit of n .<br />

PROBLEMS<br />

Types of Projects<br />

5.1 What is the difference between mutually exclusive<br />

alternatives and independent projects?<br />

5.2 (a) What is meant by the do-nothing alternative?<br />

(b) When is the do-nothing alternative not an<br />

option?<br />

5.3 (a) How many alternatives are possible from<br />

four independent projects identified as W, X,<br />

Y, and Z?<br />

(b) List all of the possibilities.<br />

5.4 What is the difference between a revenue and a<br />

cost alternative?


Problems 143<br />

5.5 What is meant by the term equal service ?<br />

5.6 What two approaches can be used to satisfy the<br />

equal-service requirement?<br />

Alternative Comparison—Equal Lives<br />

5.7 A company that manufactures magnetic membrane<br />

switches is investigating two production options<br />

that have the estimated cash flows shown ($1 million<br />

units). Which one should be selected on the<br />

basis of a present worth analysis at 10% per year?<br />

In-house<br />

Contract<br />

First cost, $ 30 0<br />

Annual cost, $ per year 5 2<br />

Annual income, $ per year 14 3.1<br />

Salvage value, $ 2 —<br />

Life, years 5 5<br />

5.8 The manager of a canned food processing plant<br />

must decide between two different labeling machines.<br />

Machine A will have a first cost of $42,000,<br />

an annual operating cost of $28,000, and a service<br />

life of 4 years. Machine B will cost $51,000 to buy<br />

and will have an annual operating cost of $17,000<br />

during its 4-year life. At an interest rate of 10% per<br />

year, which should be selected on the basis of a<br />

present worth analysis?<br />

5.9 A metallurgical engineer is considering two materials<br />

for use in a space vehicle. All estimates are<br />

made. ( a ) Which should be selected on the basis of<br />

a present worth comparison at an interest rate of<br />

12% per year? ( b ) At what first cost for the material<br />

not selected above will it become the more<br />

economic alternative?<br />

Material X<br />

Material Y<br />

First cost, $ 15,000 35,000<br />

Maintenance cost, $ per year 9,000 7,000<br />

Salvage value, $ 2,000 20,000<br />

Life, years 5 5<br />

5.10 To retain high-performing engineers, a large semiconductor<br />

company provides corporate stock as<br />

part of the compensation package. In one particular<br />

year, the company offered 1000 shares of either<br />

class A or class B stock. The class A stock was selling<br />

for $30 per share at the time, and stock market<br />

analysts predicted that it would increase at a rate of<br />

6% per year for the next 5 years. Class B stock was<br />

selling for $20 per share, but its price was expected<br />

to increase by 12% per year. At an interest rate of<br />

8% per year, which stock should the engineers select<br />

on the basis of a present worth analysis and a<br />

5-year planning horizon?<br />

5.11 The Murphy County Fire Department is considering<br />

two options for upgrading its aging physical facilities.<br />

Plan A involves remodeling the fire stations<br />

on Alameda Avenue and Trowbridge Boulevard that<br />

are 57 and 61 years old, respectively. (The industry<br />

standard is about 50 years of use for a station.) The<br />

cost for remodeling the Alameda station is estimated<br />

at $952,000 while the cost of redoing the<br />

Trowbridge station is $1.3 million. Plan B calls for<br />

buying 5 acres of land somewhere between the two<br />

stations, building a new fire station, and selling the<br />

land and structures at the previous sites. The cost of<br />

land in that area is estimated to be $366,000 per<br />

acre. The size of the new fire station would be 9000<br />

square feet with a construction cost of $151.18 per<br />

square foot. Contractor fees for overhead, profit,<br />

etc. are expected to be $340,000, and architect fees<br />

will be $81,500. (Assume all of the costs for plan B<br />

occur at time 0.) If plan A is adopted, the extra cost<br />

for personnel and equipment will be $126,000 per<br />

year. Under plan B, the sale of the old sites is anticipated<br />

to net a positive $500,000 five years in the<br />

future. Use an interest rate of 6% per year and a 50-<br />

year useful life for the remodeled and new stations<br />

to determine which plan is better on the basis of a<br />

present worth analysis.<br />

5.12 Delcon Properties is a commercial developer of<br />

shopping centers and malls in various places around<br />

the country. The company needs to analyze the economic<br />

feasibility of rainwater drains in a 60-acre<br />

area that it plans to develop. Since the development<br />

won’t be started for 3 years, this large open space<br />

will be subject to damage from heavy thunderstorms<br />

that cause soil erosion and heavy rutting. If<br />

no drains are installed, the cost of refilling and<br />

grading the washed out area is expected to be<br />

$1500 per thunderstorm. Alternatively, a temporary<br />

corrugated steel drainage pipe could be installed<br />

that will prevent the soil erosion. The cost of the<br />

pipe will be $3 per foot for the total length of<br />

7000 feet required. Some of the pipe will be salvageable<br />

for $4000 at the end of the 3-year period<br />

between now and when the construction begins.<br />

Assuming that thunderstorms occur regularly at<br />

3-month intervals, starting 3 months from now,<br />

which alternative should be selected on the basis of<br />

a present worth comparison using an interest rate of<br />

4% per quarter?<br />

5.13 A public water utility is trying to decide between<br />

two different sizes of pipe for a new water main. A<br />

250-mm line will have an initial cost of $155,000,<br />

whereas a 300-mm line will cost $210,000. Since<br />

there is more head loss through the 250-mm pipe,<br />

the pumping cost is expected to be $3000 more per<br />

year than for the 250-mm line. If the lines are


144 Chapter 5 Present Worth Analysis<br />

expected to last for 30 years, which size should be<br />

selected on the basis of a present worth analysis<br />

using an interest rate of 10% per year?<br />

5.14 The supervisor of a community swimming pool<br />

has developed two methods for chlorinating the<br />

pool. If gaseous chlorine is added, a chlorinator<br />

will be required that has an initial cost of $8000<br />

and a useful life of 5 years. The chlorine will cost<br />

$650 per year, and the labor cost will be $800 per<br />

year. Alternatively, dry chlorine can be added manually<br />

at a cost of $1000 per year for chlorine and<br />

$1900 per year for labor. Which method should be<br />

used on the basis of a present worth analysis if the<br />

interest rate is 10% per year?<br />

5.15 Anion, an environmental engineering consulting<br />

firm, is trying to be eco-friendly in acquiring an automobile<br />

for general office use. It is considering a<br />

gasoline-electric hybrid and a gasoline-free allelectric<br />

hatchback. The hybrid under consideration<br />

is GM’s Volt, which will cost $35,000 and have a<br />

range of 40 miles on the electric battery and several<br />

hundred more miles when the gasoline engine<br />

kicks in. Nissan’s Leaf, on the other hand, is a pure<br />

electric that will have a range of only 100 miles,<br />

after which its lithium-ion battery will have to be<br />

recharged. The Leaf’s relatively limited range creates<br />

a psychological effect known as range anxiety.<br />

This fact alone has caused the company to lean toward<br />

purchasing the Volt, which is assumed to have<br />

a salvage value of $15,000 in 5 years. The Leaf<br />

could be leased for $349 per month (end-of-month<br />

payments) for 5 years after an initial $1500 down<br />

payment for “account activation.” If the consulting<br />

company plans to ignore the range anxiety effect in<br />

making its decision, which automobile is the better<br />

option on the basis of a present worth analysis at an<br />

interest rate of 0.75% per month? Assume the operating<br />

cost will be the same for both vehicles.<br />

5.16 A pipeline engineer working in Kuwait for the oil<br />

giant BP wants to perform a present worth analysis on<br />

alternative pipeline routings—the first predominately<br />

by land and the second primarily undersea. The undersea<br />

route is more expensive initially due to extra<br />

corrosion protection and installation costs, but cheaper<br />

security and maintenance reduces annual costs. Perform<br />

the analysis for the engineer at 15% per year.<br />

Land<br />

Undersea<br />

Installation cost, $ million 215 350<br />

Pumping, operating, security, 22 2<br />

$ million per year<br />

Replacement of valves and<br />

30 70<br />

appurtenances in year 25, $ million<br />

Expected life, years 50 50<br />

Alternative Comparison—Different Lives<br />

5.17 An electric switch manufacturing company has to<br />

choose one of three different assembly methods.<br />

Method A will have a first cost of $40,000, an annual<br />

operating cost of $9000, and a service life of<br />

2 years. Method B will cost $80,000 to buy and<br />

will have an annual operating cost of $6000 over<br />

its 4-year service life. Method C will cost $130,000<br />

initially with an annual operating cost of $4000<br />

over its 8-year life. Methods A and B will have no<br />

salvage value, but method C will have some equipment<br />

worth an estimated $12,000. Which method<br />

should be selected? Use present worth analysis at<br />

an interest rate of 10% per year.<br />

5.18 Midwest Power and Light operates 14 coal-fired<br />

power plants in several states around the United<br />

States. The company recently settled a lawsuit by<br />

agreeing to pay $60 million in mitigation costs related<br />

to acid rain. The settlement included $21 million<br />

to reduce emissions from barges and trucks in<br />

the Ohio River Valley, $24 million for projects to<br />

conserve energy and produce alternative energy,<br />

$3 million for Chesapeake Bay, $2 million for<br />

Shenandoah National Park, and $10 million to acquire<br />

ecologically sensitive lands in Appalachia.<br />

The question of how to distribute the money over<br />

time has been posed. Plan A involves spending<br />

$5 million now and the remaining $55 million<br />

equally over a 10-year period (that is, $5.5 million<br />

in each of years 1 through 10). Plan B requires expenditures<br />

of $5 million now, $25 million 2 years<br />

from now, and $30 million 7 years from now. Determine<br />

which plan is more economical on the<br />

basis of a present worth analysis over a 10-year<br />

period at an interest rate of 10% per year.<br />

5.19 Machines that have the following costs are under<br />

consideration for a robotized welding process.<br />

Using an interest rate of 10% per year, determine<br />

which alternative should be selected on the basis<br />

of a present worth analysis. Show ( a ) hand and<br />

( b ) spreadsheet solutions.<br />

Machine X Machine Y<br />

First cost, $ 250,000 430,000<br />

Annual operating cost, $ per year 60,000 40,000<br />

Salvage value, $ 70,000 95,000<br />

Life, years 3 6<br />

5.20 Water for Semiconductor Manufacturing Case<br />

PE<br />

Throughout the present worth analyses, the decision<br />

between seawater and groundwater switched<br />

multiple times in Examples 5.2 and 5.4. A summary<br />

is given here in $1 million units.


Life n ,<br />

years<br />

First<br />

cost, $<br />

Seawater (S)<br />

PW at<br />

12%, $ Selected<br />

Life n ,<br />

years<br />

Problems 145<br />

Groundwater (G)<br />

First<br />

cost, $<br />

PW at<br />

12%, $ Selected<br />

10 20 30.64 Yes 10 22 33.16 No<br />

5<br />

(study<br />

period)<br />

5 20, plus<br />

10 after<br />

5 years<br />

36.31 No 10 22 33.16 Yes<br />

20 26.43 Yes 5<br />

(study<br />

period)<br />

22 28.32 No<br />

The confusion about the recommended source for<br />

UPW has not gone unnoticed by the general manager.<br />

Yesterday, you were asked to settle the issue<br />

by determining the first cost X S of the seawater option<br />

to ensure that it is the economic choice over<br />

groundwater. The study period is set by the manager<br />

as 10 years, simply because that is the time period on<br />

the lease agreement for the building where the fab<br />

will be located. Since the seawater equipment must<br />

be refurbished or replaced after 5 years, the general<br />

manager told you to assume that the equipment will<br />

be purchased anew after 5 years of use. What is the<br />

maximum first cost that Angular Enterprises should<br />

pay for the seawater option?<br />

5.21 Accurate airflow measurement requires straight<br />

unobstructed pipe for a minimum of 10 diameters<br />

upstream and 5 diameters downstream of the measuring<br />

device. In a field application, physical constraints<br />

compromise the pipe layout, so the engineer<br />

is considering installing the airflow probes in an<br />

elbow, knowing that flow measurement will be less<br />

accurate but good enough for process control. This<br />

is plan 1, which will be in place for only 3 years,<br />

after which a more accurate flow measurement system<br />

with the same costs as plan 1 will be available.<br />

This plan will have a first cost of $26,000 with an<br />

annual maintenance cost estimated at $5000.<br />

Plan 2 involves installation of a recently designed<br />

submersible airflow probe. The stainless<br />

steel probe can be installed in a drop pipe with the<br />

transmitter located in a waterproof enclosure on the<br />

handrail. The first cost of this system is $83,000,<br />

but because it is accurate and more durable, it will<br />

not have to be replaced for at least 6 years. Its maintenance<br />

cost is estimated to be $1400 per year plus<br />

$2500 in year 3 for replacement of signal processing<br />

software. Neither system will have a salvage<br />

value. At an interest rate of 10% per year, which<br />

one should be selected on the basis of a present<br />

worth comparison?<br />

5.22 An engineer is considering two different liners for an<br />

evaporation pond that will receive salty concentrate<br />

from a brackish water desalting plant. A plastic liner<br />

will cost $0.90 per square foot initially and will<br />

require replacement in 15 years when precipitated<br />

solids will have to be removed from the pond using<br />

heavy equipment. This removal will cost $500,000.<br />

A rubberized elastomeric liner is tougher and, therefore,<br />

is expected to last 30 years, but it will cost<br />

$2.20 per square foot. If the size of the pond is<br />

110 acres (1 acre 43,560 square feet), which liner<br />

is more cost effective on the basis of a present worth<br />

comparison at an interest rate of 8% per year?<br />

5.23 A sports mortgage is the brainchild of Stadium Capital<br />

Financing Group, a company headquartered in<br />

Chicago, Illinois. It is an innovative way to finance<br />

cash-strapped sports programs by allowing fans to<br />

sign up to pay a “mortgage” over a certain number of<br />

years for the right to buy good seats at football games<br />

for several decades with season ticket prices locked<br />

in at current prices. In California, the locked-in price<br />

period is 50 years. Assume UCLA fan X purchases a<br />

$130,000 mortgage and pays for it now to get season<br />

tickets for $290 each for 50 years, while fan Y buys<br />

season tickets at $290 in year 1, with prices increasing<br />

by $20 per year for 50 years. ( a ) Which fan made<br />

the better deal if the interest rate is 8% per year?<br />

( b ) What should fan X be willing to pay up front for<br />

the mortgage to make the two plans exactly equivalent<br />

economically? (Assume he has no reason to<br />

give extra money to UCLA at this point.)<br />

5.24 A chemical processing corporation is considering<br />

three methods to dispose of a non-hazardous chemical<br />

sludge: land application, fluidized-bed incineration,<br />

and private disposal contract. The estimates for<br />

each method are shown. Determine which has the<br />

least cost on the basis of a present worth comparison<br />

at 10% per year for the following scenarios:<br />

(a) The estimates as shown<br />

(b) The contract award cost increases by 20%<br />

every 2-year renewal<br />

Land Application Incineration<br />

Contract<br />

First cost, $ 130,000 900,000 0<br />

Annual operating 95,000 60,000 120,000<br />

cost, $ per year<br />

Salvage value, $ 25,000 300,000 0<br />

Life, years 3 6 2


146 Chapter 5 Present Worth Analysis<br />

5.25 An assistant to Stacy gave her the PW values for<br />

four alternatives they are comparing for the development<br />

of a remote control vibration control system<br />

for offshore platform application. The results<br />

in the table use a MARR of 14% per year. Determine<br />

which alternative(s) should be selected ( a ) if<br />

the alternatives are exclusive, and ( b ) if the projects<br />

are independent.<br />

I J K L<br />

Life n , years 3 4 12 6<br />

PW over n years, $ 16.08 31.12 257.46 140.46<br />

PW over 6 years, $ 26.94 15.78 653.29 140.46<br />

PW over 12 years, $ 39.21 60.45 257.46 204.46<br />

Future Worth Comparison<br />

5.26 An industrial engineer is considering two robots<br />

for purchase by a fiber-optic manufacturing company.<br />

Robot X will have a first cost of $80,000, an<br />

annual maintenance and operation (M&O) cost of<br />

$30,000, and a $40,000 salvage value. Robot Y<br />

will have a first cost of $97,000, an annual M&O<br />

cost of $27,000, and a $50,000 salvage value.<br />

Which should be selected on the basis of a future<br />

worth comparison at an interest rate of 15% per<br />

year? Use a 3-year study period.<br />

5.27 Two processes can be used for producing a polymer<br />

that reduces friction loss in engines. Process T<br />

will have a first cost of $750,000, an operating cost<br />

of $60,000 per year, and a salvage value of $80,000<br />

after its 2-year life. Process W will have a first cost<br />

of $1,350,000, an operating cost of $25,000 per<br />

year, and a $120,000 salvage value after its 4-year<br />

life. Process W will also require updating at the<br />

end of year 2 at a cost of $90,000. Which process<br />

should be selected on the basis of a future worth<br />

analysis at an interest rate of 12% per year?<br />

5.28 Compare the alternatives shown below on the<br />

basis of a future worth analysis, using an interest<br />

rate of 8% per year.<br />

First cost, $ 23,000 30,000<br />

Annual operating cost, $ per year 4,000 2,500<br />

Salvage value, $ 3,000 1,000<br />

Life, years 3 6<br />

5.29 Two manufacturers supply MRI systems for medical<br />

imaging. St. Jude’s Hospital wishes to replace its<br />

current MRI equipment that was purchased 8 years<br />

ago with the newer technology and clarity of a stateof-the-art<br />

system. System K will have a first cost of<br />

$1,600,000, an operating cost of $70,000 per year,<br />

and a salvage value of $400,000 after its 4-year life.<br />

System L will have a first cost of $2,100,000, an<br />

P<br />

Q<br />

operating cost of $50,000 the first year with an expected<br />

increase of $3000 per year thereafter, and no<br />

salvage value after its 8-year life. Which system<br />

should be selected on the basis of a future worth<br />

analysis at an interest rate of 12% per year?<br />

5.30 A retail shopping center developer signed a contract<br />

to build a $100 million high-end shopping center in<br />

City Center, because the city and county governments<br />

agreed to sales and tax rebates totaling<br />

$18.7 million over 10 years. The contract called for<br />

the developer to raze existing buildings 2 years<br />

from the date the contract was signed and to have<br />

the shopping center built by the end of year 3. However,<br />

due to a real estate–induced recession in the<br />

United States, the developer sought and was granted<br />

a new contract. The new contract required the developer<br />

to raze the existing buildings at the end of<br />

year 1, but the shopping center would not have to be<br />

completed for 7 years from the date the contract was<br />

signed. Assume that the cost for razing the existing<br />

buildings is $1.3 million and the developer does not<br />

build the shopping center until 7 years from now (at<br />

a cost of $100 million). Determine the difference in<br />

the future worth cost in year 7 of the two contracts<br />

at an interest rate of 10% per year.<br />

Capitalized Cost<br />

5.31 A wealthy businessman wants to start a permanent<br />

fund for supporting research directed toward sustainability.<br />

The donor plans to give equal amounts<br />

of money for each of the next 5 years, plus one now<br />

(i.e., six donations) so that $100,000 per year can<br />

be withdrawn each year forever, beginning in<br />

year 6. If the fund earns interest at a rate of 8% per<br />

year, how much money must be donated each time?<br />

5.32 Bob, a philanthropist, is not sure what rate of return<br />

his gifts may realize once donated to his favorite<br />

charity. Determine the capitalized cost of $10,000<br />

every 5 years forever , starting 5 years from now<br />

at an interest rate of ( a ) 3% and ( b ) 8% per year.<br />

( c ) Explain the significant difference between the<br />

two capitalized costs.<br />

5.33 Find the capitalized cost of a present cost of<br />

$300,000, annual costs of $35,000, and periodic<br />

costs every 5 years of $75,000. Use an interest rate<br />

of 12% per year.<br />

5.34 Water for Semiconductor Manufacturing Case<br />

PE<br />

It is anticipated that the needs for UPW (ultrapure<br />

water) at the new Angular Enterprises site will<br />

continue for a long time, as long as 50 years. This<br />

is the rationale for using capitalized cost as a basis<br />

for the economic decision between desalinated


Additional Problems and FE Exam Review Questions 147<br />

seawater (S) and purified groundwater (G). These<br />

costs were determined (Example 5.7) to be CC S <br />

$53.58 million and CC G $48.91 million.<br />

Groundwater is the clear economic choice.<br />

Yesterday, the general manager had lunch with<br />

the president of Brissa Water, who offered to supply<br />

the needed UPW at a cost of $5 million per year<br />

for the indefinite future. It would mean a dependence<br />

upon a contractor to supply the water, but the<br />

equipment, treatment, and other costly activities to<br />

obtain UPW on-site would be eliminated. The<br />

manager asks you to make a recommendation<br />

about this seemingly attractive alternative under<br />

the following conditions at the same MARR of<br />

12% per year as used for the other analyses:<br />

( a) The annual cost of $5 million remains constant<br />

throughout the time it is needed.<br />

(b) The annual cost starts at $5 million for the first<br />

year only, and then it increases 2% per year.<br />

(This increase is above the cost of providing<br />

UPW by either of the other two methods.)<br />

5.35 Compare the alternatives shown on the basis of<br />

their capitalized costs using an interest rate of<br />

10% per year.<br />

Alternative M<br />

Alternative N<br />

First cost, $ 150,000 800,000<br />

Annual operating 50,000 12,000<br />

cost, $ per year<br />

Salvage value, $ 8,000 1,000,000<br />

Life, years 5 <br />

5.36 The cost of maintaining a certain permanent monument<br />

in Washington, DC occurs as periodic outlays<br />

of $1000 every year and $5000 every 4 years.<br />

Calculate the capitalized cost of the maintenance<br />

using an interest rate of 10% per year.<br />

5.37 Because you are thankful for what you learned in<br />

engineering economy, you plan to start a permanent<br />

scholarship fund in the name of the professor<br />

who taught the course. You plan to deposit money<br />

now with the stipulation that the scholarships be<br />

awarded beginning 12 years from now (which<br />

happens to be the exact time that your daughter<br />

plans to begin college). The interest that is accumulated<br />

between now and year 12 is to be added to<br />

the principal of the endowment. After that, the interest<br />

that is earned each year will be awarded as<br />

scholarship money. If you want the amount of the<br />

scholarships to be $40,000 per year, how much<br />

must you donate now if the fund earns interest at a<br />

rate of 8% per year?<br />

5.38 A patriotic group of firefighters is raising money<br />

to erect a permanent (i.e., infinite life) monument<br />

in New York City to honor those killed in the line<br />

of duty. The initial cost of the monument will be<br />

$150,000, and the annual maintenance will cost<br />

$5000. There will be an additional one-time cost<br />

of $20,000 in 2 years to add names of those who<br />

were missed initially. At an interest rate of 6% per<br />

year, how much money must they raise now in<br />

order to construct and maintain the monument<br />

forever?<br />

ADDITIONAL PROBLEMS AND FE EXAM REVIEW QUESTIONS<br />

5.39 One assumption inherent in the present worth<br />

method of analysis is that:<br />

( a) The alternatives will be used only through<br />

the life of the shortest-lived alternative.<br />

( b) The alternatives will be used only through<br />

the life of the longest-lived alternative.<br />

( c) The cash flows of each alternative will<br />

change only by the inflation or deflation rate<br />

in succeeding life cycles.<br />

( d) At least one of the alternatives will have a<br />

finite life.<br />

5.40 When only one alternative can be selected from<br />

two or more, the alternatives are said to be:<br />

( a) Mutually exclusive<br />

( b) Independent alternatives<br />

( c) Cost alternatives<br />

( d) Revenue alternatives<br />

5.41 For the mutually exclusive alternatives shown, the<br />

one(s) that should be selected are:<br />

(a)<br />

(b)<br />

(c)<br />

(d)<br />

Alternative PW, $<br />

Only C<br />

Only A<br />

C and D<br />

Only D<br />

A 25,000<br />

B 12,000<br />

C 10,000<br />

D 15,000<br />

5.42 The alternatives shown are to be compared on the<br />

basis of their present worth values. At an interest<br />

rate of 10% per year, the values of n that you<br />

should use in the uniform series factors to make a


148 Chapter 5 Present Worth Analysis<br />

correct comparison by the present worth method<br />

are:<br />

Alternative A<br />

Alternative B<br />

First cost, $ 50,000 90,000<br />

Annual operating cost, 10,000 4000<br />

$ per year<br />

Salvage value, $ 13,000 15,000<br />

Life, years 3 6<br />

(a)<br />

(b)<br />

(c)<br />

(d)<br />

n 3 years for A and n 3 years for B<br />

n 3 years for A and n 6 years for B<br />

n 6 years for A and n 6 years for B<br />

None of the above<br />

5.43 The value of the future worth for alternative P at an<br />

interest rate of 8% per year is closest to:<br />

P<br />

Q<br />

First cost, $ 23,000 30,000<br />

Annual operating cost, 4,000 2,500<br />

$ per year<br />

Salvage value, $ 3,000 1,000<br />

Life, years 3 6<br />

(a)<br />

(b)<br />

(c)<br />

(d)<br />

FW P $88,036<br />

FW P $86,026<br />

FW P $81,274<br />

FW P $70,178<br />

5.44 The present worth of $50,000 now, $10,000 per year<br />

in years 1 through 15, and $20,000 per year in years<br />

16 through infinity at 10% per year is closest to:<br />

(a) Less than $169,000<br />

(b) $169,580<br />

(c) $173,940<br />

(d) $195,730<br />

5.45 A donor (you) wishes to start an endowment that<br />

will provide scholarship money of $40,000 per year<br />

beginning in year 5 and continuing indefinitely. If<br />

the university earns 10% per year on the endowment,<br />

the amount you must donate now is closest to:<br />

(a) $225,470<br />

(b) $248,360<br />

(c) $273,200<br />

(d) $293,820<br />

Problems 5.46 through 5.48 are based on the following<br />

information.<br />

Alternative I<br />

Alternative J<br />

Initial cost, $ 150,000 250,000<br />

Annual income, $ per year 20,000 40,000<br />

Annual expenses, $ per year 9,000 14,000<br />

Salvage value, $ 25,000 35,000<br />

Life, years 3 6<br />

The interest rate is 15% per year.<br />

5.46 In comparing alternatives I and J by the present<br />

worth method, the value of n that must be used in<br />

11,000( P A , i , n ) for alternative I is:<br />

(a) 3<br />

(b) 6<br />

(c) 18<br />

(d) 36<br />

5.47 In comparing alternatives I and J by the present<br />

worth method, the equation that yields the present<br />

worth of alternative J is:<br />

(a) PW J 250,000 40,000( P A ,15%,6) <br />

35,000( P F ,15%,6)<br />

(b) PW J 250,000 26,000( P A ,15%,6) <br />

( c) PW J<br />

35,000( P F ,15%,6)<br />

250,000 26,000( P A ,15%,6) <br />

35,000( P F ,15%,6)<br />

( d) PW J 250,000 26,000( P A ,15%,6) <br />

35,000( P F ,15%,6)<br />

5.48 In comparing alternatives I and J by the present<br />

worth method, the equation that yields the present<br />

worth of alternative I is:<br />

( a) PW I 150,000 11,000( P A ,15%,3) <br />

25,000( P F ,15%,3)<br />

(b) PW I 150,000 11,000( P A ,15%,6) <br />

25,000( P F ,15%,6)<br />

(c) PW I 150,000 11,000( P A ,15%,6) <br />

175,000( P F ,15%,3) 25,000( P F ,15%,6)<br />

(d) PW I 150,000 11,000( P A ,15%,6) <br />

125,000( P F ,15%,3) 25,000( P F ,15%,6)<br />

Problems 5.49 and 5.50 are based on the following<br />

information.<br />

Machine X Machine Y<br />

Initial cost, $ 80,000 95,000<br />

Annual operating cost, $ per year 20,000 15,000<br />

Salvage value, $ 10,000 30,000<br />

Life, years 2 4<br />

The interest rate is 10% per year.<br />

5.49 The equation that will calculate the present worth<br />

of machine X is:<br />

(a) PW X 80,000 15,000( P A ,10%,4) <br />

30,000( P F ,10%,4)<br />

(b) PW X 80,000 20,000( P A ,10%,4) <br />

80,000( P F ,10%,2) 10,000( P F ,10%,4)<br />

( c) PW X 80,000 20,000( P A ,10%,2) <br />

10,000( P F ,10%,2)<br />

(d) PW X 80,000 20,000( P A ,10%,4) <br />

70,000( P F ,10%,2) 10,000( P F ,10%,4)<br />

5.50 In comparing the machines on a present worth<br />

basis, the present worth of machine Y is closest to:<br />

(a) $112,320<br />

(b) $122,060<br />

(c) $163,040<br />

(d) $175,980


Case Study 149<br />

5.51 The capitalized cost of $10,000 every 5 years<br />

forever , starting now at an interest rate of 10% per<br />

year, is closest to:<br />

( a) $13,520<br />

( b) $16,380<br />

( c) $26,380<br />

( d) $32,590<br />

5.52 At an interest rate of 10% per year, the capitalized<br />

cost of $10,000 in year 0, $5000 per year in years 1<br />

through 5, and $1000 per year thereafter forever is<br />

closest to:<br />

(a) $29,652<br />

(b) $35,163<br />

(c) $38,954<br />

(d) $43,221<br />

CASE STUDY<br />

COMPARING SOCIAL SECURITY BENEFITS<br />

Background<br />

When Sheryl graduated from Northeastern University in<br />

2000 and went to work for BAE Systems, she did not pay<br />

much attention to the monthly payroll deduction for social<br />

security. It was a “necessary evil” that may be helpful in retirement<br />

years. However, this was so far in the future that she<br />

fully expected this government retirement benefit system to<br />

be broke and gone by the time she could reap any benefits<br />

from her years of contributions.<br />

This year, Sheryl and Brad, another engineer at BAE,<br />

got married. Recently, they both received notices from the<br />

Social Security Administration of their potential retirement<br />

amounts, were they to retire and start social security benefits<br />

at preset ages. Since both of them hope to retire a few<br />

years early, they decided to pay closer attention to the predicted<br />

amount of retirement benefits and to do some analysis<br />

on the numbers.<br />

Information<br />

They found that their projected benefits are substantially the<br />

same, which makes sense since their salaries are very close to<br />

each other. Although the numbers were slightly different in<br />

their two mailings, the similar messages to Brad and Sheryl<br />

can be summarized as follows:<br />

If you stop working and start receiving benefits . . .<br />

At age 62, your payment would<br />

be about<br />

At you full retirement age (67<br />

years), your payment would be<br />

about<br />

At age 70, your payment would<br />

be about<br />

$1400 per month<br />

$2000 per month<br />

$2480 per month<br />

These numbers represent a reduction of 30% for early retirement<br />

(age 62) and an increase of 24% for delayed retirement<br />

(age 70).<br />

This couple also learned that it is possible for a spouse to<br />

take spousal benefits at the time that one of them is at full<br />

retirement age. In other words, if Sheryl starts her $2000 benefit<br />

at age 67, Brad can receive a benefit equal to 50% of hers.<br />

Then, when Brad reaches 70 years of age, he can discontinue<br />

spousal benefits and start his own. In the meantime, his benefits<br />

will have increased by 24%. Of course, this strategy<br />

could be switched with Brad taking his benefits and Sheryl<br />

receiving spousal benefits until age 70.<br />

All these options led them to define four alternative plans.<br />

A: Each takes early benefits at age 62 with a 30% reduction<br />

to $1400 per month.<br />

B: Each takes full benefits at full retirement age of 67<br />

and receives $2000 per month.<br />

C: Each delays benefits until age 70 with a 24% increase<br />

to $2480 per month.<br />

D: One person takes full benefits of $2000 per month at<br />

age 67, and the other person receives spousal benefits<br />

($1000 per month at age 67) and switches to delayed<br />

benefits of $2480 at age 70.<br />

They realize, of course, that the numbers will change over<br />

time, based on their respective salaries and number of years<br />

of contribution to the social security system by them and by<br />

their employers.<br />

Case Study Exercises<br />

Brad and Sheryl are the same age. Brad determined that most<br />

of their investments make an average of 6% per year. With<br />

this as the interest rate, the analysis for the four alternatives is<br />

possible. Sheryl and Brad plan to answer the following questions,<br />

but don’t have time this week. Can you please help<br />

them? (Do the analysis for one person at a time, not the couple,<br />

and stop at the age of 85.)<br />

1. How much in total (without the time value of money considered)<br />

will each plan A through D pay through age 85?<br />

2. What is the future worth at 6% per year of each plan at<br />

age 85?<br />

3. Plot the future worth values for all four plans on one<br />

spreadsheet graph.<br />

4. Economically, what is the best combination of plans for<br />

Brad and Sheryl, assuming they both live to be 85 years<br />

old?<br />

5. Develop at least one additional question that you think<br />

Sheryl and Brad may have. Answer the question.


CHAPTER 6<br />

Annual Worth<br />

Analysis<br />

LEARNING OUTCOMES<br />

Purpose: Utilize different annual worth techniques to evaluate and select alternatives.<br />

SECTION TOPIC LEARNING OUTCOME<br />

6.1 Advantages of AW • Demonstrate that the AW value is the same for<br />

each life cycle.<br />

6.2 CR and AW values • Calculate and interpret the capital recovery (CR)<br />

and AW amounts.<br />

6.3 AW analysis • Select the best alternative using an annual worth<br />

analysis.<br />

6.4 Perpetual life • Evaluate alternatives with very long lives using<br />

AW analysis.<br />

6.5 LCC analysis • Perform a life-cycle cost (LCC) analysis using AW<br />

methods.


I<br />

n this chapter, we add to our repertoire of alternative comparison tools. In Chapter<br />

5 we learned the PW method. Here we learn the equivalent annual worth, or<br />

AW, method. AW analysis is commonly considered the more desirable of the two<br />

methods because the AW value is easy to calculate; the measure of worth—AW in monetary<br />

units per year—is understood by most individuals; and its assumptions are essentially identical<br />

to those of the PW method.<br />

Annual worth is also known by other titles. Some are equivalent annual worth (EAW),<br />

equivalent annual cost (EAC), annual equivalent (AE), and equivalent uniform annual cost<br />

(EUAC). The alternative selected by the AW method will always be the same as that selected<br />

by the PW method, and all other alternative evaluation methods, provided they are performed<br />

correctly.<br />

An additional application of AW analysis treated here is life-cycle cost (LCC) analysis. This<br />

method considers all costs of a product, process, or system from concept to phaseout.<br />

6.1 Advantages and Uses of Annual Worth Analysis<br />

For many engineering economic studies, the AW method is the best to use, when compared to<br />

PW, FW, and rate of return (Chapters 7 and 8). Since the AW value is the equivalent uniform annual<br />

worth of all estimated receipts and disbursements during the life cycle of the project or alternative,<br />

AW is easy to understand by any individual acquainted with annual amounts, for example,<br />

dollars per year. The AW value, which has the same interpretation as A used thus far, is<br />

the economic equivalent of the PW and FW values at the MARR for n years. All three can be<br />

easily determined from each other by the relation<br />

AW PW( A P,i,n ) FW( A F,i,n ) [6.1]<br />

The n in the factors is the number of years for equal-service comparison. This is the LCM or the<br />

stated study period of the PW or FW analysis.<br />

When all cash flow estimates are converted to an AW value, this value applies for every year<br />

of the life cycle and for each additional life cycle.<br />

The annual worth method offers a prime computational and interpretation advantage because<br />

the AW value needs to be calculated for only one life cycle. The AW value determined over<br />

one life cycle is the AW for all future life cycles. Therefore, it is not necessary to use the<br />

LCM of lives to satisfy the equal-service requirement.<br />

Equal-service<br />

requirement and LCM<br />

As with the PW method, there are three fundamental assumptions of the AW method that should<br />

be understood. When alternatives being compared have different lives, the AW method makes the<br />

assumptions that<br />

1. The services provided are needed for at least the LCM of the lives of the alternatives.<br />

2. The selected alternative will be repeated for succeeding life cycles in exactly the same manner<br />

as for the first life cycle.<br />

3. All cash flows will have the same estimated values in every life cycle.<br />

In practice, no assumption is precisely correct. If, in a particular evaluation, the first two assumptions<br />

are not reasonable, a study period must be established for the analysis. Note that for assumption<br />

1, the length of time may be the indefinite future (forever). In the third assumption, all<br />

cash flows are expected to change exactly with the inflation (or deflation) rate. If this is not a<br />

reasonable assumption, new cash flow estimates must be made for each life cycle, and again a<br />

study period must be used. AW analysis for a stated study period is discussed in Section 6.3.<br />

EXAMPLE 6.1<br />

In Example 5.3, National Homebuilders, Inc. evaluated cut-and-finish equipment from vendor A<br />

(6-year life) and vendor B (9-year life). The PW analysis used the LCM of 18 years. Consider<br />

only the vendor A option now. The diagram in Figure 6–1 shows the cash flows for all three life<br />

cycles (first cost $15,000; annual M&O costs $3500; salvage value $1000). Demonstrate<br />

the equivalence at i 15% of PW over three life cycles and AW over one cycle. In Example<br />

5.3, present worth for vendor A was calculated as PW $45,036.


152 Chapter 6 Annual Worth Analysis<br />

Figure 6–1<br />

PW and AW values<br />

for three life cycles,<br />

Example 6.1.<br />

PW = $45,036<br />

3 life cycles<br />

0 1 2 3 4 5 6 7 8 9 10 11 12 13 14 15 16 17 18<br />

$1000<br />

0 1 2 3 4 5 6<br />

Life cycle 1<br />

i = 15%<br />

$3500<br />

$1000<br />

$15,000<br />

0 1 2 3 4 5 6<br />

Life cycle 2<br />

$3500<br />

$1000<br />

$15,000<br />

0 1 2 3 4 5 6<br />

Life cycle 3<br />

$3500<br />

$15,000<br />

0 1 2 3 4 5 6 7 8 9 10 11 12 13 14 15 16 17 18 19 20<br />

• • •<br />

continues<br />

AW = $7349<br />

Solution<br />

Calculate the equivalent uniform annual worth value for all cash flows in the first life cycle.<br />

AW 15,000( AP ,15%,6) 1000( AF ,15%,6) 3500 $7349<br />

When the same computation is performed on each succeeding life cycle, the AW value is<br />

$7349. Now Equation [6.1] is applied to the PW value for 18 years.<br />

AW 45,036( AP ,15%,18) $7349<br />

The one-life-cycle AW value and the AW value based on 18 years are equal.<br />

Not only is annual worth an excellent method for performing engineering economy studies,<br />

but also it is applicable in any situation where PW (and FW and benefit/cost) analysis can be<br />

utilized. The AW method is especially useful in certain types of studies: asset replacement and<br />

retention time studies to minimize overall annual costs (both covered in Chapter 11), breakeven<br />

studies and make-or-buy decisions (Chapter 13), and all studies dealing with production or manufacturing<br />

costs where a cost /unit or profit /unit measure is the focus.


6.2 Calculation of Capital Recovery and AW Values 153<br />

If income taxes are considered, a slightly different approach to the AW method is used by some<br />

large corporations and financial institutions. It is termed economic value added, or EVA. This approach,<br />

covered in Chapter 17, concentrates upon the wealth-increasing potential that an alternative offers a<br />

corporation. The resulting EVA values are the equivalent of an AW analysis of after-tax cash flows.<br />

6.2 Calculation of Capital Recovery and AW Values<br />

An alternative should have the following cash flow estimates:<br />

Initial investment P. This is the total first cost of all assets and services required to initiate<br />

the alternative. When portions of these investments take place over several years, their present<br />

worth is an equivalent initial investment. Use this amount as P .<br />

Salvage value S. This is the terminal estimated value of assets at the end of their useful life.<br />

The S is zero if no salvage is anticipated; S is negative when it will cost money to dispose of<br />

the assets. For study periods shorter than the useful life, S is the estimated market value or<br />

trade-in value at the end of the study period.<br />

Annual amount A. This is the equivalent annual amount (costs only for cost alternatives;<br />

costs and receipts for revenue alternatives). Often this is the annual operating cost (AOC) or<br />

M&O cost, so the estimate is already an equivalent A value.<br />

Salvage/market value<br />

The annual worth (AW) value for an alternative is comprised of two components: capital<br />

recovery for the initial investment P at a stated interest rate (usually the MARR) and the equivalent<br />

annual amount A. The symbol CR is used for the capital recovery component. In equation form,<br />

AW CR A [6.2]<br />

Both CR and A represent costs. The total annual amount A is determined from uniform recurring<br />

costs (and possibly receipts) and nonrecurring amounts. The PA and PF factors may be necessary<br />

to first obtain a present worth amount; then the AP factor converts this amount to the A<br />

value in Equation [6.2]. (If the alternative is a revenue project, there will be positive cash flow<br />

estimates present in the calculation of the A value.)<br />

The recovery of an amount of capital P committed to an asset, plus the time value of the<br />

capital at a particular interest rate, is a fundamental principle of economic analysis .<br />

Capital recovery (CR) is the equivalent annual amount that the asset, process, or system must<br />

earn (new revenue) each year to just recover the initial investment plus a stated rate of return<br />

over its expected life. Any expected salvage value is considered in the computation of CR.<br />

Capital recovery<br />

The AP factor is used to convert P to an equivalent annual cost. If there is some anticipated<br />

positive salvage value S at the end of the asset’s useful life, its equivalent annual value is recovered<br />

using the AF factor. This action reduces the equivalent annual cost of the asset. Accordingly,<br />

CR is calculated as<br />

CR P ( A P,i,n ) S ( AF,i,n ) [6.3]<br />

EXAMPLE 6.2<br />

Lockheed Martin is increasing its booster thrust power in order to win more satellite launch contracts<br />

from European companies interested in opening up new global communications markets. A<br />

piece of earth-based tracking equipment is expected to require an investment of $13 million, with<br />

$8 million committed now and the remaining $5 million expended at the end of year 1 of the<br />

project. Annual operating costs for the system are expected to start the first year and continue at<br />

$0.9 million per year. The useful life of the tracker is 8 years with a salvage value of $0.5 million.<br />

Calculate the CR and AW values for the system, if the corporate MARR is 12% per year.<br />

Solution<br />

Capital recovery: Determine P in year 0 of the two initial investment amounts, followed by the<br />

use of Equation [6.3] to calculate the capital recovery. In $1 million units,<br />

P 8 5( PF ,12%,1) $12.46


154 Chapter 6 Annual Worth Analysis<br />

CR 12.46( AP ,12%,8) 0.5( AF ,12%,8)<br />

12.46(0.20130) 0.5(0.08130)<br />

$2.47<br />

The correct interpretation of this result is very important to Lockheed Martin. It means that<br />

each and every year for 8 years, the equivalent total net revenue from the tracker must be at<br />

least $2,470,000 just to recover the initial present worth investment plus the required return of<br />

12% per year. This does not include the AOC of $0.9 million each year.<br />

Annual worth: To determine AW, the cash flows in Figure 6–2a must be converted to an<br />

equivalent AW series over 8 years ( Figure 6–2b ). Since CR $2.47 million is an equivalent<br />

annual cost, as indicated by the minus sign, total AW is determined.<br />

AW 2.47 0.9 $3.37 million per year<br />

This is the AW for all future life cycles of 8 years, provided the costs rise at the same rate as<br />

inflation, and the same costs and services are expected to apply for each succeeding life cycle.<br />

$0.5<br />

0 1 2 3 4 5 6 7 8 0 1 2 3 4 5 6 7 8<br />

$0.9<br />

AW=?<br />

$5.0<br />

Figure 6–2<br />

( a ) Cash flow diagram<br />

for satellite tracker<br />

costs, and ( b ) conversion<br />

to an equivalent<br />

AW (in $1 million),<br />

Example 6.2.<br />

$8.0<br />

(a)<br />

(b)<br />

There is a second, equally correct way to determine CR. Either method results in the same<br />

value. There is a relation between the AP and AF factors.<br />

( AF,i,n ) ( AP,i,n ) i<br />

Both factors are present in the CR Equation [6.3]. Substitute for the AF factor to obtain<br />

CR P ( AP,i,n ) S [( AP,i,n ) i ]<br />

[( P S )( AP,i,n ) S ( i )]<br />

There is a basic logic to this formula. Subtracting S from the initial investment P before applying<br />

the AP factor recognizes that the salvage value will be recovered. This reduces CR, the annual<br />

cost of asset ownership. However, the fact that S is not recovered until year n of ownership is<br />

compensated for by charging the annual interest S ( i ) against the CR. Although either CR relation<br />

results in the same amount, it is better to consistently use the same method. The first method,<br />

Equation [6.3], will be used in this text.<br />

For solution by spreadsheet, use the PMT function to determine CR only in a single spreadsheet<br />

cell. The general function PMT( i %, n,P,F ) is rewritten using the initial investment as<br />

P and S for the salvage value. The format is<br />

PMT( i %, n,P , S ) [6.4]<br />

As an illustration, determine only the CR in Example 6.2. The equivalent initial investment<br />

in year 0 is $12.46 million. The complete function for the CR amount (in $1 million units) is<br />

PMT(12%,8,12.46,0.5). The answer of $2.47 (million) will be displayed in the spreadsheet<br />

cell.<br />

As we learned in Section 3.1, one spreadsheet function can be embedded in another function.<br />

In the case of Example 6.2, the initial investment is distributed over a 2-year period. The one-cell<br />

PMT function, with the PV function embedded (in bold), can be written as PMT(12%,8,<br />

8PV(12%,1, 5) ,0.5) to display the same value CR $2.47.


6.3 Evaluating Alternatives by Annual Worth Analysis 155<br />

6.3 Evaluating Alternatives by Annual<br />

Worth Analysis<br />

The annual worth method is typically the easiest to apply of the evaluation techniques when the<br />

MARR is specified. The AW is calculated over the respective life of each alternative, and the selection<br />

guidelines are the same as those used for the PW method. For mutually exclusive alternatives,<br />

whether cost- or revenue-based, the guidelines are as follows:<br />

One alternative: If AW 0, the requested MARR is met or exceeded and the alternative is<br />

economically justified.<br />

Two or more alternatives: Select the alternative with the AW that is numerically largest,<br />

that is, less negative or more positive. This indicates a lower AW of cost for cost alternatives<br />

or a larger AW of net cash flows for revenue alternatives.<br />

Project evaluation<br />

ME alternative<br />

selection<br />

If any of the three assumptions in Section 6.1 is not acceptable for an alternative, a study period<br />

analysis must be used. Then the cash flow estimates over the study period are converted to AW<br />

amounts. The following two examples illustrate the AW method for one project and two alternatives.<br />

EXAMPLE 6.3<br />

Heavenly Pizza, which is located in Toronto, fares very well with its competition in offering<br />

fast delivery. Many students at the area universities and community colleges work part-time<br />

delivering orders made via the web. The owner, Jerry, a software engineering graduate, plans<br />

to purchase and install five portable, in-car systems to increase delivery speed and accuracy.<br />

The systems provide a link between the web order-placement software and the On-Star system<br />

for satellite-generated directions to any address in the area. The expected result is faster, friendlier<br />

service to customers and larger income.<br />

Each system costs $4600, has a 5-year useful life, and may be salvaged for an estimated<br />

$300. Total operating cost for all systems is $1000 for the first year, increasing by $100 per<br />

year thereafter. The MARR is 10%. Perform an annual worth evaluation for the owner that<br />

answers the following questions. Perform the solution by hand and by spreadsheet.<br />

(a) How much new annual net income is necessary to recover the investment at the MARR<br />

of 10% per year?<br />

(b) Jerry estimates increased net income of $6000 per year for all five systems. Is this project<br />

financially viable at the MARR?<br />

(c) Based on the answer in part ( b ), determine how much new net income Heavenly Pizza<br />

must have to economically justify the project. Operating costs remain as estimated.<br />

Solution by Hand<br />

(a) The capital recovery amount calculated by Equation [6.3] answers the first question.<br />

CR 5[4600( AP ,10%,5)] 5[300( AF ,10%,5)]<br />

5[4600(0.26380)] 5[300(0.16380)]<br />

$5822<br />

The five systems must generate an equivalent annual new revenue of $5822 to recover the<br />

initial investment plus a 10% per year return.<br />

( b) Figure 6–3 presents the cash flows over 5 years. The annual operating cost series,<br />

combined with the estimated $6000 annual income, forms an arithmetic gradient series<br />

with a base amount of $5000 and G $100. The project is financially viable if<br />

AW 0 at i 10% per year. Apply Equation [6.2], where A is the equivalent annual<br />

net income series.<br />

AW CR A 5822 5000 100( AG ,10%,5)<br />

$1003<br />

The system is not financially justified at the net income level of $6000 per year.


156 Chapter 6 Annual Worth Analysis<br />

Figure 6–3<br />

Cash flow diagram used to<br />

compute AW, Example 6.3.<br />

$6000<br />

$1500<br />

0<br />

1 2 3 4<br />

5<br />

$1000<br />

$1100<br />

$1200<br />

$1300<br />

$1400<br />

$23,000<br />

(c) Let the required income equal R, and set the AW relation equal to zero to find the minimum<br />

income to justify the system.<br />

0 5822 ( R 1000) 100( AG ,10%,5)<br />

R 5822 1000 100(1.8101)<br />

$7003 per year<br />

Solution by Spreadsheet<br />

The spreadsheet in Figure 6–4 summarizes the estimates and answers the questions posed<br />

for Heavenly Pizza with the same values that were determined in the hand solution.<br />

Figure 6–4<br />

Spreadsheet solution<br />

of Example 6.3.<br />

( a ) Capital recovery<br />

in cell B16, (b) AW in<br />

cell E17, and ( c ) Goal<br />

Seek template and<br />

outcome in cell B5.<br />

PMT($B$2, 5, NPV($B$2,B10:B14)+B9)<br />

PMT($B$2, 5, NPV($B$2,E10:E14)+E9)<br />

(a) and (b)<br />

(c) Required income to<br />

justify system<br />

(c)


6.4 AW of a Permanent Investment 157<br />

Cell references are used in the spreadsheet functions to accommodate changes in estimated<br />

values.<br />

(a) The capital recovery CR $–5822 is displayed in column B using the PMT function<br />

with an embedded NPV function, as shown in the cell tag.<br />

( b) The annual worth AW $–1003 is displayed in column E using the PMT function<br />

shown. The arithmetic gradient series of costs and estimated income of $6000 in columns<br />

C and D, respectively, are added to obtain the net income necessary for the PMT<br />

function.<br />

( c) The minimum required income is determined in the lower part of Figure 6–4 . This is<br />

easily accomplished by setting AW 0 (column E) in the Goal Seek tool and letting it<br />

find the income per year of $7003 to balance the AW equation.<br />

EXAMPLE 6.4<br />

Luby’s Cafeterias is in the process of forming a separate business unit that provides meals to<br />

facilities for the elderly, such as assisted care and long-term care centers. Since the meals are<br />

prepared in one central location and distributed by trucks throughout the city, the equipment<br />

that keeps food and drink cold and hot is very important. Michele is the general manager of this<br />

unit, and she wishes to choose between two manufacturers of temperature retention units that<br />

are mobile and easy to sterilize after each use. Use the cost estimates below to select the more<br />

economic unit at a MARR of 8% per year.<br />

Hamilton (H)<br />

Infinity Care (IC)<br />

Initial cost P, $ 15,000 20,000<br />

Annual M&O, $year 6,000 9,000<br />

Refurbishment cost, $ 0 2,000 every 4 years<br />

Trade-in value S, % of P 20 40<br />

Life, years 4 12<br />

Solution<br />

The best evaluation technique for these different-life alternatives is the annual worth method,<br />

where AW is taken at 8% per year over the respective lives of 4 and 12 years.<br />

AW H annual equivalent of P annual M&O annual equivalent of S<br />

15,000( AP ,8%,4) 6000 0.2(15,000)( AF ,8%,4)<br />

15,000(0.30192) 6000 3000(0.22192)<br />

$9,863<br />

AW IC annual equivalent of P annual M&O annual equivalent of refurbishment<br />

annual equivalent of S<br />

20,000( AP ,8%,12) 9000 2000[( PF ,8%,4) ( PF ,8%,8)]( AP ,8%,12)<br />

0.4(20,000)( AF ,8%,12)<br />

20,000(0.13270) 9000 2000[0.7350 0.5403](0.13270) 8000(0.05270)<br />

$11,571<br />

The Hamilton unit is considerably less costly on an annual equivalent basis.<br />

If the projects are independent, the AW at the MARR is calculated. All projects with AW 0<br />

are acceptable.<br />

6.4 AW of a Permanent Investment<br />

This section discusses the annual worth equivalent of the capitalized cost introduced in Section<br />

5.5. Evaluation of public sector projects, such as flood control dams, irrigation canals,<br />

bridges, or other large-scale projects, requires the comparison of alternatives that have such long<br />

Independent project<br />

selection


158 Chapter 6 Annual Worth Analysis<br />

lives that they may be considered infinite in economic analysis terms. For this type of analysis,<br />

the annual worth (and capital recovery amount) of the initial investment is the perpetual annual<br />

interest on the initial investment, that is, A Pi (CC) i. This is Equation [5.2].<br />

Cash flows recurring at regular or irregular intervals are handled exactly as in conventional<br />

AW computations; convert them to equivalent uniform annual amounts A for one cycle. This<br />

automatically annualizes them for each succeeding life cycle. Add all the A values to the CR<br />

amount to find total AW, as in Equation [6.2].<br />

EXAMPLE 6.5<br />

The U.S. Bureau of Reclamation is considering three proposals for increasing the capacity of the<br />

main drainage canal in an agricultural region of Nebraska. Proposal A requires dredging the<br />

canal to remove sediment and weeds that have accumulated during previous years’ operation.<br />

The capacity of the canal will have to be maintained in the future near its design peak flow because<br />

of increased water demand. The Bureau is planning to purchase the dredging equipment<br />

and accessories for $650,000. The equipment is expected to have a 10-year life with a $17,000<br />

salvage value. The annual operating costs are estimated to total $50,000. To control weeds in the<br />

canal itself and along the banks, environmentally safe herbicides will be sprayed during the irrigation<br />

season. The yearly cost of the weed control program is expected to be $120,000.<br />

Proposal B is to line the canal with concrete at an initial cost of $4 million. The lining is<br />

assumed to be permanent, but minor maintenance will be required every year at a cost of<br />

$5000. In addition, lining repairs will have to be made every 5 years at a cost of $30,000.<br />

Proposal C is to construct a new pipeline along a different route. Estimates are an initial cost<br />

of $6 million, annual maintenance of $3000 for right-of-way, and a life of 50 years.<br />

Compare the alternatives on the basis of annual worth, using an interest rate of 5% per<br />

year.<br />

Solution<br />

Since this is an investment for a permanent project, compute the AW for one cycle of all recurring<br />

costs. For proposals A and C, the CR values are found using Equation [6.3], with n A 10<br />

and n C 50, respectively. For proposal B, the CR is simply P ( i ).<br />

Proposal A<br />

CR of dredging equipment:<br />

650,000( AP ,5%,10) 17,000( AF ,5%,10) $ 82,824<br />

Annual cost of dredging 50,000<br />

Annual cost of weed control 120,000<br />

$252,824<br />

Proposal B<br />

CR of initial investment: 4,000,000(0.05) $200,000<br />

Annual maintenance cost 5,000<br />

Lining repair cost: 30,000( AF ,5%,5) 5,429<br />

$210,429<br />

Proposal C<br />

CR of pipeline: 6,000,000( AP ,5%,50)<br />

$328,680<br />

Annual maintenance cost 3,000<br />

$331,680<br />

Proposal B, which is a permanent solution, is selected due to its lowest AW of costs.<br />

Comment<br />

Note the use of the AF factor for the lining repair cost in proposal B. The AF factor is used<br />

instead of AP , because the lining repair cost begins in year 5, not year 0, and continues<br />

indefinitely at 5-year intervals.<br />

If the 50-year life of proposal C is considered infinite, CR P ( i ) $300,000, instead of<br />

$328,680 for n 50. This is a small economic difference. How long lives of 40 or more<br />

years are treated economically is a matter of “local” practice.


6.4 AW of a Permanent Investment 159<br />

EXAMPLE 6.6<br />

At the end of each year, all owners and employees at Bell County Utility Cooperative are given<br />

a bonus check based on the net profit of the Coop for the previous year. Bart just received his<br />

bonus in the amount of $8530. He plans to invest it in an annuity program that returns<br />

7% per year. Bart’s long-term plans are to quit the Coop job some years in the future when he is<br />

still young enough to start his own business. Part of his future living expenses will be paid from<br />

the proceeds that this year’s bonus accumulates over his remaining years at the Coop.<br />

(a) Use a spreadsheet to determine the amount of annual year-end withdrawal that he can anticipate<br />

(starting 1 year after he quits) that will continue forever. He is thinking of working<br />

15 or 20 more years.<br />

(b) Determine the amount Bart must accumulate after 15 and 20 years to generate $3000 per<br />

year forever.<br />

Solution by Spreadsheet<br />

(a) Figure 6–5 presents the cash flow diagram for n 15 years of accumulation at 7% per<br />

year on the $8530 deposited now. The accumulated amount after n 15 years is indicated<br />

as F after 15 ? and the withdrawal series starts at the end of year 16. The diagram for<br />

n 20 would appear the same, except there is a 20-year accumulation period.<br />

The spreadsheet in Figure 6–6 shows the functions and answers for n 15 years in<br />

columns C and D. The FV function displays the total accumulated at 7% after 15 years<br />

as $23,535. The perpetual withdrawal is determined by viewing this accumulated amount<br />

as a P value and by applying the formula<br />

A P ( i ) 23,535(0.07) $1647 per year<br />

The spreadsheet function D9*B7 performs this same computation in cell reference format<br />

in column D.<br />

Answers for n 20 years are displayed in column E. At a consistent 7% per year return,<br />

Bart’s perpetual income is estimated as $1647 after 15 years, or $2311 per year if he<br />

waits for 20 years.<br />

i =7%<br />

0 1 2 3 4 13 14 15<br />

A =?<br />

16 17 18 19 20 21 22 …<br />

∞<br />

P = $8530<br />

F after 15 = ?<br />

Figure 6–5<br />

Diagram for a perpetual<br />

series starting after 15<br />

years of accumulation,<br />

Example 6.6.<br />

Figure 6–6<br />

Spreadsheet solution,<br />

Example 6.6.


160 Chapter 6 Annual Worth Analysis<br />

(b) To obtain a perpetual annual withdrawal of $3000, it is necessary to determine how much<br />

must be accumulated 1 year before the first withdrawal of $3000. This is an application of<br />

the relation A P (i) solved for P , or<br />

P — A i ——— 3000<br />

0.07 $42,857<br />

This P value is independent of how long Bart works at the Coop, because he must accumulate<br />

this amount to achieve his goal. Figure 6–6 , row 13, shows the function and result. Note that<br />

the number of years n does not enter into the function 3000B13.<br />

Comment<br />

The NPER function can be used to determine how many years it will take for the current<br />

amount of $8530 to accumulate the required $42,857 at 7% per year. The row 15 function indicates<br />

that Bart will have to work at the Coop for just under 24 additional years.<br />

6.5 Life-Cycle Cost Analysis<br />

The PW and AW analysis techniques discussed thus far have concentrated on estimates for first<br />

cost P , annual operating and maintenance costs (AOC or M&O), salvage value S , and predictable<br />

periodic repair and upgrade costs, plus any revenue estimates that may favor one alternative over<br />

another. There are usually a host of additional costs involved when the complete project life costs<br />

are evaluated. A life-cycle cost analysis includes these additional estimates to the extent that they<br />

can be reliably determined.<br />

Life-cycle cost (LCC) analysis utilizes AW or PW methods to evaluate cost estimates for the entire<br />

life cycle of one or more projects. Estimates will cover the entire life span from the early<br />

conceptual stage, through the design and development stages, throughout the operating stage, and<br />

even the phaseout and disposal stages. Both direct and indirect costs are included to the extent<br />

possible, and differences in revenue and savings projections between alternatives are included.<br />

Some typical LCC applications are life-span analysis for military and commercial aircraft,<br />

new manufacturing plants, new automobile models, new and expanded product lines, and government<br />

systems at federal and state levels. For example, the U.S. Department of Defense<br />

requires that a government contractor include an LCC budget and analysis in the originating<br />

proposal for most defense systems.<br />

Most commonly the LCC analysis includes costs, and the AW method is used for the analysis,<br />

especially if only one alternative is evaluated. If there are expected revenue or other benefit differences<br />

between alternatives, a PW analysis is recommended. Public sector projects are usually<br />

evaluated using a benefit/cost analysis (Chapter 9), rather than LCC analysis, because estimates<br />

to the citizenry are difficult to make with much accuracy. The direct costs mentioned above include<br />

material, human labor, equipment, supplies, and other costs directly related to a product,<br />

process, or system. Some examples of indirect cost components are taxes, management, legal,<br />

warranty, quality, human resources, insurance, software, purchasing, etc. Direct and indirect<br />

costs are discussed further in Chapter 15.<br />

LCC analysis is most effectively applied when a substantial percentage of the life span (postpurchase)<br />

costs, relative to the initial investment, will be expended in direct and indirect operating,<br />

maintenance, and similar costs once the system is operational. For example, the evaluation<br />

of two equipment purchase alternatives with expected useful lives of 5 years and M&O costs of<br />

5% to 10% of the initial investment does not require an LCC analysis. However, let’s assume that<br />

Exxon-Mobil wants to evaluate the design, construction, operation, and support of a new type<br />

and style of tanker that can transport oil over long distances of ocean. If the initial costs are in the<br />

$100 millions with support and operating costs ranging from 25% to 35% of this amount over a<br />

25-year life, the logic of an LCC analysis will offer a better understanding of the economic viability<br />

of the project.<br />

To understand how a LCC analysis works, first we must understand the phases and stages of<br />

systems engineering or systems development. Many books and manuals are available on systems<br />

development and analysis. Generally, the LCC estimates may be categorized into a simplified


6.5 Life-Cycle Cost Analysis 161<br />

format for the major phases of acquisition, operation, and phaseout/disposal , and their respective<br />

stages.<br />

Acquisition phase: all activities prior to the delivery of products and services.<br />

• Requirements definition stage—Includes determination of user/customer needs, assessing<br />

them relative to the anticipated system, and preparation of the system requirements<br />

documentation.<br />

• Preliminary design stage—Includes feasibility study, conceptual, and early-stage plans;<br />

final go–no go decision is probably made here.<br />

• Detailed design stage—Includes detailed plans for resources—capital, human, facilities,<br />

information systems, marketing, etc.; there is some acquisition of assets, if economically<br />

justifiable.<br />

Operation phase: all activities are functioning, products and services are available.<br />

• Construction and implementation stage—Includes purchases, construction, and implementation<br />

of system components; testing; preparation, etc.<br />

• Usage stage—Uses the system to generate products and services; the largest portion of<br />

the life cycle.<br />

Phaseout and disposal phase: covers all activities to transition to a new system; removal/<br />

recycling/disposal of old system.<br />

EXAMPLE 6.7<br />

In the 1860s, General Mills Inc. and Pillsbury Inc. both started in the flour business in the Twin<br />

Cities of Minneapolis–St. Paul, Minnesota. In the decade of 2000 to 2010, General Mills purchased<br />

Pillsbury for a combination cash and stock deal worth more than $10 billion and integrated<br />

the product lines. Food engineers, food designers, and food safety experts made many<br />

cost estimates as they determined the needs of consumers and the combined company’s ability<br />

to technologically and safely produce and market new food products. At this point only cost<br />

estimates have been addressed—no revenues or profits.<br />

Assume that the major cost estimates below have been made based on a 6-month study<br />

about two new products that could have a 10-year life span for the company. Use LCC analysis<br />

at the industry MARR of 18% to determine the size of the commitment in AW terms.<br />

(Time is indicated in product-years. Since all estimates are for costs, they are not preceded by<br />

a minus sign.)<br />

Consumer habits study (year 0)<br />

Preliminary food product design (year 1)<br />

Preliminary equipment/plant design (year 1)<br />

Detail product designs and test marketing (years 1, 2)<br />

Detail equipment/plant design (year 2)<br />

Equipment acquisition (years 1 and 2)<br />

Current equipment upgrades (year 2)<br />

New equipment purchases (years 4 and 8)<br />

Annual equipment operating cost (AOC) (years 3–10)<br />

$0.5 million<br />

0.9 million<br />

0.5 million<br />

1.5 million each year<br />

1.0 million<br />

$2.0 million each year<br />

1.75 million<br />

2.0 million (year 4) <br />

10% per purchase<br />

thereafter<br />

200,000 (year 3) <br />

4% per year thereafter<br />

Marketing, year 2<br />

$8.0 million<br />

years 3–10 5.0 million (year 3)<br />

and 0.2 million<br />

per year thereafter<br />

year 5 only<br />

3.0 million extra<br />

Human resources, 100 new employees for 2000 hours<br />

per year (years 3–10)<br />

Phaseout and disposal (years 9 and 10)<br />

$20 per hour (year 3) <br />

5% per year<br />

$1.0 million each year


162 Chapter 6 Annual Worth Analysis<br />

Solution<br />

LCC analysis can get complicated rapidly due to the number of elements involved. Calculate<br />

the PW by phase and stage, add all PW values, then find the AW over 10 years. Values are in<br />

$1 million units.<br />

Acquisition phase:<br />

Requirements definition: consumer study<br />

PW $0.5<br />

Preliminary design: product and equipment<br />

PW 1.4( PF ,18%,1) $1.187<br />

Detailed design: product and test marketing, and equipment<br />

PW 1.5( PA ,18%,2) 1.0( PF ,18%,2) $3.067<br />

Operation phase:<br />

Construction and implementation: equipment and AOC<br />

PW 2.0( PA ,18%,2) 1.75( PF ,18%,2) 2.0( PF ,18%,4) 2.2( PF ,18%,8)<br />

Use: marketing<br />

1 (——<br />

1.04<br />

0.2<br />

1.18 ) 8<br />

<br />

————— ( PF ,18%,2) $6.512<br />

0.14<br />

PW 8.0( PF ,18%,2) [5.0( PA ,18%,8) 0.2( PG ,18%,8)]( PF ,18%,2)<br />

3.0( PF ,18%,5)<br />

$20.144<br />

Use: human resources: (100 employees)(2000 hyr)($20h) $4.0 million in year 3<br />

1 (——<br />

1.05<br />

1.18 ) 8<br />

<br />

PW 4.0 ————— ( PF ,18%,2) $13.412<br />

0.13<br />

Phaseout phase:<br />

PW 1.0( PA ,18%,2)( PF ,18%,8) $0.416<br />

The sum of all PW of costs is PW $45.238 million. Finally, determine the AW over the<br />

expected 10-year life span.<br />

AW 45.238 million( AP ,18%,10) $10.066 million per year<br />

This is the LCC estimate of the equivalent annual commitment to the two proposed<br />

products.<br />

Often the alternatives compared by LCC do not have the same level of output or amount of<br />

usage. For example, if one alternative will produce 20 million units per year and a second alternative<br />

will operate at 35 million per year, the AW values should be compared on a currency unit/<br />

unit produced basis, such as dollar/unit or euro/hour operated.<br />

Figure 6–7 presents an overview of how costs may be distributed over an entire life cycle. For<br />

some systems, typically defense systems, operating and maintenance costs rise fast after acquisition<br />

and remain high until phaseout occurs.<br />

The total LCC for a system is established or locked in early in the life cycle. It is not unusual<br />

to have 75% to 85% of the entire life span LCC committed during the preliminary and detail


6.5 Life-Cycle Cost Analysis 163<br />

Costs<br />

Requirements<br />

and<br />

design<br />

Acquisition and<br />

installation<br />

Operating and<br />

maintenance<br />

Phaseout<br />

and disposal<br />

Start<br />

Time<br />

End of<br />

life cycle<br />

Figure 6–7<br />

Typical distribution of life-cycle costs of the phases for one<br />

life cycle.<br />

D<br />

%<br />

B<br />

%<br />

B<br />

Reduced<br />

total LCC<br />

Committed<br />

Committed<br />

Cumulative LCC<br />

#1<br />

Actual<br />

Cumulative LCC<br />

#2<br />

#1<br />

E<br />

F<br />

C<br />

A<br />

Time<br />

A<br />

Time<br />

Acquisition<br />

phase<br />

Operation<br />

phase<br />

Acquisition<br />

phase<br />

Operation<br />

phase<br />

(a)<br />

Figure 6–8<br />

LCC envelopes for committed and actual costs: ( a ) design 1, ( b ) improved design 2.<br />

(b)<br />

design stages. As shown in Figure 6–8 a , the actual or observed LCC (bottom curve AB ) will trail<br />

the committed LCC throughout the life span (unless some major design flaw increases the total<br />

LCC of design #1 above point B ).<br />

The potential for signifi cantly reducing total LCC occurs primarily during the early stages. A<br />

more effective design and more efficient equipment can reposition the envelope to design #2 in<br />

Figure 6–8 b . Now the committed LCC curve AEC is below AB at all points, as is the actual LCC<br />

curve AFC . It is this lower envelope #2 we seek. The shaded area represents the reduction in<br />

actual LCC.<br />

Even though an effective LCC envelope may be established early in the acquisition phase,<br />

it is not uncommon that unplanned cost-saving measures are introduced during the acquisition<br />

phase and early operation phase. These apparent “savings” may actually increase the total<br />

LCC, as shown by curve AFD . This style of ad hoc cost savings, often imposed by management<br />

early in the design stage and/or construction stage, can substantially increase costs later,<br />

especially in the after-sale portion of the use stage. For example, the use of inferior-strength<br />

concrete and steel has been the cause of structural failures many times, thus increasing the<br />

overall life span LCC.


164 Chapter 6 Annual Worth Analysis<br />

CHAPTER SUMMARY<br />

The annual worth method of comparing alternatives is often preferred to the present worth method,<br />

because the AW comparison is performed for only one life cycle. This is a distinct advantage when<br />

comparing different-life alternatives. AW for the first life cycle is the AW for the second, third, and<br />

all succeeding life cycles, under certain assumptions. When a study period is specified, the AW<br />

calculation is determined for that time period, regardless of the lives of the alternatives.<br />

For infinite-life (perpetual) alternatives, the initial cost is annualized simply by multiplying P<br />

by i. For finite-life alternatives, the AW through one life cycle is equal to the perpetual equivalent<br />

annual worth.<br />

Life-cycle cost analysis is appropriate for systems that have a large percentage of costs in<br />

operating and maintenance. LCC analysis helps in the analysis of all costs from design to<br />

operation to disposal phases.<br />

PROBLEMS<br />

Annual Worth Calculations<br />

6.1 If you are asked to provide an annual worth (AW)<br />

comparison of alternatives after a present worth<br />

(PW) comparison has already been done, what factor<br />

multiplied by the PW values provide the correct<br />

AW values?<br />

6.2 List three assumptions that are inherent in the<br />

annual worth method of comparing alternatives.<br />

6.3 In the annual worth method of comparing alternatives<br />

that have different lives, why do you calculate<br />

the AW of the alternatives over their respective<br />

life cycles instead of over the least common<br />

multiple of their lives?<br />

6.4 James developed the two cash flow diagrams shown<br />

at the bottom of this page. The cash flows for alternative<br />

B represent two life cycles of A. Calculate<br />

the annual worth value of each over the<br />

respective life cycles to demonstrate that they are<br />

the same. Use an interest rate of 10% per year.<br />

6.5 An asset with a first cost of $20,000 has an annual<br />

operating cost of $12,000 and a $4000 salvage value<br />

after its 4-year life. If the project will be needed for<br />

6 years, what would the market (salvage) value of<br />

the 2-year-old asset have to be for the annual worth<br />

to be the same as it is for one life cycle of the asset?<br />

Use an interest rate of 10% per year.<br />

6.6 A sports mortgage is an innovative way to finance<br />

cash-strapped sports programs by allowing fans to<br />

sign up to pay a “mortgage” for the right to buy<br />

good seats at football games for several decades<br />

with season tickets locked in at current prices. At<br />

Notre Dame, the locked-in price period is 50 years.<br />

If a fan pays a $130,000 “mortgage” fee now (i.e.,<br />

in year 0) when season tickets are selling for $290<br />

each, what is the equivalent annual cost of the<br />

football tickets over the 50-year period at an interest<br />

rate of 8% per year?<br />

6.7 If a fan buys a sports mortgage to USC football<br />

games by paying $130,000 in 10 equal payments<br />

starting now , and then pays a fixed price of $290<br />

per year for 50 years (starting 1 year from now)<br />

for season tickets, what is the AW in years 1<br />

through 50 of the season tickets at 8% per year<br />

interest?<br />

Alternative A<br />

Alternative B<br />

$1000<br />

i = 10% per year<br />

$1000<br />

0 1 2 3<br />

Year<br />

0<br />

1<br />

2 3 4 5 6<br />

Year<br />

$25 $25 $25<br />

$25 $25 $25<br />

$25 $25 $25<br />

$5000<br />

$5000<br />

$4000


Problems 165<br />

6.8 Eight years ago, Ohio Valley Trucking purchased a<br />

large-capacity dump truck for $115,000 to provide<br />

short-haul earthmoving services. The company<br />

sold it today for $45,000. Operating and maintenance<br />

costs averaged $10,500 per year. A complete<br />

overhaul at the end of year 4 cost an extra $3600.<br />

Calculate the annual cost of the truck at 8% per<br />

year interest.<br />

6.9 A major repair on the suspension system of a<br />

5-year-old car cost $2000 because the warranty<br />

expired after 3 years of ownership. The cost of<br />

periodic maintenance has been $800 every 2 years.<br />

If the owner donates the car to charity after 8 years<br />

of ownership, what is the equivalent annual cost of<br />

the repair and maintenance in the 8-year period<br />

of ownership? Use an interest rate of 8% per year,<br />

and assume that the owner paid the $800 maintenance<br />

cost immediately before donating the car in<br />

year 8.<br />

Capital Recovery<br />

6.10 Ten years ago, Jacobson Recovery purchased a<br />

wrecker for $285,000 to move disabled 18-wheelers.<br />

He anticipated a salvage value of $50,000 after 10<br />

years. During this time his average annual revenue<br />

totaled $52,000. ( a ) Did he recover his investment<br />

and a 12% per year return? ( b ) If the annual M&O<br />

cost was $10,000 the first year and increased by a<br />

constant $1000 per year, was the AW positive or<br />

negative at 12% per year? Assume the $50,000 salvage<br />

was realized.<br />

6.11 Sylvia has received a $500,000 inheritance from<br />

her favorite, recently deceased aunt in Hawaii.<br />

Sylvia is planning to purchase a condo in Hawaii<br />

in the same area where her aunt lived all her life<br />

and to rent it to vacationers. She hopes to make 8%<br />

per year on this purchase over an ownership period<br />

of 20 years. The condo’s total first cost is $500,000,<br />

and she conservatively expects to sell it for 90% of<br />

the purchase price. No annual M&O costs are considered<br />

in the analysis. ( a ) What is the capital recovery<br />

amount? ( b ) If there is a real boom in rental<br />

real estate 10 years in the future, what sales price<br />

(as a percentage of original purchase price) is necessary<br />

at that time (year 10) to realize the same<br />

amount as the 8% return expected over the 20-year<br />

ownership period?<br />

6.12 Humana Hospital Corporation installed a new MRI<br />

machine at a cost of $750,000 this year in its medical<br />

professional clinic in Cedar Park. This state-ofthe-art<br />

system is expected to be used for 5 years<br />

and then sold for $75,000. Humana uses a return<br />

requirement of 24% per year for all of its medical<br />

diagnostic equipment. As a bioengineering student<br />

currently serving a coop semester on the management<br />

staff of Humana Corporation in Louisville,<br />

Kentucky, you are asked to determine the minimum<br />

revenue required each year to realize the expected<br />

recovery and return. Also, you are asked to<br />

draw two cash flow diagrams, one showing the<br />

MRI purchase and sale cash flow and a second depicting<br />

the required capital recovery each year.<br />

Alternative Comparison<br />

6.13 Polypropylene wall caps, used for covering exterior<br />

vents for kitchen cooktops, bathroom fans,<br />

dryers, and other building air exhausts, can be<br />

made by two different methods. Method X will<br />

have a first cost of $75,000, an operating cost of<br />

$32,000 per year, and a $9000 salvage value after<br />

4 years. Method Y will have a first cost of $140,000,<br />

an operating cost of $24,000 per year, and a<br />

$19,000 salvage value after its 4-year life. At an<br />

interest rate of 10% per year, which method should<br />

be used on the basis of an annual worth analysis?<br />

6.14 Nissan’s all-electric car, the Leaf, has a base price<br />

of $32,780 in the United States, but it is eligible<br />

for a $7500 federal tax credit. A consulting engineering<br />

company wants to evaluate the purchase<br />

or lease of one of the vehicles for use by its employees<br />

traveling to job sites in the local area. The<br />

cost for leasing the vehicle will be $4200 per year<br />

(payable at the end of each year) after an initialization<br />

charge of $2500 paid now. If the company<br />

purchases the vehicle, it will also purchase a home<br />

charging station for $2200 that will be partially<br />

offset by a 50% tax credit. If the company expects<br />

to be able to sell the car and charging station for<br />

40% of the base price of the car alone at the end of<br />

3 years, should the company purchase or lease the<br />

car? Use an interest rate of 10% per year and annual<br />

worth analysis.<br />

6.15 A new structural design software package is available<br />

for analyzing and designing three-sided guyed<br />

towers and three- and four-sided self-supporting<br />

towers. A single-user license will cost $6000 per<br />

year. A site license has a one-time cost of $22,000.<br />

A structural engineering consulting company is<br />

trying to decide between two alternatives: buy a<br />

single-user license now and one each year for the<br />

next 3 years (which will provide 4 years of service),<br />

or buy a site license now. Determine which<br />

strategy should be adopted at an interest rate of<br />

10% per year for a 4-year planning period using<br />

the annual worth method of evaluation.<br />

6.16 The city council in a certain southwestern city is<br />

considering whether to construct permanent restrooms<br />

in 22 of its smaller parks (i.e., parks of less


166 Chapter 6 Annual Worth Analysis<br />

than 12 acres) or pay for portable toilets on a yearround<br />

basis. The cost of constructing the 22 permanent<br />

restrooms will be $3.8 million. The 22<br />

portable restrooms can be rented for $7500 each<br />

for 1 year. The service life of a permanent restroom<br />

is 20 years. Using an interest rate of 6% per<br />

year and an annual worth analysis, determine if the<br />

city should build the permanent restrooms or lease<br />

the portable ones.<br />

6.17 A remotely located air sampling station can be powered<br />

by solar cells or by running an above ground<br />

electric line to the site and using conventional<br />

power. Solar cells will cost $16,600 to install and<br />

will have a useful life of 5 years with no salvage<br />

value. Annual costs for inspection, cleaning, etc.,<br />

are expected to be $2400. A new power line will<br />

cost $31,000 to install, with power costs expected<br />

to be $1000 per year. Since the air sampling project<br />

will end in 5 years, the salvage value of the line is<br />

considered to be zero. At an interest rate of 10% per<br />

year, ( a ) which alternative should be selected on the<br />

basis of an annual worth analysis and ( b ) what must<br />

be the first cost of the above ground line to make the<br />

two alternatives equally attractive economically?<br />

6.18 The cash flows for two small raw water treatment<br />

systems are shown. Determine which should be<br />

selected on the basis of an annual worth analysis at<br />

10% per year interest.<br />

MF UF<br />

First cost, $ –33,000 –51,000<br />

Annual cost, $ per year –8,000 –3,500<br />

Salvage value, $ 4,000 11,000<br />

Life, years 3 6<br />

6.19 PGM Consulting is under contract to Montgomery<br />

County for evaluating alternatives that use a<br />

robotic, liquid-propelled “pig” to periodically inspect<br />

the interior of buried potable water pipes for<br />

leakage, corrosion, weld strength, movement over<br />

time, and a variety of other parameters. Two<br />

equivalent robot instruments are available. Robot<br />

Joeboy will have a first cost of $85,000, annual<br />

M&O costs of $30,000, and a $40,000 salvage<br />

value after 3 years. Robot Watcheye will have a<br />

first cost of $125,000, annual M&O costs of<br />

$27,000, and a $33,000 salvage value after its<br />

5-year life. Assume an interest rate of 8% per year.<br />

(a)<br />

(b)<br />

Which robot is the better economic option?<br />

Using the spreadsheet Goal Seek tool, determine<br />

the first cost of the robot not selected in<br />

( a ) so that it will be the economic selection.<br />

6.20 TT Racing and Performance Motor Corporation<br />

wishes to evaluate two alternative CNC machines<br />

for NHRA engine building. Use the AW method at<br />

10% per year to select the better alternative.<br />

Machine R<br />

Machine S<br />

First cost, $ 250,000 370,500<br />

Annual operating cost, 40,000 –50,000<br />

$ per year<br />

Salvage value, $ 20,000 30,000<br />

Life, years 3 5<br />

6.21 The maintenance and operation (M&O) cost of<br />

front-end loaders working under harsh environmental<br />

conditions tends to increase by a constant<br />

$1200 per year for the first 5 years of operation.<br />

For a loader that has a first cost of $39,000 and<br />

first-year M&O cost of $17,000, compare the<br />

equivalent annual worth of a loader kept for 4 years<br />

with one kept for 5 years at an interest rate of<br />

12% per year. The salvage value of a used loader is<br />

$23,000 after 4 years and $18,000 after 5 years.<br />

6.22 You work for Midstates Solar Power. A manager<br />

asked you to determine which of the following<br />

two machines will have the lower ( a ) capital recovery<br />

and ( b ) equivalent annual total cost. Machine<br />

Semi2 has a first cost of $80,000 and an<br />

operating cost of $21,000 in year 1, increasing<br />

by $500 per year through year 5, after which<br />

time it will have a salvage value of $13,000. Machine<br />

Auto1 has a first cost of $62,000 and an<br />

operating cost of $21,000 in year 1, increasing<br />

by 8% per year through year 5, after which time<br />

it will have a scavenge value of $2000. Utilize an<br />

interest rate of 10% per year to determine both<br />

estimates.<br />

Permanent Investments<br />

6.23 The State of Chiapas, Mexico, decided to fund a<br />

program for literacy. The first cost is $200,000<br />

now, and an update budget of $100,000 every<br />

7 years forever is requested. Determine the perpetual<br />

equivalent annual cost at an interest rate of<br />

10% per year.<br />

6.24 Calculate the perpetual equivalent annual cost<br />

(years 1 through infinity) of $5 million in year 0,<br />

$2 million in year 10, and $100,000 in years 11<br />

through infinity. Use an interest rate of 10% per<br />

year.<br />

6.25 A Pennsylvania coal mining operation has installed<br />

an in-shaft monitoring system for oxygen<br />

tank and gear readiness for emergencies. Based on<br />

maintenance patterns for previous systems, there<br />

are no maintenance costs for the first 2 years, they


Problems 167<br />

increase for a time period, and then they level off.<br />

Maintenance costs are expected to be $150,000 in<br />

year 3, $175,000 in year 4, and amounts increasing<br />

by $25,000 per year through year 6 and remain<br />

constant thereafter for the expected 10-year<br />

life of the system. If similar systems with similar<br />

costs will replace the current one, determine the<br />

perpetual equivalent annual maintenance cost at<br />

i 10% per year.<br />

6.26 Compare two alternatives for a security system<br />

surrounding a power distribution substation using<br />

annual worth analysis and an interest rate of 10%<br />

per year.<br />

Condi<br />

Torro<br />

First cost, $ 25,000 130,000<br />

Annual cost, $ per year 9,000 2,500<br />

Salvage value, $ 3,000 150,000<br />

Life, years 3 <br />

6.27 A new bridge across the Allegheny River in<br />

Pittsburgh is expected to be permanent and will<br />

have an initial cost of $30 million. This bridge must<br />

be resurfaced every 5 years at a cost of $1 million.<br />

The annual inspection and operating costs are estimated<br />

to be $50,000. Determine its equivalent annual<br />

worth at an interest rate of 10% per year.<br />

6.28 For the cash flows shown, use an annual worth<br />

comparison and an interest rate of 10% per year.<br />

( a) Determine the alternative that is economically<br />

best.<br />

( b) Determine the first cost required for each of<br />

the two alternatives not selected in ( a ) so that<br />

all alternatives are equally acceptable. Use a<br />

spreadsheet to answer this question.<br />

X Y Z<br />

First cost, $ 90,000 400,000 650,000<br />

Annual cost, 40,000 20,000 13,000<br />

$ per year<br />

Overhaul every — — 80,000<br />

10 years, $<br />

Salvage value, $ 7,000 25,000 200,000<br />

Life, years 3 10 <br />

Life-Cycle Cost<br />

6.29 Blanton Agriculture of Santa Monica, California,<br />

offers different types and levels of irrigation<br />

water conservation systems for use in areas where<br />

groundwater depletion is a serious concern. A<br />

large farming corporation in India, where depletion<br />

is occurring at an alarming rate of 1.6 inches<br />

(4 centimeters) per year due to exponential growth<br />

in usage for crop irrigation, is considering the<br />

purchase of one of the Blanton systems. There are<br />

three options with two levels of automation for<br />

the first two options. The estimated costs and associated<br />

cash flow diagrams over a 10-year period<br />

are summarized below and on the next page,<br />

respectively, for each of the five alternatives.<br />

Costs are categorized as design (Des), development<br />

(Dev), and operation (Oper). For alternatives<br />

A and B, there is an extra cost of $15,000 per<br />

installation year to maintain the manual system in<br />

place now. Level 2 development costs are distributed<br />

equally over a 2-year period. Alternative C<br />

is a retrofit of the current manual system with no<br />

design or development costs, and there is no level<br />

1 option. At an interest rate of 10% per year and a<br />

10-year study period, determine which alternative<br />

and level has the lowest LCC.<br />

Alternative<br />

Cost<br />

Component<br />

Level<br />

1<br />

Level<br />

2<br />

A Des $100,000 $200,000<br />

Dev 175,000 350,000<br />

Oper 60,000 55,000<br />

Install time 1 year 2 years<br />

B Des $ 50,000 $100,000<br />

Dev 200,000 500,000<br />

Oper 45,000 30,000<br />

Install time 1 year 2 years<br />

C Oper $100,000<br />

6.30 The Pentagon asked a defense contractor to estimate<br />

the life-cycle cost for a proposed light-duty<br />

support vehicle. The list of items included the following<br />

categories: R&D costs (R&D), nonrecurring<br />

investment costs (NRI), recurring upgrade costs<br />

(RU), scheduled and unscheduled maintenance<br />

costs (Maint), equipment usage costs (Equip), and<br />

phaseoutdisposal costs (PoD). Use the cost estimates<br />

(shown in $1 million) for the 20-year life<br />

cycle to calculate the annual LCC at an interest rate<br />

of 7% per year.<br />

Year R&D NRI RU Maint Equip PoD<br />

0 5.5 1.1<br />

1 3.5<br />

2 2.5<br />

3 0.5 5.2 1.3 0.6 1.5<br />

4 10.5 3.1 1.4 3.6<br />

5 10.5 4.2 1.6 5.3<br />

6–10 6.5 2.7 7.8<br />

11 on 2.2 3.5 8.5<br />

18–20 2.7


168 Chapter 6 Annual Worth Analysis<br />

Alternative A<br />

Level 1<br />

0 1 2 3 4 5 6 7 8 9 10<br />

$100,000<br />

$60,000<br />

$175,000<br />

15,000<br />

Alternative A<br />

Level 2<br />

0 1 2 3 4 5 6 7 8 9 10<br />

$55,000<br />

$200,000<br />

$175,000<br />

15,000<br />

0 1 2 3 4 5 6 7 8 9 10<br />

Alternative B<br />

Level 1<br />

$50,000<br />

$45,000<br />

$200,000<br />

15,000<br />

Alternative B<br />

Level 2<br />

0 1 2 3 4 5 6 7 8 9 10<br />

$30,000<br />

$100,000<br />

$250,000<br />

15,000<br />

Alternative C<br />

Level 2<br />

0 1 2 3 4 5 6 7 8 9 10<br />

Cash flow diagrams for Problem 6.29<br />

$100,000<br />

6.31 A manufacturing software engineer at a major<br />

aerospace corporation has been assigned the<br />

management responsibility of a project to design,<br />

build, test, and implement AREMSS, a<br />

new- generation automated scheduling system<br />

for routine and expedited maintenance. Reports<br />

on the disposition of each service will also be<br />

entered by field personnel, then filed and archived<br />

by the system. The initial application will<br />

be on existing Air Force in-flight refueling aircraft.<br />

The system is expected to be widely used<br />

over time for other aircraft maintenance scheduling.<br />

Once it is fully implemented, enhancements<br />

will have to be made, but the system is expected<br />

to serve as a worldwide scheduler for up to<br />

15,000 separate aircraft. The engineer, who must<br />

make a presentation next week of the best estimates<br />

of costs over a 20-year life period, has decided<br />

to use the life-cycle cost approach of cost<br />

estimations. Use the following information to<br />

determine the current annual LCC at 6% per year<br />

for the AREMSS scheduling system.


Additional Problems and FE Exam Review Questions 169<br />

Cost in Year ($ millions)<br />

Cost Category 1 2 3 4 5 6 on 10 18<br />

Field study 0.5<br />

Design of system 2.1 1.2 0.5<br />

Software design 0.6 0.9<br />

Hardware purchases 5.1<br />

Beta testing 0.1 0.2<br />

Users manual<br />

0.1 0.1 0.2 0.2 0.06<br />

development<br />

System implementation 1.3 0.7<br />

Field hardware 0.4 6.0 2.9<br />

Training trainers 0.3 2.5 2.5 0.7<br />

Software upgrades 0.6 3.0 3.7<br />

6.32 The U.S. Army received two proposals for a turnkey<br />

design-build project for barracks for infantry unit soldiers<br />

in training. Proposal A involves an off-the-shelf<br />

“bare-bones” design and standard grade construction<br />

of walls, windows, doors, and other features. With<br />

this option, heating and cooling costs will be greater,<br />

maintenance costs will be higher, and replacement<br />

will be sooner than for proposal B. The initial cost for<br />

A will be $750,000. Heating and cooling costs will<br />

average $72,000 per year, with maintenance costs<br />

averaging $24,000 per year. Minor remodeling will<br />

be required in years 5, 10, and 15 at a cost of $150,000<br />

each time in order to render the units usable for<br />

20 years. They will have no salvage value.<br />

Proposal B will include tailored design and<br />

construction costs of $1.1 million initially, with<br />

estimated heating and cooling costs of $36,000 per<br />

year and maintenance costs of $12,000 per year.<br />

There will be no salvage value at the end of the<br />

20-year life. Which proposal should be accepted<br />

on the basis of an annual life-cycle cost analysis, if<br />

the interest rate is 6% per year?<br />

6.33 A medium-size municipality plans to develop a<br />

software system to assist in project selection during<br />

the next 10 years. A life-cycle cost approach<br />

has been used to categorize costs into development,<br />

programming, operating, and support costs<br />

for each alternative. There are three alternatives<br />

under consideration, identified as M, N, and O.<br />

The costs are summarized below. Use an annual<br />

life-cycle cost approach to identify the best alternative<br />

at 8% per year.<br />

Alternative Component Estimated Cost<br />

M Development $250,000 now,<br />

$150,000 years 1–4<br />

Programming $45,000 now,<br />

$35,000 years 1, 2<br />

Operation $50,000 years 1–10<br />

Support $30,000 years 1–5<br />

N Development $10,000 now<br />

Programming $45,000 year 0,<br />

$30,000 years 1–3<br />

Operation $80,000 years 1–10<br />

Support $40,000 years 1–10<br />

O Operation $175,000 years 1–10<br />

ADDITIONAL PROBLEMS AND FE EXAM REVIEW QUESTIONS<br />

6.34 All of the following are fundamental assumptions<br />

for the annual worth method of analysis except :<br />

( a) The alternatives will be needed for only one<br />

life cycle.<br />

( b) The services provided are needed for at least<br />

the LCM of the lives of the alternatives.<br />

( c) The selected alternative will be repeated for<br />

the succeeding life cycles in exactly the same<br />

manner as for the first life cycle.<br />

( d) All cash flows will have the same estimated<br />

values in every life cycle.<br />

6.35 When comparing five alternatives that have different<br />

lives by the AW method, you must:<br />

( a) Find the AW of each over the life of the<br />

longest-lived alternative.<br />

( b) Find the AW of each over the life of the<br />

shortest-lived alternative.<br />

( c) Find the AW of each over the LCM of all of<br />

the alternatives.<br />

( d) Find the AW of each alternative over its life<br />

without considering the life of the other<br />

alternatives.<br />

6.36 The annual worth of an alternative can be calculated<br />

from the alternative’s:<br />

( a) Present worth by multiplying by ( A/P , i , n )<br />

(b) Future worth by multiplying by ( F/A , i , n )<br />

(c) Either ( a ) or ( b )<br />

(d) Neither ( a ) nor ( b )<br />

6.37 The alternatives shown are to be compared on the<br />

basis of annual worth. At an interest rate of 10%<br />

per year, the values of n that you could use in the<br />

( AP,i,n ) factors to make a correct comparison by<br />

the annual worth method are:<br />

First cost, $ 50,000 90,000<br />

Annual cost, $ per year 10,000 4,000<br />

Salvage value, $ 13,000 15,000<br />

Life, years 3 6<br />

(a) n 3 years for A and 3 years for B<br />

(b) n 3 years for A and 6 years for B<br />

(c) Either ( a ) or ( b )<br />

(d) Neither ( a ) nor ( b )<br />

A<br />

B


170 Chapter 6 Annual Worth Analysis<br />

6.38 The alternatives shown are to be compared on the<br />

basis of a perpetual (i.e., forever) equivalent annual<br />

worth. At an interest rate of 10% per year, the<br />

equation that represents the perpetual AW of X1 is:<br />

First cost, $ 50,000 90,000<br />

Annual cost, $ per year 10,000 4,000<br />

Salvage value, $ 13,000 15,000<br />

Life, years 3 6<br />

X1<br />

(a) AW X1 50,000(0.10) 10,000<br />

13,000 (0.10)<br />

(b) AW X1 50,000(0.10) 10,000<br />

13,000 ( A/F ,10%,3)<br />

(c) AW X1 50,000(0.10) 10,000<br />

37,000 ( P/F ,10%,3)(0.10)<br />

13,000(0.10)<br />

(d) AW X1 50,000( A/P ,10%,3) 10,000<br />

13,000( A/F ,10%,3)<br />

6.39 To get the AW of a cash flow of $10,000 that<br />

occurs every 10 years forever, with the first one<br />

occurring 10 years from now, you should:<br />

(a) Multiply $10,000 by ( A/P,i ,10).<br />

(b) Multiply $10,000 by ( A/F,i ,10).<br />

(c) Multiply $10,000 by i .<br />

(d) Multiply $10,000 by ( A/F,i,n ) and then<br />

multiply by i .<br />

Problems 6.40 through 6.43 refer to the following<br />

estimates.<br />

The alternatives are mutually exclusive and the MARR is<br />

6% per year.<br />

Y1<br />

Vendor 1 Vendor 2 Vendor 3<br />

First cost, $ 200,000 550,000 1,000,000<br />

Annual cost, $ per year 50,000 20,000 10,000<br />

Revenue, $ per year 120,000 120,000 110,000<br />

Salvage value, $ 25,000 0 500,000<br />

Life, years 10 15 <br />

6.40 The annual worth of vendor 2 cash flow estimates<br />

is closest to:<br />

(a) $63,370<br />

(b) $43,370<br />

(c) $43,370<br />

(d) $63,370<br />

6.41 Of the following three relations, the correct one or ones<br />

to calculate the annual worth of vendor 1 cash flow estimates<br />

is ( note : all dollar values are in thousands):<br />

Relation 1: AW 1 200( AP ,6%,10) 70 <br />

25( AF ,6%,10)<br />

Relation 2: AW 1 [200 50( PA ,6%,10) <br />

120( PA ,6%,10)<br />

25( PF ,6%,10)](AP ,6%,10)<br />

Relation 3:<br />

(a) 1 and 3<br />

(b) Only 1<br />

(c) 1 and 2<br />

(d) Only 3<br />

AW 1 200( FP ,6%,10) 25 <br />

(50 120)( AP ,6%,10)<br />

6.42 The AW values for the alternatives are listed<br />

below. The vendor or vendors that should be recommended<br />

is:<br />

AW 1 $44,723 AW 2 $43,370<br />

AW 3 $40,000<br />

(a) 1 and 2<br />

(b) 3<br />

(c) 2<br />

(d) 1<br />

6.43 The capital recovery amount for vendor 3 is:<br />

(a) $40,000 per year<br />

(b) $60,000 per year<br />

(c) $43,370 per year<br />

(d) $100,000 per year<br />

6.44 If a revenue alternative has a negative AW value and<br />

it was correctly calculated, it means the following:<br />

(a) The equivalent annual worth of revenues<br />

does not exceed that of the costs.<br />

(b)<br />

(c)<br />

The estimates are wrong somewhere.<br />

A minus or plus sign of a cash flow was entered<br />

incorrectly into the PMT spreadsheet<br />

function.<br />

(d) The alternative should have a longer life so<br />

revenues will exceed costs.<br />

6.45 Estimates for one of two process upgrades are as<br />

follows: first cost of $40,000, annual cost of $5000<br />

per year, market value that decreases by $2000 per<br />

year to the salvage value of $20,000 after the expected<br />

life of 10 years. If a 4-year study period is<br />

used for AW analysis at 15% per year, the correct<br />

AW value is closest to:<br />

(a) $15,000<br />

(b) $11,900<br />

(c) $7600<br />

(d) $12,600<br />

6.46 The perpetual annual worth of investing $50,000<br />

now and $20,000 per year starting in year 16 and<br />

continuing forever at 12% per year is closest to:<br />

(a) $4200<br />

(b) $8650<br />

(c) $9655<br />

(d) $10,655


Case Study 171<br />

6.47 All the following statements about the capital recovery<br />

amount for an alternative are false except:<br />

( a) Annual revenue can be no more than this<br />

amount, if the alternative is selected.<br />

( b) A monetary estimate of new capital funds required<br />

each year for the life of the alternative.<br />

(c) An amount of revenue required to recover<br />

the first cost plus a stated return over the life<br />

of the alternative.<br />

(d) Does not consider the salvage value, since it<br />

is returned at the end of the alternative’s life.<br />

CASE STUDY<br />

THE CHANGING SCENE OF AN ANNUAL WORTH ANALYSIS<br />

Background and Information<br />

Harry, owner of an automobile battery distributorship in<br />

Atlanta, Georgia, performed an economic analysis 3 years ago<br />

when he decided to place surge protectors in-line for all his<br />

major pieces of testing equipment. The estimates used and the<br />

annual worth analysis at MARR 15% are summarized<br />

below. Two different manufacturers’ protectors were compared.<br />

PowrUp<br />

Lloyd’s<br />

Cost and installation, $ 26,000 36,000<br />

Annual maintenance cost, 800 300<br />

$ per year<br />

Salvage value, $ 2,000 3,000<br />

Equipment repair savings, $ 25,000 35,000<br />

Useful life, years 6 10<br />

The spreadsheet in Figure 6–9 is the one Harry used to make<br />

the decision. Lloyd’s was the clear choice due to its substantially<br />

larger AW value. The Lloyd’s protectors were installed.<br />

During a quick review this last year (year 3 of operation),<br />

it was obvious that the maintenance costs and repair savings<br />

have not followed (and will not follow) the estimates made<br />

3 years ago. In fact, the maintenance contract cost (which includes<br />

quarterly inspection) is going from $300 to $1200 per<br />

year next year and will then increase 10% per year for the next<br />

10 years. Also, the repair savings for the last 3 years were<br />

$35,000, $32,000, and $28,000, as best as Harry can determine.<br />

He believes savings will decrease by $2000 per year<br />

hereafter. Finally, these 3-year-old protectors are worth nothing<br />

on the market now, so the salvage in 7 years is zero, not<br />

$3000.<br />

Case Study Exercises<br />

1. Plot a graph of the newly estimated maintenance costs<br />

and repair savings projections, assuming the protectors<br />

last for 7 more years.<br />

2. With these new estimates, what is the recalculated AW<br />

for the Lloyd’s protectors? Use the old first cost and<br />

maintenance cost estimates for the first 3 years. If these<br />

estimates had been made 3 years ago, would Lloyd’s<br />

still have been the economic choice?<br />

3. How has the capital recovery amount changed for the<br />

Lloyd’s protectors with these new estimates?<br />

Figure 6–9<br />

Annual worth analysis of surge protector alternatives, case study.


CHAPTER 7<br />

Rate of Return<br />

Analysis: One<br />

Project<br />

LEARNING OUTCOMES<br />

Purpose: Understand the meaning of rate of return and perform an ROR evaluation of a single project.<br />

SECTION TOPIC LEARNING OUTCOME<br />

7.1 Definition • State and understand the meaning of rate of<br />

return.<br />

7.2 Calculate ROR • Use a PW or AW relation to determine the<br />

ROR of a series of cash flows.<br />

7.3 Cautions • State the difficulties of using the ROR method,<br />

relative to the PW and AW methods.<br />

7.4 Multiple RORs • Determine the maximum number of possible<br />

ROR values and their values for a specific cash<br />

flow series.<br />

7.5 Calculate EROR • Determine the external rate of return using<br />

the techniques of modified ROR and return on<br />

invested capital.<br />

7.6 Bonds • Calculate the nominal and effective rate of<br />

return for a bond investment.


T<br />

he most commonly quoted measure of economic worth for a project or alternative<br />

is its rate of return (ROR). Whether it is an engineering project with cash<br />

flow estimates or an investment in a stock or bond, the rate of return is a wellaccepted<br />

way of determining if the project or investment is economically acceptable. Compared<br />

to the PW or AW value, the ROR is a generically different type of measure of worth,<br />

as is discussed in this chapter. Correct procedures to calculate a rate of return using a PW or<br />

AW relation are explained here, as are some cautions necessary when the ROR technique is<br />

applied to a single project’s cash flows.<br />

The ROR is known by other names such as the internal rate of return (IROR), which is the<br />

technically correct term, and return on investment (ROI). We will discuss the computation of<br />

ROI in the latter part of this chapter.<br />

In some cases, more than one ROR value may satisfy the PW or AW equation. This chapter<br />

describes how to recognize this possibility and an approach to find the multiple values .<br />

Alternatively, one reliable ROR value can be obtained by using additional information established<br />

separately from the project cash flows. Two of the techniques are covered: the modified<br />

ROR technique and the ROIC (return on invested capital) technique.<br />

Only one alternative is considered here; Chapter 8 applies these same principles to multiple<br />

alternatives. Finally, the rate of return for a bond investment is discussed.<br />

7.1 Interpretation of a Rate of Return Value<br />

From the perspective of someone who has borrowed money, the interest rate is applied to the<br />

unpaid balance so that the total loan amount and interest are paid in full exactly with the last loan<br />

payment. From the perspective of a lender of money, there is an unrecovered balance at each time<br />

period. The interest rate is the return on this unrecovered balance so that the total amount lent and<br />

the interest are recovered exactly with the last receipt. Rate of return describes both of these<br />

perspectives.<br />

Rate of return (ROR) is the rate paid on the unpaid balance of borrowed money , or the rate<br />

earned on the unrecovered balance of an investment , so that the final payment or receipt<br />

brings the balance to exactly zero with interest considered.<br />

Rate of return<br />

The rate of return is expressed as a percent per period, for example, i 10% per year. It is<br />

stated as a positive percentage; the fact that interest paid on a loan is actually a negative rate of<br />

return from the borrower’s perspective is not considered. The numerical value of i can range from<br />

100% to infinity, that is, 100% i . In terms of an investment, a return of i 100%<br />

means the entire amount is lost.<br />

The definition above does not state that the rate of return is on the initial amount of the investment;<br />

rather it is on the unrecovered balance , which changes each time period. Example 7.1<br />

illustrates this difference.<br />

EXAMPLE 7.1<br />

To get started in a new telecommuting position with AB Hammond Engineers, Jane took out<br />

a $1000 loan at i 10% per year for 4 years to buy home office equipment. From the lender’s<br />

perspective, the investment in this young engineer is expected to produce an equivalent net<br />

cash flow of $315.47 for each of 4 years.<br />

A $1000( A P ,10%,4) $315.47<br />

This represents a 10% per year rate of return on the unrecovered balance. Compute the amount<br />

of the unrecovered investment for each of the 4 years using ( a ) the rate of return on the unrecovered<br />

balance (the correct basis) and ( b ) the return on the initial $1000 investment. ( c ) Explain<br />

why all of the initial $1000 amount is not recovered by the final payment in part ( b ).<br />

Solution<br />

(a) Table 7–1 shows the unrecovered balance at the end of each year in column 6 using the<br />

10% rate on the unrecovered balance at the beginning of the year. After 4 years the total<br />

$1000 is recovered, and the balance in column 6 is exactly zero.


174 Chapter 7 Rate of Return Analysis: One Project<br />

TABLE 7–1 Unrecovered Balances Using a Rate of Return of 10% on the Unrecovered<br />

Balance<br />

(1) (2) (3) 0.10 (2) (4) (5) (4) (3) (6) (2) (5)<br />

Year<br />

Beginning<br />

Unrecovered<br />

Balance<br />

Interest on<br />

Unrecovered<br />

Balance<br />

Cash<br />

Flow<br />

Recovered<br />

Amount<br />

Ending<br />

Unrecovered<br />

Balance<br />

0 — — $1000.00 — $1000.00<br />

1 $1000.00 $100.00 315.47 $215.47 784.53<br />

2 784.53 78.45 315.47 237.02 547.51<br />

3 547.51 54.75 315.47 260.72 286.79<br />

4 286.79 28.68 315.47 286.79 0<br />

$261.88 $1000.00<br />

TABLE 7–2 Unrecovered Balances Using a 10% Return on the Initial Amount<br />

(1) (2) (3) 0.10 (2) (4) (5) (4) (3) (6) (2) (5)<br />

Year<br />

Beginning<br />

Unrecovered<br />

Balance<br />

Interest on<br />

Initial Amount<br />

Cash<br />

Flow<br />

Recovered<br />

Amount<br />

Ending<br />

Unrecovered<br />

Balance<br />

0 — — $1000.00 — $1000.00<br />

1 $1000.00 $100 315.47 $215.47 784.53<br />

2 784.53 100 315.47 215.47 569.06<br />

3 569.06 100 315.47 215.47 353.59<br />

4 353.59 100 315.47 215.47 138.12<br />

$400 $861.88<br />

(b) Table 7–2 shows the unrecovered balance if the 10% return is always figured on the initial<br />

$1000. Column 6 in year 4 shows a remaining unrecovered amount of $138.12,<br />

because only $861.88 is recovered in the 4 years (column 5).<br />

(c) As shown in column 3, a total of $400 in interest must be earned if the 10% return each year is<br />

based on the initial amount of $1000. However, only $261.88 in interest must be earned if a<br />

10% return on the unrecovered balance is used. There is more of the annual cash flow available<br />

to reduce the remaining loan when the rate is applied to the unrecovered balance as in part ( a )<br />

and Table 7–1 . Figure 7–1 illustrates the correct interpretation of rate of return in Table 7–1 .<br />

Loan balance, $<br />

1000.00<br />

784.53<br />

547.51<br />

Loan balance<br />

of $1000<br />

$100.00<br />

$215.47<br />

Interest<br />

Loan balance<br />

reduction<br />

$78.45<br />

$237.02<br />

$54.75<br />

$260.72<br />

286.79<br />

$28.68<br />

$286.79<br />

0 1 2 3 4 Year<br />

Loan balance<br />

of $0<br />

Figure 7–1<br />

Plot of unrecovered balances and 10% per year rate of return on a $1000<br />

amount, Table 7–1 .


7.2 Rate of Return Calculation Using a PW or AW Relation 175<br />

Each year the $315.47 receipt represents 10% interest on the unrecovered balance in column 2<br />

plus the recovered amount in column 5.<br />

Because rate of return is the interest rate on the unrecovered balance, the computations in<br />

Table 7–1 for part (a) present a correct interpretation of a 10% rate of return. Clearly, an interest<br />

rate applied only to the principal represents a higher rate than is stated. In practice, a socalled<br />

add-on interest rate is frequently based on principal only, as in part ( b ). This is sometimes<br />

referred to as the installment fi nancing problem.<br />

Installment fi nancing can be discovered in many forms in everyday finances. One popular<br />

example is a “no-interest program” offered by retail stores on the sale of major appliances, audio<br />

and video equipment, furniture, and other consumer items. Many variations are possible, but in<br />

most cases, if the purchase is not paid for in full by the time the promotion is over, usually<br />

6 months to 1 year later, fi nance charges are assessed from the original date of purchase. Further,<br />

the program’s fine print may stipulate that the purchaser use a credit card issued by the retail<br />

company, which often has a higher interest rate than that of a regular credit card, for example,<br />

24% per year compared to 15% per year. In all these types of programs, the one common theme<br />

is more interest paid over time by the consumer. Usually, the correct definition of i as interest on<br />

the unpaid balance does not apply directly; i has often been manipulated to the financial disadvantage<br />

of the purchaser. This was demonstrated by Example 4.4 using the Credit Card Case in<br />

Chapter 4.<br />

7.2 Rate of Return Calculation Using a PW<br />

or AW Relation<br />

The ROR value is determined in a generically different way compared to the PW or AW value for<br />

a series of cash flows. For a moment, consider only the present worth relation for a cash flow<br />

series. Using the MARR, which is established independent of any particular project’s cash flows,<br />

a mathematical relation determines the PW value in actual monetary units, say, dollars or euros.<br />

For the ROR values calculated in this and later sections, only the cash flows themselves are used<br />

to determine an interest rate that balances the present worth relation. Therefore, ROR may be<br />

considered a relative measure, while PW and AW are absolute measures. Since the resulting interest<br />

rate depends only on the cash flows themselves, the correct term is internal rate of return<br />

(IROR) ; however, the term ROR is used interchangeably. Another definition of rate of return is<br />

based on our previous interpretations of PW and AW.<br />

The rate of return is the interest rate that makes the present worth or annual worth of a cash<br />

flow series exactly equal to 0.<br />

Rate of return<br />

To determine the rate of return, develop the ROR equation using either a PW or AW relation, set<br />

it equal to 0, and solve for the interest rate. Alternatively, the present worth of cash outflows<br />

(costs and disbursements) PW O may be equated to the present worth of cash inflows (revenues<br />

and savings) PW I . That is, solve for i using either of the relations<br />

or<br />

0 PW [7.1]<br />

PW O PW I<br />

The annual worth approach utilizes the AW values in the same fashion to solve for i .<br />

or<br />

0 AW [7.2]<br />

AW O AW I<br />

The i value that makes these equations numerically correct is called i *. It is the root of the ROR<br />

relation. To determine if the investment project’s cash flow series is viable, compare i * with the<br />

established MARR.


176 Chapter 7 Rate of Return Analysis: One Project<br />

Figure 7–2<br />

Cash flow for which<br />

a value of i is to be<br />

determined.<br />

$1500<br />

$500<br />

0 1 2 3 4 5<br />

i * = ?<br />

$1000<br />

The guideline is as follows:<br />

Project evaluation<br />

If i * MARR, accept the project as economically viable.<br />

If i * MARR, the project is not economically viable.<br />

The purpose of engineering economy calculations is equivalence in PW or AW terms for a<br />

stated i 0%. In rate of return calculations, the objective is to fi nd the interest rate i* at which<br />

the cash flows are equivalent. The calculations are the reverse of those made in previous chapters,<br />

where the interest rate was known. For example, if you deposit $1000 now and are promised payments<br />

of $500 three years from now and $1500 five years from now, the rate of return relation<br />

using PW factors and Equation [7.1] is<br />

1000 500( P F , i *,3) 1500( P F , i *,5) [7.3]<br />

The value of i * that makes the equality correct is to be determined (see Figure 7–2 ). If the $1000<br />

is moved to the right side of Equation [7.3], we have the form 0 PW.<br />

0 1000 500( P F , i *,3) 1500( P F , i *,5)<br />

The equation is solved for i * 16.9% by hand using trial and error or using a spreadsheet function.<br />

The rate of return will always be greater than zero if the total amount of cash inflow is greater<br />

than the total amount of outflow, when the time value of money is considered. Using i * 16.9%,<br />

a graph similar to Figure 7–1 can be constructed. It will show that the unrecovered balances each<br />

year, starting with $1000 in year 1, are exactly recovered by the $500 and $1500 receipts in<br />

years 3 and 5.<br />

It should be evident that rate of return relations are merely a rearrangement of a present worth<br />

equation. That is, if the above interest rate is known to be 16.9%, and it is used to find the present<br />

worth of $500 three years from now and $1500 five years from now, the PW relation is<br />

PW 500( P F ,16.9%,3) 1500( P F ,16.9%,5) $1000<br />

This illustrates that rate of return and present worth equations are set up in exactly the same fashion.<br />

The only differences are what is given and what is sought.<br />

There are several ways to determine i * once the PW relation is established: solution via trial<br />

and error by hand, using a programmable calculator, and solution by spreadsheet function. The<br />

spreadsheet is faster; the first helps in understanding how ROR computations work. We summarize<br />

two methods here and in Example 7.2 . Refer to Appendix D for the discussion about solution<br />

by calculator.<br />

i* Using Trial and Error The general procedure of using a PW-based equation is as follows:<br />

1. Draw a cash flow diagram.<br />

2. Set up the rate of return equation in the form of Equation [7.1] .<br />

3. Select values of i by trial and error until the equation is balanced.<br />

When the trial-and-error method is applied to determine i *, it is advantageous in step 3 to get<br />

fairly close to the correct answer on the first trial. If the cash flows are combined in such a manner<br />

that the income and disbursements can be represented by a single factor such as P F or P A , it is


7.2 Rate of Return Calculation Using a PW or AW Relation 177<br />

possible to look up the interest rate (in the tables) corresponding to the value of that factor for n<br />

years. The problem, then, is to combine the cash flows into the format of only one of the factors.<br />

This may be done through the following procedure:<br />

1. Convert all disbursements into either single amounts ( P or F ) or uniform amounts ( A ) by<br />

neglecting the time value of money. For example, if it is desired to convert an A to an F<br />

value, simply multiply the A by the number of years n . The scheme selected for movement<br />

of cash flows should be the one that minimizes the error caused by neglecting the time value<br />

of money. That is, if most of the cash flow is an A and a small amount is an F , convert the F<br />

to an A rather than the other way around.<br />

2. Convert all receipts to either single or uniform values.<br />

3. Having combined the disbursements and receipts so that a P F , P A , or A F format applies,<br />

use the interest tables to find the approximate interest rate at which the P F , P A , or A F<br />

value is satisfied. The rate obtained is a good estimate for the first trial.<br />

It is important to recognize that this first-trial rate is only an estimate of the actual rate of return,<br />

because the time value of money is neglected. The procedure is illustrated in Example 7.2 .<br />

i* by Spreadsheet The fastest way to determine an i * value when there is a series of equal<br />

cash flows (A series) is to apply the RATE function. This is a powerful one-cell function, where it<br />

is acceptable to have a separate P value in year 0 and a separate F value in year n . The format is<br />

RATE (n,A,P,F) [7.4]<br />

When cash flows vary from year to year (period to period), the best way to find i * is to enter<br />

the net cash flows into contiguous cells (including any $0 amounts) and apply the IRR function<br />

in any cell. The format is<br />

IRR(first_cell:last_cell,guess) [7.5]<br />

where “guess” is the i value at which the function starts searching for i *.<br />

The PW-based procedure for sensitivity analysis and a graphical estimation of the i * value is<br />

as follows:<br />

1. Draw the cash flow diagram.<br />

2. Set up the ROR relation in the form of Equation [7.1], PW 0.<br />

3. Enter the cash flows onto the spreadsheet in contiguous cells.<br />

4. Develop the IRR function to display i *.<br />

5. Use the NPV function to develop a PW graph (PW versus i values). This graphically shows<br />

the i * value at which PW 0.<br />

EXAMPLE 7.2<br />

Applications of green, lean manufacturing techniques coupled with value stream mapping can<br />

make large financial differences over future years while placing greater emphasis on environmental<br />

factors. Engineers with Monarch Paints have recommended to management an investment<br />

of $200,000 now in novel methods that will reduce the amount of wastewater, packaging<br />

materials, and other solid waste in their consumer paint manufacturing facility. Estimated savings<br />

are $15,000 per year for each of the next 10 years and an additional savings of $300,000<br />

at the end of 10 years in facility and equipment upgrade costs. Determine the rate of return<br />

using hand and spreadsheet solutions.<br />

Solution by Hand<br />

Use the trial-and-error procedure based on a PW equation.<br />

1. Figure 7–3 shows the cash flow diagram.<br />

2. Use Equation [7.1] format for the ROR equation.<br />

0 200,000 15,000( P A , i *,10) 300,000( P F , i *,10) [7.6]


178 Chapter 7 Rate of Return Analysis: One Project<br />

i * = ?<br />

$300,000<br />

$15,000<br />

0 1 2 3 4 5 6 7 8 9 10<br />

$200,000<br />

Figure 7–3<br />

Cash flow diagram, Example 7.2 .<br />

3. Use the estimation procedure to determine i for the first trial. All income will be regarded<br />

as a single F in year 10 so that the P F factor can be used. The P F factor is selected because<br />

most of the cash flow ($300,000) already fits this factor and errors created by<br />

neglecting the time value of the remaining money will be minimized. Only for the first<br />

estimate of i , define P $200,000, n 10, and F 10(15,000) 300,000 $450,000.<br />

Now we can state that<br />

200,000 450,000( P F , i ,10)<br />

( P F , i ,10) 0.444<br />

The roughly estimated i is between 8% and 9%. Use 9% as the first trial because this approximate<br />

rate for the P F factor will be lower than the true value when the time value of money is<br />

considered.<br />

0 200,000 15,000( P A ,9%,10) 300,000( P F ,9%,10)<br />

0 $22,986<br />

The result is positive, indicating that the return is more than 9%. Try i 11%.<br />

0 200,000 15,000( P A ,11%,10) 300,000( P F ,11%,10)<br />

0 $6002<br />

Since the interest rate of 11% is too high, linearly interpolate between 9% and 11%.<br />

22,986 0<br />

i * 9.00 ————————<br />

22,986 (6002) (2.0)<br />

9.00 1.58 10.58%<br />

Solution by Spreadsheet<br />

The fastest way to find i * is to use the RATE function ( Equation [7.4]). The entry<br />

RATE(10,15000,200000,300000) displays i * 10.55% per year. It is equally correct to<br />

use the IRR function. Figure 7–4 , column B, shows the cash flows and IRR(B2:B12)<br />

function to obtain i *.<br />

For a complete spreadsheet analysis, use the procedure outlined above.<br />

1. Figure 7–3 shows cash flows.<br />

2. Equation [7.6] is the ROR relation.<br />

3. Figure 7–4 shows the net cash flows in column B.<br />

4. The IRR function in cell B14 displays i * 10.55%.<br />

5. To graphically observe i * 10.55%, column D displays the PW graph for different<br />

i values. The NPV function is used repeatedly to calculate PW for the xy scatter<br />

chart.


7.3 Special Considerations When Using the ROR Method 179<br />

NPV(C4,$B$3:$B$12) + $B$2<br />

IRR(B2:B12)<br />

Figure 7–4<br />

Spreadsheet to determine i * and develop a PW graph, Example 7.2 .<br />

Just as i * can be found using a PW equation, it may equivalently be determined using an AW<br />

relation. This method is preferred when uniform annual cash flows are involved. Solution by<br />

hand is the same as the procedure for a PW-based relation, except Equation [7.2] is used. In the<br />

case of Example 7.2 , i* 10.55% is determined using the AW-based relation.<br />

0 200,000( AP,i *,10) 15,000 300,000( AF,i *,10)<br />

The procedure for solution by spreadsheet is exactly the same as outlined above using the IRR<br />

function. Internally, IRR calculates the NPV function at different i values until NPV 0. (There is no<br />

equivalent way to utilize the PMT function, since it requires a fixed value of i to calculate an A value.)<br />

7.3 Special Considerations When Using<br />

the ROR Method<br />

The rate of return method is commonly used in engineering and business settings to evaluate<br />

one project, as discussed in this chapter, and to select one alternative from two or more, as<br />

explained in the next chapter. As mentioned earlier, an ROR analysis is performed using a different<br />

basis than PW and AW analyses. The cash flows themselves determine the (internal) rate<br />

of return. As a result, there are some assumptions and special considerations with ROR analysis<br />

that must be made when calculating i * and in interpreting its real-world meaning. A summary<br />

is provided below.<br />

• Multiple i * values. Depending upon the sequence of net cash inflows and outflows, there may<br />

be more than one real-number root to the ROR equation, resulting in more than one i* value.<br />

This possibility is discussed in Section 7.4.<br />

• Reinvestment at i*. Both the PW and AW methods assume that any net positive investment<br />

(i.e., net positive cash flows once the time value of money is considered) is reinvested at the<br />

MARR. However, the ROR method assumes reinvestment at the i * rate. When i * is not close<br />

to the MARR (e.g., if i * is substantially larger than MARR), this is an unrealistic assumption.<br />

In such cases, the i * value is not a good basis for decision making. This situation is discussed<br />

in Section 7.5.<br />

• Different procedure for multiple alternative evaluations. To correctly use the ROR method<br />

to choose from two or more mutually exclusive alternatives requires an incremental analysis<br />

procedure that is significantly more involved than PW and AW analysis. Chapter 8 explains<br />

this procedure.<br />

If possible, from an engineering economic study perspective, the AW or PW method at a<br />

stated MARR should be used in lieu of the ROR method . However, there is a strong appeal


180 Chapter 7 Rate of Return Analysis: One Project<br />

for the ROR method because rate of return values are very commonly quoted. And it is easy to<br />

compare a proposed project’s return with that of in-place projects.<br />

When it is important to know the exact value of i*, a good approach is to determine PW or<br />

AW at the MARR, then determine the specific i* for the selected alternative.<br />

As an illustration, if a project is evaluated at MARR 15% and has PW 0, there is no need to<br />

calculate i *, because i * 15%. However, if PW is positive, but close to 0, calculate the exact i *<br />

and report it along with the conclusion that the project is financially justified.<br />

7.4 Multiple Rate of Return Values<br />

In Section 7.2 a unique rate of return i * was determined. In the cash flow series presented thus<br />

far, the algebraic signs on the net cash fl ows changed only once, usually from minus in year 0 to<br />

plus at some time during the series. This is called a conventional ( or simple) cash fl ow series.<br />

However, for some series the net cash flows switch between positive and negative from one year<br />

to another, so there is more than one sign change. Such a series is called nonconventional (nonsimple).<br />

As shown in the examples of Table 7–3 , each series of positive or negative signs may be<br />

one or more in length. Relatively large net cash flow (NCF) changes in amount and sign can<br />

occur in projects that require significant spending at the end of the expected life. Nuclear plants,<br />

open-pit mines, petroleum well sites, refineries, and the like often require environmental restoration,<br />

waste disposal, and other expensive phaseout costs. The cash flow diagram will appear<br />

similar to Figure 7–5 a . Plants and systems that have anticipated major refurbishment costs or<br />

upgrade investments in future years may have considerable swings in cash flow and sign changes<br />

over the years, as shown by the pattern in Figure 7–5 b .<br />

TABLE 7–3 Examples of Conventional and Nonconventional Net Cash Flow for<br />

a 6-year Project<br />

Type of Series<br />

Sign on Net Cash Flow by Year<br />

0 1 2 3 4 5 6<br />

Number of<br />

Sign Changes<br />

Conventional 1<br />

Conventional 1<br />

Conventional 1<br />

Nonconventional 2<br />

Nonconventional 2<br />

Nonconventional 3<br />

Positive<br />

NCF<br />

Positive<br />

NCF<br />

Positive<br />

NCF<br />

0 1 2 n 1 n<br />

Year<br />

0 1 2 n 1 n<br />

Year<br />

Initial<br />

investment<br />

i * = ?<br />

Phaseout<br />

costs<br />

Initial<br />

investment<br />

Midlife<br />

investments<br />

(a)<br />

(b)<br />

Figure 7–5<br />

Typical cash flow diagrams for projects with ( a ) large restoration or remediation costs, and ( b ) upgrade or refurbishment costs.<br />

i * = ?


7.4 Multiple Rate of Return Values 181<br />

When there is more than one sign change in the net cash flows, it is possible that there will be<br />

multiple i * values in the 100% to plus infinity range. There are two tests to perform in sequence<br />

on the nonconventional series to determine if there is one unique value or possibly multiple i *<br />

values that are real numbers.<br />

Test 1: (Descartes’) rule of signs states that the total number of real-number roots is always less<br />

than or equal to the number of sign changes in the series.<br />

This rule is derived from the fact that the relation set up by Equation [7.1] or [7.2] to find i *<br />

is an n th-order polynomial. (It is possible that imaginary values or infinity may also satisfy the<br />

equation.)<br />

Test #2: Cumulative cash flow sign test, also known as Norstrom’s criterion, states that only<br />

one sign change in a series of cumulative cash flows which starts negatively indicates that there<br />

is one positive root to the polynomial relation.<br />

Zero values in the series are neglected when applying Norstrom’s criterion. This is a more discriminating<br />

test that determines if there is one, real-number, positive i * value. There may be<br />

negative roots that satisfy the ROR relation, but these are not useful i * values. To perform the<br />

test, determine the series<br />

S t cumulative cash flows through period t<br />

Observe the sign of S 0 and count the sign changes in the series S 0 , S 1 , . . . , S n . Only if S 0 0 and<br />

signs change one time in the series is there a single, real-number, positive i *.<br />

With the results of these two tests, the ROR relation is solved for either the unique i * or the<br />

multiple i * values, using trial and error by hand, using a programmable calculator, or by spreadsheet<br />

using an IRR function that incorporates the “guess” option. Development of the PW graph<br />

is recommended, especially when using a spreadsheet. Examples 7.3 and 7.4 illustrate the tests<br />

and solution for i *.<br />

EXAMPLE 7.3<br />

Sept-Îles Aluminum Company operates a bauxite mine to supply its aluminum smelter located<br />

about 2 km from the current open pit. A new branch for the pit is proposed that will<br />

supply an additional 10% of the bauxite currently available over the next 10-year period. The<br />

lease for the land will cost $400,000 immediately. The contract calls for the restoration of the<br />

land and development as part of a state park and wildlife area at the end of the 10 years. This<br />

is expected to cost $300,000. The increased production capacity is estimated to net an additional<br />

$75,000 per year for the company. Perform an ROR analysis that will provide the following<br />

information:<br />

(a) Type of cash flow series and possible number of ROR values<br />

(b) PW graph showing all i* values<br />

(c) Actual i* values determined using the ROR relation and spreadsheet function<br />

(d) Conclusions that can be drawn about the correct rate of return from this analysis<br />

Solution<br />

(a) The net cash flows will appear like those in Figure 7–5a with an initial investment of<br />

$400,000, annual net cash flow (NCF) of $75,000 for years 1 through 10, and a phaseout<br />

cost of $300,000 in year 10. Figure 7–6 details the NCF series (column B) and cumulative<br />

NCF (column C) for use in the two tests for unique and multiple i* values. The<br />

series is nonconventional based on the sign changes throughout the series.<br />

Test #1: There are two sign changes in the NCF series, which indicates a possible<br />

maximum of two roots to the polynomial equation or i* values for the ROR<br />

equation.<br />

Test #2: There is one sign change in the cumulative NCF series, which indicates a unique<br />

positive root or one positive i* value.


182 Chapter 7 Rate of Return Analysis: One Project<br />

PV(D3,$B$4:$B$13)$B$3<br />

IRR(B3:B13,guess)<br />

Figure 7–6<br />

Spreadsheet determination of multiple i* values and PW graph, Example 7.3.<br />

(b) Columns D and E of the spreadsheet in Figure 7–6 use i values ranging from 20% to<br />

20% per year to plot the PW vs i curve via the NPV function. There are two times that<br />

the parabolic-shaped curve crosses the PW 0 line; these are approximately i 1 * 18%<br />

and i 2 * 5%.<br />

(c) The ROR equation based on PW computations is<br />

0 400,000 75,000(PA,i*%,10) 300,000(PF,i*%,10) [7.7]<br />

i* values by hand If hand solution is chosen, the same procedure used in Example 7.2<br />

can be applied here. However, the technique to estimate the initial i value will not work<br />

as well in this case since the majority of the cash flows do not fit either the PF or the<br />

FP factor. In fact, using the PF factor, the initial i value is indicated to be 1.25%.<br />

Trial-and-error solution of Equation [7.7] with various i values will approximate the<br />

correct answer of about 4.5% per year. This complies with the test results of one positive<br />

i* value.<br />

i* by spreadsheet function Use the IRR(B3:B13,guess) function to determine the i*<br />

value for the NCF series in column B, Figure 7–6. Entering different values in the<br />

optional “guess” field will force the function to find multiple i* values, if they exist.<br />

As shown in row 17, two are found.<br />

i 1 * 18.70%<br />

i 2 * 4.53%<br />

This result does not conflict with test results, as there is one positive value, but a negative<br />

value also balances the ROR equation.<br />

(d) The positive i* 4.53% is accepted as the correct internal rate of return (IROR) for the<br />

project. The negative value is not useful in economic conclusions about the project.<br />

EXAMPLE 7.4<br />

The engineering design and testing group for Honda Motor Corp. does contract-based work for<br />

automobile manufacturers throughout the world. During the last 3 years, the net cash flows for<br />

contract payments have varied widely, as shown below, primarily due to a large manufacturer’s<br />

inability to pay its contract fee.<br />

Year 0 1 2 3<br />

Cash Flow ($1000) 2000 500 8100 6800


7.4 Multiple Rate of Return Values 183<br />

(a) Determine the maximum number of i* values that may satisfy the ROR equation.<br />

(b) Write the PW equation and approximate the i* value(s) by plotting PW vs i.<br />

(c) What do the i* values mean?<br />

Solution<br />

(a) Table 7–4 shows the annual cash flows and cumulative cash flows. Since there are two<br />

sign changes in the cash flow sequence, the rule of signs indicates a maximum of two i*<br />

values. The cumulative cash flow sequence starts with a positive number S 0 2000,<br />

indicating that test #2 is inconclusive. As many as two i* values can be found.<br />

TABLE 7–4 Cash Flow and Cumulative Cash Flow Sequences, Example 7.4<br />

Year<br />

Cash Flow<br />

($1000)<br />

Sequence<br />

Number<br />

Cumulative Cash Flow<br />

($1000)<br />

0 2000 S 0 2000<br />

1 500 S 1 1500<br />

2 8100 S 2 6600<br />

3 6800 S 3 200<br />

(b) The PW relation is<br />

PW 2000 500(PF,i,1) 8100(PF,i,2) 6800(PF,i,3)<br />

The PW values are shown below and plotted in Figure 7–7 for several i values. The characteristic<br />

parabolic shape for a second-degree polynomial is obtained, with PW crossing<br />

the i axis at approximately i 1 * 8% and i 2 * 41%.<br />

i% 5 10 20 30 40 50<br />

PW ($1000) 51.44 39.55 106.13 82.01 11.83 81.85<br />

100<br />

75<br />

50<br />

Figure 7–7<br />

Present worth of cash<br />

flows at several interest<br />

rates, Example 7.4.<br />

PW ( $1000)<br />

25<br />

0<br />

25<br />

i%<br />

10 20 30 40 50<br />

50<br />

75<br />

100<br />

125<br />

Figure 7–8 presents the spreadsheet PW graph with the PW curve crossing the x axis at<br />

PW 0 two times. Also, the solution for two positive i* values using the IRR function<br />

with different guess values is displayed. The values are<br />

i 1 * 7.47% i 2 * 41.35%<br />

(c) Since both i* values are positive, they are not of much value, because neither can be<br />

considered the true ROR of the cash flow series. This result indicates that additional


184 Chapter 7 Rate of Return Analysis: One Project<br />

IRR function format<br />

IRR($B4:$B7,guess)<br />

Figure 7–8<br />

Spreadsheet solution, Example 7.4.<br />

information is needed to calculate a more useful project ROR, namely, some information<br />

about the anticipated return on funds invested external to the project and the cost of capital<br />

to borrow money to continue the project. This problem is a good example of when an<br />

approach discussed in the next section should be taken.<br />

If the guess option is not used in the IRR function, the starting point is 10% per year. The function<br />

will find the one ROR closest to 10% that satisfies the PW relation. Entering various guess<br />

values will allow IRR to find multiple i * values in the range of 100% to , if they exist. Often,<br />

the results are unbelievable or unacceptable values that are rejected. Some helpful guidelines can<br />

be developed. Assume there are two i * values for a particular cash flow series.<br />

If the Results Are<br />

Both i * 0<br />

Both i * 0<br />

One i * 0; one i * 0<br />

What to Do<br />

Discard both values.<br />

Discard both values.<br />

Use i * 0 as ROR.<br />

If both i * values are discarded, proceed to the approach discussed in the next section to determine<br />

one rate of return value for the project. However, remember the prime recommendation.<br />

Always determine the PW or AW at the MARR first for a reliable measure of economic justification.<br />

If the PW or AW is greater than zero and the ROR is needed, then find the actual i* of the<br />

project cash flows.<br />

This recommendation is not to dissuade you from using the ROR method. Rather it is a recommendation<br />

that the use of the ROR technique be reserved for times when the actual i * value is<br />

essential to the decision-making process.<br />

7.5 Techniques to Remove Multiple Rates of Return<br />

The techniques developed here are used under the following conditions:<br />

• The PW or AW value at the MARR is determined and could be used to make the decision, but information<br />

on the ROR is deemed necessary to finalize the economic decision, and<br />

• The two tests of cash flow sign changes (Descartes’ and Norstrom’s) indicate multiple roots<br />

( i * values) are possible, and


7.5 Techniques to Remove Multiple Rates of Return 185<br />

• More than one positive i * value or all negative i * values are obtained when the PW graph and<br />

IRR function are developed, and<br />

• A single, reliable rate of return value is required by management or engineers to make a clear<br />

economic decision.<br />

We will present a couple of ways to remove multiple i * values. The selected approach depends<br />

upon what estimates are the most reliable for the project being evaluated. An important fact to<br />

remember is the following.<br />

The result of follow-up analysis to obtain a single ROR value when multiple, nonuseful i* values<br />

are present does not determine the internal rate of return (IROR) for nonconventional net<br />

cash flow series. The resulting rate is a function of the additional information provided to make<br />

the selected technique work, and the accuracy is further dependent upon the reliability of this<br />

information.<br />

We will refer to the resulting value as the external rate of return (EROR) as a reminder that<br />

it is different from the IROR obtained in all previous sections. First, it is necessary to identify<br />

the perspective about the annual net cash flows of a project. Take the following view: You are<br />

the project manager and the project generates cash flows each year. Some years produce positive<br />

NCF, and you want to invest the excess money at a good rate of return. We will call this<br />

the investment rate i i . This can also be called the reinvestment rate . Other years, the net cash flow<br />

will be negative and you must borrow funds from some source to continue. The interest rate you<br />

pay should be as small as possible; we will call this the borrowing rate i b , also referred to as the<br />

finance rate . Each year, you must consider the time value of money, which must utilize either the<br />

investment rate or the borrowing rate, depending upon the sign on the NCF of the preceding year.<br />

With this perspective, it is now possible to outline two approaches that rectify the multiple<br />

i * situation. The resulting ROR value will not be the same for each method, because slightly<br />

different additional information is necessary and the cash flows are treated in slightly different<br />

fashions from the time value of money viewpoint.<br />

Modified ROR (MIRR) Approach This is the easier approach to apply, and it has a spreadsheet<br />

function that can find the single EROR value quickly. However, the investment and borrowing<br />

rates must be reliably estimated, since the results may be quite sensitive to them. The<br />

symbol i will identify the result.<br />

Return on Invested Capital (ROIC) Approach Though more mathematically rigorous,<br />

this technique provides a more reliable estimate of the EROR and it requires only the investment<br />

rate i i . The symbol i is used to indentify the result.<br />

Before covering the techniques, it would be good to review the material in Section 7.1, including<br />

Example 7.1 . Though the i or i value determined here is not the ROR defined earlier in the<br />

chapter, the concepts used to make the ending cash flow balance equal to zero are used.<br />

Modified ROR Approach<br />

The technique requires that two rates external to the project net cash flows be estimated.<br />

• Investment rate i i is the rate at which extra funds are invested in some source external to the<br />

project. This applies to all positive annual NCF. It is reasonable that the MARR is used for this<br />

rate.<br />

• Borrowing rate i b is the rate at which funds are borrowed from an external source to provide<br />

funds to the project. This applies to all negative annual NCF. The weighted average cost of<br />

capital (WACC) can be used for this rate.<br />

It is possible to make all rates the same, that is, i i i b MARR WACC. However, this is not<br />

a good idea as it implies that the company is willing to borrow funds and invest in projects at the<br />

same rate. This implies no profit margin over time, so the company can’t survive for long using<br />

this strategy. Commonly MARR WACC, so usually i i i b . (See Section 1.9 for a quick review<br />

of MARR and WACC and Chapter 10 for a more detailed discussion of WACC.)


186 Chapter 7 Rate of Return Analysis: One Project<br />

PW 0 at i b<br />

Find FW n of all<br />

NCF > 0 at investment rate i i<br />

NCF 1<br />

NCF 2<br />

NCF 4<br />

NCF n1<br />

0 1 2 3 4 n 1 n<br />

Year<br />

NCF 3 NCF n<br />

NCF 0<br />

Find PW 0 of all<br />

FW<br />

NCF < 0 at borrowing rate i n at i i<br />

b<br />

Figure 7–9<br />

Typical cash flow diagram to determine modified rate of return i.<br />

Figure 7–9 is a reference diagram that has multiple i * values, since the net cash flows change<br />

sign multiple times. The modified ROR method uses the following procedure to determine a<br />

single external rate of return i and to evaluate the economic viability of the project.<br />

1. Determine the PW value in year 0 of all negative NCF at the borrowing rate i b (lightly<br />

shaded area and resulting PW 0 value in Figure 7–9 ).<br />

2. Determine the FW value in year n of all positive NCF at the investment rate i i (darker shaded<br />

area and resulting FW n value in Figure 7–9 ).<br />

3. Calculate the modified rate of return i at which the PW and FW values are equivalent over<br />

the n years using the following relation, where i is to be determined.<br />

FW n PW 0 ( F P , i %, n ) [7.8]<br />

If using a spreadsheet rather than hand computation, the MIRR function displays i directly with<br />

the format<br />

MIRR(first_cell:last_cell, i b , i i ) [7.9]<br />

4. The guideline for economic decision making compares the EROR or i to MARR.<br />

If i MARR, the project is economically justified.<br />

If i MARR, the project is not economically justified.<br />

As in other situations, on the rare occasion that i MARR, there is indifference to the project's<br />

economic acceptability; however, acceptance is the usual decision.<br />

EXAMPLE 7.5<br />

The cash flows experienced by Honda Motors in Example 7.4 are repeated below. There are<br />

two positive i * values that satisfy the PW relation, 7.47% and 41.35% per year. Use the modified<br />

ROR method to determine the EROR value. Studies indicate that Honda has a WACC of<br />

8.5% per year and that projects with an estimated return of less than 9% per year are routinely<br />

rejected. Due to the nature of this contract business, any excess funds generated are expected<br />

to earn at a rate of 12% per year.<br />

Year 0 1 2 3<br />

Net Cash Flow ($1000) 2000 500 8100 6800


7.5 Techniques to Remove Multiple Rates of Return 187<br />

Solution by Spreadsheet<br />

Using the information in the problem statement, the rate estimates are as follows:<br />

MARR:<br />

Investment rate, i i :<br />

Borrowing rate, i b :<br />

9% per year<br />

12% per year<br />

8.5% per year<br />

The fast way to find i is with the MIRR function. Figure 7–10 shows the result of i ' 9.39%<br />

per year. Since 9.39% MARR, the project is economically justified.<br />

MIRR(B2:B5,E2,E3)<br />

Figure 7–10<br />

Spreadsheet application of MIRR function, Example 7.5 .<br />

It is vital that the interpretation be correct. The 9.39% is not the internal rate of return<br />

(IROR); it is the external ROR (EROR) based on the two external rates for investing and borrowing<br />

money.<br />

Solution by Hand<br />

Figure 7–9 can serve as a reference as the procedure to find i ' manually is applied.<br />

Step 1. Find PW 0 of all negative NCF at i b 8.5%.<br />

PW 0 500( P F ,8.5%,1) 8100( P F ,8.5%,2)<br />

$7342<br />

Step 2. Find FW 3 of all positive NCF at i i 12%.<br />

FW 3 2000( F P ,12%,3) 6800<br />

$9610<br />

Step 3. Find the rate i ' at which the PW and FW are equivalent.<br />

PW 0 ( F P , i ,3) FW 3 0<br />

7342(1 i ) 3 9610 0<br />

i ' ( 9610 ———<br />

7342 ) 13 1<br />

0.939 (9.39%)<br />

Step 4. Since i ' MARR of 9%, the project is economically justified using this EROR<br />

approach.<br />

Return on Invested Capital Approach<br />

The definition of ROIC should be understood before we discuss the approach.<br />

Return on invested capital (ROIC) is a rate-of-return measure of how effectively a project utilizes<br />

the funds invested in it, that is, funds that remain internal to the project. For a corporation, ROIC<br />

is a measure of how effectively it utilizes the funds invested in its operations, including facilities,<br />

equipment, people, systems, processes, and all other assets used to conduct business.


188 Chapter 7 Rate of Return Analysis: One Project<br />

The technique requires that the investment rate i i be estimated for excess funds generated in any<br />

year that they are not needed by the project. The ROIC rate, which has the symbol i , is determined<br />

using an approach called the net-investment procedure . It involves developing a series of future<br />

worth ( F ) relations moving forward 1 year at a time. In those years that the net balance of the<br />

project cash flows is positive (extra funds generated by the project), the funds are invested at the i i<br />

rate. Usually, i i is set equal to the MARR. When the net balance is negative, the ROIC rate is used,<br />

since the project keeps all of its funds internal to itself. The ROIC method uses the following procedure<br />

to determine a single external rate of return i and to evaluate the economic viability of the<br />

project. Remember that the perspective is that you are the project manager and when the project<br />

generates extra cash flows, they are invested external to the project at the investment rate i i .<br />

1. Develop a series of future worth relations by setting up the following relation for each year t<br />

( t 1, 2, . . . , n years).<br />

F t F t 1 (1 k ) NCF t [7.10]<br />

where F t future worth in year t based on previous year and time value of money<br />

NCF t net cash flow in year t<br />

k { i i if F t1 0 (extra funds available)<br />

i if F t1 0 (project uses all available funds)<br />

2. Set the future worth relation for the last year n equal to 0, that is, F n 0, and solve for i to<br />

balance the equation. The i value is the ROIC for the specified investment rate i i .<br />

The F t series and solution for i in the F n 0 relation can become involved mathematically.<br />

Fortunately, the Goal Seek spreadsheet tool can assist in the determination of i because<br />

there is only one unknown in the F n relation and the target value is zero. (Both the hand<br />

and spreadsheet solutions are demonstrated in Example 7.6 .)<br />

3. The guideline for economic decision making is the same as above, namely,<br />

If ROIC MARR, the project is economically justified.<br />

If ROIC MARR, the project is not economically justified.<br />

It is important to remember that the ROIC is an external rate of return dependent upon<br />

the investment rate choice. It is not the same as the internal rate of return discussed at the beginning<br />

of this chapter, nor is it the multiple rates, nor is it the MIRR rate found by the previous<br />

method. This is a separate technique to find a single rate for the project.<br />

EXAMPLE 7.6<br />

Once again, we will use the cash flows experienced by Honda Motors in Example 7.4 (repeated<br />

below). Use the ROIC method to determine the EROR value. The MARR is 9% per year, and<br />

any excess funds generated by the project can earn at a rate of 12% per year.<br />

Year 0 1 2 3<br />

Net Cash Flow ($1000) 2000 500 8100 6800<br />

Solution by Hand<br />

The hand solution is presented first to provide the logic of the ROIC method. Use MARR <br />

9% and i i 12% per year in the procedure to determine i , which is the ROIC. Figure 7–11<br />

details the cash flows and tracks the progress as each F t is developed. Equation [7.10] is applied<br />

to develop each F t .<br />

Step 1. Year 0: F 0 $2000<br />

Since F 0 0, externally invest in year 1 at i i 12%.<br />

Year 1: F 1 2000(1.12) 500 $1740<br />

Since F 1 0, use i i 12% for year 2.


7.5 Techniques to Remove Multiple Rates of Return 189<br />

$6800 $6800 $6800<br />

F 0 $2000 F 1 $1740<br />

F 3 = 0<br />

at i = ?<br />

0 1 2 3 0 1 2 3 0 1 2 3 0 1 2 3<br />

$500<br />

Year<br />

F 2 $6151<br />

$8100<br />

$8100<br />

(a) (b) (c) (d)<br />

Figure 7–11<br />

Application of ROIC method at i i 12% per year: ( a ) original cash flow; equivalent form in ( b ) year 1, ( c ) year 2, and ( d ) year 3.<br />

Year 2: F 2 1740(1.12) 8100 $6151<br />

Now F 2 0, use i for year 3, according to Equation [7.10].<br />

Year 3: F 3 6151(1 i ) 6800<br />

This is the last year. See Figure 7–11 for equivalent net cash flow diagrams.<br />

Go to step 2.<br />

Step 2. Solve for i ROIC from F 3 0.<br />

6151(1 i ) 6800 0<br />

i 68006151 1<br />

0.1055 (10.55%)<br />

Step 3. Since ROIC > MARR 9%, the project is economically justified.<br />

Solution by Spreadsheet<br />

Figure 7–12 provides a spreadsheet solution. The future worth values F 1 through F 3 are determined<br />

by the conditional IF statements in rows 3 through 5. The functions are shown in column<br />

D. In each year, Equation [7.10] is applied. If there are surplus funds generated by the project,<br />

F t –1 0 and the investment rate i i (in cell E7) is used to find F t . For example, because the F 1<br />

value (in cell C3) of $1740 0, the time value of money for the next year is calculated at the<br />

investment rate of 12% per year, as shown in the hand solution above for year 2.<br />

Figure 7–12<br />

Spreadsheet application of ROIC method using Goal Seek, Example 7.6 .


190 Chapter 7 Rate of Return Analysis: One Project<br />

The Goal Seek template sets the F 3 value to zero by changing the ROIC value (cell E8). The<br />

result is i ROIC 10.55% per year. As before, since 10.55% 9%, the MARR, the project<br />

is economically justified.<br />

Comment<br />

Note that the rate by the ROIC method (10.55%) is different than the MIRR rate (9.39%). Also<br />

these are both different than the multiple rates determined earlier (7.47% and 41.35%). This<br />

shows how dependent the different methods are upon the additional information provided<br />

when multiple i * rates are indicated by the two sign tests.<br />

Now that we have learned two techniques to remove multiple i * values, there are some connections<br />

between the multiple i * values, the external rate estimates, and the resulting external<br />

rates ( i and i ) obtained by the two methods.<br />

Modified ROR technique When both the borrowing rate i b and the investment rate i i are<br />

exactly equal to any one of the multiple i * values, the rate i ' found by the MIRR function, or by<br />

hand solution, will equal the i * value. That is, all four parameters have the same value.<br />

If any i * i b i i , then i ' i *<br />

ROIC technique Similarly, if the investment rate i i is exactly equal to any one of the multiple<br />

i * values, the rate found by the Goal Seek tool, or by hand when the equation F n 0 is solved,<br />

will be i i * value.<br />

Finally, it is very important to remember the following fact.<br />

None of the details of the modified ROR (MIRR) technique or the return on invested capital<br />

(ROIC) technique are necessary if the PW or AW method of project evaluation is applied at a<br />

specific MARR. When the MARR is established, this is, in effect, fixing the i* value. Therefore,<br />

a definitive economic decision can be made directly from the PW or AW value.<br />

7.6 Rate of Return of a Bond Investment<br />

A time-tested method of raising capital funds is through the issuance of an IOU, which is financing<br />

through debt, not equity (see Chapter 1). One very common form of IOU is a bond—a longterm<br />

note issued by a corporation or a government entity (the borrower) to finance major projects.<br />

The borrower receives money now in return for a promise to pay the face value V of the<br />

bond on a stated maturity date. Bonds are usually issued in face value amounts of $1000, $5000,<br />

or $10,000. Bond dividend I, also called bond interest , is paid periodically between the time the<br />

money is borrowed and the time the face value is repaid. The bond dividend is paid c times per<br />

year. Expected payment periods are usually semiannually or quarterly. The amount of interest is<br />

determined using the stated dividend or interest rate, called the bond coupon rate b.<br />

I <br />

(face value) (bond coupon rate)<br />

————————————————<br />

number of payment periods per year<br />

I Vb ——<br />

c [7.11]<br />

There are many types or classifications of bonds. Four general classifications are summarized<br />

in Table 7–5 according to their issuing entity, fundamental characteristics, and example names or<br />

purposes. For example, Treasury securities are issued in different monetary amounts ($1000 and<br />

up) with varying periods of time to the maturity date (Bills up to 1 year; Notes for 2 to 10 years).<br />

In the United States, Treasury securities are considered a very safe bond purchase because they are<br />

backed with the “full faith and credit of the U.S. government.” The safe investment rate indicated<br />

in Figure 16 as the lowest level for establishing a MARR is the coupon rate on a U.S. Treasury<br />

security. Funds obtained through corporate bond issues are used for new product development,<br />

facilities upgrade, expansion into international markets, and similar business ventures.


7.6 Rate of Return of a Bond Investment 191<br />

TABLE 7–5<br />

Classification and Characteristics of Bonds<br />

Classification Issued by Characteristics Examples<br />

Treasury securities Federal government Backed by faith and credit of the federal<br />

government<br />

Municipal Local governments Federal tax-exempt<br />

Issued against taxes received<br />

Mortgage Corporation Backed by specified assets or mortgage<br />

Low ratelow risk on first mortgage<br />

Foreclosure, if not repaid<br />

Debenture Corporation Not backed by collateral, but by<br />

reputation of corporation<br />

Bond rate may “float”<br />

Higher interest rates and higher risks<br />

EXAMPLE 7.7<br />

General Electric just released $10 million worth of $10,000 ten-year bonds. Each bond pays dividends<br />

semiannually at a rate of 6% per year. (a) Determine the amount a purchaser will receive each<br />

6 months and after 10 years. (b) Suppose a bond is purchased at a time when it is discounted by 2%<br />

to $9800. What are the dividend amounts and the final payment amount at the maturity date?<br />

Solution<br />

(a) Use Equation [7.11] for the dividend amount.<br />

10,000 (0.06)<br />

I —————— $300 per 6 months<br />

2<br />

The face value of $10,000 is repaid after 10 years.<br />

(b) Purchasing the bond at a discount from face value does not change the dividend or final repayment<br />

amounts. Therefore, $300 per 6 months and $10,000 after 10 years remain the amounts.<br />

Bills ( 1 year)<br />

Notes (2–10 years)<br />

Bonds (10–30 years)<br />

General obligation<br />

Revenue<br />

Zero coupon<br />

Put<br />

First mortgage<br />

Second mortgage<br />

Equipment trust<br />

Convertible<br />

Subordinated<br />

Junk or high yield<br />

The cash flow series for a bond investment is conventional and has one unique i *, which is<br />

best determined by solving a PW-based rate of return equation in the form of Equation [7.1] , that<br />

is, 0 PW.<br />

EXAMPLE 7.8<br />

Allied Materials needs $3 million in debt capital for expanded composites manufacturing.<br />

It is offering small-denomination bonds at a discount price of $800 for a 4% $1000 bond<br />

that matures in 20 years with a dividend payable semiannually. What nominal and effective<br />

interest rates per year, compounded semiannually, will Allied Materials pay an investor?<br />

Solution<br />

The income that a purchaser will receive from the bond purchase is the bond dividend I $20 every<br />

6 months plus the face value in 20 years. The PW-based equation for calculating the rate of return is<br />

0 800 20(PA,i*,40) 1000(PF,i*,40)<br />

Solve by the IRR function or by hand to obtain i* 2.8435% semiannually. The nominal interest<br />

rate per year is computed by multiplying i* by 2.<br />

Nominal i (2.8435)(2) 5.6870% per year, compounded semiannually<br />

Using Equation [4.5], the effective annual rate is<br />

i a (1.028435) 2 1 5.7678%


192 Chapter 7 Rate of Return Analysis: One Project<br />

EXAMPLE 7.9<br />

Gerry is a project engineer. He took a financial risk and bought a bond from a corporation that<br />

had defaulted on its interest payments. He paid $4240 for an 8% $10,000 bond with dividends<br />

payable quarterly. The bond paid no interest for the first 3 years after Gerry bought it. If interest<br />

was paid for the next 7 years and then Gerry was able to resell the bond for $11,000, what rate<br />

of return did he make on the investment? Assume the bond is scheduled to mature 18 years<br />

after he bought it. Perform hand and spreadsheet analysis.<br />

Solution by Hand<br />

The bond interest received in years 4 through 10 was<br />

I ——————<br />

(10,000)(0.08) $200 per quarter<br />

4<br />

The effective rate of return per quarter can be determined by solving the PW equation developed<br />

on a per quarter basis.<br />

0 4240 200(PA,i* per quarter,28)(PF,i* per quarter,12)<br />

11,000(PF,i* per quarter,40)<br />

The equation is correct for i* 4.1% per quarter, which is a nominal 16.4% per year, compounded<br />

quarterly.<br />

Solution by Spreadsheet<br />

Once all the cash flows are entered into contiguous cells, the function IRR(B2:B42) is used<br />

in Figure 7–13, row 43, to display the answer of a nominal rate of return of 4.10% per quarter.<br />

(Note that many of the row entries have been hidden to conserve space.) This is the same as the<br />

nominal annual rate of<br />

i* 4.10%(4) 16.4% per year, compounded quarterly<br />

Gerry did well on his bond investment.<br />

Figure 7–13<br />

Spreadsheet solution for<br />

a bond investment,<br />

Example 7.9.<br />

If a bond investment is being considered and a required rate of return is stated, the same<br />

PW-based relation used to find i * can be used to determine the maximum amount to pay for<br />

the bond now to ensure that the rate is realized. The stated rate is the MARR, and the PW<br />

computations are performed exactly as they were in Chapter 5. As an illustration, in the last<br />

example , if 12% per year, compounded quarterly, is the target MARR, the PW relation is<br />

used to find the maximum that Gerry should pay now; P is determined to be $6004. The<br />

quarterly MARR is 12%4 3%.<br />

0 P 200( P A ,3%,28)( P F ,3%,12) 11,000( P F ,3%,40)<br />

P $6004


Problems 193<br />

CHAPTER SUMMARY<br />

The rate of return of a cash flow series is determined by setting a PW-based or AW-based relation<br />

equal to zero and solving for the value of i *. The ROR is a term used and understood by almost<br />

everybody. Most people, however, can have considerable difficulty in calculating a rate of return<br />

correctly for anything other than a conventional cash flow series. For some types of series, more<br />

than one ROR possibility exists. The maximum number of i * values is equal to the number of<br />

changes in the sign of the net cash flow series (Descartes’ rule of signs). Also, a single positive<br />

rate can be found if the cumulative net cash flow series starts negatively and has only one sign<br />

change (Norstrom’s criterion).<br />

When multiple i * values are indicated, either of the two techniques covered in this chapter can<br />

be applied to find a single, reliable rate for the nonconventional net cash flow series. In the case<br />

of the ROIC technique, additional information is necessary about the investment rate that excess<br />

project funds will realize, while the modified ROR technique requires this same information, plus<br />

the borrowing rate for the organization considering the project. Usually, the investment rate is set<br />

equal to the MARR, and the borrowing rate takes on the historical WACC rate. Each technique<br />

will result in slightly different rates, but they are reliable for making the economic decision,<br />

whereas the multiple rates are often not useful to decision making.<br />

If an exact ROR is not necessary, it is strongly recommended that the PW or AW method at<br />

the MARR be used to decide upon economic justification.<br />

PROBLEMS<br />

Understanding ROR<br />

7.1 Under what circumstances would the rate of return<br />

be ( a ) 100%, and ( b ) infinite?<br />

7.2 A shrewd investor loaned $1,000,000 to a start-up<br />

company at 10% per year interest for 3 years, but<br />

the terms of the agreement were such that interest<br />

would be charged on the principal rather than on<br />

the unpaid balance. How much extra interest did<br />

the company pay?<br />

7.3 What is the nominal rate of return per year on an<br />

investment that increases in value by 8% every<br />

3 months?<br />

7.4 Assume you borrow $50,000 at 10% per year interest<br />

and you agree to repay the loan in five equal<br />

annual payments. What is the amount of the unrecovered<br />

balance immediately after you make the<br />

third payment?<br />

7.5 International Potash got a $50 million loan amortized<br />

over a 10-year period at 10% per year interest.<br />

The loan agreement stipulates that the loan<br />

will be repaid in 10 equal annual payments with<br />

interest charged on the principal amount of the<br />

loan (not on the unrecovered balance).<br />

( a) What is the amount of each payment?<br />

( b) What is the total amount of interest paid?<br />

How does the total interest paid compare<br />

with the principal of the loan?<br />

Determination of ROR<br />

7.6 In 2010, the city of Houston, Texas, collected<br />

$24,112,054 in fines from motorists because of<br />

traffic violations caught by red-light cameras. The<br />

cost of operating the system was $8,432,372. The<br />

net profit, that is, profit after operating costs, is<br />

split equally (that is, 50% each) between the city<br />

and the operator of the camera system. What will<br />

be the rate of return over a 3-year period to the<br />

contractor that paid for, installed, and operates the<br />

system, if its initial cost was $9,000,000 and the<br />

profit for each of the 3 years is the same as it was<br />

in 2010?<br />

7.7 P&G sold its prescription drug business to<br />

Warner-Chilcott, Ltd. for $3.1 billion. If income<br />

from product sales is $2 billion per year and net<br />

profit is 20% of sales, what rate of return will<br />

the company make over a 10-year planning<br />

horizon?<br />

7.8 Water damage from a major flood in a Midwestern<br />

city resulted in damages estimated at<br />

$108 million. As a result of the claimant payouts,<br />

insurance companies raised homeowners' insurance<br />

rates by an average of $59 per year for each<br />

of the 160,000 households in the affected city. If<br />

a 20-year study period is considered, what was<br />

the rate of return on the $108 million paid by the<br />

insurance companies?


194 Chapter 7 Rate of Return Analysis: One Project<br />

7.9 Determine the rate of return for the cash flows shown<br />

in the diagram. (If requested by your instructor,<br />

show both hand and spreadsheet solutions.)<br />

to 1,694,247 in 2015. If the increase were to occur<br />

uniformly, what rate of increase would be required<br />

each year to meet the goal?<br />

0 1 2 3 4 5 6 7 8<br />

$3000<br />

$200<br />

$200 $200<br />

i =?<br />

$7000<br />

$90 $90 $90<br />

Year<br />

7.14 U.S. Census Bureau statistics show that the annual<br />

earnings for persons with a high school diploma<br />

are $35,220 versus $57,925 for someone with a<br />

bachelor’s degree. If the cost of attending college<br />

is assumed to be $30,000 per year for 4 years and<br />

the forgone earnings during those years are assumed<br />

to be $35,220 per year, what rate of return<br />

does earning a bachelor’s degree represent? Use a<br />

35-year study period. ( Hint : The investment in<br />

years 1 through 4 is the cost of college plus the<br />

foregone earnings, and the income in years 5<br />

through 35 is the difference in income between a<br />

high school diploma and a bachelor’s degree.)<br />

7.10 The Office of Naval Research sponsors a contest<br />

for college students to build underwater robots that<br />

can perform a series of tasks without human intervention.<br />

The University of Florida, with its Subju-<br />

Gator robot, won the $7000 first prize (and serious<br />

bragging rights) over 21 other universities. If the<br />

team spent $2000 for parts (at time 0) and the project<br />

took 2 years, what annual rate of return did the<br />

team make?<br />

7.11 For the cash flows shown, determine the rate of<br />

return.<br />

Year 0 1 2 3 4 5<br />

Expense, $ 17,000 2,500 2,500 2,500 2,500 2,500<br />

Revenue, $ 0 5,000 6,000 7,000 8,000 12,000<br />

7.12 In an effort to avoid foreclosure proceedings on<br />

struggling mortgage customers, Bank of America<br />

proposed an allowance that a jobless customer<br />

make no payment on their mortgage for up to<br />

9 months. If the customer did not find a job within<br />

that time period, they would have to sign over their<br />

house to the bank. The bank would give them<br />

$2000 for moving expenses.<br />

Assume John and his family had a mortgage<br />

payment of $2900 per month and he was not able<br />

to find a job within the 9-month period. If the bank<br />

saved $40,000 in foreclosure costs, what rate of<br />

return per month did the bank make on the allowance?<br />

Assume the first payment that was skipped<br />

was due at the end of month 1 and the $40,000<br />

foreclosure savings and $2000 moving expense<br />

occurred at the end of the 9-month forbearance<br />

period.<br />

7.13 The Closing the Gaps initiative by the Texas<br />

Higher Education Coordinating Board established<br />

the goal of increasing the number of students in<br />

higher education in Texas from 1,064,247 in 2000<br />

7.15 The Ester Municipal Water Utility issued 20-year<br />

bonds in the amount of $53 million for several<br />

high-priority flood control improvement projects.<br />

The bonds carried a 5.38% dividend rate with the<br />

dividend payable annually. The U.S. economy was<br />

in a recession at that time, so as part of the federal<br />

stimulus program, the Utility gets a 35% reimbursement<br />

on the dividend it pays.<br />

(a) What is the effective dividend rate that the<br />

Utility is paying on the bonds?<br />

(b)<br />

What is the total dollar amount the Utility will<br />

save in dividends over the life of the bonds?<br />

(c) What is the future worth in year 20 of the<br />

dividend savings, if the interest rate is<br />

6% per year?<br />

7.16 A contract between BF Goodrich and the Steelworkers<br />

Union of America called for the company<br />

to spend $100 million in capital investment to keep<br />

the facilities competitive. The contract also required<br />

the company to provide buyout packages<br />

for 400 workers. If the average buyout package is<br />

$100,000 and the company is able to reduce costs<br />

by $20 million per year, what rate of return will the<br />

company make over a 10-year period? Assume all<br />

of the company’s expenditures occur at time 0 and<br />

the savings begin 1 year later.<br />

7.17 Rubber sidewalks made from ground up tires are<br />

said to be environmentally friendly and easier on<br />

people’s knees. Rubbersidewalks, Inc. of Gardena,<br />

California, manufactures the small rubberized<br />

squares that are being installed where tree roots,<br />

freezing weather, and snow removal have required<br />

sidewalk replacement or major repairs every<br />

3 years. The District of Columbia spent $60,000 for<br />

a rubber sidewalk to replace broken concrete in a<br />

residential neighborhood lined with towering willow<br />

oaks. If a concrete sidewalk costs $28,000 and


Problems 195<br />

lasts only 3 years versus a 9-year life for the rubber<br />

sidewalks, what rate of return does this represent?<br />

7.18 Efficient light jets (ELJs) are smaller aircraft that<br />

may revolutionize the way people travel by plane.<br />

They cost between $1.5 and $3 million, seat 5 to 7<br />

people, and can fly up to 1100 miles at cruising<br />

speeds approaching 425 mph. Eclipse Aerospace<br />

was founded in 2009, and its sole business is making<br />

ELJs. The company invested $500 million<br />

(time 0) and began taking orders 2 years later. If<br />

the company accepted orders for 2500 planes and<br />

received 10% down (in year 2) on planes having<br />

an average cost of $1.8 million, what rate of return<br />

will the company make over a 10-year planning<br />

period? Assume 500 of the planes are delivered<br />

each year in years 6 through 10 and that the company’s<br />

M&O costs average $10 million per year in<br />

years 1 through 10. (If requested by your instructor,<br />

show both hand and spreadsheet solutions.)<br />

7.19 Betson Enterprises distributes and markets the Big<br />

Buck video game which allows players to “hunt”<br />

for elk, antelope, moose, and bucks without shivering<br />

outside in the cold. E-sports entertainment in<br />

New York City purchased five machines for $6000<br />

each and took in an average of $600 total per week<br />

in sales. What rate of return does this represent<br />

( a ) per week and ( b ) per year (nominal)? Use a<br />

3-year study period with 52 weeks per year.<br />

7.20 A 473-foot, 7000-ton World War II troop carrier<br />

(once commissioned as the USS Excambion ) was<br />

sunk in the Gulf of Mexico to serve as an underwater<br />

habitat and diving destination. The project<br />

took 10 years of planning and cost $4 million.<br />

Assume the $4 million was expended equally in<br />

years 1 through 10. What rate of return does the<br />

venture represent, if increased fishing and recreation<br />

activities are valued at $270,000 per year<br />

beginning in year 11 and they continue in perpetuity?<br />

(If assigned by your instructor, show both<br />

hand and spreadsheet solutions.)<br />

Multiple ROR Values<br />

7.21 What is meant by a nonconventional cash flow<br />

series?<br />

7.22 Explain at least three types of projects in which<br />

large net cash flow changes may cause sign changes<br />

during the life of the project, thus indicating the<br />

possible presence of multiple ROR values.<br />

7.23 Explain a situation with which you are personally<br />

familiar for which the net cash flows have<br />

changed signs in a fashion similar to those in<br />

Figure 7–5 .<br />

7.24 According to Descartes’ rule of signs, what is the<br />

maximum number of real-number values that will<br />

balance a rate of return equation?<br />

7.25 According to Descartes’ rule of signs, how many<br />

possible i * values are there for net cash flows that<br />

have the following signs?<br />

(a) <br />

(b) <br />

(c) <br />

7.26 According to Norstrom’s criterion, there are two<br />

requirements regarding the cumulative cash flows<br />

that must be satisfied to ensure that there is only<br />

one positive root in a rate of return equation. What<br />

are they?<br />

7.27 According to Descartes’ rule of signs, how many<br />

possible i * values are there for the cash flows<br />

shown?<br />

Year 1 2 3 4 5 6<br />

Net Cash<br />

Flow, $<br />

4100 2000 7000 12,000 700 800<br />

7.28 According to Descartes’ rule of signs, how many<br />

i* values are possible for the cash flows shown?<br />

Year 1 2 3 4<br />

Revenue, $ 25,000 13,000 4,000 70,000<br />

Costs, $ 30,000 7,000 6,000 12,000<br />

7.29 According to Descartes’ rule and Norstrom’s criterion,<br />

how many i* values are possible for the cash<br />

flow (CF ) sequence shown?<br />

Year 1 2 3 4 5<br />

Net Cash 16,000 32,000 25,000 50,000 8,000<br />

Flow, $<br />

Cumulative<br />

CF, $<br />

16,000 16,000 41,000 9,000 1,000<br />

7.30 For the cash flows shown, determine the sum of<br />

the cumulative cash flows.<br />

Year 0 1 2 3 4<br />

Revenue, $ 25,000 15,000 4,000 18,000<br />

Costs, $ 6,000 30,000 7,000 6,000 12,000<br />

7.31 Stan-Rite Corp of Manitowoc, Wisconsin, is a B to<br />

B company that manufactures many types of industrial<br />

products, including portable measuring<br />

arms with absolute encoders, designed to perform<br />

3D inspections of industrial parts. If the company’s<br />

cash flow (in millions) for one of its product divisions<br />

is as shown on the next page, determine<br />

( a ) the number of possible i * values and ( b ) all rate<br />

of return values between 0% and 100%.


196 Chapter 7 Rate of Return Analysis: One Project<br />

Year Expenses Revenues<br />

0 $30 $ 0<br />

1 20 18<br />

2 25 19<br />

3 15 36<br />

4 22 52<br />

5 20 38<br />

6 30 70<br />

7.32 Julie received a $50 bill for her birthday at the end<br />

of January. At the end of February, she spent this<br />

$50 and an additional $150 to buy clothes. Her<br />

parents then gave her $50 and $125 at the end of<br />

March and April, respectively, as she prepared to<br />

go to summer school and needed the clothes. Her<br />

conclusion was that over the 4 months, she had received<br />

$25 more than she spent. Determine if Julie<br />

has a multiple rate of return situation for these cash<br />

flows. If so, determine the multiple rates and comment<br />

on their validity. The cash flow values are as<br />

follows:<br />

Month Jan (0) Feb (1) Mar (2) Apr (3)<br />

Cash Flow, $ 50 200 50 125<br />

7.33 Veggie Burger Boy sells franchises to individuals<br />

who want to start small in the sandwiches-forvegeterians<br />

business and grow in net cash flow<br />

over the years. A franchisee in Mississippi provided<br />

the $5000 up-front money, but did very<br />

poorly the first year. He was allowed to borrow at<br />

the end of his first year from the corporation’s capital<br />

incentive fund with a promise to repay the loan<br />

in addition to the annual share that the corporation<br />

contractually receives from annual sales. The net<br />

cash flows from the corporation’s perspective are<br />

shown below.<br />

Year 0 1 2 3 4 5 6<br />

NCF, $ 5000 10,100 500 2000 2000 2000 2000<br />

The corporate chief financial officer (CFO) has<br />

some questions concerning this NCF series. Help<br />

her by doing the following, using a spreadsheet.<br />

(a) Plot the PW versus i graph and estimate the<br />

rate of return for this franchise.<br />

(b)<br />

Use the IRR function on a spreadsheet to find<br />

the corresponding return.<br />

(c) Basing your conclusions on Descartes’ and<br />

Norstrom’s rules, provide the CFO with<br />

some advice on what ROR value is the most<br />

reliable for this franchise over the 6-year period.<br />

The normal corporate MARR used for<br />

franchisee evaluation is 30% per year.<br />

7.34 In 2011, Vaught Industries closed its plant in<br />

Marionsville following labor, environmental, and<br />

safety problems. The plant was built in 2005<br />

based on an older technology to produce meat<br />

products. It had positive NCF until 2010 and discontinued<br />

operation in 2011 due to labor and<br />

safety problems. In <strong>2012</strong>, prior to the sale of the<br />

facility and property, Vaught spent $1 million to<br />

make the site environmentally acceptable to a potential<br />

buyer. The net cash flows in $100,000 over<br />

the years are listed below. Use a spreadsheet to do<br />

the following.<br />

(a) Check for multiple rates of return.<br />

(b) Find all rates that are real numbers between<br />

25% and 50%, and calculate the PW<br />

value for interest rates in this range.<br />

(c) Indicate which is the best and correct i * value<br />

to use in a PW analysis.<br />

Year 2005 2006 2007 2008 2009 2010 2011 <strong>2012</strong><br />

NCF, $ 25 10 10 15 15 5 6 10<br />

7.35 Five years ago, VistaCare spent $5 million to develop<br />

and introduce a new service in home health<br />

care for people who require frequent blood dialysis<br />

treatments. The service was not well received<br />

after the first year and was removed from the market.<br />

When reintroduced 4 years after its initial<br />

launch, it was much more profitable. Now, in year<br />

5, VistaCare has spent a large sum on research to<br />

broaden the application of this service. Use the<br />

NCF series below to plot the PW versus i graph<br />

and estimate the ROR over the 5 years. NCF values<br />

are in $1 million units.<br />

Year 0 1 2 3 4 5<br />

Net Cash Flow, $ 5000 5000 0 0 15,000 15,000<br />

Removing Multiple i * Values<br />

7.36 In calculating the external rate of return by the<br />

modified rate of return approach, it is necessary to<br />

use two different rates of return, the investment rate<br />

i i and the borrowing rate i b . When is each used?<br />

7.37 In the modified rate of return approach for determining<br />

a single interest rate from net cash flows,<br />

state which interest rate is usually higher, the investment<br />

rate i i or the borrowing rate i b . State why.<br />

7.38 Use the modified rate of return approach with an<br />

investment rate of 18% per year and a borrowing<br />

rate of 10% to find the external rate of return for<br />

the following cash flows.<br />

Year 0 1 2 3<br />

Net Cash Flow, $ 16,000 32,000 25,000 70,000<br />

7.39 Harley worked for many years to save enough money<br />

to start his own residential landscape design business.<br />

The cash flows shown are those he recorded for the<br />

first 6 years as his own boss. Find the external rate of


Problems 197<br />

return using the modified rate of return approach, an<br />

investment rate of 15% per year, and a borrowing rate<br />

of 8%. (After using the procedure, use the MIRR<br />

function to confirm your answer.)<br />

Year 0 1 2 3 4 5 6<br />

NCF, $ 9000 4100 2000 7000 12,000 700 800<br />

7.40 Samara, an engineer working for GE, invested her<br />

bonus money each year in company stock. Her<br />

bonus has been $8000 each year for the past<br />

6 years (i.e., at the end of years 1 to 6). At the end<br />

of year 7, she sold the stock for $52,000 to buy a<br />

condo; she purchased no stock that year. In years 8<br />

to 10, she again invested the $8000 bonus. Samara<br />

sold all of the remaining stock for $28,000 immediately<br />

after the investment at the end of year 10.<br />

( a) Determine the number of possible rate of return<br />

values in the net cash flow series using<br />

the two sign tests.<br />

( b) Determine the external rate of return by<br />

hand, using the modified rate of return approach<br />

with an investment rate of 12% per<br />

year and a borrowing rate of 8%.<br />

( c) Find the external rate of return by spreadsheet<br />

using the ROIC approach with an investment<br />

rate of 12% per year.<br />

(d) Enter the cash flows into a spreadsheet, and use<br />

the IRR function to find the i * value. You should<br />

get the same value as the ROIC in part ( c ). Explain<br />

why this is so, given that the investment<br />

rate is 12% per year. ( Hint : Look carefully at the<br />

column labeled “Future worth, F, $” when you<br />

solved part ( c ) using the spreadsheet.)<br />

7.41 Swagelok Co. of Solon, Ohio, makes variable area<br />

flowmeters (VAFs) that measure liquid and gas flow<br />

rates by means of a tapered tube and float. If tooling<br />

and setup costs were $400,000 in year 0 and an additional<br />

$190,000 in year 3, determine the external<br />

rate of return using the modified rate of return approach.<br />

The revenue was $160,000 per year in<br />

years 1 through 10. Assume the company’s MARR<br />

is 20% per year and its cost of capital is 9% per year.<br />

7.42 A company that makes clutch disks for race cars<br />

has the cash flows shown for one department.<br />

Year Cash Flow, $1000<br />

0 65<br />

1 30<br />

2 84<br />

3 10<br />

4 12<br />

( a) Determine the number of positive roots to the<br />

rate of return relation.<br />

( b) Calculate the internal rate of return.<br />

(c)<br />

Calculate the external rate of return using the<br />

return on invested capital (ROIC) approach<br />

with an investment rate of 15% per year. (As<br />

assigned by your instructor, solve by hand<br />

andor spreadsheet.)<br />

7.43 For the cash flow series below, calculate the external<br />

rate of return, using the return on invested capital<br />

approach with an investment rate of 14% per<br />

year.<br />

Year Cash Flow, $<br />

0 3000<br />

1 2000<br />

2 1000<br />

3 6000<br />

4 3800<br />

7.44 Five years ago, a company made a $500,000 investment<br />

in a new high-temperature material. The<br />

product did poorly after only 1 year on the market.<br />

However, with a new name and advertising campaign<br />

4 years later it did much better. New development<br />

funds have been expended this year<br />

(year 5) at a cost of $1.5 million. Determine the<br />

external rate of return using the ROIC approach<br />

and an investment rate of 15% per year. The i * rate<br />

is 44.1% per year.<br />

Year Cash Flow, $<br />

0 500,000<br />

1 400,000<br />

2 0<br />

3 0<br />

4 2,000,000<br />

5 1,500,000<br />

Bonds<br />

7.45 What is the bond coupon rate on a $25,000 mortgage<br />

bond that has semiannual interest payments<br />

of $1250 and a 20-year maturity date?<br />

7.46 An equipment trust bond with a face value of<br />

$10,000 has a bond coupon rate of 8% per year,<br />

payable quarterly. What are the amount and frequency<br />

of the dividend payments?<br />

7.47 What is the face value of a municipal bond that<br />

matures in 20 years and has a bond coupon rate<br />

of 6% per year with semiannual payments of<br />

$900?<br />

7.48 What is the present worth of a $50,000 debenture<br />

bond that has a bond coupon rate of 8% per year,<br />

payable quarterly? The bond matures in 15 years.<br />

The interest rate in the marketplace is 6% per year,<br />

compounded quarterly.


198 Chapter 7 Rate of Return Analysis: One Project<br />

7.49 Best Buy issued collateral bonds 4 years ago that<br />

have a face value of $20,000 each and a coupon<br />

rate of 8% per year, payable semiannually. If the<br />

bond maturity date is 20 years from the date they<br />

were issued and the interest rate in the marketplace<br />

is now 12% per year, compounded semiannually,<br />

what is the present worth (now) of one bond?<br />

7.50 In 2011, El Paso Water Utilities (EPWU) issued<br />

bonds worth $9.125 million to improve the Van<br />

Buren dam in central El Paso and to finance three<br />

other drainage projects. The bonds were purchased<br />

by the Texas Water Development Board under the<br />

federal stimulus program wherein EPWU did not<br />

have to pay any dividend on the bonds. If the bond<br />

dividend rate would have been 4% per year, payable<br />

quarterly, with a bond maturity date 18 years<br />

after issuance, what is the present worth of the<br />

dividend savings to EPWU rate payers? Assume<br />

the market interest rate is 6% per year.<br />

7.51 A recently issued industrial bond with a face value<br />

of $10,000 has a coupon rate of 8% per year, payable<br />

annually. The bond matures 20 years from now. Jeremy<br />

is interested in buying one bond. If he pays<br />

$10,000 for the bond and plans to hold it to maturity,<br />

what rate of return per year will he realize?<br />

7.52 Due to a significant troop buildup at the local military<br />

base, a school district issued $10,000,000 in<br />

bonds to build new schools. The bond coupon rate<br />

is 6% per year, payable semiannually, with a maturity<br />

date of 20 years. If an investor is able to purchase<br />

one of the bonds that has a face value of<br />

$5000 for $4800, what rate of return per 6 months<br />

will the investor realize? Assume the bond is kept<br />

to maturity.<br />

7.53 As the name implies, a zero-coupon bond pays no<br />

dividend, only the face value when it matures. If a<br />

zero coupon bond that has a face value of $10,000<br />

and a maturity date of 15 years is for sale for<br />

$2000, what rate of return will the purchaser make,<br />

provided the bond is held to maturity?<br />

7.54 To provide infrastructure in the outlying areas of<br />

Morgantown, West Virginia, the city council<br />

issued 30-year bonds with a face value of $25 million.<br />

The bond coupon rate was set at 5% per year,<br />

payable semiannually. Because the market interest<br />

rate increased immediately before the bonds were<br />

sold, the city received only $23.5 million from the<br />

bond sale. What was the semiannual interest rate<br />

when the bonds were sold?<br />

7.55 An investor who purchased a $10,000 mortgage<br />

bond today paid only $6000 for it. The bond coupon<br />

rate is 8% per year, payable quarterly, and the<br />

maturity date is 18 years from the year of issuance.<br />

Because the bond is in default, it will pay no dividend<br />

for the next 2 years. If the bond dividend is in<br />

fact paid for the following 5 years (after the<br />

2 years) and the investor then sells the bond for<br />

$7000, what rate of return will be realized ( a ) per<br />

quarter and ( b ) per year (nominal)?<br />

7.56 Five years ago, GSI, an oil services company<br />

headquartered in Texas, issued $10 million worth<br />

of 12% 30-year bonds with the dividend payable<br />

quarterly. The bonds have a call date of this year if<br />

GSI decides to take advantage of it. The interest<br />

rate in the marketplace decreased enough that the<br />

company is considering calling the bonds since the<br />

coupon rate is relatively high. If the company buys<br />

the bonds back now for $11 million, determine the<br />

rate of return that the company will make ( a ) per<br />

quarter and ( b ) per year (nominal). ( Hint : The<br />

“call” option means the following: By spending<br />

$11 million now, the company will not have to<br />

make the quarterly bond dividend payments or pay<br />

the face value of the bonds when they come due<br />

25 years from now.)<br />

7.57 Four years ago, Chevron issued $5 million worth<br />

of debenture bonds with a coupon rate of 10% per<br />

year, payable semiannually. Market interest rates<br />

dropped, and the company called the bonds (i.e.,<br />

paid them off in advance) at a 10% premium on the<br />

face value. Therefore, it cost the corporation<br />

$5.5 million to retire the bonds. What semiannual<br />

rate of return did an investor make who purchased<br />

a $5000 bond 4 years ago and held it until it was<br />

called 4 years later?<br />

ADDITIONAL PROBLEMS AND FE EXAM REVIEW QUESTIONS<br />

7.58 All of the following mean the same as rate of<br />

return except:<br />

(a) Internal rate of return<br />

(b) Time for return of capital<br />

(c) Interest rate<br />

(d) Return on investment<br />

7.59 The numerical value of i in a rate of return equation<br />

can range from:<br />

(a) 0% to 100%<br />

(b) 0 to <br />

(c) 100% to 100%<br />

(d) 100% to


Additional Problems and FE Exam Review Questions 199<br />

7.60 The internal rate of return on an investment refers<br />

to the interest rate earned on the:<br />

( a) Initial investment<br />

( b) Unrecovered balance of the investment<br />

( c) Money recovered from an investment<br />

( d) Income from an investment<br />

7.61 A conventional (or simple) cash flow series is one<br />

wherein :<br />

( a) The algebraic signs on the net cash flows<br />

change only once.<br />

( b) The interest rate you get is a simple interest<br />

rate.<br />

( c) The total of the net cash flows is equal to 0.<br />

( d) The total of the cumulative cash flows is<br />

equal to 0.<br />

7.62 According to Descartes’ rule of signs, for a net<br />

cash flow sequence of , the number of<br />

possible i values is:<br />

( a) 2<br />

( b) 3<br />

( c) 4<br />

( d) 5<br />

7.63 According to Norstrom’s criterion, the one statement<br />

below that is correct is :<br />

( a) The cumulative cash flow must start out<br />

positively.<br />

( b) The cumulative cash flow must start out<br />

negatively.<br />

( c) The cumulative cash flow must equal 0.<br />

( d) The net cash flow must start out positively.<br />

7.64 According to Descartes’ rule and Norstrom’s criterion,<br />

the number of positive i * values for the<br />

following cash flow sequence is:<br />

Year 1 2 3 4<br />

Revenue, $ 25,000 15,000 4,000 18,000<br />

Cost, $ 30,000 7,000 6,000 12,000<br />

( a) 1<br />

( b) 2<br />

( c) 3<br />

( d) 4<br />

7.65 For the net cash flows and cumulative cash flows<br />

shown, the value of x is:<br />

Year 1 2 3 4 5<br />

NCF, $ 13,000 29,000 25,000 x 8000<br />

Cumulative 13,000 16,000 41,000 9000 1000<br />

NCF, $<br />

( a) $7000<br />

( b) $16,000<br />

( c) $36,000<br />

( d) $50,000<br />

7.66 A company that uses a minimum attractive rate of<br />

return of 10% per year is evaluating new processes<br />

to improve operational efficiency. The estimates<br />

associated with candidate processes are shown.<br />

Alternative I<br />

Alternative J<br />

First cost, $ 40,000 50,000<br />

Annual cost, $ per year 15,000 12,000<br />

Salvage value, $ 5,000 5,000<br />

Life, years 3 6<br />

The statement that is most correct is :<br />

(a) The alternatives are revenue alternatives.<br />

(b) The alternatives are cost alternatives.<br />

(c) The alternatives are revenue alternatives and<br />

DN is an option.<br />

(d) The alternatives are cost alternatives and DN<br />

is an option.<br />

7.67 Scientific Instruments, Inc. uses a MARR of 8% per<br />

year. The company is evaluating a new process to<br />

reduce water effluents from its manufacturing processes.<br />

The estimate associated with the process follows.<br />

In evaluating the process on the basis of a rate<br />

of return analysis, the correct equation to use is:<br />

(a)<br />

(b)<br />

(c)<br />

(d)<br />

New Process<br />

First cost, $ 40,000<br />

NCF, $ per year 13,000<br />

Salvage value, $ 5,000<br />

Life, years 3<br />

0 40,000 13,000( P A , i ,3) <br />

5000( P F , i ,3)<br />

0 40,000( AP,i ,3) 13,000 <br />

5000( A F , i ,3)<br />

0 40,000( FP,i ,3) 13,000( FA,i, 3) <br />

5000<br />

Any of the above<br />

7.68 When one is using the modified ROR method to remove<br />

multiple ROR values, an additional estimate<br />

needed besides the cash flows and their timings is:<br />

(a) The ROIC value<br />

(b) External rate of return<br />

(c) Investment rate<br />

(d) Internal rate of return<br />

7.69 For the following cash flows, the modified rate of<br />

return method uses a borrowing rate of 10%, and<br />

an investment rate is 12% per year. The correct<br />

computation for the present worth in year 0 is:<br />

Year 1 2 3 4 5<br />

NCF, $ 10,000 0 0 19,000 25,000<br />

(a) 10,000 19,000( P F ,12%,4)<br />

(b) 10,000 19,000( P F ,12%,4) <br />

25,000( P F ,10%,5)


200 Chapter 7 Rate of Return Analysis: One Project<br />

(c) 25,000( P F ,10%,5)<br />

(d) 10,000 19,000( P F ,10%,4)<br />

7.70 The return on invested capital (ROIC) method removes<br />

multiple ROR values from a cash flow sequence.<br />

If the future worth computation in year t is<br />

F t 0, the ROIC rate i is used. The interpretation<br />

of F t 0 most closely means:<br />

(a) The net balance of project cash flows in year<br />

t is negative.<br />

(b) The resulting external rate of return will be<br />

(c)<br />

positive.<br />

The net balance of project cash flows is positive<br />

in year t .<br />

(d) The sequence has nonremovable negative<br />

ROR values.<br />

7.71 The meaning of return on invested capital for a<br />

corporation is best stated as :<br />

(a) A rate-of-return measure that equates the internal<br />

and external ROR<br />

(b) A measure of how effectively the corporation<br />

uses capital funds invested in it<br />

(c) The value at which borrowing ROR and investing<br />

ROR are equal<br />

(d) The external rate of return value is based on<br />

total capital invested<br />

7.72 A corporate $10,000 bond has a coupon rate of 8%<br />

per year, payable semiannually. The bond matures<br />

20 years from now. At an interest rate of 6% per<br />

year, compounded semiannually, the amount and<br />

frequency of the bond dividend payments are :<br />

(a) $600 every 6 months<br />

(b) $800 every 6 months<br />

(c) $300 every 6 months<br />

(d) $400 every 6 months<br />

7.73 A $20,000 mortgage bond that is due in 1 year<br />

pays interest of $500 every 3 months. The bond’s<br />

coupon rate is:<br />

(a) 2.5% per year, payable quarterly<br />

(b) 5% per year, payable quarterly<br />

(c) 5% per year, payable semiannually<br />

(d) 10% per year, payable quarterly<br />

7.74 A $10,000 municipal bond due in 10 years pays<br />

interest of $400 every 6 months. If an investor purchases<br />

the bond now for $9000 and holds it to maturity,<br />

the rate of return received can be determined<br />

by the following equation:<br />

(a) 0 9000 400( P A , i ,10)<br />

10,000( P F , i ,10)<br />

(b) 0 9000 400( P A , i ,20)<br />

10,000( P F , i ,20)<br />

(c) 0 10,000 400( P A , i ,20)<br />

10,000( P F , i ,20)<br />

(d) 0 9000 800( P A , i ,10)<br />

10,000( P F , i ,10)<br />

7.75 A debenture bond issued 3 years ago has a face<br />

value of $5000, a coupon rate of 4% per year, payable<br />

semiannually, and a maturity date of 20 years<br />

from the date it was issued . The bond is for sale<br />

now for $4500. If the interest rate in the marketplace<br />

is compounded quarterly, the value of n that<br />

must be used in the P A factor to calculate the rate<br />

of return for the bond is:<br />

( a) 34<br />

( b) 40<br />

( c) 68<br />

( d) 80<br />

CASE STUDY<br />

DEVELOPING AND SELLING AN INNOVATIVE IDEA<br />

Background<br />

Three engineers who worked for Mitchell <strong>Engineering</strong>, a<br />

company specializing in public housing development, went<br />

to lunch together several times a week. Over time they decided<br />

to work on solar energy production ideas. After a lot of<br />

weekend time over several years, they had designed and developed<br />

a prototype of a low-cost, scalable solar energy plant<br />

for use in multifamily dwellings on the low end and mediumsized<br />

manufacturing facilities on the upper end. For residential<br />

applications, the collector could be mounted along side a<br />

TV dish and be programmed to track the sun. The generator<br />

and additional equipment are installed in a closet-sized area<br />

in an apartment or on a floor for multiple-apartment supply.<br />

The system serves as a supplement to the electricity provided<br />

by the local power company. After some 6 months of testing,<br />

it was agreed that the system was ready to market and reliably<br />

state that an electricity bill in high-rises could be reduced by<br />

approximately 40% per month. This was great news for lowincome<br />

dwellers on government subsidy that are required to<br />

pay their own utility bills.<br />

Information<br />

With a hefty bank loan and $200,000 of their own capital,<br />

they were able to install demonstration sites in three cities in<br />

the sunbelt. Net cash flow after all expenses, loan repayment,<br />

and taxes for the first 4 years was acceptable; $55,000 at the


Case Study 201<br />

end of the first year, increasing by 5% each year thereafter. A<br />

business acquaintance introduced them to a potential buyer of<br />

the patent rights and current subscriber base with an estimated<br />

$500,000 net cashout after only these 4 years of ownership.<br />

However, after serious discussion replaced the initial<br />

excitement of the sales offer, the trio decided to not sell at this<br />

time. They wanted to stay in the business for a while longer<br />

to develop some enhancement ideas and to see how much<br />

revenue may increase over the next few years.<br />

During the next year, the fifth year of the partnership, the<br />

engineer who had received the patents upon which the collector<br />

and generator designs were based became very displeased<br />

with the partnering arrangements and left the trio to go into<br />

partnership with an international firm in the energy business.<br />

With new research and development funds and the patent<br />

rights, a competing design was soon on the market and took<br />

much of the business away from the original two developers.<br />

Net cash flow dropped to $40,000 in year 5 and continued to<br />

decrease by $5000 per year. Another offer to sell in year 8<br />

was presented, but it was only for $100,000 net cash. This<br />

was considered too much of a loss, so the two owners did not<br />

accept. Instead, they decided to put $200,000 more of their<br />

own savings into the company to develop additional applications<br />

in the housing market.<br />

It is now 12 years since the system was publicly launched.<br />

With increased advertising and development, net cash flow<br />

has been positive the last 4 years, starting at $5000 in year 9<br />

and increasing by $5000 each year until now.<br />

Case Study Exercises<br />

It is now 12 years after the products were developed, and the<br />

engineers invested most of their savings in an innovative<br />

idea. However, the question of “When do we sell?” is always<br />

present in these situations. To help with the analysis, determine<br />

the following:<br />

1. The rate of return at the end of year 4 for two situations:<br />

( a ) The business is sold for the net cash amount of<br />

$500,000 and ( b ) no sale.<br />

2. The rate of return at the end of year 8 for two situations:<br />

( a ) The business is sold for the net cash amount of<br />

$100,000 and ( b ) no sale.<br />

3. The rate of return now at the end of year 12.<br />

4. Consider the cash flow series over the 12 years. Is there<br />

any indication that multiple rates of return may be present?<br />

If so, use the spreadsheet already developed to<br />

search for ROR values in the range 100% other than<br />

the one determined in exercise 3 above.<br />

5. Assume you are an investor with a large amount of<br />

ready cash, looking for an innovative solar energy product.<br />

What amount would you be willing to offer for the<br />

business at this point (end of year 12) if you require a<br />

12% per year return on all your investments and, if purchased,<br />

you plan to own the business for 12 additional<br />

years? To help make the decision, assume the current<br />

NCF series continues increasing at $5000 per year for<br />

the years you would own it. Explain your logic for<br />

offering this amount.


CHAPTER 8<br />

Rate of Return<br />

Analysis:<br />

Multiple<br />

Alternatives<br />

LEARNING OUTCOMES<br />

Purpose: Select the best alternative on the basis of incremental rate of return analysis.<br />

SECTION TOPIC LEARNING OUTCOME<br />

8.1 Incremental analysis • State why the ROR method of comparing<br />

alternatives requires an incremental cash flow<br />

analysis.<br />

8.2 Incremental cash flows • Calculate the incremental cash flow series for two<br />

alternatives.<br />

8.3 Meaning of i * • Interpret the meaning of the incremental ROR ( i *)<br />

determined from the incremental cash flow series.<br />

8.4 i * from PW relation • Based on a PW relation, select the better of two<br />

alternatives using incremental ROR analysis or a<br />

breakeven ROR value.<br />

8.5 i * from AW relation • Select the better of two alternatives using<br />

incremental ROR analysis based on an AW<br />

relation.<br />

8.6 More than two alternatives • Select the best from several alternatives using<br />

incremental ROR analysis.<br />

8.7 All-in-one spreadsheet • Use a single spreadsheet to perform PW, AW, ROR,<br />

and incremental ROR analyses for mutually<br />

exclusive and independent alternatives.


T<br />

his chapter presents the methods by which two or more alternatives can be evaluated<br />

using a rate of return (ROR) comparison based on the methods of the previous<br />

chapter. The ROR evaluation, correctly performed, will result in the same selection<br />

as the PW and AW analyses, but the computational procedure is considerably different for ROR<br />

evaluations. The ROR analysis evaluates the increments between two alternatives in pairwise<br />

comparisons. As the cash flow series becomes more complex, spreadsheet functions help speed<br />

computations.<br />

8.1 Why Incremental Analysis Is Necessary<br />

When two or more mutually exclusive alternatives are evaluated, engineering economy can identify<br />

the one alternative that is the best economically. As we have learned, the PW and AW techniques<br />

can be used to do so, and are the recommended methods. Now the procedure using ROR<br />

to identify the best is presented.<br />

Let’s assume that a company uses a MARR of 16% per year, that the company has $90,000<br />

available for investment, and that two alternatives (A and B) are being evaluated. Alternative<br />

A requires an investment of $50,000 and has an internal rate of return i * A of 35% per year. Alternative<br />

B requires $85,000 and has an i * B of 29% per year. Intuitively we may conclude that<br />

the better alternative is the one that has the larger return, A in this case. However, this is not<br />

necessarily so. While A has the higher projected return, its initial investment ($50,000) is much<br />

less than the total money available ($90,000). What happens to the investment capital that is<br />

left over? It is generally assumed that excess funds will be invested at the company’s MARR,<br />

as we learned in previous chapters. Using this assumption, it is possible to determine the consequences<br />

of the two alternative investments. If alternative A is selected, $50,000 will return<br />

35% per year. The $40,000 left over will be invested at the MARR of 16% per year. The rate<br />

of return on the total capital available, then, will be the weighted average. Thus, if alternative<br />

A is selected,<br />

50,000(0.35) 40,000(0.16)<br />

Overall ROR A ———————————— 26.6%<br />

90,000<br />

If alternative B is selected, $85,000 will be invested at 29% per year, and the remaining $5000<br />

will earn 16% per year. Now the weighted average is<br />

85,000(0.29) 5000(0.16)<br />

Overall ROR B ——————————— 28.3%<br />

90,000<br />

These calculations show that even though the i * for alternative A is higher, alternative B presents<br />

the better overall ROR for the $90,000. If either a PW or AW comparison is conducted using the<br />

MARR of 16% per year as i , alternative B will be chosen.<br />

This simple example illustrates a major fact about the rate of return method for ranking and<br />

comparing alternatives:<br />

Under some circumstances, project ROR values do not provide the same ranking of alternatives<br />

as do PW and AW analyses. This situation does not occur if we conduct an incremental ROR<br />

analysis (discussed below).<br />

When independent projects are evaluated, no incremental analysis is necessary between<br />

projects. Each project is evaluated separately from others, and more than one can be selected.<br />

Therefore, the only comparison is with the do-nothing alternative for each project. The project<br />

ROR can be used to accept or reject each one.<br />

Independent project<br />

selection<br />

8.2 Calculation of Incremental Cash Flows for<br />

ROR Analysis<br />

To conduct an incremental ROR analysis, it is necessary to calculate the incremental cash flow<br />

series over the lives of the alternatives. Based upon the equivalence relations (PW and AW), ROR<br />

evaluation makes the equal-service assumption.


204 Chapter 8 Rate of Return Analysis: Multiple Alternatives<br />

TABLE 8–1<br />

Year<br />

0<br />

1<br />

Format for Incremental Cash Flow Tabulation<br />

Cash Flow<br />

Alternative A<br />

(1)<br />

Alternative B<br />

(2)<br />

Incremental<br />

Cash Flow<br />

(3) (2) (1)<br />

Equal-service requirement<br />

The incremental ROR method requires that the equal-service requirement be met. Therefore,<br />

the LCM (least common multiple) of lives for each pairwise comparison must be used. All<br />

the assumptions of equal service present for PW analysis are necessary for the incremental<br />

ROR method.<br />

A format for hand or spreadsheet solutions is helpful (Table 8–1). Equal-life alternatives have<br />

n years of incremental cash flows, while unequal-life alternatives require the LCM of lives for<br />

analysis. At the end of each life cycle, the salvage value and initial investment for the next cycle<br />

must be included for the LCM case.<br />

When a study period is established, only this number of years is used for the evaluation. All<br />

incremental cash flows outside the period are neglected. As we learned earlier, using a study period,<br />

especially one shorter than the life of either alternative, can change the economic decision<br />

from that rendered when the full lives are considered.<br />

Only for the purpose of simplification, use the convention that between two alternatives, the<br />

one with the larger initial investment will be regarded as alternative B . Then, for each year in<br />

Table 8–1,<br />

Incremental cash flow cash flow B cash flow A [8.1]<br />

The initial investment and annual cash flows for each alternative (excluding the salvage value)<br />

are one of the types identified in Chapter 5:<br />

Revenue alternative , where there are both negative and positive cash flows<br />

Cost alternative, where all cash flow estimates are negative<br />

Revenue or cost<br />

alternative<br />

In either case, Equation [8.1] is used to determine the incremental cash flow series with the sign<br />

of each cash flow carefully determined.<br />

EXAMPLE 8.1<br />

A tool and die company in Hanover is considering the purchase of a drill press with fuzzy-logic<br />

software to improve accuracy and reduce tool wear. The company has the opportunity to buy a<br />

slightly used machine for $15,000 or a new one for $21,000. Because the new machine is a<br />

more sophisticated model, its operating cost is expected to be $7000 per year, while the used<br />

machine is expected to require $8200 per year. Each machine is expected to have a 25-year life<br />

with a 5% salvage value. Tabulate the incremental cash flow.<br />

Solution<br />

Incremental cash flow is tabulated in Table 8–2. The subtraction performed is (new – used)<br />

since the new machine has a larger initial cost. The salvage values in year 25 are separated<br />

from ordinary cash flow for clarity. When disbursements are the same for a number of consecutive<br />

years, for hand solution only , it saves time to make a single cash flow listing, as is<br />

done for years 1 to 25. However, remember that several years were combined when performing<br />

the analysis. This approach cannot be used for spreadsheets, when the IRR or NPV function is<br />

used, as each year must be entered separately.


8.2 Calculation of Incremental Cash Flows for ROR Analysis 205<br />

TABLE 8–2 Cash Flow Tabulation for Example 8.1<br />

Year<br />

Cash Flow<br />

Used Press New Press<br />

Incremental<br />

Cash Flow<br />

(New – Used)<br />

0 $15,000 $21,000 $6,000<br />

1–25 8,200 7,000 1,200<br />

25 750 1,050 300<br />

EXAMPLE 8.2<br />

A sole-source vendor can supply a new industrial park with large transformers suitable for<br />

underground utilities and vault-type installation. Type A has an initial cost of $70,000 and<br />

a life of 8 years. Type B has an initial cost of $95,000 and a life expectancy of 12 years.<br />

The annual operating cost for type A is expected to be $9000, while the AOC for type B is<br />

expected to be $7000. If the salvage values are $5000 and $10,000 for type A and type B,<br />

respectively, tabulate the incremental cash flow using their LCM for hand and spreadsheet<br />

solutions.<br />

Solution by Hand<br />

The LCM of 8 and 12 is 24 years. In the incremental cash flow tabulation for 24 years<br />

(Table 8–3), note that the reinvestment and salvage values are shown in years 8 and 16 for type<br />

A and in year 12 for type B.<br />

Solution by Spreadsheet<br />

Figure 8–1 shows the incremental cash flows for the LCM of 24 years. As in the hand tabulation,<br />

reinvestment is made in the last year of each intermediate life cycle. The incremental<br />

values in column D are the result of subtractions of column B from C.<br />

Note that the final row includes a summation check. The total incremental cash flow should<br />

agree in both the column D total and the subtraction C29 B29. Also note that the incremental<br />

values change signs three times, indicating the possibility of multiple i* values, per Descartes’<br />

rule of signs. This possible dilemma is discussed later in the chapter.<br />

TABLE 8–3 Incremental Cash Flow Tabulation, Example 8.2<br />

Cash Flow<br />

Incremental<br />

Cash Flow<br />

Year<br />

Type A<br />

Type B<br />

(B A)<br />

0 $ 70,000 $ 95,000 $25,000<br />

1–7 9,000 –7,000 2,000<br />

70,000<br />

8 9,000 –7,000 67,000<br />

5,000<br />

9–11 9,000 –7,000 2,000<br />

–95,000<br />

12 9,000 –7,000 83,000<br />

10,000<br />

13–15 9,000 –7,000 2,000<br />

70,000<br />

16 9,000 –7,000 67,000<br />

5,000<br />

17–23 9,000 –7,000 2,000<br />

24<br />

9,000 –7,000<br />

5,000 10,000<br />

7,000<br />

$411,000 $338,000 $73,000


206 Chapter 8 Rate of Return Analysis: Multiple Alternatives<br />

Figure 8–1<br />

Spreadsheet computation<br />

of incremental<br />

cash flows for<br />

unequal-life alternatives,<br />

Example 8.2.<br />

Starting new life cycle for A<br />

initial cost AOC salvage<br />

70,000 9,000 5,000<br />

Check on summations<br />

Incremental column should<br />

equal difference of columns<br />

8.3 Interpretation of Rate of Return<br />

on the Extra Investment<br />

The incremental cash flows in year 0 of Tables 8–2 and 8–3 reflect the extra investment or cost<br />

required if the alternative with the larger first cost is selected. This is important in an incremental<br />

ROR analysis in order to determine the ROR earned on the extra funds expended for the largerinvestment<br />

alternative. If the incremental cash flows of the larger investment don’t justify it, we<br />

must select the cheaper one. In Example 8.1 the new drill press requires an extra investment of<br />

$6000 (Table 8–2). If the new machine is purchased, there will be a “savings” of $1200 per year<br />

for 25 years, plus an extra $300 in year 25. The decision to buy the used or new machine can<br />

be made on the basis of the profitability of investing the extra $6000 in the new machine. If the<br />

equivalent worth of the savings is greater than the equivalent worth of the extra investment at the<br />

MARR, the extra investment should be made (i.e., the larger first-cost proposal should<br />

be accepted). On the other hand, if the extra investment is not justified by the savings, select the<br />

lower-investment proposal.<br />

It is important to recognize that the rationale for making the selection decision is the same as<br />

if only one alternative were under consideration, that alternative being the one represented by the<br />

incremental cash flow series. When viewed in this manner, it is obvious that unless this investment<br />

yields a rate of return equal to or greater than the MARR, the extra investment should not<br />

be made. As further clarification of this extra investment rationale, consider the following: The<br />

rate of return attainable through the incremental cash flow is an alternative to investing at the<br />

MARR. Section 8.1 states that any excess funds not invested in the alternative are assumed to<br />

be invested at the MARR. The conclusion is clear:<br />

ME alternative selection<br />

If the rate of return available through the incremental cash flow equals or exceeds the MARR,<br />

the alternative associated with the extra investment should be selected.


8.4 Rate of Return Evaluation Using PW: Incremental and Breakeven 207<br />

Not only must the return on the extra investment meet or exceed the MARR, but also the return<br />

on the investment that is common to both alternatives must meet or exceed the MARR. Accordingly,<br />

prior to performing an incremental ROR analysis, it is advisable to determine the internal rate of<br />

return i * for each alternative. This can be done only for revenue alternatives, because cost alternatives<br />

have only cost (negative) cash flows and no i * can be determined. The guideline is as follows:<br />

For multiple revenue alternatives, calculate the internal rate of return i * for each alternative,<br />

and eliminate all alternatives that have an i * MARR. Compare the remaining alternatives<br />

incrementally.<br />

As an illustration, if the MARR 15% and two alternatives have i * values of 12% and 21%,<br />

the 12% alternative can be eliminated from further consideration. With only two alternatives, it<br />

is obvious that the second one is selected. If both alternatives have i * MARR, no alternative is<br />

justified and the do-nothing alternative is the best economically. When three or more alternatives<br />

are evaluated, it is usually worthwhile, but not required, to calculate i * for each alternative for<br />

preliminary screening. Alternatives that cannot meet the MARR may be eliminated from further<br />

evaluation using this option. This option is especially useful when performing the analysis by<br />

spreadsheet. The IRR function applied to each alternative’s cash flow estimates can quickly indicate<br />

unacceptable alternatives, as demonstrated in Section 8.6.<br />

When independent projects are evaluated, there is no comparison on the extra investment.<br />

The ROR value is used to accept all projects with i * MARR, assuming there is no budget<br />

limitation. For example, assume MARR 10%, and three independent projects are available<br />

with ROR values of<br />

Independent project<br />

selection<br />

i A<br />

* 12%<br />

i * B 9% i C<br />

* 23%<br />

Projects A and C are selected, but B is not because i * B MARR.<br />

8.4 Rate of Return Evaluation Using PW:<br />

Incremental and Breakeven<br />

In this section we discuss the primary approach to making mutually exclusive alternative selections<br />

by the incremental ROR method. A PW-based relation is developed for the incremental cash<br />

flows and set equal to zero. Use hand solution or spreadsheet functions to find i * B–A , the internal<br />

ROR for the series. Placing (delta) before i * B–A distinguishes it from the overall ROR values i A<br />

*<br />

and i B<br />

*. ( i * may replace i * B–A when only two alternatives are present.)<br />

Since incremental ROR requires equal-service comparison, the LCM of lives must be used in<br />

the PW formulation. Because of the reinvestment requirement for PW analysis for different-life<br />

assets, the incremental cash flow series may contain several sign changes, indicating multiple i *<br />

values. Though incorrect, this indication is usually neglected in actual practice. The correct<br />

approach is to follow one of the techniques of Section 7.5. This means that the single external<br />

ROR ( i or i ) for the incremental cash flow series is determined.<br />

These three elements— incremental cash flow series, LCM, and multiple roots —are the primary<br />

reasons that the ROR method is often applied incorrectly in engineering economy analyses of<br />

multiple alternatives. As stated earlier, it is always possible, and generally advisable, to use a PW or<br />

AW analysis at an established MARR in lieu of the ROR method when multiple rates are indicated.<br />

The complete procedure for hand or spreadsheet solution for an incremental ROR analysis<br />

for two alternatives is as follows:<br />

1. Order the alternatives by initial investment or cost, starting with the smaller one, called<br />

A. The one with the larger initial investment is in the column labeled B in Table 8–1.<br />

2. Develop the cash flow and incremental cash flow series using the LCM of years, assuming<br />

reinvestment in alternatives.<br />

3. Draw an incremental cash flow diagram, if needed.


208 Chapter 8 Rate of Return Analysis: Multiple Alternatives<br />

ME alternative<br />

selection<br />

4. Count the number of sign changes in the incremental cash flow series to determine if<br />

multiple rates of return may be present. If necessary, use Norstrom’s criterion to determine<br />

if a single positive root exists.<br />

5. Set up the PW 0 equation and determine i* B–A .<br />

6. Select the economically better alternative as follows:<br />

If i* BA MARR, select alternative A.<br />

If i* BA MARR, the extra investment is justified; select alternative B.<br />

If i* is exactly equal to or very near the MARR, noneconomic considerations help in<br />

the selection of the “better” alternative.<br />

EXAMPLE 8.3<br />

In step 5, if trial and error by hand is used, time may be saved if the i * BA value is bracketed,<br />

rather than approximated by a point value using linear interpolation, provided that a single ROR<br />

value is not needed. For example, if the MARR is 15% per year and you have established that<br />

i * BA is in the 15% to 20% range, an exact value is not necessary to accept B since you already<br />

know that i * BA MARR.<br />

The IRR function on a spreadsheet will normally determine one i * value. Multiple guess<br />

values can be input to find multiple roots in the range 100% to for a nonconventional series,<br />

as illustrated in Example 7.4. If this is not the case, to be correct, the indication of multiple roots<br />

in step 4 requires that one of the techniques of Section 7.5 be applied to find an EROR.<br />

As Ford Motor Company retools an old truck assembly plant in Michigan to produce a fuelefficient<br />

economy model, the Ford Focus. Ford and its suppliers are seeking additional sources for<br />

light, long-life transmissions. Automatic transmission component manufacturers use highly finished<br />

dies for precision forming of internal gears and other moving parts. Two United States–<br />

based vendors make the required dies. Use the per unit estimates below and a MARR of 12% per<br />

year to select the more economical vendor bid. Show both hand and spreadsheet solutions.<br />

Initial cost, $ 8,000 13,000<br />

Annual costs, $ per year 3,500 1,600<br />

Salvage value, $ 0 2,000<br />

Life, years 10 5<br />

Solution by Hand<br />

These are cost alternatives, since all cash flows are costs. Use the procedure described above to<br />

determine i * B–A .<br />

1. Alternatives A and B are correctly ordered with the higher first-cost alternative in column 2<br />

of Table 8–4.<br />

2. The cash flows for the LCM of 10 years are tabulated.<br />

TABLE 8–4 Incremental Cash Flow Tabulation, Example 8.3<br />

Year<br />

Cash Flow A<br />

(1)<br />

A<br />

Cash Flow B<br />

(2)<br />

B<br />

Incremental<br />

Cash Flow<br />

(3) (2) (1)<br />

0 $ 8,000 $13,000 $ 5,000<br />

1–5 3,500 1,600 1,900<br />

5 —<br />

2,000<br />

13,000<br />

11,000<br />

6–10 3,500 1,600 1,900<br />

10 — 2,000 2,000<br />

$43,000 $38,000 $ 5,000


8.4 Rate of Return Evaluation Using PW: Incremental and Breakeven 209<br />

$1900<br />

$2000<br />

Figure 8–2<br />

Diagram of incremental cash<br />

flows, Example 8.3 .<br />

0 1 2 3 4 5 6 7 8 9<br />

10<br />

$5000<br />

$11,000<br />

3. The incremental cash flow diagram is shown in Figure 8–2 .<br />

4. There are three sign changes in the incremental cash flow series, indicating as many as<br />

three roots. There are also three sign changes in the cumulative incremental series, which<br />

starts negatively at S 0 $5000 and continues to S 10 $5000, indicating that more<br />

than one positive root may exist.<br />

5. The rate of return equation based on the PW of incremental cash flows is<br />

0 5000 1900( P A , i *,10) 11,000( P F , i *,5) 2000( P F , i *,10) [8.2]<br />

In order to resolve any multiple-root problem, we can assume that the investment rate i i in the<br />

ROIC technique will equal the i* found by trial and error. Solution of Equation [8.2] for the<br />

first root discovered results in i * between 12% and 15%. By interpolation i * 12.65%.<br />

6. Since the rate of return of 12.65% on the extra investment is greater than the 12% MARR,<br />

the higher-cost vendor B is selected.<br />

Comment<br />

In step 4, the presence of up to three i * values is indicated. The preceding analysis finds one of<br />

the roots at 12.65%. When we state that the incremental ROR is 12.65%, we assume that any<br />

positive net cash flows are reinvested at 12.65%. If this is not a reasonable assumption, the<br />

ROIC or modified ROR technique (Section 7.5) must be applied to find a different single i '<br />

or i '' to compare with MARR 12%.<br />

The other two roots are very large positive and negative numbers, as the IRR function of<br />

Excel reveals. So they are not useful to the analysis.<br />

Solution by Spreadsheet<br />

Steps 1 through 4 are the same as above.<br />

5. Figure 8–3 includes the same incremental net cash flows from Table 8–4 calculated in<br />

column D. Cell D15 displays the i * value of 12.65% using the IRR function.<br />

Figure 8–3<br />

Spreadsheet solution using LCM of<br />

lives and IRR function, Example 8.3 .<br />

NPV(12%,D5:D14) + D4


210 Chapter 8 Rate of Return Analysis: Multiple Alternatives<br />

6. Since the rate of return on the extra investment is greater than the 12% MARR, the highercost<br />

vendor B is selected.<br />

Comment<br />

Once the spreadsheet is set up, there are a wide variety of analyses that can be performed. For<br />

example, row 17 uses the NPV function to verify that the present worth is positive at MARR12%.<br />

Charts such as PW versus i and PW versus i help graphically interpret the situation.<br />

The rate of return determined for the incremental cash flow series or the actual cash flows can<br />

be interpreted as a breakeven rate of return value.<br />

The breakeven rate of return is the incremental i* value, i*, at which the PW (or AW)<br />

value of the incremental cash fl ows is exactly zero. Equivalently, the breakeven ROR is the i<br />

value, i*, at which the PW (or AW) values of two alternatives’ actual cash flows are exactly<br />

equal to each other.<br />

Breakeven ROR<br />

If the incremental cash flow ROR ( i *) is greater than the MARR, the larger-investment<br />

alternative is selected. For example, if the PW versus i graph for the incremental cash flows in<br />

Table 8–4 (and spreadsheet Figure 8–3 ) is plotted for various interest rates, the graph shown in<br />

Figure 8–4 is obtained. It shows the i * breakeven at 12.65%. The conclusions are that<br />

• For MARR 12.65%, the extra investment for B is justified.<br />

• For MARR 12.65%, the opposite is true—the extra investment in B should not be made,<br />

and vendor A is selected.<br />

• If MARR is exactly 12.65%, the alternatives are equally attractive.<br />

Figure 8–5 , which is a breakeven graph of PW versus i for the cash flows (not incremental) of<br />

each alternative in Example 8.3 , provides the same results. Since all net cash flows are negative<br />

1800<br />

1600<br />

1400<br />

1200<br />

For MARR<br />

in this range,<br />

select B<br />

Breakeven i<br />

is 12.65%<br />

For MARR<br />

in this range,<br />

select A<br />

PW of incremental cash flows, $<br />

1000<br />

800<br />

600<br />

400<br />

200<br />

0<br />

– 200<br />

6 7 8 9<br />

10 11 12<br />

14 15<br />

16 i%<br />

– 400<br />

– 600<br />

– 800<br />

Vendor B<br />

Vendor A<br />

Figure 8–4<br />

Plot of present worth of incremental cash flows for Example 8.3 at various<br />

i values.


8.4 Rate of Return Evaluation Using PW: Incremental and Breakeven 211<br />

PW of alternative cash flows, $<br />

25,000<br />

26,000<br />

27,000<br />

28,000<br />

29,000<br />

30,000<br />

Interest rate, % Figure 8–5<br />

10 11 12 13 14 15 16<br />

Breakeven graph of<br />

Example 8.3 cash flows<br />

(not incremental).<br />

B<br />

A<br />

A<br />

B<br />

(cost alternatives), the PW values are negative. Now, the same conclusions are reached using the<br />

following logic:<br />

• If MARR 12.65%, select B since its PW of cost cash flows is smaller (numerically larger).<br />

• If MARR 12.65%, select A since its PW of costs is smaller.<br />

• If MARR is exactly 12.65%, either alternative is equally attractive.<br />

Example 8.4 illustrates incremental ROR evaluation and breakeven rate of return graphs for<br />

revenue alternatives. More of breakeven analysis is covered in Chapter 13.<br />

EXAMPLE 8.4<br />

New filtration systems for commercial airliners are available that use an electric field to remove up<br />

to 99.9% of infectious diseases and pollutants from aircraft air. This is vitally important, as many<br />

of the flu germs, viruses, and other contagious diseases are transmitted through the systems that<br />

recirculate aircraft air many times per hour. Investments in the new filtration equipment can cost<br />

from $100,000 to $150,000 per aircraft, but savings in fuel, customer complaints, legal actions,<br />

etc., can also be sizable. Use the estimates below (in $100 units) from two suppliers provided to an<br />

international carrier to do the following, using a spreadsheet and an MARR of 15% per year.<br />

• Plot two graphs: PW versus i values for both alternatives’ cash flows and PW versus i<br />

values for incremental cash flows.<br />

• Estimate the breakeven ROR values from both graphs, and use this estimate to select one<br />

alternative.<br />

Air Cleanser<br />

(Filter 1)<br />

Purely Heaven<br />

(Filter 2)<br />

Initial cost per aircraft, $ 1000 1500<br />

Estimated savings, $ per year 375 700 in year 1, decreasing by<br />

100 per year thereafter<br />

Estimated life, years 5 5<br />

Solution by Spreadsheet<br />

Refer to Figure 8–6 as the solution is explained. For information, row 10 shows the ROR values<br />

calculated using the IRR function for filter 1 and filter 2 cash flows and the incremental cash<br />

flow series (filter 2 filter 1).<br />

The cash flow sign tests for each filter indicate no multiple rates. The incremental cash flow<br />

sign test does not indicate the presence of a unique positive root; however, the second rate is an<br />

extremely large and useless value. The PW values of filter 1 and filter 2 cash flows are plotted<br />

on the right side for i values ranging from 0% to 60%. Since the PW curves cross each other at<br />

approximately i * 17%, the rate does exceed the MARR of 15%. The higher-cost filter 2<br />

(Purely Heaven) is selected.<br />

The PW curve for the incremental cash flows series (column G) is plotted at the bottom left<br />

of Figure 8–6 . As expected, the curve crosses the PW 0 line at approximately 17%, indicating<br />

the same economic conclusion of filter 2.


MARR<br />

Breakeven<br />

Incremental ROR 17%<br />

Figure 8–6<br />

PW versus i graph and PW versus incremental i graph, Example 8.4 .<br />

Breakeven ROR 17%<br />

MARR<br />

Filter 1 ROR 25%<br />

Filter 2 ROR 23%<br />

212


8.5 Rate of Return Evaluation Using AW 213<br />

Figure 8–6 provides an excellent opportunity to see why the ROR method can result in selecting<br />

the wrong alternative when only i * values are used to select between two alternatives. This is<br />

sometimes called the ranking inconsistency problem of the ROR method. The inconsistency<br />

occurs when the MARR is set less than the breakeven rate between two revenue alternatives.<br />

Since the MARR is established based on conditions of the economy and market, MARR is established<br />

external to any particular alternative evaluation. In Figure 8–6 the incremental breakeven<br />

rate is 16.89%, and the MARR is 15%. The MARR is lower than breakeven; therefore, the incremental<br />

ROR analysis results in correctly selecting filter 2. But if only the i * values were used,<br />

filter 1 would be wrongly chosen, because its i * exceeds that of filter 2 (25.41% > 23.57%). This<br />

error occurs because the rate of return method assumes reinvestment at the alternative’s ROR<br />

value, while PW and AW analyses use the MARR as the reinvestment rate. The conclusion is<br />

simple:<br />

If the ROR method is used to evaluate two or more alternatives, use the incremental cash flows<br />

and i* to make the decision between alternatives.<br />

8.5 Rate of Return Evaluation Using AW<br />

Comparing alternatives by the ROR method (correctly performed) always leads to the same selection<br />

as PW and AW analyses, whether the ROR is determined using a PW-based or an AW-based<br />

relation. However, for the AW-based technique, there are two equivalent ways to perform the<br />

evaluation: (1) using the incremental cash flows over the LCM of alternative lives, just as for the<br />

PW-based relation (Section 8.4), or (2) finding the AW for each alternative’s actual cash flows and<br />

setting the difference of the two equal to zero to find the i * value. There is no difference between<br />

the two approaches if the alternative lives are equal. Both methods are summarized here.<br />

Since the ROR method requires comparison for equal service, the incremental cash flows<br />

must be evaluated over the LCM of lives. There may be no real computational advantage to<br />

using AW, as was found in Chapter 6. The same six-step procedure of the previous section (for<br />

PW-based calculation) is used, except in step 5 the AW-based relation is developed.<br />

Equal-service<br />

requirement<br />

The second AW-based method mentioned above takes advantage of the AW technique’s assum p-<br />

tion that the equivalent AW value is the same for each year of the first and all succeeding life<br />

cycles. Whether the lives are equal or unequal, set up the AW relation for the cash fl ows of each<br />

alternative, form the relation below, and solve for i *.<br />

0 AW B AW A [8.3]<br />

For both methods, all equivalent values are on an AW basis, so the i * that results from Equation<br />

[8.3] is the same as the i * found using the first approach. Example 8.5 illustrates ROR<br />

analysis using AW-based relations for unequal lives.<br />

EXAMPLE 8.5<br />

Compare the alternatives of vendors A and B for Ford in Example 8.3 , using an AW-based<br />

incremental ROR method and the same MARR of 12% per year.<br />

Solution<br />

For reference, the PW-based ROR relation, Equation [8.2], for the incremental cash flow in<br />

Example 8.3 shows that vendor B should be selected with i * 12.65%.<br />

For the AW relation, there are two equivalent solution approaches. Write an AW-based<br />

relation on the incremental cash flow series over the LCM of 10 years, or write Equation [8.3]<br />

for the two actual cash flow series over one life cycle of each alternative.<br />

For the incremental method, the AW equation is<br />

0 5000( A P , i *,10) 11,000( P F , i *,5)( A P , i *,10) 2000( A F , i *,10) 1900


214 Chapter 8 Rate of Return Analysis: Multiple Alternatives<br />

It is easy to enter the incremental cash flows onto a spreadsheet, as in Figure 8–3 , column D,<br />

and use the IRR(D4:D14) function to display i * 12.65%.<br />

For the second method, the ROR is found using the actual cash flows and the respective<br />

lives of 10 years for A and 5 years for B.<br />

Now develop 0 AW B AW A .<br />

AW A 8000( A P , i ,10) 3500<br />

AW B 13,000( A P , i ,5) 2000( A F , i ,5) 1600<br />

0 13,000( A P , i *,5) 2000( A F , i *,5) 8000( A P , i *,10) 1900<br />

Solution again yields i * 12.65%.<br />

Comment<br />

It is very important to remember that when an incremental ROR analysis using an AW-based<br />

equation is made on the incremental cash fl ows, the LCM must be used.<br />

8.6 Incremental ROR Analysis of Multiple Alternatives<br />

This section treats selection from multiple alternatives that are mutually exclusive, using the incremental<br />

ROR method. Acceptance of one alternative automatically precludes acceptance of<br />

any others. The analysis is based upon PW (or AW) relations for incremental cash flows between<br />

two alternatives at a time.<br />

When the incremental ROR method is applied, the entire investment must return at least the<br />

MARR. When the i * values on several alternatives exceed the MARR, incremental ROR evaluation<br />

is required. (For revenue alternatives, if not even one i * MARR, the do-nothing alternative is selected.)<br />

For all alternatives (revenue or cost), the incremental investment must be separately justified.<br />

If the return on the extra investment equals or exceeds the MARR, then the extra investment should<br />

be made in order to maximize the total return on the money available, as discussed in Section 8.1.<br />

For ROR analysis of multiple, mutually exclusive alternatives, the following criteria are used.<br />

ME alternative<br />

selection<br />

Select the one alternative<br />

That requires the largest investment, and<br />

Indicates that the extra investment over another acceptable alternative is justified.<br />

An important rule to apply when evaluating multiple alternatives by the incremental ROR method<br />

is that an alternative should never be compared with one for which the incremental investment is<br />

not justifi ed.<br />

The incremental ROR evaluation procedure for multiple, equal-life alternatives is summarized<br />

below. Step 2 applies only to revenue alternatives, because the first alternative is compared<br />

to DN only when revenue cash flows are estimated. The terms defender and challenger are dynamic<br />

in that they refer, respectively, to the alternative that is currently selected (the defender)<br />

and the one that is challenging it for acceptance based on i *. In every pairwise evaluation, there<br />

is one of each. The steps for solution by hand or by spreadsheet are as follows:<br />

1. Order the alternatives from smallest to largest initial investment. Record the annual cash<br />

flow estimates for each equal-life alternative.<br />

2. Revenue alternatives only: Calculate i * for the first alternative. In effect, this makes DN the<br />

defender and the first alternative the challenger. If i * MARR, eliminate the alternative and<br />

go to the next one. Repeat this until i * MARR for the first time, and define that alternative<br />

as the defender. The next alternative is now the challenger. Go to step 3. ( Note: This is where<br />

solution by spreadsheet can be a quick assist. Calculate the i * for all alternatives first using<br />

the IRR function, and select as the defender the first one for which i * MARR. Label it the<br />

defender and go to step 3.)<br />

3. Determine the incremental cash flow between the challenger and defender, using the relation<br />

Incremental cash flow challenger cash flow defender cash flow<br />

Set up the ROR relation.


8.6 Incremental ROR Analysis of Multiple Alternatives 215<br />

4. Calculate i * for the incremental cash flow series using a PW- or AW-based equation. (PW<br />

is most commonly used.)<br />

5. If i * MARR, the challenger becomes the defender and the previous defender is eliminated.<br />

Conversely, if i * MARR, the challenger is removed, and the defender remains<br />

against the next challenger.<br />

6. Repeat steps 3 to 5 until only one alternative remains. It is the selected one.<br />

Note that only two alternatives are compared at any one time. It is vital that the correct alternatives<br />

be compared, or the wrong alternative may be selected.<br />

EXAMPLE 8.6<br />

Caterpillar Corporation wants to build a spare parts storage facility in the Phoenix, Arizona,<br />

vicinity. A plant engineer has identified four different location options. The initial cost of earthwork<br />

and prefab building and the annual net cash flow estimates are detailed in Table 8–5. The<br />

annual net cash flow series vary due to differences in maintenance, labor costs, transportation<br />

charges, etc. If the MARR is 10%, use incremental ROR analysis to select the one economically<br />

best location.<br />

TABLE 8–5 Estimates for Four Alternative Building Locations, Example 8.6<br />

A B C D<br />

Initial cost, $ 200,000 275,000 190,000 350,000<br />

Annual cash flow, $ per year 22,000 35,000 19,500 42,000<br />

Life, years 30 30 30 30<br />

Solution<br />

All sites have a 30-year life, and they are revenue alternatives. The procedure outlined above<br />

is applied.<br />

1. The alternatives are ordered by increasing initial cost in Table 8–6.<br />

2. Compare location C with the do-nothing alternative. The ROR relation includes only the<br />

P A factor.<br />

0 190,000 19,500( P A , i *,30)<br />

Table 8–6, column 1, presents the calculated ( P A , i *,30) factor value of 9.7436 and<br />

i c<br />

* 9.63%. Since 9.63% 10%, location C is eliminated. Now the comparison is A to<br />

DN, and column 2 shows that i A<br />

* 10.49%. This eliminates the do-nothing alternative;<br />

the defender is now A and the challenger is B.<br />

TABLE 8–6 Computation of Incremental Rate of Return for Four Alternatives,<br />

Example 8.6<br />

C<br />

(1)<br />

Initial cost, $ 190,000 200,000 275,000 350,000<br />

Cash flow, $ per year 19,500 22,000 35,000 42,000<br />

Alternatives compared C to DN A to DN B to A D to B<br />

Incremental cost, $ 190,000 200,000 75,000 75,000<br />

Incremental cash flow, $ 19,500 22,000 13,000 7,000<br />

Calculated ( P A , i *,30) 9.7436 9.0909 5.7692 10.7143<br />

i *,% 9.63 10.49 17.28 8.55<br />

Increment justified? No Yes Yes No<br />

Alternative selected DN A B B<br />

A<br />

(2)<br />

B<br />

(3)<br />

D<br />

(4)


216 Chapter 8 Rate of Return Analysis: Multiple Alternatives<br />

3. The incremental cash flow series, column 3, and i * for B-to-A comparison are determined<br />

from<br />

0 275,000 (200,000) (35,000 22,000)( P A , i *,30)<br />

75,000 13,000( P A , i *,30)<br />

4. From the interest tables, look up the P A factor at the MARR, which is ( P A ,10%,30) <br />

9.4269. Now, any P A value greater than 9.4269 indicates that the i * will be less than<br />

10% and is unacceptable. The P A factor is 5.7692, so B is acceptable. For reference<br />

purposes, i * 17.28%.<br />

5. Alternative B is justified incrementally (new defender), thereby eliminating A.<br />

6. Comparison D-to-B (steps 3 and 4) results in the PW relation 0 75,000 <br />

7000( P A , i *,30) and a P A value of 10.7143 ( i * 8.55%). Location D is eliminated,<br />

and only alternative B remains; it is selected.<br />

Comment<br />

An alternative must always be incrementally compared with an acceptable alternative, and the<br />

do-nothing alternative can end up being the only acceptable one. Since C was not justified in<br />

this example, location A was not compared with C. Thus, if the B-to-A comparison had not<br />

indicated that B was incrementally justified, then the D-to-A comparison would be correct<br />

instead of D-to-B.<br />

To demonstrate how important it is to apply the ROR method correctly, consider the following.<br />

If the i * of each alternative is computed initially, the results by ordered alternatives are<br />

Location C A B D<br />

i *, % 9.63 10.49 12.35 11.56<br />

Now apply only the first criterion stated earlier; that is, make the largest investment that has a<br />

MARR of 10% or more. Location D is selected. But, as shown above, this is the wrong selection,<br />

because the extra investment of $75,000 over location B will not earn the MARR. In fact,<br />

it will earn only 8.55%. This is another example of the ranking inconsistency problem of the<br />

ROR method mentioned in Section 8.4.<br />

For cost alternatives, the incremental cash flow is the difference between costs for two alternatives.<br />

There is no do-nothing alternative and no step 2 in the solution procedure. Therefore, the<br />

lowest-investment alternative is the initial defender against the next-lowest investment (c hallenger).<br />

This procedure is illustrated in Example 8.7 using a spreadsheet solution.<br />

EXAMPLE 8.7<br />

The complete failure of an offshore platform and the resulting spillage of up to 800,000 to<br />

1,000,000 gallons per day into the Gulf of Mexico in the spring of 2010 have made major oil producers<br />

and transporters very conscious of the harm done to people’s livelihood and all forms of<br />

aquatic life by spills of this magnitude. To address the specific danger to birds that are shoreline<br />

feeders and dwellers, environmental engineers from several international petroleum corporations<br />

and transport companies—Exxon-Mobil, BP, Shell, and some transporters for OPEC producers—<br />

have developed a plan to strategically locate throughout the world newly developed equipment<br />

that is substantially more effective than manual procedures in cleaning crude oil residue from bird<br />

feathers. The Sierra Club, Greenpeace, and other international environmental interest groups are<br />

in favor of the initiative. Alternative machines from manufacturers in Asia, America, Europe, and<br />

Africa are available with the cost estimates in Table 8–7. Annual cost estimates are expected to be<br />

high to ensure readiness at any time. The company representatives have agreed to use the average<br />

of the corporate MARR values, which results in MARR 13.5%. Use a spreadsheet and incremental<br />

ROR analysis to determine which manufacturer offers the best economic choice.


8.6 Incremental ROR Analysis of Multiple Alternatives 217<br />

TABLE 8–7 Costs for Four Alternative Machines, Example 8.7<br />

Machine 1 Machine 2 Machine 3 Machine 4<br />

First cost, $ 5,000 6,500 10,000 15,000<br />

Annual operating cost, $ 3,500 3,200 3,000 1,400<br />

Salvage value, $ 500 900 700 1,000<br />

Life, years 8 8 8 8<br />

Solution by Spreadsheet<br />

Follow the procedure for incremental ROR analysis. The spreadsheet in Figure 8–7 contains<br />

the complete solution.<br />

1. The alternatives are already ordered by increasing first costs.<br />

2. These are cost alternatives, so there is no comparison to DN, since i * values cannot be<br />

calculated.<br />

3. Machine 2 is the first challenger to machine 1; the incremental cash flows for the 2-to-1<br />

comparison are in column D.<br />

4. The 2-to-1 comparison results in i * 14.57% by applying the IRR function.<br />

5. This return exceeds MARR 13.5%, so machine 2 is the new defender (cell D17).<br />

The comparison continues for 3-to-2 in column E, where the return is negative at i * <br />

18.77%; machine 2 is retained as the defender. Finally the 4-to-2 comparison has an incremental<br />

ROR of 13.60%, which is slightly larger than MARR 13.5%. The conclusion is to<br />

purchase machine 4 because the extra investment is (marginally) justified.<br />

IRR(D6:D14)<br />

Figure 8–7<br />

Spreadsheet solution to select from multiple cost alternatives, Example 8.7 .<br />

Comment<br />

As mentioned earlier, it is not possible to generate a PW versus i graph for each cost alternative<br />

because all cash flows are negative. However, it is possible to generate PW versus i graphs<br />

for the incremental series in the same fashion as we have done previously. The curves will<br />

cross the PW 0 line at the i * values determined by the IRR functions.<br />

Selection from multiple, mutually exclusive alternatives with unequal lives using i * values<br />

requires that the incremental cash flows be evaluated over the LCM of the two alternatives being<br />

compared. This is another application of the principle of equal-service comparison. The spreadsheet<br />

application in the next section illustrates the computations.<br />

It is always possible to rely on PW or AW analysis of the incremental cash flows at the MARR<br />

to make the selection. In other words, don’t find i * for each pairwise comparison; find PW or


218 Chapter 8 Rate of Return Analysis: Multiple Alternatives<br />

AW at the MARR instead. However, it is still necessary to make the comparison over the LCM<br />

number of years for an incremental analysis to be performed correctly.<br />

8.7 All-in-One Spreadsheet Analysis (Optional)<br />

For professors and students who like to pack a spreadsheet, Example 8.8 combines many of the<br />

economic analysis techniques we have learned so far—(internal) ROR analysis, incremental<br />

ROR analysis, PW analysis, and AW analysis. Now that the IRR, NPV, and PV functions are<br />

mastered, it is possible to perform a wide variety of evaluations for multiple alternatives on a<br />

single spreadsheet. No cell tags are provided in this example. A nonconventional cash flow series<br />

for which multiple ROR values may be found, and selection from both mutually exclusive alternatives<br />

and independent projects, are included in this example.<br />

EXAMPLE 8.8<br />

In-flight texting, phone, and Internet connections provided at airline passenger seats are an<br />

expected service by many customers. Singapore Airlines knows it will have to replace 15,000<br />

to 24,000 units in the next few years on its Boeing 757, 777, and A300 aircraft. Four optional<br />

data handling features that build upon one another are available from the manufacturer, but at<br />

an added cost per unit. Besides costing more, the higher-end options (e.g., satellite-based plugin<br />

video service) are estimated to have longer lives before the next replacement is forced by<br />

new, advanced features expected by flyers. All four options are expected to boost annual revenues<br />

by varying amounts. Figure 8–8 spreadsheet rows 2 through 6 include all the estimates<br />

for the four options.<br />

(a) Using MARR 15%, perform ROR, PW, and AW evaluations to select the one level of<br />

options that is the most promising economically.<br />

(b) If more than one level of options can be selected, consider the four that are described as<br />

independent projects. If no budget limitations are considered at this time, which options<br />

are acceptable if the MARR is increased to 20% when more than one option may be<br />

implemented?<br />

Figure 8–8<br />

Spreadsheet analysis using ROR, PW, and AW methods for unequal-life, revenue alternatives, Example 8.8 .


Chapter Summary 219<br />

Solution by Spreadsheet<br />

(a) The spreadsheet ( Figure 8–8 ) is divided into six sections:<br />

Section 1 (rows 1, 2): MARR value and the alternative names (A through D) are in increasing<br />

order of initial cost.<br />

Section 2 (rows 3 to 6): Per-unit net cash flow estimates for each alternative. These are<br />

revenue alternatives with unequal lives.<br />

Section 3 (rows 7 to 20): Actual and incremental cash flows are displayed here.<br />

Section 4 (rows 21, 22): Because these are all revenue alternatives, i * values are determined<br />

by the IRR function. If an alternative passes the MARR test ( i * 15%), it is<br />

retained and a column is added to the right of its actual cash flows so the incremental<br />

cash flows can be determined. Columns F and H were inserted to make space for the<br />

incremental evaluations. Alternative A does not pass the i * test.<br />

Section 5 (rows 23 to 25): The IRR functions display the i * values in columns F and H.<br />

Comparison of C to B takes place over the LCM of 12 years. Since i * CB 19.42% <br />

15%, eliminate B; alternative C is the new defender and D is the next challenger. The<br />

final comparison of D to C over 12 years results in i * DC 11.23% 15%, so D is<br />

eliminated. Alternative C is the chosen one.<br />

Section 6 (rows 26 to 28): These include the AW and PW analyses. The AW value over the life<br />

of each alternative is calculated using the PMT function at the MARR with an embedded<br />

NPV function. Also, the PW value is determined from the AW value for 12 years using the<br />

PV function. For both measures, alternative C has the numerically largest value, as expected.<br />

Conclusion: All methods result in the same, correct choice of alternative C.<br />

(b) Since each option is independent of the others, and there is no budget limitation at this<br />

time, each i * value in row 21 of Figure 8–8 is compared to MARR 20%. This is a comparison<br />

of each option with the do-nothing alternative. Of the four, options B and C have<br />

i * 20%. They are acceptable; the other two are not.<br />

Comment<br />

In part ( a ), we should have applied the two multiple-root sign tests to the incremental cash flow<br />

series for the C-to-B comparison. The series itself has three sign changes, and the cumulative<br />

cash flow series starts negatively and also has three sign changes. Therefore, up to three realnumber<br />

roots may exist. The IRR function is applied in cell F23 to obtain i * CB 19.42%<br />

without using a supplemental (Section 7.5) procedure. This means that the investment assumption<br />

of 19.42% for positive cash flows is a reasonable one. If the MARR 15%, or some other<br />

earning rate were more appropriate, the ROIC procedure could be applied to determine a single<br />

rate, which would be different from 19.42%. Depending upon the investment rate chosen,<br />

alternative C may or may not be incrementally justified against B. Here, the assumption is<br />

made that the i * value is reasonable, so C is justified.<br />

CHAPTER SUMMARY<br />

Just as present worth and annual worth methods find the best alternative from among several,<br />

incremental rate of return calculations can be used for the same purpose. In using the ROR technique,<br />

it is necessary to consider the incremental cash flows when selecting between mutually<br />

exclusive alternatives. The incremental investment evaluation is conducted between only two<br />

alternatives at a time, beginning with the lowest initial investment alternative. Once an alternative<br />

has been eliminated, it is not considered further.<br />

Rate of return values have a natural appeal to management, but the ROR analysis is often<br />

more difficult to set up and complete than the PW or AW analysis using an established MARR.<br />

Care must be taken to perform a ROR analysis correctly on the incremental cash flows; otherwise<br />

it may give incorrect results.<br />

If there is no budget limitation when independent projects are evaluated, the ROR value of<br />

each project is compared to the MARR. Any number, or none, of the projects can be accepted.


220 Chapter 8 Rate of Return Analysis: Multiple Alternatives<br />

PROBLEMS<br />

Understanding Incremental ROR<br />

8.1 In a tabulation of cash flow, the column entitled<br />

“Rate of return on the incremental cash flow” represents<br />

the rate of return on what?<br />

8.2 If the rate of return on the incremental cash flow<br />

between two alternatives is less than the minimum<br />

attractive rate of return, which alternative should<br />

be selected, if any?<br />

8.3 An engineer is comparing three projects by the incremental<br />

ROR method. There are revenue and<br />

cost cash flow estimates. He used the IRR function<br />

of Excel to determine the ROR values for<br />

each project. The first one is 3.5% above the<br />

MARR, the second is 1.2% below the MARR, and<br />

the third is 2.4% above the MARR. Which alternatives,<br />

if any, must he include in the incremental<br />

ROR analysis?<br />

8.4 The rates of return on alternatives X and Y are<br />

15% and 12%, respectively. Alternative Y requires<br />

a larger investment than alternative X.<br />

(a)<br />

What is known about the rate of return on the<br />

increment of investment between the two alternatives?<br />

(b) If the MARR is 12%, which alternative<br />

should be selected and why?<br />

8.5 Victoria is comparing two mutually exclusive<br />

alternatives, A and B. The overall ROR on alternative<br />

A is greater than the MARR, and the overall<br />

ROR on alternative B, which requires the larger<br />

investment, is exactly equal to the MARR.<br />

(a) What is known about the ROR on the increment<br />

between A and B?<br />

(b) Which alternative should be selected?<br />

8.6 A food processing company is considering two<br />

types of moisture analyzers. Only one can be selected.<br />

The company expects an infrared model to<br />

yield a rate of return of 27% per year. A more expensive<br />

microwave model will yield a rate of return<br />

of 22% per year. If the company’s MARR is<br />

19% per year, can you determine which model<br />

should be purchased solely on the basis of the<br />

overall rate of return information provided? Why<br />

or why not?<br />

8.7 If $80,000 is invested at 30% and another $50,000<br />

is invested at 20% per year, what is the overall rate<br />

of return on the entire $130,000?<br />

8.8 A total of $100,000 was invested in two different<br />

projects identified as Z1 and Z2. If the overall rate<br />

of return on the $100,000 was 30% and the rate of<br />

return on the $30,000 invested in Z1 was 15%,<br />

what was the rate of return on Z2?<br />

8.9 Tuggle, Inc., which manufactures rigid shaft couplings,<br />

has $600,000 to invest. The company is<br />

considering three different projects that will yield<br />

the following rates of return.<br />

Project X i X 24%<br />

Project Y i Y 18%<br />

Project Z i Z 30%<br />

The initial investment required for each project is<br />

$100,000, $300,000, and $200,000, respectively.<br />

If Tuggle’s MARR is 15% per year and it invests<br />

in all three projects, what rate of return will the<br />

company make?<br />

8.10 Two options are available for setting up a wireless<br />

meter scanner and controller. A simple setup is<br />

good for 2 years and has an initial cost of $12,000,<br />

no salvage value, and an operating cost of $27,000<br />

per year. A more permanent system has a higher<br />

first cost of $73,000, but it has an estimated life of<br />

6 years and a salvage value of $15,000. It costs<br />

only $14,000 per year to operate and maintain. If<br />

the two options are compared using an incremental<br />

rate of return, what are the incremental cash flows<br />

in ( a ) year 0 and ( b ) year 2?<br />

8.11 Prepare a tabulation of incremental cash flows for<br />

the two machine alternatives below.<br />

Machine X<br />

Machine Y<br />

First cost, $ 35,000 90,000<br />

Annual operating cost, $ per year 31,600 19,400<br />

Salvage value, $ 0 8,000<br />

Life, years 2 4<br />

8.12 For the alternatives shown, determine the sum of<br />

the incremental cash flows for Q – P.<br />

Alternative P Alternative Q<br />

First cost, $ 50,000 85,000<br />

Annual operating cost, $ per year 8,600 2,000<br />

Annual revenue, $ per year 22,000 45,000<br />

Salvage value, $ 3,000 8,000<br />

Life, years 3 6<br />

8.13 The tabulation of the incremental cash flows between<br />

alternatives A and B is shown on the next<br />

page. Alternative A has a 3-year life and alternative<br />

B a 6-year life. If neither alternative has a salvage<br />

value, what is ( a ) the first cost of alternative<br />

A and ( b ) the first cost of alternative B?


Problems 221<br />

Year<br />

Incremental<br />

Cash Flow (B A), $<br />

0 –20,000<br />

1 5,000<br />

2 5,000<br />

3 12,000<br />

4 5,000<br />

5 5,000<br />

6 5,000<br />

8.14 Standby power for pumps at water distribution<br />

booster stations can be provided by either gasolineor<br />

diesel-powered engines. The costs for the gasoline<br />

engines are as follows:<br />

Gasoline<br />

First cost, $ 150,000<br />

Annual M&O, $ per year 41,000<br />

Salvage value, $ 23,000<br />

Life, years 15<br />

The incremental PW cash flow equation associated<br />

with (diesel – gasoline) is<br />

0 40,000 11,000( P A , i ,15) 16,000( P F , i ,15)<br />

Determine the following:<br />

( a) First cost of the diesel engines<br />

( b) Annual M&O cost of the diesel engines<br />

( c) Salvage value of the diesel engines<br />

8.15 Several high-value parts for NASA’s reusable<br />

space exploration vehicle can be either anodized<br />

or powder-coated. Some of the costs for each<br />

process are shown below:<br />

Anodize<br />

Powder Coat<br />

First cost, $ ? 65,000<br />

Annual cost, $ per year 21,000 ?<br />

Resale value, $ ? 6,000<br />

Life, years 3 3<br />

The incremental AW cash flow equation associated<br />

with (powder coat – anodize) is<br />

0 –14,000( A P , i ,3) 5000 2000( A F , i ,3)<br />

What is ( a ) the first cost for anodizing, ( b ) the annual<br />

cost for powder coating, and ( c ) the resale<br />

(salvage) value of the anodized parts?<br />

Incremental ROR Comparison (Two Alternatives)<br />

8.16 Specialty Gases & Chemicals manufactures nitrogen<br />

trifluoride, a highly specialized gas used as an<br />

industrial cleansing agent for flat panels installed in<br />

laptop computers, televisions, and desktop monitors.<br />

The incremental cash flow associated with two alternatives<br />

for chemical storage and handling systems<br />

(identified as P3 and X3) has been calculated in<br />

$1000 units. Determine ( a ) the rate of return on the<br />

incremental cash flows and ( b ) which one should be<br />

selected if the company’s MARR is 25% per year.<br />

Alternative X3 requires the larger initial investment.<br />

Year<br />

Incremental Cash Flow<br />

(X3 P3), $1000<br />

0 $4600<br />

1–9 1100<br />

10 2000<br />

8.17 As groundwater wells age, they sometimes begin to<br />

pump sand (and they become known as “sanders”),<br />

and this can cause damage to downstream desalting<br />

equipment. This situation can be dealt with by drilling<br />

a new well at a cost of $1,000,000 or by installing<br />

a tank and self-cleaning screen ahead of the<br />

desalting equipment. The tank and screen will cost<br />

$230,000 to install and $61,000 per year to operate<br />

and maintain. A new well will have a pump that is<br />

more efficient than the old one, and it will require<br />

almost no maintenance, so its operating cost will be<br />

only $18,000 per year. If the salvage values are estimated<br />

at 10% of the first cost, use a present worth<br />

relation to ( a ) calculate the incremental rate of return<br />

and ( b ) determine which alternative is better at<br />

a MARR of 6% per year over a 20-year study<br />

period.<br />

8.18 Konica Minolta plans to sell a copier that prints<br />

documents on both sides simultaneously, cutting in<br />

half the time it takes to complete big commercial jobs.<br />

The costs associated with producing chemicallytreated<br />

vinyl rollers and fiber-impregnated rubber<br />

rollers are shown below. Determine which of<br />

the two types should be selected by calculating the<br />

rate of return on the incremental investment.<br />

Assume the company’s MARR is 21% per year.<br />

(Solve by hand and/or spreadsheet, as instructed.)<br />

Treated<br />

Impregnated<br />

First cost, $ 50,000 95,000<br />

Annual cost, $ per year 100,000 85,000<br />

Salvage value, $ 5,000 11,000<br />

Life, years 3 6<br />

8.19 The Texas Department of Transportation (TxDOT)<br />

is considering two designs for crash barriers along<br />

a reconstructed portion of I-10. Design 2B will<br />

cost $3 million to install and $135,000 per year to<br />

maintain. Design 4R will cost $3.7 million to install<br />

and $70,000 per year to maintain. Calculate<br />

the rate of return and determine which design is<br />

preferred if TxDOT uses a MARR of 6% per year<br />

and a 20-year project period.


222 Chapter 8 Rate of Return Analysis: Multiple Alternatives<br />

8.20 Chem-Tex Chemical is considering two additives<br />

for improving the dry-weather stability of its lowcost<br />

acrylic paint. Additive A has a first cost of<br />

$110,000 and an annual operating cost of $60,000.<br />

Additive B has a first cost of $175,000 and an annual<br />

operating cost of $35,000. If the company<br />

uses a 3-year recovery period for paint products<br />

and a MARR of 20% per year, which process is<br />

economically favored? Use an incremental ROR<br />

analysis.<br />

8.21 The manager of Liquid Sleeve, Inc., a company<br />

that makes a sealing solution for machine shaft surfaces<br />

that have been compromised by abrasion,<br />

high pressures, or inadequate lubrication, is considering<br />

adding metal-based nanoparticles of either<br />

type Al or Fe to its solution to increase the product’s<br />

performance at high temperatures. The costs<br />

associated with each are shown below. The company’s<br />

MARR is 20% per year. Do the following<br />

using a PW-based rate of return analysis and a<br />

spreadsheet:<br />

(a)<br />

Determine which nanoparticle type the company<br />

should select using the i * value.<br />

(b) On the same graph, plot the PW versus different<br />

i values for each alternative. Indicate<br />

the breakeven i * value and the MARR value<br />

on the plot.<br />

(c)<br />

Use the plot of PW versus i values to select<br />

the better alternative with MARR 20% per<br />

year. Is the answer the same as in part ( a )?<br />

Type Fe<br />

Type Al<br />

First cost, $ 150,000 280,000<br />

Annual operating cost, $ per year 92,000 74,000<br />

Salvage value, $ 30,000 70,000<br />

Life, years 2 4<br />

8.22 A chemical company is considering two processes<br />

for isolating DNA material. The incremental cash<br />

flows between the two alternatives, J and S, have an<br />

incremental rate of return that is less than 40%,<br />

which is the MARR of the company. However, the<br />

company CEO prefers the more expensive process<br />

S. She believes the company can implement cost<br />

controls to reduce the annual cost of the more<br />

expensive process. By how much would she have to<br />

reduce the annual operating cost of alternative S (in<br />

$ per year) for it to have an incremental rate of<br />

return of exactly 40%?<br />

Year<br />

Incremental Cash Flow<br />

(S J), $<br />

0 900,000<br />

1 400,000<br />

2 400,000<br />

3 400,000<br />

8.23 The incremental cash flows for two alternative<br />

electrode setups are shown. The MARR is 12%<br />

per year, and alternative Dryloc requires a larger<br />

initial investment compared to NPT.<br />

(a)<br />

Determine which should be selected using an<br />

AW-based rate of return analysis.<br />

(b) Use a graph of incremental values to determine<br />

the largest MARR value that will justify<br />

the NPT alternative.<br />

Year<br />

Incremental Cash Flow<br />

(Dryloc NPT), $<br />

0 56,000<br />

18 8,900<br />

9 12,000<br />

8.24 Hewett Electronics manufactures amplified pressure<br />

transducers. It must decide between two machines<br />

for a finishing operation. Select one for them<br />

on the basis of AW-based rate of return analysis.<br />

The company’s MARR is 18% per year.<br />

Variable<br />

Speed<br />

Dual<br />

Speed<br />

First cost, $ 270,000 245,000<br />

Annual operating cost, $ per year 135,000 139,000<br />

Salvage value, $ 75,000 35,000<br />

Life, years 6 6<br />

8.25 A manufacturer of hydraulic equipment is trying to<br />

determine whether it should use monoflange double<br />

block and bleed (DBB) valves or a multi-valve<br />

system (MVS) for chemical injection. The costs<br />

are shown below. Use an AW-based rate of return<br />

analysis and a MARR of 18% per year to determine<br />

the better of the two options.<br />

DBB<br />

MVS<br />

First cost, $ 40,000 71,000<br />

Annual cost, $ per year 60,000 65,000<br />

Salvage value, $ 0 18,000<br />

Life, years 2 4<br />

8.26 Poly-Chem Plastics is considering two types of injection<br />

molding machines—hydraulic and electric.<br />

The hydraulic press (HP) will have a first cost of<br />

$600,000, annual costs of $200,000, and a salvage<br />

value of $70,000 after 5 years. Electric machine<br />

technology (EMT) will have a first cost of $800,000,<br />

annual costs of $150,000, and a salvage value of<br />

$130,000 after 5 years.<br />

(a) Use an AW-based rate of return equation to<br />

determine the ROR on the increment of<br />

investment between the two.


Problems 223<br />

(b) Determine which machine the company<br />

should select, if the MARR 16% per year.<br />

( c) Plot the AW versus i graph for each alternative’s<br />

cash flows, and utilize it to determine<br />

the largest MARR that will justify the EMT<br />

extra investment of $200,000.<br />

8.27 Last week Eduardo calculated the overall project<br />

ROR values for two alternatives A and B using the<br />

estimates below. He calculated i A<br />

* 34.2% and<br />

i B<br />

* 31.2% and recommended acceptance of A<br />

since its rate of return exceeded the established<br />

MARR of 30% by a greater amount than project B.<br />

Yesterday, the general manager of the company<br />

announced a major capital investment program,<br />

which includes a large drop in the MARR from<br />

30% to 20% per year. Do the following to help<br />

Eduardo better understand the rate of return method<br />

and what this reduction in MARR means.<br />

( a) Explain the error that Eduardo made in performing<br />

the rate of return analysis.<br />

( b) Perform the correct analysis using each<br />

MARR value.<br />

( c) Illustrate the ranking inconsistency problem<br />

using the two MARR values, and determine<br />

the maximum MARR that will justify alternative<br />

B.<br />

Alternative A<br />

Alternative B<br />

First cost, $ 40,000 85,000<br />

Annual operating cost, 5,500 15,000<br />

$ per year<br />

Annual revenue, $ per year 22,000 45,000<br />

Salvage value, $ 0 20,000<br />

Life, years 6 6<br />

i *, % 34.2 29.2<br />

Multiple Alternative Comparison<br />

8.28 Four mutually exclusive revenue alternatives<br />

are under consideration to automate a baking<br />

and packaging process at Able Bakery Products.<br />

The alternatives are ranked in order of increasing<br />

initial investment and compared by<br />

incremental rate of return analysis. The rate of<br />

return on each increment of investment was less<br />

than the MARR. Which alternative should be<br />

selected?<br />

8.29 A WiMAX wireless network integrated with a<br />

satellite network can provide connectivity to<br />

any location within 10 km of the base station.<br />

The number of sectors per base station can be<br />

varied to increase the bandwidth. An independent<br />

cable operator is considering three bandwidth<br />

alternatives. Assume a life of 20 years and<br />

a MARR of 10% per year to determine which<br />

alternative is best using an incremental rate of<br />

return analysis.<br />

Bandwidth,<br />

Mbps<br />

First Cost,<br />

$1000<br />

Operating<br />

Cost,<br />

$1000 per Year<br />

Annual<br />

Income,<br />

$1000 per Year<br />

44 –40,000 –2000 5000<br />

55 –46,000 –1000 5000<br />

88 –61,000 –500 8000<br />

8.30 Xerox’s iGenX high-speed commercial printers<br />

cost $1.5 billion to develop. The machines cost<br />

$500,000 to $750,000 depending on what options<br />

the client selects. Spectrum Imaging Systems is<br />

considering the purchase of a new printer based<br />

on recent contracts it received for printing weekly<br />

magazine and mail-out advertising materials. The<br />

operating costs and revenues generated are related<br />

to a large extent to the speed and other capabilities<br />

of the copier. Spectrum is considering the four<br />

machines shown below. The company uses a<br />

3-year planning period and a MARR of 15% per<br />

year. Determine which copier the company should<br />

acquire on the basis of an incremental rate of return<br />

analysis.<br />

Copier<br />

Initial<br />

Investment,<br />

$<br />

Operating<br />

Cost,<br />

$ per Year<br />

Annual<br />

Revenue,<br />

$ per Year<br />

Salvage<br />

Value, $<br />

iGen-1 –500,000 –350,000 450,000 70,000<br />

iGen-2 –600,000 –300,000 460,000 85,000<br />

iGen-3 –650,000 –275,000 480,000 95,000<br />

iGen-4 –750,000 –200,000 510,000 120,000<br />

8.31 Ashley Foods, Inc. has determined that any one of<br />

five machines can be used in one phase of its chili<br />

canning operation. The costs of the machines are<br />

estimated below, and all machines are estimated to<br />

have a 4-year useful life. If the minimum attractive<br />

rate of return is 20% per year, determine which<br />

machine should be selected on the basis of a rate of<br />

return analysis.<br />

Machine First Cost, $<br />

Annual<br />

Operating<br />

Cost, $ per Year<br />

1 31,000 16,000<br />

2 29,000 19,300<br />

3 34,500 17,000<br />

4 49,000 12,200<br />

5 41,000 15,500<br />

8.32 Five revenue projects are under consideration by<br />

General Dynamics for improving material flow<br />

through an assembly line. The initial cost in


224 Chapter 8 Rate of Return Analysis: Multiple Alternatives<br />

$1000 and the life of each project are as follows<br />

(revenue estimates are not shown):<br />

Project<br />

A B C D E<br />

Initial cost, $1000 700 2300 900 300 1600<br />

Life, years 5 8 5 5 6<br />

An engineer made the comparisons shown below.<br />

From the calculations, determine which project, if<br />

any, should be undertaken if the company’s MARR<br />

is ( a ) 11.5% per year and ( b ) 13.5% per year. If<br />

other calculations are necessary to make a decision,<br />

state which ones.<br />

Comparison Incremental Rate of Return, %<br />

B vs DN 13%<br />

A vs B 19%<br />

D vs DN 11%<br />

E vs B 15%<br />

E vs D 24%<br />

E vs A 21%<br />

C vs DN 7%<br />

C vs A 19%<br />

E vs DN 12%<br />

A vs DN 10%<br />

E vs C 33%<br />

D vs C 33%<br />

D vs B 29%<br />

8.33 The five alternatives shown here are being evaluated<br />

by the rate of return method.<br />

Alternative<br />

Initial<br />

Investment,<br />

$<br />

ROR<br />

versus<br />

DN, %<br />

Incremental<br />

Rate of Return, %<br />

A B C D E<br />

A –25,000 9.6 — 27.3 9.4 35.3 25.0<br />

B –35,000 15.1 — 0 38.5 24.4<br />

C –40,000 13.4 — 46.5 27.3<br />

D –60,000 25.4 — 6.8<br />

E –75,000 20.2 —<br />

(a)<br />

(b)<br />

If the alternatives are mutually exclusive and<br />

the MARR is 26% per year, which alternative<br />

should be selected?<br />

If the alternatives are mutually exclusive and<br />

the MARR is 15% per year, which alternative<br />

should be selected?<br />

(c) If the alternatives are independent and the<br />

MARR is 15% per year, which alternative(s)<br />

should be selected?<br />

8.34 Five mutually exclusive revenue alternatives that<br />

have infi nite live s are under consideration for increasing<br />

productivity in a manufacturing operation.<br />

The initial costs and cash flows of each<br />

project are shown. If the MARR is 14.9% per year,<br />

which alternative should be selected?<br />

Alternative<br />

A B C D E<br />

Initial cost, $ 7,000 23,000 9,000 3,000 16,000<br />

Cash flow, $ per 1,000 3,500 1,400 500 2,200<br />

year<br />

Rate of return<br />

(vs DN), %<br />

14.3 15.2 15.6 16.7 13.8<br />

8.35 The plant manager at Automaton Robotics is looking<br />

at the summarized incremental rate of return<br />

information shown below for five mutually exclusive<br />

alternatives, one of which must be chosen.<br />

The table includes the overall ROR and the incremental<br />

comparison of alternatives. Which alternative<br />

is best if the minimum attractive rate of return<br />

is ( a ) 15% per year and ( b ) 12% per year?<br />

Alternative<br />

First<br />

Cost, $<br />

Overall<br />

ROR, %<br />

Incremental Rate of Return, %<br />

A B C D E<br />

A 80,000 14 — 12 11 17 24<br />

B 60,000 16 12 — 14 23 21<br />

C 40,000 17 11 14 — 35 29<br />

D 30,000 12 17 23 35 — 17<br />

E 20,000 8 24 21 29 17 —<br />

8.36 Only one of four different machines can be purchased<br />

for Glass Act Products. An engineer performed<br />

the following analysis to select the best<br />

machine, all of which have a 10-year life. Which<br />

machine, if any, should the company select at a<br />

MARR of 22% per year?<br />

Machine<br />

1 2 3 4<br />

Initial cost, $ 44,000 60,000 72,000 98,000<br />

Annual cost, $ per year 70,000 64,000 61,000 58,000<br />

Annual savings, 80,000 80,000 80,000 82,000<br />

$ per year<br />

Overall ROR, % 18.6 23.4 23.1 20.8<br />

Machines compared 2 to 1 3 to 2 4 to 3<br />

Incremental<br />

16,000 12,000 26,000<br />

investment, $<br />

Incremental cash flow,<br />

6,000 3,000 5,000<br />

$ per year<br />

ROR on increment, % 35.7 21.4 14.1<br />

8.37 The U.S. Bureau of Reclamation is considering five<br />

national park projects shown below, all of which can<br />

be considered to last indefinitely. At a MARR of<br />

7.5% per year, determine which should be selected, if<br />

they are ( a ) independent and ( b ) mutually exclusive.<br />

Project ID<br />

First<br />

Cost, $1000<br />

Annual<br />

Income, $1000<br />

Rate of<br />

Return, %<br />

A 20,000 2000 10.0<br />

B 10,000 1300 13.0<br />

C 15,000 1000 6.6<br />

D 70,000 4000 5.7<br />

E 50,000 2600 5.2


Additional Problems and FE Exam Review Questions 225<br />

ADDITIONAL PROBLEMS AND FE EXAM REVIEW QUESTIONS<br />

8.38 When conducting a rate of return (ROR) analysis<br />

involving multiple mutually exclusive alternatives,<br />

the first step is to:<br />

( a) Rank the alternatives according to decreasing<br />

initial investment cost<br />

( b) Rank the alternatives according to increasing<br />

initial investment cost<br />

( c) Calculate the present worth of each alternative<br />

using the MARR<br />

( d) Find the LCM between all of the alternatives<br />

8.39 In comparing mutually exclusive alternatives by<br />

the ROR method, you should:<br />

( a) Find the ROR of each alternative and pick<br />

the one with the highest ROR<br />

( b) Select the alternative whose incremental<br />

ROR is the highest<br />

( c) Select the alternative with ROR MARR<br />

that has the lowest initial investment cost<br />

(d) Select the alternative with the largest initial investment<br />

that has been incrementally justified<br />

8.40 When comparing independent projects by the<br />

ROR method, you should:<br />

( a) Find the ROR of each project and pick the<br />

ones with the highest ROR<br />

( b) Select all projects that have an overall ROR <br />

MARR<br />

( c) Select the project with an overall ROR <br />

MARR that involves the lowest initial<br />

investment cost<br />

(d) Select the project with the largest initial investment<br />

that has been incrementally justified<br />

8.41 Of the following scenarios, alternative Y requires a<br />

higher initial investment than alternative X, and<br />

the MARR is 20% per year. The only scenario that<br />

requires an incremental investment analysis to<br />

s elect an alternative is that:<br />

( a) X has an overall ROR of 22% per year, and Y<br />

has an overall ROR of 24% per year<br />

( b) X has an overall ROR of 19% per year, and Y<br />

has an overall ROR of 23% per year<br />

( c) X has an overall ROR of 18% per year, and Y<br />

has an overall ROR of 19% per year<br />

( d) X has an overall ROR of 28% per year, and Y<br />

has an overall ROR of 26% per year<br />

8.42 Alternatives whose cash flows (excluding the<br />

salvage value) are all negative are called:<br />

( a) Revenue alternatives<br />

( b) Nonconventional alternatives<br />

( c) Cost alternatives<br />

( d) Independent alternatives<br />

8.43 For these alternatives, the sum of the incremental<br />

cash flows is:<br />

Year A B<br />

0 10,000 14,000<br />

1 2,500 4,000<br />

2 2,500 4,000<br />

3 2,500 4,000<br />

4 2,500 4,000<br />

5 2,500 4,000<br />

(a) $2500<br />

(b) $3500<br />

(c) $6000<br />

(d) $8000<br />

8.44 Helical Systems, Inc. uses a minimum attractive<br />

rate of return of 8% per year, compounded annually.<br />

The company is evaluating two new processes<br />

for improving the efficiency of its manufacturing<br />

operation. The cash flow estimates<br />

associated with each process are shown below. A<br />

correct equation to use for an incremental rate of<br />

return analysis is:<br />

Process I<br />

Process J<br />

First cost, $ 40,000 50,000<br />

Annual cost, $ per year 15,000 12,000<br />

Salvage value, $ 5,000 6,000<br />

Life, years 3 3<br />

(a)<br />

(b)<br />

(c)<br />

(d)<br />

0 10,000 3000( P A , i *,3) <br />

1000( P F , i *,3)<br />

0 40,000( A P , i *,3) 15,000 <br />

5000( A F , i *,3)<br />

0 50,000( A P , i *,3) 12,000 <br />

6000( A F , i *,3)<br />

0 10,000 3000( P A , i *,3) <br />

1000( P F , i *,3)<br />

8.45 For the four independent projects shown, the one or<br />

ones to select using a MARR of 14% per year are:<br />

Project<br />

Rate of Return,<br />

% per Year<br />

A 14<br />

B 12<br />

C 15<br />

D 10<br />

(a) Only C<br />

(b) Only A and C<br />

(c) Only A<br />

( d) Can’t tell; need to conduct incremental<br />

analysis


226 Chapter 8 Rate of Return Analysis: Multiple Alternatives<br />

Problems 8.46 through 8.48 are based on the following<br />

information.<br />

Five alternatives are being evaluated by the incremental<br />

rate of return method.<br />

Alternative<br />

Initial<br />

Investment, $<br />

Overall ROR<br />

versus DN, %<br />

Incremental<br />

Rate of Return, %<br />

A B C D E<br />

A 25,000 9.6 — 27.3 9.4 35.3 25.0<br />

B 35,000 15.1 — 0 38.5 24.4<br />

C 40,000 13.4 — 46.5 27.3<br />

D 60,000 25.4 — 6.8<br />

E 75,000 20.2 —<br />

8.46 If the projects are mutually exclusive and the minimum<br />

attractive rate of return is 14% per year, the<br />

best alternative is:<br />

(a) B (b) C (c) D (d) E<br />

8.47 If the projects are mutually exclusive and the<br />

MARR is 20% per year, the best alternative is:<br />

(a) B<br />

( b) C<br />

( c) D<br />

( d) E<br />

8.48 If the projects are independent, instead of mutually<br />

exclusive, the one or ones to select at an MARR of<br />

18% per year are:<br />

( a) B and C<br />

(b) B, D, and E<br />

(c) D and E<br />

(d) B, C, and E<br />

CASE STUDY<br />

ROR ANALYSIS WITH ESTIMATED LIVES THAT VARY<br />

Background<br />

Make-to-Specs is a software system under development by<br />

ABC Corporation. It will be able to translate digital versions of<br />

three-dimensional computer models, containing a wide variety<br />

of part shapes with machined and highly finished (ultrasmooth)<br />

surfaces. The product of the system is the numerically<br />

controlled (NC) machine code for the part’s manufacturing.<br />

Additionally, Make-to-Specs will build the code for superfine<br />

finishing of surfaces with continuous control of the finishing<br />

machines.<br />

Information<br />

There are two alternative computers that can provide the<br />

server function for the software interfaces and shared database<br />

updates on the manufacturing floor while Make-to-<br />

Specs is operating in parallel mode. The server first cost<br />

and estimated contribution to annual net cash flow are<br />

summarized below.<br />

Server 1 Server 2<br />

First cost, $ 100,000 200,000<br />

Net cash flow, $/year 35,000 50,000 year 1, plus 5000 per<br />

year for years 2, 3, and 4<br />

(gradient)<br />

70,000 maximum for years<br />

5 on, even if the server is<br />

replaced<br />

Life, years 3 or 4 5 or 8<br />

The life estimates were developed by two different individuals:<br />

a design engineer and a manufacturing manager. They have<br />

asked that, at this stage of the project, all analyses be performed<br />

using both life estimates for each system.<br />

Case Study Exercises<br />

Use spreadsheet analysis to determine the following:<br />

1. If the MARR 12%, which server should be selected?<br />

Use the PW or AW method to make the selection.<br />

2. Use incremental ROR analysis to decide between the<br />

servers at MARR 12%.<br />

3. Use any method of economic analysis to display on the<br />

spreadsheet the value of the incremental ROR between<br />

server 2 with a life estimate of 5 years and a life estimate<br />

of 8 years.


Case Study 227<br />

CASE STUDY<br />

HOW A NEW ENGINEERING GRADUATE CAN HELP HIS FATHER 1<br />

Background<br />

“I don’t know whether to sell it, expand it, lease it, or what. But<br />

I don’t think we can keep doing the same thing for many more<br />

years. What I really want to do is to keep it for 5 more years,<br />

then sell it for a bundle,” Elmer Kettler said to his wife, Janise,<br />

their son, John Kettler, and new daughter-in-law, Suzanne<br />

Gestory, as they were gathered around the dinner table. Elmer<br />

was sharing thoughts on Gulf Coast Wholesale Auto Parts, a<br />

company he has owned and operated for 25 years on the southern<br />

outskirts of Houston, Texas. The business has excellent contracts<br />

for parts supply with several national retailers operating<br />

in the area—NAPA, AutoZone, O’Reilly, and Advance. Additionally,<br />

Gulf Coast operates a rebuild shop serving these same<br />

retailers for major automobile components, such as carburetors,<br />

transmissions, and air conditioning compressors.<br />

At his home after dinner, John decided to help his father<br />

with an important and difficult decision: What to do with his<br />

business? John graduated just last year with an engineering<br />

degree from a major state university in Texas, where he completed<br />

a course in engineering economy. Part of his job at<br />

Energcon Industries is to perform basic rate of return and<br />

present worth analyses on energy management proposals.<br />

Information<br />

Over the next few weeks, John outlined five options, including<br />

his dad’s favorite of selling in 5 years. John summarized<br />

all the estimates over a 10-year horizon. The options and<br />

estimates were given to Elmer, and he agreed with them.<br />

Option 1: Remove rebuild. Stop operating the rebuild<br />

shop and concentrate on selling wholesale<br />

parts. The removal of the rebuild operations and the<br />

switch to an “all-parts house” are expected to cost<br />

$750,000 in the first year. Overall revenues will<br />

drop to $1 million the first year with an expected 4%<br />

increase per year thereafter. Expenses are projected<br />

at $0.8 million the first year, increasing 6% per year<br />

thereafter.<br />

Option 2: Contract rebuild operations. To get the<br />

rebuild shop ready for an operations contractor to<br />

take over will cost $400,000 immediately. If expenses<br />

stay the same for 5 years, they will average<br />

$1.4 million per year, but they can be expected to<br />

rise to $2 million per year in year 6 and thereafter.<br />

Elmer thinks revenues under a contract arrangement<br />

can be $1.4 million the first year and can rise 5% per<br />

year for the duration of a 10-year contract.<br />

Option 3: Maintain status quo and sell out after 5 years<br />

(Elmer’s personal favorite). There is no cost now, but<br />

the current trend of negative net profit will probably<br />

continue. Projections are $1.25 million per year for expenses<br />

and $1.15 million per year in revenue. Elmer<br />

had an appraisal last year, and the report indicated Gulf<br />

Coast Wholesale Auto Parts is worth a net $2 million.<br />

Elmer’s wish is to sell out completely after 5 more<br />

years at this price, and to make a deal that the new<br />

owner pay $500,000 per year at the end of year 5 (sale<br />

time) and the same amount for the next 3 years.<br />

Option 4: Trade-out. Elmer has a close friend in the<br />

antique auto parts business who is making a “ killing,”<br />

so he says, with e-commerce. Although the possibility<br />

is risky, it is enticing to Elmer to consider a whole new<br />

line of parts, but still in the basic business that he already<br />

understands. The trade-out would cost an estimated<br />

$1 million for Elmer immediately. The 10-year<br />

horizon of annual expenses and revenues is considerably<br />

higher than for his current business. Expenses are<br />

estimated at $3 million per year and revenues at<br />

$3.5 million each year.<br />

Option 5: Lease arrangement. Gulf Coast could be<br />

leased to some turnkey company with Elmer remaining<br />

the owner and bearing part of the expenses for building,<br />

delivery trucks, insurance, etc. The first-cut estimates<br />

for this option are $1.5 million to get the business ready<br />

now, with annual expenses at $500,000 per year and<br />

revenues at $1 million per year for a 10-year contract.<br />

Case Study Exercises<br />

Help John with the analysis by doing the following:<br />

1. Develop the actual cash flow series and incremental<br />

cash flow series (in $1000 units) for all five options in<br />

preparation for an incremental ROR analysis.<br />

2. Discuss the possibility of multiple rate of return values<br />

for all the actual and incremental cash flow series. Find<br />

any multiple rates in the range of 0% to 100%.<br />

3. If John’s father insists that he make 25% per year or more<br />

on the selected option over the next 10 years, what should<br />

he do? Use all the methods of economic analysis you<br />

have learned so far (PW, AW, ROR) so John’s father can<br />

understand the recommendation in one way or another.<br />

4. Prepare plots of the PW versus i for each of the five options.<br />

Estimate the breakeven rate of return between options.<br />

5. What is the minimum amount that must be received in<br />

each of years 5 through 8 for option 3 (the one Elmer<br />

wants) to be best economically? Given this amount,<br />

what does the sale price have to be, assuming the same<br />

payment arrangement as presented above?<br />

1 Based upon a study by Alan C. Stewart, Consultant, Communications and High Tech Solutions <strong>Engineering</strong>, Accenture LLP.


CHAPTER 9<br />

Benefit/Cost<br />

Analysis and<br />

Public Sector<br />

Economics<br />

LEARNING OUTCOMES<br />

Purpose: Understand public sector projects and select the best alternative on the basis of incremental benefit /cost analysis.<br />

SECTION TOPIC LEARNING OUTCOME<br />

9.1 Public sector • Explain some of the fundamental differences<br />

between private and public sector projects.<br />

9.2 B/C for single project • Calculate the benefit/cost ratio and use it to<br />

evaluate a single project.<br />

9.3 Incremental B/C • Select the better of two alternatives using the<br />

incremental B/C ratio method.<br />

9.4 More than two alternatives • Based on the incremental B/C ratios, select the<br />

best of multiple alternatives.<br />

9.5 Service projects and CEA • Explain service sector projects and use costeffectiveness<br />

analysis (CEA) to evaluate projects.<br />

9.6 Ethical considerations • Explain the major aspects of public project<br />

activities, and describe how ethical compromise<br />

may enter public sector project analysis.


T<br />

he evaluation methods of previous chapters are usually applied to alternatives<br />

in the private sector, that is, for-profit and not-for-profit corporations and businesses.<br />

This chapter introduces public sector and service sector alternatives and<br />

their economic consideration. In the case of public projects, the owners and users (beneficiaries)<br />

are the citizens and residents of a government unit—city, county, state, province,<br />

or nation. Government units provide the mechanisms to raise capital and operating funds.<br />

Public-private partnerships have become increasingly common, especially for large infrastructure<br />

projects such as major highways, power generation plants, water resource developments,<br />

and the like.<br />

The benefit/cost (B/C) ratio introduces objectivity into the economic analysis of public sector<br />

evaluation, thus reducing the effects of politics and special interests. The different formats<br />

of B/C analysis, and associated disbenefits of an alternative, are discussed here. The B/C<br />

analysis can use equivalency computations based on PW, AW, or FW values. Performed correctly,<br />

the benefit/cost method will always select the same alternative as PW, AW, and ROR<br />

analyses.<br />

This chapter also introduces service sector projects and discusses how their economic evaluation<br />

is different from that for other projects. Finally, there is a discussion on professional<br />

ethics and ethical dilemmas in the public sector.<br />

Water Treatment Facility #3 Case: Allen<br />

Water Utilities has planned for the last<br />

25 years to construct a new drinking<br />

water treatment facility that will supply<br />

the rapidly growing north and northwest<br />

areas of the city. An expectation of<br />

over 100,000 new residents in the next<br />

several years and 500,000 by 2040<br />

prompted the development of the plant<br />

starting in <strong>2012</strong>. The supply is from a<br />

large surface lake currently used to provide<br />

water to all of Allen and the surrounding<br />

communities. The project is<br />

termed WTF3, and its initial capital investment<br />

is $540 million for the treatment<br />

plant and two large steel-pipe<br />

transmission mains (84- and 48-inch)<br />

that will be installed via tunneling approximately<br />

100 to 120 feet under suburban<br />

areas of the city to reach current<br />

r eservoirs.<br />

Tunneling was selected after geotechnical<br />

borings indicated that open trenching<br />

was not supportable by the soil and<br />

based upon a large public outcry against<br />

trenching in the living areas along the<br />

selected transmission routes. Besides the<br />

treatment plant construction on the 95-<br />

acre site, there must be at least three<br />

large vertical shafts (25 to 50 feet in diameter)<br />

bored along each transmission<br />

main to gain underground access for<br />

equipment and debris removal during<br />

the tunneling operations.<br />

PE<br />

The stated criteria used to make decisions<br />

for WTF3 and the transmission<br />

mains were economics, environment,<br />

community impact, and constructability.<br />

There are major long-term benefits for<br />

the new facility. These are some mentioned<br />

by city engineers:<br />

• It will meet projected water needs<br />

of the city for the next 50 years.<br />

• The new treatment plant is at a<br />

higher elevation than the current<br />

two plants, allowing gravity flow to<br />

replenish reservoirs, thereby using<br />

little or no electric pumping.<br />

• There will be an increase in the<br />

diversity and reliability of supply as<br />

other plants age.<br />

• It will provide a water quality that<br />

is more consistent due to the location<br />

of the raw water intakes.<br />

• The facility uses water supplies<br />

already purchased; therefore, there<br />

is no need to negotiate additional<br />

allowances.<br />

The disbenefits are mostly short-term<br />

during the construction of WTF3 and<br />

transmission mains. Some of these are<br />

mentioned by citizen groups and one retired<br />

city engineer:<br />

• There will be disruption of habitat<br />

for some endangered species<br />

of birds, lizards, and trees not


230 Chapter 9 Benefit/Cost Analysis and Public Sector Economics<br />

found in any other parts of the<br />

country.<br />

• Large amounts of dust and smoke<br />

will enter the atmosphere in a<br />

residential area during the 3½ years<br />

of construction, tunneling, and<br />

transmission main completion.<br />

• Noise pollution and traffic congestion<br />

will result during an estimated<br />

26,000 truck trips to remove debris<br />

from the plant site and tunnel<br />

shafts, in addition to the problems<br />

from regular construction traffic.<br />

• Natural landscape in plant and tunnel<br />

shaft sites will be destroyed.<br />

• Safety will be compromised for children<br />

in a school where large trucks<br />

will pass about every 5 minutes for<br />

approximately 12 hours per day,<br />

6 days per week for 2½ years.<br />

• There may be delays in fire and<br />

ambulance services in emergencies,<br />

since many neighborhood streets are<br />

country-road width and offer only<br />

single ingress/egress streets for neighborhoods<br />

along the indicated routes.<br />

• The need for the facility has not<br />

been proved, as the water will be<br />

sold to developers outside the city<br />

limits, not provided to residences<br />

within Allen.<br />

• Newly generated revenues will be<br />

used to pay off the capital funding<br />

bonds approved for the plant’s construction.<br />

Last year, the city engineers did a benefit/cost<br />

analysis for this massive public<br />

sector project; none of the results were<br />

publicized. Public and elected official intervention<br />

has now caused some of the<br />

conclusions using the criteria mentioned<br />

above to be questioned by the general<br />

manager of Allen Water Utilities.<br />

This case is used in the following topics<br />

of this chapter:<br />

Public sector projects (Section 9.1)<br />

Incremental B/C analysis, two alternatives<br />

(Section 9.3)<br />

Incremental B/C analysis, more than<br />

two alternatives (Section 9.4)<br />

9.1 Public Sector Projects<br />

Virtually all the examples and problems of previous chapters have involved the private sector,<br />

where products, systems, and services are developed and offered by corporations and businesses<br />

for use by individual customers and clients, the government, or other companies. (Notable exceptions<br />

are the long-life alternatives discussed in Chapters 5 (PW) and 6 (AW) where capitalized<br />

cost analysis was applied.) Now we will explore projects that concentrate on government units<br />

and the citizens they serve. These are called public sector projects.<br />

A public sector project is a product, service, or system used, financed, and owned by the citizens<br />

of any government level. The primary purpose is to provide service to the citizenry for the<br />

public good at no profit. Areas such as public health, criminal justice, safety, transportation,<br />

welfare, and utilities are publically owned and require economic evaluation.<br />

Upon reflection, it is surprising how much of what we use on a daily or as-needed basis is<br />

publicly owned and financed to serve us—the citizenry. These are some public sector<br />

examples:<br />

Hospitals and clinics<br />

Parks and recreation<br />

Utilities: water, electricity, gas,<br />

sewer, sanitation<br />

Schools: primary, secondary, community<br />

colleges, universities<br />

Economic development projects<br />

Convention centers<br />

Sports arenas<br />

Transportation: highways, bridges,<br />

waterways


9.1 Public Sector Projects 231<br />

Police and fire protection<br />

Courts and prisons<br />

Food stamp and rent relief programs<br />

Job training<br />

Public housing<br />

Emergency relief<br />

Codes and standards<br />

There are significant differences in the characteristics of private and public sector alternatives.<br />

They are summarized here.<br />

Characteristic Public sector Private sector<br />

Size of investment Large Some large; more medium to small<br />

Often alternatives developed to serve public needs require large initial investments, possibly<br />

distributed over several years. Modern highways, public transportation systems, universities,<br />

airports, and flood control systems are examples.<br />

Characteristic Public sector Private sector<br />

Life estimates Longer (30–50 years) Shorter (2–25 years)<br />

The long lives of public projects often prompt the use of the capitalized cost method, where<br />

infinity is used for n and annual costs are calculated as A P ( i ). As n gets larger, especially<br />

over 30 years, the differences in calculated A values become small. For example, at i 7%,<br />

there will be a very small difference in 30 and 50 years, because ( A/P ,7%,30) 0.08059 and<br />

( A/P ,7%,50) 0.07246.<br />

Characteristic Public sector Private sector<br />

Annual cash flow<br />

estimates<br />

No profit; costs, benefits, and<br />

disbenefits are estimated<br />

Revenues contribute to<br />

profits; costs are estimated<br />

Public sector projects (also called publicly owned) do not have profits; they do have costs that are<br />

paid by the appropriate government unit; and they benefit the citizenry. Public sector projects<br />

often have undesirable consequences, as interpreted by some sectors of the public. It is these<br />

consequences that can cause public controversy about the projects. The economic analysis should<br />

consider these consequences in monetary terms to the degree estimable. (Often in private sector<br />

analysis, undesirable consequences are not considered, or they may be directly addressed as<br />

costs.) To perform a benefit/cost economic analysis of public alternatives, the costs (initial and<br />

annual), the benefits, and the disbenefits, if considered, must be estimated as accurately as<br />

possible in monetary units.<br />

Costs—estimated expenditures to the government entity for construction, operation, and maintenance<br />

of the project, less any expected salvage value.<br />

Benefits—advantages to be experienced by the owners, the public.<br />

Disbenefits—expected undesirable or negative consequences to the owners if the alternative is<br />

implemented. Disbenefits may be indirect economic disadvantages of the alternative.<br />

It is difficult to estimate and agree upon the economic impact of benefits and disbenefits for a<br />

public sector alternative. For example, assume a short bypass around a congested area in town is<br />

recommended. How much will it benefit a driver in dollars per driving minute to be able to bypass<br />

five traffic lights while averaging 35 miles per hour, as compared to currently driving<br />

through the lights averaging 20 miles per hour and stopping at an average of two lights for an<br />

average of 45 seconds each? The bases and standards for benefits estimation are always difficult<br />

to establish and verify. Relative to revenue cash flow estimates in the private sector, benefit estimates<br />

are much harder to make, and vary more widely around uncertain averages. (The inability<br />

to make economic estimates for benefits may be overcome by using the evaluation technique<br />

discussed in Section 9.5.) And the disbenefits that accrue from an alternative are even harder to<br />

estimate. In fact, the disbenefit itself may not be known at the time the evaluation is performed.


232 Chapter 9 Benefit/Cost Analysis and Public Sector Economics<br />

Characteristic Public sector Private sector<br />

Funding<br />

Taxes, fees, bonds,<br />

private funds<br />

Stocks, bonds, loans,<br />

individual owners<br />

The capital used to finance public sector projects is commonly acquired from taxes, bonds,<br />

and fees. Taxes are collected from those who are the owners—the citizens (e.g., federal gasoline<br />

taxes for highways are paid by all gasoline users, and health care costs are covered by insurance<br />

premiums). This is also the case for fees, such as toll road fees for drivers. Bonds are often issued:<br />

U.S. Treasury bonds, municipal bond issues, and special-purpose bonds, such as utility district<br />

bonds. Private lenders can provide up-front financing. Also, private donors may provide funding<br />

for museums, memorials, parks, and garden areas through gifts.<br />

Characteristic Public sector Private sector<br />

Interest rate Lower Higher, based on cost of capital<br />

Because many of the financing methods for public sector projects are classified as low-interest,<br />

the interest rate is virtually always lower than for private sector alternatives. Government<br />

agencies are exempt from taxes levied by higher-level units. For example, municipal projects<br />

do not have to pay state taxes. (Private corporations and individual citizens do pay taxes.)<br />

Many loans are very low-interest, and grants with no repayment requirement from federal<br />

programs may share project costs. This results in interest rates in the 4% to 8% range. It is<br />

common that a government agency will direct that all projects be evaluated at a specific rate.<br />

As a matter of standardization, directives to use a specific interest rate are beneficial because<br />

different government agencies are able to obtain varying types of funding at different rates.<br />

This can result in projects of the same type being rejected in one state or city but accepted in<br />

another. Standardized rates tend to increase the consistency of economic decisions and to reduce<br />

gamesmanship.<br />

The determination of the interest rate for public sector evaluation is as important as the determination<br />

of the MARR for a private sector analysis. The public sector interest rate is identified<br />

as i ; however, it is referred to by other names to distinguish it from the private sector rate. The<br />

most common terms are discount rate and social discount rate.<br />

Characteristic Public sector Private sector<br />

Alternative selection Multiple criteria Primarily based on rate<br />

criteria<br />

of return<br />

Multiple categories of users, economic as well as noneconomic interests, and special-interest<br />

political and citizen groups make the selection of one alternative over another much more difficult<br />

in public sector economics. Seldom is it possible to select an alternative on the sole basis of<br />

a criterion such as PW or ROR. It is important to describe and itemize the criteria and selection<br />

method prior to the analysis. This helps determine the perspective or viewpoint when the evaluation<br />

is performed. Viewpoint is discussed below.<br />

Characteristic Public sector Private sector<br />

Environment of the evaluation Politically inclined Primarily economic<br />

There are often public meetings and debates associated with public sector projects to accommodate<br />

the various interests of citizens (owners). Elected officials commonly assist with the selection,<br />

especially when pressure is brought to bear by voters, developers, environmentalists, and<br />

others. The selection process is not as “clean” as in private sector evaluation.<br />

The viewpoint of the public sector analysis must be determined before cost, benefit, and disbenefit<br />

estimates are made and before the evaluation is formulated and performed. There are<br />

several viewpoints for any situation, and the different perspectives may alter how a cash flow<br />

estimate is classified.


9.1 Public Sector Projects 233<br />

Some example perspectives are the citizen; the city tax base; number of students in the school district;<br />

creation and retention of jobs; economic development potential; a particular industry interest<br />

(agriculture, banking, electronics manufacturing); even the reelection of a public officeholder (often<br />

termed pork projects ). In general, the viewpoint of the analysis should be as broadly defined as those<br />

who will bear the costs of the project and reap its benefits. Once established, the viewpoint assists in<br />

categorizing the costs, benefits, and disbenefits of each alternative, as illustrated in Example 9.1 .<br />

EXAMPLE 9.1 Water Treatment Facility #3 Case<br />

The situation with the location and construction of the new WTF3 and associated transmission<br />

mains described in the chapter’s introduction has reached a serious level because of recent<br />

questions posed by some city council members and citizen groups. Before going public to the<br />

city council with the analysis performed last year, the director of Allen Water Utilities has<br />

asked an engineering management consultant to review it and determine if it was an acceptable<br />

analysis and correct economic decision, then and now. The lead consultant, Joel Whiterson,<br />

took engineering economy as a part of his B.S. education and has previously worked on<br />

economic studies in the government sector, but never as the lead person.<br />

Within the first hour of checking background notes, Joel found several initial estimates<br />

(shown below) from last year for expected consequences if WTF3 were built. He realized that<br />

no viewpoint of the study was defined, and, in fact, the estimates were never classified as costs,<br />

benefits, or disbenefits. He did determine that disbenefits were considered at some point in the<br />

analysis, though the estimates for them are very sketchy.<br />

Joel defined two viewpoints: a citizen of Allen and the Allen Water Utilities budget. He<br />

wants to identify each of the estimates as a cost, benefit, or disbenefit from each viewpoint.<br />

Please help with this classification.<br />

Economic Dimension<br />

1. Cost of water: 10% annual increase to Allen<br />

households<br />

2. Bonds: Annual debt service at 3% per year on<br />

$540 million<br />

3. Use of land: Payment to Parks and Recreation<br />

for shaft sites and construction areas<br />

4. Property values: Loss in value, sales price,<br />

and property taxes<br />

5. Water sales: Increases in sales to surrounding<br />

communities<br />

6. M&O: Annual maintenance and operations<br />

costs<br />

7. Peak load purchases: Savings in purchases of<br />

treated water from secondary sources<br />

Monetary Estimate<br />

Average of $29.7 million (years 1–5, steady<br />

thereafter)<br />

$16.2 million (years 1–19); $516.2 million (year 20)<br />

$300,000 (years 1–4)<br />

$4 million (years 1–5)<br />

$5 million (year 4) plus 5% per year (years 5–20)<br />

$300,000 plus 4% per year increase (years 1–20)<br />

$500,000 (years 5–20)<br />

PE<br />

Solution<br />

The perspective of each viewpoint is identified and estimates are classified. (How this classification<br />

is done will vary depending upon who does the analysis. This solution offers only one<br />

logical answer.)<br />

Viewpoint 1: Citizen of the city of Allen. Goal: Maximize the quality of life and wellness of<br />

citizens with family and neighborhood as prime concerns.<br />

Costs: 1, 2, 4, 6 Benefits: 5, 7 Disbenefits: 3<br />

Viewpoint 2: Allen Water Utilities budget. Goal: Ensure the budget is balanced and of sufficient<br />

size to fund rapidly growing city service demands.<br />

Costs: 2, 3, 6 Benefits: 1, 5, 7 Disbenefits: 4<br />

Citizens view costs in a different light than a city budget employee does. For example, the loss<br />

of property values (item 4) is considered a real cost to a citizen, but is an unfortunate disbenefit


234 Chapter 9 Benefit/Cost Analysis and Public Sector Economics<br />

from the city budget perspective. Similarly, the Allen Water Utilities budget interprets estimate<br />

3 (payment for use of land to Parks and Recreation) as a real cost; but a citizen might interpret<br />

this as merely a movement of funds between two municipal budgets—therefore, it is a disbenefit,<br />

not a real cost.<br />

Comment<br />

The inclusion of disbenefits can easily change the economic decision. However, agreement on<br />

the disbenefits and their monetary estimates is difficult (to impossible) to develop, often resulting<br />

in the exclusion of any disbenefits from the economic analysis. Unfortunately, this usually<br />

transfers the consideration of disbenefits to the noneconomic (i.e., political) realm of public<br />

project decision making.<br />

Most of the large public sector projects are developed through public-private partnerships<br />

(PPPs). A partnership is advantageous in part because of the greater efficiency of the private sector<br />

and in part because of the sizable cost to design, construct, and operate such projects. Full<br />

funding by the government unit may not be possible using traditional means—fees, taxes, and<br />

bonds. Some examples of the projects are as follows:<br />

Project<br />

Mass transportation<br />

Bridges and tunnels<br />

Ports and harbors<br />

Airports<br />

Water resources<br />

Some Purposes of the Project<br />

Reduce transit time; reduce congestion; improve environment; decrease road<br />

accidents<br />

Speed traffic flows; reduce congestion; improve safety<br />

Increase cargo capacity; support industrial development; increase tourism<br />

Increase capacity; improve passenger safety; support development<br />

Desalination for drinking water; meet irrigation and industrial needs; improve<br />

wastewater treatment<br />

In these joint ventures, the public sector (government) is responsible for the funding and service<br />

to the citizenry, and the private sector partner (corporation) is responsible for varying aspects of<br />

the projects as detailed below. The government unit cannot make a profit, but the corporation(s)<br />

involved can realize a reasonable profit; in fact, the profit margin is usually written into the contract<br />

that governs the design, construction, and operation of the project.<br />

Traditional methods of contracting were fi xed-price (historically called lump-sum) and cost<br />

reimbursable (also called cost-plus). In these formats, a government unit took responsibility for<br />

funding and possibly some of the design elements, and later all operation activities, while the<br />

contractor did not share in the risks involved—liability, natural disasters, funding shortfalls, etc.<br />

More recently, the PPP has become the arrangement of choice for most large public projects.<br />

Commonly these are called design-build contracts, under which contractors take on more and<br />

more of the functions from design to operation. Details are explained in publications such as<br />

Design-Build Contracting Handbook (Cushman and Loulakis). The most reliance is placed upon<br />

a contractor or contractors with a DBOMF contract, as described below.<br />

The Design-Build-Operate-Maintain-Finance (DBOMF) contract is considered a turnkey approach<br />

to a project. It requires the contractor(s) to perform all the DBOMF activities with collaboration<br />

and approval of the owner (the government unit). The activity of financing is the<br />

management of cash fl ow to support project implementation by a contracting firm. Although a<br />

contractor may assist in some instances, the funding (obtaining the capital funds) remains the<br />

government’s responsibility through bonding, commercial loans, taxation, grants, and gifts.<br />

When the financing activity is not managed by a contractor, the contract is a DBOM; it is also<br />

common to develop a design-build contract. In virtually all cases, some forms of design-build<br />

arrangements for public projects are made because they offer several advantages to the government<br />

and citizens served:<br />

• Cost and time savings in the design, build, and operate phases<br />

• Earlier and more reliable (less variable) cost estimates


9.2 Benefit/Cost Analysis of a Single Project 235<br />

• Reduced administrative responsibilities for the owner<br />

• Better efficiency of resource allocation of private enterprise<br />

• Environmental, liability, and safety issues addressed by the private sector, where there usually<br />

is greater expertise<br />

Many of the projects in international settings and in developing countries utilize the publicprivate<br />

partnership. There are, of course, disadvantages to this arrangement. One risk is that the<br />

amount of funding committed to the project may not cover the actual build cost because it is<br />

considerably higher than estimated. Another risk is that a reasonable profit may not be realized<br />

by the private corporation due to low usage of the facility during the operate phase. To prevent<br />

such problems, the original contract may provide for special subsidies and loans guaranteed by<br />

the government unit. The subsidy may cover costs plus (contractually agreed-to) profit if usage<br />

is lower than a specified level. The level used may be the breakeven point with the agreed-to<br />

profit margin considered.<br />

9.2 Benefit/Cost Analysis of a Single Project<br />

The benefit/cost ratio is relied upon as a fundamental analysis method for public sector projects.<br />

The B/C analysis was developed to introduce greater objectivity into public sector economics,<br />

and as one response to the U.S. Congress approving the Flood Control Act of 1936. There are<br />

several variations of the B/C ratio; however, the fundamental approach is the same. All cost and<br />

benefit estimates must be converted to a common equivalent monetary unit (PW, AW, or FW) at<br />

the discount rate (interest rate). The B/C ratio is then calculated using one of these relations:<br />

B/C ———————<br />

PW of benefits<br />

———————<br />

AW of benefits<br />

———————<br />

FW of benefits<br />

PW of costs AW of costs FW of costs<br />

[9.1]<br />

Present worth and annual worth equivalencies are preferred to future worth values. The sign<br />

convention for B/C analysis is positive signs; costs are preceded by a sign. Salvage values<br />

and additional revenues to the government, when they are estimated, are subtracted from costs in<br />

the denominator. Disbenefits are considered in different ways depending upon the model used.<br />

Most commonly, disbenefits are subtracted from benefits and placed in the numerator. The<br />

different formats are discussed below.<br />

The decision guideline is simple:<br />

If B/C 1.0, accept the project as economically justified for the estimates and discount rate<br />

applied.<br />

If B/C 1.0, the project is not economically acceptable.<br />

Project evaluation<br />

If the B/C value is exactly or very near 1.0, noneconomic factors will help make the decision.<br />

The conventional B/C ratio, probably the most widely used, is calculated as follows:<br />

B/C ——————————<br />

benefits disbenefits<br />

costs<br />

B D ———<br />

C<br />

[9.2]<br />

In Equation [9.2] disbenefits are subtracted from benefits, not added to costs. The B/C value<br />

could change considerably if disbenefits are regarded as costs. For example, if the numbers<br />

10, 8, and 5 are used to represent the PW of benefits, disbenefits, and costs, respectively, the<br />

correct procedure results in B/C (10 8)5 0.40. The incorrect placement of disbenefits<br />

in the denominator results in B/C 10(8 5) 0.77, which is approximately twice the correct<br />

B/C value of 0.40. Clearly, then, the method by which disbenefits are handled affects the<br />

magnitude of the B/C ratio. However, regardless of whether disbenefits are (correctly) subtracted<br />

from the numerator or (incorrectly) added to costs in the denominator, a B/C ratio of<br />

less than 1.0 by the first method will always yield a B/C ratio less than 1.0 by the second<br />

method, and vice versa.<br />

The modified B/C ratio includes all the estimates associated with the project, once operational.<br />

Maintenance and operation (M&O) costs are placed in the numerator and treated in a manner


236 Chapter 9 Benefit/Cost Analysis and Public Sector Economics<br />

similar to disbenefits. The denominator includes only the initial investment. Once all amounts are<br />

expressed in PW, AW, or FW terms, the modified B/C ratio is calculated as<br />

Modified B/C ————————————————<br />

benefits disbenefits M&O costs<br />

initial investment<br />

[9.3]<br />

Salvage value is usually included in the denominator as a negative cost. The modified B/C ratio<br />

will obviously yield a different value than the conventional B/C method. However, as with disbenefits,<br />

the modifi ed procedure can change the magnitude of the ratio but not the decision to<br />

accept or reject the project.<br />

The benefit and cost difference measure of worth, which does not involve a ratio, is based on<br />

the difference between the PW, AW, or FW of benefits and costs, that is, B − C . If ( B − C ) 0,<br />

the project is acceptable. This method has the advantage of eliminating the discrepancies noted<br />

above when disbenefits are regarded as costs, because B represents net benefi ts. Thus, for the<br />

numbers 10, 8, and 5, the same result is obtained regardless of how disbenefits are treated.<br />

Subtracting disbenefits from benefits: B C (10 8) 5 3<br />

Adding disbenefits to costs:<br />

B C 10 (8 5) 3<br />

Before calculating the B/C ratio by any formula, check whether the alternative with the larger<br />

AW or PW of costs also yields a larger AW or PW of benefits. It is possible for one alternative<br />

with larger costs to generate lower benefits than other alternatives, thus making it unnecessary to<br />

further consider the larger-cost alternative.<br />

By the very nature of benefits and especially disbenefits, monetary estimates are difficult to<br />

make and will vary over a wide range. The extensive use of sensitivity analysis on the more<br />

questionable parameters helps determine how sensitive the economic decision is to estimate<br />

variation. This approach assists in determining the economic and public acceptance risk associated<br />

with a defined project. Also, the use of sensitivity analysis can alleviate some of the public’s<br />

concerns commonly expressed that people (managers, engineers, consultants, contractors,<br />

and elected officials) designing (and promoting) the public project are narrowly receptive to different<br />

approaches to serving the public’s interest.<br />

EXAMPLE 9.2<br />

In the past, the Afram Foundation has awarded many grants to improve the living and medical<br />

conditions of people in war-torn and poverty-stricken countries throughout the world. In a<br />

proposal for the foundation’s board of directors to construct a new hospital and medical clinic<br />

complex in a deprived central African country, the project manager has developed some estimates.<br />

These are developed, so she states, in a manner that does not have a major negative<br />

effect on prime agricultural land or living areas for citizens.<br />

Award amount: $20 million (end of) first year, decreasing by $5 million per year for 3<br />

additional years; local government will fund during the first year only<br />

Annual costs: $2 million per year for 10 years, as proposed<br />

Benefits: Reduction of $8 million per year in health-related expenses for citizens<br />

Disbenefits: $0.1 to $0.6 million per year for removal of arable land and commercial<br />

districts<br />

Use the conventional and modified B/C methods to determine if this grant proposal is economically<br />

justified over a 10-year study period. The foundation’s discount rate is 6% per year.<br />

Solution<br />

Initially, determine the AW for each parameter over 10 years. In $1 million units,<br />

Award:<br />

Annual costs:<br />

Benefits:<br />

Disbenefits:<br />

20 − 5(A/G,6%,4) $12.864 per year<br />

$2 per year<br />

$8 per year<br />

Use $0.6 for the first analysis


9.2 Benefit/Cost Analysis of a Single Project 237<br />

The conventional B/C analysis applies Equation [9.2].<br />

B/C —————— 8.0 0.6<br />

12.864 2.0 0.50<br />

The modifi ed B/C analysis uses Equation [9.3].<br />

Modified B/C ———————<br />

8.0 0.6 2.0<br />

0.42<br />

12.864<br />

The proposal is not justified economically since both measures are less than 1.0. If the low<br />

disbenefits estimate of $0.1 million per year is used, the measures increase slightly, but not<br />

enough to justify the proposal.<br />

It is possible to develop a direct formula connection between the B/C of a public sector and<br />

B/C of a private sector project that is a revenue alternative ; that is, both revenues and costs are<br />

estimated. Further, we can identify a direct correspondence between the modified B/C relation in<br />

Equation [9.3] and the PW method we have used repeatedly. (The following development also<br />

applies to AW or FW values.) Let’s neglect the initial investment in year 0 for a moment, and<br />

concentrate on the cash flows of the project for year 1 through its expected life. For the private<br />

sector, the PW for project cash flows is<br />

PW of project PW of revenue PW of costs<br />

Since private sector revenues are approximately the same as public sector benefits minus disbenefits<br />

(B D), the modified B/C relation in Equation [9.3] may be written as<br />

Modified B/C <br />

PW of (B D) PW of C<br />

————————————<br />

PW of initial investment<br />

This relation can be slightly rewritten to form the profitability index (PI), which can be used to<br />

evaluate revenue projects in the public or private sector. For years t 1, 2, . . . , n ,<br />

PI <br />

PW of NCF t<br />

———————————<br />

PW of initial investment<br />

[9.4]<br />

Note that the denominator includes only first cost (initial investment) items, while the numerator<br />

has only cash flows that result from the project for years 1 through its life. The PI measure of<br />

worth provides a sense of getting the most for the investment dollar (euro, yen, etc.). That is, the<br />

result is in PW units per PW of money invested at the beginning. This is a “bang for the buck”<br />

measure. When used solely for a private sector project, the disbenefits are usually omitted,<br />

whereas they should be estimated and included in the modified B/C version of this measure for a<br />

public project.<br />

The evaluation guideline for a single project using the PI is the same as for the conventional<br />

B/C or modified B/C.<br />

If PI 1.0, the project is economically acceptable at the discount rate.<br />

If PI 1.0, the project is not economically acceptable at the discount rate.<br />

Remember, the computations for PI and modified B/C are essentially the same, except the PI is<br />

usually applied without disbenefits estimated. The PI has another name: the present worth index<br />

(PWI). It is often used to rank and assist in the selection of independent projects when the capital<br />

budget is limited. This application is discussed in Chapter 12, Section 12.5.<br />

Project evaluation<br />

EXAMPLE 9.3<br />

The Georgia Transportation Directorate is considering a public-private partnership with Young<br />

Construction as the prime contractor using a DBOMF contract for a new 22.51-mile toll road on<br />

the outskirts of Atlanta’s suburban area. The design includes three 4-mile-long commercial/<br />

retail corridors on both sides of the toll road. Highway construction is expected to require


238 Chapter 9 Benefit/Cost Analysis and Public Sector Economics<br />

5 years at an average cost of $3.91 million per mile. The discount rate is 4% per year, and the<br />

study period is 30 years. Evaluate the economics of the proposal using (a) the modified B/C<br />

analysis from the State of Georgia perspective and (b) the profitability index from the Young<br />

corporate viewpoint in which disbenefits are not included.<br />

Initial investment: $88 million distributed over 5 years; $4 million now and in year 5 and<br />

$20 million in each of years 1 through 4.<br />

Annual M&O cost: $1 million per year, plus an additional $3 million each fifth year, including<br />

year 30.<br />

Annual revenue/benefits: Include tolls and retail/commercial growth; start at $2 million in<br />

year 1, increasing by a constant $0.5 million annually through year 10, and then increasing by<br />

a constant $1 million per year through year 20 and remaining constant thereafter.<br />

Estimable disbenefits: Include loss of business income, taxes, and property value in surrounding<br />

areas; start at $10 million in year 1, decrease by $0.5 million per year through year<br />

21, and remain at zero thereafter.<br />

Solution<br />

The PW values in year 0 for all estimates must be developed initially usually by hand, calculator,<br />

or spreadsheet computations. If the 30 years of estimates are entered into a spreadsheet and<br />

NPV functions at 4% are applied, the results in $1 million units are obtained. All values are<br />

positive because of the sign convention for B/C and PI measures.<br />

PW of investment $71.89 PW of benefits $167.41<br />

PW of costs $26.87 PW of disbenefits $80.12<br />

(a) From the public project perspective, the State will apply Equation [9.3].<br />

Modified B/C ——————————<br />

167.41 80.12 26.87<br />

0.84<br />

71.89<br />

The toll road proposal is not economically acceptable, since B/C 1.0.<br />

(b) From the private corporation viewpoint, Young Construction will apply Equation [9.4].<br />

PI ———————<br />

167.41 26.87<br />

1.95<br />

71.89<br />

The proposal is clearly justified without the disbenefits, since PI > 1.0. The private project<br />

perspective predicts that every investment dollar will return an equivalent of $1.95 over<br />

30 years at a 4% per year discount rate.<br />

Comment<br />

The obvious question that arises concerns the correct measure to use. When PI is used in the private<br />

project setting, there is no problem, since disbenefits are virtually never considered in the<br />

economic analysis. The public project setting will commonly use some form of the B/C ratio with<br />

the disbenefit considered. When a public-private partnership is initiated, there should be some<br />

agreement beforehand that establishes the economic measure acceptable for analysis and decision<br />

making throughout the project. Then the numerical dilemma presented above should not occur.<br />

9.3 Alternative Selection Using Incremental<br />

B/C Analysis<br />

The technique to compare two mutually exclusive alternatives using benefit/cost analysis is virtually<br />

the same as that for incremental ROR in Chapter 8. The incremental (conventional) B/C ratio,<br />

which is identified as B/C, is determined using PW, AW, or FW calculations. The higher-cost<br />

alternative is justified if B/C is equal to or larger than 1.0. The selection rule is as follows:<br />

ME alternative<br />

selection<br />

If B/C 1.0, choose the higher-cost alternative, because its extra cost is economically justified.<br />

If B/C 1.0, choose the lower-cost alternative.


9.3 Alternative Selection Using Incremental B/C Analysis 239<br />

To perform a correct incremental B/C analysis, it is required that each alternative be compared<br />

only with another alternative for which the incremental cost is already justified. This same rule<br />

was used for incremental ROR analysis.<br />

There are two dimensions of an incremental B/C analysis that differ from the incremental<br />

ROR method in Chapter 8. We already know the first, all costs have a positive sign in the B/C<br />

ratio. The second, and significantly more important, concerns the ordering of alternatives prior to<br />

incremental analysis.<br />

Alternatives are ordered by increasing equivalent total costs, that is, PW or AW of all cost<br />

estimates that will be utilized in the denominator of the B/C ratio. When not done correctly, the<br />

incremental B/C analysis may reject a justified higher-cost alternative.<br />

If two alternatives, A and B, have equal initial investments and lives, but B has a larger equivalent<br />

annual cost, then B must be incrementally justified against A. (This is illustrated in Example 9.4<br />

below.) If this convention is not correctly followed, it is possible to get a negative cost value in<br />

the denominator, which can incorrectly make B/C 1 and reject a higher-cost alternative that is<br />

actually justified.<br />

Follow these steps to correctly perform a conventional B/C ratio analysis of two alternatives.<br />

Equivalent values can be expressed in PW, AW, or FW terms.<br />

1. Determine the equivalent total costs for both alternatives.<br />

2. Order the alternatives by equivalent total cost: first smaller, then larger. Calculate the incremental<br />

cost (C) for the larger-cost alternative. This is the denominator in B/C.<br />

3. Calculate the equivalent total benefits and any disbenefits estimated for both alternatives.<br />

Calculate the incremental benefits (B) for the larger-cost alternative. This is (B D) if<br />

disbenefits are considered.<br />

4. Calculate the B/C ratio using Equation [9.2], (B D)/C.<br />

5. Use the selection guideline to select the higher-cost alternative if B/C 1.0.<br />

When the B/C ratio is determined for the lower-cost alternative, it is a comparison with the donothing<br />

(DN) alternative. If B/C 1.0, then DN should be selected and compared to the second<br />

alternative. If neither alternative has an acceptable B/C value and one of the alternatives does not<br />

have to be selected, the DN alternative must be selected. In public sector analysis, the DN alternative<br />

is usually the current condition.<br />

EXAMPLE 9.4<br />

The city of Garden Ridge, Florida, has received designs for a new patient room wing to the<br />

municipal hospital from two architectural consultants. One of the two designs must be accepted<br />

in order to announce it for construction bids. The costs and benefits are the same in most<br />

categories, but the city financial manager decided that the estimates below should be considered<br />

to determine which design to recommend at the city council meeting next week and to<br />

present to the citizenry in preparation for an upcoming bond referendum next month.<br />

Design A<br />

Design B<br />

Construction cost, $ 10,000,000 15,000,000<br />

Building maintenance cost, $/year 35,000 55,000<br />

Patient usage copay, $/year 450,000 200,000<br />

The patient usage copay is an estimate of the amount paid by patients over the insurance coverage<br />

generally allowed for a hospital room. The discount rate is 5%, and the life of the building is<br />

estimated at 30 years.<br />

(a) Use incremental B/C analysis to select design A or B.<br />

(b) Once the two designs were publicized, the privately owned hospital in the directly adjacent<br />

city of Forest Glen lodged a complaint that design A will reduce its own municipal hospital’s<br />

income by an estimated $500,000 per year because some of the day-surgery features<br />

of design A duplicate its services. Subsequently, the Garden Ridge merchants’ association<br />

argued that design B could reduce its annual revenue by an estimated $400,000, because it


240 Chapter 9 Benefit/Cost Analysis and Public Sector Economics<br />

will eliminate an entire parking lot used by their patrons for short-term parking. The city<br />

financial manager stated that these concerns would be entered into the evaluation as disbenefits<br />

of the respective designs. Redo the B/C analysis to determine if the economic decision<br />

is still the same as when disbenefits were not considered.<br />

Solution<br />

( a) Since most of the cash flows are already annualized, the incremental B/C ratio will use<br />

AW values. No disbenefit estimates are considered. Follow the steps of the procedure<br />

above:<br />

1. The AW of costs is the sum of construction and maintenance costs.<br />

AW A 10,000,000(A/P,5%,30) 35,000 $685,500<br />

AW B 15,000,000(A/P,5%,30) 55,000 $1,030,750<br />

2. Design B has the larger AW of costs, so it is the alternative to be incrementally justified.<br />

The incremental cost is<br />

C AW B AW A $345,250 per year<br />

3. The AW of benefits is derived from the patient usage copays, since these are consequences<br />

to the public. The benefits for the B/C analysis are not the estimates themselves,<br />

but the difference if design B is selected. The lower usage copay is a positive<br />

benefit for design B.<br />

B usage A usage B $450,000 $200,000 $250,000 per year<br />

4. The incremental B/C ratio is calculated by Equation [9.2].<br />

BC ————<br />

$250,000<br />

$345,250 0.72<br />

5. The B/C ratio is less than 1.0, indicating that the extra costs associated with design B<br />

are not justified. Therefore, design A is selected for the construction bid.<br />

(b) The revenue loss estimates are considered disbenefits. Since the disbenefits of design B are<br />

$100,000 less than those of A, this positive difference is added to the $250,000 benefits of<br />

B to give it a total benefit of $350,000. Now<br />

BC ————<br />

$350,000<br />

$345,250 1.01<br />

Design B is slightly favored. In this case the inclusion of disbenefits has reversed the previous<br />

economic decision. This has probably made the situation more difficult politically.<br />

New disbenefits will surely be claimed in the near future by other special-interest groups.<br />

Like other methods, incremental B/C analysis requires equal-service comparison of alternatives.<br />

Usually, the expected useful life of a public project is long (25 or 30 or more years), so<br />

alternatives generally have equal lives. However, when alternatives do have unequal lives, the<br />

use of PW or AW to determine the equivalent costs and benefits requires that the LCM of lives be<br />

used to calculate B/C. As with ROR analysis of two alternatives, this is an excellent opportunity<br />

to use the AW equivalency of estimated (not incremental) costs and benefits, if the implied assumption<br />

that the project could be repeated is reasonable. Therefore, use AW-based analysis of<br />

actual costs and benefits for B/C ratios when different-life alternatives are compared.<br />

EXAMPLE 9.5 Water Treatment Facility #3 Case<br />

PE<br />

As our case unfolds, the consultant, Joel Whiterson, has pieced together some of the B/C<br />

analysis estimates for the 84-inch Jolleyville transmission main study completed last year. The<br />

two options for constructing this main were open trench (OT) for the entire 6.8-mile distance<br />

or a combination of trenching and bore tunneling (TT) for a shorter route of 6.3 miles. One of<br />

the two options had to be selected to transport approximately 300 million gallons per day (gpd)<br />

of treated water from the new WTF3 to an existing aboveground reservoir.


9.3 Alternative Selection Using Incremental B/C Analysis 241<br />

The general manager of Allen Water Utilities has stated publicly several times that the<br />

trench-tunnel combination option was selected over the open-trench alternative based on analysis<br />

of both quantitative and nonquantitative data. He stated the equivalent annual costs in an<br />

internal e-mail some months ago, based on the expected construction periods of 24 and<br />

36 months, respectively, as equivalent to<br />

AW OT $1.20 million per year<br />

AW TT $2.37 million per year<br />

This analysis indicated that the open-trench option was economically better, at that time. The<br />

planning horizon for the transmission mains is 50 years; this is a reasonable study period, Joel<br />

concluded. Use the estimates below that Joel has unearthed to perform a correct incremental<br />

B/C analysis and comment on the results. The interest (discount) rate is 3% per year, compounded<br />

annually, and 1 mile is 5280 feet.<br />

Open trench (OT)<br />

Trench-tunnel (TT)<br />

Distance, miles 6.8 6.3<br />

First cost, $ per foot 700 Trench for 2.0 miles: 700<br />

Tunnel for 4.3 miles: 2100<br />

Time to complete, months 24 36<br />

Construction support costs, $ per month 250,000 175,000<br />

Ancillary expenses, $ per month:<br />

Environmental 150,000 20,000<br />

Safety 140,000 60,000<br />

Community interface 20,000 5,000<br />

Solution<br />

One of the alternatives must be selected, and the construction lives are unequal. Since it is not<br />

reasonable to assume that this construction project will be repeated many cycles in the future,<br />

it is incorrect to conduct an AW analysis over the respective completion periods of 24 and<br />

36 months, or the LCM of these time periods. However, the study period of 50 years is a reasonable<br />

evaluation time frame, since the mains are considered permanent installations. We can<br />

assume that the construction first costs are a present worth value in year 0, but the equivalent<br />

PW and 50-year AW of other monthly costs must be determined.<br />

PW OT PW of construction PW of construction support costs<br />

700(6.8)(5280) 250,000(12)( PA ,3%,2)<br />

$30,873,300<br />

AW OT 30,873,300( AP ,3%,50)<br />

$1.20 million per year<br />

PW TT [700(2.0) 2100(4.3)](5280) 175,000(12)( PA ,3%,3)<br />

$61,010,460<br />

AW TT 61,010,460( AP ,3%,50)<br />

$2.37 million per year<br />

The trench-tunnel (TT) alternative has a larger equivalent cost ; it must be justified against the<br />

OT alternative. The incremental cost is<br />

C AW TT AW OT 2.37 − 1.20 $1.17 million per year<br />

The difference between ancillary expenses defines the incremental benefit for TT.<br />

PW OT-anc<br />

310,000(12)( PA ,3%,2)<br />

$7,118,220<br />

AW OT-anc 7,118,220( AP ,3%,50)<br />

$276,685 per year


242 Chapter 9 Benefit/Cost Analysis and Public Sector Economics<br />

PW TT-anc<br />

85,000(12)( PA ,3%,3)<br />

$2,885,172<br />

AW TT-anc 2,885,172( AP ,3%,50)<br />

$112,147 per year<br />

B AW OT-anc AW TT-anc 276,685 112,147 $164,538 per year ($0.16 million)<br />

Calculate the incremental B/C ratio.<br />

B/C 0.16/1.17 0.14<br />

Since B/C 1.0, the trench-tunnel option is not economically justified. Joel can now conclude<br />

that the general manager’s earlier comment that the TT option was selected based on<br />

quantitative and nonquantitative data must have had heavy dependence on nonquantitative<br />

information not yet discovered.<br />

9.4 Incremental B/C Analysis of Multiple, Mutually<br />

Exclusive Alternatives<br />

The procedure necessary to select one from three or more mutually exclusive alternatives using<br />

incremental B/C analysis is essentially the same as that of Section 9.3. The procedure also parallels<br />

that for incremental ROR analysis in Section 8.6. The selection guideline is as follows:<br />

Choose the largest-cost alternative that is justified with an incremental B/C 1.0 when this selected<br />

alternative has been compared with another justified alternative.<br />

There are two types of benefit estimates—estimation of direct benefits, and implied benefits<br />

based on usage cost estimates. The previous two examples (9.4 and 9.5) are good illustrations of<br />

the second type of implied benefit estimation. When direct benefi ts are estimated, the B/C ratio<br />

for each alternative may be calculated first as an initial screening mechanism to eliminate unacceptable<br />

alternatives. At least one alternative must have B/C 1.0 to perform the incremental<br />

B/C analysis. If all alternatives are unacceptable, the DN alternative is the choice. (This is the<br />

same approach as that of step 2 for “revenue alternatives only” in the ROR procedure of Section<br />

8.6. However, the term revenue alternative is not applicable to public sector projects.)<br />

As in the previous section when comparing two alternatives, selection from multiple alternatives<br />

by incremental B/C ratio utilizes equivalent total costs to initially order alternatives from<br />

smallest to largest. Pairwise comparison is then undertaken. Also, remember that all costs are<br />

considered positive in B/C calculations. The terms defender and challenger alternative are used<br />

in this procedure, as in a ROR-based analysis. The procedure for incremental B/C analysis of<br />

multiple alternatives is as follows:<br />

1. Determine the equivalent total cost for all alternatives. Use AW, PW, or FW equivalencies.<br />

2. Order the alternatives by equivalent total cost, smallest first.<br />

3. Determine the equivalent total benefits (and any disbenefits estimated) for each alternative.<br />

4. Direct benefi ts estimation only: Calculate the B/C for the first ordered alternative. If B/C<br />

1.0, eliminate it. By comparing each alternative to DN in order, we eliminate all that have<br />

B/C 1.0. The lowest-cost alternative with B/C 1.0 becomes the defender and the next<br />

higher-cost alternative is the challenger in the next step. (For analysis by spreadsheet, determine<br />

the B/C for all alternatives initially and retain only acceptable ones.)<br />

5. Calculate incremental costs ( C ) and benefits ( B ) using the relations<br />

C challenger cost defender cost<br />

B challenger benefits defender benefits<br />

If relative usage costs are estimated for each alternative, rather than direct benefits, B may<br />

be found using the relation<br />

B defender usage costs challenger usage costs [9.5]


9.4 Incremental B/C Analysis of Multiple, Mutually Exclusive Alternatives 243<br />

6. Calculate the B/C for the first challenger compared to the defender.<br />

B/C B C [9.6]<br />

If B/C 1.0 in Equation [9.6], the challenger becomes the defender and the previous defender<br />

is eliminated. Conversely, if B/C 1.0, remove the challenger and the defender<br />

remains against the next challenger.<br />

7. Repeat steps 5 and 6 until only one alternative remains. It is the selected one.<br />

In all the steps above, incremental disbenefits may be considered by replacing B with ( B D ).<br />

EXAMPLE 9.6<br />

Schlitterbahn Waterparks of Texas, a very popular water and entertainment park headquartered<br />

in New Braunfels, has been asked by four different cities outside of Texas to consider<br />

building a park in their area. All the offers include some version of the following<br />

incentives:<br />

• Immediate cash incentive (year 0)<br />

• A 10% of first-year incentive as a direct property tax reduction for 8 years<br />

• Sales tax rebate sharing plan for 8 years<br />

• Reduced entrance (usage) fees for area residents for 8 years<br />

Table 9–1 (top section) summarizes the estimates for each proposal, including the present<br />

worth of the initial construction cost and anticipated annual revenue. The annual M&O costs<br />

are expected to be the same for all locations. Use incremental B/C analysis at 7% per year and<br />

an 8-year study period to advise the board of directors if they should consider any of the offers<br />

to be economically attractive.<br />

Solution<br />

The viewpoint is that of Schlitterbahn, and the benefits are direct estimates. Develop the AW<br />

equivalents over 8 years, and use the procedure detailed above. The results are presented in<br />

Table 9–1.<br />

1. AW of total costs and an example for city 1 are determined in $1 million units.<br />

AW of costs first cost(AP,7%,8) entrance fee reduction to residents<br />

38.5(0.16747) 0.5<br />

$6.948 ($6,948,000 per year)<br />

2. The four alternatives are correctly ordered by increasing equivalent total cost in<br />

Table 9–1.<br />

TABLE 9–1 Incremental B/C Analysis of Water Park Proposals, Example 9.6<br />

City 1 City 2 City 3 City 4<br />

First cost, $ million 38.5 40.1 45.9 60.3<br />

Entrance fee costs, $/year 500,000 450,000 425,000 250,000<br />

Annual revenue, $ million/year 7.0 6.2 10.0 10.4<br />

Initial cash incentive, $ 250,000 350,000 500,000 800,000<br />

Property tax reduction, $/year 25,000 35,000 50,000 80,000<br />

Sales tax sharing, $/year 310,000 320,000 320,000 340,000<br />

AW of total costs, $ million/year 6.948 7.166 8.112 10.348<br />

AW of total benefits, $ million/year 7.377 6.614 10.454 10.954<br />

Overall B/C 1.06 0.92 1.29 1.06<br />

Alternatives compared 1-to-DN B/C 1.0 3-to-1 4-to-3<br />

Incremental costs C, $/year 6.948 1.164 2.236<br />

Incremental benefits B, $/year 7.377 3.077 0.50<br />

B/C 1.06 2.64 0.22<br />

Increment justified? Yes Eliminated Yes No<br />

City selected 1 3 3


244 Chapter 9 Benefit/Cost Analysis and Public Sector Economics<br />

3. AW of total benefits and an example for city 1 are also determined in $1 million units.<br />

AW of benefits revenue initial incentive(AP,7%,8)<br />

property tax reduction sales tax sharing<br />

7.0 0.25(0.16747) 0.025 0.31<br />

$7.377 ($7,377,000 per year)<br />

4. Since benefits are directly estimated (and no disbenefits are included), determine the overall<br />

B/C for each alternative using Equation [9.1]. In the case of city 1,<br />

BC 1 7.3776.948 1.06<br />

City 2 is eliminated with B/C 2 0.92; the rest are initially acceptable.<br />

5. The C and B values are the actual estimates for the 1-to-DN comparison.<br />

6. The overall B/C is the same as B/C 1.06, using Equation [9.6]. City 1 is economically<br />

justified and becomes the defender.<br />

7. Repeat steps 5 and 6. Since city 2 is eliminated, the 3-to-1 comparison results in<br />

C 8.112 6.948 1.164<br />

B 10.454 7.377 3.077<br />

BC 3.0771.164 2.64<br />

City 3 is well justified and becomes the defender against city 4. From Table 9–1, B/C 0.22<br />

for the 4-to-3 comparison. City 4 falls out easily, and city 3 is the one to recommend to the<br />

board. Note that the DN alternative could have been selected had no proposal met the B/C or<br />

B/C requirements.<br />

Independent project<br />

selection<br />

When two or more independent projects are evaluated using B/C analysis and there is no<br />

budget limitation, no incremental comparison is necessary. The only comparison is between<br />

each project separately with the do-nothing alternative. The project B/C values are calculated,<br />

and those with B/C 1.0 are accepted.<br />

This is the same procedure as that used to select from independent projects using the ROR method<br />

(Chapter 8). When a budget limitation is imposed, the capital budgeting procedure discussed in<br />

Chapter 12 must be applied.<br />

When the lives of mutually exclusive alternatives are so long that they can be considered<br />

infinite, the capitalized cost is used to calculate the equivalent PW or AW values for costs<br />

and benefits. Equation [5.3], A P ( i ), is used to determine the equivalent AW values in the<br />

incremental B/C analysis. Example 9.7 illustrates this using the progressive example and a<br />

spreadsheet.<br />

EXAMPLE 9.7 Water Treatment Facility #3 Case<br />

PE<br />

Land for Water Treatment Facility #3 was initially purchased in the year 2010 for $19.3 million;<br />

however, when it was publicized, influential people around Allen spoke strongly against<br />

the location. We will call this location 1. Some of the plant design had already been completed<br />

when the general manager announced that this site was not the best choice anyway, and that it<br />

would be sold and a different, better site (location 2) would be purchased for $28.5 million.<br />

This was well over the budget amount of $22.0 million previously set for land acquisition. As<br />

it turns out, there was a third site (location 3) available for $35.0 million that was never seriously<br />

considered.<br />

In his review and after much resistance from Allen Water Utilities staff, the consultant, Joel,<br />

received a copy of the estimated costs and benefits for the three plant location options. The<br />

revenues, savings, and sale of bulk water rights to other communities are estimated as increments<br />

from a base amount for all three locations. Using the assumption of a very long life for<br />

the WTF3 facility and the established discount rate of 3% per year, determine what Joel discovered<br />

when he did the B/C analysis. Was the general manager correct in concluding that<br />

location 2 was the best, all said and done?


9.4 Incremental B/C Analysis of Multiple, Mutually Exclusive Alternatives 245<br />

Location 1 Location 2 Location 3<br />

Land cost, $ million 19.3 28.5 35.0<br />

Facility first cost, $ million 460.0 446.0 446.0<br />

Benefits, $ per year:<br />

Pumping cost savings 5 3 0<br />

Sales to area communities 12 10 8<br />

Added revenue from Allen 6 6 6<br />

Total benefits, $ per year 23 19 14<br />

Solution<br />

A spreadsheet can be very useful when performing an incremental B/C analysis of three or<br />

more alternatives. Figure 9–1a presents the analysis with the preliminary input of AW values<br />

for costs using the relation A P(i) and annual benefits. Figure 9–1b details all the functions<br />

used in the analysis. Logical IF statements indicate alternative elimination and selection decisions.<br />

In $1 million units,<br />

Location 1: AW of costs A of land cost A of facility first cost<br />

(19.3 460.0)(0.03)<br />

$14.379 per year<br />

AW of benefits $23<br />

Location 2: AW of costs $14.235 AW of benefits $19<br />

Location 3: AW of costs $14.430 AW of benefits $14<br />

Though the AW of cost values are close to one another, the increasing order is locations 2, 1,<br />

and 3 to determine B/C values. The benefits are direct estimates; therefore, the overall B/C<br />

ratios indicate that location 3 (row 5; B/C 3 0.97) is not economically justified at the outset.<br />

It is eliminated, and one of the remaining locations must be selected. Location 2 is justified<br />

against the DN alternative (B/C 2 1.33); the only remaining comparison is 1-to-2 as detailed<br />

in column C of Figure 9–1. Location 1 is a clear winner with B/C 27.78.<br />

In conclusion, Joel has learned that location 1 is indeed the best and that, from the economic<br />

perspective, the general manager was incorrect in stating that location 2 was better. However,<br />

given the original evaluation criteria listed in the introduction—economics, environment, community<br />

impact, and constructability—location 2 is likely a good compromise selection.<br />

(a)<br />

Figure 9–1<br />

Incremental B/C analysis for WTF3 case: (a) numerical results and (b) functions developed for the analysis.<br />

(b)


246 Chapter 9 Benefit/Cost Analysis and Public Sector Economics<br />

Comment<br />

This is an actual situation with changed names and values. Location 1 was initially purchased<br />

and planned for WTF3. However, the presence of political, community, and environmental<br />

stress factors changed the decision to location 2, when all was said and done.<br />

9.5 Service Sector Projects and Cost-Effectiveness<br />

Analysis<br />

Much of the GDP of the United States and some countries in Europe and Asia is generated by<br />

what has become known as the service sector of the economy. A large percentage of service sector<br />

projects are generated by and dependent upon the private sector (corporations, businesses,<br />

and other for-profit institutions). However, many projects in the public sector are also service<br />

sector projects.<br />

A service sector project is a process or system that provides services to individuals, businesses,<br />

or government units. The economic value is developed primarily by the intangibles of the process<br />

or system, not the physical entities , such as buildings, machines, and equipment. Manufacturing<br />

and construction activities are commonly not considered a service sector project, though<br />

they may support the theme of the service provided.<br />

Service projects have a tremendous range of variety and purpose; to name a few: health care<br />

systems, health and life insurance, airline reservation systems, credit card services, police and<br />

court systems, security programs, safety training programs, and all types of consulting projects.<br />

The intangible and intellectual work done by engineers and other professionals is often a part of<br />

a service sector project.<br />

The economic evaluation of a service project is difficult to a great degree because the cost and<br />

benefit estimates are not accurate and often not within an acceptable degree of error. In other<br />

words, undue risk may be introduced into the decision because of poor monetary estimates. For<br />

example, consider the decision to place red-light cameras at stop lights to ticket drivers who run<br />

the red light. This is a public and a service project, but its ( economic ) benefi ts are quite diffi cult<br />

to estimate . Depending upon the viewpoint, benefits could be in terms of accidents averted,<br />

deaths prevented, police personnel released from patrolling the intersection, or, from a more<br />

mercenary viewpoint, amount of fines collected. In all but the last case, benefits in monetary<br />

terms will be poor estimates. These are examples where B/C analysis does not work well and a<br />

different form of analysis is needed.<br />

In service and public sector projects, as expected, it is the benefits that are the more difficult<br />

to estimate. An evaluation method that combines monetary cost estimates with nonmonetary<br />

benefit estimates is cost-effectiveness analysis (CEA) . The CEA approach utilizes a costeffectiveness<br />

measure or the cost-effectiveness ratio (CER) as a basis of ranking projects and<br />

selecting the best of independent projects or mutually exclusive alternatives. The CER ratio is<br />

defined as<br />

equivalent total costs<br />

CER —————————————<br />

total of effectiveness measure — C E<br />

[9.7]<br />

In the red-light camera example, the effectiveness measure (the benefit) may be one of the samples<br />

mentioned earlier, accidents averted or deaths prevented. Different from the B/C ratio of<br />

costs to benefits, CER places the PW or AW of total costs in the numerator and the effectiveness<br />

measure in the denominator. (The reciprocal of Equation [9.7] can also be used as the measure of<br />

worth, but we will use CER as defined above.) With costs in the numerator, smaller ratio values<br />

are more desirable for the same value of the denominator, since smaller ratio values indicate a<br />

lower cost for the same level of effectiveness.<br />

Like ROR and B/C analysis, cost-effectiveness analysis requires the ordering (ranking) of<br />

alternatives prior to selection and the use of incremental analysis for mutually exclusive


9.5 Service Sector Projects and Cost-Effectiveness Analysis 247<br />

alternative selection. Cost-effectiveness analysis utilizes a different ranking criterion than ROR or<br />

BC analysis. The ordering criteria are as follows:<br />

Independent projects: Initially rank projects by CER value.<br />

Mutually exclusive alternatives: Initially rank alternatives by effectiveness measure, then<br />

perform an incremental CER analysis.<br />

Return again to the public/service project of red-light cameras. If the CER is defined as “cost per<br />

total accidents averted ” and the projects are independent, increasing CER value is the ranking<br />

basis. If the projects are mutually exclusive, “total accidents averted ” is the correct ranking basis<br />

and an incremental analysis is necessary.<br />

There are significantly different analysis procedures for independent and mutually exclusive<br />

proposals. To select some from several (independent) projects, a budget limit, termed b, is inherently<br />

necessary once ordering is complete. However, for selecting one from several (mutually<br />

exclusive) alternatives, a pairwise incremental analysis is necessary and selection is made on the<br />

basis of C/E ratios. The procedures and examples follow.<br />

For independent projects, the procedure is as follows:<br />

1. Determine the equivalent total cost C and effectiveness measure E , and calculate the CER<br />

measure for each project.<br />

2. Order projects from the smallest to the largest CER value.<br />

3. Determine cumulative cost for each project and compare with the budget limit b .<br />

4. The selection criterion is to fund all projects such that b is not exceeded.<br />

Independent project<br />

selection<br />

EXAMPLE 9.8<br />

Recent research indicates that corporations throughout the world need employees who demonstrate<br />

creativity and innovation for new processes and products. One measure of these talents<br />

is the number of patents approved each year through the R&D efforts of a company. Rollings<br />

Foundation for Innovative Thinking has allocated $1 million in grant funds to award to corporations<br />

that enroll their top R&D personnel in a 1- to 2-month professional training program in<br />

their home state that has a historically proven track record over the last 5 years in helping<br />

individuals earn patents.<br />

Table 9–2 summarizes data for six corporations that submitted proposals. Columns 2 and 3<br />

give the proposed number of attendees and cost per person, respectively, and column 4 provides<br />

the historical track record of program graduates in patents per year. Use cost- effectiveness<br />

analysis to select the corporations and programs to fund.<br />

Solution<br />

We assume that across all programs and all patent awards there is equal quality. Use the procedure<br />

for independent projects and b $1 million to select from the proposals.<br />

TABLE 9–2<br />

Program<br />

(1)<br />

Data for Programs to Increase Patents Used for CEA<br />

Total Personnel<br />

(2)<br />

Cost/Person, $<br />

(3)<br />

5-Year History,<br />

Patents/Graduate/Year<br />

(4)<br />

1 50 5000 0.5<br />

2 35 4500 3.1<br />

3 57 8000 1.9<br />

4 24 2500 2.1<br />

5 12 5500 2.9<br />

6 87 3800 0.6


248 Chapter 9 Benefit/Cost Analysis and Public Sector Economics<br />

1. Using Equation [9.7], the effectiveness measure E is patents per year, and the CER is<br />

program cost per person<br />

CER ——————————<br />

patents per graduate<br />

C —<br />

E<br />

The program cost C is a PW value, and the E values are obtained from the proposals.<br />

2. The CER values are shown in Table 9–3 in increasing order, column 5.<br />

3. Cost per course, column 6, and cumulative costs, column 7, are determined.<br />

4. Programs 4, 2, 5, 3, and 6 (68 of the 87 people) are selected to not exceed $1 million.<br />

TABLE 9–3 Programs Ordered by CER Value, Example 9.8<br />

Program<br />

(1)<br />

Total<br />

Personnel<br />

(2)<br />

Cost/Person<br />

C, $<br />

(3)<br />

Patents<br />

per Year E<br />

(4)<br />

CER, $ per<br />

Patent<br />

(5) (3)/(4)<br />

Program<br />

Cost, $<br />

(6) (2)(3)<br />

Cumulative<br />

Cost, $<br />

(7) (6)<br />

4 24 2,500 2.1 1,190 60,000 60,000<br />

2 35 4,500 3.1 1,452 157,500 217,500<br />

5 12 5,500 2.9 1,897 66,000 283,500<br />

3 57 8,000 1.9 4,211 456,000 739,500<br />

6 87 3,800 0.6 6,333 330,600 1,070,100<br />

1 50 5,000 0.5 10,000 250,000 1,320,100<br />

Comment<br />

This is the first time that a budget limit has been imposed for the selection among independent<br />

projects. This is often referred to as capital budgeting, which is discussed further in Chapter 12.<br />

For mutually exclusive alternatives and no budget limit, the alternative with the highest effectiveness<br />

measure E is selected without further analysis. Otherwise, an incremental CER analysis<br />

is necessary and the budget limit is applied to the selected alternative(s). The analysis is<br />

based on the incremental ratio C/E, and the procedure is similar to that we have applied for<br />

incremental ROR and B/C, except now the concept of dominance is utilized.<br />

Dominance occurs when the incremental analysis indicates that the challenger alternative offers<br />

an improved incremental CER measure compared to the defender’s CER, that is,<br />

ME alternative<br />

selection<br />

(C/E) challenger (C/E) defender<br />

Otherwise, no dominance is present, and both alternatives remain in the analysis.<br />

For mutually exclusive alternatives, the selection procedure is as follows:<br />

1. Order the alternatives from smallest to largest effectiveness measure E . Record the cost for<br />

each alternative.<br />

2. Calculate the CER measure for the first alternative. This, in effect, makes DN the defender<br />

and the first alternative the challenger. This CER is a baseline for the next incremental comparison,<br />

and the first alternative becomes the new defender.<br />

3. Calculate incremental costs ( C ) and effectiveness ( E ) and the incremental measure C/E<br />

for the new challenger using the relation<br />

cost of challenger cost of defender<br />

C/E ———————————————————————<br />

effectiveness of challenger effectiveness of defender —— C<br />

E<br />

4. If C/E C/E defender , the challenger dominates the defender and it becomes the new defender;<br />

the previous defender is eliminated. Otherwise, no dominance is present and both<br />

alternatives are retained for the next incremental evaluation.<br />

5. Dominance present: Repeat steps 3 and 4 to compare the next ordered alternative (challenger)<br />

and new defender. Determine if dominance is present.


9.5 Service Sector Projects and Cost-Effectiveness Analysis 249<br />

Dominance not present: The current challenger becomes the new defender, and the next alternative<br />

is the new challenger. Repeat steps 3 and 4 to compare the new challenger and new<br />

defender. Determine if dominance is present.<br />

6. Continue steps 3 through 5 until only one alternative or only nondominated alternatives remain.<br />

7. Apply the budget limit (or other criteria) to determine which of the remaining alternative(s)<br />

can be funded.<br />

EXAMPLE 9.9<br />

One of the corporations not selected for funding in Example 9.8 decided to fund its 50 R&D<br />

personnel to attend one of the innovation and creativity programs at its own expense. One<br />

criterion is that the program must have a historical average for a graduate of at least 2.0 patents<br />

per year. Use the data in Table 9–2 to select the best program.<br />

TABLE 9–4<br />

Program<br />

(1)<br />

Total<br />

Personnel<br />

(2)<br />

Mutually Exclusive Alternatives Evaluated by Cost-Effectiveness<br />

Analysis, Example 9.9<br />

Cost/Person<br />

C , $<br />

(3)<br />

Patents per<br />

Year E<br />

(4)<br />

CER, $ per<br />

Patent<br />

(5) (3)/(4)<br />

Program Cost,<br />

$<br />

(6) (2)(3)<br />

4 50 2,500 2.1 1190 125,000<br />

5 50 5,500 2.9 1897 275,000<br />

2 50 4,500 3.1 1452 225,000<br />

Solution<br />

From Table 9–2 , three programs—2, 4, and 5—have a historical record of at least two patents<br />

per graduate per year. Since only one program will be selected, these are now mutually exclusive<br />

alternatives. Use the procedure to perform the incremental analysis.<br />

1. The alternatives are ranked by increasing patents per year in Table 9–4 , column 4.<br />

2. The CER measure for program 4 is compared to the DN alternative.<br />

program cost per person<br />

C/E 4 —————————— ——— 2500<br />

patents per graduate 2.1 1190<br />

3. Program 5 is now the challenger.<br />

5-to-4 comparison: C/E C ——<br />

E<br />

——————<br />

5500 2500 3750<br />

2.9 2.1<br />

4. In comparison to C/E 4 1190, it costs $3750 per additional patent if program 5 is chosen<br />

over 4. Program 5 is more expensive for more patents; however, clear dominance is not<br />

present; both programs are retained for further evaluation.<br />

5. Dominance not present: Program 5 becomes the new defender, and program 2 is the new<br />

challenger. Perform the 2-to-5 comparison.<br />

2-to-5 comparison: C/E C ——<br />

E<br />

——————<br />

4500 5500 5000<br />

3.1 2.9<br />

Compared to C/E 5 1897, this increment is much cheaper—more patents for less money<br />

per person. Dominance is present; eliminate program 5 and compare 2 to 4.<br />

6. Repeat steps 3 through 5 and compare C/E to C/E 4 1190.<br />

2-to-4 comparison: C/E C ——<br />

E<br />

——————<br />

4500 2500 2000<br />

3.1 2.1<br />

This does not represent dominance of program 2 over 4. The conclusion is that both programs<br />

are eligible for funding, that is, CEA in this case does not indicate only one program.<br />

This occurs when there is not lower cost and higher effectiveness of one alternative<br />

over another; that is, one alternative does not dominate all the others.<br />

7. Now the budget and other considerations (probably noneconomic) are brought to bear to<br />

make the final decision. The fact that program 4 costs $125,000—significantly less than<br />

program 2 at $225,000—will likely enter into the decision.


250 Chapter 9 Benefit/Cost Analysis and Public Sector Economics<br />

Cost-effectiveness analysis is a form of multiattribute decision-making in which economic<br />

and noneconomic dimensions are integrated to evaluate alternatives from several perspectives by<br />

different decision makers. See Chapter 10 for further discussion of alternative analysis using<br />

multiple attributes.<br />

9.6 Ethical Considerations in the Public Sector<br />

Usual expectations by citizens of their elected officials—locally, nationally, and internationally—<br />

are that they make decisions for the good of the public, ensuring safety and minimizing risk and cost<br />

to the public. Above these is the long-standing expectation that public servants have integrity.<br />

Similarly, the expectations of engineers employed by government departments, and those<br />

serving as consultants to government agencies, are held to high standards. Impartiality, consideration<br />

of a wide range of circumstances, and the use of realistic assumptions are but three of<br />

the foundation elements upon which engineers should base their recommendations to decision<br />

makers. This implies that engineers in public service avoid<br />

• Self-serving, often greedy individuals and clients with goals of excessive profits and future<br />

contract awards<br />

• Using a politically favorable perspective that compromises the results of a study<br />

• Narrowly-defined assumptions that serve special interest groups and subcommunities potentially<br />

affected by the findings<br />

Many people are disappointed and discouraged with government when elected officials and<br />

public employees (engineers and others, alike) do not have real commitment to integrity and<br />

unbiasedness in their work.<br />

Engineers are routinely involved in two of the major aspects of public sector activities:<br />

Public policy making —the development of strategy for public service, behavior, fairness,<br />

and justice. This may involve literature study, background discovery, data collection, opinion<br />

giving, and hypothesis testing. An example is transportation management . Engineers make<br />

virtually all the recommendations based on data and long-standing decision algorithms for<br />

policy items such as capacity of roads, expansion of highways, planning and zoning rules,<br />

traffic signal usage, speed limit corridors, and many related topics in transportation policy.<br />

Public officials use these findings to establish public transportation policy.<br />

Public planning —the development of projects that implement strategy and affect people,<br />

the environment, and financial resources in a variety of ways. Consider traffi c control ,<br />

where the use and placement of traffic control signs, signals, speed limits, parking restrictions,<br />

etc. are detailed based upon established policy and current data. (In effect, this is<br />

systems engineering, that is, an application of the life-cycle phases and stages explained in<br />

Section 6.5.)<br />

Whether in the arena of policy making or public planning, engineers can find ethical compromise<br />

a possibility when working with the public sector. A few circumstances are summarized here.<br />

• Use of technology Many public projects involve the use of new technology. The public risks<br />

and safety factors are not always known for these new advances. It is common and expected<br />

that engineers make every attempt to apply the latest technology while ensuring that the public<br />

is not exposed to undue risk.<br />

• Scope of study A client may pressure the engineers to limit the range of options, the assumption<br />

base, or the breadth of alternative solutions. These restrictions may be based on financial<br />

reasons, politically-charged topics, client-favored options, or a wide variety of other reasons.<br />

To remain impartial, it is the responsibility of the engineer to submit a fully unbiased analysis,<br />

report, and recommendation, even though it may jeopardize future contract possibilities, promote<br />

public disfavor, or generate other negative consequences.<br />

• Negative community impact It is inevitable that public projects will adversely affect some<br />

groups of people, or the environment, or businesses. The intentional silencing of these projected<br />

effects is often the cause of strong public outcries against what may be a project that is<br />

in the best interest of the community at large. Engineers who find (stumble onto) such negative<br />

impacts may be pressured by clients, managers, or public figures to overlook them, though


Chapter Summary 251<br />

TABLE 9–5 Some Ethical Considerations When Performing B/C and CEA Analysis<br />

What the Study Includes Ethical Dimension<br />

Audience for study<br />

Impact time of decision<br />

Greater good for community<br />

as a whole<br />

Reliance on economic<br />

measures only<br />

Scope of disbenefits estimated<br />

and evaluated<br />

Is it ethical to select a specific<br />

group of people affected by the<br />

project and neglect possible<br />

effects on other groups?<br />

Is it ethical to decide now for future<br />

generations who may be adversely<br />

and economically affected<br />

by the current project decision?<br />

Vulnerable minority groups, especially<br />

economically deprived<br />

ones, may be disproportionally<br />

affected. Is this ethical if the impact<br />

is predictable?<br />

Is it acceptable to reduce all<br />

costs and benefits to monetary<br />

estimates for a decision, then<br />

subjectively impute nonquantified<br />

factors in the final decision?<br />

Is it ethical to disregard any disbenefits<br />

in the B/C study or use<br />

indirect effectiveness measures<br />

in a CEA study based on the difficulty<br />

to estimate some of them?<br />

Example<br />

Construct children’s health care clinics<br />

for city dwellers, but neglect rural families<br />

with poor transportation means.<br />

Accomplish financial bailouts of corporations<br />

when future generations’ taxes<br />

will be significantly higher to recover the<br />

costs, plus interest and inflation effects.<br />

Allow a chemical plant that is vital to<br />

the community’s employment to pollute<br />

a waterway when a minority group is<br />

known to eat fish from the water that is<br />

predictably contaminated.<br />

Softening of building codes can improve<br />

the financial outlook for home<br />

builders; however, increased risks of<br />

fire loss, storm and water damage to<br />

structures, and reduced future resale<br />

values are considered only in passing as<br />

a new subdivision is approved by the<br />

planning and zoning committee.<br />

Noise and air pollution caused by a<br />

planned open-pit quarry will have a negative<br />

effect on area ranchers, residents,<br />

wildlife, and plant life; but the effectiveness<br />

measure considers only suburban<br />

residents due to estimation difficulty of<br />

effects on other constituencies.<br />

the Code of Ethics for Engineers dictates a full and fair analysis and report. For example, a<br />

planned rerouting of a city street may effectively cut off a section of international citizens’<br />

businesses, thus resulting in a clearly predictable economic downturn. Considering this outcome<br />

in the recommendation to the transportation department should be a goal of the analyzing<br />

engineers, yet pressure to bias the results may be quite high.<br />

The results of a B/C or CEA analysis are routinely depended upon by public officials and<br />

government staff members to assist in making public planning decisions. As discussed earlier,<br />

estimations for benefits, disbenefits, effectiveness measures, and costs can be difficult and inaccurate,<br />

but these analysis tools are often the best available to structure a study. Some example<br />

ethically-oriented challenges that may be confronted during B/C and CEA analyses are summarized<br />

in Table 9–5 .<br />

CHAPTER SUMMARY<br />

The benefit/cost method is used primarily to evaluate alternatives in the public sector. When one<br />

is comparing mutually exclusive alternatives, the incremental B/C ratio must be greater than or<br />

equal to 1.0 for the incremental equivalent total cost to be economically justified. The PW, AW,<br />

or FW of the initial costs and estimated benefits can be used to perform an incremental B/C<br />

analysis. For independent projects, no incremental B/C analysis is necessary. All projects with<br />

B/C 1.0 are selected provided there is no budget limitation. It is usually quite diffi cult to make<br />

accurate estimates of benefi ts for public sector projects. The characteristics of public sector projects<br />

are substantially different from those of the private sector: initial costs are larger; expected<br />

life is longer; additional sources of capital funds include taxation, user fees, and government<br />

grants; and interest (discount) rates are lower.


252 Chapter 9 Benefit/Cost Analysis and Public Sector Economics<br />

Service projects develop economic value largely based on the intangibles of the services provided<br />

to users, not the physical items associated with the process or system. Evaluation by B/C<br />

analysis can be difficult with no good way to make monetary estimates of benefits. Cost-effectiveness<br />

analysis (CEA) combines cost estimates and a nonmonetary effectiveness measure (the<br />

benefit) to evaluate independent or mutually exclusive projects using procedures that are similar<br />

to incremental ROR and B/C analysis. The concept of dominance is incorporated into the procedure<br />

for comparing mutually exclusive alternatives.<br />

As a complement to the discussion on professional ethics in Chapter 1, some potential ethical<br />

challenges in the public sector for engineers, elected officials, and government consultants are<br />

discussed here. Examples are included.<br />

PROBLEMS<br />

Public Projects and B/C Concepts<br />

9.1 What is the difference between disbenefits and<br />

costs?<br />

9.2 Identify the following as primarily public or private<br />

sector undertakings: eBay, farmer’s market,<br />

state police department, car racing facility, social<br />

security, EMS, ATM, travel agency, amusement<br />

park, gambling casino, swap meet, football<br />

stadium.<br />

9.3 State whether the following characteristics are primarily<br />

associated with public or private sector<br />

projects: large initial investment, park user fees,<br />

short-life projects, profit, disbenefits, tax-free<br />

bonds, subsidized loans, low interest rate, income<br />

tax, water quality regulations.<br />

9.4 Identify the following cash flows as a benefit, disbenefit,<br />

or cost.<br />

(a) Loss of income to local businesses because<br />

of a new freeway<br />

(b) Less travel time because of a loop bypass<br />

(c) $400,000 annual income to local businesses<br />

because of tourism created by a national park<br />

(d) Cost of fish from a hatchery to stock a lake at<br />

the state park<br />

(e) Less tire wear because of smoother road<br />

(f)<br />

surfaces<br />

Decrease in property values due to the closure<br />

of a government research lab<br />

(g) School overcrowding because of a military<br />

base expansion<br />

(h) Revenue to local motels because of an extended<br />

weekend holiday<br />

9.5 What is a fundamental difference between DBOM<br />

and DBOMF contracts?<br />

9.6 Buster County has proposed a strict water conservation<br />

policy for all industrial plants within the<br />

county limits. Enforcement is proposed to be a<br />

public-private partnership between the sheriff’s<br />

office and a private security company. In preparing<br />

for a B/C analysis of the proposal, estimates must<br />

be categorized as a benefit, cost, or disbenefit. The<br />

categories chosen will vary depending upon a person’s<br />

viewpoint. Assume that possible viewpoints<br />

include sales revenues, politics, service to the<br />

public, the environment, contract obligations, future<br />

generations of residents, legal matters, revenue<br />

and budget (in general), and customers.<br />

( a) Select the top two viewpoints (in your opinion)<br />

for each of the following individuals as<br />

they would categorize estimates as a cost,<br />

benefit, or disbenefit.<br />

1. An industrial plant manager in the county<br />

2. County sheriff’s deputy (appointed office)<br />

3. County commissioner (elected office)<br />

4. Security company president<br />

( b) Explain your answers by writing a short description<br />

of why you selected as you did.<br />

Project B/C Value<br />

9.7 If an alternative has a salvage value, how is it handled<br />

in the calculation of a B/C ratio relative to<br />

benefits, disbenefits, costs, or savings?<br />

9.8 The cost of grading and spreading gravel on a<br />

short rural road is expected to be $300,000. The<br />

road will have to be maintained at a cost of $25,000<br />

per year. Even though the new road is not very<br />

smooth, it allows access to an area that previously<br />

could only be reached with off-road vehicles. The<br />

improved accessibility has led to a 150% increase<br />

in the property values along the road. If the previous<br />

market value of a property was $900,000,<br />

calculate the B/C ratio using an interest rate of 6%<br />

per year and a 20-year study period.<br />

9.9 Arsenic enters drinking water supplies from natural<br />

deposits in the earth or from agricultural and<br />

industrial practices. Since it has been linked to


Problems 253<br />

cancer of the bladder, kidney, and other internal<br />

organs, the EPA has lowered the arsenic standard<br />

for drinking water from 0.050 parts per million to<br />

0.010 parts per million (10 parts per billion). The<br />

annual cost to public water utilities to meet the<br />

new standard is estimated to be $200 per household.<br />

If it is estimated that there are 90 million<br />

households in the United States and that the lower<br />

standard can save 50 lives per year valued at<br />

$4,000,000 per life, what is the benefit/cost ratio of<br />

the regulation?<br />

9.10 From the following estimates, determine the B/C<br />

ratio for a project that has a 20-year life. Use an<br />

interest rate of 8% per year.<br />

Consequences to<br />

the People<br />

Annual<br />

benefits<br />

$90,000<br />

per year<br />

Annual $10,000<br />

disbenefits per year<br />

Consequences to<br />

the Government<br />

First<br />

cost<br />

Annual<br />

cost<br />

Annual<br />

savings<br />

$750,000<br />

$50,000<br />

per year<br />

$30,000<br />

per year<br />

9.11 A project to extend irrigation canals into an area<br />

that was recently cleared of mesquite trees (a nuisance<br />

tree in Texas) and large weeds is projected to<br />

have a capital cost of $2,000,000. Annual maintenance<br />

and operation costs will be $100,000 per<br />

year. Annual favorable consequences to the general<br />

public of $820,000 per year will be offset to<br />

some extent by annual adverse consequences of<br />

$400,000 to a portion of the general public. If the<br />

project is assumed to have a 20-year life, what is<br />

the B/C ratio at an interest rate of 8% per year?<br />

9.12 Calculate the B/C ratio for the following cash flow<br />

estimates at a discount rate of 7% per year.<br />

Item<br />

Cash Flow<br />

FW of benefits, $ 30,800,000<br />

AW of disbenefits, $ per year 105,000<br />

First cost, $ 1,200,000<br />

M&O costs, $ per year 400,000<br />

Life of project, years 20<br />

9.13 The benefits associated with a nuclear power plant<br />

cooling water filtration project located on the Ohio<br />

River are $10,000 per year forever, starting in<br />

year 1. The costs are $50,000 in year 0 and $50,000<br />

at the end of year 2. Calculate the B/C ratio at i <br />

10% per year.<br />

9.14 A privately funded wind-based electric power<br />

generation company in the southern part of the<br />

country has developed the following estimates<br />

(in $1000) for a new turbine farm. The MARR is<br />

10% per year, and the project life is 25 years. Calculate<br />

( a ) the profitability index and ( b ) the modified<br />

B/C ratio.<br />

Benefits: $20,000 in year 0 and $30,000 in<br />

year 5<br />

Savings: $2000 in years 1–20<br />

Cost: $50,000 in year 0<br />

Disbenefits: $3000 in years 1–10<br />

9.15 For the values shown, calculate the conventional<br />

B/C ratio at i 10% per year.<br />

PW, $ AW, $/Year FW, $<br />

First cost 100,000 — 259,370<br />

M&O cost 61,446 10,000 159,374<br />

Benefits — 40,000 637,496<br />

Disbenefits 30,723 5,000 —<br />

9.16 A proposal to reduce traffic congestion on I-5 has<br />

a B/C ratio of 1.4. The annual worth of benefits<br />

minus disbenefits is $560,000. What is the first<br />

cost of the project if the interest rate is 6% per year<br />

and the project is expected to have a 20-year life?<br />

9.17 Oil spills in the Gulf of Mexico have been known<br />

to cause extensive damage to both public and private<br />

oyster grounds along the Louisiana and Mississippi<br />

shores. One way to protect shellfish along<br />

the shoreline is to release large volumes of freshwater<br />

from the Mississippi River to flush oil out to<br />

sea. This procedure inevitably results in death to<br />

some of the saltwater shellfish while preventing<br />

more widespread destruction to public reefs. Oil<br />

containment booms and other temporary structures<br />

can also be used to intercept floating oil before<br />

it damages sensitive fishing grounds. If the<br />

Fish and Wildlife Service spent $110 million in<br />

year 0 and $50 million in years 1 and 2 to minimize<br />

environmental damage from one particular<br />

oil spill, what is the benefit-to-cost ratio provided<br />

the efforts resulted in saving 3000 jobs valued at a<br />

total of $175 million per year? Assume disbenefits<br />

associated with oyster deaths amounted to $30 million<br />

in year 0. Use a 5-year study period and an<br />

interest rate of 8% per year.<br />

9.18 On-site granular ferric hydroxide (GFH) systems<br />

can be used to remove arsenic from water when<br />

daily flow rates are relatively low. If the operating<br />

cost is $600,000 per year and the public health<br />

benefits are assumed to be $800,000 per year, what<br />

initial investment in the GFH system is necessary<br />

to guarantee a modified B/C ratio of at least 1.0?<br />

Assume that the equipment life is 10 years and the<br />

interest rate is 6% per year.


254 Chapter 9 Benefit/Cost Analysis and Public Sector Economics<br />

9.19 The Parks and Recreation Department of Burkett<br />

County has estimated that the initial cost of a<br />

“bare-bones” permanent river park will be<br />

$2.3 million. Annual upkeep costs are estimated at<br />

$120,000. Benefits of $340,000 per year and disbenefits<br />

of $40,000 per year have also been identified.<br />

Using a discount rate of 6% per year, calculate<br />

( a ) the conventional B/C ratio and ( b ) the<br />

modified B/C ratio.<br />

9.20 From the following data, calculate the ( a ) conventional<br />

and ( b ) modified benefit/cost ratios using an<br />

interest rate of 6% per year and an infinite project<br />

period.<br />

Benefits:<br />

To the<br />

People<br />

$300,000 now and<br />

$100,000 per<br />

year thereafter<br />

Disbenefits: $40,000 per year<br />

To the<br />

Government<br />

Costs: $1.5 million now<br />

and $200,000<br />

three years<br />

from now<br />

Savings: $70,000 per year<br />

9.21 In 2010, Brazil began construction of the Belo<br />

Monte hydroelectric dam on the Xingu River<br />

(which feeds the Amazon River). The project is<br />

funded by a consortium of investors and is expected<br />

to cost $11 billion. It will begin producing<br />

electricity in 2015. Even though the dam will provide<br />

clean energy for millions of people, environmentalists<br />

are sharply opposed. They say it will<br />

devastate wildlife and the livelihoods of 40,000<br />

people who live in the area to be flooded.<br />

Assume that the funding will occur evenly over<br />

the 5-year period from 2010 through 2014 at<br />

$2.2 billion per year. The disbenefits are estimated<br />

to be $100,000 for each displaced person and $1<br />

billion for wildlife destruction. Assume that the<br />

disbenefits will occur evenly through the 5-year<br />

construction period and anticipated benefits will<br />

begin at the end of 2015 and continue indefinitely.<br />

Use an interest rate of 8% per year to determine<br />

what the equivalent annual benefits must be to ensure<br />

a B/C ratio of at least 1.0.<br />

9.22 In the United States, the average number of airplanes<br />

in the sky on an average morning is 4000.<br />

There are another 16,000 planes on the ground.<br />

Aerospace company Rockwell Collins developed<br />

what it calls a digital parachute —a panic-button<br />

technology that will land any plane in a pinch at<br />

the closest airport, no matter what the weather or<br />

geography and without the help of a pilot. The<br />

technology can be applied if a pilot is no longer<br />

capable of flying the plane or is panicked and confused<br />

about what to do in an emergency. Assume<br />

that the cost of retrofitting 20,000 commercial airplanes<br />

is $100,000 each and the plane stays in service<br />

for 15 years. If the technology saves an average<br />

of 30 lives per year, with the value of a human<br />

life placed at $4,000,000, what is the B/C ratio?<br />

Use an interest rate of 10% per year.<br />

9.23 Although the lower Rio Grande is regulated by<br />

the Elephant Butte Dam and the Caballo Reservoir,<br />

serious flooding has occurred in El Paso and<br />

other cities located along the river. This has required<br />

homeowners living in valley areas near the<br />

river to purchase flood insurance costing between<br />

$145 and $2766 per year. To alleviate the possibility<br />

of flooding, the International Boundary and<br />

Water Commission undertook a project costing<br />

$220 million to raise the levees along flood-prone<br />

portions of the river. As a result, 13,000 properties<br />

were freed of the federal mandate to purchase<br />

flood insurance. In addition, historical records indicate<br />

that damage to infrastructure will be<br />

avoided, which amounts to an average benefit of<br />

$8,200,000 per year. If the average cost of flood<br />

insurance is $460 per household per year, calculate<br />

the benefit-to-cost ratio of the levee-raising<br />

project. Use an interest rate of 6% per year and a<br />

30-year study period.<br />

9.24 For the data shown, calculate the conventional B/C<br />

ratio at i 6% per year.<br />

Benefits:<br />

$20,000 in year 0 and<br />

$30,000 in year 5<br />

Disbenefits: $7000 in year 3<br />

Savings (to government): $25,000 in years 1–4<br />

Cost: $100,000 in year 0<br />

Project life:<br />

5 years<br />

9.25 Explain a fundamental difference between the<br />

modified B/C ratio and the profitability index.<br />

9.26 Gerald Corporation entered a public-private partnership<br />

using a DBOM contract with the state of<br />

Massachusetts 10 years ago for railroad system<br />

upgrades. Determine the profitability index for the<br />

financial results listed below using a MARR of<br />

8% per year.<br />

Investments: Year 0 $4.2 million<br />

Year 5 $3.5 million<br />

Net savings: Years 15 $1.2 million per year<br />

Years 610 $2.5 million per year<br />

9.27 A project had a staged investment distributed over<br />

the 6-year contract period. For the cash flows<br />

shown (next page) and an interest rate of 10% per<br />

year, determine the profitability index and determine<br />

if the project was economically justified.


Problems 255<br />

Year 0 1 2 3 4 5 6<br />

Investment, $1000 25 0 10 0 5 0 0<br />

NCF, $1000 per year 0 5 7 9 11 13 15<br />

Two Alternative Comparison<br />

9.28 In comparing two alternatives by the B/C method,<br />

if the overall B/C ratio for both alternatives is calculated<br />

to be exactly 1.0, which alternative should<br />

you select?<br />

9.29 In comparing alternatives X and Y by the B/C<br />

method, if B/C X 1.6 and B/C Y 1.8, what is<br />

known about the B/C ratio on the increment of investment<br />

between X and Y?<br />

9.30 Logan Well Services Group is considering two sites<br />

for storage and recovery of reclaimed water. The<br />

mountain site (MS) will use injection wells that cost<br />

$4.2 million to develop and $280,000 per year for<br />

M&O costs. This site will be able to accommodate<br />

150 million gallons per year. The valley site (VS) will<br />

involve recharge basins that cost $11 million to construct<br />

and $400,000 to operate and maintain. At this<br />

site, 890 million gallons can be injected each year. If<br />

the value of the injected water is $3.00 per 1000 gallons,<br />

which alternative, if either, should be selected<br />

according to the B/C ratio method? Use an interest<br />

rate of 8% per year and a 20-year study period.<br />

9.31 The estimates shown are for a bridge under consideration<br />

for a river crossing in Wheeling, West<br />

Virginia. Use the B/C ratio method at an interest<br />

rate of 6% per year to determine which bridge, if<br />

either, should be built.<br />

East Location<br />

West Location<br />

Initial cost, $ 11 10 6 27 10 6<br />

Annual M&O, $/year 100,000 90,000<br />

Benefits, $/year 990,000 2,400,000<br />

Disbenefits, $/year 120,000 100,000<br />

Life, years <br />

9.32 Select the better of two proposals to improve street<br />

safety and lighting in a colonia in south central<br />

New Mexico. Use a B/C analysis and an interest<br />

rate of 8% per year.<br />

Proposal 1 Proposal 2<br />

Initial cost, $ 900,000 1,700,000<br />

Annual M&O cost, $/year 120,000 60,000<br />

Annual benefits, $/year 530,000 650,000<br />

Annual disbenefits, $/year 300,000 195,000<br />

Life, years 10 20<br />

9.33 Conventional and solar alternatives are available<br />

for providing energy at a remote radar site. Use the<br />

B/C ratio to determine which method should be<br />

selected at an interest rate of 8% per year and a<br />

5-year study period.<br />

Conventional<br />

Solar<br />

Initial cost, $ 300,000 2,500,000<br />

Annual cost, $ per year 700,000 5,000<br />

9.34 The two alternatives shown are under consideration<br />

for improving security at a county jail in Travis<br />

County, New York. Determine which one should be<br />

selected, based on a B/C analysis, an interest rate of<br />

7% per year and a 10-year study period.<br />

Extra Cameras<br />

(EC)<br />

New Sensors<br />

(NS)<br />

First cost, $ 38,000 87,000<br />

Annual M&O, $/year 49,000 64,000<br />

Benefits, $/year 110,000 160,000<br />

Disbenefits, $/year 26,000 —<br />

9.35 The U.S. government recently released an RFP to<br />

construct a second-story floor on an existing building<br />

at the Pentagon Complex. Separate contractors proposed<br />

two methods. Method 1 will use lightweight<br />

expanded shale on a metal deck with open web joists<br />

and steel beams. For this method, the costs will be<br />

$14,100 for concrete, $6000 for metal decking,<br />

$4300 for joists, and $2600 for beams. Method 2 will<br />

construct a reinforced concrete slab costing $5200<br />

for concrete, $1400 for rebar, $2600 for equipment<br />

rental, and $1200 for expendable supplies. Special<br />

additives will be included in the lightweight concrete<br />

that will improve the heat-transfer properties of the<br />

floor. If the energy costs for method 1 will be $600<br />

per year lower than for method 2, which one is more<br />

attractive? Use an interest rate of 7% per year, a<br />

20 -year study period, and the B/C method.<br />

9.36 A project to control flooding from rare, but sometimes<br />

heavy rainfalls in the arid southwest will<br />

have the cash flows shown below. Determine<br />

which project should be selected on the basis of a<br />

B/C analysis at i 8% per year and a 20-year<br />

study period.<br />

Sanitary<br />

Sewers<br />

Open<br />

Channels<br />

First cost, $ 26 million 53 million<br />

M&O cost, $ per year 400,000 30,000<br />

Homeowner cleanup costs,<br />

$ per year<br />

60,000 0<br />

9.37 Two routes are under consideration for a new interstate<br />

highway. The long intervalley route would be<br />

25 kilometers in length and would have an initial<br />

cost of $25 million. The short transmountain route


256 Chapter 9 Benefit/Cost Analysis and Public Sector Economics<br />

would be 10 kilometers long and would have an initial<br />

cost of $45 million. Maintenance costs are estimated<br />

at $150,000 per year for the long route and<br />

$35,000 per year for the short route. Regardless of<br />

which route is selected, the volume of traffic is expected<br />

to be 400,000 vehicles per year. If the vehicle<br />

operating expenses are assumed to be $0.30 per kilometer,<br />

determine which route should be selected on<br />

the basis of ( a ) conventional B/C analysis and<br />

( b ) modified B/C analysis. Assume an infinite life for<br />

each road, and use an interest rate of 8% per year.<br />

9.38 The Idaho Department of Fish and Wildlife (IDFW)<br />

is considering two locations for a new state park.<br />

Location E would require an investment of $3 million<br />

and $50,000 per year to maintain. Location W<br />

would cost $7 million to construct, but the IDFW<br />

would receive an additional $25,000 per year in<br />

park fees. The operating cost of location W will be<br />

$65,000 per year. The revenue to park concessionaires<br />

will be $500,000 per year at location E and<br />

$700,000 at location W. The disbenefits associated<br />

with each location are $30,000 per year for location<br />

E and $40,000 per year for location W. Assume the<br />

park will be maintained indefinitely. Use an interest<br />

rate of 12% per year to determine which location, if<br />

either, should be selected on the basis of ( a ) the<br />

B/C method and ( b ) the modified B/C method.<br />

9.39 Three engineers made the estimates shown below<br />

for two optional methods by which new construction<br />

technology would be implemented at a site for public<br />

housing. Either one of the two options or the current<br />

method may be selected. Set up a spreadsheet<br />

for B/C sensitivity analysis and determine if option<br />

1, option 2 or the do-nothing option is selected by<br />

each of the three engineers. Use a life of 5 years and<br />

a discount rate of 10% per year for all analyses.<br />

Engineer Bob Engineer Judy Engineer Chen<br />

Option 1 Option 2 Option 1 Option 2 Option 1 Option 2<br />

Alternative A B C D E F<br />

PW of capital, $ 80 50 72 43 89 81<br />

PW of benefits, $ 70 55 76 52 85 84<br />

9.41 Comparison of five mutually exclusive alternatives<br />

is shown. One must be accepted. According<br />

to the B/C ratio, which alternative should be selected<br />

(costs increase from A to E).<br />

Comparison<br />

B/C<br />

Ratio<br />

A versus B 0.75<br />

B versus C 1.4<br />

C versus D 1.3<br />

A versus C 1.1<br />

A versus D 0.2<br />

B versus D 1.9<br />

C versus E 1.2<br />

D versus E 0.9<br />

9.42 A consulting engineer is currently evaluating four<br />

different projects for the Department of Housing<br />

and Urban Development. The future worth of<br />

costs, benefits, disbenefits, and cost savings is<br />

shown. The interest rate is 10% per year, compounded<br />

continuously. Determine which of the<br />

projects, if any, should be selected, if the projects<br />

are ( a ) independent and ( b ) mutually exclusive.<br />

Project ID<br />

Good Better Best Best of All<br />

FW of first costs, $ 10,000 8,000 20,000 14,000<br />

FW of benefits, $ 15,000 11,000 25,000 42,000<br />

FW of disbenefits, $ 6,000 1,000 20,000 32,000<br />

FW of cost savings, $ 1,500 2,000 16,000 3,000<br />

9.43 From the data shown below for six mutually<br />

exclusive projects, determine which project, if<br />

any, should be selected.<br />

Project ID<br />

Initial cost, $ 50,000 90,000 75,000 90,000 60,000 70,000<br />

Cost, $/year 3,000 4,000 3,800 3,000 6,000 3,000<br />

Benefits, $/year 20,000 29,000 30,000 35,000 30,000 35,000<br />

Disbenefits, $/year 500 1,500 1,000 0 5,000 1,000<br />

Multiple Alternatives<br />

9.40 A group of engineers responsible for developing<br />

advanced missile detection and tracking technologies,<br />

such as shortwave infrared, thermal infrared<br />

detection, target tracking radar, etc., recently came<br />

up with six proposals for consideration. The present<br />

worth (in $ billions) of the capital requirements<br />

and benefits is shown for each alternative in the<br />

table. Determine which one(s) should be undertaken,<br />

if they are ( a ) independent and ( b ) mutually<br />

exclusive.<br />

Annual cost, $<br />

per year<br />

Annual benefits,<br />

$ per year<br />

B/C ratio (alternative<br />

vs. DN)<br />

A B C D E F<br />

8000 25,000 15,000 32,000 17,000 20,000<br />

? ? ? ? ? ?<br />

1.23 1.12 0.87 0.97 0.71 1.10<br />

Selected Incremental B/C Ratios<br />

A versus B 1.07<br />

A versus C 0.46<br />

A versus F 1.02<br />

B versus D 0.43<br />

B versus E 2.00<br />

B versus F 1.20<br />

C versus D 1.06<br />

C versus F 1.80


Problems 257<br />

9.44 Four mutually exclusive revenue alternatives are<br />

being compared using the B/C method. Which alternative,<br />

if any, should be selected?<br />

Alternative<br />

Initial<br />

Cost, $ Millions<br />

B/C Ratio<br />

vs. DN<br />

Incremental B/C<br />

When Compared<br />

with Alternative<br />

A B C D<br />

A 30 0.87 — 2.38 1.30 1.38<br />

B 38 1.18 — 0.58 1.13<br />

C 52 1.04 — 1.45<br />

D 81 1.16 —<br />

9.45 The city of St. Louis, Missouri, is considering various<br />

proposals regarding the disposal of used tires.<br />

All of the proposals involve shredding, but the<br />

charges for the service and the handling of the tire<br />

shreds differ in each plan. An incremental B/C<br />

analysis was initiated but never completed.<br />

( a) Fill in all the missing blanks in the table.<br />

(b) Determine which alternative should be selected.<br />

Alternative<br />

PW of<br />

Costs,$<br />

PW of<br />

Benefits, $<br />

PW of<br />

Disbenefits, $<br />

B/C<br />

Ratio<br />

Incremental<br />

B/C When<br />

Compared with<br />

Alternative<br />

J K L M<br />

J 20 ? 1 1.05 — ? ? ?<br />

K 23 28 ? 1.13 — ? ?<br />

L 28 35 3 ? — ?<br />

M ? 51 4 1.34 —<br />

Cost-Effectiveness Analysis<br />

9.46 In an effort to improve productivity in a large<br />

semiconductor manufacturing plant, the plant<br />

manager decided to undertake on a trial basis a series<br />

of actions directed toward improving employee<br />

morale. Six different strategies were implemented,<br />

such as increased employee autonomy,<br />

flexible work schedules, improved training, company<br />

picnics, electronic suggestion box, and better<br />

work environment. Periodically, the company surveyed<br />

the employees to measure the change in morale.<br />

The measure of effectiveness is the difference<br />

between the number of employees who rate their<br />

job satisfaction as very high and those who rate it<br />

very low. The per-employee cost of each strategy<br />

(identified as A through F) and the resultant measurement<br />

score are shown in the next column.<br />

The manager has a maximum of $50 per employee<br />

to spend on the permanent implementation of as<br />

many of the strategies as are justified from both the<br />

effectiveness and economic viewpoints. Determine<br />

which strategies are the best to implement.<br />

(Hand or spreadsheet solution is acceptable, as you<br />

are instructed.)<br />

Strategy Cost/Employee, $ Measurement Score<br />

A 5.20 50<br />

B 23.40 182<br />

C 3.75 40<br />

D 10.80 75<br />

E 8.65 53<br />

F 15.10 96<br />

9.47 There are a number of techniques to help people<br />

stop smoking, but their cost and effectiveness vary<br />

widely. One accepted measure of effectiveness of<br />

a program is the percentage of enrollees quitting.<br />

The table below shows several techniques touted<br />

as effective stop-smoking methods, some historical<br />

data on the approximate cost of each program<br />

per person, and the percentage of people smokefree<br />

3 months after the program ended.<br />

Technique<br />

The Cancer Society provides annual cost-offset<br />

funding to cancer patients so more people can afford<br />

these programs. A large clinic in St. Louis has the<br />

capacity to treat each year the number of people<br />

shown. If the clinic plans to place a proposal with the<br />

Cancer Society to treat a specified number of people<br />

annually, estimate the amount of money the clinic<br />

should ask for in its proposal to do the following:<br />

( a) Conduct programs at the capacity level<br />

for the technique with the lowest costeffectiveness<br />

ratio.<br />

(b) Offer programs using as many techniques as<br />

possible to treat up to 1300 people per year<br />

using the most cost-effective techniques.<br />

Cost, $/<br />

Enrollee<br />

% Quitting<br />

Treatment<br />

Capacity per Year<br />

Acupuncture 700 9 250<br />

Subliminal message 150 1 500<br />

Aversion therapy 1700 10 200<br />

Outpatient clinic 2500 39 400<br />

In-patient clinic 1800 41 550<br />

Nicotine replacement<br />

therapy (NRT)<br />

1300 20 100<br />

9.48 The cost in $ per year and effectiveness measure<br />

in items salvaged per year for four mutually<br />

exclusive service sector alternatives have been<br />

collected. ( a ) Calculate the cost effectiveness ratio<br />

for each alternative, and ( b ) use the CER to identify<br />

the best alternative.<br />

Alternative Cost C, $/Year Salvaged Items/Year E<br />

W 355 20<br />

X 208 17<br />

Y 660 41<br />

Z 102 7<br />

9.49 An engineering student has only 30 minutes before<br />

the final exam in statics and dynamics. He wants to


258 Chapter 9 Benefit/Cost Analysis and Public Sector Economics<br />

get help solving a type of problem that he knows<br />

will be on the test from the professor’s review during<br />

the previous class. There is time for using only<br />

one method of assistance before the exam; he must<br />

select well. In a rapid process of estimation, he determines<br />

how many minutes it would take for each<br />

method of assistance and how many points it might<br />

gain for him on the final. The method and estimates<br />

follow. Where should he seek help to be<br />

most effective?<br />

Assistance from<br />

Minutes for Assistance Points Gained<br />

Teaching assistant 15 15<br />

Course slides on Web 20 10<br />

Friend in class 10 5<br />

Professor 20 15<br />

Public Sector Ethics<br />

9.50 During the design and specifications development<br />

stages of a remote meter reading system for residential<br />

electricity use (a system that allows monthly<br />

usage to be transmitted via phone lines with no<br />

need to physically view meters), the two engineers<br />

working on the project for the city of Forest Ridge<br />

noted something different from what they expected.<br />

The first, an electrical/software engineer, noted that<br />

the city liaison staff member provided all the information<br />

on the software options, but only one option,<br />

the one from Lorier Software, was ever discussed<br />

and detailed. The second designer, an<br />

industrial/systems engineer, further noted that all<br />

the hardware specifications provided to them by<br />

this same liaison came from the same distributor,<br />

namely, Delsey Enterprises. Coincidently, at a<br />

weekend family picnic for city employees, to which<br />

the engineers had been invited, they met a couple<br />

named Don Delsey and Susan Lorier. Upon review,<br />

they learned that Don is the son-in-law of the city<br />

liaison and Susan is his stepdaughter. Based on<br />

these observations and before they complete the<br />

system design and specifications, what should the<br />

two engineers do, if anything, about their suspicions<br />

that the city liaison person is trying to bias the<br />

design to favor of the use of his relatives’ software<br />

and hardware businesses?<br />

9.51 Explain the difference between public policy making<br />

and public planning.<br />

9.52 Since transportation via automobile was introduced,<br />

drivers throughout the country of Yalturia in eastern<br />

Europe have driven on the left side of the road. Recently<br />

the Yalturian National Congress passed a law<br />

that within 3 years, a right-hand driving convention<br />

will be adopted and implemented throughout the<br />

country. This is a major policy change for the country<br />

and will require significant public planning and<br />

project development to implement successfully and<br />

safely. Assume you are the lead engineering consultant<br />

from Halcrow Engineers, responsible for developing<br />

and describing many of these major projects.<br />

Identify six of the projects you deem necessary.<br />

State a name and provide a one- or two-sentence<br />

description of each project.<br />

ADDITIONAL PROBLEMS AND FE EXAM REVIEW QUESTIONS<br />

9.53 All of the following are examples of public sector<br />

projects, except:<br />

(a) Bridges (b) Emergency relief<br />

(c) Prisons (d) Oil wells<br />

9.54 All of the following are usually associated with<br />

public sector projects except:<br />

(a) Funding from taxes (b) Profit<br />

(c) Disbenefits<br />

(d) Infinite life<br />

9.55 All of the following would be examples of public<br />

projects except:<br />

(a) Air traffic control system<br />

(b) Establishing a dot.com company<br />

(c) Dam with irrigation canals<br />

(d) Mass transit system<br />

9.56 In a conventional B/C ratio, revenue received by<br />

the government from admission to national parks<br />

should be:<br />

(a) Added to benefits in the numerator<br />

(b) Subtracted from benefits in the numerator<br />

(c)<br />

(d)<br />

Subtracted from costs in the denominator<br />

Added to costs in the denominator<br />

9.57 In a conventional B/C ratio:<br />

(a) Disbenefits and M&O costs are subtracted<br />

from benefits.<br />

(b) Disbenefits are subtracted from benefits and<br />

M&O costs are included in costs.<br />

(c) Disbenefits and M&O costs are added to<br />

costs.<br />

(d) Disbenefits are added to costs and M&O<br />

costs are subtracted from benefits.<br />

9.58 In a modified B/C ratio:<br />

( a) Disbenefits are put in the denominator.<br />

( b) Benefits are subtracted from costs.<br />

( c) M&O costs are put in the denominator.<br />

( d) M&O costs are put in the numerator.<br />

9.59 If two mutually exclusive alternatives have B/C<br />

ratios of 1.4 and 1.5 for the lower- and higher-cost<br />

alternatives, respectively, the following is correct:


Case Study 259<br />

( a) The B/C ratio on the increment between<br />

them is equal to 1.5.<br />

( b) The B/C ratio on the increment between<br />

them is between 1.4 and 1.5.<br />

( c) The B/C ratio on the increment between<br />

them is less than 1.5.<br />

( d) The higher-cost alternative is the better one<br />

economically.<br />

9.60 In evaluating three independent alternatives by the B/C<br />

method, the alternatives were ranked A, B, and C,<br />

respectively, in terms of increasing cost, and the following<br />

results were obtained for overall B/C ratios: 1.1,<br />

0.9, and 1.3. On the basis of these results, you should:<br />

( a) Select only alternative A<br />

( b) Select only alternative C<br />

( c) Compare A and C incrementally<br />

( d) Select alternatives A and C<br />

9.61 An alternative has the following cash flows:<br />

Benefits of $50,000 per year<br />

Disbenefits of $27,000 per year<br />

Initial cost of $250,000<br />

M&O costs of $10,000 per year<br />

If the alternative has an infinite life and the interest<br />

rate is 10% per year, the B/C ratio is closest to:<br />

( a) 0.52 (b) 0.66<br />

( c) 0.91 (d) 1.16<br />

9.62 At the interest rate of 10% per year, an alternative<br />

with the following estimates has a modified B/C<br />

ratio that is closest to:<br />

Benefits of $60,000 per year<br />

Disbenefits of $29,000 per year<br />

Amortized first cost of $20,000 per year<br />

M&O costs of $15,000 per year<br />

( a) 0.65 (b) 0.72<br />

( c) 0.80 (d) 1.04<br />

9.63 In evaluating three mutually exclusive alternatives<br />

by the B/C method, the alternatives are ranked A,<br />

B, and C, respectively, in terms of increasing cost,<br />

and the following results are obtained for the<br />

overall B/C ratios : 1.1, 0.9, and 1.06. On the basis<br />

of these results, you should:<br />

( a) Select A<br />

( b) Select C<br />

(c)<br />

(d)<br />

Select A and C<br />

Compare A and C incrementally<br />

9.64 An alternative with an infinite life has a B/C ratio<br />

of 1.5. The alternative has benefits of $50,000 per<br />

year and annual maintenance costs of $10,000 per<br />

year. The first cost of the alternative at an interest<br />

rate of 10% per year is closest to:<br />

(a) $23,300 (b) $85,400<br />

(c) $146,100 (d) $233,000<br />

9.65 Cost-effectiveness analysis (CEA) differs from<br />

cost-benefit (B/C) analysis in that:<br />

(a) CEA cannot handle multiple alternatives.<br />

(b) CEA expresses outcomes in natural units<br />

rather than in currency units.<br />

(c) CEA cannot handle independent alternatives.<br />

(d) CEA is more time-consuming and resourceintensive.<br />

9.66 Several private colleges claim to have programs<br />

that are very effective at teaching enrollees how<br />

to become entrepreneurs. Two programs, identified<br />

as program X and program Y, have produced<br />

4 and 6 persons per year, respectively, who were<br />

recognized as entrepreneurs. If the total cost of<br />

the programs is $25,000 and $33,000, respectively,<br />

the incremental cost-effectiveness ratio is<br />

closest to:<br />

(a) 6250 (b) 5500<br />

(c) 4000 (d) 1333<br />

9.67 The statements contained in a code of ethics are<br />

variously known as all of the following except:<br />

(a) Canons (b) Laws<br />

(c) Standards (d) Norms<br />

9.68 Of the following, the word not related to ethics is:<br />

( a) Virtuous (b) Honest<br />

(c) Lucrative (d) Proper<br />

9.69 All of the following are examples of unethical<br />

behavior except:<br />

(a) Offering services at prices lower than the<br />

competition<br />

(b) Price fixing<br />

(c) Bait and switch<br />

(d) Selling on the black market<br />

CASE STUDY<br />

COMPARING B/C ANALYSIS AND CEA OF TRAFFIC ACCIDENT REDUCTION<br />

Background<br />

This case study compares benefit/cost analysis and costeffectiveness<br />

analysis on the same information about highway<br />

lighting and its role in accident reduction.<br />

Poor highway lighting may be one reason that proportionately<br />

more traffic accidents occur at night. Traffic accidents<br />

are categorized into six types by severity and value.<br />

For example, an accident with a fatality is valued at


260 Chapter 9 Benefit/Cost Analysis and Public Sector Economics<br />

approximately $4 million, while an accident in which there<br />

is property damage (to the car and contents) is valued at<br />

$6000. One method by which the impact of lighting is measured<br />

compares day and night accident rates for lighted and<br />

unlighted highway sections with similar characteristics. Observed<br />

reductions in accidents seemingly caused by too low<br />

lighting can be translated into either monetary estimates of<br />

the benefits B of lighting or used as the effectiveness<br />

measure E of lighting.<br />

Information<br />

Freeway accident data were collected in a 5-year study. The<br />

property damage category is commonly the largest based on<br />

the accident rate. The number of accidents recorded on a section<br />

of highway is presented here.<br />

Number of Accidents Recorded 1<br />

Unlighted<br />

Lighted<br />

Accident Type Day Night Day Night<br />

Property damage 379 199 2069 839<br />

The ratios of night to day accidents involving property damage<br />

for the unlighted and lighted freeway sections are 199/379 <br />

0.525 and 839/2069 0.406, respectively. These results indicate<br />

that the lighting was beneficial. To quantify the benefit, the<br />

accident rate ratio from the unlighted section will be applied to<br />

the lighted section. This will yield the number of accidents that<br />

were prevented. Thus, there would have been (2069)(0.525) <br />

1086 accidents instead of 839 if there had not been lights on the<br />

freeway. This is a difference of 247 accidents. At a cost of $6000<br />

per accident, this results in a net annual benefit of<br />

B (247)($6000) $1,482,000<br />

For an effectiveness measure of number of accidents prevented,<br />

this results in E 247.<br />

To determine the cost of the lighting, it will be assumed that<br />

the light poles are center poles 67 meters apart with 2 bulbs<br />

each. The bulb size is 400 watts, and the installation cost is<br />

$3500 per pole. Since these data were collected over 87.8 kilometers<br />

of lighted freeway, the installed cost of the lighting is<br />

(with number of poles rounded off):<br />

Installation cost $3500 ( 87.8 ———<br />

0.067 )<br />

3500(1310)<br />

$4,585,000<br />

There are a total of 87.8/0.0671310 poles, and electricity<br />

costs $0.10 per kWh. Therefore, the annual power cost is<br />

Annual power cost<br />

1310 poles(2 bulbs/pole)(0.4 kilowatt/bulb)<br />

(12 hours/day)(365 days/year)<br />

($0.10/kilowatt-hour)<br />

$459,024 per year<br />

The data were collected over a 5-year period. Therefore, the<br />

annualized cost C at i 6% per year is<br />

Total annual cost $4,585,000( A/P ,6%,5)<br />

459,024<br />

$1,547,503<br />

If a benefit/cost analysis is the basis for a decision on additional<br />

lighting, the B/C ratio is<br />

B/C —————<br />

1,482,000<br />

1,547,503 0.96<br />

Since B/C 1.0, the lighting is not justified. Consideration<br />

of other categories of accidents is necessary to obtain a better<br />

basis for decisions. If a cost-effectiveness analysis (CEA) is<br />

applied, due to a judgment that the monetary estimates for<br />

lighting’s benefit is not accurate, the C/E ratio is<br />

C/E —————<br />

1,547,503 6265<br />

247<br />

This can serve as a base ratio for comparison when an incremental<br />

CEA is performed for additional accident reduction proposals.<br />

These preliminary B/C and C/E analyses prompted the<br />

development of four lighting options:<br />

W) Implement the plan as detailed above; light poles<br />

every 67 meters at a cost of $3500 per pole.<br />

X) Install poles at twice the distance apart (134 meters).<br />

This is estimated to cause the accident prevention<br />

benefit to decrease by 40%.<br />

Y) Install cheaper poles and surrounding safety guards,<br />

plus slightly lowered lumen bulbs (350 watts) at a<br />

cost of $2500 per pole; place the poles 67 meters<br />

apart. This is estimated to reduce the benefit by 25%.<br />

Z) Install cheaper equipment for $2500 per pole with<br />

350-watt lightbulbs and place them 134 meters apart.<br />

This plan is estimated to reduce the accident prevention<br />

measure by 50% from 247 to 124.<br />

Case Study Exercises<br />

Determine if a definitive decision on lighting can be determined<br />

by doing the following:<br />

1. Use a benefi t/cost analysis to compare the four alternatives<br />

to determine if any are economically justified.<br />

2. Use a cost-effectiveness analysis to compare the four<br />

alternatives.<br />

From an understanding viewpoint, consider the following:<br />

3. How many property-damage accidents could be prevented<br />

on the unlighted portion if it were lighted?<br />

4. What would the lighted, night-to-day accident ratio<br />

have to be to make alternative Z economically justified<br />

by the B/C ratio?<br />

5. Discuss the analysis approaches of B/C and C/E. Does<br />

one seem more appropriate in this type of situation than<br />

the other? Why? Can you think of other bases that might<br />

be better for decisions for public projects such as this one?<br />

1 Portion of data reported in Michael Griffin, “Comparison of the Safety of Lighting on Urban Freeways,” Public Roads, vol. 58, pp. 8–15, 1994.


LEARNING STAGE 2: EPILOGUE<br />

Selecting the Basic Analysis Tool<br />

In the previous five chapters, several equivalent evaluation techniques have been discussed. Any<br />

method—PW, AW, FW, ROR, or B/C—can be used to select one alternative from two or more and<br />

obtain the same, correct answer. Only one method is needed to perform the engineering economy<br />

analysis, because any method, correctly performed, will select the same alternative. Yet different information<br />

about an alternative is available with each different method. The selection of a method and<br />

its correct application can be confusing.<br />

Table LS2–1 gives a recommended evaluation method for different situations, if it is not specified<br />

by the instructor in a course or by corporate practice in professional work. The primary criteria for<br />

selecting a method are speed and ease of performing the analysis. Interpretation of the entries in each<br />

column follows.<br />

Evaluation period: Most private sector alternatives (revenue and cost) are compared over their<br />

equal or unequal estimated lives, or over a specific period of time. Public sector projects are<br />

commonly evaluated using the B/C ratio and usually have long lives that may be considered infinite<br />

for economic computation purposes.<br />

Type of alternatives: Private sector alternatives have cash flow estimates that are revenuebased<br />

(includes income and cost estimates) or cost-based (cost estimates only). For cost alternatives,<br />

the revenue cash flow series is assumed to be equal for all alternatives. For public sector<br />

projects, the difference between costs and timing is used to select one alternative over<br />

another. Service sector projects for which benefits are estimated using a nonmonetary effectiveness<br />

measure are usually evaluated with a method such as cost-effectiveness analysis. This<br />

applies to all evaluation periods.<br />

Recommended method: Whether an analysis is performed by hand, calculator, or spreadsheet, the<br />

method(s) recommended in Table LS2–1 will correctly select one alternative from two or more as<br />

TABLE LS2–1<br />

Recommended Method to Compare Mutually Exclusive Alternatives,<br />

Provided the Method Is Not Preselected<br />

Evaluation Period Type of Alternatives Recommended Method<br />

Equal lives of<br />

alternatives<br />

Unequal lives of<br />

alternatives<br />

Series to<br />

Evaluate<br />

Revenue or cost AW or PW Cash flows<br />

Public sector B/C, based on AW or PW Incremental<br />

cash flows<br />

Revenue or cost AW Cash flows<br />

Public sector B/C, based on AW Incremental<br />

cash flows<br />

Study period Revenue or cost AW or PW Updated cash<br />

flows<br />

Public sector B/C, based on AW or PW Updated incremental<br />

cash flows<br />

Long to infinite Revenue or cost AW or PW Cash flows<br />

Public sector B/C, based on AW Incremental<br />

cash flows


262 Chapter 9 Benefit/Cost Analysis and Public Sector Economics<br />

rapidly as possible. Any other method can be applied subsequently to obtain additional information<br />

and, if needed, verification of the selection. For example, if lives are unequal and the<br />

rate of return is needed, it is best to first apply the AW method at the MARR and then determine<br />

the selected alternative’s i * using the same AW relation with i as the unknown.<br />

Series to evaluate: The estimated cash flow series for one alternative and the incremental<br />

series between two alternatives are the only two options for present worth or annual worth<br />

evaluation. For spreadsheet analyses, this means that the NPV or PV functions (for present<br />

worth) or the PMT function (for annual worth) is applied. The word updated is added as a<br />

reminder that a study period analysis requires that cash flow estimates (especially salvage/<br />

market values) be reexamined and updated before the analysis is performed.<br />

Once the evaluation method is selected, a specific procedure must be followed. These procedures<br />

were the primary topics of the last five chapters. Table LS2–2 summarizes the important<br />

TABLE LS2–2 Characteristics of an Economic Analysis of Mutually Exclusive Alternatives Once the<br />

Evaluation Method Is Determined<br />

Evaluation<br />

Method<br />

Equivalence<br />

Relation<br />

Lives of<br />

Alternatives<br />

Time<br />

Period for<br />

Analysis<br />

Series to<br />

Evaluate<br />

Rate of<br />

Return;<br />

Interest Rate<br />

Decision<br />

Guideline:<br />

Select 1<br />

PW Equal Lives Cash flows MARR Numerically<br />

largest PW<br />

PW Unequal LCM Cash flows MARR Numerically<br />

largest PW<br />

Present worth PW Study period Study period Updated MARR<br />

Numerically<br />

CC<br />

Long to<br />

infinite<br />

cash flows<br />

largest PW<br />

Infinity Cash flows MARR Numerically<br />

largest CC<br />

Future worth FW Same as present worth for equal lives, unequal lives,<br />

and study period<br />

Numerically<br />

largest FW<br />

AW Equal or<br />

unequal<br />

Lives Cash flows MARR Numerically<br />

largest AW<br />

Annual worth AW Study period Study period Updated<br />

cash flows<br />

MARR<br />

Numerically<br />

largest AW<br />

AW Long to<br />

infinite<br />

Infinity Cash flows MARR Numerically<br />

largest AW<br />

PW or AW Equal Lives Incremental<br />

cash flows<br />

Find i* Last i * <br />

MARR<br />

PW or AW Unequal LCM of pair Incremental<br />

cash flows<br />

Find i * Last i * <br />

MARR<br />

Rate of return AW Unequal Lives Cash flows Find i * Last i * <br />

MARR<br />

PW or AW Study period Study period Updated<br />

incremental<br />

cash flows<br />

Find i * Last i * <br />

MARR<br />

PW Equal or<br />

unequal<br />

Benefit/cost AW Equal or<br />

unequal<br />

AW or PW Long to<br />

infinite<br />

LCM of pairs<br />

Lives<br />

Infinity<br />

Incremental<br />

cash flows<br />

Incremental<br />

cash flows<br />

Incremental<br />

cash flows<br />

Discount rate<br />

Discount rate<br />

Discount rate<br />

Last B/C <br />

1.0<br />

Last B/C <br />

1.0<br />

Last B/C <br />

1.0<br />

1 Lowest equivalent cost or largest equivalent income.


Learning Stage 2: Epilogue 263<br />

elements of the procedure for each method—PW, AW, ROR, and B/C. FW is included as an extension<br />

of PW. The meaning of the entries in Table LS2–2 follows.<br />

Equivalence relation The basic equation written to perform any analysis is either a PW<br />

or an AW relation. The capitalized cost (CC) relation is a PW relation for infinite life, and<br />

the FW relation is likely determined from the PW equivalent value. Additionally, as we<br />

learned in Chapter 6, AW is simply PW times the A/P factor over the LCM or study<br />

period.<br />

Lives of alternatives and time period for analysis The length of time for an evaluation (the<br />

n value) will always be one of the following: equal lives of the alternatives, LCM of unequal<br />

lives, specified study period, or infinity because the lives are very long.<br />

• PW analysis always requires the LCM of compared alternatives.<br />

• Incremental ROR and B/C methods require the LCM of the two alternatives being compared.<br />

• The AW method allows analysis over the respective alternative lives.<br />

• CC analysis has an infinite time line and uses the relation P Ai.<br />

The one exception is for the incremental ROR method for unequal-life alternatives using an AW<br />

relation for incremental cash fl ows . The LCM of the two alternatives compared must be used.<br />

This is equivalent to using an AW relation for the actual cash fl ows over the respective lives. Both<br />

approaches find the incremental rate of return i *.<br />

Series to evaluate Either the estimated cash flow series or the incremental series is used to<br />

determine the PW value, the AW value, the i * value, or the B/C ratio.<br />

Rate of return (interest rate) The MARR value must be stated to complete the PW, FW,<br />

or AW method. This is also correct for the discount rate for public sector alternatives analyzed<br />

by the B/C ratio. The ROR method requires that the incremental rate be found in order<br />

to select one alternative. It is here that the dilemma of multiple rates appears, if the sign tests<br />

indicate that a unique, real number root does not necessarily exist for a nonconventional<br />

series.<br />

Decision guideline The selection of one alternative is accomplished using the general guideline<br />

in the rightmost column. Always select the alternative with the numerically largest PW,<br />

FW, or AW value. This is correct for both revenue and cost alternatives. The incremental cash<br />

flow methods—ROR and B/C—require that the largest initial cost and incrementally justified<br />

alternative be selected, provided it is justified against an alternative that is itself justified. This<br />

means that the i * exceeds MARR, or the B/C exceeds 1.0.<br />

EXAMPLE LS2–1<br />

Read through the problem statement of the following examples, neglecting the evaluation<br />

method used in the example. Determine which evaluation method is probably the fastest and<br />

easiest to apply. Is this the method used in the example? (a) 8.6, (b) 6.5, (c) 5.8, (d) 5.4.<br />

Solution<br />

Referring to the contents of Table LS2–1, the following methods should be applied first.<br />

(a) Example 8.6 involves four revenue alternatives with equal lives. Use the AW or PW value<br />

at the MARR of 10%. The incremental ROR method was applied in the example.<br />

(b) Example 6.5 requires selection between three public sector alternatives with unequal lives,<br />

one of which is 50 years and another is infinite. The B/C ratio of AW values is the best<br />

choice. This is how the problem was solved.


264 Chapter 9 Benefit/Cost Analysis and Public Sector Economics<br />

(c) Since Example 5.8 involves two cost alternatives with one having a long life, either AW or<br />

PW can be used. Since one life is long, capitalized cost, based on P A/i, is best in this<br />

case. This is the method applied in the example.<br />

(d) Example 5.4 is in the series of progressive examples. It involves 5-year and 10-year cost<br />

alternatives. The AW method is the best to apply in this case. The PW method for the LCM<br />

of 10 years and a study period of 5 years were both presented in the example.


LEARNING STAGE 3<br />

Making Better Decisions<br />

LEARNING STAGE 3<br />

Making Better<br />

Decisions<br />

CHAPTER 10<br />

Project Financing and<br />

Noneconomic<br />

Attributes<br />

CHAPTER 11<br />

Replacement and<br />

Retention Decisions<br />

CHAPTER 12<br />

Independent<br />

Projects with<br />

Budget Limitation<br />

CHAPTER 13<br />

Breakeven and<br />

Payback Analysis<br />

Most of the evaluations in the real world involve more than<br />

a simple economic selection of new assets or projects. The<br />

chapters in this stage introduce information-gathering and<br />

techniques that make decisions better. For example, noneconomic<br />

parameters can be introduced into the project analysis study<br />

through multiple attribute evaluation, and the appropriate MARR<br />

for a corporation or type of alternative can tailor and improve the<br />

economic decision.<br />

The future is certainly not exact. However, techniques such as<br />

replacement/retention studies, breakeven analysis, and payback<br />

analysis help make informed decisions about future uses of<br />

existing assets and systems.<br />

After completing these chapters, you will be able to go beyond<br />

the basic alternative analysis tools of the previous chapters. The<br />

techniques covered in this learning stage take into consideration the<br />

moving targets of change over time.<br />

Important note: If asset depreciation and taxes are to be considered<br />

by an after-tax analysis, Chapters 16 and 17 should be covered<br />

before or in conjunction with these chapters.


CHAPTER 10<br />

Project<br />

Financing and<br />

Noneconomic<br />

Attributes<br />

LEARNING OUTCOMES<br />

Purpose: Explain debt and equity financing, select the appropriate MARR, and consider multiple attributes when<br />

comparing alternatives.<br />

SECTION TOPIC LEARNING OUTCOME<br />

10.1 COC and MARR • Explain the relation between cost of capital and<br />

the MARR; explain why MARR values vary.<br />

10.2 D-E mix and WACC • Understand debt-to-equity mix and calculate the<br />

weighted average cost of capital.<br />

10.3 Cost of debt capital • Estimate the cost of debt capital, considering tax<br />

advantages.<br />

10.4 Cost of equity capital • Estimate the cost of equity capital and describe its<br />

relation to MARR and WACC.<br />

10.5 High D-E mixes • Demonstrate the connection between high D-E<br />

mixes and corporate (or personal) financial risk.<br />

10.6 Multiple attributes • Develop weights for multiple attributes used in<br />

alternative evaluation and selection.<br />

10.7 Additive weights • Apply the weighted attribute method to<br />

alternative evaluations that include noneconomic<br />

attributes.


T<br />

his chapter discusses the different ways to finance a project through debt and<br />

equity sources and explains how the MARR is established. The descriptions here<br />

complement the introductory material of Chapter 1 on the same topics. Some of<br />

the parameters specified earlier are unspecified here, and in future chapters. As a result,<br />

some of the textbook aspects apparent in previous chapters are removed, thus coming closer<br />

to treating the more complex, real-world situations in which professional practice and decision<br />

making occur.<br />

Until now, only one dimension—the economic one—has been the basis for judging the<br />

economic viability of one project, or the selection basis from two or more alternatives. In this<br />

chapter, guidelines and techniques explain the determination and use of multiple (noneconomic)<br />

attributes helpful in selecting between alternatives.<br />

10.1 MARR Relative to the Cost of Capital<br />

The MARR value used in alternative evaluation is one of the most important parameters of<br />

a study. In Chapter 1 the MARR was described relative to the weighted costs of debt and<br />

equity capital. This and the next four sections explain how to establish a MARR under varying<br />

conditions.<br />

To form the basis for a realistic MARR, the types and cost of each source of project financing<br />

should be understood and estimated. There is a strong connection between the costs of debt and<br />

equity capital and the MARR used to evaluate one or more alternatives, whether they are mutually<br />

exclusive or independent. There are several terms and relationships important to the understanding<br />

of project financing and the MARR that is specified to evaluate projects using PW, AW,<br />

FW, or B/C methods. (Reference to Section 1.9 will complement the following material.)<br />

The cost of capital is the weighted average interest rate paid based on the proportion of<br />

i nvestment capital from debt and equity sources .<br />

The MARR is then set relative to the cost of capital. The MARR can be set for one project, a<br />

series of projects, a division of a corporation, or the entire company. MARR values change<br />

over time due to changing circumstances.<br />

When no specific MARR is established, the estimated net cash flows and available capital<br />

establish an inherent MARR. This rate is determined by finding the ROR ( i *) value of the<br />

project cash flows. This rate is utilized as the opportunity cost , which is the ROR of the first<br />

project not funded due to the lack of capital funds.<br />

Cost of capital<br />

MARR<br />

Opportunity cost<br />

Before we discuss cost of capital, we review the two primary sources of capital.<br />

Debt capital represents borrowing from outside the company, with the principal repaid at a<br />

stated interest rate following a specified time schedule. Debt financing includes borrowing via<br />

bonds, loans, and mortgages. The lender does not share in the profits made using the debt<br />

funds, but there is risk in that the borrower could default on part of or all the borrowed funds.<br />

The amount of outstanding debt financing is indicated in the liabilities section of the corporate<br />

balance sheet.<br />

Equity capital is corporate money comprised of the funds of owners and retained earnings.<br />

Owners’ funds are further classified as common and preferred stock proceeds or owners’ capital<br />

for a private (non-stock- issuing) company. Retained earnings are funds previously retained<br />

in the corporation for capital investment. The amount of equity is indicated in the net worth<br />

section of the corporate balance sheet.<br />

To illustrate the relation between cost of capital and MARR, assume a new greenhouse gas<br />

emission control system will be completely financed by a $25,000,000 bond issue (100% debt<br />

financing), and assume the dividend rate on the bonds is 8%. Therefore, the cost of debt capital<br />

is 8% as shown in Figure 10–1 . This 8% is the minimum for MARR. Management may increase<br />

this MARR in increments that reflect its desire for added return and its perception of risk. For<br />

example, management may add an amount for all capital commitments in this area. Suppose this<br />

amount is 2%. This increases the expected return to 10% ( Figure 10–1 ). Also, if the risk associated<br />

with the investment is considered substantial enough to warrant an additional 1% return requirement,<br />

the final MARR is 11%.


268 Chapter 10 Project Financing and Noneconomic Attributes<br />

Established<br />

MARR<br />

11%<br />

Risk factor added<br />

1%<br />

10% Expected return<br />

requirement added<br />

2%<br />

Minimum<br />

MARR<br />

8% Cost of capital<br />

Figure 10–1<br />

A fundamental relation between cost of capital<br />

and MARR used in practice.<br />

The recommended approach does not follow the logic presented above. Rather, the cost of<br />

capital (8% here) should be the established MARR. Then the i * value is determined from the<br />

estimated net cash flows. Using this approach, suppose the control system is estimated to return<br />

11%. Now, additional return requirements and risk factors are considered to determine if 3%<br />

above the MARR of 8% is sufficient to justify the capital investment. After these considerations,<br />

if the project is not funded, the effective MARR is now 11%. This is the opportunity cost discussed<br />

previously—the unfunded project i * has established the effective MARR for emission<br />

control system alternatives at 11%, not 8%.<br />

The setting of the MARR for an economy study is not an exact process. The debt and equity<br />

capital mix changes over time and between projects. Also, the MARR is not a fixed value established<br />

corporatewide. It is altered for different opportunities and types of projects. For example,<br />

a corporation may use a MARR of 10% for evaluating the purchase of assets (equipment, cars)<br />

and a MARR of 20% for expansion investments, such as acquiring smaller companies.<br />

The effective MARR varies from one project to another and through time because of factors<br />

such as the following:<br />

Project risk. Where there is greater risk (perceived or actual) associated with proposed<br />

projects, the tendency is to set a higher MARR. This is encouraged by the higher cost of debt<br />

capital for projects considered risky. This usually means that there is some concern that the<br />

project will not realize its projected revenue requirements.<br />

Investment opportunity. If management is determined to expand in a certain area, the<br />

MARR may be lowered to encourage investment with the hope of recovering lost revenue in<br />

other areas. This common reaction to investment opportunity can create havoc when the<br />

guidelines for setting a MARR are too strictly applied. Flexibility becomes very important.<br />

Government intervention. Depending upon the state of the economy, international relations,<br />

and a host of other factors, the federal government (and possibly lower levels) can dictate<br />

the forces and direction of the free market. This may occur through price limits, subsidies,<br />

import tariffs, and limitation on availability. Both short-term and long-term government interventions<br />

are commonly present in different areas of the economy. Examples are steel imports,<br />

foreign capital investment, car imports, and agricultural product exports. During the time that<br />

such government actions are in force, there is a strong impact to increase or decrease taxes,<br />

prices, etc., thus tending to move the MARR up or down.<br />

Tax structure. If corporate taxes are rising (due to increased profits, capital gains, local<br />

taxes, etc.), pressure to increase the MARR is present. Use of after-tax analysis may assist in<br />

eliminating this reason for a fluctuating MARR, since accompanying business expenses will<br />

tend to decrease taxes and after-tax costs.<br />

Limited capital. As debt and equity capital become limited, the MARR is increased. If the<br />

demand for limited capital exceeds supply, the MARR may tend to be set even higher. The<br />

opportunity cost has a large role in determining the MARR actually used.


10.2 Debt-Equity Mix and Weighted Average Cost of Capital 269<br />

Market rates at other corporations. If the MARR increases at other corporations, especially<br />

competitors, a company may alter its MARR upward in response. These variations are<br />

often based on changes in interest rates for loans, which directly impact the cost of capital.<br />

If the details of after-tax analysis are not of interest, but the effects of income taxes are important,<br />

the MARR may be increased by incorporating an effective tax rate using the formula<br />

Before-tax MARR <br />

after-tax MARR<br />

————————<br />

1 tax rate<br />

[10.1]<br />

The total or effective tax rate, including federal, state, and local taxes, for most corporations is in<br />

the range of 30% to 50%. If an after-tax rate of return of 10% is required and the effective tax rate<br />

is 35%, the MARR for the before-tax economic analysis is 10%/(1 0.35) 15.4%.<br />

EXAMPLE 10.1<br />

Twin brother and sister, Carl and Christy graduated several years ago from college. Carl, an<br />

architect, has worked in home design with Bulte Homes since graduation. Christy, a civil engineer,<br />

works with Butler Industries in structural components and analysis. They both reside in<br />

Richmond, Virginia. They have started a creative e-commerce network through which<br />

Virginia-based builders can buy their “spec home” plans and construction materials much<br />

more cheaply. Carl and Christy want to expand into a regional e-business corporation. They<br />

have gone to the Bank of America (BA) in Richmond for a business development loan. Identify<br />

some factors that might cause the loan rate to vary when BA provides the quote. Also, indicate<br />

any impact on the established MARR when Carl and Christy make economic decisions for<br />

their business.<br />

Solution<br />

In all cases the direction of the loan rate and the MARR will be the same. Using the six factors<br />

mentioned above, some loan rate considerations are as follows:<br />

Project risk: The loan rate may increase if there has been a noticeable downturn in housing<br />

starts, thus reducing the need for the e-commerce connection.<br />

Investment opportunity: The rate could increase if other companies offering similar services<br />

have already applied for a loan at other BA branches regionally or nationwide.<br />

Government intervention: The loan rate may decrease if the federal government has<br />

recently offered Federal Reserve loan money at low rates to banks. The intervention may<br />

be designed to boost the housing economic sector in an effort to offset a significant<br />

slowdown in new home construction.<br />

Taxes: If the state recently removed house construction materials from the list of items<br />

subject to sales tax, the rate might be lowered slightly.<br />

Capital limitation: Assume the computer equipment and software rights held by Carl and<br />

Christy were bought with their own funds and there are no outstanding loans. If additional<br />

equity capital is not available for this expansion, the rate for the loan (debt capital)<br />

should be lowered.<br />

Market loan rates: The local BA branch probably obtains its development loan money<br />

from a large national pool. If market loan rates to this BA branch have increased, the rate<br />

for this loan will likely increase, because money is becoming “tighter.”<br />

10.2 Debt-Equity Mix and Weighted<br />

Average Cost of Capital<br />

The debt-to-equity (D-E) mix identifies the percentages of debt and equity financing for a corporation.<br />

A company with a 40–60 D-E mix has 40% of its capital originating from debt capital<br />

sources (bonds, loans, and mortgages) and 60% derived from equity sources (stocks and retained


270 Chapter 10 Project Financing and Noneconomic Attributes<br />

Figure 10–2<br />

General shape of different<br />

cost of capital curves.<br />

Equity<br />

Cost of capital for each source<br />

Normal<br />

operating<br />

range<br />

WACC<br />

Minimum<br />

WACC<br />

Debt<br />

0% debt<br />

45% 100% debt<br />

Percent debt capital (D-E mix)<br />

earnings). Most projects are funded with a combination of debt and equity capital made available<br />

specifically for the project or taken from a corporate pool of capital. The weighted average cost<br />

of capital (WACC) of the pool is estimated by the relative fractions from debt and equity sources.<br />

If known exactly, these fractions are used to estimate WACC; otherwise the historical fractions<br />

for each source are used in the relation<br />

WACC (equity fraction)(cost of equity capital)<br />

(debt fraction)(cost of debt capital) [10.2]<br />

The two cost terms are expressed as percentage interest rates.<br />

Since virtually all corporations have a mixture of capital sources, the WACC is a value between<br />

the debt and equity costs of capital. If the fraction of each type of equity financing—<br />

common stock, preferred stock, and retained earnings—is known, Equation [10.2] is expanded.<br />

WACC (common stock fraction)(cost of common stock capital)<br />

(preferred stock fraction)(cost of preferred stock capital)<br />

(retained earnings fraction)(cost of retained earnings capital)<br />

(debt fraction)(cost of debt capital) [10.3]<br />

Figure 10–2 indicates the usual shape of cost of capital curves. If 100% of the capital is derived<br />

from equity or 100% is from debt sources, the WACC equals the cost of capital of that source of<br />

funds. There is virtually always a mixture of capital sources involved for any capitalization program.<br />

As an illustration only, Figure 10–2 indicates a minimum WACC at about 45% debt capital.<br />

Most firms operate over a range of D-E mixes. For example, a range of 30% to 50% debt financing<br />

for some companies may be very acceptable to lenders, with no increases in risk or MARR. However,<br />

another company may be considered “risky” with only 20% debt capital. It takes knowledge<br />

about management ability, current projects, and the economic health of the specific industry to<br />

determine a reasonable operating range of the D-E mix for a particular company.<br />

EXAMPLE 10.2<br />

Historically, Hong Kong has imported over 95% of its fresh vegetables each day. In an effort<br />

to develop sustainable and renewable vegetable sources, a new commercial vertical crop technology<br />

is being installed through a public-private partnership with Valcent Products. 1 For<br />

1<br />

“Valcent Announces Agreement to Supply Verticrop Vertical Farming Technology to Hong Kong’s<br />

VF Innovations Ltd.,” www.valcent.net, June 16, 2010 news release.


10.3 Determination of the Cost of Debt Capital 271<br />

illustration purposes, assume that the present worth of the total system cost is $20 million with<br />

financing sources and costs as follows.<br />

Commercial loan for debt financing<br />

Retained earnings from partnering corporations<br />

Sale of stock (common and preferred)<br />

$10 million at 6.8% per year<br />

$4 million at 5.2% per year<br />

$6 million at 5.9% per year<br />

There are three existing international vertical farming projects with capitalization and WACC<br />

values as follows:<br />

Project 1: $5 million with WACC 1 7.9%<br />

Project 2: $30 million with WACC 2 10.2%<br />

Project 3: $7 million with WACC 3 4.8%<br />

Compare the WACC for the Hong Kong (HK) project with the WACC of the existing projects.<br />

Solution<br />

To apply Equation [10.3] to this new project, the fraction of equity (stock and retained earnings)<br />

and debt financing is needed. These are 0.3 for stock ($6 out of $20 million), 0.2 for retained<br />

earnings, and 0.5 for debt ($10 out of $20 million).<br />

WACC HK 0.3(5.9%) 0.2(5.2%) 0.5(6.8%) 6.210%<br />

To correctly weight the other three project WACCs by size, determine the fraction in each one<br />

of the $42 million in total capital: project 1 has $5 million/$42 million 0.119; project 2 has<br />

0.714; project 3 has 0.167. The WACC weighted by project size is WACC W .<br />

WACC W 0.119(7.9%) 0.714(10.2%) 0.167(4.8%) 9.025%<br />

The Hong Kong project has a considerably lower cost of capital than the weighted average of<br />

other projects, considering all sources of funding.<br />

The WACC value can be computed using before-tax or after-tax values for cost of capital. The<br />

after-tax method is the correct one since debt financing has a distinct tax advantage, as discussed<br />

in Section 10.3 below. Approximations of after-tax or before-tax cost of capital are made using<br />

the effective tax rate T e in the relation<br />

After-tax cost of debt capital (before-tax cost)(1 T e ) [10.4]<br />

The effective tax rate is a combination of federal, state, and local tax rates. They are reduced to a<br />

single number T e to simplify computations. Equation [10.4] may be used to approximate the cost<br />

of debt capital separately or inserted into Equation [10.2] for an after-tax WACC rate. Chapter 17<br />

treats taxes and after-tax economic analysis in detail.<br />

10.3 Determination of the Cost of Debt Capital<br />

Debt financing includes borrowing, primarily via bonds and loans. (We learned about bonds in<br />

Section 7.6.) In most industrialized countries, bond dividends and loan interest payments are taxdeductible<br />

as a corporate expense. This reduces the taxable income base upon which taxes are<br />

calculated, with the end result of less taxes paid. The cost of debt capital is, therefore, reduced<br />

because there is an annual tax savings of the expense cash flow times the effective tax rate T e .<br />

This tax savings is subtracted from the debt-capital expense in order to calculate the cost of debt<br />

capital. In formula form,<br />

Tax savings (expenses) (effective tax rate) expenses ( T e ) [10.5]<br />

Net cash flow expenses tax savings expenses (1 T e ) [10.6]<br />

To fi nd the cost of debt capital, develop a PW- or AW-based relation of the net cash flow<br />

(NCF) series with i * as the unknown. Find i * by trial and error, by calculator, or by the RATE


272 Chapter 10 Project Financing and Noneconomic Attributes<br />

or IRR function on a spreadsheet. This is the cost of debt capital used in the WACC computation,<br />

Equation [10.2].<br />

EXAMPLE 10.3<br />

AT&T will generate $5 million in debt capital by issuing five thousand $1000 8% per year<br />

10-year bonds. If the effective tax rate of the company is 30% and the bonds are discounted<br />

2%, compute the cost of debt capital ( a ) before taxes and ( b ) after taxes from the company<br />

perspective. Obtain the answers by hand and spreadsheet.<br />

Solution by Hand<br />

(a) The annual bond dividend is $1000(0.08) $80, and the 2% discounted sales price is $980<br />

now. Using the company perspective, find the i * in the PW relation<br />

0 980 80( P / A , i *,10) 1000( P / F , i *,10)<br />

i * 8.3%<br />

The before-tax cost of debt capital is i * 8.3%, which is slightly higher than the<br />

8% bond interest rate, because of the 2% sales discount.<br />

( b) With the allowance to reduce taxes by deducting the bond dividend, Equation [10.5]<br />

shows a tax savings of $80(0.3) $24 per year. The bond dividend amount for the PW<br />

relation is now $80 24 $56. Solving for i * after taxes reduces the cost of debt<br />

capital to 5.87%.<br />

Solution by Spreadsheet<br />

Figure 10–3 is a spreadsheet image for both before-tax (column B) and after-tax (column C)<br />

analysis using the IRR function. The after-tax net cash flow is calculated using Equation [10.6]<br />

with T e 0.3.<br />

Bond dividend before taxes<br />

1000 * 0.08<br />

Bond dividend after taxes<br />

(1000 * 0.08) * (1 0.3)<br />

IRR(C3:C13)<br />

Figure 10–3<br />

Use of IRR function to determine cost of debt capital before taxes and after taxes, Example10.3.<br />

EXAMPLE 10.4<br />

LST Trading Company will purchase a $20,000 ten-year-life asset. Company managers<br />

have decided to put $10,000 down now from retained earnings and borrow $10,000 at an<br />

interest rate of 6%. The simplified loan repayment plan is $600 in interest each year, with<br />

the entire $10,000 principal paid in year 10. ( a ) What is the after-tax cost of debt capital if<br />

the effective tax rate is 42%? ( b ) How are the interest rate and cost of debt capital used to<br />

calculate WACC?


10.4 Determination of the Cost of Equity Capital and the MARR 273<br />

Solution<br />

( a) The after-tax net cash flow for interest on the $10,000 loan is an annual amount of<br />

600(1 0.42) $348 by Equation [10.6]. The loan repayment is $10,000 in year 10.<br />

PW is used to estimate a cost of debt capital of 3.48%.<br />

0 10,000 348( P / A , i *,10) 10,000( P / F , i *,10)<br />

(b) The 6% annual interest on the $10,000 loan is not the WACC because 6% is paid only on<br />

the borrowed funds. Nor is 3.48% the WACC, since it is only the cost of debt capital. The<br />

cost of the $10,000 equity capital is needed to determine the WACC.<br />

10.4 Determination of the Cost of Equity<br />

Capital and the MARR<br />

Equity capital is usually obtained from the following sources:<br />

Sale of preferred stock<br />

Sale of common stock<br />

Use of retained earnings<br />

Use of owner’s private capital<br />

The cost of each type of financing is estimated separately and entered into the WACC computation.<br />

A summary of one commonly accepted way to estimate each source’s cost of capital is<br />

presented here. One additional method for estimating the cost of equity capital via common stock<br />

is presented. There are no tax savings for equity capital, because dividends paid to stockholders<br />

and owners are not tax-deductible.<br />

Issuance of preferred stock carries with it a commitment to pay a stated dividend annually.<br />

The cost of capital is the stated dividend percentage, for example, 10%, or the dividend amount<br />

divided by the price of the stock. Preferred stock may be sold at a discount to speed the sale, in<br />

which case the actual proceeds from the stock should be used as the denominator. For example,<br />

if a 10% dividend preferred stock with a value of $200 is sold at a 5% discount for $190 per<br />

share, there is a cost of equity capital of ($20/$190) 100% 10.53%.<br />

Estimating the cost of equity capital for common stock is more involved. The dividends paid<br />

are not a true indication of what the stock issue will actually cost in the future. Usually a valuation<br />

of the common stock is used to estimate the cost. If R e is the cost of equity capital (in<br />

decimal form),<br />

first-year dividend<br />

R e ———————— expected dividend growth rate<br />

price of stock<br />

DV 1<br />

——<br />

P g [10.7]<br />

The growth rate g is an estimate of the annual increase in returns that the shareholders receive.<br />

Stated another way, it is the compound growth rate on dividends that the company believes is<br />

required to attract stockholders. For example, assume a multinational corporation plans to raise<br />

capital through its U.S. subsidiary for a new plant in South America by selling $2,500,000 worth<br />

of common stock valued at $20 each. If a 5% or $1 dividend is planned for the first year and an<br />

appreciation of 4% per year is anticipated for future dividends, the cost of capital for this common<br />

stock issue from Equation [10.7] is 9%.<br />

R e ——<br />

20 1 0.04 0.09<br />

The retained earnings and owner’s funds cost of equity capital is usually set equal to the<br />

common stock cost, since it is the shareholders and owners who will realize any returns from<br />

projects in which these funds are invested.


274 Chapter 10 Project Financing and Noneconomic Attributes<br />

Figure 10–4<br />

Expected return on common<br />

stock issue using<br />

CAPM.<br />

R m<br />

R e<br />

Market<br />

security<br />

line<br />

1<br />

Premium<br />

increases<br />

for more<br />

risky securities<br />

R m – R f<br />

Premium<br />

R f<br />

Selected<br />

market<br />

portfolio<br />

0 1.0<br />

<br />

EXAMPLE 10.5<br />

Once the cost of capital for all planned equity sources is estimated, the WACC is calculated<br />

using Equation [10.3].<br />

A second method used to estimate the cost of common stock capital is the capital asset pricing<br />

model (CAPM) . Because of the fluctuations in stock prices and the higher return demanded<br />

by some corporations’ stocks compared to others, this valuation technique is commonly applied.<br />

The cost of equity capital from common stock R e , using CAPM, is<br />

R e risk-free return premium above risk-free return<br />

R f ( R m R f ) [10.8]<br />

where volatility of a company’s stock relative to other stocks in the market ( 1.0 is<br />

the norm)<br />

R m return on stocks in a defined market portfolio measured by a prescribed index<br />

R f return on a “safe investment,” which is usually the U.S. Treasury bill rate<br />

The term ( R m R f ) is the premium paid above the safe or risk-free rate. The coefficient <br />

(beta) indicates how the stock is expected to vary compared to a selected portfolio of stocks in<br />

the same general market area, often the Standard and Poor’s 500 stock index. If 1.0, the<br />

stock is less volatile, so the resulting premium can be smaller; when 1.0, larger price movements<br />

are expected, so the premium is increased.<br />

Security is a word that identifies a stock, bond, or any other instrument used to develop capital.<br />

To better understand how CAPM works, consider Figure 10–4 . This is a plot of a market security<br />

line, which is a linear fit by regression analysis to indicate the expected return for different<br />

values. When 0, the risk-free return R f is acceptable (no premium). As increases, the<br />

premium return requirement grows. Beta values are published periodically for most stock-issuing<br />

corporations. Once complete, this estimated cost of common stock equity capital can be included<br />

in the WACC computation in Equation [10.3].<br />

The lead software engineer at SafeSoft, a food industry service corporation, has convinced the<br />

president to develop new software technology for the meat and food safety industry. It is envisioned<br />

that processes for prepared meats can be completed more safely and faster using this<br />

automated control software. A common stock issue is a possibility to raise capital if the cost of<br />

equity capital is below 9%. SafeSoft, which has a historical beta value of 1.09, uses CAPM to<br />

determine the premium of its stock compared to other software corporations. The security<br />

market line indicates that a 5% premium above the risk-free rate is desirable. If U.S. Treasury<br />

bills are paying 2%, estimate the cost of common stock capital.


10.5 Effect of Debt-Equity Mix on Investment Risk 275<br />

Solution<br />

The premium of 5% represents the term R m R f in Equation [10.8].<br />

R e 2.0 1.09(5.0) 7.45%<br />

Since this cost is lower than 9%, SafeSoft should issue common stock to finance this new<br />

venture.<br />

In theory, a correctly performed engineering economy study uses a MARR equal to the cost<br />

of the capital committed to the specific alternatives in the study. Of course, such detail is not<br />

known. For a combination of debt and equity capital, the calculated WACC sets the minimum for<br />

the MARR. The most rational approach is to set MARR between the cost of equity capital and<br />

the corporation’s WACC. The risks associated with an alternative should be treated separately<br />

from the MARR determination, as stated earlier. This supports the guideline that the MARR<br />

should not be arbitrarily increased to account for the various types of risk associated with the<br />

cash flow estimates. Unfortunately, the MARR is often set above the WACC because management<br />

does want to account for risk by increasing the MARR.<br />

EXAMPLE 10.6<br />

The <strong>Engineering</strong> Products Division of 4M Corporation has two mutually exclusive alternatives<br />

A and B with ROR values of i * A 9.2% and i * B 5.9%. The financing scenario is yet unsettled,<br />

but it will be one of the following: plan 1—use all equity funds, which are currently earning<br />

8% for the corporation; plan 2—use funds from the corporate capital pool which is 25% debt<br />

capital costing 14.5% and the remainder from the same equity funds mentioned above. The<br />

cost of debt capital is currently high because the company has narrowly missed its projected<br />

revenue on common stock for the last two quarters, and banks have increased the borrowing<br />

rate for 4M. Make the economic decision on alternative A versus B under each financing<br />

scenario. The MARR is set by the calculated WACC.<br />

Solution<br />

The capital is available for one of the two mutually exclusive alternatives. For plan 1, 100%<br />

equity, the financing is specifically known, so the cost of equity capital is the MARR, that is,<br />

8%. Only alternative A is acceptable; alternative B is not since the estimated return of 5.9%<br />

does not exceed this MARR.<br />

Under financing plan 2, with a D-E mix of 25–75,<br />

WACC 0.25(14.5) 0.75(8.0) 9.625%<br />

Now, neither alternative is acceptable since both ROR values are less than MARR <br />

WACC 9.625%. The selected alternative should be to do nothing, unless one alternative<br />

absolutely must be selected, in which case noneconomic attributes must be considered.<br />

10.5 Effect of Debt-Equity Mix on Investment Risk<br />

The D-E mix was introduced in Section 10.2. As the proportion of debt capital increases, the<br />

calculated cost of capital decreases due to the tax advantages of debt capital.<br />

The leverage offered by larger debt capital percentages increases the riskiness of projects undertaken<br />

by the company. When large debts are already present, additional financing using debt (or<br />

equity) sources gets more difficult to justify, and the corporation can be placed in a situation<br />

where it owns a smaller and smaller portion of itself. This is sometimes referred to as a highly<br />

leveraged corporation .<br />

Inability to obtain operating and investment capital means increased difficulty for the company<br />

and its projects. Thus, a reasonable balance between debt and equity financing is important for


276 Chapter 10 Project Financing and Noneconomic Attributes<br />

the financial health of a corporation. Example 10.7 illustrates the disadvantages of unbalanced<br />

D-E mixes.<br />

EXAMPLE 10.7<br />

Three auto parts manufacturing companies have the following debt and equity capital amounts<br />

and D-E mixes. Assume all equity capital is in the form of common stock.<br />

Company<br />

Debt<br />

($ in Millions)<br />

Amount of Capital<br />

Equity<br />

($ in Millions) D-E Mix (%–%)<br />

A 10 40 20−80<br />

B 20 20 50−50<br />

C 40 10 80−20<br />

Assume the annual revenue is $15 million for each one and that after interest on debt is considered,<br />

the net incomes are $14.4, $13.4, and $10.0 million, respectively. Compute the return on<br />

common stock for each company, and comment on the return relative to the D-E mixes.<br />

Solution<br />

Divide the net income by the stock (equity) amount to compute the common stock return. In<br />

million dollars,<br />

Return A —— 14.4 0.36 (36%)<br />

40<br />

Return B —— 13.4 0.67 (67%)<br />

20<br />

Return C —— 10.0 1.00 (100%)<br />

10<br />

As expected, the return is by far the largest for highly leveraged C, where only 20% of the<br />

company is in the hands of the ownership. The return is excellent, but the risk associated with<br />

this firm is high compared to A, where the D-E mix is only 20% debt.<br />

The use of large percentages of debt fi nancing greatly increases the risk taken by lenders and<br />

stock owners. Long-term confidence in the corporation diminishes, no matter how large the<br />

short-term return on stock.<br />

The leverage of large D-E mixes does increase the return on equity capital, as shown in previous<br />

examples; but it can also work against the owners and investors. A decrease in asset value<br />

will more negatively affect a highly debt-leveraged company compared to one with small leveraging.<br />

Example 10.8 illustrates this fact.<br />

EXAMPLE 10.8<br />

During the last several years, the U.S. airline industry has had financial problems, in part due<br />

to high fuel costs, fewer customers, security problems, government regulations, aging aircraft,<br />

and union dissatisfaction. As a consequence, the D-E mixes of the so-called traditional companies<br />

(American, United, Delta, and others) have become larger on the debt side than is historically<br />

acceptable. Meanwhile, the D-E mixes of so-called low-cost airlines (Southwest, JetBlue,<br />

and others) have suffered, but not to the same degree. In an effort to reduce costs, assume that<br />

three airlines joined forces to cooperate on a range of services (baggage handling, onboard<br />

food preparation, ticket services, and software development) by forming a new company called<br />

FullServe, Inc. This required $5 billion ($5 B) up-front funding from each airline.<br />

Table 10–1 summarizes the D-E mixes and the total equity capitalization for each airline<br />

after its share of $5 B was removed from available equity funds. The percentage of the $5 B<br />

obtained as debt capital was the same proportion as the debt in the company’s D-E mix. For


10.5 Effect of Debt-Equity Mix on Investment Risk 277<br />

TABLE 10–1 Debt and Equity Statistics, Example 10.8<br />

Airline<br />

Company<br />

Corporate D-E<br />

Mix, %<br />

Amount Borrowed,<br />

$ B<br />

Equity Capital<br />

Available, $ B<br />

National 30–70 1.50 5.0<br />

Global 65–35 3.25 3.7<br />

PanAm 91–9 4.55 6.7<br />

example, National had 30% of its capitalization in debt capital; therefore, 30% of $5 B was<br />

borrowed, and 70% was provided from National’s equity fund.<br />

Unfortunately, after a short time, it was clear that the three-way collaborative effort was a<br />

complete failure, and FullServe was dissolved and its assets were distributed or sold for a total<br />

of $3.0 billion, only 20% of its original value. A total of $1.0 billion in equity capital was returned<br />

to each airline. The commercial banks that provided the original loans then required that<br />

the airlines each pay back the entire borrowed amount now, since FullServe was dissolved and<br />

no profit from the venture could be realized. Assuming the loan and equity amounts are the<br />

same as shown in Table 10–1 , determine the resulting equity capital situation for each airline<br />

after it pays off the loan from its own equity funds. Also, describe one impact on each company<br />

as a result of this expensive and risky venture that failed.<br />

Solution<br />

Determine the level of post-FullServe equity capital using the following relation, in $ billions.<br />

Equity capital pre-FullServe level returned capital loan repayment<br />

National: Equity capital 5.0 1.0 1.50 $4.50<br />

Global: Equity capital 3.7 1.0 3.25 $1.45<br />

PanAm: Equity capital 6.7 1.0 4.55 $3.15<br />

Comparing the equity capital levels ( Table 10–1 ) with the levels above indicates that the<br />

FullServe effort reduced equity amounts by 10% for National, 60% for Global, and 53%<br />

for PanAm. The debt capital to fund the failed FullServe effort has affected National<br />

airlines the least, in large part due to its low D-E mix of 30%–70%. However, Global<br />

and PanAm are in much worse shape financially, and they now must maintain business<br />

with a significantly lower ownership level and much reduced ability to obtain future<br />

capital—debt or equity.<br />

The same principles discussed above for corporations are applicable to individuals. The person<br />

who is highly leveraged has large debts in terms of credit card balances, personal loans, and<br />

house mortgages. As an example, assume two engineers each have a take-home amount of<br />

$40,000 after all income tax, social security, and insurance premiums are deducted from their<br />

annual salaries. Further, assume that the cost of the debt (money borrowed via credit cards and<br />

loans) averages 15% per year and that the total debt is being repaid in equal amounts over<br />

20 years. If Jamal has a total debt of $25,000 and Barry owes $100,000, the remaining amount of<br />

the annual take-home pay may be calculated as follows:<br />

Person<br />

Total<br />

Debt, $<br />

Amount Paid, $ per Year<br />

Cost of<br />

Debt at 15%, $ Repayment of Debt, $<br />

Amount Remaining<br />

from $40,000, $<br />

Jamal 25,000 3,750 1,250 35,000<br />

Barry 100,000 15,000 5,000 20,000<br />

Jamal has 87.5% of his base available while Barry has only 50% available.


278 Chapter 10 Project Financing and Noneconomic Attributes<br />

10.6 Multiple Attribute Analysis: Identification<br />

and Importance of Each Attribute<br />

In Chapter 1 the role and scope of engineering economy in decision making were outlined. The<br />

decision-making process explained in that chapter (Figure 1–1) included the seven steps listed on<br />

the right side of Figure 10–5 . Step 4 is to identify the one or multiple attributes (criteria) upon<br />

which the selection will be based. In all prior evaluations, only one attribute—the economic<br />

one—has been identified and used to select the best alternative. The criterion has been the maximization<br />

of the equivalent value of PW, AW, FW, ROR, B/C ratio, or the CER for service projects.<br />

As we are all aware, most evaluations do and should take into account multiple attributes in<br />

decision making. These are the factors labeled as noneconomic in step 5 of Figure 1–1. However,<br />

these noneconomic dimensions tend to be intangible and often difficult, if not impossible, to<br />

quantify with economic and other scales. Nonetheless, among the many attributes that can be<br />

identified, there are key ones that must be considered in earnest before the alternative selection<br />

process is complete. This section and the next describe some of the techniques that accommodate<br />

multiple attributes in an engineering study.<br />

Multiple attributes enter into the decision-making process in many studies. Public and service sector<br />

projects are excellent examples of multiple-attribute problem solving. For example, the proposal<br />

to construct a dam to form a lake in a low-lying area or to widen the catch basin of a river usually has<br />

several purposes, such as flood control; drinking water; industrial use; commercial development;<br />

recreation; nature conservation for fish, plants, and birds; and possibly other less obvious purposes.<br />

High levels of complexity are introduced into the selection process by the multiple attributes thought<br />

to be important in selecting an alternative for the dam’s location, design, environmental impact, etc.<br />

The left side of Figure 10–5 expands steps 4 and 5 to consider multiple attributes. The discussion<br />

below concentrates on the expanded step 4 and the next section focuses on the evaluation<br />

measure and alternative selection of step 5.<br />

4-1 Attribute Identification Attributes to be considered in the evaluation methodology can<br />

be identified and defined by several methods, some much better than others depending upon the<br />

situation surrounding the study itself. To seek input from individuals other than the analyst is<br />

important; it helps focus the study on key attributes. The following is an incomplete listing of<br />

ways in which key attributes are identified.<br />

• Comparison with similar studies that include multiple attributes<br />

• Input from experts with relevant past experience<br />

• Surveys of constituencies (customers, employees, managers) impacted by the alternatives<br />

• Small group discussions using approaches such as focus groups, brainstorming, or nominal<br />

group technique<br />

• Delphi method, which is a progressive procedure to develop reasoned consensus from different<br />

perspectives and opinions<br />

Figure 10–5<br />

Expansion of the decisionmaking<br />

process to include<br />

multiple attributes.<br />

Consider multiple attributes<br />

Emphasis on one attribute<br />

1. Understand the problem; define the objective.<br />

2. Collect relevant information; define alternatives.<br />

3. Make estimates.<br />

4-1. Identify the attributes for decision<br />

making.<br />

4-2. Determine the relative<br />

importance (weights) of attributes.<br />

4-3. For each alternative, determine<br />

each attribute’s value rating.<br />

4. Identify the selection criteria (one or more<br />

attributes).<br />

5. Evaluate each alternative using<br />

a multiple-attribute technique.<br />

Use sensitivity analysis for key<br />

attributes.<br />

5. Evaluate each alternative; use sensitivity and<br />

risk analysis.<br />

6. Select the best alternative.<br />

7. Implement the solution and monitor results.


10.6 Multiple Attribute Analysis: Identification and Importance of Each Attribute 279<br />

As an illustration, assume that Delta Airlines has decided to purchase five new Boeing 787s<br />

for overseas flights, primarily between the North American west coast and Asian cities, principally<br />

Hong Kong, Tokyo, and Singapore. There are approximately 8000 options for each plane<br />

that must be decided upon by Delta’s engineering, purchasing, maintenance, and marketing personnel<br />

before the order to Boeing is placed. Options range in scope from the material and color<br />

of the plane’s interior to the type of latching devices used on the engine cowlings, and in function<br />

from maximum engine thrust to pilot instrument design. An economic study based on the equivalent<br />

AW of the estimated passenger income per trip has determined that 150 of these options are<br />

clearly advantageous. But other noneconomic attributes are to be considered before some of the<br />

more expensive options are specified. A Delphi study was performed using input from 25 individuals.<br />

Concurrently, option choices for another, unidentified airline’s recent order were shared with<br />

Delta personnel. From these two studies it was determined that there are 10 key attributes for options<br />

selection. Four of the most important attributes are<br />

• Repair time: mean time to repair or replace (MTTR) if the option is or affects a flight-critical<br />

component.<br />

• Safety: mean time to failure (MTTF) of flight-critical components.<br />

• Economic: estimated extra revenue for the option. (Basically, this is the attribute evaluated by<br />

the economic study already performed.)<br />

• Crewmember needs: some measure of the necessity and/or benefits of the option as judged by<br />

representative crewmembers—pilots and attendants.<br />

The economic attribute of extra revenue may be considered an indirect measure of customer satisfaction,<br />

one that is more quantitative than customer opinion/satisfaction survey results. Of course, there<br />

are many other attributes that can be, and are, used. However, the point is that the economic study<br />

may directly address only one or a few of the key attributes vital to alternative decision making.<br />

An attribute routinely identified by individuals and groups is risk .<br />

Risk is a possible variation in a parameter from an expected, desired, or predicted value that<br />

may be detrimental to observing the intended outcome(s) of the product, process, or system.<br />

It represents the absence of or deviation from certainty. Risk is present when there are two or<br />

more observable values of a parameter and it is possible to assume or estimate the chance that<br />

each value may occur.<br />

Risk<br />

Actually, risk is not a stand-alone attribute, because it is a part of every attribute in one form or<br />

another. Considerations of variation, probabilistic estimates, etc., in the decision-making process<br />

are treated in Chapters 18 and 19. Formalized sensitivity analysis, expected values, simulation,<br />

and decision trees are some of the techniques useful in handling risk.<br />

4-2 Importance (Weights) for the Attributes Determination of the extent of importance<br />

for each attribute i results in a weight W i that is incorporated into the final evaluation measure. The<br />

weight, a number between 0 and 1, is based upon the experienced opinion of one individual or a<br />

group of persons familiar with the attributes, and possibly the alternatives. If a group is utilized to<br />

determine the weights, there must be consensus among the members for each weight. Otherwise,<br />

some averaging technique must be applied to arrive at one weight value for each attribute.<br />

Table 10–2 is a tabular layout of attributes and alternatives used to perform a multiple attribute<br />

evaluation. Weights W i for each attribute are entered on the left side. The remainder of the table<br />

is discussed as we proceed through steps 4 and 5 of the expanded decision-making process.<br />

Attribute weights are usually normalized such that their sum over all the alternatives is 1.0. This<br />

normalizing implies that each attribute’s importance score is divided by the sum S over all attributes.<br />

Expressed in formula form, these two properties of weights for attribute i (i 1, 2, . . . , m ) are<br />

m<br />

Normalized weights: W i 1.0 [10.9]<br />

i1<br />

importance score<br />

Weight calculation: W i ————————— i<br />

m<br />

importance score i<br />

<br />

i 1<br />

importance score i<br />

————————<br />

S<br />

[10.10]


280 Chapter 10 Project Financing and Noneconomic Attributes<br />

TABLE 10–2 Tabular Layout of Attributes and Alternatives<br />

Used for Multiple Attribute Evaluation<br />

Alternatives<br />

Attributes Weights 1 2 3 . . . n<br />

1 W 1<br />

2 W 2<br />

3 W 3<br />

Value ratings V ij<br />

m<br />

W m<br />

Of the many procedures developed to assign weights to an attribute, an analyst is likely to rely<br />

upon one that is relatively simple, such as equal weighting, rank order, or weighted rank order.<br />

Pairwise comparison is another technique. Each is briefly presented below.<br />

Equal Weighting All attributes are considered to be of approximately the same importance<br />

, or there is no rationale to distinguish the more important from the less important attribute.<br />

This is the default approach. Each weight in Table 10–2 will be 1 m , according to Equation<br />

[10.10]. Alternatively, the normalizing can be omitted, in which case each weight is 1 and<br />

their sum is m . In this case, the final evaluation measure for an alternative will be the sum over<br />

all attributes.<br />

Rank Order The m attributes are ordered (ranked) by increasing importance with a score of 1<br />

assigned to the least important and m assigned to the most important. By Equation [10.10], the<br />

weights follow the pattern 1 S , 2 S , . . . , m S . With this method, the difference in weights between<br />

attributes of increasing importance is constant .<br />

Weighted Rank Order The m attributes are again placed in the order of increasing importance.<br />

However, now differentiation between attributes is possible. The most important attribute<br />

is assigned a score, usually 100, and all other attributes are scored relative to it between 100<br />

and 0. Now, define the score for each attribute as s i , and Equation [10.10] takes the form<br />

s i<br />

m<br />

s i<br />

i1<br />

W i ——<br />

[10.11]<br />

This is a very practical method to determine weights because one or more attributes can<br />

be heavily weighted if they are significantly more important than the remaining ones, and<br />

Equation [10.11] automatically normalizes the weights. For example, suppose the four key<br />

attributes in the previous aircraft purchase example are ordered: safety, repair time, crewmember<br />

needs, and economic. If repair time is only one-half as important as safety, and the<br />

last two attributes are each one-half as important as repair time, the scores and weights are<br />

as follows.<br />

Attribute Score Weights<br />

Safety 100 100/200 0.50<br />

Repair time 50 50/200 0.25<br />

Crewmember needs 25 25/200 0.125<br />

Economic 25 25/200 0.125<br />

Sum of scores and weights 200 1.000<br />

Pairwise Comparison Each attribute is compared to each other attribute in a pairwise fashion<br />

using a rating scale that indicates the importance of one attribute over the other. Assume the


10.6 Multiple Attribute Analysis: Identification and Importance of Each Attribute 281<br />

TABLE 10–3 Pairwise Comparison of Three Attributes to Determine Weights<br />

Attribute i 1 Cost 2 Constructability 3 Environment<br />

Cost — 0 1<br />

Constructability 2 — 1<br />

Environment 1 1 —<br />

Sum of scores 3 1 2<br />

Weight W i 0.500 0.167 0.333<br />

three criteria (attributes) upon which a public works project decision is based are cost, constructability,<br />

and environmental impact . Define the importance comparison scale as follows:<br />

0 if attribute is less important than one compared to<br />

1 if attribute is equally important as one compared to<br />

2 if attribute is more important than one compared to<br />

Set up a table listing attributes across the top and down the side, and perform the pairwise comparison<br />

for each column attribute with each row attribute. Table 10–3 presents a comparison with<br />

importance scores included. The arrow to the right of the table indicates the direction of comparison,<br />

i.e., column with row attribute. For example, cost is judged more important than constructability,<br />

thus a score of 2. The complement score of 0 is placed in the reverse comparison of<br />

constructability with cost. The weights are determined by normalizing the scores using Equation<br />

[10.11], where the sum for each column is s i . For the first attribute, cost i 1.<br />

s 1 3<br />

s i 3 1 2 6<br />

Cost weight W 1 36 0.500<br />

Similarly, the other weights are W 2 16 0.167 and W 3 26 0.333.<br />

There are other attribute weighting techniques, especially for group processes, such as utility<br />

functions, and the Dunn-Rankin procedure. These become increasingly sophisticated, but they are<br />

able to provide an advantage that these simple methods do not afford the analyst: consistency of<br />

ranks and scores between attributes and between individuals. If this consistency is important in that<br />

several decision makers with diverse opinions about attribute importance are involved in a study, a<br />

more sophisticated technique may be warranted. There is substantial literature on this topic.<br />

4-3 Value Rating of Each Alternative by Attribute This is the final step prior to calculating<br />

the evaluation measure. Each alternative j is awarded a value rating V ij for each attribute i .<br />

These are the entries within the cells in Table 10–2 . The ratings are appraisals by decision makers<br />

of how well an alternative will perform as each attribute is considered.<br />

The scale for the value rating can vary depending upon what is easiest to understand for those<br />

who do the valuation. A scale of 0 to 100 can be used for attribute importance scoring. However,<br />

the most popular is a scale of 4 or 5 gradations about the perceived ability of an alternative to<br />

accomplish the intent of the attribute. This is called a Likert scale, which can have descriptions<br />

for the gradations (e.g., very poor, poor, good, very good), or numbers assigned between 0 and<br />

10, or 1 to 1, or 2 to 2. The last two scales can give a negative impact to the evaluation<br />

measure for poor alternatives. An example numerical scale of 0 to 10 is as follows:<br />

If You Value the<br />

Alternative as<br />

Give It a Rating<br />

between the Numbers<br />

Very poor 0–2<br />

Poor 3–5<br />

Good 6–8<br />

Very good 9–10<br />

It is preferable to have a Likert scale with four choices (an even number) so that the central tendency<br />

of “fair” is not overrated.


282 Chapter 10 Project Financing and Noneconomic Attributes<br />

TABLE 10–4 Completed Layout for Four Attributes and Three<br />

Alternatives for Multiple Attribute Evaluation<br />

Alternatives<br />

Attributes Weights 1 2 3<br />

Safety 0.50 6 4 8<br />

Repair 0.25 9 3 1<br />

Crew needs 0.125 5 6 6<br />

Economic 0.125 5 9 7<br />

If we now build upon the aircraft purchase illustration to include value ratings, the cells are<br />

filled with ratings awarded by a decision maker. Table 10–4 includes example ratings V ij and the<br />

weights W i determined above. Initially, there will be one such table for each decision maker. Prior<br />

to calculating the final evaluation measure R j , the ratings can be combined in some fashion; or a<br />

different R j can be calculated using each decision maker’s ratings. Determination of this evaluation<br />

measure is discussed below.<br />

10.7 Evaluation Measure for Multiple Attributes<br />

The need for an evaluation measure that accommodates multiple attributes is indicated in step 5<br />

of Figure 10–5 . The measure can be one that attempts to retain all of the ratings, values, and<br />

complexity of previous assessments by multiple decision makers, or the measure can reduce<br />

these inputs to a single-dimension measure. This section introduces a single-dimension measure<br />

that is widely accepted.<br />

A single-dimension measure effectively combines the different aspects addressed by the attribute<br />

importance weights W i and the alternative value ratings V ij . The resulting evaluation measure<br />

is a formula that calculates an aggregated measure for use in selecting from two or more<br />

alternatives. The approach applied in this process is called the rank-and-rate method .<br />

This reduction process removes much of the complexity of trying to balance the different attributes;<br />

however, it also eliminates much of the robust information captured by the process of ranking attributes<br />

for their importance and rating each alternative’s performance against each attribute.<br />

There are additive, multiplicative, and exponential measures, but by far the most commonly<br />

applied is the additive model. The most used additive model is the weighted attribute method ,<br />

also called the additive weight technique . The evaluation measure, symbolized by R j for each<br />

alternative j , is defined as<br />

R j sum of (weight value rating)<br />

m<br />

W i V ij [10.12]<br />

i1<br />

The W i numbers are the attribute importance weights, and V ij is the value rating by attribute i for<br />

each alternative j . If the attributes are of equal weight (also called unweighted ), all W i 1 m , as<br />

determined by Equation [10.10]. This means that W i can be moved outside of the summation in<br />

the formula for R j . (If an equal weight of W i 1.0 is used for all attributes, in lieu of 1 m , then<br />

the R j value is simply the sum of all ratings for the alternative.)<br />

The selection guideline is as follows:<br />

ME alternative selection<br />

Choose the alternative with the largest R j value. This measure assumes that increasing<br />

weights W i mean more important attributes and increasing ratings V ij mean better performance<br />

of an alternative.<br />

Sensitivity analysis for any score, weight, or value rating is used to determine sensitivity of the decision<br />

to it. The Chapter 18 case study includes an example of multiple attribute sensitivity analysis.


Chapter Summary 283<br />

EXAMPLE 10.9<br />

The Island of Niue in the South Pacific Ocean (www.niueisland.com) released a request for proposal<br />

(RFP) 1 for a new or reconditioned workboat as it upgrades the infrastructure of its port services.<br />

The spreadsheet in Figure 10–6 , left two columns, presents the attributes and normalized<br />

weights W i published in the RFP for use in selecting one of the tenders presenting proposals. Four<br />

acceptable proposals were received. The next four columns (C through F ) include value ratings<br />

between 0 and 100 developed by a group of decision makers when the details of each proposal were<br />

evaluated against each attribute. For example, proposal 2 received a perfect score of 100 on delivery<br />

time, but lifetime costs were considered too high (rating of 20) and the price was considered relatively<br />

high (rating of 55). Use these weights and ratings to determine which proposal to pursue first.<br />

Solution<br />

Assume an additive weighting model is appropriate and apply the weighted attribute method. Equation<br />

[10.12] determines the R j measure for the four alternatives. As an illustration, for proposal 3,<br />

R 3 0.30(95) 0.20(60) 0.05(90) 0.35(85) 0.10(100)<br />

28.5 12.0 4.5 29.8 10.0<br />

84.8<br />

The four totals in Figure 10–6 (columns G through J, row 8) indicate that proposal 3 is the<br />

overall best choice for the attributes and weights published in the RFP.<br />

Comment<br />

Any economic measure can be incorporated into a multiple attribute evaluation using this method.<br />

All measures of worth—PW, AW, ROR, B/C, and C/E—can be included; however, their impact<br />

on the final selection will vary relative to the importance placed on the noneconomic attributes.<br />

Figure 10–6<br />

Attributes, weights, ratings, and evaluation measure for Niue workboat proposals, Example 10.9.<br />

1 Used with permission of Government of Niue, Infrastructure Department, “Request for Proposal: Supply<br />

of Workboat,” released March 30, 2010, www.gov.nu/Documents/workboattender3022.pdf.<br />

CHAPTER SUMMARY<br />

The interest rate at which the MARR is established depends principally upon the cost of capital<br />

and the mix between debt and equity financing. The MARR is strongly influenced by the weighted<br />

average cost of capital (WACC). Risk, profit, and other factors can be considered after the AW,<br />

PW, or ROR analysis is completed and prior to final alternative selection. A high debt-to-equity<br />

mix can significantly increase the riskiness of a project and make further debt financing difficult<br />

to acquire for the corporation.<br />

If multiple attributes, which include more than the economic dimension of a study, are to be<br />

considered in making the alternative decision, first the attributes must be identified and their<br />

relative importance assessed. Then each alternative can be value-rated for each attribute. The<br />

evaluation measure is determined using a model such as the weighted attribute method, where the<br />

measure is calculated by Equation [10.12]. The largest value indicates the best alternative.


284 Chapter 10 Project Financing and Noneconomic Attributes<br />

PROBLEMS<br />

Working with MARR<br />

10.1 List at least three factors that affect the MARR,<br />

and discuss how each one affects it.<br />

10.2 State whether each of the following involves debt<br />

financing or equity financing.<br />

(a) $10,000 taken from one partner’s savings account<br />

to pay for equipment repair<br />

(b) Issuance of preferred stock worth $1.3 million<br />

(c) Short-term loan of $75,000 from a local bank<br />

(d) Issuance of $3 million worth of 20-year<br />

bonds<br />

(e) Del <strong>Engineering</strong> buyback of $8 million of its<br />

own stock using internal funds<br />

10.3 Helical Products, Inc. uses an after-tax MARR of<br />

12% per year. If the company’s effective tax rate<br />

(federal, state, and local taxes) is 40%, determine<br />

the company’s before-tax MARR.<br />

10.4 The owner of a small pipeline construction company<br />

is trying to figure out how much he should<br />

bid in his attempt to win his first “big” contract. He<br />

estimates that his cost to complete the project will<br />

be $7.2 million. He wants to bid an amount that<br />

will give him an after-tax rate of return of 15% per<br />

year if he gets the job, but he doesn’t know how<br />

much he should bid on a before-tax basis. He told<br />

you that his effective state tax rate is 7% and his<br />

effective federal tax rate is 22%.<br />

(a) The expression for determining the overall<br />

effective tax rate is<br />

state rate (1 state rate)(federal rate)<br />

What should his before-tax MARR be in<br />

order for him to make an after-tax MARR<br />

of 15% per year?<br />

(b) How much should he bid on the job?<br />

10.5 A group of private equity investors provided<br />

$16 million to a start-up company involved in<br />

making high-technology detection systems for<br />

drugs and other types of contraband. Immediately<br />

after the investment was made, another investment<br />

opportunity came up for which the investors<br />

didn’t have enough capital. That project would<br />

have yielded an estimated rate of return of 29%<br />

per year before taxes. If the group’s effective tax<br />

rate is 32%, what after-tax rate of return would<br />

the forgone project have yielded?<br />

10.6 Five projects were ranked in decreasing order by<br />

two measures—rate of return (ROR) and present<br />

worth (PW)—to determine which ones should be<br />

Project<br />

ROR,<br />

%<br />

funded, with the total initial investment not to<br />

exceed $18 million. Use the results below to determine<br />

the opportunity cost for each measure.<br />

PW at Initial<br />

15%, Investment,<br />

$1000 $1000 Project<br />

Ranking by ROR<br />

Ranking by PW<br />

Cumulative<br />

Cumulative<br />

Investment, Investment,<br />

$1000 Project $1000<br />

A 44.5 7,138 8,000 A 8,000 A 8,000<br />

B 12.8 1,162 15,000 E 13,000 C 16,000<br />

C 20.4 1,051 8,000 C 21,000 E 21,000<br />

D 9.6 863 8,000 B 36,000 D 29,000<br />

E 26.0 936 5,000 D 44,000 B 44,000<br />

10.7 Tom, the owner of Burger Palace, determined that<br />

his weighted average cost of capital is 8%. He expects<br />

a return of 4% per year on all of his investments.<br />

A proposal presented by the owner of the<br />

Dairy Choice next door seems quite risky to Tom,<br />

but is an intriguing partnership opportunity. Tom<br />

has determined that the proposal’s “risk factor”<br />

will require an additional 3% per year return for<br />

him to accept it.<br />

(a) Use the recommended approach to determine<br />

the MARR that Tom should use, and explain<br />

how the 3% risk factor is compensated for in<br />

this MARR.<br />

(b) Determine the effective MARR for his business<br />

if Tom turns down the proposal.<br />

D-E Mix and WACC<br />

10.8 Electrical generators produce not only electricity,<br />

but also heat from conductor resistance and from<br />

friction losses in bearings. A company that manufactures<br />

generator coolers for nuclear and gas turbine<br />

power plants undertook a plant expansion<br />

through financing that had a debt-equity mix of<br />

45–55. If $18 million came from mortgages and<br />

bond sales, what was the total amount of the<br />

financing?<br />

10.9 Determine the debt-to-equity mix when Applied<br />

Technology bought out Southwest Semiconductor<br />

using financing as follows: $12 million from mortgages,<br />

$5 million from retained earnings, $10 million<br />

from cash on hand, and $20 million from<br />

bonds.<br />

10.10 Business and engineering seniors are comparing<br />

methods of financing their college education during<br />

their senior year. The business student has<br />

$30,000 in student loans that come due at graduation.<br />

Interest is an effective 4% per year. The<br />

engineering senior owes $50,000, 50% from his<br />

parents with no interest due and 50% from a


Problems 285<br />

credit union loan. This latter amount is also due at<br />

graduation with an effective rate of 7% per year.<br />

( a) What is the D-E mix for each student?<br />

( b) If their grandparents pay the loans in full at<br />

graduation, what are the amounts on the<br />

checks they write for each graduate?<br />

( c) When grandparents pay the full amount at<br />

graduation, what percent of the principal<br />

does the interest represent?<br />

10.11 Two public corporations, First <strong>Engineering</strong> and<br />

Midwest Development, each show capitalization<br />

of $175 million in their annual reports. The balance<br />

sheet for First <strong>Engineering</strong> indicates a total<br />

debt of $87 million, and that of Midwest Development<br />

indicates a net worth of $62 million. Determine<br />

the D-E mix for each company.<br />

10.12 Forest Products, Inc. invested $50 million. The<br />

company’s overall D-E mix is 60–40.What is the<br />

return on the company’s equity, if the net income is<br />

$5 million on a revenue base of $6 million?<br />

10.13 Determine the weighted average cost of capital for<br />

a company that manufactures miniature triaxial accelerometers<br />

for space-restricted applications. The<br />

financing profile, with interest rates, is as follows:<br />

$3 million in stock sales at 15% per year, $4 million<br />

in bonds at 9%, and $6 million in retained<br />

earnings at 7% per year.<br />

10.14 Growth Transgenics Enterprises (GTE) is contemplating<br />

the purchase of its rival. One of GTE’s genetics<br />

engineers got interested in the financing<br />

strategy of the buyout. He learned there are two<br />

plans being considered. Plan 1 requires 50% equity<br />

funds from GTE’s retained earnings that currently<br />

earn 9% per year, with the balance borrowed<br />

externally at 6%, based on the company’s excellent<br />

stock rating. Plan 2 requires only 20% equity<br />

funds with the balance borrowed at a higher rate of<br />

8% per year.<br />

( a) Which plan has the lower average cost of<br />

capital?<br />

( b) If GTE’s owners decide that the current corporate<br />

WACC of 8.2% will not be exceeded,<br />

what is the maximum cost of debt capital allowed<br />

for each plan? Are these rates higher<br />

or lower than the current estimates?<br />

10.15 Midac Corporation wants to arrange for $50 million<br />

in capital to finance the manufacturing of a<br />

new consumer product. The current plan is 60%<br />

equity capital and 40% debt financing. Calculate<br />

the WACC for the following scenario:<br />

Equity capital : 60%, or $35 million, via common<br />

stock sales for 40% of this amount that will pay<br />

dividends at a rate of 5% per year, and the remaining<br />

60% from retained earnings, which currently<br />

earn 9% per year.<br />

Debt capital : 40%, or $15 million, obtained<br />

through two sources—bank loans for $10 million<br />

borrowed at 8% per year, and the remainder in<br />

convertible bonds at an estimated 10% per year<br />

bond dividend rate.<br />

10.16 A public corporation in which you own common<br />

stock reported a WACC of 11.1% for the year in its<br />

report to stockholders. The common stock that you<br />

own has averaged a total return of 7% per year<br />

over the last 3 years. The annual report also mentions<br />

that projects within the corporation are 75%<br />

funded by its own capital. Estimate the company’s<br />

cost of debt capital.<br />

10.17 BASF will invest $14 million this year to upgrade<br />

its ethylene glycol processes. This chemical is<br />

used to produce polyester resins to manufacture<br />

products varying from construction materials to<br />

aircraft, and from luggage to home appliances. Equity<br />

capital costs 14.5% per year and will supply<br />

65% of the capital funds. Debt capital costs 10%<br />

per year before taxes. The effective tax rate for<br />

BASF is 36%.<br />

(a) Determine the amount of annual revenue<br />

after taxes that is consumed in covering the<br />

interest on the project’s initial cost.<br />

(b) If the corporation does not want to use 65%<br />

of its own funds, the financing plan may include<br />

75% debt capital. Determine the<br />

amount of annual revenue needed to cover<br />

the interest with this plan, and explain the effect<br />

it may have on the corporation’s ability<br />

to borrow in the future.<br />

10.18 A couple planning for their child’s college education<br />

can fund part of or all the expected $100,000<br />

tuition cost from their own funds (through an education<br />

IRA) or borrow all or part of it. The average<br />

return for their own funds is 7% per year, but the<br />

loan is expected to have a higher interest rate as<br />

the loan amount increases. Use a spreadsheet to<br />

generate a plot of the WACC curve with the estimated<br />

loan interest rates below and determine the<br />

best D-E mix for the couple.<br />

Loan Amount, $<br />

Interest Rate, % per year<br />

10,000 5.0<br />

30,000 6.0<br />

50,000 8.0<br />

60,000 9.0<br />

75,000 11.0<br />

100,000 13.0


286 Chapter 10 Project Financing and Noneconomic Attributes<br />

10.19 Over the last few years, Carol’s Fashion Store, a<br />

statewide franchise, has experienced the D-E<br />

mixes and costs of debt and equity capital on several<br />

projects summarized below.<br />

(a) Plot debt, equity, and weighted average cost<br />

of capital.<br />

(b) Determine what mix of debt and equity capital<br />

provided the lowest WACC.<br />

Debt capital<br />

Equity capital<br />

Project Percent Rate Percent Rate<br />

A 100 % 14.5%<br />

B 70 13.0 30% 7.8%<br />

C 65 12.0 35 7.8<br />

D 50 11.5 50 7.9<br />

E 35 9.9 65 9.8<br />

F 20 12.4 80 12.5<br />

G 100 12.5<br />

10.20 For Problem 10.19, use a spreadsheet to ( a ) determine<br />

the best D-E mix and ( b ) determine the best<br />

D-E mix if the cost of debt capital increases by<br />

10% per year, for example, 13.0% increases<br />

to 14.3%.<br />

Cost of Debt Capital<br />

10.21 The cash flow plan associated with a debt financing<br />

transaction allowed a company to receive<br />

$2,800,000 now in lieu of future interest payments<br />

of $196,000 per year for 10 years plus a lump sum<br />

of $2,800,000 in year 10. If the company’s effective<br />

tax rate is 33%, determine the company’s cost<br />

of debt capital ( a ) before taxes and ( b ) after taxes.<br />

10.22 A company that makes several different types of<br />

skateboards, Jennings Outdoors, incurred interest<br />

expenses of $1,200,000 per year from various<br />

types of debt financing. The company borrowed<br />

$19,000,000 in year 0 and repaid the principal of<br />

the loans in year 15 in a lump-sum payment of<br />

$20,000,000. If the company’s effective tax rate is<br />

29%, what was the company’s cost of debt capital<br />

( a ) before taxes and ( b ) after taxes?<br />

10.23 Molex Inc., a manufacturer of cable assemblies for<br />

polycrystalline photovoltaic solar modules, requires<br />

$3.1 million in debt capital. The company<br />

plans to sell 15-year bonds that carry a dividend of<br />

6% per year, payable semiannually. Molex has an<br />

effective tax rate of 32% per year. Determine<br />

( a ) the nominal annual after-tax cost of debt capital<br />

and ( b ) the effective annual after-tax cost of<br />

debt capital.<br />

10.24 Tri-States Gas Producers expects to borrow<br />

$800,000 for field engineering improvements. Two<br />

methods of debt financing are possible—borrow it<br />

all from a bank or issue debenture bonds. The company<br />

will pay an effective 8% per year to the bank<br />

for 8 years. The principal on the loan will be reduced<br />

uniformly over the 8 years, with the remainder of<br />

each annual payment going toward interest. The<br />

bond issue will be for 800 10-year bonds of $1000<br />

each that require a 6% per year dividend payment.<br />

( a) Which method of financing is cheaper after<br />

an effective tax rate of 40% is considered?<br />

( b) Which is the cheaper method using a beforetax<br />

analysis?<br />

10.25 The Sullivan Family Partnership plans to purchase<br />

a refurbished condo in their hometown for investment<br />

purposes. The negotiated $200,000 purchase<br />

price will be financed with 20% of savings (retained<br />

earnings) which consistently makes 6.5%<br />

per year after all relevant income taxes are paid.<br />

Eighty percent will be borrowed at a before-tax<br />

rate of 9% per year for 15 years with the principal<br />

repaid in equal annual installments. If the effective<br />

tax rate is 22% per year, based only on these data,<br />

answer the following.<br />

(a) What is the partnership’s annual loan payment<br />

for each of the 15 years?<br />

(b) What is the net present worth difference between<br />

the $200,000 now and the PW of the<br />

cost of the 80–20 D-E mix series of cash<br />

flows necessary to finance the purchase?<br />

What does this PW value mean?<br />

( c) What is the after-tax WACC for this<br />

purchase?<br />

Cost of Equity Capital<br />

10.26 Determine the cost of equity capital to Hy-Lok<br />

USA if the company sells 500,000 shares of its<br />

preferred stock at a 5% discount from its price of<br />

$130. The stock carries a $10 per year dividend.<br />

10.27 The initial public offering price for the common<br />

stock of SW Refining is $23 per share, and it will<br />

pay a first-year dividend of $0.92 per share. If the<br />

appreciation rate in dividends is anticipated to be<br />

3.2% per year, determine the cost of equity capital<br />

for the stock offering.<br />

10.28 The cost of debt capital is lower after taxes than<br />

before taxes. The cost of equity capital is more difficult<br />

to estimate using the dividend method or the<br />

CAPM model, for example, yet the after-tax and<br />

before-tax cost of equity capital is the same. Why<br />

are the after-tax rates not the same for both types<br />

of financing?<br />

10.29 H2W Technologies is considering raising capital<br />

to expand its offerings of 2-phase and 4-phase


Problems 287<br />

linear stepper motors. The beta value for its stock<br />

is 1.41. Use the capital asset pricing model and a<br />

3.8% premium above the risk-free return to determine<br />

the cost of equity capital if the risk-free return<br />

is 3.2%.<br />

10.30 Management at Hirschman <strong>Engineering</strong> has asked<br />

you to determine the cost of equity capital based<br />

on the company’s common stock. The company<br />

wants you to use two methods: the dividend<br />

method and the CAPM. Last year, the first year for<br />

dividends, the stock paid $0.75 per share on the<br />

average of $11.50 on the New York Stock Exchange.<br />

Management hopes to grow the dividend<br />

rate at 3% per year. Hirschman <strong>Engineering</strong> stock<br />

has a volatility that is higher than the norm at 1.3.<br />

If safe investments are returning 5.5% and the 3%<br />

growth on common stocks is also the premium<br />

above the risk-free investments that Hirschman<br />

<strong>Engineering</strong> plans to pay, calculate the cost of equity<br />

capital using the two methods.<br />

10.31 Common stocks issued by Meggitt Sensing Systems<br />

paid stockholders $0.93 per share on an average<br />

price of $18.80 last year. The company expects<br />

to grow the dividend rate at a maximum of 1.5%<br />

per year. The stock volatility is 1.19, and other<br />

stocks in the same industry are paying an average<br />

of 4.95% per year dividend. U.S Treasury bills are<br />

returning 4.5%. Determine the company’s cost of<br />

equity capital last year using ( a ) the dividend<br />

method and ( b ) the CAPM.<br />

10.32 Last year a Japanese engineering materials corporation,<br />

Yamachi Inc., purchased some U.S. Treasury<br />

bonds that return an average of 4% per year.<br />

Now, Euro bonds are being purchased with a realized<br />

average return of 3.9% per year. The volatility<br />

factor of Yamachi stock last year was 1.10 and has<br />

increased this year to 1.18. Other publicly traded<br />

stocks in this same business are paying an average<br />

of 5.1% dividends per year. Determine the cost of<br />

equity capital for each year, and explain why the<br />

increase or decrease seems to have occurred.<br />

10.33 The engineering manager at FXO Plastics wants to<br />

complete an alternative evaluation study. She<br />

asked the finance manager for the corporate<br />

MARR. The finance manager gave her some data<br />

on the project and stated that all projects must<br />

clear their average (pooled) cost by at least 4%.<br />

Funds Source Amount, $ Average Cost, %<br />

Retained earnings 4,000,000 7.4<br />

Stock sales 6,000,000 4.8<br />

Long-term loans 5,000,000 9.8<br />

(a) Use the data to determine the minimum<br />

MARR.<br />

(b) The study is after taxes and part ( a ) provided<br />

the before-tax MARR. Determine the correct<br />

MARR to use if the tax rate was 32% last year<br />

and the finance manager meant that the 4%<br />

above the cost is for after-tax evaluations.<br />

Different D-E Mixes<br />

10.34 Why is it financially unhealthy for an individual to<br />

maintain a large percentage of debt financing over<br />

a long time, that is, to be highly leveraged?<br />

10.35 In a leveraged buyout of one company by another,<br />

the purchasing company usually obtains borrowed<br />

money and inserts as little of its own equity as possible<br />

into the purchase. Explain some circumstances<br />

under which such a buyout may put the<br />

purchasing company at economic risk.<br />

10.36 Grainger and Company has an opportunity to invest<br />

$500,000 in a new line of direct-drive rotary<br />

screw compressors. Financing will be equally split<br />

between common stock ($250,000) and a loan<br />

with an 8% after-tax interest rate. The estimated<br />

annual NCF after taxes is $48,000 for the next<br />

7 years. The effective tax rate is 50%. Grainger<br />

uses the capital asset pricing model for evaluation<br />

of its common stock. Recent analysis shows that it<br />

has a volatility rating of 0.95 and is paying a premium<br />

of 5% above a safe return on its common<br />

stock. Nationally, the safest investment is currently<br />

paying 3% per year. Is the investment financially<br />

attractive if Grainger uses as the MARR its<br />

( a ) equity cost of capital and ( b ) WACC?<br />

10.37 Fairmont Industries primarily relies on 100% equity<br />

financing to fund projects. A good opportunity<br />

is available that will require $250,000 in capital.<br />

The Fairmont owner can supply the money from<br />

personal investments that currently earn an average<br />

of 7.5% per year. The annual net cash flow<br />

from the project is estimated at $30,000 for the<br />

next 15 years. Alternatively, 60% of the required<br />

amount can be borrowed for 15 years at 7% per<br />

year. If the MARR is the WACC, determine which<br />

plan, if either, should be undertaken. This is a<br />

before-tax analysis.<br />

10.38 Omega <strong>Engineering</strong> Inc. has an opportunity to invest<br />

$10,000,000 in a new engineering remote<br />

control system for offshore drilling platforms. Financing<br />

will be split between common stock sales<br />

($5,000,000) and a loan with an 8% per year interest<br />

rate. Omega’s share of the annual net cash flow<br />

is estimated to be $1.35 million for each of the<br />

next 5 years. Omega is about to initiate CAPM as


288 Chapter 10 Project Financing and Noneconomic Attributes<br />

its common stock evaluation model. Recent analysis<br />

shows that it has a volatility rating of 1.22 and<br />

is paying a premium of 5% on its common stock<br />

dividend. The U.S. Treasury bills are currently<br />

paying 4% per year. Is the venture financially attractive<br />

if the MARR equals ( a ) the cost of equity<br />

capital and ( b ) the WACC?<br />

10.39 A new annular die process is to be installed for extruding<br />

pipes, tubes, and tubular films. The phase I<br />

installed price for the dies and machinery is<br />

$2,000,000. The manufacturer has not decided<br />

how to finance the system. The WACC over the<br />

last 5 years has averaged 10% per year.<br />

(a)<br />

Two financing alternatives have been defined.<br />

The first requires an investment of<br />

40% equity funds at 9% and a loan for the<br />

balance at an interest rate of 10% per year.<br />

The second alternative requires only 25% equity<br />

funds and the balance borrowed at<br />

10.5% per year. Which approach will result<br />

in the smaller average cost of capital?<br />

(b) Yesterday, the corporate finance committee<br />

decided that the WACC for all new projects<br />

must not exceed the 5-year historical average<br />

of 10% per year. With this restriction, what is<br />

the maximum loan interest rate that can be incurred<br />

for each of the financing alternatives?<br />

10.40 Shadowland, a manufacturer of air-freightable pet<br />

crates, has identified two projects that, though having<br />

a relatively high risk, are expected to move the<br />

company into new revenue markets. Utilize a<br />

spreadsheet solution to ( a ) select any combination<br />

of the projects if the MARR is equal to the aftertax<br />

WACC and ( b ) determine if the same projects<br />

should be selected if the risk factors are enough to<br />

require an additional 2% per year for the investment<br />

to be made.<br />

Project<br />

Initial<br />

Investment, $<br />

After-Tax Cash<br />

Flow, $/Year<br />

Life, Years<br />

Wildlife (W) 250,000 48,000 10<br />

Reptiles (R) 125,000 30,000 5<br />

Financing will be developed using a D-E mix of<br />

60–40 with equity funds costing 7.5% per year.<br />

Debt financing will be developed from $10,000,<br />

5% per year, paid quarterly, 10-year bonds. The effective<br />

tax rate is 30% per year.<br />

10.41 Two friends each invested $20,000 of their own<br />

(equity) funds. Stan, being more conservative, purchased<br />

utility and manufacturing corporation<br />

stocks. Theresa, being a risk taker, leveraged the<br />

$20,000 and purchased a $100,000 condo for rental<br />

property. Considering no taxes, dividends, or revenues,<br />

analyze these two purchases by doing the following<br />

for one year after the funds were invested.<br />

(a) Determine the year-end values of their equity<br />

funds if there was a 10% increase in the<br />

value of the stocks and the condo.<br />

(b) Determine the year-end values of their equity<br />

funds if there was a 10% decrease in the<br />

value of the stocks and the condo.<br />

(c) Use your results to explain why leverage can<br />

be financially risky.<br />

Multiple, Noneconomic Attributes<br />

10.42 In multiple attribute analysis, if three different alternatives<br />

are to be evaluated on the basis of eight<br />

attributes that are considered of equal importance,<br />

what is the weight of each attribute?<br />

10.43 A consulting engineer asked a company manager to<br />

assign importance values (0 to 100) to five attributes<br />

that will be included in an alternative evaluation<br />

process. Determine the weight of each attribute<br />

using the importance scores.<br />

Attribute<br />

Importance Score<br />

1. Safety 60<br />

2. Cost 40<br />

3. Impact 80<br />

4. Environmental 30<br />

5. Acceptability 20<br />

10.44 Ten attributes were rank-ordered in terms of increasing<br />

importance and were identified as A, B,<br />

C, . . . , and J. Determine the weight of ( a ) attribute<br />

C and ( b ) attribute J.<br />

10.45 A team of three people submitted the following<br />

statements about the attributes to be used in a<br />

weighted attribute evaluation. Use the statements<br />

to determine the normalized weights if assigned<br />

scores are between 0 and 100.<br />

Attribute<br />

Comment<br />

1. Flexibility (F) The most important factor<br />

2. Safety (S) 70% as important as uptime<br />

3. Uptime (U) One-half as important as flexibility<br />

4. Rate of return (R) Twice as important as safety<br />

10.46 Different types and capacities of crawler hoes are<br />

being considered for use in a major excavation on<br />

a pipe-laying project. Several supervisors who<br />

served on similar projects in the past have identified<br />

some of the attributes and their view of relative<br />

importance. For the information that follows,<br />

determine the weighted rank order, using a 0-to-10<br />

scale and the normalized weights.


Additional Problems and FE Exam Review Questions 289<br />

Attribute<br />

Comment<br />

1. Truck versus hoe height 90% as important as trenching speed<br />

2. Type of topsoil Only 10% of most important<br />

attribute<br />

3. Type of subsoil 30% as important as trenching speed<br />

4. Hoe cycle time Twice as important as type of<br />

subsoil<br />

5. Hoe trenching speed Most important attribute<br />

6. Pipe laying speed 80% as important as hoe cycle time<br />

10.47 John, who works at Swatch, has decided to use the<br />

weighted attribute method to compare three systems<br />

for manufacturing a watchband. The vice<br />

president and her assistant VP have evaluated each<br />

of three attributes in terms of importance to them,<br />

and John has placed an evaluation from 0 to 100<br />

on each alternative for the three attributes. John’s<br />

ratings for each alternative are as follows:<br />

Alternative<br />

Attribute 1 2 3<br />

Economic return MARR 50 70 100<br />

High throughput 100 60 30<br />

Low scrap rate 100 40 50<br />

Use the weights below to evaluate the alternatives.<br />

Are the results the same for both persons’ weights?<br />

Why?<br />

Importance Score VP Assistant VP<br />

Economic return MARR 20 100<br />

High throughput 80 80<br />

Low scrap rate 100 20<br />

10.48 The Athlete’s Shop has evaluated two proposals<br />

for weight lifting and exercise equipment. A present<br />

worth analysis at i 15% per year of estimated<br />

revenues and costs resulted in PW A <br />

$440,000 and PW B $390,000. In addition to<br />

this economic measure, three more attributes<br />

were independently assigned a relative importance<br />

score from 0 to 100 by the shop manager<br />

and the lead trainer.<br />

Importance Score<br />

Attribute Manager Trainer<br />

Economics 80 80<br />

Durability 35 80<br />

Flexibility 30 100<br />

Maintainability 20 50<br />

Separately, you have used the four attributes to<br />

rate the two equipment proposals on a scale of 0 to<br />

1.0 as shown in the following table. The economic<br />

attribute was rated using the PW values.<br />

Attribute Proposal A Proposal B<br />

Economics 1.00 0.90<br />

Durability 0.35 1.00<br />

Flexibility 1.00 0.90<br />

Maintainability 0.25 1.00<br />

Select the better proposal using each of the following<br />

methods.<br />

(a) Present worth<br />

(b) Weighted evaluations of the shop manager<br />

(c) Weighted evaluations of the lead trainer<br />

ADDITIONAL PROBLEMS AND FE EXAM REVIEW QUESTIONS<br />

10.49 The term opportunity cost refers to:<br />

( a) The first cost of an alternative that has been<br />

accepted for funding<br />

( b) The total cost of an alternative that has been<br />

accepted for funding<br />

( c) The rate of return or profit available on the<br />

next-best alternative that had to be forgone<br />

due to lack of capital funds<br />

( d) The cost of an alternative that was not recognized<br />

as an alternative that actually represented<br />

a good opportunity<br />

10.50 The cost of capital is established on the basis of:<br />

( a) The cost of debt financing<br />

( b) The weighted average of debt and equity<br />

financing<br />

(c) The cost of equity financing<br />

(d) The cost of debt financing plus the expected<br />

inflation rate<br />

10.51 All of the following are examples of debt capital<br />

except:<br />

(a) Retained earnings<br />

(b) Long-term bonds<br />

(c) Loan from a local bank<br />

(d) Purchase of equipment using a credit card<br />

10.52 All of the following are examples of equity capital<br />

except:<br />

(a) Sale of preferred stock<br />

(b) Long-term bonds<br />

(c) Company cash on hand<br />

(d) Use of retained earnings


290 Chapter 10 Project Financing and Noneconomic Attributes<br />

10.53 If a public utility expands its capacity to generate<br />

electricity by obtaining $41 million from retained<br />

earnings and $30 million from municipal bond<br />

sales, the utilities’ debt-to-equity mix is closest to:<br />

(a) 58% debt and 42% equity<br />

(b) 73% debt and 27% equity<br />

(c) 27% debt and 73% equity<br />

(d) 42% debt and 58% equity<br />

10.54 Gentech, Inc. financed a new product as follows:<br />

$5 million in stock sales at 13.7% per year, $2 million<br />

in retained earnings at 8.9% per year, and<br />

$3 million through convertible bonds at 7.8% per<br />

year. The company’s WACC is closest to:<br />

(a) 9% per year<br />

(b) 10% per year<br />

(c) 11% per year<br />

(d) 12% per year<br />

10.55 If the after-tax rate of return for a cash flow series<br />

is 11.2% and the corporate effective tax rate is<br />

39%, the approximated before-tax rate of return is<br />

closest to:<br />

(a) 6.8%<br />

(b) 5.4%<br />

(c) 18.4%<br />

(d) 28.7%<br />

10.56 Medzyme Pharmaceuticals has maintained a 50-50<br />

D-E mix for capital investments. Equity capital<br />

has cost 11%; however, debt capital that historically<br />

cost 9% has now increased by 20% per year.<br />

If Medzyme does not want to exceed its historical<br />

weighted average cost of capital (WACC), and it is<br />

forced to go to a D-E mix of 75–25, the maximum<br />

acceptable cost of equity capital is closest to:<br />

(a) 7.6%<br />

(b) 9.2%<br />

(c) 9.8%<br />

(d) 10.9%<br />

10.57 The importance values (0 to 100) for five attributes<br />

are shown below. The weight to assign to attribute<br />

1 is:<br />

(a) 0.16<br />

(b) 0.20<br />

(c) 0.22<br />

(d) 0.25<br />

Attribute<br />

Importance Score<br />

1 55<br />

2 45<br />

3 85<br />

4 30<br />

5 60<br />

10.58 For eight attributes rank-ordered in terms of increasing<br />

importance, the weighting of the sixth attribute<br />

is closest to:<br />

(a) 0.17<br />

(b) 0.14<br />

(c) 0.08<br />

(d) 0.03<br />

CASE STUDY<br />

WHICH IS BETTER—DEBT OR EQUITY FINANCING?<br />

Background<br />

Information<br />

Pizza Hut Corporation has decided to enter the catering business<br />

in three states within its Southeastern U.S. Division,<br />

using the name Pizza Hut At-Your-Place. To deliver the meals<br />

and serving personnel, it is about to purchase 200 vans with<br />

custom interiors for a total of $1.5 million. Each van is expected<br />

to be used for 10 years and have a $1000 salvage<br />

value.<br />

A feasibility study completed last year indicated that the<br />

At-Your-Place business venture could realize an estimated<br />

annual net cash flow of $300,000 before taxes in the three<br />

states. After-tax considerations would have to take into account<br />

the effective tax rate of 35% paid by Pizza Hut.<br />

An engineer with Pizza Hut’s Distribution Division has<br />

worked with the corporate finance office to determine how to<br />

best develop the $1.5 million capital needed for the purchase<br />

of vans. There are two viable financing plans.<br />

Plan A is debt financing for 50% of the capital ($750,000)<br />

with the 8% per year compound interest loan repaid over<br />

10 years with uniform year-end payments. (A simplifying assumption<br />

that $75,000 of the principal is repaid with each<br />

annual payment can be made.)<br />

Plan B is 100% equity capital raised from the sale of $15<br />

per share common stock. The financial manager informed the<br />

engineer that stock is paying $0.50 per share in dividends and<br />

that this dividend rate has been increasing at an average of<br />

5% each year. This dividend pattern is expected to continue,<br />

based on the current financial environment.<br />

Case Study Exercises<br />

1. What values of MARR should the engineer use to determine<br />

the better financing plan?


Case Study 291<br />

2. The engineer must make a recommendation on the financing<br />

plan by the end of the day. He does not know<br />

how to consider all the tax angles for the debt financing<br />

in plan A. However, he does have a handbook that gives<br />

these relations for equity and debt capital about taxes<br />

and cash flows:<br />

Equity capital: no income tax advantages<br />

After-tax net cash flow<br />

(before-tax net cash flow)(1 tax rate)<br />

Debt capital: income tax advantage comes from interest<br />

paid on loans<br />

Taxes (taxable income)(tax rate)<br />

Taxable income net cash flow<br />

loan interest<br />

He decides to forget any other tax consequences and use<br />

this information to prepare a recommendation. Is A or B<br />

the better plan?<br />

3. The division manager would like to know how much<br />

the WACC varies for different D-E mixes, especially<br />

about 15% to 20% on either side of the 50% debt financing<br />

option in plan A. Plot the WACC curve and compare<br />

its shape with that of Figure 10–2 .<br />

After-tax net cash flow before-tax net cash flow<br />

loan principal<br />

loan interest taxes


CHAPTER 11<br />

Replacement<br />

and Retention<br />

Decisions<br />

LEARNING OUTCOMES<br />

Purpose: Perform a replacementretention study between an in-place asset, process, or system and one that could<br />

replace it.<br />

SECTION TOPIC LEARNING OUTCOME<br />

11.1 Replacement study basics • Explain the fundamental approach and<br />

terminology of replacement analysis.<br />

11.2 Economic service life • Determine the ESL that minimizes the total AW<br />

for estimated costs and salvage value.<br />

11.3 Replacement analysis • Perform a replacementretention study between<br />

a defender and the best challenger.<br />

11.4 Additional considerations • Understand the approach to special situations in<br />

a replacement study.<br />

11.5 Study period analysis • Perform a replacementretention study over a<br />

specified number of years.<br />

11.6 Replacement value • Calculate the minimum market (trade-in) value<br />

required to make the challenger economically<br />

attractive.


O<br />

ne of the most common and important issues in industrial practice is that of replacement<br />

or retention of an asset, process, or system that is currently installed.<br />

This differs from previous situations where all the alternatives were new. The fundamental<br />

question answered by a replacement study (also called a replacementretention<br />

study) about a currently installed system is, Should it be replaced now or later ? When an<br />

asset is currently in use and its function is needed in the future, it will be replaced at some<br />

time. In reality, a replacement study answers the question of when, not if, to replace.<br />

A replacement study is usually designed to first make the economic decision to retain or<br />

replace now . If the decision is to replace, the study is complete. If the decision is to retain,<br />

the cost estimates and decision can be revisited each year to ensure that the decision to<br />

retain is still economically correct. This chapter explains how to perform the initial-year and<br />

follow-on year replacement studies.<br />

A replacement study is an application of the AW method of comparing unequal-life alternatives,<br />

first introduced in Chapter 6. In a replacement study with no specified study period,<br />

the AW values are determined by a technique called the economic service life (ESL) analysis.<br />

If a study period is specified, the replacement study procedure is different from that used<br />

when no study period is set.<br />

If asset depreciation and taxes are to be considered in an after-tax replacement analysis,<br />

Chapters 16 and 17 should be covered before or in conjunction with this chapter. After-tax<br />

replacement analysis is included in Chapter 17.<br />

Keep or Replace the Kiln Case: B&T<br />

Enterprises manufactures and sells highmelting-temperature<br />

ceramics and highperformance<br />

metals to other corporations.<br />

The products are sold to a wide range of<br />

industries from the nuclear and solar power<br />

industry to sports equipment manufacturers<br />

of specialty golf and tennis gear, where kiln<br />

temperatures up to approximately 1700°C<br />

are needed. For years, B&T has owned and<br />

been very satisfied with Harper International<br />

pusher-plate tunnel kilns. Two are in use<br />

currently at plant locations on each coast<br />

of the country; one kiln is 10 years old, and<br />

the second was purchased only 2 years ago<br />

and serves, primarily, the ceramics industry<br />

needs on the west coast. This newer kiln<br />

can reach temperatures of 2800°C.<br />

During the last two or three quarterly<br />

maintenance visits, the Harper team<br />

leader and the head of B&T quality have<br />

discussed the ceramic and metal industry<br />

needs for higher temperatures. In some<br />

cases the temperatures are as high as<br />

3000°C for emerging nitride, boride, and<br />

carbide transition metals that form very<br />

high-melting-temperature oxides. These<br />

may find use in hypersonic vehicles,<br />

engines, plasma arc electrodes, cutting<br />

tools, and high-temperature shielding.<br />

A looming question on the mind of the<br />

senior management and financial officers<br />

of B&T revolves around the need to seriously<br />

consider a new graphite hearth kiln,<br />

which can meet higher temperature and<br />

other needs of the current and projected<br />

PE<br />

customer base. This unit will have lower<br />

operating costs and significantly greater<br />

furnace efficiency in heat time, transit,<br />

and other crucial parameters. Since virtually<br />

all of this business is on the west coast,<br />

the graphite hearth kiln would replace the<br />

newer of the two kilns currently in use.<br />

For identification, let<br />

PT identify the currently installed<br />

pusher-plate tunnel kiln (defender)<br />

GH identify the proposed new graphite<br />

hearth kiln (challenger)<br />

Relevant estimates follow in $ millions<br />

for monetary units.<br />

First cost,<br />

$ M<br />

AOC, $ M<br />

per year<br />

PT<br />

$25; 2 years<br />

ago<br />

year 1: $5.2;<br />

year 2: $6.4<br />

GH<br />

$38; with no<br />

trade-in<br />

starts at $3.4,<br />

increases<br />

10%year<br />

Life, years 6 (remaining) 12 (estimated)<br />

Heating<br />

element,<br />

$ M<br />

—<br />

$2.0 every<br />

6 years<br />

This case is used in the following topics<br />

of this chapter:<br />

Economic service life (Section 11.2)<br />

Replacement study (Section 11.3)<br />

Replacement study with study period<br />

(Section 11.5)<br />

Replacement value (Section 11.6)<br />

Problems 11.18 and 11.41


294 Chapter 11 Replacement and Retention Decisions<br />

11.1 Basics of a Replacement Study<br />

The need for a replacement study can develop from several sources:<br />

Reduced performance. Because of physical deterioration, the ability to perform at an<br />

expected level of reliability (being available and performing correctly when needed) or productivity<br />

(performing at a given level of quality and quantity) is not present. This usually<br />

results in increased costs of operation, higher scrap and rework costs, lost sales, reduced quality,<br />

diminished safety, and larger maintenance expenses.<br />

Altered requirements. New requirements of accuracy, speed, or other specifications cannot<br />

be met by the existing equipment or system. Often the choice is between complete replacement<br />

or enhancement through retrofitting or augmentation.<br />

Obsolescence. International competition and rapidly changing technology make currentlyused<br />

systems and assets perform acceptably but less productively than equipment coming<br />

available. The ever-decreasing development cycle time to bring new products to market is<br />

often the reason for premature replacement studies, that is, studies performed before the<br />

estimated useful or economic life is reached.<br />

Replacement studies use some terminology that is closely related to terms in previous<br />

chapters.<br />

Salvagemarket value<br />

Economic service life<br />

Defender and challenger are the names for two mutually exclusive alternatives. The defender<br />

is the currently installed asset, and the challenger is the potential replacement. A<br />

replacement study compares these two alternatives. The challenger is the “best” challenger<br />

because it has been selected as the best one to possibly replace the defender. (This is the<br />

same terminology used earlier for incremental ROR and BC analysis, but both alternatives<br />

were new).<br />

Market value is the current value of the installed asset if it were sold or traded on the<br />

open market. Also called trade-in value , this estimate is obtained from professional appraisers,<br />

resellers, or liquidators familiar with the industry. As in previous chapters, salvage<br />

value is the estimated value at the end of the expected life. In replacement analysis,<br />

the salvage value at the end of one year is used as the market value at the beginning of the<br />

next year.<br />

AW values are used as the primary economic measure of comparison between the defender<br />

and challenger. The term equivalent uniform annual cost ( EUAC) may be used in lieu of<br />

AW, because often only costs are included in the evaluation; revenues generated by the<br />

defender or challenger are assumed to be equal. (Since EUAC calculations are exactly the<br />

same as for AW, we use the term AW.) Therefore, all values will be negative when only<br />

costs are involved. Salvage or market value is an exception; it is a cash inflow and carries<br />

a plus sign.<br />

Economic service life (ESL) for an alternative is the number of years at which the lowest AW<br />

of cost occurs . The equivalency calculations to determine ESL establish the life n for the best<br />

challenger and the lowest cost life for the defender in a replacement study. The next section<br />

explains how to find the ESL.<br />

Defender first cost is the initial investment amount P used for the defender. The current<br />

market value ( MV ) is the correct estimate to use for P for the defender in a replacement<br />

study. The estimated salvage value at the end of one year becomes the market value at the<br />

beginning of the next year, provided the estimates remain correct as the years pass. It is<br />

incorrect to use the following as MV for the defender first cost: trade-in value that does<br />

not represent a fair market value , or the depreciated book value taken from accounting<br />

records. If the defender must be upgraded or augmented to make it equivalent to the challenger<br />

(in speed, capacity, etc.), this cost is added to the MV to obtain the estimated<br />

defender first cost.<br />

Challenger first cost is the amount of capital that must be recovered (amortized) when<br />

replacing a defender with a challenger. This amount is almost always equal to P , the first cost<br />

of the challenger.


11.1 Basics of a Replacement Study 295<br />

If an unrealistically high trade-in value is offered for the defender compared to its fair<br />

market value, the net cash flow required for the challenger is reduced, and this fact should be<br />

considered in the analysis. The correct amount to recover and use in the economic analysis for<br />

the challenger is its first cost minus the difference between the trade-in value (TIV) and market<br />

value (MV) of the defender. In equation form, this is P (TIV MV). This amount<br />

represents the actual cost to the company because it includes both the opportunity cost (i.e.,<br />

market value of the defender) and the out-of-pocket cost (i.e., first cost trade-in) to acquire<br />

the challenger. Of course, when the trade-in and market values are the same, the challenger P<br />

value is used in all computations.<br />

The challenger first cost is the estimated initial investment necessary to acquire and install it.<br />

Sometimes, an analyst or manager will attempt to increase this first cost by an amount equal to the<br />

unrecovered capital remaining in the defender, as shown on the accounting records for the asset.<br />

This incorrect treatment of capital recovery is observed most often when the defender is working<br />

well and in the early stages of its life, but technological obsolescence, or some other reason, has<br />

forced consideration of a replacement. This leads us to identify two additional characteristics of<br />

replacement analysis, in fact, of any economic analysis: sunk costs and nonowner’s viewpoint .<br />

A sunk cost is a prior expenditure or loss of capital (money) that cannot be recovered by a<br />

decision about the future. The replacement alternative for an asset, system, or process that has<br />

incurred a nonrecoverable cost should not include this cost in any direct fashion; sunk costs<br />

should be handled in a realistic way using tax laws and write-off allowances.<br />

Sunk cost<br />

A sunk cost should never be added to the challenger’s first cost, because it will make the challenger<br />

appear to be more costly than it actually is . For example, assume an asset costing $100,000<br />

two years ago has a depreciated value of $80,000 on the corporate books. It must be replaced<br />

prematurely due to rapidly advancing technology. If the replacement alternative (challenger) has<br />

a first cost of $150,000, the $80,000 from the current asset is a sunk cost were the challenger<br />

purchased. For the purposes of an economic analysis, it is incorrect to increase the challenger’s<br />

first cost to $230,000 or any number between this and $150,000.<br />

The second characteristic is the perspective taken when conducting a replacement study. You,<br />

the analyst, are a consultant from outside the company.<br />

The nonowner’s viewpoint , also called the outsider’s viewpoint or consultant’s viewpoint, provides<br />

the greatest objectivity in a replacement study. This viewpoint performs the analysis without<br />

bias; it means the analyst owns neither the defender nor the challenger. Additionally, it assumes<br />

the services provided by the defender can be purchased now by making an “initial<br />

investment” equal to the market value of the defender.<br />

Besides being unbiased, this perspective is correct because the defender’s market value is a forgone<br />

opportunity of cash inflow were the replacement not selected, and the defender chosen.<br />

As mentioned in the introduction, a replacement study is an application of the annual worth<br />

method. As such, the fundamental assumptions for a replacement study parallel those of an AW<br />

analysis. If the planning horizon is unlimited , that is, a study period is not specified, the assumptions<br />

are as follows:<br />

1. The services provided are needed for the indefinite future.<br />

2. The challenger is the best challenger available now and in the future to replace the defender.<br />

When this challenger replaces the defender (now or later), it will be repeated for succeeding<br />

life cycles.<br />

3. Cost estimates for every life cycle of the defender and challenger will be the same as in their<br />

first cycle.<br />

As expected, none of these assumptions is precisely correct. We discussed this previously for the<br />

AW method (and the PW method). When the intent of one or more of the assumptions becomes<br />

incorrect, the estimates for the alternatives must be updated and a new replacement study conducted.<br />

The replacement procedure discussed in Section 11.3 explains how to do this. When the<br />

planning horizon is limited to a specified study period, the assumptions above do not hold. The<br />

procedure of Section 11.5 discusses replacement analysis over a fixed study period.


296 Chapter 11 Replacement and Retention Decisions<br />

EXAMPLE 11.1<br />

Only 2 years ago, Techtron purchased for $275,000 a fully loaded SCADA (supervisory control<br />

and data acquisition) system including hardware and software for a processing plant operating<br />

on the Houston ship channel. When it was purchased, a life of 5 years and salvage of 20%<br />

of first cost were estimated. Actual M&O costs have been $25,000 per year, and the book value<br />

is $187,000. There has been a series of insidious malware infections targeting Techtron’s command<br />

and control software, plus next-generation hardware marketed only recently could<br />

greatly reduce the competitiveness of the company in several of its product lines. Given these<br />

factors, the system is likely worth nothing if kept in use for the final 3 years of its anticipated<br />

useful life.<br />

Model K2-A1, a new replacement turnkey system, can be purchased for $300,000 net cash,<br />

that is, $400,000 first cost and a $100,000 trade-in for the current system. A 5-year life, salvage<br />

value of 15% of stated first cost or $60,000, and an M&O cost of $15,000 per year are good<br />

estimates for the new system. The current system was appraised this morning, and a market<br />

value of $100,000 was confirmed for today; however, with the current virus discovery, the appraiser<br />

anticipates that the market value will fall rapidly to the $80,000 range once the virus<br />

problem and new model are publicized.<br />

Using the above values as the best possible today, state the correct defender and challenger<br />

estimates for P, M&O, S, and n in a replacement study to be performed today.<br />

Solution<br />

Defender: Use the current market value of $100,000 as the first cost for the defender. All<br />

others—original cost of $275,000, book value of $187,000, and trade-in value of<br />

$100,000—are irrelevant to a replacement study conducted today. The estimates are as<br />

follows:<br />

First cost P $100,000<br />

M&O cost A $25,000 per year<br />

Expected life n 3 years<br />

Salvage value S 0<br />

Challenger: The $400,000 stated first cost is the correct one to use for P, because the<br />

trade-in and market values are equal.<br />

First cost P $400,000<br />

M&O cost A $15,000 per year<br />

Expected life n 5 years<br />

Salvage value S $60,000<br />

Comment<br />

If the replacement study is conducted next week when estimates will have changed, the defender’s<br />

first cost will be $80,000, the new market value according to the appraiser. The challenger’s<br />

first cost will be $380,000, that is, P (TIV MV) 400,000 (100,000 – 80,000).<br />

11.2 Economic Service Life<br />

Until now the estimated life n of an alternative or asset has been stated. In reality, the best life<br />

estimate to use in the economic analysis is not known initially. When a replacement study or an<br />

analysis between new alternatives is performed, the best value for n should be determined using<br />

current cost estimates. The best life estimate is called the economic service life .<br />

Economic service life<br />

The economic service life (ESL) is the number of years n at which the equivalent uniform<br />

annual worth (AW) of costs is the minimum, considering the most current cost estimates over<br />

all possible years that the asset may provide a needed service.


11.2 Economic Service Life 297<br />

Larger<br />

costs<br />

Figure 11–1<br />

Annual worth curves of<br />

cost elements that determine<br />

the economic<br />

service life.<br />

AW of costs, $/year<br />

Total AW of costs<br />

AW of AOC<br />

Capital recovery<br />

0 Years<br />

Economic<br />

service life<br />

The ESL is also referred to as the economic life or minimum cost life . Once determined, the ESL<br />

should be the estimated life for the asset used in an engineering economy study, if only economics<br />

are considered. When n years have passed, the ESL indicates that the asset should be replaced<br />

to minimize overall costs. To perform a replacement study correctly, it is important that the ESL<br />

of the challenger and the ESL of the defender be determined, since their n values are usually not<br />

preestablished.<br />

The ESL is determined by calculating the total AW of costs if the asset is in service 1 year,<br />

2 years, 3 years, and so on, up to the last year the asset is considered useful. Total AW of costs is<br />

the sum of capital recovery (CR), which is the AW of the initial investment and any salvage<br />

value, and the AW of the estimated annual operating cost (AOC), that is,<br />

Total AW capital recovery AW of annual operating costs<br />

CR AW of AOC [11.1]<br />

The ESL is the n value for the smallest total AW of costs . (Remember: These AW values are cost<br />

estimates, so the AW values are negative numbers. Therefore, $–200 is a lower cost than $500.)<br />

Figure 11–1 shows the characteristic shape of a total AW of cost curve. The CR component of total<br />

AW decreases, while the AOC component increases, thus forming the concave shape. The two AW<br />

components are calculated as follows.<br />

Decreasing cost of capital recovery. The capital recovery is the AW of investment; it decreases<br />

with each year of ownership. Capital recovery is calculated by Equation [6.3], which<br />

is repeated here. The salvage value S , which usually decreases with time, is the estimated<br />

market value (MV) in that year.<br />

Capital recovery P ( AP , i,n ) S ( A F , i,n ) [11.2]<br />

Increasing cost of AW of AOC. Since the AOC (or M&O) estimates usually increase over the<br />

years, the AW of AOC increases. To calculate the AW of the AOC series for 1, 2, 3, . . . years,<br />

determine the present worth of each AOC value with the PF factor, then redistribute this<br />

P value over the years of ownership, using the AP factor.<br />

The complete equation for total AW of costs over k years (k 1, 2, 3, . . . ) is<br />

jk<br />

Total AW k P(AP,i,k) S k (AF,i,k) <br />

AOC j (PF,i,j) (AP,i,k) [11.3]<br />

<br />

j1<br />

Capital recovery<br />

where<br />

P initial investment or current market value<br />

S k salvage value or market value after k years<br />

AOC j annual operating cost for year j ( j 1 to k )


298 Chapter 11 Replacement and Retention Decisions<br />

The current MV is used for P when the asset is the defender, and the estimated future MV values<br />

are substituted for the S values in years 1, 2, 3, . . . . Plotting the AW k series as in Figure 11–1<br />

clearly indicates where the ESL is located and the trend of the AW k curve on each side of the ESL.<br />

To determine ESL by spreadsheet, the PMT function (with embedded NPV functions as<br />

needed) is used repeatedly for each year to calculate capital recovery and the AW of AOC. Their<br />

sum is the total AW for k years of ownership. The PMT function formats for the capital recovery<br />

and AOC components for each year k ( k 1, 2, 3, . . .) are as follows:<br />

Capital recovery for the challenger:<br />

Capital recovery for the defender:<br />

AW of AOC:<br />

PMT( i %,years, P, −MV_in_year_k)<br />

PMT( i %,years,current_MV,MV_in_<br />

year_k) [11.4]<br />

PMT( i %,years,NPV( i %,year_1_AOC:<br />

year_k_AOC)0)<br />

EXAMPLE 11.2<br />

When the spreadsheet is developed, it is recommended that the PMT functions in year 1 be developed<br />

using cell-reference format; then drag down the function through each column. A final<br />

column summing the two PMT results displays total AW. Augmenting the table with an Excel xy<br />

scatter chart graphically displays the cost curves in the general form of Figure 11–1 , and the ESL<br />

is easily identified. Example 11.2 illustrates ESL determination by hand and by spreadsheet.<br />

A 3-year-old backup power system is being considered for early replacement. Its current market<br />

value is $20,000. Estimated future market values and annual operating costs for the next<br />

5 years are given in Table 11–1, columns 2 and 3. What is the economic service life of this<br />

defender if the interest rate is 10% per year? Solve by hand and by spreadsheet.<br />

Solution by Hand<br />

Equation [11.3] is used to calculate total AW k for k 1, 2, . . . , 5. Table 11–1, column 4, shows<br />

the capital recovery for the $20,000 current market value ( j 0) plus 10% return. Column 5<br />

gives the equivalent AW of AOC for k years. As an illustration, the computation of total AW<br />

for k 3 from Equation [11.3] is<br />

Total AW 3 P(AP,i,3) MV 3 (AF,i,3) [PW of AOC 1 ,AOC 2 , and AOC 3 ](AP,i,3)<br />

20,000(AP,10%,3) 6000(AF,10%,3) [5000(PF,10%,1)<br />

6500(PF,10%,2) 8000(PF,10%,3)](AP,10%,3)<br />

6230 6405 $12,635<br />

A similar computation is performed for each year 1 through 5. The lowest equivalent cost<br />

( numerically largest AW value) occurs at k 3. Therefore, the defender ESL is n 3 years,<br />

and the AW value is $12,635. In the replacement study, this AW will be compared with the<br />

best challenger AW determined by a similar ESL analysis.<br />

Solution by Spreadsheet<br />

See Figure 11–2 for the spreadsheet screen shot and chart that shows the ESL is n 3 years<br />

and AW $12,634. (This format is a template for any ESL analysis; simply change the<br />

TABLE 11–1<br />

Computation of Economic Service Life<br />

Year j<br />

(1)<br />

MV j , $<br />

(2)<br />

AOC j , $<br />

(3)<br />

Capital<br />

Recovery, $<br />

(4)<br />

AW of<br />

AOC, $<br />

(5)<br />

Total<br />

AW k , $<br />

(6) (4) (5)<br />

1 10,000 5,000 12,000 5,000 17,000<br />

2 8,000 6,500 7,714 5,714 13,428<br />

3 6,000 8,000 6,230 6,405 12,635<br />

4 2,000 9,500 5,878 7,072 12,950<br />

5 0 12,500 5,276 7,961 13,237


11.2 Economic Service Life 299<br />

Current market value<br />

PMT($B$1,$A9,$B$2,$B9)<br />

PMT($B$1,$A9,NPV($B$1,$C$5:$C9)0)<br />

ESL of defender<br />

Total AW is minimum<br />

at n 3 years<br />

Total AW<br />

curve<br />

Capital<br />

recovery<br />

curve<br />

Figure 11–2<br />

Determination of ESL and plot of curves, Example 11.2<br />

estimates and add rows for more years.) Contents of columns D and E are described below. The<br />

PMT functions apply the formats as described in Equation [11.4]. Cell tags show detailed cellreference<br />

format for year 5. The $ symbols are included for absolute cell referencing, needed<br />

when the entry is dragged down through the column.<br />

Column D: Capital recovery is the AW of the $20,000 investment in year 0 for each year 1<br />

through 5 with the estimated MV in that year. For example, in actual numbers, the<br />

cell-reference PMT function in year 5 shown on the spreadsheet reads PMT (10%,5,20000,<br />

0), resulting in $5276. This series is plotted in Figure 11–2.<br />

Column E: The NPV function embedded in the PMT function obtains the present worth in<br />

year 0 of all AOC estimates through year k. Then PMT calculates the AW of AOC over<br />

k years. For example, in year 5, the PMT in numbers is PMT(10%,5,NPV<br />

(10%,C5:C9)0). The 0 is the AOC in year 0; it is optional. The graph plots the AW of<br />

AOC curve, which constantly increases in cost because the AOC estimates increase<br />

each year.<br />

Comment<br />

The capital recovery curve in Figure 11–2 (middle curve) is not the expected shape (see year 4)<br />

because the estimated market value changes each year. If the same MV were estimated for each<br />

year, the curve would appear like Figure 11–1. When several total AW values are approximately<br />

equal, the curve will be flat over several periods. This indicates that the ESL is relatively<br />

insensitive to costs.


300 Chapter 11 Replacement and Retention Decisions<br />

It is reasonable to ask about the difference between the ESL analysis above and the AW<br />

analyses performed in previous chapters. Previously we had a specifi c life estimated to be n years<br />

with associated other estimates: first cost in year 0, possibly a salvage value in year n , and an<br />

AOC that remained constant or varied each year. For all previous analyses, the calculation of AW<br />

using these estimates determined the AW over n years. This is the economic service life when n<br />

is fixed. Also, in all previous cases, there were no year-by-year market value estimates. Therefore,<br />

we can conclude the following:<br />

When the expected life n is known and specified for the challenger or defender, no ESL computations<br />

are necessary. Determine the AW over n years, using the first cost or current market<br />

value, estimated salvage value after n years, and AOC estimates. This AW value is the correct one<br />

to use in the replacement study.<br />

However, when n is not fixed, the following is useful. First the market/salvage series is needed.<br />

It is not difficult to estimate this series for a new or current asset. For example, an asset with a<br />

first cost of P can lose market value of, say, 20% per year, so the market value series for years<br />

0, 1, 2, . . . is P , 0.8 P , 0.64 P , . . . , respectively. If it is reasonable to predict the MV series on a<br />

year-by-year basis, it can be combined with the AOC estimates to produce what is called the<br />

marginal costs for the asset.<br />

Marginal costs (MC) are year-by-year estimates of the costs to own and operate an asset for that<br />

year. Three components are added to determine the marginal cost:<br />

• Cost of ownership (loss in market value is the best estimate of this cost)<br />

• Forgone interest on the market value at the beginning of the year<br />

• AOC for each year<br />

Once the marginal costs are estimated for each year, their equivalent AW value is calculated. The<br />

sum of the AW values of the fi rst two of these components is the capital recovery amount . Now, it<br />

should be clear that the total AW of all three marginal cost components over k years is the same<br />

value as the total annual worth for k years calculated in Equation [11.3]. That is, the following<br />

relation is correct.<br />

AW of marginal costs total AW of costs [11.5]<br />

Therefore, there is no need to perform a separate, detailed marginal cost analysis when yearly<br />

market values are estimated. The ESL analysis presented in Example 11.2 is sufficient in that<br />

it results in the same numerical values. This is demonstrated in Example 11.3 using the progressive<br />

example.<br />

EXAMPLE 11.3 Keep or Replace the Kiln Case<br />

PE<br />

In our progressive example, B&T Enterprises is considering the replacement of a 2-year-old<br />

kiln with a new one to meet emerging market needs. When the current tunnel kiln was purchased<br />

2 years ago for $25 million, an ESL study indicated that the minimum cost life was between<br />

3 and 5 years of the expected 8-year life. The analysis was not very conclusive because<br />

the total AW cost curve was flat for most years between 2 and 6, indicating insensitivity of the<br />

ESL to changing costs. Now, the same type of question arises for the proposed graphite hearth<br />

model that costs $38 million new: What are the ESL and the estimated total AW of costs? The<br />

Manager of Critical Equipment at B&T estimates that the market value after only 1 year will<br />

drop to $25 million and then retain 75% of the previous year’s value over the 12-year expected<br />

life. Use this market value series and i 15% per year to illustrate that an ESL analysis and<br />

marginal cost analysis result in exactly the same total AW of cost series.<br />

Solution<br />

Figure 11–3 is a spreadsheet screen shot of the two analyses in $ million units. The market<br />

value series is detailed in column B starting at $25 (million) and decreasing by 25% per year.<br />

A brief description of each analysis follows.


11.2 Economic Service Life 301<br />

ESL analysis<br />

Two AW series<br />

are identical<br />

Marginal cost analysis<br />

Figure 11–3<br />

Comparison of annual worth series resulting from ESL analysis and marginal cost analysis, Example 11.3.<br />

ESL analysis: Equation [11.4] is applied repeatedly for k 1, 2, . . . , 12 years (columns C,<br />

D, and E) in the top of Figure 11–3. Row 16 details the spreadsheet functions for year 12.<br />

The result in column F is the total AW series that is of interest now.<br />

Marginal cost (MC): The functions in the bottom of Figure 11–3 (columns C, D, and E)<br />

develop the three components added to obtain the MC series. Row 33 details the functions<br />

for year 12. The resulting AW of marginal costs (column G) is the series to compare with<br />

the corresponding ESL series above (column F).<br />

The two AW series are identical, thus demonstrating that Equation [11.5] is correct. Therefore,<br />

either an ESL or a marginal cost analysis will provide the same information for a replacement<br />

study. In this case, the results show that the new kiln will have a minimum AW of costs<br />

of $–12.32 million at its full 12-year life.<br />

We can draw two important conclusions about the n and AW values to be used in a replacement<br />

study. These conclusions are based on the extent to which detailed annual estimates are<br />

made for the market value.<br />

1. Year-by-year market value estimates are made. Use them to perform an ESL analysis,<br />

and determine the n value with the lowest total AW of costs. These are the best n and AW<br />

values for the replacement study.<br />

2. Yearly market value estimates are not available. The only estimate available is market<br />

value (salvage value) in year n . Use it to calculate the AW over n years. These are<br />

the n and AW values to use; however, they may not be the “best” values in that they may<br />

not represent the best equivalent total AW of cost value found if an ESL analysis were<br />

performed.


302 Chapter 11 Replacement and Retention Decisions<br />

Upon completion of the ESL analysis (item 1 above), the replacement study procedure in Section<br />

11.3 is applied using the values<br />

Challenger alternative (C):<br />

Defender alternative (D):<br />

AW C for n C years<br />

AW D for n D years<br />

11.3 Performing a Replacement Study<br />

Replacement studies are performed in one of two ways: without a study period specified or with<br />

one defined. Figure 11–4 gives an overview of the approach taken for each situation. The procedure<br />

discussed in this section applies when no study period (planning horizon) is specified. If a<br />

specific number of years is identified for the replacement study, for example, over the next<br />

5 years, with no continuation considered after this time period in the economic analysis, the procedure<br />

in Section 11.5 is applied.<br />

A replacement study determines when a challenger replaces the in-place defender. The complete<br />

study is finished if the challenger (C) is selected to replace the defender (D) now. However,<br />

if the defender is retained now, the study may extend over a number of years equal to the life of<br />

the defender n D , after which a challenger replaces the defender. Use the annual worth and life<br />

values for C and D determined in the ESL analysis in the following procedure. Assume the services<br />

provided by the defender could be obtained at the AW D amount.<br />

The replacement study procedure is:<br />

New replacement study:<br />

1. On the basis of the better AW C or AW D value, select the challenger C or defender D. When the<br />

challenger is selected, replace the defender now, and expect to keep the challenger for n C<br />

years. This replacement study is complete. If the defender is selected, plan to retain it for up<br />

to n D more years. (This is the leftmost branch of Figure 11–4 .) Next year, perform the<br />

following steps.<br />

One-year-later analysis:<br />

2. Determine if all estimates are still current for both alternatives, especially first cost, market<br />

value, and AOC. If not, proceed to step 3. If yes and this is year n D , replace the defender. If<br />

this is not year n D , retain the defender for another year and repeat this same step. This step<br />

may be repeated several times.<br />

3. Whenever the estimates have changed, update them and determine new AW C and AW D values.<br />

Initiate a new replacement study (step 1).<br />

Figure 11–4<br />

Overview of replacement<br />

study approaches.<br />

Replacement study<br />

No study period<br />

specified<br />

Study period<br />

specified<br />

Perform ESL<br />

analysis<br />

Develop succession<br />

options for D and C<br />

using AW of<br />

respective cash flows<br />

AW D<br />

AW C<br />

PW or AW for<br />

each option<br />

Select better AW<br />

Select best option


11.3 Performing a Replacement Study 303<br />

If the defender is selected initially (step 1), estimates may need updating after 1 year of retention<br />

(step 2). Possibly there is a new best challenger to compare with D. Either significant<br />

changes in defender estimates or availability of a new challenger indicates that a new replacement<br />

study is to be performed. In actuality, a replacement study can be performed each year or<br />

more frequantly to determine the advisability of replacing or retaining any defender, provided<br />

a competitive challenger is available.<br />

Example 11.4 illustrates the application of ESL analysis for a challenger and defender, followed<br />

by the use of the replacement study procedure. The planning horizon is unspecified in this<br />

example.<br />

EXAMPLE 11.4<br />

Two years ago, Toshiba Electronics made a $15 million investment in new assembly line<br />

machinery. It purchased approximately 200 units at $70,000 each and placed them in plants<br />

in 10 different countries. The equipment sorts, tests, and performs insertion-order kitting on<br />

electronic components in preparation for special-purpose circuit boards. This year, new international<br />

industry standards will require a $16,000 retrofit on each unit, in addition to the expected<br />

operating cost. Due to the new standards, coupled with rapidly changing technology, a<br />

new system is challenging the retention of these 2-year-old machines. The chief engineer at<br />

Toshiba USA realizes that the economics must be considered, so he has asked that a replacement<br />

study be performed this year and each year in the future, if need be. The i is 10% and the<br />

estimates are below.<br />

Challenger: First cost: $50,000<br />

Future market values: decreasing by 20% per year<br />

Estimated retention period: no more than 10 years<br />

AOC estimates: $5000 in year 1 with increases of $2000 per year<br />

thereafter<br />

Defender: Current international market value: $15,000<br />

Future market values: decreasing by 20% per year<br />

Estimated retention period: no more than 3 more years<br />

AOC estimates: $4000 next year, increasing by $4000 per year<br />

thereafter, plus the $16,000 retrofit next year<br />

(a) Determine the AW values and economic service lives necessary to perform the replacement<br />

study.<br />

(b) Perform the replacement study now.<br />

(c) After 1 year, it is time to perform the follow-up analysis. The challenger is making large<br />

inroads to the market for electronic components assembly equipment, especially with the<br />

new international standards features built in. The expected market value for the defender is<br />

still $12,000 this year, but it is expected to drop to virtually nothing in the future—$2000<br />

next year on the worldwide market and zero after that. Also, this prematurely outdated<br />

equipment is more costly to keep serviced, so the estimated AOC next year has been increased<br />

from $8000 to $12,000 and to $16,000 two years out. Perform the follow-up replacement<br />

study analysis.<br />

Solution<br />

(a) The results of the ESL analysis, shown in Figure 11–5, include all the MV and AOC estimates<br />

in columns B and C. For the challenger, note that P $50,000 is also the MV in<br />

year 0. The total AW of costs is for each year, should the challenger be placed into service<br />

for that number of years. As an example, the year k 4 amount of $19,123 is determined<br />

using Equation [11.3], where the AG factor accommodates the arithmetic gradient<br />

series in the AOC.<br />

Total AW 4 50,000(AP,10%,4) 20,480(AF,10%,4)<br />

[5000 2000(AG,10%,4)]<br />

$19,123


304 Chapter 11 Replacement and Retention Decisions<br />

ESL Analysis of Challenger<br />

ESL results<br />

ESL Analysis of Defender<br />

Figure 11–5<br />

ESL analysis of challenger and defender, Example 11.4.<br />

For spreadsheet-based ESL analysis, this same result is achieved in cell F8 using Equation<br />

[11.4]. The functions are<br />

Total AW 4 PMT(10%,4,50000,20480) PMT(10%,4,NPV(10%,C5:C8)0)<br />

11,361 7,762<br />

19,123<br />

The defender costs are analyzed in the same way up to the maximum retention period of<br />

3 years.<br />

The lowest AW cost (numerically largest) values for the replacement study are as follows:<br />

Challenger: AW C $19,123 for n C 4 years<br />

Defender: AW D $17,307 for n D 3 years<br />

The challenger total AW of cost curve (Figure 11–5) is classically shaped and relatively flat<br />

between years 3 and 6; there is virtually no difference in the total AW for years 4 and 5. For<br />

the defender, note that the estimated AOC values change substantially over 3 years, and<br />

they do not constantly increase or decrease.<br />

(b) To perform the replacement study now, apply only the first step of the procedure. Select the<br />

defender because it has the better AW of costs ($17,307), and expect to retain it for<br />

3 more years. Prepare to perform the one-year-later analysis 1 year from now.<br />

(c) One year later, the situation has changed significantly for the equipment Toshiba retained<br />

last year. Apply the steps for the one-year-later analysis:<br />

2. After 1 year of defender retention, the challenger estimates are still reasonable, but the<br />

defender market value and AOC estimates are substantially different. Go to step 3 to<br />

perform a new ESL analysis for the defender.<br />

3. The defender estimates in Figure 11–5 are updated below for the ESL analysis. New<br />

AW values are calculated using Equation [11.3]. There is now a maximum of 2 more<br />

years of retention, 1 year less than the 3 years determined last year.<br />

Year k<br />

Market<br />

Value, $ AOC, $<br />

Total AW<br />

If Retained k More Years, $<br />

0 12,000 — —<br />

1 2,000 12,000 23,200<br />

2 0 16,000 20,819


11.3 Performing a Replacement Study 305<br />

The AW and n values for the new replacement study are as follows:<br />

Challenger:<br />

Defender:<br />

unchanged at AW C $19,123 for n C 4 years<br />

new AW D $20,819 for n D 2 more years<br />

Now select the challenger based on its favorable AW value. Therefore, replace the defender<br />

now, not 2 years from now. Expect to keep the challenger for 4 years, or until a better challenger<br />

appears on the scene.<br />

EXAMPLE 11.5 Keep or Replace the Kiln Case<br />

We continue with the progressive example of possibly replacing a kiln at B&T Enterprises. A<br />

marketing study revealed that the improving business activity on the west coast implies that<br />

the revenue profile between the installed kiln (PT) and the proposed new one (GH) would be<br />

the same, with the new kiln possibly bringing in new revenue within the next couple of years.<br />

The president of B&T decided it was time to do a replacement study. Assume you are the lead<br />

engineer and that you previously completed the ESL analysis on the challenger (Example<br />

11.3). It indicates that for the GH system the ESL is its expected useful life.<br />

Challenger: ESL n GH 12 years with total equivalent annual cost AW GH $12.32 million<br />

The president asked you to complete the replacement study, stipulating that, due to the rapidly<br />

rising annual operating costs (AOC), the defender would be retained a maximum of 6 years.<br />

You are expected to make the necessary estimates for the defender (PT) and perform the study<br />

at a 15% per year return.<br />

PE<br />

Solution<br />

After some data collection, you have good evidence that the market value for the PT system<br />

will stay high, but that the increasing AOC is expected to continue rising about $1.2 million per<br />

year. The best estimates for the next 6 years in $ million units are these:<br />

Year 1 2 3 4 5 6<br />

Market Value, $ M 22.0 22.0 22.0 20.0 18.0 18.0<br />

AOC, $ M per year 5.2 6.4 7.6 8.8 10.0 11.2<br />

You developed a spreadsheet and performed the analysis in Figure 11–6. As an illustration,<br />

total AW computation for 3 years of retention, in $ million units, is<br />

Total AW 3 22.0(AP,15%,3) 22.0(AF,15%,3) [5.2(PF,15%,1) 6.4(PF,15%,2)<br />

7.6(PF,15%,3)](AP,15%,3)<br />

–9.63 6.34 [14.36] (0.43798)<br />

$9.59 per year<br />

Though the system could be retained up to 6 years, the ESL is much shorter at 1 year.<br />

Defender: ESL n PT 1 year with total equivalent annual cost AW PT $8.50 million<br />

Figure 11–6<br />

ESL analysis of defender kiln PT for progressive example, Example 11.5.


306 Chapter 11 Replacement and Retention Decisions<br />

To make the replacementretention decision, apply step 1 of the procedure. Since AW PT <br />

$8.50 million per year is considerably less than AW $12.32 million, you should recommend<br />

keeping the current kiln only 1 more year and doing another study during the year<br />

to determine if the current estimates are still reliable.<br />

Comment<br />

An observation of the trends of the two final AW series in this problem is important. A comparison<br />

of Figure 11–3 (top), column F, and Figure 11–6, column F, shows us that the largest<br />

total AW of the current system ($–11.47 M for 6 years) is still below the smallest total AW of<br />

the proposed system ($12.32 M for 12 years). This indicates that the graphite hearth system<br />

(the challenger) will not be chosen on an economic basis, if the decision to consider the installed<br />

kiln as a defender in the future were made. It would take some significant estimate<br />

changes to justify the challenger.<br />

11.4 Additional Considerations in a<br />

Replacement Study<br />

There are several additional aspects of a replacement study that may be introduced. Three of<br />

these are identified and discussed in turn.<br />

• Future-year replacement decisions at the time of the initial replacement study<br />

• Opportunity cost versus cash flow approaches to alternative comparison<br />

• Anticipation of improved future challengers<br />

In most cases when management initiates a replacement study, the question is best framed as,<br />

“Replace now, 1 year from now, 2 years from now, etc.?” The procedure above does answer this<br />

question provided the estimates for C and D do not change as each year passes. In other words,<br />

at the time it is performed, step 1 of the procedure does answer the replacement question for<br />

multiple years. It is only when estimates change over time that the decision to retain the defender<br />

may be prematurely reversed (prior to n D years) in favor of the then-best challenger.<br />

The first costs ( P values) for the challenger and defender have been correctly taken as the<br />

initial investment for the challenger C and current market value for the defender D. This is called<br />

the opportunity cost approach because it recognizes that a cash inflow of funds equal to the<br />

market value is forgone if the defender is selected. This approach, also called the conventional<br />

approach, is correct for every replacement study. A second approach, called the cash flow approach,<br />

recognizes that when C is selected, the market value cash inflow for the defender is received<br />

and, in effect, immediately reduces the capital needed to invest in the challenger. Use of<br />

the cash fl ow approach is strongly discouraged for at least two reasons: possible violation of the<br />

equal-service requirement and incorrect capital recovery value for C. As we are aware, all economic<br />

evaluations must compare alternatives with equal service. Therefore, the cash flow approach<br />

can work only when challenger and defender lives are exactly equal. This is commonly<br />

not the case; in fact, the ESL analysis and the replacement study procedure are designed to compare<br />

two mutually exclusive, unequal-life alternatives via the annual worth method. If this equalservice<br />

comparison reason is not enough to avoid the cash flow approach, consider what happens<br />

to the challenger’s capital recovery amount when its first cost is decreased by the market value of<br />

the defender. The capital recovery (CR) terms in Equation [11.3] will decrease, resulting in a<br />

falsely low value of CR for the challenger, were it selected. From the vantage point of the economic<br />

study itself, the decision for C or D will not change; but when C is selected and implemented,<br />

this CR value is not reliable. The conclusion is simple:<br />

Use the initial investment of C and the market value of D as the first costs in the ESL analysis and<br />

in the replacement study.<br />

A basic premise of a replacement study is that some challenger will replace the defender at a future<br />

time, provided the service continues to be needed and a worthy challenger is available. The expectation<br />

of ever-improving challengers can offer strong encouragement to retain the defender until<br />

some situational elements—technology, costs, market fluctuations, contract negotiations, etc.—


11.5 Replacement Study over a Specified Study Period 307<br />

stabilize. This was the case in the previous two examples. A large expenditure on equipment when<br />

the standards changed soon after purchase forced an early replacement consideration and a large loss<br />

of invested capital. The replacement study is no substitute for forecasting challenger availability. It is<br />

important to understand trends, new advances, and competitive pressures that can complement the<br />

economic outcome of a good replacement study . It is often better to compare a challenger with an<br />

augmented defender in the replacement study. Adding needed features to a currently installed defender<br />

may prolong its useful life and productivity until challenger choices are more appealing.<br />

It is possible that a significant tax impact may occur when a defender is traded early in its<br />

expected life. If taxes should be considered, proceed now, or after the next section, to Chapter 17<br />

and the after-tax replacement analysis in Section 17.7.<br />

11.5 Replacement Study over a<br />

Specified Study Period<br />

When the time period for the replacement study is limited to a specified study period or planning<br />

horizon, for example, 6 years, the ESL analysis is not performed.<br />

The AW values for the challenger and for the remaining life of the defender are not based on the<br />

economic service life; the AW is calculated over the study period only. What happens to the alternatives<br />

after the study period is not considered in the replacement analysis.<br />

This means that the defender or challenger is not needed beyond the study period. In fact,<br />

a study period of fixed duration does not comply with the three assumptions stated in<br />

Section 11.1—service needed for indefinite future, best challenger available now, and estimates<br />

will be identical for future life cycles.<br />

When performing a replacement study over a fixed study period, it is crucial that the estimates<br />

used to determine the AW values be accurate and used in the study. This is especially important for<br />

the defender. Failure to do the following violates the requirement of equal-service comparison.<br />

When the defender’s remaining life is shorter than the study period , the cost of providing<br />

the defender’s services from the end of its expected remaining life to the end of the study<br />

period must be estimated as accurately as possible and included in the replacement study.<br />

Study period<br />

The right branch of Figure 11–4 presents an overview of the replacement study procedure for a<br />

stated study period.<br />

1. Succession options and AW values . Develop all the viable ways to use the defender and challenger<br />

during the study period. There may be only one option or many options; the longer the<br />

study period, the more complex this analysis becomes. The AW values for the challenger and<br />

defender cash flows are used to build the equivalent cash flow values for each option.<br />

2. Selection of the best option . The PW or AW for each option is calculated over the study<br />

period. Select the option with the lowest cost, or highest income if revenues are estimated.<br />

(As before, the best option will have the numerically largest PW or AW value.)<br />

The following examples use this procedure and illustrate the importance of making cost estimates<br />

for the defender alternative when its remaining life is less than the study period.<br />

EXAMPLE 11.6<br />

Claudia works with Lockheed-Martin (LMCO) in the aircraft maintenance division. She is<br />

preparing for what she and her boss, the division chief, hope to be a new 10-year defense contract<br />

with the U.S. Air Force on C-5A cargo aircraft. A key piece of equipment for maintenance<br />

operations is an avionics circuit diagnostics system. The current system was purchased 7 years<br />

ago on an earlier contract. It has no capital recovery costs remaining, and the following are<br />

reliable estimates: current market value $70,000, remaining life of 3 more years, no salvage<br />

value, and AOC $30,000 per year. The only options for this system are to replace it now or<br />

retain it for the full 3 additional years.<br />

Claudia has found that there is only one good challenger system. Its cost estimates are: first<br />

cost $750,000, life 10 years, S 0, and AOC $50,000 per year.


308 Chapter 11 Replacement and Retention Decisions<br />

Realizing the importance of accurate defender alternative cost estimates, Claudia asked the<br />

division chief what system would be a logical follow-on to the current one 3 years hence, if<br />

LMCO wins the contract. The chief predicted LMCO would purchase the very system she had<br />

identified as the challenger, because it is the best on the market. The company would keep it<br />

for the entire 10 additional years for use on an extension of this contract or some other application<br />

that could recover the remaining 3 years of invested capital. Claudia interpreted the response<br />

to mean that the last 3 years would also be capital recovery years, but on some project<br />

other than this one. Claudia’s estimate of the first cost of this same system 3 years from now is<br />

$900,000. Additionally, the $50,000 per year AOC is the best estimate at this time.<br />

The division chief mentioned any study had to be conducted using the interest rate of 10%,<br />

as mandated by the U.S. Office of Management and Budget (OMB). Perform a replacement<br />

study for the fixed contract period of 10 years.<br />

Solution<br />

The study period is fixed at 10 years, so the intent of the replacement study assumptions is not<br />

present. This means the defender follow-on estimates are very important to the analysis.<br />

Further, any analyses to determine the ESL values are unnecessary since alternative lives are<br />

already set and no projected annual market values are available. The first step of the replacement<br />

study procedure is to define the options. Since the defender will be replaced now or in<br />

3 years, there are only two options:<br />

1. Challenger for all 10 years.<br />

2. Defender for 3 years, followed by the challenger for 7 years.<br />

Cash flows are diagrammed in Figure 11–7. For option 1, the challenger is used for all 10 years.<br />

Equation [11.3] is applied to calculate AW using the following estimates:<br />

Challenger: P $750,000 AOC $50,000<br />

n 10 years S 0<br />

AW C 750,000(AP,10%,10) 50,000 $172,063<br />

0 1 2 3 4 5 6 7 8 9 10<br />

Year<br />

P = $750,000<br />

AOC = $50,000<br />

(a) Challenger (option 1)<br />

End of<br />

study period<br />

0 1 2 3 4 5 6 7 8 9 10 11 12 13<br />

Year<br />

AOC = $30,000<br />

MV = $70,000<br />

Defendercurrent<br />

AOC = $50,000<br />

CR = $146,475<br />

Defender-follow-on<br />

CR for another<br />

project<br />

(b) Defender alternative (option 2)<br />

Figure 11–7<br />

Cash flow diagrams for a 10-year study period replacement study, Example 11.6.


11.5 Replacement Study over a Specified Study Period 309<br />

The second option has more complex cost estimates. The AW for the in-place system is calculated<br />

over the first 3 years. Added to this is the capital recovery for the defender follow-on for the<br />

next 7 years. However in this case, the CR amount is determined over its full 10-year life. (It is not<br />

unusual for the recovery of invested capital to be moved between projects, especially for contract<br />

work.) Refer to the AW components as AW DC (subscript DC for defender current) and AW DF (subscript<br />

DF for defender follow-on). The final cash flows are shown in Figure 11–7b.<br />

Defender current: market value $70,000 AOC $30,000<br />

n 3 years S 0<br />

AW DC [70,000 30,000(PA,10%,3)](AP,10%,10) $23,534<br />

Defender follow-on: P $900,000, n 10 years for capital recovery calculation<br />

only, AOC $–50,000 for years 4 through 10, S 0.<br />

The CR and AW for all 10 years are<br />

CR DF 900,000(AP,10%,10) $146,475 [11.6]<br />

AW DF (146,475 50,000)(FA,10%,7)(AF,10%,10) $116,966<br />

Total AW D for the defender is the sum of the two annual worth values above. This is the AW<br />

for option 2.<br />

AW D AW DC AW DF 23,534 116,966 $140,500<br />

Option 2 has a lower cost ($140,500 versus $172,063). Retain the defender now and expect<br />

to purchase the follow-on system 3 years hence.<br />

Comment<br />

The capital recovery cost for the defender follow-on will be borne by some yet-to-be-identified<br />

project for years 11 through 13. If this assumption were not made, its capital recovery cost would<br />

be calculated over 7 years, not 10, in Equation [11.6], increasing CR to $184,869. This raises the<br />

annual worth to AW D $163,357. The defender alternative (option 2) is still selected.<br />

EXAMPLE 11.7<br />

Three years ago Chicago’s O’Hare Airport purchased a new fire truck. Because of flight increases,<br />

new fire-fighting capacity is needed once again. An additional truck of the same capacity<br />

can be purchased now, or a double-capacity truck can replace the current fire truck. Estimates<br />

are presented below. Compare the options at 12% per year using (a) a 12-year study<br />

period and (b) a 9-year study period.<br />

Presently Owned New Purchase Double Capacity<br />

First cost P, $ 151,000 (3 years ago) 175,000 190,000<br />

AOC, $ 1,500 1,500 2,500<br />

Market value, $ 70,000 — —<br />

Salvage value, $ 10% of P 12% of P 10% of P<br />

Life, years 12 12 12<br />

Solution<br />

Identify option 1 as retention of the presently owned truck and augmentation with a new samecapacity<br />

vehicle. Define option 2 as replacement with the double-capacity truck.<br />

Option 1 Option 2<br />

Presently Owned Augmentation Double Capacity<br />

P, $ 70,000 175,000 190,000<br />

AOC, $ 1,500 1,500 2,500<br />

S, $ 15,100 21,000 19,000<br />

n, years 9 12 12


310 Chapter 11 Replacement and Retention Decisions<br />

(a) For a full-life 12-year study period of option 1,<br />

AW 1 (AW of presently owned) (AW of augmentation)<br />

[70,000(AP,12%,9) 15,100(AF,12%,9) 1500]<br />

[175,000(AP,12%,12) 21,000(AF,12%,12) 1500]<br />

13,616 28,882<br />

$42,498<br />

This computation assumes the equivalent services provided by the current fire truck can be<br />

purchased at $−13,616 per year for years 10 through 12.<br />

AW 2 190,000(AP,12%,12) 19,000(AF,12%,12) 2500<br />

$32,386<br />

Replace now with the double-capacity truck (option 2) at an advantage of $10,112 per year.<br />

(b) The analysis for an abbreviated 9-year study period is identical, except that n 9 in each<br />

factor; that is, 3 fewer years are allowed for the augmentation and double-capacity trucks<br />

to recover the capital investment plus a 12% per year return. The salvage values remain the<br />

same since they are quoted as a percentage of P for all years.<br />

Option 2 is again selected.<br />

AW 1 $46,539<br />

AW 2 $36,873<br />

The previous two examples indicate an important consideration for setting the length of the<br />

study period for a replacement analysis. It involves the capital recovery amount for the challenger,<br />

when the strict definition of a study period is applied.<br />

Study period<br />

Capital recovery<br />

When a study period shorter than the life of the challenger is defined, the challenger’s capital<br />

recovery amount increases in order to recover the initial investment plus a return in this<br />

shortened time period . Highly abbreviated study periods tend to disadvantage the challenger<br />

because no consideration of time beyond the end of the study period is made in calculating the<br />

challenger’s capital recovery amount.<br />

If there are several options for the number of years that the defender may be retained before<br />

replacement with the challenger, the first step of the replacement study—succession options and<br />

AW values—must include all the viable options. For example, if the study period is 5 years and<br />

the defender will remain in service 1 year, or 2 years, or 3 years, cost estimates must be made to<br />

determine AW values for each defender retention period. In this case, there are four options; call<br />

them W, X, Y, and Z.<br />

Option<br />

Defender<br />

Retained, Years<br />

Challenger<br />

Serves, Years<br />

W 3 2<br />

X 2 3<br />

Y 1 4<br />

Z 0 5<br />

The respective AW values for defender retention and challenger use define the cash flows for<br />

each option. Example 11.8 illustrates the procedure using the progressive example.<br />

EXAMPLE 11.8 Keep or Replace the Kiln Case<br />

PE<br />

We have progressed to the point that the replacement study between the defender PT and<br />

challenger GH was completed (Example 11.5). The defender was the clear choice with a<br />

much smaller AW value ($8.50 M) than that of the challenger ($12.32 M). Now the<br />

management of B&T is in a dilemma. They know the current tunnel kiln is much cheaper<br />

than the new graphite hearth, but the prospect of future new business should not be dismissed.


11.5 Replacement Study over a Specified Study Period 311<br />

TABLE 11–2 Replacement Study Options and Total AW Values, Example 11.8<br />

Defender PT<br />

Challenger GH<br />

Option Years Retained AW, $ MYear Years Retained AW, $ MYear<br />

A 1 8.50 5 14.21<br />

B 2 9.06 4 15.08<br />

C 3 9.59 3 16.31<br />

D 4 10.49 2 18.21<br />

E 5 11.16 1 22.10<br />

F 6 11.47 0 —<br />

The president asked, “Is it possible to determine when it is economically the cheapest to<br />

purchase the new kiln, provided the current one is kept at least 1 year, but no more than 6<br />

years, its remaining expected life?” The chief financial officer answered, yes, of course.<br />

(a) Determine the answer for the president. (b) Discuss the next step in the analysis based on<br />

the conclusion reached here.<br />

Solution<br />

(a) Actually, this is a quite easy question to answer, because all the information has been determined<br />

previously. We know the MARR is 15% per year, the study period has been established<br />

at 6 years, and the defender PT will stay in place between 1 and 6 years. Therefore,<br />

the challenger GH will be considered for 0 to 5 years of service. The total AW values<br />

were determined for the defender in Example 11.5 (Figure 11–6) and for the challenger in<br />

Example 11.3 (Figure 11–3). They are repeated in Table 11–2 for convenience. Use the<br />

procedure for a replacement study with a fixed study period.<br />

Step 1: Succession options and AW values. There are six options in this case; the defender<br />

is retained from 1 to 6 years while the challenger is installed from 0 to 5 years. We will<br />

label them A through F. Figure 11–8 presents the options and the AW series for each option<br />

from Table 11–2. No consideration of the fact that the challenger has an expected life of<br />

12 years is made since the study period is fixed at 6 years.<br />

Step 2: Selection of the best option. The PW value for each option is determined over the<br />

6-year study period in column J of Figure 11–8. The conclusion is clearly to keep the<br />

defender in place for 6 more years.<br />

(b) Every replacement analysis has indicated that the defender should be retained for the near<br />

future. If the analysis is to be carried further, the possibility of increased revenue based on<br />

services of the challenger’s high-temperature and operating efficiency should be considered<br />

next. In the introductory material, new business opportunities were mentioned. A revenue<br />

increase for the challenger will reduce its AW of costs and possibly make it more<br />

economically viable.<br />

Conclusion: Keep defender all 6 years<br />

Figure 11–8<br />

PW values for 6-year study period replacement analysis, Example 11.8.


312 Chapter 11 Replacement and Retention Decisions<br />

11.6 Replacement Value<br />

Often it is helpful to know the minimum market value of the defender necessary to make the<br />

challenger economically attractive. If a realizable market value or trade-in of at least this amount<br />

can be obtained, from an economic perspective the challenger should be selected immediately.<br />

This is a breakeven value between AW C and AW D ; it is referred to as the replacement value<br />

(RV). Set up the relation AW C AW D with the market value for the defender identified as RV,<br />

which is the unknown. The AW C is known, so RV can be determined. The selection guideline is<br />

as follows:<br />

If the actual market trade-in exceeds the breakeven replacement value , the challenger is the better<br />

alternative and should replace the defender now.<br />

Determination of the RV for a defender is an excellent opportunity to utilize the Goal Seek tool<br />

in Excel. The target cell is the current market value, and the AW D value is forced to equal the<br />

AW C amount. Example 11.9 discusses a replacement value of the progressive example.<br />

EXAMPLE 11.9 Keep or Replace the Kiln Case<br />

As one final consideration of the challenger kiln, you decide to determine what the trade-in<br />

amount would have to be so that the challenger is the economic choice next year. This is based<br />

on the ESL analysis that concluded the following (Examples 11.3 and 11.5):<br />

Defender:<br />

Challenger:<br />

ESL n PT 1 year with AW PT $8.50 million<br />

ESL n GH 12 years with AW GH $12.32 million<br />

The original defender price was $25 million, and a current market value of $22 million was<br />

estimated earlier (Figure 11–6). Since the installed kiln is known for retention of its market<br />

value (MV), you are hopeful the difference between RV and estimated MV may not be so significant.<br />

What will you discover RV to be? The MARR is 15% per year.<br />

Solution<br />

Set the AW relation for the defender for the ESL time of 1 year equal to AW GH $12.32<br />

and solve for RV. The estimates for AOC and MV next year are in Figure 11–6; they are, in<br />

$ million,<br />

Year 1: AOC $−5.20 MV $22.0<br />

12.32 RV(AP,15%,1) 22.00(AF,15%,1) 5.20<br />

1.15RV 12.32 22.00 5.20<br />

RV $25.32<br />

Though the RV is larger than the defender’s estimated MV of $22 million, some flexibility in<br />

the trade-in offer or the challenger’s first cost may cause the challenger to be economically<br />

justifiable.<br />

Comment<br />

To find RV using a spreadsheet, return to Figure 11–6. In the Goal Seek template, the “set” cell<br />

is the AW for 1 year (currently $8.50), and the required value is $12.32, the AW GH for its<br />

ESL of 12 years. The “changing” cell is the current market value (cell F2), currently $22.00.<br />

When “OK” is touched, $25.32 is displayed as the breakeven market value. This is the RV.<br />

PE<br />

CHAPTER SUMMARY<br />

It is important in a replacement study to compare the best challenger with the defender. Best<br />

(economic) challenger is described as the one with the lowest annual worth (AW) of costs for<br />

some number of years . If the expected remaining life of the defender and the estimated life of the<br />

challenger are specified, the AW values over these years are determined and the replacement<br />

study proceeds. However, if reasonable estimates of the expected market value (MV) and AOC


for each year of ownership can be made, these year-by-year (marginal) costs help determine the<br />

best challenger.<br />

The economic service life (ESL) analysis is designed to determine the best challenger’s years<br />

of service and the resulting lowest total AW of costs. The resulting n C and AW C values are used<br />

in the replacement study. The same analysis can be performed for the ESL of the defender.<br />

Replacement studies in which no study period (planning horizon) is specified utilize the annual<br />

worth method of comparing two unequal-life alternatives. The better AW value determines<br />

how long the defender is retained before replacement.<br />

When a study period is specified for the replacement study, it is vital that the market value and<br />

cost estimates for the defender be as accurate as possible. When the defender’s remaining life is<br />

shorter than the study period, it is critical that the cost for continuing service be estimated carefully.<br />

All the viable options for using the defender and challenger are enumerated, and their AW<br />

equivalent cash flows are determined. For each option, the PW or AW value is used to select the<br />

best option. This option determines how long the defender is retained before replacement.<br />

Problems 313<br />

PROBLEMS<br />

Foundations of Replacement<br />

11.1 In a replacement study, what is meant by “taking<br />

the nonowner’s viewpoint”?<br />

11.2 An asset that was purchased 3 years ago for<br />

$100,000 is becoming obsolete faster than expected.<br />

The company thought the asset would last<br />

5 years and that its book value would decrease by<br />

$20,000 each year and, therefore, be worthless at<br />

the end of year 5. In considering a more versatile,<br />

more reliable high-tech replacement, the company<br />

discovered that the presently owned asset has a<br />

market value of only $15,000. If the replacement is<br />

purchased immediately at a first cost of $75,000<br />

and if it will have a lower annual worth, what is the<br />

amount of the sunk cost? Assume the company’s<br />

MARR is 15% per year.<br />

11.3 As a muscle car aficionado, a friend of yours likes<br />

to restore cars of the 60s and 70s and sell them for<br />

a profit. He started his latest project (a 1965 Shelby<br />

GT350) four months ago and has a total of<br />

$126,000 invested so far. Another opportunity has<br />

come up (a 1969 Dodge Charger) that he is thinking<br />

of buying because he believes he could sell it<br />

for a profit of $60,000 after it is completely restored.<br />

To do so, however, he would have to sell<br />

the unfinished Shelby first. He thought that the<br />

completely restored Shelby would be worth<br />

$195,000, resulting in a tidy profit of $22,000, but<br />

in its half-restored condition, the most he could get<br />

now is $115,000. In discussing the situation with<br />

you, he stated that if he could sell the Shelby now<br />

and buy the Charger at a reduced price, he would<br />

make up for the money he will lose in selling the<br />

Shelby at a lower-than-desired price.<br />

( a) What is wrong with this thinking?<br />

( b) What is his sunk cost in the Shelby?<br />

11.4 In conducting a replacement study wherein the<br />

planning horizon is unspecified, list three assumptions<br />

that are inherent in an annual worth analysis<br />

of the defender and challenger.<br />

11.5 A civil engineer who owns his own designbuild<br />

operate company purchased a small crane 3 years<br />

ago at a cost of $60,000. At that time, it was expected<br />

to be used for 10 years and then traded in<br />

for its salvage value of $10,000. Due to increased<br />

construction activities, the company would prefer<br />

to trade for a new, larger crane now that will cost<br />

$80,000. The company estimates that the old crane<br />

can be used, if necessary, for another 3 years, at<br />

which time it would have a $23,000 estimated<br />

market value. Its current market value is estimated<br />

to be $39,000, and if it is used for another 3 years,<br />

it will have M&O costs (exclusive of operator<br />

costs) of $17,000 per year. Determine the values of<br />

P , n , S , and AOC that should be used for the existing<br />

crane in a replacement analysis.<br />

11.6 Equipment that was purchased by Newport Corporation<br />

for making pneumatic vibration isolators<br />

cost $90,000 two years ago. It has a market value<br />

that can be described by the relation $90,000 <br />

8000 k , where k is the years from time of purchase.<br />

Experience with this type of equipment has shown<br />

that the operating cost for the first 4 years is<br />

$65,000 per year, after which it increases by $6300<br />

per year. The asset’s salvage value was originally<br />

estimated to be $7000 after a predicted 10-year<br />

useful life. Determine the values of P , S , and AOC<br />

if a replacement study is done ( a ) now and<br />

( b ) 1 year from now.<br />

11.7 A piece of equipment that was purchased 2 years<br />

ago by Toshiba Imaging for $50,000 was expected<br />

to have a useful life of 5 years with a $5000 salvage


314 Chapter 11 Replacement and Retention Decisions<br />

value. Its performance was less than expected, and<br />

it was upgraded for $20,000 one year ago. Increased<br />

demand now requires that the equipment<br />

be upgraded again for another $17,000 so that it<br />

can be used for 3 more years. If upgraded, its annual<br />

operating cost will be $27,000 and it will<br />

have a $12,000 salvage after 3 years. Alternatively,<br />

it can be replaced with new equipment priced at<br />

$65,000 with operating costs of $14,000 per year<br />

and a salvage value of $23,000 after 6 years. If replaced<br />

now, the existing equipment will be sold for<br />

$7000. Determine the values of P , S , AOC, and n<br />

for the defender in a replacement study.<br />

Economic Service Life<br />

11.8 For equipment that has a first cost of $10,000 and<br />

the estimated operating costs and year-end salvage<br />

values shown below, determine the economic service<br />

life at i 10% per year.<br />

Year<br />

Operating Cost,<br />

$ per Year<br />

Salvage<br />

Value, $<br />

1 1000 7000<br />

2 1200 5000<br />

3 1300 4500<br />

4 2000 3000<br />

5 3000 2000<br />

11.9 To improve package tracking at a UPS transfer facility,<br />

conveyor equipment was upgraded with<br />

RFID sensors at a cost of $345,000. The operating<br />

cost is expected to be $148,000 per year for the<br />

first 3 years and $210,000 for the next 3 years. The<br />

salvage value of the equipment is expected to be<br />

$140,000 for the first 3 years, but due to obsolescence,<br />

it won’t have a significant value after that.<br />

At an interest rate of 10% per year, determine<br />

(a) The economic service life of the equipment<br />

and associated annual worth<br />

( b) The percentage increase in the AW of cost if<br />

the equipment is retained 2 years longer than<br />

the ESL<br />

11.10 Economic service life calculations for an asset are<br />

shown below. If an interest rate of 10% per year<br />

was used in making the calculations, determine the<br />

values of P and S that were used in calculating the<br />

AW for year 3.<br />

Years<br />

Retained<br />

AW of First<br />

Cost, $<br />

AW of Operating<br />

Cost, $ per Year<br />

AW of<br />

Salvage Value, $<br />

1 51,700 15,000 35,000<br />

2 27,091 17,000 13,810<br />

3 18,899 19,000 6,648<br />

4 14,827 21,000 4,309<br />

5 12,398 23,000 2,457<br />

11.11 The initial cost of a bridge that is expected to be in<br />

place forever is $70 million. Maintenance can be<br />

done at 1-, 2-, 3-, or 4-year intervals, but the longer<br />

the interval between servicing, the higher the cost.<br />

The costs of servicing are estimated at $83,000,<br />

$91,000, $125,000, and $183,000 for intervals of 1<br />

through 4 years, respectively. What interval should<br />

be scheduled for maintenance to minimize the<br />

overall equivalent annual cost? The interest rate is<br />

8% per year.<br />

11.12 A construction company bought a 180,000 metric<br />

ton earth sifter at a cost of $65,000. The company<br />

expects to keep the equipment a maximum of<br />

7 years. The operating cost is expected to follow the<br />

series described by 40,000 10,000 k , where k is<br />

the number of years since it was purchased ( k 1,<br />

2, . . . , 7). The salvage value is estimated to be<br />

$30,000 for years 1 and 2 and $20,000 for years 3<br />

through 7. At an interest rate of 10% per year, determine<br />

the economic service life and the associated<br />

equivalent annual cost of the sifter.<br />

11.13 An engineer determined the ESL of a new<br />

$80,000 piece of equipment and recorded the<br />

calculations shown below. [Note that the numbers<br />

are annual worth values associated with<br />

various years of retention; that is, if the equipment<br />

is kept for, say, 3 years, the AW (years 1<br />

through 3) of the first cost is $32,169, the AW of<br />

the operating cost is $51,000, and the AW of the<br />

salvage value is $6042.] The engineer forgot to<br />

enter the AW of the salvage value for 2 years of<br />

retention. From the information available, determine<br />

the following:<br />

(a) The interest rate used in the ESL calculations.<br />

( b) The salvage value after 2 years, if the total<br />

AW of the equipment in year 2 was<br />

$78,762. Use the interest rate determined<br />

in part ( a ).<br />

Years<br />

Retained<br />

AW of<br />

First Cost, $<br />

AW of<br />

Operating Cost, $<br />

AW of<br />

Salvage Value, $<br />

1 88,000 45,000 50,000<br />

2 46,095 46,000 ?<br />

3 32,169 51,000 6,042<br />

11.14 A large, standby electricity generator in a hospital<br />

operating room has a first cost of $70,000 and may<br />

be used for a maximum of 6 years. Its salvage<br />

value, which decreases by 15% per year, is described<br />

by the equation S 70,000(1 – 0.15) n ,<br />

where n is the number of years after purchase. The<br />

operating cost of the generator will be constant at<br />

$75,000 per year. At an interest rate of 12% per<br />

year, what are the economic service life and the<br />

associated AW value?


Problems 315<br />

11.15 A piece of equipment has a first cost of $150,000, a<br />

maximum useful life of 7 years, and a market (salvage)<br />

value described by the relation S 120,000 <br />

20,000 k , where k is the number of years since it was<br />

purchased. The salvage value cannot go below zero.<br />

The AOC series is estimated using AOC 60,000 <br />

10,000 k . The interest rate is 15% per year. Determine<br />

the economic service life ( a ) by hand solution,<br />

using regular AW computations, and ( b ) by spreadsheet,<br />

using annual marginal cost estimates.<br />

11.16 Determine the economic service life and corresponding<br />

AW value for a machine that has the following<br />

cash flow estimates. Use an interest rate of<br />

14% per year and hand solution.<br />

Year<br />

Salvage<br />

Value, $<br />

Operating<br />

Cost, $ per Year<br />

0 100,000 —<br />

1 75,000 –28,000<br />

2 60,000 –31,000<br />

3 50,000 –34,000<br />

4 40,000 –34,000<br />

5 25,000 –34,000<br />

6 15,000 –45,000<br />

7 0 –49,000<br />

11.17 Use the annual marginal costs to find the economic<br />

service life for Problem 11.16 on a spreadsheet. Assume<br />

the salvage values are the best estimates of future<br />

market value. Develop an Excel chart of annual<br />

marginal costs (MC) and AW of MC over 7 years.<br />

11.18 Keep or Replace the Kiln Case<br />

PE<br />

In Example 11.3 , the market value (salvage value)<br />

series of the proposed $38 million replacement kiln<br />

(GH) dropped to $25 million in only 1 year and then<br />

retained 75% of the previous year’s market value<br />

through the remainder of its 12-year expected life.<br />

Based on the experience with the current kiln, and<br />

the higher-temperature capability of the replacement,<br />

the new kiln is actually expected to retain<br />

only 50% of the previous year’s value starting in<br />

year 5. Additionally, the heating element replacement<br />

in year 6 will probably cost $4 million, not<br />

$2 million. And finally, the maintenance costs will<br />

be considerably higher as the kiln ages. Starting in<br />

year 5, the AOC is expected to increase by 25% per<br />

year, not 10% as predicted earlier. The Manager of<br />

Critical Equipment is now very concerned that the<br />

ESL will be significantly decreased from the<br />

12 years calculated earlier ( Example 11.3 ).<br />

( a) Determine the new ESL and associated AW<br />

value.<br />

( b) In percentage changes, estimate how much<br />

these new cost estimates may affect the<br />

minimum-cost life and AW of cost estimate.<br />

Replacement Study<br />

11.19 In a one-year-later analysis, what action should be<br />

taken if ( a ) all estimates are still current and the<br />

year is n D , ( b ) all estimates are still current and the<br />

year is not n D , and ( c ) the estimates have changed?<br />

11.20 Based on company records of similar equipment, a<br />

consulting aerospace engineer at Aerospatiale estimated<br />

AW values for a presently owned, highly<br />

accurate steel rivet inserter.<br />

If Retained This<br />

Number of Years<br />

AW Value,<br />

$ per Year<br />

1 62,000<br />

2 51,000<br />

3 49,000<br />

4 53,000<br />

5 70,000<br />

A challenger has ESL 2 years and AW C <br />

$48,000 per year. ( a ) If the consultant must recommend<br />

a replaceretain decision today, should<br />

the company keep the defender or purchase the<br />

challenger? Why? The MARR is 15% per year.<br />

( b ) When should the next replacement evaluation<br />

take place?<br />

11.21 In planning a plant expansion, MedImmune has an<br />

economic decision to make—upgrade the existing<br />

controlled-environment rooms or purchase new<br />

ones. The presently owned ones were purchased 4<br />

years ago for $250,000. They have a current “quick<br />

sale” value of $20,000, but for an investment of<br />

$100,000 now, they would be adequate for another<br />

4 years, after which they would be sold for $40,000.<br />

Alternatively, new controlled-environment rooms<br />

could be purchased at a cost of $270,000. They are<br />

expected to have a 10-year life with a $50,000 salvage<br />

value at that time. Determine whether the<br />

company should upgrade or replace. Use a MARR<br />

of 20% per year.<br />

11.22 Three years ago, Witt Gas Controls purchased<br />

equipment for $80,000 that was expected to have a<br />

useful life of 5 years with a $9000 salvage value.<br />

Increased demand necessitated an upgrade costing<br />

$30,000 one year ago. Technology changes now require<br />

that the equipment be upgraded again for another<br />

$25,000 so that it can be used for 3 more<br />

years. Its annual operating cost will be $47,000, and<br />

it will have a $22,000 salvage after 3 years. Alternatively,<br />

it can be replaced with new equipment that<br />

will cost $68,000 with operating costs of $35,000<br />

per year and a salvage value of $21,000 after<br />

3 years. If replaced now, the existing equipment will<br />

be sold for $9000. Calculate the annual worth of the<br />

defender at an interest rate of 10% per year.


316 Chapter 11 Replacement and Retention Decisions<br />

11.23 A recent environmental engineering graduate is trying<br />

to decide whether he should keep his presently<br />

owned car or purchase a more environmentally<br />

friendly hybrid. A new car will cost $26,000 and have<br />

annual operation and maintenance costs of $1200 per<br />

year with an $8000 salvage value in 5 years (which is<br />

its estimated economic service life).<br />

The presently owned car has a resale value now<br />

of $5000; one year from now it will be $3000, two<br />

years from now $2500, and 3 years from now<br />

$2200. Its operating cost is expected to be $1900<br />

this year, with costs increasing by $200 per year.<br />

The presently owned car will definitely not be kept<br />

longer than 3 more years. Assuming used cars like<br />

the one presently owned will always be available,<br />

should the presently owned car be sold now, 1 year<br />

from now, 2 years from now, or 3 years from now?<br />

Use annual worth calculations at i 10% per year<br />

and show your work.<br />

11.24 A pulp and paper company is evaluating whether it<br />

should retain the current bleaching process that<br />

uses chlorine dioxide or replace it with a proprietary<br />

“oxypure” process. The relevant information<br />

for each process is shown. Use an interest rate of<br />

15% per year to perform the replacement study.<br />

Current Process<br />

Oxypure Process<br />

Original cost 6 450,000 —<br />

years ago, $<br />

Investment cost<br />

— 700,000<br />

now, $<br />

Current market<br />

25,000 —<br />

value, $<br />

Annual operating 180,000 70,000<br />

cost, $year<br />

Salvage value, $ 0 50,000<br />

Remaining life,<br />

years<br />

5 10<br />

11.25 A machine that is critical to the Phelps-Dodge copper<br />

refining operation was purchased 7 years ago<br />

for $160,000. Last year a replacement study was<br />

performed with the decision to retain it for 3 more<br />

years. The situation has changed. The equipment is<br />

estimated to have a value of $8000 if “scavenged”<br />

for parts now or anytime in the future. If kept in<br />

service, it can be minimally upgraded at a cost of<br />

$43,000, which will make it usable for up to<br />

2 more years. Its operating cost is expected to be<br />

$22,000 the first year and $25,000 the second year.<br />

Alternatively, the company can purchase a new<br />

system that will have an equivalent annual worth<br />

of $47,063 per year over its ESL. The company<br />

uses a MARR of 10% per year. Calculate the relevant<br />

annual worth values, and determine when the<br />

company should replace the machine.<br />

11.26 A crushing machine that is a basic component of a<br />

metal recycling operation is wearing out faster than<br />

expected. The machine was purchased 2 years ago<br />

for $400,000. At that time, the buyer thought the<br />

machine would serve its needs for at least 5 years,<br />

at which time the machine would be sold to a<br />

smaller independent recycler for $80,000. Now,<br />

however, the company thinks the market value of<br />

the diminished machine is only $50,000. If it is<br />

kept, the machine’s operating cost will be $37,000<br />

per year for the next 2 years, after which it will be<br />

scrapped for $1000. If it is kept for only 1 year, the<br />

market value is estimated to be $10,000. Alternatively,<br />

the company can outsource the process now<br />

for a fixed cost of $56,000 per year. At an interest<br />

rate of 10% per year, should the company continue<br />

with the current machine or outsource the process?<br />

11.27 The data associated with operating and maintaining<br />

an asset are shown below. The company manager<br />

has already decided to keep the machine for 1<br />

more year (i.e., until the end of year 1), but you<br />

have been asked to determine the cost of keeping it<br />

1 more year after that. At an interest rate of 10%<br />

per year, estimate the AW of keeping the machine<br />

from year 1 to year 2.<br />

Year Market Value, $<br />

Operating Cost,<br />

$ per Year<br />

0 30,000 15,000<br />

1 25,000 15,000<br />

2 14,000 15,000<br />

3 10,000 15,000<br />

11.28 A machine that cost $120,000 three years ago can<br />

be sold now for $54,000. Its market value for the<br />

next 2 years is expected to be $40,000 and $20,000<br />

one year and 2 years from now, respectively. Its<br />

operating cost was $18,000 for the first 3 years of<br />

its life, but the M&O cost is expected to be<br />

$23,000 for the next 2 years. A new improved machine<br />

that can be purchased for $138,000 will<br />

have an economic life of 5 years, an operating<br />

cost of $9000 per year, and a salvage value of<br />

$32,000 after 5 years. At an interest rate of 10%<br />

per year, determine if the presently owned machine<br />

should be replaced now, 1 year from now, or<br />

2 years from now.<br />

11.29 The projected market value and M&O costs associated<br />

with a presently owned machine are shown<br />

(next page). An outside vendor of services has offered<br />

to provide the service of the existing machine<br />

at a fixed price per year. If the presently<br />

owned machine is replaced now, the cost of the<br />

fixed-price contract will be $33,000 for each of the<br />

next 3 years. If the presently owned machine is


Problems 317<br />

replaced next year or the year after that, the contract<br />

price will be $35,000 per year. Determine if<br />

and when the defender should be replaced with the<br />

outside vendor using an interest rate of 10% per<br />

year. Assume used equipment similar to the defender<br />

will always be available.<br />

Year Market Value, $<br />

M&O Cost,<br />

$ per Year<br />

0 30,000 —<br />

1 25,000 24,000<br />

2 14,000 25,000<br />

3 10,000 26,000<br />

4 8,000 —<br />

11.30 BioHealth, a biodevice systems leasing company,<br />

is considering a new equipment purchase to replace<br />

a currently owned asset that was purchased<br />

2 years ago for $250,000. It is appraised at a current<br />

market value of only $50,000. An upgrade is<br />

possible for $200,000 now that would be adequate<br />

for another 3 years of lease rights, after which the<br />

entire system could be sold on the international<br />

circuit for an estimated $40,000. The challenger,<br />

which can be purchased for $300,000, has an expected<br />

life of 10 years and a $50,000 salvage<br />

value. Determine whether the company should upgrade<br />

or replace at a MARR of 12% per year. Assume<br />

the AOC estimates are the same for both<br />

alternatives.<br />

11.31 For the estimates in Problem 11.30, use a<br />

spreadsheet-based analysis to determine the first<br />

cost for the augmentation of the current system<br />

that will make the defender and challenger break<br />

even. Is this a maximum or minimum for the upgrade,<br />

if the current system is to be retained?<br />

11.32 Herald Richter and Associates, 5 years ago, purchased<br />

for $45,000 a microwave signal graphical<br />

plotter for corrosion detection in concrete structures.<br />

It is expected to have the market values and<br />

annual operating costs shown below for its remaining<br />

useful life of up to 3 years. It could be<br />

traded now at an appraised market value of $8000.<br />

Year<br />

Market Value<br />

at End of Year, $<br />

AOC,<br />

$ per Year<br />

1 6000 –50,000<br />

2 4000 –53,000<br />

3 1000 –60,000<br />

A replacement plotter with new Internet-based,<br />

digital technology costing $125,000 has an estimated<br />

$10,000 salvage value after its 5-year life<br />

and an AOC of $31,000 per year. At an interest rate<br />

of 15% per year, determine how many more years<br />

Richter should retain the present plotter. Solve<br />

( a ) by hand and ( b ) by using a spreadsheet.<br />

11.33 In the opportunity cost approach to replacement<br />

analysis, what does the opportunity refer to?<br />

11.34 State what is meant by the cash fl ow approach to<br />

replacement analysis, and list two reasons why it is<br />

not a good idea to use this method.<br />

Replacement Study over a Specified Study Period<br />

11.35 ABB Communications is considering replacing<br />

equipment that had a first cost of $300,000 five<br />

years ago. The company CEO wants to know if the<br />

equipment should be replaced now or at any other<br />

time over the next 3 years to minimize the cost of<br />

producing miniature background suppression sensors.<br />

Since the present equipment or the proposed<br />

equipment can be used for any or all of the 3-year<br />

period, one of the company’s industrial engineers<br />

produced AW cost information for the defender<br />

and challenger as shown below. The values represent<br />

the annual costs of the respective equipment if<br />

used for the indicated number of years. Determine<br />

when the defender should be replaced to minimize<br />

the cost to ABB for the 3-year study period using<br />

an interest rate of 10% per year.<br />

Number of<br />

Years Kept<br />

AW If Kept Stated<br />

Number of Years, $ per Year<br />

Defender<br />

Challenger<br />

1 22,000 29,000<br />

2 24,000 26,000<br />

3 27,000 25,000<br />

11.36 The table below shows present worth calculations<br />

of the costs associated with using a presently<br />

owned machine (defender) and a possible replacement<br />

(challenger) for different numbers of years.<br />

Determine when the defender should be replaced<br />

using an interest rate of 10% per year and a 5-year<br />

study period. Show solutions ( a ) by hand and<br />

( b ) by spreadsheet.<br />

Number of Years<br />

KeptUsed<br />

PW If KeptUsed Stated<br />

Number of Years, $<br />

Defender<br />

Challenger<br />

1 36,000 89,000<br />

2 75,000 96,000<br />

3 125,000 102,000<br />

4 166,000 113,000<br />

5 217,000 149,000<br />

11.37 Nano Technologies intends to use the newest and<br />

finest equipment in its labs. Accordingly, a senior<br />

engineer has recommended that a 2-year-old piece


318 Chapter 11 Replacement and Retention Decisions<br />

of precision measurement equipment be replaced<br />

immediately. This engineer believes it can be<br />

shown that the proposed equipment is economically<br />

advantageous at a 10% per year return and a<br />

planning horizon of 3 years.<br />

(a) Perform the replacement analysis using the<br />

annual worth method for a specified 3-year<br />

study period.<br />

(b) Determine the challenger’s capital recovery<br />

amount for the 3-year study period and the<br />

expected full life. Comment on the effect<br />

made by the 3-year study period.<br />

Current<br />

Proposed<br />

Original purchase price, $ 30,000 40,000<br />

Current market value, $ 17,000 —<br />

Remaining life, years 5 15<br />

Estimated value in 3 years, $ 9,000 20,000<br />

Salvage value after 15 years, $ — 5,000<br />

Annual operating cost, $ per year 8,000 3,000<br />

11.38 An industrial engineer at a fiber-optic manufacturing<br />

company is considering two robots to reduce<br />

costs in a production line. Robot X will have a first<br />

cost of $82,000, an annual maintenance and operation<br />

(M&O) cost of $30,000, and salvage values of<br />

$50,000, $42,000, and $35,000 after 1, 2, and<br />

3 years, respectively. Robot Y will have a first cost<br />

of $97,000, an annual M&O cost of $27,000, and<br />

salvage values of $60,000, $51,000, and $42,000<br />

after 1, 2, and 3 years, respectively. Which robot<br />

should be selected if a 2-year study period is specified<br />

at an interest rate of 15% per year and replacement<br />

after 1 year is not an option?<br />

11.39 A 3-year-old machine purchased for $140,000 is not<br />

able to meet today’s market demands. The machine<br />

can be upgraded now for $70,000 or sold to a subcontracting<br />

company for $40,000. The current machine<br />

will have an annual operating cost of $85,000<br />

per year and a $30,000 salvage value in 3 years. If<br />

upgraded, the presently owned machine will be retained<br />

for only 3 more years, then replaced with a<br />

machine to be used in the manufacture of several<br />

other product lines. The replacement machine,<br />

which will serve the company now and for at least<br />

8 years, will cost $220,000. Its salvage value will be<br />

$50,000 for years 1 through 4; $20,000 after 5 years;<br />

and $10,000 thereafter. It will have an estimated operating<br />

cost of $65,000 per year. You want to perform<br />

an economic analysis at 15% per year using a<br />

3-year planning horizon.<br />

(a) Should the company replace the presently<br />

owned machine now, or do it 3 years from now?<br />

(b) Compare the capital recovery requirements for<br />

the replacement machine (challenger) over the<br />

study period and an expected life of 8 years.<br />

11.40 Two processes can be used for producing a polymer<br />

that reduces friction loss in engines. Process<br />

K, which is currently in place, has a market value<br />

of $165,000 now, an operating cost of $69,000 per<br />

year, and a salvage value of $50,000 after 1 more<br />

year and $40,000 after its maximum 2-year remaining<br />

life. Process L, the challenger, will have a<br />

first cost of $230,000, an operating cost of $65,000<br />

per year, and salvage values of $100,000 after 1<br />

year, $70,000 after 2 years, $45,000 after 3 years,<br />

and $26,000 after its maximum expected 4-year<br />

life. The company’s MARR is 12% per year. You<br />

have been asked to determine which process to select<br />

when ( a ) a 2-year study period is used and<br />

( b ) a 3-year study period is used.<br />

11.41 Keep or Replace the Kiln Case<br />

In Example 11.8 , the in-place kiln and replacement<br />

kiln (GH) were evaluated using a fixed study period<br />

of 6 years. This is a significantly shortened<br />

period compared to the expected 12-year life of<br />

the challenger. Use the best estimates available<br />

throughout this case to determine the impact on the<br />

capital recovery amount for the GH kiln of shortening<br />

the evaluation time from 12 to 6 years.<br />

11.42 Nabisco Bakers currently employs staff to operate<br />

the equipment used to sterilize much of the mixing,<br />

baking, and packaging facilities in a large cookie<br />

and cracker manufacturing plant in Iowa. The plant<br />

manager, who is dedicated to cutting costs but not<br />

sacrificing quality and hygiene, has the projected<br />

data shown in the table below if the current system<br />

were retained for up to its maximum expected life<br />

of 5 years. A contract company has proposed a<br />

turnkey sanitation system for $5.0 million per year<br />

if Nabisco signs on for 4 to 10 years, and $5.5 million<br />

per year for a shorter number of years.<br />

Years Retained AW, $ per Year Close-Down Expense, $<br />

0 — 3,000,000<br />

1 2,300,000 2,500,000<br />

2 2,300,000 2,000,000<br />

3 3,000,000 1,000,000<br />

4 3,000,000 1,000,000<br />

5 3,500,000 500,000<br />

(a)<br />

PE<br />

At a MARR 8% per year, perform a replacement<br />

study for the plant manager with a<br />

fixed study period of 5 years, when it is anticipated<br />

that the plant will be shut down due<br />

to the age of the facility and projected technological<br />

obsolescence. As you perform the<br />

study, take into account that regardless of the<br />

number of years that the current sanitation<br />

system is retained, a one-time close-down


Additional Problems and FE Exam Review Questions 319<br />

cost will be incurred for personnel and equipment<br />

during the last year of operation. ( Hint:<br />

Calculate AW values for all combinations of<br />

defenderchallenger options.)<br />

( b) What is the percentage change in the AW<br />

amount each year for the 5-year period? If<br />

the decision to retain the current sanitation<br />

system for all 5 years is made, what is the<br />

economic disadvantage in AW compared to<br />

that of the most economic retention period?<br />

Replacement Value<br />

11.43 In 2008, Amphenol Industrial purchased a new<br />

quality inspection system for $550,000. The estimated<br />

salvage value was $50,000 after 8 years.<br />

Currently the expected remaining life is 3 years<br />

with an AOC of $27,000 per year and an estimated<br />

salvage value of $30,000. The new president has<br />

recommended early replacement of the system<br />

with one that costs $400,000 and has a 5-year<br />

economic service life, a $45,000 salvage value,<br />

and an estimated AOC of $50,000 per year. If the<br />

MARR for the corporation is 12% per year, find<br />

the minimum trade-in value now necessary to<br />

make the president’s replacement economically<br />

advantageous.<br />

11.44 A CNC milling machine purchased by Proto Tool<br />

and Die 10 years ago for $75,000 can be used for 3<br />

more years. Estimates are an annual operating cost<br />

of $63,000 and a salvage value of $25,000. A challenger<br />

will cost $130,000 with an economic life of<br />

6 years and an operating cost of $32,000 per year.<br />

Its salvage value will be $45,000. On the basis of<br />

these estimates, what market value for the existing<br />

asset will render the challenger equally attractive?<br />

Use an interest rate of 12% per year.<br />

11.45 Hydrochloric acid, which fumes at room temperatures,<br />

creates a very corrosive work environment,<br />

causing steel tools to rust and equipment to fail<br />

prematurely. A distillation system purchased 4<br />

years ago for $80,000 can be used for only 2 more<br />

years, at which time it will be scrapped with no<br />

value. Its operating cost is $75,000 per year. A<br />

more corrosion-resistant challenger will cost<br />

$220,000 with an operating cost of $49,000 per<br />

year. It is expected to have a $30,000 salvage<br />

value after its 6-year ESL. At an interest rate of<br />

10% per year, what minimum replacement value<br />

now for the present system will render the challenger<br />

attractive?<br />

11.46 Engine oil purifier machines can effectively remove<br />

acid, pitch, particles, water, and gas from<br />

used oil. Purifier A was purchased 5 years ago for<br />

$90,000. Its operating cost is higher than expected,<br />

so if it is not replaced now, it will likely be used for<br />

only 3 more years. Its operating cost this year will<br />

be $140,000, increasing by $2000 per year through<br />

the end of its useful life, at which time it will be<br />

donated for its recyclable scrap value. A more efficient<br />

challenger, purifier B, will cost $150,000<br />

with a $50,000 salvage value after its 8-year ESL.<br />

Its operating cost is expected to be $82,000 for<br />

year 1, increasing by $500 per year thereafter.<br />

What is the market value for A that will make the<br />

two purifiers equally attractive at an interest rate of<br />

12% per year?<br />

ADDITIONAL PROBLEMS AND FE EXAM REVIEW QUESTIONS<br />

11.47 In conducting a replacement study, all of the following<br />

are correct viewpoints for the analyst except:<br />

( a) Consultant’s<br />

( b) Owner’s<br />

( c) Outsider’s<br />

( d) Nonowner’s<br />

11.48 A sunk cost is the difference between:<br />

( a) The first cost and the salvage value<br />

( b) The market value and the salvage value<br />

( c) The first cost and the market value<br />

( d) The book value and the market value<br />

11.49 A truck was purchased 3 years ago for $45,000 and<br />

can be sold today for $24,000. The operating costs<br />

are $9000 per year, and it is expected to last 4 more<br />

years with a $5000 salvage value. A new truck,<br />

which will perform that same service, can be purchased<br />

for $50,000, and it will have a life of<br />

10 years with operating costs of $28,000 per year<br />

and a $10,000 salvage value. The value that should<br />

be used as P for the presently owned vehicle in a<br />

replacement study is:<br />

(a) $45,000<br />

(b) $5000<br />

(c) $50,000<br />

(d) $24,000<br />

11.50 The economic service life of an asset with the PW<br />

and AW values on the next page is:<br />

(a) 1 year<br />

(b) 2 years<br />

(c) 3 years<br />

(d) 4 years


320 Chapter 11 Replacement and Retention Decisions<br />

Years n<br />

Present Worth If<br />

Kept n Years, $<br />

Annual Worth If<br />

Kept n Years,<br />

$Year<br />

1 15,455 17,000<br />

2 28,646 16,286<br />

3 45,019 18,102<br />

4 57,655 18,188<br />

5 72,867 19,222<br />

11.51 In looking for ways to cut costs and increase<br />

profit (to make the company’s stock go up), one<br />

of the company’s industrial engineers (IEs) determined<br />

that the equivalent annual worth of an<br />

existing machine over its remaining useful life of<br />

3 years will be $–70,000 per year. The IE also<br />

determined that a replacement with more advanced<br />

features will have an AW of $–80,000 per<br />

year if it is kept for 2 years or less, $–75,000 if it<br />

is kept between 3 and 4 years, and $–65,000 if it<br />

is kept for 5 to 10 years. If the engineer uses a<br />

3-year study period and an interest rate of 15%<br />

per year, she should recommend that the existing<br />

machine be:<br />

(a) Replaced now<br />

(b) Replaced 1 year from now<br />

(c) Replaced 2 years from now<br />

(d) Not replaced<br />

11.52 The equivalent annual worth of an existing machine<br />

at American Semiconductor is estimated to<br />

be $–70,000 per year over its remaining useful life<br />

of 3 years. It can be replaced now or later with a<br />

machine that will have an AW of $–90,000 per<br />

year if it is kept for 2 years or less, –$65,000 if it<br />

is kept between 3 and 5 years, and $–110,000 if it<br />

is kept for 6 to 8 years. The company wants an<br />

analysis of what it should do for a 3-year study<br />

period at an interest rate of 15% per year. The replacement<br />

must be made now or 3 years from<br />

now, according to the department supervisor. You<br />

should recommend that the existing machine be<br />

replaced:<br />

(a) Now<br />

(b) 1 year from now<br />

(c) 2 years from now<br />

(d) Can’t tell; must find AW of different retention<br />

combinations for the two machines<br />

11.53 The cost characteristics of a CO testing machine at<br />

Dytran Instruments are shown below. The cost of a<br />

new tester is $100,000. The equation for determining<br />

the AW of keeping the tester for 2 years is:<br />

(a) AW 100,000( AP,i,8) [42,000( PF,i,1)<br />

47,000( PF,i ,2)](AP,i,8) 40,000( AF,i,8)<br />

(b) AW 100,000( AP,i ,2) [42,000( PF,i,1)<br />

47,000( PF,i,2)](AP,i ,2) 40,000( AF,i ,2)<br />

( c) AW 100,000( AP,i, 2) 47,000<br />

40,000( AF,i ,2)<br />

(d) AW 100,000( AP,i, 2) 42,000<br />

40,000( AF,i, 2)<br />

Machine Age,<br />

Years<br />

M&O Costs,<br />

$ per Year<br />

Salvage Value<br />

at End of Year, $<br />

1 42,000 60,000<br />

2 47,000 40,000<br />

3 49,000 31,000<br />

4 50,000 24,000<br />

5 52,000 15,000<br />

6 54,000 10,000<br />

7 63,000 10,000<br />

8 67,000 10,000<br />

11.54 An engineer with Haliburton calculated the AW<br />

values shown for a presently owned machine by<br />

using estimates he obtained from the vendor and<br />

company records.<br />

Retention<br />

Period, Years<br />

AW Value,<br />

$ per Year<br />

1 92,000<br />

2 81,000<br />

3 87,000<br />

4 89,000<br />

5 95,000<br />

A challenger has an economic service life of<br />

7 years with an AW of $–86,000 per year. Assume<br />

that used machines like the one presently owned<br />

will always be available and that the MARR is<br />

12% per year. If all future costs remain as estimated<br />

for the analysis, the company should purchase<br />

the challenger:<br />

(a) Now<br />

(b) After 2 years<br />

(c) After 3 years<br />

(d) Never<br />

11.55 The annual worth values for a defender, which can<br />

be replaced with a similar used asset, and a challenger<br />

are estimated. The defender should be replaced:<br />

(a) Now<br />

(b) 1 year from now<br />

(c) 2 years from now<br />

(d) 3 years from now<br />

Number of<br />

Years Retained<br />

AW Value, $ per Year<br />

Defender<br />

Challenger<br />

1 14,000 21,000<br />

2 13,700 18,000<br />

3 16,900 13,100<br />

4 17,000 15,600<br />

5 18,000 17,500


Case Study 321<br />

CASE STUDY<br />

WILL THE CORRECT ESL PLEASE STAND?<br />

Background<br />

New pumper system equipment is under consideration by a<br />

Gulf Coast chemical processing plant. One crucial pump<br />

moves highly corrosive liquids from specially lined tanks on<br />

intercoastal barges into storage and preliminary refining facilities<br />

dockside. Because of the variable quality of the raw<br />

chemical and the high pressures imposed on the pump chassis<br />

and impellers, a close log is maintained on the number of<br />

hours per year that the pump operates. Safety records and<br />

pump component deterioration are considered critical control<br />

points for this system. As currently planned, rebuild and<br />

M&O cost estimates are increased accordingly when cumulative<br />

operating time reaches the 6000-hour mark.<br />

Information<br />

You are the safety engineer at the plant. Estimates made for<br />

this pump are as follows:<br />

First cost:<br />

Rebuild cost:<br />

$800,000<br />

$150,000 whenever 6000 cumulative<br />

hours are logged. Each<br />

rework will cost 20% more than<br />

the previous one. A maximum of<br />

3 rebuilds is allowed.<br />

M&O costs: $25,000 for each year 1 through 4<br />

$40,000 per year starting the<br />

year after the first rebuild, plus<br />

15% per year thereafter<br />

MARR: 10% per year<br />

Based on previous logbook data, the current estimates for<br />

number of operating hours per year are as follows:<br />

Year<br />

Hours per Year<br />

1 500<br />

2 1500<br />

3 on 2000<br />

Case Study Questions<br />

1. Determine the economic service life of the pump. How<br />

does the ESL compare with the maximum allowed<br />

rebuilds?<br />

2. The plant superintendent told you, the safety engineer,<br />

that only one rebuild should be planned for, because<br />

these types of pumps usually have their minimum-cost<br />

life before the second rebuild. Determine a market<br />

value for this pump that will force the ESL to be 6 years.<br />

Comment on the practicality of ESL 6 years, given<br />

the MV calculated.<br />

3. In a separate conversation, the line manager told you<br />

to not plan for a rebuild after 6000 hours, because<br />

the pump will be replaced after a total of 10,000<br />

hours of operation. The line manager wants to know<br />

what the base AOC in year 1 can be to make the ESL<br />

6 years. He also told you to assume now that the 15%<br />

growth rate applies from year 1 forward. How does<br />

this base AOC value compare with the rebuild cost<br />

after 6000 hours?<br />

4. What do you think of these suggestions from the plant<br />

superintendent and the line manager?


CHAPTER 12<br />

Independent<br />

Projects with<br />

Budget<br />

Limitation<br />

LEARNING OUTCOMES<br />

Purpose: Select independent projects for funding when there is a limitation on the amount of capital available for investment.<br />

SECTION TOPIC LEARNING OUTCOME<br />

12.1 Capital rationing • Explain how a capital budgeting problem is<br />

approached.<br />

12.2 Equal-life projects • Use PW-based capital budgeting to select from<br />

several equal-life independent projects.<br />

12.3 Unequal-life projects • Use PW-based capital budgeting to select from<br />

several unequal-life independent projects.<br />

12.4 Linear programming • Set up the linear programming model and use the<br />

Solver spreadsheet tool to select projects.<br />

12.5 Ranking options • Use the internal rate of return (IROR) and<br />

profitability index (PI) to rank and select from<br />

independent projects.


I<br />

n most of the previous economic comparisons, the alternatives have been mutually<br />

exclusive; only one could be selected. If the projects are not mutually exclusive,<br />

they are categorized as independent of one another, as discussed at the beginning<br />

of Chapter 5. Now we learn techniques to select from several independent projects. It<br />

is possible to select any number of projects from none (do nothing) to all viable projects.<br />

There is virtually always some upper limit on the amount of capital available for investment<br />

in new projects. This limit is considered as each independent project is economically<br />

evaluated. The techniques applied are called capital budgeting methods, also referred to<br />

as capital rationing. They determine the economically best rationing of initial investment<br />

capital among independent projects based upon different measures, such as PW, ROR, and<br />

the profitability index. These three are discussed here.<br />

12.1 An Overview of Capital Rationing<br />

among Projects<br />

Investment capital is a scarce resource for all corporations; thus there is virtually always a limited<br />

amount to be distributed among competing investment opportunities. When a corporation has<br />

several options for placing investment capital, a “reject or accept” decision must be made for<br />

each project. Effectively, each option is independent of other options, so the evaluation is performed<br />

on a project-by-project basis. Selection of one project does not impact the selection decision<br />

for any other project. This is the fundamental difference between mutually exclusive alternatives<br />

and independent projects.<br />

The term project is used to identify each independent option. We use the term bundle to<br />

identify a collection of independent projects. The term mutually exclusive alternative continues<br />

to identify a project when only one may be selected from several.<br />

There are two exceptions to purely independent projects: A contingent project is one that has<br />

a condition placed upon its acceptance or rejection. Two examples of contingent projects A and<br />

B are as follows: A cannot be accepted unless B is accepted; and A can be accepted in lieu of B,<br />

but both are not needed. A dependent project is one that must be accepted or rejected based on<br />

the decision about another project(s). For example, B must be accepted if both A and C are accepted.<br />

In practice, these complicating conditions can be bypassed by forming packages of related<br />

projects that are economically evaluated themselves as independent projects along with the<br />

remaining, unconditioned projects.<br />

A capital budgeting study has the following characteristics:<br />

• Several independent projects are identified, and net cash flow estimates are available.<br />

• Each project is either selected entirely or not selected; that is, partial investment in a project<br />

is not possible.<br />

• A stated budgetary constraint restricts the total amount available for investment. Budget<br />

constraints may be present for the first year only or for several years. This investment limit<br />

is identified by the symbol b .<br />

• The objective is to maximize the return on the investments using a measure of worth, such<br />

as the PW value.<br />

Independent p roject<br />

selection<br />

Limited budget<br />

By nature, independent projects are usually quite different from one another. For example, in the<br />

public sector, a city government may develop several projects to choose from: drainage, city<br />

park, metro rail, and an upgraded public bus system. In the private sector, sample projects may<br />

be a new warehousing facility, expanded product base, improved quality program, upgraded information<br />

system, automation, and acquisition of another firm. The size of the study can range<br />

from only four or five projects to a complex study involving 50 to 100 projects. The typical<br />

capital budgeting problem is illustrated in Figure 12–1. For each independent project there is an<br />

initial investment, project life, and estimated net cash flows that can include a salvage value.<br />

Present worth analysis using the capital budgeting process is the recommended method to<br />

select projects. The general selection guideline is as follows:<br />

Accept projects with the best PW values determined at the MARR over the project life, provided<br />

the investment capital limit is not exceeded.


324 Chapter 12 Independent Projects with Budget Limitation<br />

Figure 12–1<br />

Basic characteristics of a<br />

capital budgeting study.<br />

Independent<br />

projects<br />

Initial<br />

investment<br />

Estimates<br />

Estimated<br />

net cash flows<br />

A.<br />

Life<br />

Capital<br />

investment<br />

limit<br />

Investment<br />

B.<br />

Life<br />

(can’t invest<br />

more than<br />

this much)<br />

C.<br />

Life<br />

Select 0 to all 3 projects<br />

Objective: Maximize PW<br />

value of selection<br />

within capital limit<br />

Equal-service requirement<br />

This guideline is not different from that used for selection in previous chapters for independent<br />

projects. As before, each project is compared with the do-nothing project; that is, incremental<br />

analysis between projects is not necessary. The primary difference now is that the amount of<br />

money available to invest is limited, thus the title capital budgeting or rationing . A specific solution<br />

procedure that incorporates this constraint is needed.<br />

Previously, PW analysis had the requirement of equal service between alternatives. This assumption<br />

is not necessary for capital rationing, because there is no life cycle of a project beyond<br />

its estimated life. Rather, the selection guideline has the following implied assumption.<br />

When the present worth at the MARR over the respective project life is used to select projects,<br />

the reinvestment assumption is that all positive net cash flows are reinvested at the MARR from<br />

the time they are realized until the end of the longest-lived project.<br />

Opportunity cost<br />

This fundamental assumption is demonstrated to be correct at the end of Section 12.3, which<br />

treats PW-based capital rationing for unequal-life projects.<br />

Another dilemma of capital rationing among independent projects concerns the flexibility of the<br />

capital investment limit b . The limit may marginally disallow an acceptable project that is next in<br />

line for acceptance. For example, assume project A has a positive PW value at the MARR. If A will<br />

cause the capital limit of $5,000,000 to be exceeded by only $1000, should A be included in the PW<br />

analysis? Commonly, a capital investment limit is somewhat flexible, so project A would usually be<br />

included. However, in the examples here, we will not exceed a stated investment limit.<br />

As we learned earlier (Sections 1.9 and 10.1), the rate of return on the first unfunded project<br />

is an opportunity cost . The lack of capital to fund the next project defines the ROR level that<br />

is forgone. The opportunity cost will vary with each set of independent projects evaluated, but<br />

over time it provides information to fine-tune the MARR and other measures used by the<br />

company in future evaluations.


12.2 Capital Rationing Using PW Analysis of Equal-Life Projects 325<br />

It is common to rank independent projects and select from them based on measures other than<br />

PW at the MARR. Two are the internal rate of return (IROR), discussed in Chapter 7, and the<br />

profi tability index (PI) , also called the present worth index (PWI), introduced in Chapter 9. Neither<br />

of these measures guarantees an optimal PW-based selection. The capital budgeting process,<br />

covered in the next 3 sections, does find the optimal solution for PW values. We recommend use<br />

of this PW-based technique; however, it should be recognized that both PI and IROR usually<br />

provide excellent, near-optimal selections, and they both work very well when the number of<br />

projects is large. Application of these two measures is presented in Section 12.5.<br />

12.2 Capital Rationing Using PW Analysis<br />

of Equal-Life Projects<br />

To select from projects that have the same expected life while investing no more than the limit b ,<br />

first formulate all mutually exclusive bundles —one project at a time, two at a time, etc. Each<br />

feasible bundle must have a total investment that does not exceed b . One of these bundles is the<br />

do-nothing (DN) project. The total number of bundles for m projects is 2 m . The number increases<br />

rapidly with m . For m 4, there are 2 4 16 bundles, and for m 16, 2 16 65,536 bundles.<br />

Then the PW of each bundle is determined at the MARR. The bundle with the largest PW value<br />

is selected.<br />

To illustrate the development of mutually exclusive bundles, consider these four projects with<br />

equal lives.<br />

Project Initial Investment, $<br />

A 10,000<br />

B 5,000<br />

C 8,000<br />

D 15,000<br />

If the investment limit is b $25,000, of the 16 bundles, there are 12 feasible ones to evaluate.<br />

The bundles ABD, ACD, BCD, and ABCD have investment totals that exceed $25,000. The<br />

viable bundles are shown below.<br />

Projects<br />

Total Initial<br />

Investment, $<br />

Projects<br />

Total Initial<br />

Investment, $<br />

A 10,000 AD 25,000<br />

B 5,000 BC 13,000<br />

C 8,000 BD 20,000<br />

D 15,000 CD 23,000<br />

AB 15,000 ABC 23,000<br />

AC 18,000 Do nothing 0<br />

The procedure to conduct a capital budgeting study using PW analysis is as follows:<br />

1. Develop all mutually exclusive bundles with a total initial investment that does not exceed<br />

the capital limit b .<br />

2. Sum the net cash flows NCF jt for all projects in each bundle j ( j 1, 2, . . . , 2 m ) and each<br />

year t ( t 1, 2, . . . , n j ). Refer to the initial investment of bundle j at time t 0 as NCF j 0 .<br />

3. Compute the present worth value PW j for each bundle at the MARR.<br />

PW j PW of bundle net cash flows initial investment<br />

tn j<br />

NCF jt (PF,i,t) NCF j0 [12.1]<br />

t1<br />

4. Select the bundle with the (numerically) largest PW j value.<br />

Selecting the maximum PW j means that this bundle has a PW value larger than any other bundle.<br />

Any bundle with PW j 0 is discarded, because it does not produce a return of at least the MARR.


326 Chapter 12 Independent Projects with Budget Limitation<br />

EXAMPLE 12.1<br />

The projects review committee of Microsoft has $20 million to allocate next year to new software<br />

product development. Any or all of five projects in Table 12–1 may be accepted. All<br />

amounts are in $1000 units. Each project has an expected life of 9 years. Select the project(s)<br />

if a 15% return is expected.<br />

TABLE 12–1 Five Equal-Life Independent Projects ($1000 Units)<br />

Project<br />

Initial<br />

Investment, $<br />

Annual<br />

Net Cash Flow, $<br />

Project<br />

Life, Years<br />

A 10,000 2870 9<br />

B 15,000 2930 9<br />

C 8,000 2680 9<br />

D 6,000 2540 9<br />

E 21,000 9500 9<br />

Solution<br />

Use the procedure above with b $20,000 to select one bundle that maximizes present worth.<br />

Remember the units are in $1000.<br />

5<br />

1. There are 2 32 possible bundles. The eight bundles that require no more than $20,000<br />

in initial investments are described in columns 2 and 3 of Table 12–2. The $21,000 investment<br />

for E eliminates it from all bundles.<br />

TABLE 12–2 Summary of Present Worth Analysis of Equal-Life Independent<br />

Projects ($1000 Units)<br />

Bundle<br />

j<br />

(1)<br />

Projects<br />

Included<br />

(2)<br />

Initial<br />

Investment<br />

NCF j0 , $<br />

(3)<br />

Annual Net<br />

Cash Flow<br />

NCF j , $<br />

(4)<br />

Present<br />

Worth<br />

PW j , $<br />

(5)<br />

1 A 10,000 2,870 3,694<br />

2 B 15,000 2,930 1,019<br />

3 C 8,000 2,680 4,788<br />

4 D 6,000 2,540 6,120<br />

5 AC 18,000 5,550 8,482<br />

6 AD 16,000 5,410 9,814<br />

7 CD 14,000 5,220 10,908<br />

8 Do nothing 0 0 0<br />

2. The bundle net cash flows, column 4, are the sum of individual project net cash<br />

flows.<br />

3. Use Equation [12.1] to compute the present worth for each bundle. Since the annual NCF<br />

and life estimates are the same for a bundle, PW j reduces to<br />

PW j NCF j ( P A ,15%,9) NCF j 0<br />

4. Column 5 of Table 12–2 summarizes the PW j values at i 15%. Bundle 2 does not return<br />

15%, since PW 2 0. The largest is PW 7 $10,908; therefore, invest $14 million in C and<br />

D. This leaves $6 million uncommitted.<br />

This analysis assumes that the $6 million not used in this initial investment will return the<br />

MARR by placing it in some other, unspecified investment opportunity.


12.3 Capital Rationing Using PW Analysis of Unequal-Life Projects 327<br />

12.3 Capital Rationing Using PW Analysis<br />

of Unequal-Life Projects<br />

Usually independent projects do not have the same expected life. As stated in Section 12.1, the<br />

PW method for solution of the capital budgeting problem assumes that each project will last for<br />

the period of the longest-lived project n L . Additionally, reinvestment of any positive net cash<br />

flows is assumed to be at the MARR from the time they are realized until the end of the longestlived<br />

project, that is, from year n j through year n L . Therefore, use of the LCM of lives is not<br />

necessary , and it is correct to use Equation [12.1] to select bundles of unequal-life projects by<br />

PW analysis using the procedure of the previous section.<br />

EXAMPLE 12.2<br />

For MARR 15% per year and b $20,000, select from the following independent projects.<br />

Solve by hand and by spreadsheet.<br />

Project<br />

Initial<br />

Investment, $<br />

Annual Net<br />

Cash Flow, $<br />

Project<br />

Life, Years<br />

A 8,000 3870 6<br />

B 15,000 2930 9<br />

C 8,000 2680 5<br />

D 8,000 2540 4<br />

Solution by Hand<br />

The unequal-life values make the net cash flows vary over a bundle’s life, but the selection<br />

procedure is the same as above. Of the 2 4 16 bundles, 8 are economically feasible. Their PW<br />

values by Equation [12.1] are summarized in Table 12–3. As an illustration, for bundle 7:<br />

PW 7 16,000 5220( P A ,15%,4) 2680( P F ,15%,5) $235<br />

Select bundle 5 (projects A and C) for a $16,000 investment.<br />

TABLE 12–3 Present Worth Analysis for Unequal-Life Independent Projects,<br />

Example 12.2<br />

Bundle J<br />

(1)<br />

Project<br />

(2)<br />

Initial<br />

Investment,<br />

NCF j0 , $<br />

(3)<br />

Net Cash Flows<br />

Year t<br />

(4)<br />

NCF jt , $<br />

(5)<br />

Present<br />

Worth<br />

PW j , $<br />

(6)<br />

1 A 8,000 1–6 3,870 6,646<br />

2 B 15,000 1–9 2,930 1,019<br />

3 C 8,000 1–5 2,680 984<br />

4 D 8,000 1–4 2,540 748<br />

5 AC 16,000 1–5 6,550 7,630<br />

6 3,870<br />

6 AD 16,000 1–4 6,410 5,898<br />

5–6 3,870<br />

7 CD 16,000 1–4 5,220 235<br />

5 2,680<br />

8 Do nothing 0 0 0<br />

Solution by Spreadsheet<br />

Figure 12–2 presents a spreadsheet with the same information as in Table 12–3. It is necessary<br />

to initially develop the mutually exclusive bundles manually and total net cash flows each year<br />

using each project’s NCF. The NPV function is used to determine PW for each bundle j over<br />

its respective life. Bundle 5 (projects A and C) has the largest PW value (row 16).


328 Chapter 12 Independent Projects with Budget Limitation<br />

D7E7<br />

NPV($B$1,F7:F15) F6<br />

Figure 12–2<br />

Computation of PW values for independent project selection, Example 12.2.<br />

The rest of this section will help you understand why solution of the capital budgeting problem<br />

by PW evaluation using Equation [12.1] is correct. The following logic verifies the assumption of<br />

reinvestment at the MARR for all net positive cash flows when project lives are unequal. Refer to<br />

Figure 12–3, which uses the general layout of a two-project bundle. Assume each project has the<br />

same net cash flow each year. The P A factor is used for PW computation. Define n L as the life of<br />

the longer-lived project. At the end of the shorter-lived project, the bundle has a total future worth<br />

of NCF j (F A ,MARR,n j ) as determined for each project. Now, assume reinvestment at the MARR<br />

from year n j 1 through year n L (a total of n L − n j years). The assumption of the return at the MARR<br />

is important; this PW approach does not necessarily select the correct projects if the return is not<br />

at the MARR. The results are the two future worth arrows in year n L in Figure 12–3. Finally,<br />

Figure 12–3<br />

Representative cash flows<br />

used to compute PW for a<br />

bundle of two independent<br />

unequal-life projects<br />

by Equation [12.1].<br />

PW B<br />

FW B<br />

Project B<br />

Investment<br />

for B<br />

n B = n L<br />

FW A<br />

PW A<br />

Future worth<br />

Period of<br />

reinvestment<br />

at MARR<br />

Project A<br />

Investment<br />

for A<br />

n A<br />

n L<br />

Bundle PW = PW A +PW B


12.4 Capital Budgeting Problem Formulation Using Linear Programming 329<br />

PW = $235<br />

$5220<br />

$2680<br />

FW = $57,111<br />

Figure 12–4<br />

Initial investment and<br />

cash flows for bundle 7,<br />

projects C and D,<br />

Example 17.2.<br />

0<br />

1 2 3 4 5 6 7 8 9<br />

$16,000<br />

compute the bundle PW value in the initial year. This is the bundle PW PW A PW B . In general<br />

form, the bundle j present worth is<br />

PW j NCF j ( F A ,MARR, n j )( F P ,MARR, n L n j )( P F ,MARR, n L ) [12.2]<br />

Substitute the symbol i for the MARR, and use the factor formulas to simplify.<br />

PW j NCF (1 i) n j<br />

1<br />

j<br />

——————(1 i)<br />

i<br />

n L n j<br />

———— 1<br />

(1 i) n L<br />

NCF j [ (1 i) n j<br />

1<br />

——————<br />

i(1 i) n j ] [12.3]<br />

NCF j (PA,i,n j )<br />

Since the bracketed expression in Equation [12.3] is the ( P A , i , n j ) factor, computation of PW j for<br />

n j years assumes reinvestment at the MARR of all positive net cash flows until the longest-lived<br />

project is completed in year n L .<br />

To demonstrate numerically, consider bundle j 7 in Example 12.2. The evaluation is in<br />

Table 12–3, and the net cash flow is pictured in Figure 12–4. Calculate the future worth in year 9,<br />

which is the life of the longest-lived project (B).<br />

FW 5220( F A ,15%,4)( F P ,15%,5) 2680( F P ,15%,4) $57,111<br />

The present worth at the initial investment time is<br />

PW 16,000 57,111( P F ,15%,9) $235<br />

The PW value is the same as PW 7 in Table 12–3 and Figure 12–2. This demonstrates the reinvestment<br />

assumption for positive net cash flows. If this assumption is not realistic, the PW analysis<br />

must be conducted using the LCM of all project lives.<br />

12.4 Capital Budgeting Problem Formulation<br />

Using Linear Programming<br />

The procedure discussed above requires the development of mutually exclusive bundles one<br />

project at a time, two projects at a time, etc., until all 2 m bundles are developed and each one is<br />

compared with the capital limit b . As the number of independent projects increases, this process<br />

becomes prohibitively cumbersome and unworkable. Fortunately, the capital budgeting problem<br />

can be stated in the form of a linear programming model. The problem is formulated using the<br />

integer linear programming (ILP) model, which means simply that all relations are linear and that<br />

the variable x can take on only integer values. In this case, the variables can only take on the<br />

values 0 or 1, which makes it a special case called the 0-or-1 ILP model. The formulation in<br />

words follows.


330 Chapter 12 Independent Projects with Budget Limitation<br />

Maximize: Sum of PW of net cash flows of independent projects.<br />

Constraints:<br />

• Capital investment constraint is that the sum of initial investments must not exceed a specified limit.<br />

• Each project is completely selected or not selected.<br />

For the math formulation, define b as the capital investment limit, and let x k ( k 1 to m projects)<br />

be the variables to be determined. If x k 1, project k is completely selected; if x k 0, project k<br />

is not selected. Note that the subscript k represents each independent project, not a mutually<br />

exclusive bundle.<br />

If the sum of PW of the net cash flows is Z , the math programming formulation is as follows:<br />

Maximize:<br />

Constraints:<br />

km<br />

<br />

k1<br />

km<br />

<br />

k1<br />

PW k x k Z<br />

NCF k 0 x k b<br />

x k 0 or 1 for k 1, 2, . . . , m<br />

[12.4]<br />

EXAMPLE 12.3<br />

The PW k of each project is calculated using Equation [12.1] at MARR i .<br />

PW k PW of project net cash flows for n k years<br />

tn k<br />

NCF kt (PF,i,t) NCF k0 [12.5]<br />

t1<br />

Computer solution is accomplished by a linear programming software package which treats the ILP<br />

model. Also, Excel and its optimizing tool Solver can be used to develop the formulation and select<br />

the projects. The Solver tool is similar to Goal Seek with significantly more capabilities. For example,<br />

Solver allows the target cell to be maximized, minimized, or set to a specific value. This means that the<br />

function Z in Equation [12.4] can be maximized. Also, multiple changing cells can be identified, so<br />

the 0 or 1 value of the unknowns can be determined. Additionally, with the added capability to include<br />

constraints, the investment limit b and 0-or-1 requirement on the unknowns in Equation [12.4] can be<br />

accommodated. Solver is explained in detail in Appendix A, and Example 12.3 illustrates its use.<br />

Review Example 12.2. ( a ) Formulate the capital budgeting problem using the math programming<br />

model presented in Equation [12.4]. ( b ) Select the projects using Solver.<br />

Solution<br />

(a) Define the subscript k 1 through 4 for the four projects, which are relabeled as 1, 2, 3,<br />

and 4. The capital investment limit is b $20,000 in Equation [12.4].<br />

Maximize:<br />

Constraints:<br />

k4<br />

<br />

k1<br />

k4<br />

<br />

k1<br />

PW k x k Z<br />

NCF k0 x k 20,000<br />

x k 0 or 1 for k 1, 2, 3, 4<br />

Now, substitute the PW k and NCF k 0 values from Table 12–3 into the model. Plus signs are<br />

used for all values in the budget constraint. We have the complete 0-or-1 ILP formulation.<br />

Maximize:<br />

6646 x 1 1019 x 2 984 x 3 748 x 4 Z<br />

Constraints: 8000 x 1 15,000 x 2 8000 x 3 8000 x 4 20,000<br />

x 1 , x 2 , x 3 , and x 4 0 or 1<br />

The maximum PW is $7630, and the solution from Example 12.2 is written as<br />

x 1 1 x 2 0 x 3 1 x 4 0


12.4 Capital Budgeting Problem Formulation Using Linear Programming 331<br />

NPV($B$1,E7:E18) E6<br />

Figure 12–5<br />

Spreadsheet and Solver template configured to solve a capital budgeting problem, Example 12.3.<br />

( b ) Figure 12–5 presents a spreadsheet template developed to select from six or fewer independent<br />

projects with 12 years or less of net cash flow estimates per project. The spreadsheet<br />

template can be expanded in either direction if needed. Figure 12–5 (inset) shows the<br />

Solver parameters set to solve this example for four projects and an investment limit of<br />

$20,000. The descriptions below and the cell tag identify the contents of the rows and cells<br />

in Figure 12–5, and their linkage to Solver parameters.<br />

Rows 4 and 5: Projects are identified by numbers to distinguish them from spreadsheet<br />

column letters. Cell I5 is the expression for Z , the sum of the PW values for<br />

the projects. This is the target cell for Solver to maximize.<br />

Rows 6 to 18: These are initial investments and net cash flow estimates for each<br />

project. Zero values that occur after the life of a project need not be entered;<br />

however, any $0 estimates that occur during a project’s life must be entered.<br />

Row 19: The entry in each cell is 1 for a selected project and 0 if not selected. These<br />

are the changing cells for Solver. Since each entry must be 0 or 1, a binary constraint<br />

is placed on all row 19 cells in Solver, as shown in Figure 12–5. When a<br />

problem is to be solved, it is best to initialize the spreadsheet with 0s for all projects.<br />

Solver will find the solution to maximize Z .<br />

Row 20: The NPV function is used to find the PW for each net cash flow series. The<br />

NPV functions are developed for any project with a life up to 12 years at the<br />

MARR entered in cell B1.<br />

Row 21: When a project is selected, the contribution to the Z function is shown by<br />

multiplying rows 19 and 20.<br />

Row 22: This row shows the initial investment for the selected projects. Cell I22 is<br />

the total investment. This cell has the budget limitation placed on it by the constraint<br />

in Solver. In this example, the constraint is I22 $20,000.<br />

To solve the example, set all values in row 19 to 0, set up the Solver parameters as described<br />

above, and click on Solve. (Since this is a linear model, the Solver options choice “Assume Linear<br />

Model” may be checked, if desired.) If needed, further directions on saving the solution,<br />

making changes, etc., are available in Appendix A, Section A.5, and on the Excel help function.<br />

For this problem, the selection is projects 1 and 3 with Z $7630, the same as determined<br />

previously, and $16,000 of the $20,000 limit is invested.


332 Chapter 12 Independent Projects with Budget Limitation<br />

12.5 Additional Project Ranking Measures<br />

The PW-based method of solving a capital budgeting problem covered in Sections 2.2 to 2.4<br />

provides an optimal solution that maximizes the PW of the competing projects. However, it is<br />

very common in industrial, professional, and government settings to learn that the rate of return<br />

is the basis for ranking projects. The internal rate of return (IROR), as we learned in Section<br />

7.2, is determined by setting a PW or AW relation equal to zero and solving for i * — the<br />

IROR. Using a PW basis and the estimated net cash flow (NCF) series for each project j , solve<br />

for i * in the relation<br />

tn j<br />

0 NCF jt (PF,i*,t) NCF j0 [12.6]<br />

t1<br />

This is the same as Equation [12.1] set equal to zero with i i * as the unknown. The spreadsheet<br />

function RATE or IRR will provide the same answer. The selection guideline is as follows:<br />

Independent project<br />

selection<br />

Once the project ranking by IROR is complete, select all projects in order without exceeding<br />

the investment limit b.<br />

If there is no budget limit, select all projects that have IROR MARR.<br />

The ordering of projects using IROR ranking may differ from the PW-based ranking we used in<br />

previous sections. This can occur because IROR ranking maximizes the overall rate of return, not<br />

necessarily the PW value. The use of IROR ranking is illustrated in the next example.<br />

Another common ranking method is the profitability index (PI) that we learned in<br />

Section 9.2. This is a “bang for the buck” measure that provides a sense of getting the most for<br />

the investment dollar over the life of the project. (Refer to Section 9.2 for more details.) When<br />

utilized for project ranking, it is often called the present worth index (PWI); however, we will<br />

use the PI term for consistency, and because there are other ways to mathematically define measures<br />

also referred to as a PW index. The PI measure is defined as<br />

tn j<br />

NCF jt (PF, i, t)<br />

———————————<br />

PW of net cash flows<br />

PW of initial investment t1<br />

————————<br />

NCF j0 <br />

[12.7]<br />

Note that the denominator includes only the initial investment, and it is its absolute value that is<br />

used. The numerator has only cash flows that result from the project for years 1 through its life<br />

n j . Salvage value, if there is one estimated, is incorporated into the numerator. Similar to the<br />

previous case, the selection guideline is as follows:<br />

Independent project<br />

selection<br />

Once the project ranking by PI is complete, select all projects in order without exceeding the<br />

investment limit b.<br />

If there is no budget limit, select all projects that have PI 1.0.<br />

Depending upon the project NCF estimates, the PI ranking can differ from the IROR ranking . Example<br />

12.4 compares results using the different ranking methods—IROR, PI, and PW values. None<br />

of these results are incorrect; they simply maximize different measures, as you will see. The use of<br />

IROR, PI, or other measures is common when there are a large number of projects, because the PW<br />

basis (when solved by hand, Solver, or ILP software) becomes increasingly cumbersome as the number<br />

of possible mutually exclusive bundles grows using the formula 2 m . Additionally, greater complexity<br />

is introduced when dependent and contingent projects are involved.<br />

EXAMPLE 12.4<br />

Georgia works as a financial analyst in the Management Science Group of General Electronics.<br />

She has been asked to recommend which of the five projects detailed in Table 12–4 should be<br />

funded if the MARR is 15% per year and the investment budget limit for next year is a firm


12.5 Additional Project Ranking Measures 333<br />

TABLE 12–4 IROR, PI, and PW Values for Five Projects, Example 12.4<br />

Projects 1 2 3 4 5<br />

Investment, NCF 0 , $1000 8,000 15,000 8,000 8,000 5,000<br />

NCF, $1000 per year 4,000 2,900 2,700 2,500 2,600<br />

Life n j , years 6 9 5 4 3<br />

IROR, % 44.5 12.8 20.4 9.6 26.0<br />

PI 1.89 0.92 1.13 0.89 1.19<br />

PW at 15%, $1000 7,138 1,162 1,051 863 936<br />

$18 million. She has confirmed the computations and is ready to do the ranking and make the<br />

selection. Help her by doing the following:<br />

(a) Use the IROR measure to rank and select projects.<br />

(b) Use the PI measure to rank and select projects.<br />

(c) Use the PW measure to rank and select projects.<br />

(d) Compare the selected projects by the three methods and determine which one will maximize<br />

the overall ROR value of the $18 million budget.<br />

Solution<br />

Refer to Table 12–5 for the ranking, cumulative investment for each project, and selection<br />

based on the ranking and budget limit b $18 million.<br />

(a) Ranking by overall IROR values indicates that projects 1 and 5 should be selected with<br />

$13 million of the $18 million budget expended. The remaining $5 million is assumed to<br />

be invested at the MARR of 15% per year.<br />

(b) As an example, the PI for project 1 is calculated using Equation [12.7].<br />

PI 1 ————————<br />

4000(PA,15%,6)<br />

|8000|<br />

15,1388,000<br />

1.89<br />

Ranking and projects selected are the same as in the IROR-based analysis. Again, the remaining<br />

$5 million is assumed to generate a return of MARR 15% per year.<br />

(c) Ranking by PW value results in a different selection from that of IROR and PI ranking.<br />

Projects 1 and 3, rather than 1 and 5, are selected for a total PW $8.189 million and an<br />

investment of $16 million. The remaining $2 million is assumed to earn 15% per year.<br />

(d) IROR and PI rankings result in projects 1 and 5 being selected. PW ranking results in the<br />

selection of projects 1 and 3. With MARR 15%, the $18 million will earn at the<br />

following ROR values. In $1000 units,<br />

Projects 1 and 5 NCF, year 0: $13,000<br />

NCF, years 1–3: $6,600<br />

NCF, years 4–6: $4,000<br />

0 13,000 6600(PA,i,3) 4000(PA,i,3)(PF,i,3)<br />

TABLE 12–5 Ranking of Projects by Different Measures, Example 12.4 (Monetary units in $1000)<br />

IROR, %<br />

(1)<br />

Ranking by IROR Ranking by PI Ranking by PW<br />

Project<br />

(2)<br />

Cumulative<br />

Investment, $<br />

(3)<br />

PI<br />

(4)<br />

Project<br />

(5)<br />

Cumulative<br />

Investment, $<br />

(6)<br />

PW, $<br />

(7)<br />

Project<br />

(8)<br />

Cumulative<br />

Investment, $<br />

(9)<br />

44.5 1 8,000 1.89 1 8,000 7,138 1 8,000<br />

26.0 5 13,000 1.19 5 13,000 1,051 3 16,000<br />

20.4 3 21,000 1.13 3 21,000 936 5 21,000<br />

12.8 2 0.92 2 863 4<br />

9.6 4 0.89 4 1,162 2


334 Chapter 12 Independent Projects with Budget Limitation<br />

By IRR function, the rate of return is 39.1%. The overall return on the entire budget is<br />

ROR [39.1(13,000) 15.0(5000)]18,000<br />

32.4%<br />

Projects 1 and 3 NCF, year 0: $16,000<br />

NCF, years 1–5: $6,700<br />

NCF, year 6: $4,000<br />

0 16,000 6700(PA,i,5) 4000(PF,i,6)<br />

By IRR function, the rate of return is 33.5%. The overall return on the entire budget is<br />

ROR [33.5(16,000) 15.0(2000)]18,000<br />

31.4%<br />

In conclusion, the IROR and PI selections maximize the overall rate of return at 32.4%. The<br />

PW selection maximizes the PW value at $8.189 (i.e., 7.138 1.051) million, as determined<br />

from Table 12–5, column 7.<br />

CHAPTER SUMMARY<br />

Investment capital is always a scarce resource, so it must be rationed among competing projects<br />

using economic and noneconomic criteria. Capital budgeting involves proposed projects, each<br />

with an initial investment and net cash flows estimated over the life of the project. The fundamental<br />

capital budgeting problem has specific characteristics (Figure 12–1).<br />

• Selection is made from among independent projects.<br />

• Each project must be accepted or rejected as a whole.<br />

• Maximizing the present worth of the net cash flows is the objective.<br />

• The total initial investment is limited to a specified maximum.<br />

The present worth method is used for evaluation. To start the procedure, formulate all mutually<br />

exclusive bundles that do not exceed the investment limit, including the do-nothing bundle.<br />

There are a maximum of 2 m bundles for m projects. Calculate the PW at MARR for each bundle,<br />

and select the bundle with the largest PW value. Reinvestment of net positive cash flows at the<br />

MARR is assumed for all projects with lives shorter than that of the longest-lived project.<br />

The capital budgeting problem may be formulated as a linear programming problem to select<br />

projects directly in order to maximize the total PW. Excel’s Solver tool solves this problem by<br />

spreadsheet.<br />

Projects can be ranked and selected on bases other than PW values. Two measures are the<br />

internal rate of return (IROR) and the profitability index (PI), also called the PW index. The<br />

ordering of projects may differ between the various ranking bases since different measures are<br />

optimized. When there are a large number of projects, the IROR basis is commonly applied in<br />

industrial settings.<br />

PROBLEMS<br />

Understanding Capital Rationing<br />

12.1 Define the following terms: bundle, contingent<br />

project, dependent project.<br />

12.2 State two assumptions made when doing capital rationing<br />

using a PW analysis for unequal-life projects.<br />

12.3 For independent projects identified as A, B, C, D,<br />

E, F, and G, how many mutually exclusive bundles<br />

can be formed?<br />

12.4 For independent projects identified as W, X, Y, and<br />

Z, develop all of the mutually exclusive bundles.<br />

Projects X and Y perform the same function with<br />

different processes; both should not be selected.<br />

12.5 Five projects have been identified for possible<br />

implementation by a company that makes dry ice<br />

blasters—machines that propel tiny dry ice pellets<br />

at supersonic speeds so they flash-freeze and<br />

then lift grime, paint, rust, mold, asphalt, and


Problems 335<br />

other contaminants off in-place machines and a<br />

wide range of surfaces. The total present worth<br />

of the initial investment for each project is<br />

shown. Determine which bundles are possible,<br />

provided the budget limitation is ( a ) $34,000 and<br />

( b ) $45,000.<br />

Project<br />

PW of Investment<br />

at 15%, $<br />

L 28,000<br />

M 11,000<br />

N 43,000<br />

O 38,000<br />

P 6,000<br />

12.6 Four independent projects (1, 2, 3, and 4) are proposed<br />

for investment by Perfect Manufacturing,<br />

Inc. List all the acceptable mutually exclusive<br />

bundles based on the following selection restrictions<br />

developed by the department of engineering<br />

production:<br />

Project 2 can be selected only if project 3 is<br />

selected.<br />

Projects 1 and 4 should not both be selected;<br />

they are essentially duplicates.<br />

12.7 Develop all acceptable mutually exclusive bundles<br />

for the four independent projects described<br />

below if the investment limit is $400 and the following<br />

project selection restriction applies: Project<br />

1 can be selected only if both projects 3 and 4<br />

are selected.<br />

Project Investment, $<br />

1 100<br />

2 150<br />

3 75<br />

4 235<br />

Selection from Independent Projects<br />

12.8 Listed below are bundles, each comprised of three<br />

independent proposals for which the PW has been<br />

estimated. Select the best bundle if the capital budget<br />

limit is $45,000 and the MARR is the cost of<br />

capital, which is 9% per year.<br />

Proposal<br />

Bundle<br />

Initial Investment,<br />

For Bundle, $<br />

PW at 9%,<br />

$<br />

1 18,000 1,400<br />

2 26,000 8,500<br />

3 34,000 7,100<br />

4 41,000 10,500<br />

12.9 The general manager for Woodslome Appliance<br />

Company Plant #A14 in Mexico City has four<br />

independent projects that she can fund this year to<br />

improve surface durability on stainless steel<br />

products. The project costs and 18% per year PW<br />

values are as shown. What projects should be accepted<br />

if the budget limit is ( a ) no limit and<br />

( b ) $55,000?<br />

Project<br />

Initial<br />

Investment, $<br />

PW at 18%,<br />

$<br />

1 15,000 400<br />

2 25,000 8500<br />

3 20,000 500<br />

4 40,000 9600<br />

12.10 The capital fund for research project investment at<br />

SummaCorp is limited to $100,000 for next year.<br />

Select any or all of the following proposals if a<br />

MARR of 15% per year is established by the board<br />

of directors.<br />

Project<br />

Initial<br />

Investment, $<br />

Annual Net Cash<br />

Flow, $Year<br />

Life,<br />

Years<br />

Salvage<br />

Value, $<br />

I 25,000 6,000 4 4,000<br />

II 30,000 9,000 4 1,000<br />

III 50,000 15,000 4 20,000<br />

12.11 An electrical engineer at GE is assigned the responsibility<br />

to determine how to invest up to $100,000<br />

in none, some, or all of the following independent<br />

proposals. Use ( a ) hand and ( b ) spreadsheet-based<br />

PW analysis and a 15% per year return requirement<br />

to help this engineer make the best decision from a<br />

purely economic perspective.<br />

Project<br />

Initial<br />

Investment, $<br />

Annual Net Cash<br />

Flow, $Year<br />

Life,<br />

Years<br />

Salvage<br />

Value, $<br />

A 25,000 6,000 4 4,000<br />

B 20,000 9,000 4 0<br />

C 50,000 15,000 4 20,000<br />

12.12 Dwayne has four independent vendor proposals to<br />

contract the nationwide oil recycling services for<br />

Ford Corporation manufacturing plants. All combinations<br />

are acceptable, except that vendors B<br />

and C cannot both be chosen. Revenue sharing of<br />

recycled oil sales with Ford is a part of the requirement.<br />

Develop all possible mutually exclusive<br />

bundles under the additional following restrictions<br />

and select the best projects. The corporate MARR<br />

is 10% per year.<br />

(a) A maximum of $4 million can be spent.<br />

( b) A larger budget of $5.5 million is allowed,<br />

but no more than two vendors can be<br />

selected.<br />

(c) There is no limit on spending.


336 Chapter 12 Independent Projects with Budget Limitation<br />

Vendor<br />

Initial<br />

Investment, $<br />

Life,<br />

Years<br />

Annual Net<br />

Revenue, $ per Year<br />

A 1.5 million 8 360,000<br />

B 3.0 million 10 600,000<br />

C 1.8 million 5 620,000<br />

D 2.0 million 4 630,000<br />

12.13 Use the PW method at 8% per year to select up to<br />

three projects from the four available ones if no<br />

more than $20,000 can be invested. Estimated<br />

lives and annual net cash flows vary.<br />

Project<br />

Initial<br />

Investment, $<br />

Net Cash Flow, $ per Year<br />

1 2 3 4 5 6<br />

W 5,000 1,000 1,700 1,800 2,500 2,000<br />

X 8,000 900 950 1,000 1,050 10,500<br />

Y 8,000 4,000 3,000 1,000 500 500 2,000<br />

Z 10,000 0 0 0 17,000<br />

12.14 Charlie’s Garage has $70,000 to spend on new<br />

equipment that may increase revenue for his car<br />

repair shop. Use the PW method to determine<br />

which of these independent investments are financially<br />

acceptable at 6% per year, compounded<br />

monthly. All are expected to last 3 years.<br />

Feature<br />

Installed<br />

Cost, $<br />

Estimated Added<br />

Revenue, $ per Month<br />

Diagnostics system 45,000 2200<br />

Exhaust analyzer 30,000 2000<br />

Hybrid engine tester 22,000 1500<br />

Project<br />

Initial<br />

Investment, $<br />

Life, Years<br />

PW at 12%<br />

per Year, $<br />

S 15,000 6 8,540<br />

A 25,000 8 12,325<br />

M 10,000 6 3,000<br />

E 25,000 4 10<br />

H 40,000 12 15,350<br />

12.17 The independent project estimates below have<br />

been developed by the engineering and finance<br />

managers. The corporate MARR is 8% per year,<br />

and the capital investment limit is $4 million. Select<br />

the economically best projects using the PW<br />

method and ( a ) hand solution and ( b ) spreadsheet<br />

solution.<br />

Project<br />

Project<br />

Cost, $ M<br />

Life, Years<br />

NCF,<br />

$ per Year<br />

1 1.5 8 360,000<br />

2 3.0 10 600,000<br />

3 1.8 5 520,000<br />

4 2.0 4 820,000<br />

12.18 Use the PW method to evaluate four independent<br />

projects. Select as many as three of the four projects.<br />

The MARR is 12% per year, and up to<br />

$16,000 in capital investment funds are available.<br />

Project<br />

1 2 3 4<br />

Investment, $ 5000 8,000 9,000 10,000<br />

Life, years 5 5 3 4<br />

12.15 (a) Determine which of the following independent<br />

projects should be selected for investment<br />

if $315,000 is available and the MARR<br />

is 10% per year. Use the PW method to<br />

evaluate mutually exclusive bundles to<br />

make the selection. (Solve by hand or<br />

spreadsheet as instructed.)<br />

Project<br />

Initial<br />

Investment, $<br />

NCF,<br />

$ per Year Life, Years<br />

A 100,000 50,000 8<br />

B 125,000 24,000 8<br />

C 120,000 75,000 8<br />

D 220,000 39,000 8<br />

E 200,000 82,000 8<br />

(b)<br />

If the five projects are mutually exclusive<br />

alternatives, perform the present worth analysis<br />

and select the best alternative.<br />

12.16 Use the following analysis of five independent<br />

projects to select the best, if the capital limitation<br />

is ( a ) $30,000, ( b ) $52,000, and ( c ) unlimited.<br />

Year<br />

NCF Estimates, $ per Year<br />

1 1000 500 5000 0<br />

2 1700 500 5000 0<br />

3 2400 500 2000 0<br />

4 3000 500 17,000<br />

5 3800 10,500<br />

12.19 Work Problem 12.18 using a spreadsheet.<br />

12.20 A capital rationing problem is defined for you as<br />

follows: Three projects are to be evaluated at a<br />

MARR of 12.5% per year. No more than $3.0<br />

million can be invested.<br />

(a) Use a spreadsheet to select from the independent<br />

projects.<br />

(b) If the life of project 3 can be increased from<br />

5 to 10 years for the same $1 million investment,<br />

use Goal Seek to determine the NCF in<br />

year 1 for project 3 alone to have the same<br />

PW as the best bundle in part ( a ). All other<br />

estimates remain the same. With these new<br />

NCF and life estimates, what are the best<br />

projects for investment?


Problems 337<br />

Project<br />

Investment,<br />

$ M<br />

Life,<br />

Years Year 1<br />

Estimated NCF, $ per Year<br />

Gradient<br />

after Year 1<br />

1 0.9 6 250,000 5000<br />

2 2.1 10 485,000 5000<br />

3 1.0 5 200,000 20%<br />

Linear Programming and Capital Budgeting<br />

12.21 Formulate the linear programming model, develop<br />

a spreadsheet, and solve the capital rationing problem<br />

in Example 12.1 ( a ) as presented and ( b ) using<br />

an investment limit of $13 million.<br />

12.22 Use linear programming and a spreadsheet-based<br />

solution technique to select from the independent<br />

unequal-life projects in Problem 12.17.<br />

12.23 Solve the capital budgeting problem in Problem<br />

12.20( a ), using the linear programming model and<br />

a spreadsheet.<br />

12.24 Johnson and Johnson is expanding its first-aid<br />

products line for individuals allergic to latex. The<br />

research team comprised of doctors, engineers,<br />

and chemists has proposed the four projects estimated<br />

in Problem 12.18. Develop the linear<br />

programming model and use a spreadsheet to select<br />

the projects to be funded. The MARR is 12%<br />

per year, and the budget limit is $16 million. All<br />

monetary values are in $1000 units.<br />

12.25 Using the estimates in Problem 12.18 and repeated<br />

spreadsheet solution of the capital budgeting problem<br />

for budget limits ranging from b $5000 to b<br />

$25,000, develop a spreadsheet chart that plots b<br />

versus the value of Z .<br />

Other Ranking Measures<br />

12.26 A newly proposed project has a first cost of<br />

$325,000 and estimated annual income of $60,000<br />

per year for 8 years.<br />

( a) Determine the IROR, PI, and PW values if<br />

the MARR is 15% per year.<br />

( b) Is the project economically justified?<br />

12.27 An engineer at Delphi Systems is considering the<br />

projects below, all of which can be considered to<br />

last indefinitely. The company’s MARR is 13%<br />

per year.<br />

( a) Determine which projects should be selected<br />

on the basis of IROR if the budget limitation<br />

is $39,000.<br />

( b) What is the overall rate of return if the money<br />

not invested in projects is assumed to earn a<br />

rate of return equal to the MARR?<br />

Project First Cost, $<br />

Net Income,<br />

$ per Year<br />

Rate of<br />

Return, %<br />

A 20,000 4,000 20.0<br />

B 10,000 1,900 19.0<br />

C 15,000 2,600 17.3<br />

D 70,000 10,000 14.3<br />

E 50,000 6, 000 12.0<br />

12.28 The five independent projects shown below are under<br />

consideration for implementation by KNF Neuberger,<br />

Inc. The company’s MARR is 15% per year.<br />

(a) Determine which projects should be undertaken<br />

on the basis of IROR if the budget<br />

limitation is $97,000. (Solve by hand or<br />

spreadsheet as instructed.)<br />

(b) Determine the overall rate of return if the<br />

money not invested in projects is assumed to<br />

earn a rate of return equal to the MARR.<br />

Project First Cost, $<br />

Annual<br />

Income, $ per Year<br />

Project<br />

Life, Years<br />

L 30,000 9,000 10<br />

A 15,000 4,900 10<br />

N 45,000 11,100 10<br />

D 70,000 9,000 10<br />

T 40,000 10,000 10<br />

12.29 An estimated 6 billion gallons of clean drinking<br />

water disappear each day across the United States<br />

due to aging, leaky pipes and water mains before<br />

it gets to the consumer or industrial user. The American<br />

Society of Civil Engineers (ASCE) has teamed<br />

with municipalities, counties, and several excavation<br />

companies to develop robots that can travel through<br />

mains, detect leaks, and repair many of them immediately.<br />

Four proposals have been received for funding.<br />

There is a $100 million limit on capital funding,<br />

and the MARR is established at 12% per year.<br />

(a) Use the IROR method to rank and determine<br />

which of the four independent projects should<br />

be funded. Solve by spreadsheet, unless instructed<br />

to use hand solution.<br />

Project<br />

(b)<br />

Determine the rate of return for the combination<br />

of projects selected in part ( a ). Is it economically<br />

justified?<br />

(c) Determine the overall rate of return for the<br />

$100 million with the projects selected in<br />

part ( a ). Assume that excess funds are invested<br />

at the MARR.<br />

First<br />

Cost, $ M<br />

Estimated Annual<br />

Savings, $M per Year<br />

Project<br />

Life, Years<br />

W 12 5.0 3<br />

X 25 7.3 4<br />

Y 45 12.1 6<br />

Z 60 9.0 8


338 Chapter 12 Independent Projects with Budget Limitation<br />

12.30 Determine the profitability index at 10% per year<br />

interest for a project that has a first cost of<br />

$200,000 in year 0 and $200,000 in year 2, an annual<br />

operating cost of $80,000 per year, revenue<br />

of $170,000 per year, and a salvage value of<br />

$60,000 after its 5-year life.<br />

12.31 The budget limit is $120,000, and the interest rate<br />

is 10% per year. All projects have a 10-year life.<br />

Use ( a ) the PI method and ( b ) the IROR method to<br />

rank and select from the independent projects.<br />

( c ) Are different projects selected using the two<br />

methods?<br />

Project First Cost, $<br />

Net Income,<br />

$ per Year IROR, %<br />

A 18,000 4,000 18.0<br />

B 15,000 2,800 13.3<br />

C 35,000 12,600 34.1<br />

D 60,000 13,000 17.3<br />

E 50,000 8,000 9.6<br />

12.32 The six independent projects shown below are<br />

under consideration by Peyton Packing under<br />

budget-constrained conditions. The company always<br />

has more projects to engage in than it has<br />

capital to fund projects. Therefore, it uses a relatively<br />

high MARR of 25% per year. Since all projects<br />

are considered long-term ventures, the company<br />

uses an infinite period for their life. Determine<br />

which projects the company should fund and the<br />

total investment for a capital budget of $700,000 if<br />

the capital budgeting method used is ( a ) the IROR<br />

method, ( b ) the PI method, and ( c ) the PW method.<br />

Project First Cost, $<br />

Estimated Annual<br />

Income, $ per Year<br />

F 200,000 54,000<br />

G 120,000 21,000<br />

H 250,000 115,000<br />

I 370,000 205,000<br />

J 50,000 26,000<br />

K 9,000 2,100<br />

ADDITIONAL PROBLEMS AND FE EXAM REVIEW QUESTIONS<br />

12.33 A project that has a condition associated with its<br />

acceptance or rejection is known as:<br />

(a) A mutually exclusive alternative<br />

(b) A contingent project<br />

(c) A dependent project<br />

(d) Both ( b ) and ( c )<br />

(a)<br />

(b)<br />

(c)<br />

(d)<br />

One of the bundles is the do-nothing project.<br />

A bundle may consist of only one project.<br />

The capital limit may be exceeded as long as<br />

it is exceeded by less than 3%.<br />

A bundle may include contingent and dependent<br />

projects.<br />

12.34 An assumption inherent in capital budgeting is that<br />

all positive net cash flows are reinvested at the<br />

MARR from the time they are realized until:<br />

(a) The end of the shortest-lived project<br />

(b)<br />

(c)<br />

The end of the longest-lived project<br />

The end of the respective project that generated<br />

the cash flow<br />

(d) The average of the lives of the projects included<br />

in the bundle<br />

12.35 All of the following are correct when a capital<br />

budgeting problem is solved using the 0-1 integer<br />

linear programming model except:<br />

(a) Partial investment in a project is acceptable.<br />

(b) The objective is to maximize the present<br />

worth of the investments.<br />

(c) Budget constraints may be present for the<br />

first year only or for several years.<br />

(d) Contingent and dependent project restrictions<br />

may be considered.<br />

12.36 All of the following are true when formulating<br />

mutually exclusive bundles of independent projects<br />

except:<br />

12.37 When there are 5 projects involved in a capital<br />

budgeting study, the maximum number of bundles<br />

that can be formulated is:<br />

( a ) 6<br />

( b ) 10<br />

( c ) 31<br />

( d ) 32<br />

12.38 For a capital limit of $25,000, the selected independent<br />

projects are:<br />

( a ) P only<br />

( b ) Q only<br />

( c ) R only<br />

( d ) P and R<br />

Project<br />

Initial<br />

Investment, $<br />

Life, Years<br />

PW at<br />

12%, $<br />

P 15,000 6 8,540<br />

Q 25,000 8 12,325<br />

R 10,000 6 3,000<br />

S 25,000 4 10<br />

T 40,000 12 15,350


Additional Problems and FE Exam Review Questions 339<br />

12.39 The independent projects shown below are under<br />

consideration for possible implementation by<br />

Renishaw Inc. of Hoffman Estates, Rhode Island.<br />

If the company’s MARR is 14% per year and it<br />

uses the IROR method of capital budgeting, the<br />

projects it should select under a budget limitation<br />

of $105,000 are:<br />

( a) A, B, and C<br />

( b) A, B, and D<br />

( c) B, C, and D<br />

( d) A, C, and D<br />

12.40 For a project that requires an initial investment<br />

of $26,000 and yields $10,000 per year for<br />

4 years, the PI at an interest rate of 10% per year<br />

is closest to:<br />

( a ) 1.03<br />

( b ) 1.22<br />

( c ) 1.38<br />

( d ) 1.56<br />

Project First Cost, $<br />

Annual<br />

Income, $ per Year<br />

Rate of<br />

Return, %<br />

A 20,000 4,000 20.0<br />

B 10,000 1,900 19.0<br />

C 15,000 2,600 17.3<br />

D 70,000 10,000 14.3<br />

E 50,000 6,000 12.0


CHAPTER 13<br />

Breakeven and<br />

Payback<br />

Analysis<br />

LEARNING OUTCOMES<br />

Purpose: Determine the breakeven for one or two alternatives and calculate the payback period with and without a<br />

return required.<br />

SECTION TOPIC LEARNING OUTCOME<br />

13.1 Breakeven point • Determine the breakeven point for one<br />

parameter.<br />

13.2 Two alternatives • Calculate the breakeven point of a parameter<br />

and use it to select between two alternatives.<br />

13.3 Payback period • Determine the payback period of a project at<br />

i 0% and i 0%. Illustrate the cautions when<br />

using payback analysis.<br />

13.4 Spreadsheet • Answer breakeven and payback questions that<br />

are best resolved by spreadsheet and Goal Seek<br />

tools.


B<br />

reakeven analysis is performed to determine the value of a variable or parameter<br />

of a project or alternative that makes two elements equal, for example, the<br />

sales volume that will equate revenues and costs. A breakeven study is performed<br />

for two alternatives to determine when either alternative is equally acceptable. Breakeven<br />

analysis is commonly applied in make-or-buy decisions when a decision is needed about the<br />

source for manufactured components, services, etc.<br />

Payback analysis determines the required minimum life of an asset, process, or system to<br />

recover the initial investment. There are two types of payback: return ( i 0%) and no return<br />

( i 0%). Payback analysis should not be considered the final decision maker; it is used as<br />

a screening tool or to provide supplemental information for a PW, AW, or other analysis.<br />

These aspects are discussed in depth in this chapter.<br />

Breakeven and payback studies use estimates that are considered to be certain; that is, if<br />

the estimated values are expected to vary enough to possibly change the outcome, another<br />

study is necessary using different estimates. If the variable of interest is allowed to vary, the<br />

approaches of sensitivity analysis (Chapter 18) should be used. Additionally, if probability<br />

and risk assessment are considered, the tools of simulation (Chapter 19) can be used to<br />

supplement the static nature of a breakeven or payback study.<br />

13.1 Breakeven Analysis for a Single Project<br />

When one of the engineering economy symbols— P , F , A , i , or n —is not known or not estimated,<br />

a breakeven quantity can be determined by setting an equivalence relation for PW or AW equal<br />

to zero. This form of breakeven analysis has been used many times so far. For example, we have<br />

solved for the rate of return i *, found the replacement value for a defender, and determined the P ,<br />

F , A , or salvage value S at which a series of cash flow estimates return a specific MARR. Methods<br />

used to determine the quantity include<br />

Direct solution by hand if only one factor is present (say, P A ) or only single amounts are<br />

estimated (for example, P and F )<br />

Trial and error by hand or calculator when multiple factors are present<br />

Spreadsheet when cash flow and other estimates are entered into cells and used in resident<br />

functions (PV, FV, RATE, IRR, NPV, PMT, and NPER) or tools (Goal Seek and Solver).<br />

We now concentrate on the determination of the breakeven quantity Q BE for one parameter<br />

or decision variable. For example, the variable may be a design element to minimize cost or the<br />

production level needed to realize revenues that exceed costs by 10%.<br />

Breakeven analysis finds the value of a parameter that makes two elements equal . The breakeven<br />

point Q BE is determined from mathematical relations, e.g., product revenue and costs or<br />

materials supply and demand or other parameters that involve the parameter Q . Breakeven<br />

analysis is fundamental to evaluations such as make-buy decisions.<br />

Breakeven<br />

The unit of the parameter Q may vary widely: units per year, cost per kilogram, hours per month,<br />

percentage of full plant capacity, etc.<br />

Figure 13–1 a presents different shapes of a revenue relation identified as R . A linear revenue<br />

relation is commonly assumed, but a nonlinear relation is often more realistic. It can model an<br />

increasing per unit revenue with larger volumes (curve 1 in Figure 13–1 a ) or a decreasing per<br />

unit revenue that usually prevails at higher quantities (curve 2).<br />

Costs, which may be linear or nonlinear, usually include two components—fixed and<br />

variable—as indicated in Figure 13–1 b .<br />

Fixed costs (FC). These include costs such as buildings, insurance, fixed overhead, some<br />

minimum level of labor, equipment capital recovery, and information systems.<br />

Variable costs (VC). These include costs such as direct labor, materials, indirect costs, contractors,<br />

marketing, advertisement, and warranty.<br />

The fixed-cost component is essentially constant for all values of the variable, so it does not vary<br />

for a large range of operating parameters, such as production level or workforce size. Even if no<br />

units are produced, fixed costs are incurred at some threshold level. Of course, this situation


342 Chapter 13 Breakeven and Payback Analysis<br />

Figure 13–1<br />

Linear and nonlinear<br />

revenue and cost relations.<br />

R, revenue per year<br />

(2)<br />

(1)<br />

Linear<br />

Nonlinear<br />

Q, units per year<br />

(a) Revenue relations—(1) increasing and<br />

(2) decreasing revenue per unit<br />

TC FC VC<br />

TC FC VC<br />

Cost per year<br />

Total<br />

cost, TC<br />

Variable, VC<br />

Cost per year<br />

TC<br />

VC<br />

Fixed, FC<br />

Q, units per year<br />

(b) Linear cost relations<br />

FC<br />

Q, units per year<br />

(c) Nonlinear cost relations<br />

cannot last long before the plant must shut down to reduce fixed costs. Fixed costs are reduced<br />

through improved equipment, information systems, and workforce utilization; and less costly<br />

fringe benefit packages, subcontracting specific functions, and so on.<br />

Variable costs change with production level, workforce size, and other parameters. It is usually<br />

possible to decrease variable costs through better product design, manufacturing efficiency,<br />

improved quality and safety, and higher sales volume.<br />

When FC and VC are added, they form the total cost relation TC . Figure 13–1 b illustrates<br />

the TC relation for linear fixed and variable costs. Figure 13–1 c shows a general TC curve for a<br />

nonlinear VC in which unit variable costs decrease as the quantity level rises.<br />

At a specific but unknown value Q of the decision variable, the revenue R and total cost TC<br />

relations will intersect to identify the breakeven point Q BE (Figure 13–2). If Q Q BE , there is a<br />

predictable profit ; but if Q Q BE , there is a loss. For linear models of R and VC, the greater the<br />

quantity, the larger the profit. Profit is calculated as<br />

Profit revenue total cost<br />

R TC<br />

R (FC VC) [13.1]<br />

A relation for the breakeven point may be derived when revenue and total cost are linear functions<br />

of quantity Q by setting the relations for R and TC equal to each other, indicating a profit of zero.<br />

R TC<br />

rQ FC vQ<br />

where<br />

r revenue per unit<br />

v variable cost per unit<br />

Solve for the breakeven quantity Q Q BE for linear R and TC functions.<br />

Q BE FC ———<br />

r v<br />

[13.2]


13.1 Breakeven Analysis for a Single Project 343<br />

$<br />

R<br />

Breakeven<br />

TC<br />

Breakeven with<br />

lowered VC<br />

TC with<br />

lowered VC<br />

Profit<br />

maximized<br />

Loss<br />

Breakeven<br />

point moves<br />

Profit<br />

Q BE<br />

Q, units per year<br />

Figure 13–2<br />

Effect on the breakeven point when the variable cost per unit is reduced.<br />

$<br />

TC<br />

Profit<br />

maximized<br />

R<br />

Profit range<br />

Loss<br />

Loss<br />

Q BE<br />

Q P Q BE<br />

Q, units per year<br />

Figure 13–3<br />

Breakeven points and maximum profit point for a nonlinear analysis.<br />

The breakeven graph is an important management tool because it is easy to understand and may be used<br />

in decision making in a variety of ways. For example, if the variable cost per unit is reduced, then the<br />

TC line has a smaller slope (Figure13–2) and the breakeven point will decrease. This is an advantage<br />

because the smaller the value of Q BE , the greater the profit for a given amount of revenue. A similar<br />

analysis is possible for fixed VC and increased levels of production, as shown in the next example.<br />

If nonlinear R or TC models are used, there may be more than one breakeven point. Figure 13–3<br />

presents this situation for two breakeven points. The maximum profit occurs at Q P between the<br />

two breakeven points where the distance between the R and TC relations is greatest.


344 Chapter 13 Breakeven and Payback Analysis<br />

EXAMPLE 13.1<br />

Of course, no static R and TC relations—linear or nonlinear—are able to estimate exactly the<br />

revenue and cost amounts over an extended period of time. But the breakeven point is an excellent<br />

target for planning purposes.<br />

Indira Industries is a major producer of diverter dampers used in the gas turbine power industry<br />

to divert gas exhausts from the turbine to a side stack, thus reducing the noise to acceptable<br />

levels for human environments. Normal production level is 60 diverter systems per month, but<br />

due to significantly improved economic conditions in Asia, production is at 72 per month. The<br />

following information is available.<br />

Fixed costs<br />

FC $2.4 million per month<br />

Variable cost per unit v $35,000<br />

Revenue per unit r $75,000<br />

(a) How does the increased production level of 72 units per month compare with the current<br />

breakeven point?<br />

(b) What is the current profit level per month for the facility?<br />

(c) What is the difference between the revenue and variable cost per damper that is necessary<br />

to break even at a significantly reduced monthly production level of 45 units, if fixed costs<br />

remain constant?<br />

Solution<br />

(a) Use Equation [13.2] to determine the breakeven number of units. All dollar amounts are<br />

in $1000 units.<br />

Q BE ———<br />

r FC v<br />

———— 2400<br />

60 units per month<br />

75 35<br />

Figure 13–4 is a plot of R and TC lines. The breakeven value is 60 damper units. The<br />

increased production level of 72 units is above the breakeven value.<br />

7000<br />

Revenue line<br />

6000<br />

$1000<br />

5000<br />

4500<br />

4000<br />

3000<br />

2400<br />

2000<br />

Total cost line<br />

Profit<br />

1000<br />

Q BE = 60<br />

0 15 30 45 60 75 90<br />

Q, units per month<br />

Figure 13–4<br />

Breakeven graph, Example 13.1.


13.2 Breakeven Analysis Between Two Alternatives 345<br />

(b) To estimate profit (in $1000 units) at Q 72 units per month, use Equation [13.1].<br />

Profit R TC rQ (FC vQ)<br />

(r v)Q FC<br />

(75 35)72 2400 [13.3]<br />

$480<br />

There is a profit of $480,000 per month currently.<br />

(c) To determine the required difference r v, use Equation [13.3] with profit 0, Q 45,<br />

and FC $2.4 million. In $1000 units,<br />

0 (r v) (45) 2400<br />

r v ——— 2400 $53.33 per unit<br />

45<br />

The spread between r and v must be $53,330. If v stays at $35,000, the revenue per damper<br />

must increase from $75,000 to $88,330 (i.e., 35,000 53,330) just to break even at a production<br />

level of Q 45 per month.<br />

In some circumstances, breakeven analysis performed on a per unit basis is more meaningful.<br />

The value of Q BE is still calculated using Equation [13.2], but the TC relation is divided by Q to<br />

obtain an expression for cost per unit, also termed average cost per unit C u .<br />

C u TC ——<br />

Q<br />

FC vQ ————<br />

Q<br />

FC ——<br />

Q v [13.4]<br />

At the breakeven quantity Q Q BE , the revenue per unit is exactly equal to the cost per unit. If<br />

graphed, the FC per unit term in Equation [13.4] takes on the shape of a hyperbola.<br />

The breakeven point for a project with one unknown variable can always be determined by<br />

equating revenue and total cost. This is the same as setting profit equal to zero in Equation [13.1].<br />

It may be necessary to perform some dimensional analysis initially to obtain the correct revenue<br />

and total cost relations in order to use the same dimension for both relations, for example, $ per<br />

unit, miles per month, or units per year.<br />

13.2 Breakeven Analysis Between Two Alternatives<br />

Now we consider breakeven analysis between two mutually exclusive alternatives.<br />

Breakeven analysis determines the value of a common variable or parameter between two<br />

alternatives. Equating the two PW or AW relations determines the breakeven point. Selection<br />

of the alternative is different depending upon two facts: slope of the variable cost curve and<br />

the parameter value relative to the breakeven point .<br />

Breakeven<br />

The parameter can be the interest rate i , first cost P , annual operating cost (AOC), or any parameter.<br />

We have already performed breakeven analysis between alternatives on several parameters.<br />

For example, the incremental ROR value ( i *) is the breakeven rate between alternatives. If the<br />

MARR is lower than i *, the extra investment of the larger-investment alternative is justified. In<br />

Section 11.6, the replacement value (RV) of a defender was determined. If the market value is<br />

larger than RV, the decision should favor the challenger.<br />

Often breakeven analysis involves revenue or cost variables common to both alternatives,<br />

such as price per unit, operating cost, cost of materials, or labor cost. Figure 13–5 illustrates<br />

the breakeven concept for two alternatives with linear cost relations. The fixed cost of alternative<br />

2 is greater than that of alternative 1. However, alternative 2 has a smaller variable cost,<br />

as indicated by its lower slope. The intersection of the total cost lines locates the breakeven<br />

point, and the variable cost establishes the slope. Thus, if the number of units of the common<br />

variable is greater than the breakeven amount, alternative 2 is selected, since the total cost<br />

will be lower. Conversely, an anticipated level of operation below the breakeven point favors<br />

alternative 1.


346 Chapter 13 Breakeven and Payback Analysis<br />

Figure 13–5<br />

Breakeven between two<br />

alternatives with linear<br />

cost relations.<br />

Total cost<br />

Alt. 2 FC<br />

Alt. 1 TC<br />

Alt. 2 TC<br />

Alt. 1 FC<br />

Breakeven<br />

point<br />

Common variable, units<br />

Instead of plotting the total costs of each alternative and estimating the breakeven point graphically,<br />

it may be easier to calculate the breakeven point numerically using engineering economy<br />

expressions for the PW or AW at the MARR. The AW is preferred when the variable units are<br />

expressed on a yearly basis, and AW calculations are simpler for alternatives with unequal lives.<br />

The following steps determine the breakeven point of the common variable and the slope of a<br />

linear total cost relation.<br />

1. Define the common variable and its dimensional units.<br />

2. Develop the PW or AW relation for each alternative as a function of the common variable.<br />

3. Equate the two relations and solve for the breakeven value of the variable.<br />

Selection between alternatives is based on this guideline:<br />

If the anticipated level of the common variable is below the breakeven value, select the alternative<br />

with the higher variable cost (larger slope).<br />

If the level is above the breakeven point, select the alternative with the lower variable cost.<br />

(Refer to Figure 13–5.)<br />

EXAMPLE 13.2<br />

A small aerospace company is evaluating two alternatives: the purchase of an automatic feed<br />

machine and a manual feed machine for a finishing process. The auto feed machine has an<br />

initial cost of $23,000, an estimated salvage value of $4000, and a predicted life of 10 years.<br />

One person will operate the machine at a rate of $12 per hour. The expected output is 8 tons per<br />

hour. Annual maintenance and operating cost is expected to be $3500.<br />

The alternative manual feed machine has a first cost of $8000, no expected salvage value, a<br />

5-year life, and an output of 6 tons per hour. However, three workers will be required at $8 per<br />

hour each. The machine will have an annual maintenance and operation cost of $1500. All<br />

projects are expected to generate a return of 10% per year. How many tons per year must be<br />

finished to justify the higher purchase cost of the auto feed machine?<br />

Solution<br />

Use the steps above to calculate the breakeven point between the two alternatives.<br />

1. Let x represent the number of tons per year.<br />

2. For the auto feed machine, the annual variable cost is<br />

Annual VC ——<br />

$12<br />

hour<br />

1.5x<br />

1 hour ———<br />

8 tons<br />

x tons ———<br />

year<br />

The VC is developed in dollars per year. The AW expression for the auto feed<br />

machine is<br />

AW auto 23,000(AP,10%,10) 4000(AF,10%,10) 3500 1.5x<br />

$–6992 1.5x


13.2 Breakeven Analysis Between Two Alternatives 347<br />

Similarly, the annual variable cost and AW for the manual feed machine are<br />

Annual VC ——<br />

$8<br />

hour<br />

4x<br />

(3 operators)<br />

1 hour ———<br />

6 tons<br />

x tons ———<br />

year<br />

AW manual 8000(AP,10%,5) 1500 4x<br />

$3610 4x<br />

3. Equate the two cost relations and solve for x.<br />

AW auto AW manual<br />

6992 1.5x 3610 – 4x<br />

x 1353 tons per year<br />

If the output is expected to exceed 1353 tons per year, purchase the auto feed machine, since<br />

its VC slope of 1.5 is smaller than the manual feed VC slope of 4.<br />

The breakeven analysis approach is commonly used for make-or-buy decisions . This means<br />

the company contracts to buy the product or service from the outside , or makes it within the company.<br />

The alternative to buy usually has no fixed cost and a larger variable cost than the option to<br />

make. Where the two cost relations cross is the make-buy decision quantity. Amounts above this<br />

indicate that the item should be made, not purchased outside.<br />

EXAMPLE 13.3<br />

Guardian is a national manufacturing company of home health care appliances. It is faced with<br />

a make-or-buy decision. A newly engineered lift can be installed in a car trunk to raise and<br />

lower a wheelchair. The steel arm of the lift can be purchased internationally for $3.50 per unit<br />

or made in-house. If manufactured on site, two machines will be required. Machine A is estimated<br />

to cost $18,000, have a life of 6 years, and have a $2000 salvage value; machine B will<br />

cost $12,000, have a life of 4 years, and have a $500 salvage value (carry-away cost). Machine<br />

A will require an overhaul after 3 years costing $3000. The annual operating cost for<br />

machine A is expected to be $6000 per year and for machine B is $5000 per year. A total of four<br />

operators will be required for the two machines at a rate of $12.50 per hour per operator. In a<br />

normal 8-hour period, the operators and two machines can produce parts sufficient to manufacture<br />

1000 units. Use a MARR of 15% per year to determine the following.<br />

(a) Number of units to manufacture each year to justify the in-house (make) option.<br />

(b) The maximum capital expense justifiable to purchase machine A, assuming all other estimates<br />

for machines A and B are as stated. The company expects to produce 10,000 units<br />

per year.<br />

Solution<br />

(a) Use steps 1 to 3 stated previously to determine the breakeven point.<br />

1. Define x as the number of lifts produced per year.<br />

2. There are variable costs for the operators and fixed costs for the two machines for the<br />

make option.<br />

Annual VC (cost per unit)(units per year)<br />

—————<br />

4 operators $12.50<br />

———(8 hours)x<br />

1000 units hour<br />

0.4x<br />

The annual fixed costs for machines A and B are the AW amounts.<br />

AW A 18,000(AP,15%,6) 2000(AF,15%,6)<br />

6000 3000(PF,15%,3)(AP,15%,6)<br />

AW B 12,000(AP,15%,4) 500(AF,15%,4) 5000<br />

Total cost is the sum of AW A , AW B , and VC.


348 Chapter 13 Breakeven and Payback Analysis<br />

3. Equating the annual costs of the buy option (3.50x) and the make option yields<br />

3.50x AW A AW B VC<br />

18,000(AP,15%,6) 2000(AF,15%,6) 6000<br />

3000(PF,15%,3)(AP,15%,6) 12,000(AP,15%,4)<br />

500(AF,15%,4) 5000 0.4x [13.5]<br />

3.10x 20,352<br />

x 6565 units per year<br />

A minimum of 6565 lifts must be produced each year to justify the make option, which<br />

has the lower variable cost of 0.4x.<br />

(b) Substitute 10,000 for x and P A for the to-be-determined first cost of machine A (currently<br />

$18,000) in Equation [13.5]. Solution yields P A $58,295. This is approximately three<br />

times the estimated first cost of $18,000, because the production of 10,000 per year is<br />

considerably larger than the breakeven amount of 6565.<br />

Even though the preceding examples treat only two alternatives, the same type of analysis can<br />

be performed for three or more alternatives. To do so, compare the alternatives in pairs to find<br />

their respective breakeven points. The results are the ranges through which each alternative is<br />

more economical. For example, in Figure 13–6, if the output is less than 40 units per hour, alternative<br />

1 should be selected. Between 40 and 60, alternative 2 is more economical; and above 60,<br />

alternative 3 is favored.<br />

If the variable cost relations are nonlinear, analysis is more complicated. If the costs increase<br />

or decrease uniformly, mathematical expressions that allow direct determination of the breakeven<br />

point can be developed.<br />

13.3 Payback Analysis<br />

Payback analysis is another use of the present worth technique. It is used to determine the amount<br />

of time, usually expressed in years, required to recover the first cost of an asset or project. Payback<br />

is allied with breakeven analysis; this is illustrated later in the section. The payback period ,<br />

also called payback or payout period , has the following definition and types.<br />

Alternative 1<br />

Alternative 2<br />

Alternative 3<br />

Total cost, $/year<br />

Breakeven<br />

points<br />

Figure 13–6<br />

Breakeven points for three alternatives.<br />

40<br />

60<br />

Output, units/hour


13.3 Payback Analysis 349<br />

The payback period n p is an estimated time for the revenues, savings, and any other monetary<br />

benefits to completely recover the initial investment plus a stated rate of return i.<br />

There are two types of payback analysis as determined by the required return.<br />

No return; i0%: Also called simple payback, this is the recovery of only the initial investment.<br />

Discounted payback; i 0%: The time value of money is considered in that some return, for<br />

example, 10% per year, must be realized in addition to recovering the initial investment.<br />

Payback period<br />

An example application of payback may be a corporate senior manager who insists that every<br />

proposal return the initial cost and some stated return within 3 years. Using payback as an initial<br />

screening tool, no proposal with n p 3 years can become a viable alternative. The payback period<br />

should be determined using a required i 0%. Unfortunately in practice, no-return payback<br />

is used too often to make economic decisions. After the formulas are presented, a couple of cautions<br />

about payback usage are provided.<br />

The equations used to determine n p differ for each type of analysis. For both types, the terminology<br />

is P for the initial investment in the asset, project, contract, etc., and NCF for the estimated<br />

annual net cash flow. Using Equation [1.5], annual NCF is<br />

NCF cash inflows cash outflows<br />

To calculate the payback period for i 0% or i 0%, determine the pattern of the NCF series.<br />

Note that n p is usually not an integer. For t 1, 2, . . . , n p ,<br />

No return, i 0%; NCF t varies annually:<br />

No return, i 0%; annual uniform NCF:<br />

tn p<br />

0 P NCF t [13.6]<br />

t1<br />

n p ——— P<br />

[13.7]<br />

NCF<br />

Discounted, i 0%; NCF t varies annually:<br />

tn p<br />

0 P NCF t (PF, i, t) [13.8]<br />

t1<br />

Discounted, i 0%; annual uniform NCF: 0 P NCF(PA, i, n p ) [13.9]<br />

After n p years, the cash flows will recover the investment in year 0 plus the required return of<br />

i %. If the alternative is used more than n p years, with the same or similar cash flows, a larger<br />

return results. If the estimated life is less than n p years, there is not enough time to recover the<br />

investment and i % return. It is important to understand that payback analysis neglects all cash<br />

fl ows after the payback period of n p years. Consequently, it is preferable to use payback as an<br />

initial screening method or supplemental tool rather than as the primary means to select an<br />

alternative. The reasons for this caution are that<br />

• No-return payback neglects the time value of money, since no return on an investment is required.<br />

• Either type of payback disregards all cash flows occurring after the payback period . These<br />

cash flows may increase the return on the initial investment.<br />

Payback analysis utilizes a significantly different approach to alternative evaluation than the<br />

primary methods of PW, AW, ROR, and BC. It is possible for payback analysis to select a different<br />

alternative than these techniques. However, the information obtained from discounted payback<br />

analysis performed at an appropriate i 0% can be very useful in that a sense of the risk<br />

involved in undertaking an alternative is provided. For example, if a company plans to utilize a<br />

machine for only 3 years and payback is 6 years, indication is that the equipment should not be<br />

obtained. Even here, the 6-year payback is considered supplemental information and does not<br />

replace a complete economic analysis.<br />

EXAMPLE 13.4<br />

The board of directors of Halliburton International has just approved an $18 million worldwide<br />

engineering construction design contract. The services are expected to generate new<br />

annual net cash flows of $3 million. The contract has a potentially lucrative repayment clause


350 Chapter 13 Breakeven and Payback Analysis<br />

to Halliburton of $3 million at any time that the contract is canceled by either party during the<br />

10 years of the contract period. (a) If i 15%, compute the payback period. (b) Determine<br />

the no-return payback period and compare it with the answer for i 15%. This is an initial<br />

check to determine if the board made a good economic decision. Show both hand and spreadsheet<br />

solutions.<br />

Solution by Hand<br />

(a) The net cash flow each year is $3 million. The single $3 million payment (call it CV for<br />

cancellation value) could be received at any time within the 10-year contract period.<br />

Equation [13.9] is altered to include CV.<br />

In $1,000,000 units,<br />

0 P NCF(PA,i,n) CV(PF,i,n)<br />

0 18 3(PA,15%,n) 3(PF,15%,n)<br />

The 15% payback period is n p 15.3 years, found by trail and error. During the period of<br />

10 years, the contract will not deliver the required return.<br />

(b) If Halliburton requires absolutely no return on its $18 million investment, Equation [13.6]<br />

results in n p 5 years, as follows (in $ million).<br />

0 18 5(3) 3<br />

There is a very significant difference in n p for 15% and 0%. At 15% this contract would<br />

have to be in force for 15.3 years, while the no-return payback period requires only 5 years.<br />

A longer time is always required for i 0% for the obvious reason that the time value of<br />

money is considered.<br />

Solution by Spreadsheet<br />

Enter the function NPER(15%,3,18,3) to display 15.3 years. Change the rate from 15% to<br />

0% to display the no-return payback period of 5 years.<br />

If two or more alternatives are evaluated using payback periods to indicate that one may<br />

be better than the other(s), the second shortcoming of payback analysis (neglect of cash<br />

flows after n p ) may lead to an economically incorrect decision. When cash flows that occur<br />

after n p are neglected, it is possible to favor short-lived assets even when longer-lived assets<br />

produce a higher return. In these cases, PW (or AW) analysis should always be the primary<br />

selection method. Comparison of short- and long-lived assets in Example 13.5 illustrates<br />

this situation.<br />

EXAMPLE 13.5<br />

Two equivalent pieces of quality inspection equipment are being considered for purchase by<br />

Square D Electric. Machine 2 is expected to be versatile and technologically advanced enough<br />

to provide net income longer than machine 1.<br />

Machine 1 Machine 2<br />

First cost, $ 12,000 8,000<br />

Annual NCF, $ 3,000 1,000 (years 1–5),<br />

3,000 (years 6–14)<br />

Maximum life, years 7 14<br />

The quality manager used a return of 15% per year and software that incorporates Equations<br />

[13.8] and [13.9] to recommend machine 1 because it has a shorter payback period of<br />

6.57 years at i 15%. The computations are summarized here.


13.3 Payback Analysis 351<br />

$3000 per year<br />

0 1 2 3 4 5 6 7<br />

Cash flow neglected<br />

by payback analysis<br />

Machine 1 n p = 6.57<br />

$12,000<br />

$1000 per year<br />

$3000 per year<br />

Cash flows neglected<br />

by payback analysis<br />

0 1 2 3 4 5 6 7 8 9 10 11 12 13 14<br />

Machine 2 n p = 9.52<br />

$8000<br />

Figure 13–7<br />

Illustration of payback periods and neglected net cash flows, Example 13.5.<br />

Machine 1: n p 6.57 years, which is less than the 7-year life.<br />

Equation used: 0 12,000 3000(PA,15%,n p )<br />

Machine 2: n p 9.52 years, which is less than the 14-year life.<br />

Equation used:<br />

Recommendation: Select machine 1.<br />

0 8000 1000(PA,15%,5)<br />

3000(PA,15%,n p 5)(PF,15%,5)<br />

Now, use a 15% PW analysis to compare the machines and comment on any difference in the<br />

recommendation.<br />

Solution<br />

For each machine, consider the net cash flows for all years during the estimated (maximum)<br />

life. Compare them over the LCM of 14 years.<br />

PW 1 12,000 12,000(PF,15%,7) 3000(PA,15%,14) $663<br />

PW 2 8000 1000(PA,15%,5) 3000(PA,15%,9)(PF,15%,5)<br />

$2470<br />

Machine 2 is selected since its PW value is numerically larger than that of machine 1 at 15%.<br />

This result is the opposite of the payback period decision. The PW analysis accounts for the<br />

increased cash flows for machine 2 in the later years. As illustrated in Figure 13–7 (for one life<br />

cycle for each machine), payback analysis neglects all cash flow amounts that may occur after<br />

the payback time has been reached.<br />

Comment<br />

This is a good example of why payback analysis is best used for initial screening and supplemental<br />

risk assessment. Often a shorter-lived alternative evaluated by payback analysis may<br />

appear to be more attractive, when the longer-lived alternative has cash flows later in its life<br />

that make it more economically attractive.<br />

As mentioned in the introduction to this section, breakeven and payback analyses are allied.<br />

They can be used in conjunction to determine the payback period when a desired level of breakeven<br />

is specified. The reverse is also possible; when a desired payback period is established, the<br />

breakeven value with or without a return requirement can be determined. By working together in<br />

this fashion, better economic decisions can be made. Example 13.6 illustrates the second of the<br />

situations mentioned above.


352 Chapter 13 Breakeven and Payback Analysis<br />

EXAMPLE 13.6<br />

The president of a local company expects a product to have a profitable life of between 1 and<br />

5 years. Help her determine the breakeven number of units that must be sold annually (without<br />

any return) to realize payback for each of the time periods 1 year, 2 years, and so on up to<br />

5 years. The cost and revenue estimates are as follows:<br />

Fixed costs: Initial investment of $80,000 with $1000 annual operating cost.<br />

Variable cost: $8 per unit.<br />

Revenue: Twice the variable cost for the first 5 years and 50% of the variable cost thereafter.<br />

Solution by Hand<br />

Define X BE as the breakeven quantity and n p as the payback period. Since values of X BE are<br />

sought for n p 1, 2, 3, 4, 5, solve for breakeven by substituting each payback period. First<br />

develop the FC, r, and v terms.<br />

80,000<br />

Fixed cost, FC<br />

———<br />

n 1000<br />

p<br />

Revenue per unit, r $16 (years 1 through 5 only)<br />

Variable cost per unit, v $8<br />

The breakeven relation from Equation [13.2] is<br />

X BE 80,000n p 1000<br />

————————<br />

[13.10]<br />

8<br />

Insert n p values and solve for X BE , the breakeven value.<br />

n p , payback years 1 2 3 4 5<br />

X BE , units per year 10,125 5125 3458 2625 2125<br />

Solution by Spreadsheet<br />

Figure 13–8 presents a spreadsheet solution for the breakeven values. Equation [13.10] is encoded<br />

to display the answers in column C. The breakeven values are the same as those above,<br />

e.g., sell 5125 units per year to pay back in 2 years. The breakeven curve rapidly flattens out as<br />

shown in the accompanying chart in Figure 13–8.<br />

-<br />

Figure 13–8<br />

Breakeven number of units for different payback periods, Example 13.6.<br />

13.4 More Breakeven and Payback<br />

Analysis on Spreadsheets<br />

The Goal Seek tool that we have used previously is an excellent tool to perform breakeven and payback<br />

analysis. Examples 13.7 and 13.8 demonstrate the use of Goal Seek for both types of problems.


13.4 More Breakeven and Payback Analysis on Spreadsheets 353<br />

EXAMPLE 13.7<br />

The Naruse brake-accelerator pedal (www.autoblog.comtagMasuyukiNaruse) is designed<br />

to minimize the chances that a driver will accidently step on the accelerator pedal of the car<br />

when the brake pedal is the intended target. The design is based on the fact that a person naturally<br />

steps downward on his or her foot when surprised, shocked, or struck with a medical<br />

emergency. In this pedal design, downward motion of the foot will always engage the brake,<br />

never the accelerator. Assume that for the manufacture of pedal components, two equally qualified<br />

machines have been identified and estimates made.<br />

Machine 1 Machine 2<br />

First cost, $ 80,000 110,000<br />

Net cash flow, $ per year 25,000 22,000<br />

Salvage value, $ 2,000 3,000<br />

Life, years 4 6<br />

Using an AW analysis at MARR 10%, the spreadsheet screen shot in Figure 13–9 indicates<br />

that machine 1 is the economic choice with a positive AW value of $193. However, the automated<br />

controls, safety features, and ergonomic design of machine 2 make it a better choice for<br />

the plant in the opinion of the project engineer. Use breakeven analysis to find the threshold<br />

values for each of several parameters that will make machine 2 equally qualified economically.<br />

The parameters to concentrate on are (a) first cost, (b) net cash flow, and (c) life of machine 2,<br />

if all other estimates remain the same.<br />

PMT($B$1,4,NPV($B1,B5:B8)+B4)<br />

Figure 13–9<br />

AW values for two machines, Example 13.7.<br />

Solution<br />

Figure 13–10 shows the spreadsheet and Goal Seek templates that determine breakeven values<br />

for first cost and NCF.<br />

(a) Figure 13–10a: By forcing the AW for machine 2 to equal $193, Goal Seek finds a breakeven<br />

of $96,669. If the first cost can be negotiated down to this cost from $110,000, machine<br />

2 will be economically equivalent to machine 1.<br />

(b) Figure 13–10b: (Remember to reset the first cost to $ –110,000 on the spreadsheet.) By setting<br />

all NCFs equal to the value in year 1 (using the function $C$5), Goal Seek determines<br />

a breakeven of $25,061 per year. Therefore, if the NCF estimate can realistically be<br />

increased from $22,000 to $25,061, again machine 2 will be economically equivalent.<br />

(c) Finding an extended life estimate for machine 2 is a payback question; Goal Seek is not<br />

needed. The easiest approach is to use the NPER function to find the payback period.<br />

Entering NPER(10%,22000,–110000,3000) displays n p 7.13 years. Therefore, extending<br />

the estimated life from 6 to between 7 and 8 years and retaining the salvage value<br />

of $3000 will select machine 2 over 1.


354 Chapter 13 Breakeven and Payback Analysis<br />

(a) First cost<br />

(b) NCF<br />

Figure 13–10<br />

Breakeven values for (a) first cost and (b) annual net cash flow using Goal Seek, Example 13.7.<br />

EXAMPLE 13.8<br />

Chris and her father just purchased a small office building for $160,000 that is in need of a lot<br />

of repairs, but is located in a prime commercial area of the city. The estimated costs each year<br />

for repairs, insurance, etc. are $18,000 the first year, increasing by $1000 per year thereafter.<br />

At an expected 8% per year return, use spreadsheet analysis to determine the payback period<br />

if the building is (a) kept for 2 years and sold for $290,000 sometime beyond year 2 or (b) kept<br />

for 3 years and sold for $370,000 sometime beyond 3 years.<br />

Solution<br />

Figure 13–11 shows the annual costs (column B) and the sales prices if the building is kept 2<br />

or 3 years (columns C and E, respectively). The NPV function is applied (columns D and F) to<br />

determine when the PW changes sign from plus to minus. These results bracket the payback<br />

period for each retention period and sales price. When PW 0, the 8% return is exceeded.<br />

(a) The 8% return payback period is between 3 and 4 years (column D). If the building is sold<br />

after exactly 3 years for $290,000, the payback period was not exceeded; but after 4 years<br />

it is exceeded.<br />

(b) At a sales price of $370,000, the 8% return payback period is between 5 and 6 years (column<br />

F). If the building is sold after 4 or 5 years, the payback is not exceeded; however, a<br />

sale after 6 years is beyond the 8%-return payback period.<br />

If kept 2 years and<br />

sold, payback is<br />

between 3 and 4<br />

NPV(8%,$B$4:B7)+$B$3 PV(8%,A7,,290000)<br />

If kept 3 years and<br />

sold, payback is<br />

between 5 and 6<br />

Figure 13–11<br />

Payback period analysis, Example 13.8


Problems 355<br />

CHAPTER SUMMARY<br />

The breakeven point for a variable for one project is expressed in terms such as units per year or<br />

hours per month. At the breakeven amount Q BE , there is indifference to accept or reject the project.<br />

Use the following decision guideline:<br />

Single Project (Refer to Figure 13–2.)<br />

Estimated quantity is larger than Q BE → accept project<br />

Estimated quantity is smaller than Q BE → reject project<br />

For two or more alternatives, determine the breakeven value of the common variable. Use the<br />

following guideline to select an alternative:<br />

Two Alternatives (Refer to Figure 13–5.)<br />

Estimated level is below breakeven → select alternative with higher<br />

variable cost (larger slope)<br />

Estimated level is above breakeven → select alternative with lower<br />

variable cost (smaller slope)<br />

Payback analysis estimates the number of years necessary to recover the initial investment plus<br />

a stated rate of return. This is a supplemental analysis technique used primarily for initial screening<br />

prior to a full evaluation by PW or some other method. The technique has some drawbacks,<br />

especially for no-return payback analysis, where i 0% is the stated return.<br />

PROBLEMS<br />

Breakeven Analysis for a Project<br />

13.1 A design-to-cost approach to product pricing involves<br />

determining the selling price of the product<br />

and then figuring out if it can be made at a cost lower<br />

than that. Banner <strong>Engineering</strong>’s QT50R radar-based<br />

sensor features frequency-modulated technology to<br />

accurately monitor or detect objects up to 15 miles<br />

away while resisting rain, wind, humidity, and extreme<br />

temperatures. It has a list price of $589, and<br />

the variable cost of manufacturing the unit is $340.<br />

( a) What could the company’s fixed cost per<br />

year be in order for Banner to break even<br />

with sales of 9000 units per year?<br />

( b) If Banner’s fixed cost is actually $750,000<br />

per year, what is the profit at a sales level of<br />

7000 units per year?<br />

13.2 Handheld fiber-optic meters with white light polarization<br />

interferometry are useful for measuring<br />

temperature, pressure, and strain in electrically<br />

noisy environments. The fixed costs associated<br />

with manufacturing are $800,000 per year. If a<br />

base unit sells for $2950 and its variable cost is<br />

$2075, ( a ) how many units must be sold each year<br />

for breakeven and ( b ) what will the profit be for<br />

sales of 3000 units per year?<br />

13.3 A metallurgical engineer has estimated that the<br />

capital investment cost for recovering valuable<br />

metals (nickel, silver, platinum, gold, etc.) from<br />

the copper refinery’s wastewater stream will be<br />

$12 million. The equipment will have a useful life<br />

of 15 years with no salvage value. Its operating<br />

cost is represented by the relation ($2,600,000) E 1.9 ,<br />

where E is the efficiency of the metal recovery operation<br />

(in decimal form). The amount of metal<br />

currently discharged is 2880 pounds per year prior<br />

to recovery operations, and the efficiency of<br />

recovery is estimated at 71%. What must the<br />

average selling price per pound be for the precious<br />

metals that are recovered and sold in order<br />

for the company to break even at its MARR of<br />

15% per year?<br />

Problems 13.4 through 13.7 are based on the following<br />

information.<br />

Hambry Enterprises produces a component for recycling<br />

uranium used as a nuclear fuel in power plant generators<br />

in France and the United States. Use the following cost<br />

and revenue figures, quoted in U.S. dollars per hundredweight<br />

(cwt), recorded for this year to calculate the answers<br />

for each plant.<br />

Location<br />

Fixed Cost,<br />

$ million<br />

Revenue,<br />

$ per cwt<br />

Cost,<br />

$ per cwt<br />

France 3.50 8,500 3,900<br />

United States 2.65 12,500 9,900


356 Chapter 13 Breakeven and Payback Analysis<br />

13.4 Determine the breakeven point for each plant.<br />

13.5 Estimate the minimum revenue per hundredweight<br />

required for next year if breakeven values and<br />

variable costs remain constant, but fixed costs<br />

increase by 10%.<br />

13.6 During this year, the French plant sold 950 units in<br />

Europe, and the U.S. plant sold 850 units. Determine<br />

the year’s profit (loss) for each plant.<br />

13.7 Hambry’s president has a goal of $1 million profit<br />

next year at each plant with no revenue or fixed<br />

cost increases. Determine the decreases in dollar<br />

amounts and percentages in variable cost necessary<br />

to meet this goal, if the number of units sold is<br />

the same as this year.<br />

13.8 The National Highway Traffic Safety Administration<br />

raised the average fuel efficiency standard to<br />

35.5 miles per gallon (mpg) for cars and light<br />

trucks by the year 2016. The rules will cost consumers<br />

an average of $926 extra per vehicle in the<br />

2016 model year. Assume a person purchases a<br />

new car in 2016 that gets 35.5 mpg and keeps it<br />

for 5 years. If the person drives an average of<br />

1000 miles per month and gets an extra 10 miles<br />

per gallon of gasoline, how much will the gasoline<br />

have to cost in order for the buyer to recover the<br />

extra investment in 5 years at an interest rate of<br />

0.75% per month?<br />

13.9 A call center in India used by U.S. and U.K. credit<br />

card holders has a capacity of 1,400,000 calls annually.<br />

The fixed cost of the center is $775,000<br />

with an average variable cost of $1 and revenue of<br />

$2.50 per call.<br />

(a) Find the percentage of the call capacity that<br />

must be placed each year to break even.<br />

(b) The center manager expects to dedicate the<br />

equivalent of 500,000 of the 1,400,000 capacity<br />

to a new product line. This is expected to<br />

increase the center’s fixed cost to $900,000 of<br />

which 50% will be allocated to the new product<br />

line. Determine the average revenue per<br />

call necessary to make 500,000 calls the<br />

breakeven point for only the new product.<br />

How does this required revenue compare with<br />

the current center revenue of $2.50 per call?<br />

13.10 The addition of a turbocharger to a small V-6 engine<br />

that gets 18 miles per gallon of gasoline can<br />

boost its power to that of a V-8 engine and increase<br />

fuel efficiency at the same time. If Bill will pay<br />

$800 to turbocharge his engine and his fuel efficiency<br />

increases by 3 miles per gallon, how many<br />

miles will he have to drive each month for 3 years<br />

in order to break even? Assume the cost of gasoline<br />

is $3.25 per gallon and the interest rate is 1%<br />

per month.<br />

13.11 Transporting extremely heavy patients (people<br />

who weigh more than 500 pounds) is much more<br />

difficult than transporting normal-weight patients.<br />

Various cities in Colorado, Nebraska, and Kansas<br />

charge $1421 for an extremely obese patient compared<br />

to $758 for a typical patient. The extra fees<br />

are justified by the ambulance companies on the<br />

basis of the specialty equipment required and the<br />

extra personnel involved. If it is assumed that<br />

50 extremely obese patients are transported every<br />

year, how much could the ambulance companies<br />

afford to spend on the specialty equipment now<br />

and break even on the initial cost in 5 years just<br />

from the extra charges? Assume the extra costs are<br />

$400 per patient and the company’s MARR is<br />

10% per year.<br />

13.12 High-profile vehicles have poor fuel efficiency because<br />

of increased wind resistance from their large<br />

front area. A number of companies make devices<br />

that they claim will significantly increase a vehicle’s<br />

fuel efficiency. One company claims that by<br />

spending $560 on its friction-reducing device, the<br />

fuel efficiency of a pickup truck will increase by<br />

25%. Assuming the device works as claimed, for a<br />

vehicle that currently gets 20 miles per gallon<br />

(mpg), how many miles would the owner have to<br />

drive each year to break even in 5 years? Assume<br />

the cost of gasoline is $3.50 per gallon and the<br />

interest rate is 10% per year.<br />

13.13 As the price of gasoline goes up, people are willing<br />

to drive farther to fill their tank in order to save<br />

money. Assume you had been buying gasoline for<br />

$2.90 per gallon and that it went up to $2.98 per<br />

gallon at the station where you usually go. If you<br />

drive an F-150 pickup that gets 18 miles per gallon,<br />

what is the round-trip distance you can drive<br />

to break even if it will take 20 gallons to fill your<br />

tank? Use an interest rate of 8% per year.<br />

13.14 An automobile company is investigating the advisability<br />

of converting a plant that manufactures<br />

economy cars into one that will make retro sports<br />

cars. The initial cost for equipment conversion<br />

will be $200 million with a 20% salvage value<br />

anytime within a 5-year period. The cost of producing<br />

a car will be $21,000, but it is expected to<br />

have a selling price of $33,000 to dealers. The<br />

production capacity for the first year will be<br />

4000 units. At an interest rate of 12% per year, by<br />

what uniform amount will production have to increase<br />

each year in order for the company to recover<br />

its investment in 3 years?


Problems 357<br />

13.15 For the last 2 years, The Health Company has<br />

experienced a fixed cost of $850,000 per year<br />

and an ( r v ) value of $1.25 per unit for its<br />

multivitamin line of products. International<br />

competition has become severe enough that<br />

some financial changes must be made to keep<br />

market share at the current level.<br />

( a) Perform a spreadsheet-based graphical analysis<br />

to estimate the effect on the breakeven<br />

point if the difference between revenue and<br />

variable cost per unit increases somewhere<br />

between 1% and 15% of its current value.<br />

( b) If fixed costs and revenue per unit remain at<br />

their current values, what type of change<br />

must take place to make the breakeven point<br />

go down?<br />

13.16 (This is an extension of Problem 13.15) Expand<br />

the analysis performed in Problem 13.15 by changing<br />

the variable cost per unit. The financial manager<br />

estimates that fixed costs will fall to $750,000<br />

when the required production rate to break even is<br />

at or below 600,000 units. What happens to the<br />

breakeven points over the ( r v ) range of 1% to<br />

15% increase as evaluated previously?<br />

Breakeven Analysis Between Alternatives<br />

13.17 Providing restrooms at parks, zoos, and other cityowned<br />

recreation facilities is a considerable expense<br />

for municipal governments. City councils<br />

usually opt for permanent restrooms in larger<br />

parks and portable restrooms in smaller ones. The<br />

cost of renting and servicing a portable restroom<br />

is $7500 per year. In one northeastern municipality,<br />

the parks director informed the city council<br />

that the cost of constructing a permanent restroom<br />

is $218,000 and the annual cost of maintaining it<br />

is $12,000. He remarked that the rather high cost<br />

is due to the necessity to use expensive materials<br />

and construction techniques that are tailored to<br />

minimize damage from vandalism that often occurs<br />

in unattended public facilities. If the useful<br />

life of a permanent restroom is assumed to be 20<br />

years, how many portable restrooms could the<br />

city afford to rent each year and break even with<br />

the cost of one permanent facility? Let the interest<br />

rate be 6% per year.<br />

13.18 A consulting engineer is considering two methods<br />

for lining ponds used for evaporating concentrate<br />

generated during reverse osmosis treatment of<br />

brackish groundwater for the Clay County Industrial<br />

Park. A geosynthetic bentonite clay liner<br />

(GCL) will cost $1.8 million to install, and if it is<br />

renovated after 4 years at a cost of $375,000, its<br />

life can be extended another 2 years. Alternatively,<br />

a high-density polyethylene (HDPE) geomembrane<br />

can be installed that will have a useful life of<br />

12 years. At an interest rate of 6% per year, how<br />

much money can be spent on the HDPE liner for<br />

the two methods to break even?<br />

13.19 An irrigation canal contractor wants to determine<br />

whether he should purchase a used Caterpillar<br />

mini excavator or a Toro powered rotary tiller for<br />

servicing irrigation ditches in an agricultural area<br />

of California. The initial cost of the excavator is<br />

$26,500 with a $9000 salvage value after 10 years.<br />

Fixed costs for insurance, license, etc. are expected<br />

to be $18,000 per year. The excavator will require<br />

one operator at $15 per hour and maintenance at<br />

$1 per hour. In 1 hour, 0.15 mile of ditch can be<br />

prepared. Alternatively, the contractor can purchase<br />

a tiller and hire 2 workers at $11 per hour<br />

each. The tiller costs $1200 and has a useful life of<br />

5 years with no salvage value. Its operating cost is<br />

expected to be $1.20 per hour, and with the tiller,<br />

the two workers can prepare 0.04 mile of ditch in<br />

1 hour. The contractor’s MARR is 10% per year.<br />

Determine the number of miles of ditch per year<br />

the contractor would have to service for the two<br />

options to break even.<br />

13.20 An effective method to recover water used for regeneration<br />

of ion exchange resins is to use a reverse<br />

osmosis system in a batch treatment mode.<br />

Such a system involves recirculation of the partially<br />

treated water back into the feed tank, causing<br />

the water to heat up. The water can be cooled<br />

using one of two systems: a single-pass heat exchanger<br />

or a closed-loop heat exchange system.<br />

The single-pass system, good for 3 years, requires<br />

a small chiller costing $920 plus stainless steel<br />

tubing, connectors, valves, etc. costing $360. The<br />

cost of water, treatment charges, electricity, etc.<br />

will be $3.10 per hour. The closed-loop system<br />

will cost $3850 to buy, will have a useful life of<br />

5 years, and will cost $1.28 per hour to operate.<br />

What is the minimum number of hours per year<br />

that the cooling system must be used in order to<br />

justify purchase of the closed-loop system? The<br />

MARR is 10% per year, and the salvage values<br />

are negligible.<br />

13.21 Samsung Electronics is trying to reduce supply<br />

chain risk by making more responsible make-buy<br />

decisions through improved cost estimation. A<br />

high-use component (expected usage is 5000 units<br />

per year) can be purchased for $25 per unit with<br />

delivery promised within a week. Alternatively,<br />

Samsung can make the component in-house and<br />

have it readily available at a cost of $5 per unit, if<br />

equipment costing $150,000 is purchased. Labor


358 Chapter 13 Breakeven and Payback Analysis<br />

and other operating costs are estimated to be<br />

$35,000 per year over the study period of 5 years.<br />

Salvage is estimated at 10% of first cost and<br />

i 12% per year. Neglect the element of availability<br />

( a ) to determine the breakeven quantity<br />

and ( b ) to recommend making or buying at the expected<br />

usage level.<br />

13.22 A partner in a medium-size A/E (architecturalengineering)<br />

design firm is evaluating two alternatives<br />

for improving the exterior appearance of the building<br />

they occupy. The building can be completely<br />

painted at a cost of $6500. The paint is expected to<br />

remain attractive for 4 years, at which time repainting<br />

will be necessary. Every time the building is<br />

repainted, the cost will be 20% higher than the previous<br />

time. Alternatively, the building can be sandblasted<br />

now and every 6 years at a cost 40% greater<br />

than the previous time. If the company’s MARR is<br />

10% per year, what is the maximum amount that<br />

could be spent now on the sandblasting alternative<br />

that would render the two alternatives indifferent<br />

over a study period of 12 years?<br />

13.23 A junior mechanical engineering student is cooping<br />

this semester at Regency Aircraft, which<br />

customizes the interiors of private and corporate<br />

jets. Her first assignment is to develop the specifications<br />

for a new machine to cut, shape, and sew<br />

leather or vinyl covers and trims. The first cost is<br />

not easy to estimate due to many options, but the<br />

annual revenue and M&O costs should net out at<br />

$15,000 per year over a 10-year life. Salvage is<br />

expected to be 20% of the first cost. Determine the<br />

breakeven first cost of the machine to just recover<br />

its first cost and a return of 8% per year under two<br />

scenarios:<br />

I: No outside revenue will be developed by<br />

the machine.<br />

II: Outside contracting will occur with estimated<br />

revenue of $10,000 the first year, increasing<br />

by $5000 per year thereafter.<br />

Solve using ( a ) hand and ( b ) spreadsheet solutions.<br />

13.24 Ascarate Fishing Club (a nonprofit organization<br />

dedicated to teaching kids how to fish) is considering<br />

two options for providing a heavily stocked<br />

pond for kids who have never caught a fish before.<br />

Option 1 is an above-ground swimming pool made<br />

of heavy vinyl plastic that will be assembled and<br />

disassembled for each quarterly event. The purchase<br />

price will be $400. Leaks from hooks piercing<br />

the fabric will be repaired with a vinyl repair<br />

kit at a cost of $70 per year, but the pool will have<br />

to be replaced when too many repairs have been<br />

made.<br />

Option 2 is an in-ground pond that will be excavated<br />

by club members at no cost and lined with<br />

fabric that costs $1 per square foot. The pond will<br />

be 15 ft in diameter and 3 ft deep. Assume 300 ft 2<br />

of liner will be purchased. A chain link fence at<br />

$10 per lineal foot will be installed around the<br />

pond (100 ft of fence). Maintenance inside the<br />

fence is expected to cost $20 per year. The park<br />

where the pond will be constructed has committed<br />

the land for only 10 years. At an interest rate of 6%<br />

per year, how long would the above-ground pool<br />

have to last to break even?<br />

13.25 A rural subdivision has several miles of access<br />

roads that need a new surface treatment. Alternative<br />

1 is a gravel base and pavement with an initial<br />

cost of $500,000 that will last for 15 years<br />

and has an annual upkeep cost of $100 per mile.<br />

Alternative 2 is to enhance the gravel base now at<br />

a cost of $50,000 and immediately coat the surface<br />

with a durable hot oil mix, which costs $130<br />

per barrel applied. Annual reapplication of the<br />

mix is required. A barrel covers 0.05 mile. ( a ) If<br />

the discount rate is 6% per year, determine the<br />

number of miles at which the two alternatives<br />

break even. ( b ) A drive in a pickup indicates a<br />

total of 12.5 miles of road. Which is the more<br />

economical alternative?<br />

13.26 A waste-holding lagoon situated near the main<br />

plant receives sludge daily. When the lagoon is<br />

full, it is necessary to remove the sludge to a site<br />

located 8.2 kilometers from the main plant. Currently,<br />

when the lagoon is full, the sludge is removed<br />

by pump into a tank truck and hauled away.<br />

This process requires the use of a portable pump<br />

that initially costs $800 and has an 8-year life. The<br />

company pays a contract individual to operate the<br />

pump and oversee environmental and safety factors<br />

at a rate of $100 per day, plus the truck and<br />

driver must be rented for $200 per day. The company<br />

has the option to install a pump and pipeline<br />

to the remote site. The pump would have an initial<br />

cost of $1600 and a life of 10 years and will cost<br />

$3 per day to operate. The company’s MARR is<br />

10% per year.<br />

(a)<br />

If the pipeline will cost $12 per meter to construct<br />

and will have a 10-year life, how many<br />

days per year must the lagoon require pumping<br />

to justify construction of the pipeline?<br />

(b) If the company expects to pump the lagoon<br />

once per week every week of the year, how<br />

much money can it afford to spend now on<br />

the 10-year life pipeline to just break even?<br />

13.27 Lorraine can select from two nutrient injection<br />

systems for her cottage industry of hydroponic


Problems 359<br />

tomato and lettuce greenhouses. ( a ) Use an AW<br />

relation to determine the minimum number of<br />

hours per year to operate the pumps that will justify<br />

the Auto Green system, if the MARR is 10%<br />

per year. ( b ) Which pump is economically better if<br />

it operates 7 hours per day, 365 days per year?<br />

Nutra Jet (N) Auto Green (A)<br />

Initial cost, $ 4,000 10,300<br />

Life, years 3 6<br />

Rebuild cost, $ 1,000 2,200<br />

Time before rebuild, annually 2,000 8,000<br />

or minimum hours<br />

Cost to operate, $ per hour 1.00 0.90<br />

13.28 An engineering practitioner can lease a fully<br />

equipped computer and color printer system for<br />

$800 per month or purchase one for $8500 now and<br />

pay a $75 per month maintenance fee. If the nominal<br />

interest rate is 15% per year, determine the<br />

months of use necessary for the two to break even.<br />

Show both ( a ) hand and ( b ) spreadsheet solutions.<br />

13.29 The office manager of an environmental engineering<br />

consulting firm was instructed to make an ecofriendly<br />

decision in acquiring an automobile for<br />

general office use. He is considering a gasolineelectric<br />

hybrid or a gasoline-free, all-electric<br />

hatchback. The hybrid under consideration is<br />

GM’s Volt, which will cost $35,000, will have a<br />

salvage value of $15,000 after 5 years, and will<br />

have a range of 40 miles on the electric battery,<br />

plus several hundred more miles when the gasoline<br />

engine kicks in. Nissan’s Leaf, on the other<br />

hand, is a pure electric that will have a range of<br />

only 100 miles, after which its lithium ion battery<br />

must be recharged. The Leaf’s relatively limited<br />

range creates a psychological effect known as<br />

range anxiety (RA), which has the company leaning<br />

toward purchasing the Volt. The Leaf can be<br />

leased for $349 per month after an initial $500<br />

down payment.<br />

The accountant for the consulting firm told the<br />

office manager that the Leaf is the better economic<br />

option based on an evaluation she performed earlier.<br />

If the office manager purchases the Volt anyway<br />

(instead of leasing the Leaf), what is the<br />

monthly equivalent (AW value) of the extra<br />

amount of money the company will be paying to<br />

eliminate range anxiety? Assume the operating<br />

costs will be the same for both vehicles and the<br />

MARR is 0.75% per month.<br />

Payback Analysis<br />

13.30 How long will you have to sell a product that has<br />

an income of $5000 per month and expenses of<br />

$1500 per month if your initial investment is<br />

$28,000 and your MARR is ( a ) 0% and ( b ) 3% per<br />

month? Solve by formula. ( c ) Write the spreadsheet<br />

functions to display the payback period for<br />

both 0% and 3% per month.<br />

13.31 (a) Determine the payback period at an interest<br />

rate of 8% per year for an asset that initially<br />

cost $28,000, has a scrap value of $1500<br />

whenever it is sold, and generates cash flow<br />

of $2900 per year.<br />

(b) If the asset will be in service for 12 years,<br />

should it be purchased?<br />

13.32 ABB purchased fieldbus communication equipment<br />

for a project in South Africa for $3.15 million.<br />

The net cash flow is estimated at $500,000<br />

per year, and a salvage value of $400,000 is anticipated<br />

regardless of when it is sold. Determine the<br />

number of years the equipment must be used to<br />

obtain payback at MARR values of ( a ) 0% and 8%<br />

per year and ( b ) 15% and 16% per year. ( c ) Use a<br />

spreadsheet to plot the payback years for all four<br />

return values.<br />

13.33 Sundance Detective Agency purchased new surveillance<br />

equipment with the following estimates.<br />

The year index is k 1, 2, 3, . . . .<br />

First cost $1050<br />

Annual maintenance cost $70 5 k per year<br />

Extra annual revenue $200 50 k per year<br />

Salvage value<br />

$600 for all years<br />

( a) Calculate the payback period to make a return<br />

of 10% per year.<br />

( b) For a preliminary conclusion, should the<br />

equipment be purchased if the actual useful<br />

life is 7 years?<br />

13.34 Clarisa, an engineering manager, wants to purchase<br />

a resort accommodation to rent to skiers.<br />

She is considering the purchase of a three- bedroom<br />

lodge in upper Montana that will cost $250,000.<br />

The property in the area is rapidly appreciating in<br />

value because people anxious to get away from<br />

urban developments are bidding up the prices. If<br />

Clarisa spends an average of $500 per month for<br />

utilities and the investment increases at a rate of<br />

2% per month, how long would it be before she<br />

could sell the property for $100,000 more than she<br />

has invested in it?<br />

13.35 Laura’s grandparents helped her purchase a small<br />

self-serve laundry business to make extra money<br />

during her 5 college years. When she completed<br />

her engineering management degree, she sold the<br />

business and her grandparents told her to keep the


360 Chapter 13 Breakeven and Payback Analysis<br />

money as a graduation present. For the net cash<br />

flows listed below, determine the following:<br />

(a) The percentage of the investment recovered<br />

during the 5 years<br />

(b) The actual rate of return over the 5-year<br />

period<br />

(c) How long it took to pay back the $75,000 investment<br />

in year 0, plus a 7% per year return<br />

Year 0 1 2 3 4 5<br />

NCF, $ per year 75,000 10,500 18,600 2000 28,000 105,000<br />

13.36 Buhler Tractor sold a tractor for $45,000 to Tom<br />

Edwards 10 years ago. ( a ) What is the uniform net<br />

cash flow that Tom must make each year to realize<br />

payback and a return of 5% per year on his investment<br />

over a period of 3 years? 5 years? 8 years?<br />

All 10 years? ( b ) If the net cash flow was actually<br />

$5000 per year, what is the amount Tom should<br />

have paid for the tractor to realize payback plus the<br />

5% per year return over these 10 years?<br />

13.37 National Parcel Service has historically owned and<br />

maintained its own delivery trucks. Leasing is an<br />

option being seriously considered because costs<br />

for maintenance, fuel, insurance, and some liability<br />

issues will be transferred to Pacific Leasing, the<br />

truck leasing company. The study period is no<br />

more than 24 months for either alternative. The annual<br />

lease cost is paid at the beginning of each year<br />

and is not refundable for partially used years. Use<br />

the first cost and net cash flow estimates to determine<br />

the payback in months with a nominal 9%<br />

per year return for the ( a ) purchase option and<br />

( b ) lease option.<br />

Purchase:<br />

Lease:<br />

P $– 30,000 now<br />

Monthly cost $1000<br />

Monthly revenue $4500<br />

P $10,000 at the beginning<br />

of each year (months 0 and 12)<br />

Monthly cost $−2500<br />

Monthly revenue $4500<br />

13.38 Julian Browne, owner of Clear Interior Environments,<br />

purchased an air scrubber, HEPA vacuum,<br />

and other equipment for mold removal for $15,000<br />

eight months ago. Net cash flows were $−2000 for<br />

each of the first 2 months, followed by $1000 per<br />

month for months 3 and 4. For the last 4 months, a<br />

contract generated a net $6000 per month. Julian<br />

sold the equipment yesterday for $3000 to a friend.<br />

Determine ( a ) the no-return payback period and<br />

( b ) the nominal 18%-per-year payback period.<br />

13.39 Explain why payback analysis may favor an alternative<br />

with a shorter payback period when it is not<br />

the better choice economically.<br />

13.40 When comparing two alternatives, why is it best to<br />

use no-return payback analysis as a preliminary<br />

screening tool prior to conducting a complete PW<br />

or AW evaluation?<br />

Spreadsheet Problems<br />

13.41 Benjamin used regression analysis to fit quadratic<br />

relations to monthly revenue and cost data with the<br />

following results:<br />

R 0.007 Q 2 32 Q<br />

TC 0.004 Q 2 2.2 Q 8<br />

( a) Plot R and TC. Estimate the quantity Q p at<br />

which the maximum profit should occur.<br />

Estimate the amount of profit at this quantity.<br />

(b) The profit relation P R TC and calculus<br />

can be used to determine the quantity Q p at<br />

which the maximum profit will occur and the<br />

amount of the profit. The equations are<br />

Profit aQ 2 bQ c<br />

Q p —— b<br />

2a<br />

Maximum profit b2 ——<br />

4a c<br />

Use these relations to confirm the graphical<br />

estimates you made in ( a ). (Your instructor<br />

may ask you to derive the relations above.)<br />

13.42 The National Potato Cooperative purchased a deskinning<br />

machine last year for $150,000. Revenue<br />

for the first year was $50,000. Over the total estimated<br />

life of 8 years, what must the remaining<br />

equivalent annual revenues (years 2 through 8)<br />

equal to break even by recovering the investment<br />

and a return of 10% per year? Costs are expected<br />

to be constant at $42,000 per year, and a salvage<br />

value of $20,000 is anticipated.<br />

Problems 13.43 and 13.44 are based on the following<br />

information.<br />

Wilson Partners manufactures thermocouples for electronics<br />

applications. The current system has a fixed cost of $300,000<br />

per year and a variable cost of $10 per unit. Wilson sells the<br />

units for $14 each. A newly proposed process will add onboard<br />

features that allow the revenue to increase to $16 per<br />

unit, but the fixed cost will now be $500,000 per year. The<br />

variable cost of the new system will be based on a $48 per<br />

hour rate with 0.2 hour required to produce each unit.<br />

13.43 Determine the annual breakeven quantity for<br />

( a ) the current system and ( b ) the new system.<br />

13.44 Plot the two profit relations and estimate graphically<br />

the breakeven quantity between the two<br />

alternatives.


Additional Problems and FE Exam Review Questions 361<br />

Problems 13.45 through 13.48 are based on the following<br />

information.<br />

Mid-Valley Industrial Extension Service, a state- sponsored<br />

agency, provides water quality sampling services to all<br />

business and industrial firms in a 10-county region. Just<br />

last month, the service purchased all necessary lab equipment<br />

for full in-house testing and analysis. Now an outsourcing<br />

agency has offered to take over this function on a<br />

per sample basis. Data and quotes for the two options have<br />

been collected. The MARR for government projects is 5%<br />

per year, and a study period of 8 years is chosen.<br />

In-house: Equipment and supplies initially cost<br />

$125,000 for a life of 8 years, an AOC of<br />

$15,000, and annual salaries of $175,000.<br />

Sample costs average $25 each. There is<br />

no significant market value for the equipment<br />

and supplies currently owned.<br />

Outsourced: Contractors quote sample cost averages<br />

of $100 for the first 5 years, increasing to<br />

$125 per sample for years 6 through 8.<br />

13.45 Determine the breakeven number of tests between<br />

the two options.<br />

13.46 Use a spreadsheet to graph the AW curves for both<br />

options for test loads between 0 and 4000 per year<br />

in increments of 1000 tests. What is the estimated<br />

breakeven quantity?<br />

13.47 The service director has asked the outsource company<br />

to reduce the per sample costs by 25% across<br />

the board over the 8-year study period. What will this<br />

do to the breakeven point? ( Hint : Look carefully at<br />

your graph from Problem 13.46 before answering.)<br />

13.48 Assume the Extension Service can reduce its annual<br />

salaries from $175,000 to $100,000 per<br />

year and the per sample cost from $25 to $20.<br />

What will this do to the breakeven point? ( Hint :<br />

Again, look carefully at your graph from the<br />

previous problem before answering.) What is<br />

the new annual breakeven test quantity?<br />

ADDITIONAL PROBLEMS AND FE EXAM REVIEW QUESTIONS<br />

13.49 In linear breakeven analysis, if a company expects<br />

to operate at a point above the breakeven point, it<br />

should select the alternative:<br />

( a) With the lower fixed cost<br />

( b) With the higher fixed cost<br />

( c) With the lower variable cost<br />

( d) With the higher variable cost<br />

13.50 A company is considering two alternatives to automate<br />

the pH of process liquids. Alternative A will<br />

have fixed costs of $42,000 per year and will require<br />

2 workers at $48 per day each. Together,<br />

these workers can generate 100 units of product<br />

per day. Alternative B will have fixed costs of<br />

$56,000 per year, but with this alternative, 3 workers<br />

will generate 200 units of product. If x is the<br />

number of units per year, the variable cost (VC) in<br />

$ per year for alternative B is represented by:<br />

( a) [2(48)100] x<br />

( b) [3(48)200] x<br />

( c) [3(48)200] x 56,000<br />

( d) [2(48)100] x 42,000<br />

13.52 AW 1 23,000( AP,10%,10) 4000( A F ,10%,10)<br />

3000 3 x<br />

AW 2 8,000(AP,10%,4) 2000 − 6 x<br />

For these two AW relations, the breakeven point x ,<br />

in miles per year, is closest to:<br />

( a ) 1130<br />

( b ) 1224<br />

( c ) 1590<br />

( d ) 655<br />

13.53 To make an item in-house, equipment costing<br />

$250,000 must be purchased. It will have a life of<br />

4 years, an annual cost of $80,000, and each unit<br />

will cost $40 to manufacture. Buying the item externally<br />

will cost $100 per unit. At i 15% per<br />

year, it is cheaper to make the item in-house if the<br />

number per year needed is:<br />

( a) Above 1047 units<br />

( b) Above 2793 units<br />

( c) Equal to 2793 units<br />

( d) Below 2793 units<br />

13.51 When the variable cost is reduced for linear total<br />

cost and revenue lines, the breakeven point decreases.<br />

This is an economic advantage because:<br />

( a) The revenue per unit will increase.<br />

( b) The two lines will now cross at zero.<br />

( c) The profit will increase for the same revenue<br />

per unit.<br />

( d) The total cost line becomes nonlinear.<br />

13.54 A procedure at Mercy Hospital has fixed costs of<br />

$10,000 per year and variable costs of $50 per test.<br />

If the procedure is automated, its fixed cost will be<br />

$21,500 per year, but its variable cost will be only<br />

$10 per test. The number of tests that must be performed<br />

each year for the two operations to break<br />

even is closest to:<br />

( a ) 290 ( b ) 455 ( c ) 750 ( d ) Over 800


362 Chapter 13 Breakeven and Payback Analysis<br />

13.55 An assembly process can be completed using<br />

either alternative X or Y. Alternative X has fixed<br />

costs of $10,000 per year with a variable cost of<br />

$50 per unit. If the process is automated per alternative<br />

Y, its fixed cost will be $5000 per year<br />

and its variable cost will be only $10 per unit.<br />

The number of units that must be produced each<br />

year in order for alternative Y to be favored is<br />

closest to:<br />

(a) Y will be favored for any level of production<br />

(b) 125<br />

(c) 375<br />

(d) X will be favored for any level of production<br />

13.56 Two different methods are under consideration<br />

for building a bypass road. Material C will cost<br />

$100,000 per mile and last for 10 years. Its annual<br />

maintenance cost will be $10,000 per year<br />

per mile. Material D will cost $30,000 per mile<br />

and last for 5 years. At an interest rate of 6% per<br />

year, the annual maintenance cost for material D<br />

that will make the two methods cost the same is<br />

closest to:<br />

( a ) Less than $14,000 ( b ) $14,270<br />

( c ) $16,470 ( d ) $19,510<br />

13.57 A construction company can purchase a piece of<br />

equipment for $50,000 and spend $100 per day in<br />

operating costs. The equipment will have a 5-year<br />

life with no salvage value. Alternatively, the company<br />

can lease the equipment for $400 per day.<br />

The number of days per year the company must<br />

require the equipment to justify its purchase at an<br />

interest rate of 8% per year is closest to:<br />

( a ) 10 days ( b ) 42 days<br />

( c ) 51 days ( d ) 68 days<br />

13.58 A tractor has a first cost of $40,000, a monthly operating<br />

cost of $1500, and a salvage value of<br />

$12,000 in 10 years. The MARR is 12% per year.<br />

An identical tractor can be rented for $3200 per<br />

month (operating cost not included). If n is the<br />

minimum number of months per year the tractor<br />

must be used in order to justify its purchase, the<br />

relation to find n is represented by:<br />

(a) 40,000( A P ,1%,10) 1500 n<br />

12,000( A F ,1%,10) 3200 n<br />

(b) 40,000( A P ,12%,10) 1500 n<br />

12,000( A F ,12%,10) 3200 n<br />

(c) 40,000( A P ,1%,120) 1500 n<br />

12,000( A F ,1%,120) 3200 n<br />

(d) 40,000( A P ,11.4%,10) 1500 n<br />

12,000( A F ,11.4%,10) 3200 n<br />

13.59 An anticorrosive coating for a chemical storage<br />

tank will cost $5000 and last 5 years if touched<br />

up at the end of 3 years at a cost of $1000. If an<br />

oil-base enamel coating could be used that will<br />

last 2 years, the amount the enamel coating can<br />

cost for the two to breakeven at i 8% per year<br />

is closest to:<br />

( a ) $2120 ( b ) $2390<br />

( c ) $2590 ( d ) $2725<br />

13.60 The price of a car is $50,000 today. Its price is expected<br />

to increase by $2400 each year. You now<br />

have $25,000 in an investment that is earning 20%<br />

per year. The number of years before you have<br />

enough money to buy the car, without borrowing<br />

any money, is closest to:<br />

( a ) 3 years ( b ) 5 years<br />

( c ) 7 years ( d ) 9 years<br />

13.61 Process A has a fixed cost of $16,000 per year and<br />

a variable cost of $40 per unit. For process B,<br />

5 units can be produced in 1 day at a cost of $125.<br />

If the company’s MARR is 10% per year, the fixed<br />

cost of process B that will make the two alternatives<br />

have the same annual cost at a production<br />

rate of 1000 units per year is closest to:<br />

(a) Less than $10,000<br />

(b) $18,000<br />

(c) $27,000<br />

(d) Over $30,000<br />

13.62 The profit relation for the following estimates at a<br />

quantity that is 20% above breakeven is:<br />

Fixed cost $500,000 per year<br />

Variable cost per unit $200<br />

Revenue per unit $250<br />

(a) Profit 200(12,000) 250(12,000) 500,000<br />

(b) Profit 250(12,000) 500,000 200 (12,000)<br />

(c) Profit 250(12,000) 200(12,000) 500,000<br />

(d) Profit 250(10,000) 200(10,000) 500,000<br />

13.63 Two methods of weed control in an irrigation canal<br />

are under consideration. Method A involves lining<br />

at a cost of $4000. The lining will last 20 years.<br />

The maintenance cost with this method will be<br />

$3 per mile per year. Method B involves spraying<br />

a chemical that costs $40 per gallon. One gallon<br />

will treat 8 miles, but the treatment must be applied<br />

4 times per year. In determining the number of<br />

miles per year that would result in breakeven, the<br />

variable cost for method B is closest to:<br />

( a ) $5 per mile ( b ) $15 per mile<br />

( c ) $20 per mile ( d ) $40 per mile<br />

13.64 How long will you have to maintain a business that<br />

has an income of $5000 per year and expenses of<br />

$1500 per year if your initial investment was<br />

$28,000 and your MARR is 10% per year?<br />

( a ) Less than 6 years ( b ) 8 years<br />

( c ) 12 years ( d ) 17 years


Case Study 363<br />

CASE STUDY<br />

WATER TREATMENT PLANT PROCESS COSTS<br />

Background<br />

Aeration and sludge recirculation have been practiced for<br />

many years at municipal and industrial water treatment<br />

plants. Aeration is used primarily for the physical removal of<br />

gases or volatile compounds, while sludge recirculation can<br />

be beneficial for turbidity removal and hardness reduction.<br />

When the advantages of aeration and sludge recirculation<br />

in water treatment were first recognized, energy costs were so<br />

low that such considerations were seldom of concern in treatment<br />

plant design and operation. With the huge increases in<br />

electricity cost that have occurred in some localities, however,<br />

it became necessary to review the cost-effectiveness of<br />

all water treatment processes that consume significant<br />

amounts of energy. This study was conducted at a municipal<br />

water treatment plant for evaluating the cost-effectiveness of<br />

the pre-aeration and sludge recirculation practices.<br />

Information<br />

This study was conducted at a 106 m 3 per minute watertreatment<br />

plant where, under normal operating circumstances,<br />

sludge from the secondary clarifiers is returned to<br />

the aerator and subsequently removed in the primary clarifiers.<br />

Figure 13–12 is a schematic of the process.<br />

To evaluate the effect of sludge recirculation, the sludge<br />

pump was turned off, but aeration was continued. Next, the<br />

sludge pump was turned back on, and aeration was discontinued.<br />

Finally, both processes were discontinued. Results obtained<br />

during the test periods were averaged and compared to<br />

the values obtained when both processes were operational.<br />

The results obtained from the four operating modes<br />

showed that the hardness decreased by 4.7% when both processes<br />

were in operation (i.e., sludge recirculation and aeration).<br />

When only sludge was recirculated, the reduction was<br />

3.8%. There was no reduction due to aeration only, or when<br />

there was neither aeration nor recirculation. For turbidity, the<br />

reduction was 28% when both recirculation and aeration<br />

were used. The reduction was 18% when neither aeration nor<br />

recirculation was used. The reduction was also 18% when<br />

aeration alone was used, which means that aeration alone was<br />

of no benefit for turbidity reduction. With sludge recirculation<br />

alone, the turbidity reduction was only 6%, meaning that<br />

sludge recirculation alone actually resulted in an increase in<br />

turbidity—the difference between 18% and 6%.<br />

Since aeration and sludge recirculation did cause readily<br />

identifiable effects on treated water quality (some good and<br />

others bad), the cost-effectiveness of each process for turbidity<br />

and hardness reduction was investigated. The calculations<br />

are based on the following data:<br />

Aerator motor 40 hp<br />

Aerator motor efficiency 90%<br />

Sludge recirculation motor 5 hp<br />

Recirculation pump efficiency 90%<br />

Electricity cost 9 ¢kWh (previous analysis)<br />

Lime cost 7.9 ¢kg<br />

Lime required 0.62 mgL per mgL hardness<br />

Coagulant cost 16.5 ¢kg<br />

Daysmonth 30.5<br />

As a first step, the costs associated with aeration and sludge<br />

recirculation were calculated. In each case, costs are independent<br />

of flow rate.<br />

Aeration cost:<br />

40 hp 0.75 kWhp 0.09 $kWh 24 hday<br />

0.90 $72 per day or $2196 per month<br />

Sludge recirculation cost:<br />

5 hp 0.75 kWhp 0.09 $kWh 24 hday<br />

0.90 $9 per day or $275 per month<br />

Chemical<br />

additions<br />

Flash<br />

mix<br />

Flocculation<br />

Figure 13–12<br />

Schematic of water<br />

treatment plant.<br />

Primary<br />

clarifier<br />

Secondary<br />

clarifier<br />

Aerator<br />

Filter<br />

To clear<br />

well<br />

Canal


364 Chapter 13 Breakeven and Payback Analysis<br />

TABLE 13–1<br />

Cost Summary in Dollars per Month<br />

Alt.<br />

I.D.<br />

Alternative<br />

Description<br />

Savings from<br />

Discontinuation of<br />

Aeration<br />

(1)<br />

Recirculation<br />

(2)<br />

Total Savings<br />

(3) (1) (2)<br />

Extra Cost for<br />

Removal of<br />

Hardness<br />

(4)<br />

Turbidity<br />

(5)<br />

Total<br />

Extra Cost<br />

(6) (4) (5)<br />

Net<br />

Savings<br />

(7) (3) (6)<br />

1 Sludge recirculation<br />

Normal operating condition<br />

and aeration<br />

2 Aeration only — 275 275 1380 469 1849 1574<br />

3 Sludge recirculation 2196 — 2196 262 845 1107 1089<br />

only<br />

4 Neither aeration nor<br />

sludge recirculation<br />

2196 275 2471 1380 469 1849 622<br />

The estimates appear in columns 1 and 2 of the cost summary<br />

in Table 13–1.<br />

Costs associated with turbidity and hardness removal are<br />

a function of the chemical dosage required and the water flow<br />

rate. The calculations below are based on a design flow of<br />

53 m 3 minute.<br />

As stated earlier, there was less turbidity reduction<br />

through the primary clarifier without aeration than there was<br />

with it (28% versus 6%). The extra turbidity reaching the<br />

flocculators could require further additions of the coagulating<br />

chemical. If it is assumed that, as a worst case, these chemical<br />

additions would be proportional to the extra turbidity, then<br />

22 percent more coagulant would be required. Since the average<br />

dosage before discontinuation of aeration was 10 mgL,<br />

the incremental chemical cost incurred because of the increased<br />

turbidity in the clarifier effluent would be<br />

(10 0.22) mgL 10 –6 kgmg 53 m 3 min<br />

1000 Lm 3 0.165 $kg 60 minh<br />

24 hday $27.70day or $845month<br />

Similar calculations for the other operating conditions (i.e.,<br />

aeration only, and neither aeration nor sludge recirculation)<br />

reveal that the additional cost for turbidity removal would be<br />

$469 per month in each case, as shown in column 5 of<br />

Table 13–1.<br />

Changes in hardness affect chemical costs by virtue of the<br />

direct effect on the amount of lime required for water softening.<br />

With aeration and sludge recirculation, the average hardness<br />

reduction was 12.1 mgL (that is, 258 mgL 4.7%).<br />

However, with sludge recirculation only, the reduction was<br />

9.8 mgL, resulting in a difference of 2.3 mgL attributed to<br />

aeration. The extra cost of lime incurred because of the discontinuation<br />

of aeration, therefore, was<br />

2.3 mgL 0.62 mgL lime 10 6 kgmg<br />

53m 3 min 1000 Lm 3 0.079 $kg<br />

60 minh 24 hday $8.60day or<br />

$262month<br />

When sludge recirculation was discontinued, there was no<br />

hardness reduction through the clarifier, so that the extra lime<br />

cost would be $1380 per month.<br />

The total savings and total costs associated with changes<br />

in plant operating conditions are tabulated in columns 3 and<br />

6 of Table 13–1, respectively, with the net savings shown in<br />

column 7. Obviously, the optimum condition is represented<br />

by “sludge recirculation only.” This condition would result in<br />

a net savings of $1089 per month, compared to a net savings<br />

of $622 per month when both processes are discontinued and<br />

a net cost of $1574 per month for aeration only. Since the<br />

calculations made here represent worst-case conditions, the<br />

actual savings that resulted from modifying the plant operating<br />

procedures were greater than those indicated.<br />

In summary, the commonly applied water treatment practices<br />

of sludge recirculation and aeration can significantly affect<br />

the removal of some compounds in the primary clarifier.<br />

However, increasing energy and chemical costs warrant continued<br />

investigations on a case-by-case basis of the costeffectiveness<br />

of such practices.<br />

Case Study Exercises<br />

1. What will be the monthly savings in electricity from<br />

discontinuation of aeration if the cost of electricity is<br />

now 12 ¢kWh?<br />

2. Does a decrease in the efficiency of the aerator motor<br />

make the selected alternative of sludge recirculation only<br />

more attractive, less attractive, or the same as before?<br />

3. If the cost of lime were to increase by 50%, would the<br />

cost difference between the best alternative and secondbest<br />

alternative increase, decrease, or remain the same?<br />

4. If the efficiency of the sludge recirculation pump were<br />

reduced from 90% to 70%, would the net savings difference<br />

between alternatives 3 and 4 increase, decrease, or<br />

stay the same?<br />

5. If hardness removal were to be discontinued at the treatment<br />

plant, which alternative would be the most costeffective?<br />

6. If the cost of electricity decreased to 8 ¢kWh, which<br />

alternative would be the most cost-effective?<br />

7. At what electricity cost would the following alternatives<br />

just break even? ( a ) Alternatives 1 and 2, ( b ) alternatives<br />

1 and 3, ( c ) alternatives 1 and 4.


LEARNING STAGE 4<br />

Rounding Out the Study<br />

LEARNING STAGE 4<br />

Rounding Out<br />

the Study<br />

CHAPTER 14<br />

Effects of Inflation<br />

CHAPTER 15<br />

Cost Estimation and<br />

Indirect Cost<br />

Allocation<br />

CHAPTER 16<br />

Depreciation Methods<br />

CHAPTER 17<br />

After-Tax Economic<br />

Analysis<br />

This stage includes topics to enhance your ability to perform a<br />

thorough engineering economic study of one project or several<br />

alternatives. The effects of inflation, depreciation,<br />

income taxes in all types of studies, and indirect costs are incorporated<br />

into the methods of previous chapters. Techniques of cost<br />

estimation to better predict cash flows are treated in order to base<br />

alternative selection on more accurate estimates. The last two chapters<br />

include additional material on the use of engineering economics<br />

in decision making. An expanded version of sensitivity analysis is<br />

developed to examine parameters that vary over a predictable range<br />

of values. The use of decision trees and an introduction to real<br />

options are included. Finally, the elements of risk and probability<br />

are explicitly considered using expected values, probabilistic analysis,<br />

and spreadsheet-based Monte Carlo simulation.<br />

Several of these topics can be covered earlier in the text, depending<br />

on the objectives of the course. Use the chart in the Preface to<br />

determine appropriate points at which to introduce the material in<br />

Learning Stage 4.<br />

CHAPTER 18<br />

Sensitivity Analysis<br />

and Staged Decisions<br />

CHAPTER 19<br />

More on Variation<br />

and Decision Making<br />

under Risk


CHAPTER 14<br />

Effects of<br />

Inflation<br />

LEARNING OUTCOMES<br />

Purpose: Consider the effects of inflation when performing an engineering economy evaluation.<br />

SECTION TOPIC LEARNING OUTCOME<br />

14.1 Inflationary impact • Demonstrate the difference that inflation makes<br />

on money now and money in the future; also,<br />

explain deflation.<br />

14.2 PW with inflation • Calculate the PW of cash flows with an<br />

adjustment made for inflation.<br />

14.3 FW with inflation • Determine the real interest rate and calculate<br />

the inflation-adjusted FW with different<br />

interpretations of future worth values.<br />

14.4 CR with inflation • Calculate capital recovery of an investment<br />

using the AW value with inflation considered.


T<br />

his chapter concentrates upon understanding and calculating the effects of inflation<br />

in time value of money computations. Inflation is a reality that we deal with<br />

nearly everyday in our professional and personal lives.<br />

The annual inflation rate is closely watched and historically analyzed by government<br />

units, businesses, and industrial corporations. An engineering economy study can have different<br />

outcomes in an environment in which inflation is a serious concern compared to one<br />

in which it is of minor consideration. In the first decade of the 21st century, inflation has not<br />

been a major concern in the United States or most industrialized nations. But the inflation<br />

rate is sensitive to real, as well as perceived, factors of the economy. Factors such as the cost<br />

of energy, interest rates, availability and cost of skilled people, scarcity of materials, political<br />

stability, and other, less tangible factors have short-term and long-term impacts on the<br />

inflation rate. In some industries, it is vital that the effects of inflation be integrated into an<br />

economic analysis. The basic techniques to do so are covered here.<br />

14.1 Understanding the Impact of Inflation<br />

We are all very well aware that $20 now does not purchase the same amount as $20 did in 2005<br />

and purchases significantly less than in 2000. Why? Primarily this is due to inflation and the<br />

purchasing power of money.<br />

Inflation is an increase in the amount of money necessary to obtain the same amount of<br />

goods or services before the inflated price was present.<br />

Purchasing power , or buying power , measures the value of a currency in terms of the quantity<br />

and quality of goods or services that one unit of money will purchase. Inflation decreases<br />

the purchasing ability of money in that less goods or services can be purchased for the same<br />

one unit of money.<br />

Inflation<br />

Inflation occurs because the value of the currency has changed—it has gone down in value. The<br />

value of money has decreased, and as a result, it takes more money for the same amount of<br />

goods or services. This is a sign of inflation . To make comparisons between monetary amounts<br />

that occur in different time periods, the different-valued money first must be converted to constant-value<br />

money in order to represent the same purchasing power over time . This is especially<br />

important when future sums of money are considered, as is the case with all alternative<br />

evaluations.<br />

Money in one period of time t 1 can be brought to the same value as money in another period<br />

of time t 2 by using the equation<br />

amount in period t<br />

Amount in period t 1 ————————————— 2<br />

[14.1]<br />

inflation rate between t 1 and t 2<br />

Using dollars as the currency, dollars in period t 1 are called constant-value dollars or today’s<br />

dollars. Dollars in period t 2 are called future dollars or then-current dollars and have inflation<br />

taken into account. If f represents the inflation rate per period (year) and n is the number of time<br />

periods (years) between t 1 and t 2 , Equation [14.1] is<br />

Constant-value dollars ——————<br />

future dollars<br />

(1 f ) n [14.2]<br />

Future dollars constant-value dollars(1 f ) n [14.3]<br />

We can express future dollars in terms of constant-value dollars, and vice versa, by applying the<br />

last two equations. This is how the Consumer Price Index (CPI) and cost estimation indices<br />

(of Chapter 15) are determined. As an illustration, use the price of a cheese pizza.<br />

$8.99 March 2011<br />

If inflation on food prices averaged 5% during the last year, in constant-value 2010 dollars, this<br />

cost is last year’s equivalent of<br />

$8.991.05 $8.56 March 2010


368 Chapter 14 Effects of Inflation<br />

A predicted price in <strong>2012</strong>, according to Equation [14.3], is<br />

$8.99(1.05) $9.44 March <strong>2012</strong><br />

The price of $9.44 in <strong>2012</strong> buys exactly the same cheese pizza as $8.56 did in 2010. If inflation<br />

averages 5% per year over the next 10 years, Equation [14.3] is used to predict a price in 2020<br />

based on 2010.<br />

$8.56(1.05) 10 $13.94 March 2020<br />

This is a 63% increase over the 2010 price at 5% inflation for prepared food prices, which is generally<br />

not considered excessive. In some areas of the world, hyperinflation may average 50% per<br />

year. In such an unfortunate economy, the cheese pizza in 10 years rises from the dollar equivalent<br />

of $8.99 to $518.44! This is why countries experiencing hyperinflation must devalue the<br />

currency by factors of 100 and 1000 when unacceptable inflation rates persist.<br />

Placed into an industrial or business context, at a reasonably low inflation rate averaging 4%<br />

per year, equipment or services with a first cost of $209,000 will increase by 48% to $309,000<br />

over a 10-year span. This is before any consideration of the rate of return requirement is placed<br />

upon the equipment’s revenue-generating ability. Make no mistake: Inflation is a formidable<br />

force in our economy .<br />

There are three different rates that are important to understanding inflation: the real interest<br />

rate ( i ), the market interest rate ( i f ), and the inflation rate ( f ). Only the first two are interest rates.<br />

Real or inflation-free interest rate i . This is the rate at which interest is earned when the<br />

effects of changes in the value of currency (inflation) have been removed. Thus, the real interest<br />

rate presents an actual gain in purchasing power. (The equation used to calculate i , with the<br />

influence of inflation removed, is derived later in Section 14.3.) The real rate of return that<br />

generally applies for individuals is approximately 3.5% per year. This is the “safe investment”<br />

rate. The required real rate for corporations (and many individuals) is set above this safe rate<br />

when a MARR is established without an adjustment for inflation.<br />

Inflation-adjusted or market interest rate i f . As its name implies, this is the interest rate that<br />

has been adjusted to take inflation into account. This is the interest rate we hear everyday. It is<br />

a combination of the real interest rate i and the inflation rate f , and, therefore, it changes as the<br />

inflation rate changes. It is also known as the infl ated interest rate. A company’s MARR adjusted<br />

for inflation is referred to as the inflation-adjusted or market MARR. The determination<br />

of this value is discussed in Section 14.3.<br />

Inflation rate f . As described above, this is a measure of the rate of change in the value of the<br />

currency.<br />

Deflation is the opposite of inflation in that when deflation is present, the purchasing<br />

power of the monetary unit is greater in the future than at present. That is, it will take fewer<br />

dollars in the future to buy the same amount of goods or services as it does today. Inflation<br />

occurs much more commonly than deflation, especially at the national economy level. In<br />

deflationary economic conditions, the market interest rate is always less than the real interest<br />

rate.<br />

Temporary price deflation may occur in specific sectors of the economy due to the introduction<br />

of improved products, cheaper technology, or imported materials or products that<br />

force current prices down. In normal situations, prices equalize at a competitive level after a<br />

short time. However, deflation over a short time in a specific sector of an economy can be<br />

orchestrated through dumping . An example of dumping may be the importation of materials,<br />

such as steel, cement, or cars, into one country from international competitors at very<br />

low prices compared to current market prices in the targeted country. The prices will go<br />

down for the consumer, thus forcing domestic manufacturers to reduce their prices in order<br />

to compete for business. If domestic manufacturers are not in good financial condition, they<br />

may fail, and the imported items replace the domestic supply. Prices may then return to normal<br />

levels and, in fact, become inflated over time, if competition has been significantly<br />

reduced.<br />

On the surface, having a moderate rate of deflation sounds good when inflation has been<br />

present in the economy over long periods. However, if deflation occurs at a more general


14.2 Present Worth Calculations Adjusted for Inflation 369<br />

level, say nationally, it is likely to be accompanied by the lack of money for new capital. Another<br />

result is that individuals and families have less money to spend due to fewer jobs, less<br />

credit, and fewer loans available; an overall “tighter” money situation prevails. As money<br />

gets tighter, less is available to be committed to industrial growth and capital investment. In<br />

the extreme case, this can evolve over time into a deflationary spiral that disrupts the entire<br />

economy. This has happened on occasion, notably in the United States during the Great Depression<br />

of the 1930s.<br />

<strong>Engineering</strong> economy computations that consider deflation use the same relations as those for<br />

inflation. For basic equivalence between constant-value dollars and future dollars, Equations<br />

[14.2] and [14.3] are used, except the deflation rate is a − f value. For example, if deflation is estimated<br />

to be 2% per year, an asset that costs $10,000 today would have a first cost 5 years from<br />

now determined by Equation [14.3].<br />

10,000(1 f )<br />

n<br />

10,000(0.98) 5 10,000(0.9039) $9039<br />

14.2 Present Worth Calculations<br />

Adjusted for Inflation<br />

When the dollar amounts in different time periods are to be expressed in constant-value dollars,<br />

the equivalent present and future amounts must be determined using the real interest<br />

rate i . The calculations involved in this procedure are illustrated in Table14–1, where the inflation<br />

rate is 4% per year. Column 2 shows the inflation-driven increase for each of the next<br />

4 years for an item that has a cost of $5000 today. Column 3 shows the cost in future dollars,<br />

and column 4 verifies the cost in constant-value dollars via Equation [14.2]. When the future<br />

dollars of column 3 are converted to constant-value dollars (column 4), the cost is always<br />

$5000, the same as the cost at the start. This is predictably true when the costs are increasing<br />

by an amount exactly equal to the inflation rate. The actual cost (in inflated dollars) of the<br />

item 4 years from now will be $5849, but in constant-value dollars the cost in 4 years will still<br />

amount to $5000. Column 5 shows the present worth of future amounts of $5000 at a real<br />

interest rate of i 10% per year.<br />

Two conclusions can be drawn. At f 4%, $5000 today inflates to $5849 in 4 years. And<br />

$5000 four years in the future has a PW of only $3415 now in constant-value dollars at a real<br />

interest rate of 10% per year.<br />

Figure 14–1 graphs the differences over a 4-year period of the constant-value amount of<br />

$5000, the future-dollar costs at 4% inflation, and the present worth at 10% real interest with<br />

inflation considered. The effect of compounded inflation and interest rates can be large, as you<br />

can see by the shaded area.<br />

An alternative, less complicated method of accounting for inflation in a present worth analysis<br />

involves adjusting the interest formulas themselves to account for inflation. Consider the P F<br />

formula, where i is the real interest rate.<br />

P F ———— 1<br />

(1 i) n<br />

TABLE 14–1 Inflation Calculations Using Constant-Value Dollars (f 4%, i 10%)<br />

Year<br />

n<br />

(1)<br />

Cost<br />

Increase<br />

due to 4%<br />

Inflation, $<br />

(2)<br />

Cost in<br />

Future<br />

Dollars, $<br />

(3)<br />

Future Cost in<br />

Present Worth<br />

Constant-Value<br />

at Real<br />

Dollars, $<br />

i 10%,$<br />

(4) (3)1.04 n (5) (4)(PF,10%,n)<br />

0 5000 5000 5000<br />

1 5000(0.04) 200 5200 5200(1.04) 1 5000 4545<br />

2 5200(0.04) 208 5408 5408(1.04) 2 5000 4132<br />

3 5408(0.04) 216 5624 5624(1.04) 3 5000 3757<br />

4 5624(0.04) 225 5849 5849(1.04) 4 5000 3415


370 Chapter 14 Effects of Inflation<br />

Figure 14–1<br />

Comparison of constantvalue<br />

dollars, future dollars,<br />

and their present<br />

worth values.<br />

6000<br />

Actual cost<br />

at 4% inflation<br />

$5849<br />

Increased<br />

cost<br />

5000<br />

Future dollars<br />

Constant value<br />

10% Present worth<br />

$5000<br />

Decrease in<br />

PW due<br />

to inflation<br />

and interest<br />

4000<br />

$3415<br />

3000<br />

0<br />

1<br />

2<br />

Time, years<br />

3<br />

4<br />

The F , which is a future-dollar amount with inflation built in, can be converted to constant-value<br />

dollars by using Equation [14.2].<br />

P ———— F 1<br />

(1 f ) n ————<br />

(1 i) n<br />

F ——————— 1<br />

(1 i f if ) n [14.4]<br />

If the term i f if is defined as i f , the equation becomes<br />

P F<br />

1 ————<br />

(1 i f ) n F(PF,i f ,n) [14.5]<br />

As described earlier, i f is the inflation-adjusted or market interest rate and is defined as<br />

i f i f if [14.6]<br />

where i real interest rate<br />

f inflation rate<br />

For a real interest rate of 10% per year and an inflation rate of 4% per year, Equation [14.6] yields<br />

a market interest rate of 14.4%.<br />

i f 0.10 0.04 0.10(0.04) 0.144<br />

Table 14–2 illustrates the use of i f 14.4% in PW calculations for $5000 now, which inflates<br />

to $5849 in future dollars 4 years hence. As shown in column 4, the present worth for each year<br />

is the same as column 5 of Table 14–1.<br />

The present worth of any series of cash flows—uniform, arithmetic gradient, or geometric<br />

gradient—can be found similarly. That is, either i or i f is introduced into the P A, P G, or P g factors,<br />

depending upon whether the cash flow is expressed in constant-value (today’s) dollars or<br />

future dollars, respectively.<br />

If a cash flow series is expressed in today’s (constant-value) dollars, then its PW is the discounted<br />

value using the real interest rate i .<br />

If the cash flow is expressed in future dollars, the PW value is obtained using i f .<br />

It is always acceptable to first convert all future dollars to constant-value dollars using<br />

Equation [14.2] and then find the PW at the real interest rate i .


14.2 Present Worth Calculations Adjusted for Inflation 371<br />

TABLE 14–2<br />

Present Worth Calculation Using an Inflated Interest Rate<br />

Year<br />

n<br />

(1)<br />

Cost in<br />

Future Dollars, $<br />

(2)<br />

(PF,14.4%,n)<br />

(3)<br />

PW, $<br />

(4) (2)(3)<br />

0 5000 1 5000<br />

1 5200 0.8741 4545<br />

2 5408 0.7641 4132<br />

3 5624 0.6679 3757<br />

4 5849 0.5838 3415<br />

EXAMPLE 14.1<br />

Glyphosate is the active ingredient in the herbicide Roundup ® marketed by Monsanto Co.<br />

Roundup has been a dependable product used by farmers, municipalities, and suburbanites<br />

alike to control weeds in fields, yards, gardens, streets, and parks. Contributions to Monsanto’s<br />

revenue have been reduced significantly by international dumping of generic glyphosate, as<br />

announced in mid-2010. 1 Monsanto’s sales price was decreased from $16 to $12 per gallon to<br />

compete with the highly competitive pricing, and it is expected that the international price<br />

will settle at approximately $10 per gallon. Assume when the price was set at $16 per gallon,<br />

there was a prediction that in 5 years the price would inflate to $19 per gallon. Perform the<br />

following analysis.<br />

(a) Determine the annual rate of inflation over 5 years to increase the price from $16 to $19.<br />

(b) Using the same annual rate determined above as the rate at which the price continues to<br />

decline from the new $12 price, calculate the expected price in 5 years. Compare this result<br />

with $10 per gallon that Monsanto predicted would be the longer-term price.<br />

(c) Provided Monsanto were somehow able to recover the same market share as it had previously,<br />

and the same inflation rate was applied to the reduced $12 per gallon price, determine<br />

the price 5 years in the future and compare it with the pre-dumping price of $16 per<br />

gallon.<br />

(d) Determine the market interest rate that must be used in economic equivalence<br />

computations, if inflation is considered and an 8% per year real return is expected<br />

by Monsanto.<br />

Solution<br />

The first three parts involve inflation only—no return on investments.<br />

(a) Solve Equation [14.2] for the annual inflation rate f with known constant-value and future<br />

amounts.<br />

16 19(PF,f,5) ———— 19<br />

(1 f ) 5<br />

1 f (1.1875) 0.2<br />

f 0.035 (3.5% per year)<br />

(b) If the price deflation rate is 3.5% per year, find the F value in 5 years with P $12.<br />

F P(FP,3.5%,5) 12(1 0.035) 5<br />

12(0.8368)<br />

$10.04<br />

The price will fall to exactly $10 per gallon after 5 years, as Monsanto predicted.<br />

1 S. Kilman and I. Berry, “Monsanto Cuts Roundup Prices as Knockoffs Flood Farm Belt,” Wall Street<br />

Journal, May 28, 2010.


372 Chapter 14 Effects of Inflation<br />

(c) Five years in the future, at 3.5% per year inflation, the price will be<br />

F P(FP,3.5%,5) 12(1.035) 5<br />

12(1.1877)<br />

$14.25<br />

After 5 years of recovery at the same level as historically experienced, the price will still<br />

be considerably lower than it was at the pre-dumping point ($14.25 versus $16 per gallon).<br />

(d) With inflation at 3.5% per year and a real return of 8% per year, Equation [14.6] results in<br />

a market rate of 11.78% per year.<br />

i f 0.08 0.035 (0.08)(0.035)<br />

0.1178 (11.78% per year)<br />

EXAMPLE 14.2<br />

A 15-year $50,000 bond that has a dividend rate of 10% per year, payable semiannually, is currently<br />

for sale. If the expected rate of return of the purchaser is 8% per year, compounded<br />

semiannually, and if the inflation rate is expected to be 2.5% each 6-month period, what is the<br />

bond worth now (a) without an adjustment for inflation and (b) when inflation is considered?<br />

Show both hand and spreadsheet solutions.<br />

Solution by Hand<br />

(a) Without infl ation adjustment: The semiannual dividend is I [(50,000)(0.10)]2 $2500.<br />

At a nominal 4% per 6 months for 30 periods,<br />

PW 2500(PA,4%,30) 50,000(PF,4%,30) $58,645<br />

(b) With infl ation: Use the inflated rate i f .<br />

i f 0.04 0.025 (0.04)(0.025) 0.066 per semiannual period<br />

PW 2500(PA,6.6%,30) 50,000(PF,6.6%,30)<br />

2500(12.9244) 50,000(0.1470)<br />

$39,660<br />

Solution by Spreadsheet<br />

Both (a) and (b) require simple, single-cell functions on a spreadsheet (Figure 14–2). Without<br />

an inflation adjustment, the PV function is developed at the nominal 4% rate for 30 periods;<br />

with inflation considered the rate is i f 6.6%, as determined above.<br />

Comment<br />

The $18,985 difference in PW values illustrates the tremendous negative impact made by only<br />

2.5% inflation each 6 months (5.06% per year). Purchasing the $50,000 bond means receiving<br />

$75,000 in dividends over 15 years and the $50,000 principal in year 15. Yet, this is worth only<br />

$39,660 in constant-value dollars.<br />

Figure 14–2<br />

PW computation of a bond purchase (a) without and (b) with an inflation adjustment,<br />

Example 14.2.


14.2 Present Worth Calculations Adjusted for Inflation 373<br />

EXAMPLE 14.3<br />

A self-employed chemical engineer is on contract with Dow Chemical, currently working in a<br />

relatively high-inflation country in Central America. She wishes to calculate a project’s PW<br />

with estimated costs of $35,000 now and $7000 per year for 5 years beginning 1 year from now<br />

with increases of 12% per year thereafter for the next 8 years. Use a real interest rate of 15%<br />

per year to make the calculations (a) without an adjustment for inflation and (b) considering<br />

inflation at a rate of 11% per year.<br />

Solution<br />

(a) Figure 14–3 presents the cash flows. The PW without an adjustment for inflation is found<br />

using i 15% and g 12% in Equations [2.34] and [2.35] for the geometric series.<br />

PW 35,000 7000(PA,15%,4)<br />

<br />

{ 7000 [ 1 (<br />

—— 1.12<br />

1.15 ) 9 ]<br />

———————— } (PF,15%,4)<br />

0.15 0.12<br />

35,000 19,985 28,247<br />

$83,232<br />

In the PA factor, n 4 because the $7000 cost in year 5 is the A 1 term in Equation [2.34].<br />

PW = ?<br />

PW g = ?<br />

i =15% per year<br />

0 1 2 3 4 5 6 7 8 9 10 11 12<br />

0 1 2 3 4 5 6 7 8<br />

13<br />

9<br />

Year<br />

Geometric series year<br />

$7000<br />

$7840<br />

$35,000<br />

12% increase<br />

per year<br />

$17,331<br />

Figure 14–3<br />

Cash flow diagram, Example 14.3.<br />

(b) To adjust for inflation, calculate the inflated interest rate by Equation [14.6] and use it to<br />

calculate PW.<br />

i f 0.15 0.11 (0.15)(0.11) 0.2765<br />

PW 35,000 7000(PA,27.65%,4)<br />

<br />

{ 7000 [ 1 (<br />

——— 1.12<br />

1.2765 ) 9 ]<br />

————————— } (PF,27.65%,4)<br />

0.2765 0.12<br />

−35,000 7000(2.2545) 30,945(0.3766)<br />

$62,436<br />

Comment<br />

This result demonstrates that in a high-inflation economy, when negotiating the amount of the<br />

payments to repay a loan, it is economically advantageous for the borrower to use future<br />

(inflated) dollars whenever possible to make the payments. The present value of future inflated<br />

dollars is significantly less when the inflation adjustment is included. And the higher the inflation<br />

rate, the larger the discounting because the PF and PA factors decrease in size.


374 Chapter 14 Effects of Inflation<br />

Examples 14.2 and 14.3 above add credence to the “buy now, pay later” philosophy. However, at<br />

some point, the debt-ridden company or individual will have to pay off the debts and the accrued<br />

interest with the inflated dollars. If cash is not readily available at that time, the debts cannot be repaid.<br />

This can happen, for example, when a company unsuccessfully launches a new product, when there<br />

is a serious downturn in the economy, or when an individual loses a salary. In the longer term, this<br />

buy now, pay later approach must be tempered with sound financial practices now, and in the future.<br />

14.3 Future Worth Calculations<br />

Adjusted for Inflation<br />

In future worth calculations, a future amount F can have any one of four different interpretations:<br />

Case 1. The actual amount of money that will be accumulated at time n .<br />

Case 2. The purchasing power of the actual amount accumulated at time n , but stated in<br />

today’s (constant-value) dollars.<br />

Case 3. The number of future dollars required at time n to maintain the same purchasing<br />

power as today; that is, inflation is considered, but interest is not.<br />

Case 4. The amount of money required at time n to maintain purchasing power and earn a<br />

stated real interest rate.<br />

Depending upon which interpretation is intended, the F value is calculated differently, as<br />

described below. Each case is illustrated.<br />

Case 1: Actual Amount Accumulated It should be clear that F, the actual amount of money<br />

accumulated, is obtained using the inflation-adjusted (market) interest rate.<br />

F P (1 i f ) n P( F P,i f ,n) [14.7]<br />

For example, when we quote a market rate of 10%, the inflation rate is included. Over a 7-year<br />

period, $1000 invested at 10% per year will accumulate to<br />

F 1000( F P ,10%,7) $1948<br />

Case 2: Constant-Value Dollars with Purchasing Power The purchasing power of future<br />

dollars is determined by first using the market rate i f to calculate F and then deflating the<br />

future amount through division by (1 f )<br />

n<br />

.<br />

F P(1 i f<br />

————<br />

)n<br />

(1 f) n P (FP, i f ,n)<br />

——————<br />

(1 f ) n [14.8]<br />

This relation, in effect, recognizes the fact that inflated prices mean $1 in the future purchases<br />

less than $1 now. The percentage loss in purchasing power is a measure of how much less. As an<br />

illustration, consider the same $1000 now, and a 10% per year market rate, which includes an<br />

inflation rate of 4% per year. In 7 years, the purchasing power has risen, but only to $1481.<br />

1000 (FP, 10%, 7)<br />

F ————————<br />

(1.04) $1948<br />

7 ———<br />

1.3159 $1481<br />

This is $467 (or 24%) less than the $1948 actually accumulated at 10% (case 1). Therefore, we<br />

conclude that 4% inflation over 7 years reduces the purchasing power of money by 24%.<br />

Also for case 2, the future amount of money accumulated with today’s buying power could equivalently<br />

be determined by calculating the real interest rate and using it in the F P factor to compensate<br />

for the decreased purchasing power. This real interest rate is the i in Equation [14.6].<br />

i f i f if<br />

i (1 f ) f<br />

i i f f ———<br />

1 f<br />

[14.9]


14.3 Future Worth Calculations Adjusted for Inflation 375<br />

The real interest rate i represents the rate at which today’s dollars expand with their same purchasing<br />

power into equivalent future dollars. An inflation rate larger than the market interest<br />

rate leads to a negative real interest rate. The use of this interest rate is appropriate for calculating<br />

the future worth of an investment (such as a savings account or money market fund) when<br />

the effect of inflation must be removed. For the example of $1000 in today’s dollars from<br />

Equation [14.9]<br />

i ————— 0.10 0.04<br />

0.0577 (5.77%)<br />

1 0.04<br />

F 1000( F P ,5.77%,7) $1481<br />

The market interest rate of 10% per year has been reduced to a real rate that is less than 6% per<br />

year because of the erosive effects of 4% per year inflation.<br />

Case 3: Future Amount Required, No Interest This case recognizes that prices increase<br />

when inflation is present. Simply put, future dollars are worth less, so more are needed. No interest<br />

rate is considered in this case—only inflation. This is the situation if someone asks, “How<br />

much will a car cost in 5 years if its current cost is $20,000 and its price will increase by the inflation<br />

rate of 6% per year?” (The answer is $26,765.) No interest rate—only inflation—is involved.<br />

To find the future cost, substitute f for the interest rate in the F P factor.<br />

F P (1 f ) n P ( F P, f,n) [14.10]<br />

Reconsider the $1000 used previously. If it is escalating at exactly the inflation rate of 4% per<br />

year, the amount 7 years from now will be<br />

F 1000( F P ,4%,7) $1316<br />

Case 4: Inflation and Real Interest This is the case applied when a market MARR is established.<br />

Maintaining purchasing power and earning interest must account for both increasing<br />

prices (case 3) and the time value of money. If the growth of capital is to keep up, funds must<br />

grow at a rate equal to or above the real interest rate i plus the inflation rate f . Thus, to make a real<br />

rate of return of 5.77% when the inflation rate is 4%, i f is the market (inflation-adjusted) rate that<br />

must be used. For the same $1000 amount,<br />

i f 0.0577 0.04 0.0577(0.04) 0.10<br />

F 1000( F P ,10%,7) $1948<br />

This calculation shows that $1948 seven years in the future will be equivalent to $1000 now with<br />

a real return of i 5.77% per year and inflation of f 4% per year.<br />

Table 14–3 summarizes which rate is used in the equivalence formulas for the different interpretations<br />

of F . The calculations made in this section explain the following:<br />

• The amount of $1000 now at a market rate of 10% per year will accumulate to $1948 in 7 years.<br />

• The $1948 will have the purchasing power of $1481 of today’s dollars if f 4% per year.<br />

• An item with a cost of $1000 now will cost $1316 in 7 years at an inflation rate of 4% per year.<br />

• It will take $1948 of future dollars to be equivalent to $1000 now at a real interest rate of 5.77%<br />

with inflation considered at 4% per year.<br />

Most corporations evaluate alternatives at a MARR large enough to cover inflation plus some<br />

return greater than their cost of capital, and significantly higher than the safe investment return of<br />

approximately 3.5% mentioned earlier. Therefore, for case 4, the resulting MARR will normally<br />

be higher than the market rate i f . Define the symbol MARR f as the inflation-adjusted or market<br />

MARR, which is calculated in a fashion similar to i f .<br />

MARR f i f i ( f ) [14.11]


376 Chapter 14 Effects of Inflation<br />

TABLE 14–3<br />

Calculation Methods for Various Future Worth Interpretations<br />

Future Worth<br />

Desired<br />

Case 1: Actual dollars<br />

accumulated<br />

Case 2: Purchasing power<br />

of accumulated dollars in<br />

terms of constant-value<br />

dollars<br />

Case 3: Dollars required for<br />

same purchasing power<br />

Case 4: Future dollars to<br />

maintain purchasing<br />

power and to earn a return<br />

Method of<br />

Calculation<br />

Use stated market<br />

rate i f in equivalence<br />

formulas<br />

Use market rate i f in<br />

equivalence and<br />

divide by (1 f ) n<br />

or<br />

Use real i<br />

Use f in place of i in<br />

equivalence<br />

formulas<br />

Calculate i f and use in<br />

equivalence<br />

formulas<br />

Example for<br />

P $1000, n 7,<br />

i f 10%, f 4%<br />

F 1000(FP,10%,7)<br />

1000 (FP,10%,7)<br />

F ————————<br />

(1.04) 7<br />

or<br />

F 1000(FP,5.77%,7)<br />

F 1000(FP,4%,7)<br />

F 1000(FP,10%,7)<br />

The real rate of return i used here is the required rate for the corporation relative to its cost of<br />

capital. Now the future worth F , or FW, is calculated as<br />

F P (1 MARR f ) n P ( F P ,MARR f , n )<br />

For example, if a company has a WACC (weighted average cost of capital) of 10% per year and<br />

requires that a project return 3% per year above its WACC, the real return is i 13%. The<br />

inflation-adjusted MARR is calculated by including the inflation rate of, say, 4% per year. Then<br />

the project PW, AW, or FW will be determined at the rate obtained from Equation [14.11].<br />

MARR f 0.13 0.04 0.13(0.04) 0.1752 (17.52%)<br />

EXAMPLE 14.4<br />

Abbott Mining Systems wants to determine whether it should upgrade a piece of equipment<br />

used in deep mining operations in one of its international operations now or later. If the company<br />

selects plan A, the upgrade will be purchased now for $200,000. However, if the company<br />

selects plan I, the purchase will be deferred for 3 years when the cost is expected to rise<br />

to $300,000. Abbott is ambitious; it expects a real MARR of 12% per year. The inflation rate<br />

in the country has averaged 3% per year. From only an economic perspective, determine<br />

whether the company should purchase now or later (a) when inflation is not considered and<br />

(b) when inflation is considered.<br />

Solution<br />

(a) Infl ation not considered: The real rate, or MARR, is i 12% per year. The cost of plan I is<br />

$300,000 three years hence. Calculate the FW value for plan A three years from now and<br />

select the lower cost.<br />

FW A 200,000(FP,12%,3) $280,986<br />

FW I $−300,000<br />

Select plan A (purchase now).<br />

(b) Infl ation considered: This is case 4; the real rate (12%), and inflation of 3% must be<br />

accounted for. First, compute the inflation-adjusted MARR by Equation [14.11].<br />

MARR f 0.12 0.03 0.12(0.03) 0.1536


14.4 Capital Recovery Calculations Adjusted for Inflation 377<br />

Use MARR f to compute the FW value for plan A in future dollars.<br />

FW A 200,000(FP,15.36%,3) $307,040<br />

FW I $−300,000<br />

Purchase later (plan I) is now selected, because it requires fewer equivalent future dollars.<br />

The inflation rate of 3% per year has raised the equivalent future worth of costs by 9.3%<br />

from $280,986 to $307,040. This is the same as an increase of 3% per year, compounded<br />

over 3 years, or (1.03) 3 1 9.3%.<br />

Most countries have inflation rates in the range of 2% to 8% per year, but hyperinflation is a<br />

problem in countries where political instability, overspending by the government, weak international<br />

trade balances, etc., are present. Hyperinflation rates may be very high—10% to 100% per<br />

month . In these cases, the government may take drastic actions: redefine the currency in terms of<br />

the currency of another country, control banks and corporations, and control the flow of capital<br />

into and out of the country in order to decrease inflation.<br />

In a hyperinflated environment, people usually spend all their money immediately since the<br />

cost of goods and services will be so much higher the next month, week, or day. To appreciate<br />

the disastrous effect of hyperinflation on a company’s ability to keep up, we can rework Example<br />

14.4 b using an inflation rate of 10% per month, that is, a nominal 120% per year (not<br />

considering the compounding effect of inflation). The FW A amount skyrockets and plan I is a<br />

clear choice. Of course, in such an environment the $300,000 purchase price for plan I three<br />

years hence would obviously not be guaranteed, so the entire economic analysis is unreliable.<br />

Good economic decisions in a hyperinflated economy are very difficult to make using traditional<br />

engineering economy methods, since the estimated future values are totally unreliable<br />

and the future availability of capital is uncertain.<br />

14.4 Capital Recovery Calculations<br />

Adjusted for Inflation<br />

It is particularly important in capital recovery (CR) calculations used for AW analysis to include<br />

inflation because current capital dollars must be recovered with future inflated dollars. Since future<br />

dollars have less buying power than today’s dollars, it is obvious that more dollars will be<br />

required to recover the present investment. This suggests the use of the inflated interest rate in the<br />

A P formula. For example, if $1000 is invested today at a real interest rate of 10% per year when<br />

the inflation rate is 8% per year, the equivalent amount that must be recovered each year for<br />

5 years in future dollars is<br />

A 1000( A P ,18.8%,5) $325.59<br />

On the other hand, the decreased value of dollars through time means that investors can spend<br />

fewer present (higher-value) dollars to accumulate a specified amount of future (inflated) dollars.<br />

This suggests the use of a higher interest rate, that is, the i f rate, to produce a lower A value in the<br />

A F formula. The annual equivalent (with adjustment for inflation) of F $1000 five years<br />

from now in future dollars is<br />

A 1000( A F ,18.8%,5) $137.59<br />

For comparison, the equivalent annual amount to accumulate F $1000 at a real i <br />

10% (without adjustment for inflation) is 1000( A F ,10%,5) $163.80. When F is a future<br />

known cost, uniformly distributed payments should be spread over as long a time period as<br />

possible so that the leveraging effect of inflation will reduce the effective annual payment<br />

($137.59 versus $163.80 here).


378 Chapter 14 Effects of Inflation<br />

EXAMPLE 14.5<br />

What annual deposit is required for 5 years to accumulate an amount of money with the same<br />

purchasing power as $680.58 today, if the market interest rate is 10% per year and inflation is<br />

8% per year?<br />

Solution<br />

First, find the actual number of inflated dollars required 5 years in the future that is equivalent<br />

to $680.58 today. This is case 3; Equation [14.10] applies.<br />

F (present purchasing power)(1 f ) 5 680.58(1.08) 5 $1000<br />

The actual amount of the annual deposit is calculated using the market interest rate of 10%.<br />

This is case 4 where A is calculated for a given F.<br />

A 1000(AF,10%,5) $163.80<br />

Comment<br />

The real interest rate is i 1.85% as determined using Equation [14.9]. To put these calculations<br />

into perspective, if the inflation rate is zero when the real interest rate is 1.85%, the future<br />

amount of money with the same purchasing power as $680.58 today is obviously $680.58.<br />

Then the annual amount required to accumulate this future amount in 5 years is A <br />

680.58(AF,1.85%,5) $131.17. This is $32.63 lower than the $163.80 calculated above for<br />

f 8%. This difference is due to the fact that during inflationary periods, dollars deposited now<br />

have more purchasing power than the dollars returned at the end of the period. To make up the<br />

purchasing power difference, more higher-value dollars are required. That is, to maintain<br />

equivalent purchasing power at f 8% per year, an extra $32.63 per year is required.<br />

The logic discussed here explains why, in times of increasing inflation, lenders of money<br />

(credit card companies, mortgage companies, and banks) tend to further increase their market<br />

interest rates. People tend to pay off less of their incurred debt at each payment because they<br />

use any excess money to purchase additional items before the price is further inflated. Also, the<br />

lending institutions must have more money in the future to cover the expected higher costs of<br />

lending money. All this is due to the spiraling effect of increasing inflation. Breaking this cycle<br />

is difficult to do at the individual level and much more difficult to alter at a national level.<br />

CHAPTER SUMMARY<br />

Inflation, treated computationally as an interest rate, makes the cost of the same product or service<br />

increase over time due to the decreased value of money. There are several ways to consider<br />

inflation in engineering economy computations in terms of today’s (constant-value) dollars and<br />

in terms of future dollars. Some important relations are the following:<br />

Inflated interest rate: i f i f if<br />

Real interest rate: i ( i f − f ) (1 f )<br />

PW of a future amount with inflation considered: P F ( P F , i f , n )<br />

Future worth in constant-value dollars of a present amount with the same purchasing power:<br />

F P ( F P , i , n )<br />

Future amount to cover a current amount with inflation only: F P ( F P , f , n )<br />

Future amount to cover a current amount with inflation and interest: F P ( F P , i f , n )<br />

Annual equivalent of a future amount: A F ( A F , i f , n )<br />

Annual equivalent of a present amount in future dollars: A P ( A P , i f , n )<br />

Hyperinflation implies very high f values. Available funds are expended immediately because<br />

costs increase so rapidly that larger cash inflows cannot offset the fact that the currency is losing<br />

value. This can, and usually does, cause a national financial disaster when it continues over extended<br />

periods of time.


Problems 379<br />

PROBLEMS<br />

Interest Rate and Currency Considerations with<br />

Inflation<br />

14.1 What is the difference between today’s dollars and<br />

constant-value dollars ( a ) when using today as the<br />

reference point in time and ( b ) when using 2 years<br />

ago as the reference point?<br />

14.2 State the conditions under which the market interest<br />

rate is ( a ) higher than, ( b ) lower than, and<br />

( c ) the same as the real interest rate.<br />

14.3 What annual inflation rate is implied from an<br />

inflation-adjusted interest rate of 10% per year,<br />

when the real interest rate is 4% per year?<br />

14.4 Determine the inflation-adjusted interest rate for a<br />

growth company that wants to earn a real rate of<br />

return of 20% per year when the inflation rate is<br />

5% per year.<br />

14.5 For a high-growth company that wants to make a<br />

real rate of return of 30% per year, compounded<br />

monthly, determine the inflation-adjusted nominal<br />

interest rate per year. Assume the inflation rate is<br />

1.5% per month.<br />

14.6 A high-tech company whose stock trades on the<br />

NASDAQ stock exchange uses a MARR of 35% per<br />

year. If the chief financial officer (CFO) said the<br />

company expects to make a real rate of return of<br />

25% per year on its investments over the next 3-year<br />

period, what is the company expecting the inflation<br />

rate per year to be over that time period?<br />

14.7 Calculate the inflation-adjusted interest rate per<br />

quarter when the real interest rate is 4% per quarter<br />

and the inflation rate is 1% per quarter.<br />

14.8 Calculate the real interest rate per month if the<br />

nominal inflation-adjusted interest rate per year,<br />

compounded monthly, is 18% and the inflation rate<br />

per month is 0.5%.<br />

14.9 A southwestern city has a contract with Firestone<br />

Vehicle Fleet Maintenance to provide maintenance<br />

services on its fleet of city-owned vehicles such as<br />

street sweepers, garbage trucks, backhoes, and<br />

carpool vehicles. The contract price is fixed at<br />

$45,000 per year for 4 years. If you were asked to<br />

convert the future amounts into constant-value<br />

amounts per today’s dollars, what would the respective<br />

amounts be? Assume the current market<br />

interest rate of 10% per year and inflation rate of<br />

5% per year are expected to remain the same over<br />

the next 4-year period.<br />

14.10 When the inflation rate is 5% per year, how many<br />

inflated dollars will be required 10 years from now<br />

to buy the same things that $10,000 buys now?<br />

14.11 Assume that you want to retire 30 years from now<br />

with an amount of money that will have the same<br />

value (same purchasing power) as $1.5 million<br />

today. If you estimate the inflation rate will be 4%<br />

per year, how many future (then-current) dollars<br />

will you need?<br />

Adjusting for Inflation<br />

14.12 If the inflation rate is 7% per year, how many years<br />

will it take for the cost of something to double when<br />

prices increase at exactly the same rate as inflation?<br />

14.13 The inflation rate in a Central American country is<br />

6% per year. What real rate of return will an investor<br />

make on a $100,000 investment in a copper<br />

mine stock that yields an overall internal rate of<br />

return of 28% per year?<br />

14.14 During periods of hyperinflation, prices increase<br />

rapidly over short periods of time. In 1993, the<br />

Consumer Price Index (CPI) in Brazil was<br />

113.6 billion. In 1994, the CPI was 2472.4 billion.<br />

(a)<br />

What was the inflation rate per year between<br />

1993 and 1994?<br />

( b) Assuming the inflation rate calculated in<br />

part ( a ) occurred uniformly throughout the<br />

year and represented a nominal rate, what<br />

were the monthly and daily inflation rates<br />

over that time period?<br />

14.15 A trust was set up by your grandfather that states<br />

you are to receive $250,000 exactly 5 years from<br />

today. Determine the buying power of the $250,000<br />

in terms of today’s dollars if the market interest<br />

rate is 10% per year and the inflation rate is 4% per<br />

year.<br />

14.16 Assume the inflation rate is 4% per year and the<br />

market interest rate is 5% above the inflation rate.<br />

Determine ( a ) the number of constant-value dollars<br />

5 years in the future that is equivalent to<br />

$30,000 now and ( b ) the number of future dollars<br />

that will be equivalent to $30,000 now.<br />

14.17 The Pell Grant program of the federal government<br />

provides financial aid to needy college students. If<br />

the average grant is slated to increase from $4050<br />

to $5400 over the next 5 years “to keep up with<br />

inflation,” what is the average inflation rate per<br />

year expected to be?


380 Chapter 14 Effects of Inflation<br />

14.18 Ford Motor Company announced that the price of<br />

its F-150 pickup trucks is going to increase by only<br />

the inflation rate for the next 3 years. If the current<br />

price of a well-equipped truck is $28,000 and the<br />

inflation rate averages 2.1% per year, what is the<br />

expected price of a comparably equipped truck<br />

next year? 3 years from now?<br />

14.19 In an effort to reduce pipe breakage, water hammer,<br />

and product agitation, a chemical company plans to<br />

install several chemically resistant pulsation dampeners.<br />

The cost of the dampeners today is $120,000,<br />

but the company has to wait until a permit is approved<br />

for its bidirectional port-to-plant product<br />

pipeline. The permit approval process will take at<br />

least 2 years because of the time required for preparation<br />

of an environmental impact statement.<br />

Because of intense foreign competition, the manufacturer<br />

plans to increase the price only by the inflation<br />

rate each year. If the inflation rate is 2.8% per<br />

year and the company’s MARR is 20% per year,<br />

estimate the cost of the dampeners in 2 years in<br />

terms of ( a ) today’s dollars and ( b ) future dollars.<br />

14.20 A machine currently under consideration by<br />

Holzmann Industries has a cost of $45,000. When<br />

the purchasing manager complained that a similar<br />

machine the company purchased 5 years ago was<br />

much cheaper, the salesman responded that the<br />

cost of the machine has increased solely in accordance<br />

with the inflation rate, which has averaged<br />

3% per year. When the purchasing manager<br />

checked the invoice for the machine he purchased<br />

5 years ago, he saw that the price was $29,000.<br />

Was the salesman telling the truth about the increase<br />

in the cost of the machine? If not, what<br />

should the machine cost now, provided the price<br />

increased by only the inflation rate?<br />

14.21 A report by the National Center for Public Policy<br />

and Higher Education stated that tuition and fees<br />

(T&F) at public colleges and universities increased<br />

by 439% over the last 25 years. During this same<br />

time period, the median family income (MFI) rose<br />

147%. When the report was written, T&F at a<br />

4-year public university constituted 28% of the<br />

MFI of $52,000 (tuition and fees at a private university<br />

constituted 76% of MFI).<br />

(a) What was the tuition and fee cost per year<br />

when the report was written?<br />

(b) What was the T&F cost 25 years ago?<br />

(c) What percentage was T&F of the MFI<br />

25 years ago?<br />

14.22 The headline on a Chronicle of Higher Education<br />

article reads “College Costs Rise Faster than Inflation.”<br />

The article states that tuition at public colleges<br />

and universities increased by 58% over the<br />

past 5 years. ( a ) What was the average annual percentage<br />

increase over that period of time? (b) If the<br />

real increase in tuition (i.e., without inflation) was<br />

5% per year, what was the inflation rate per year?<br />

14.23 Stadium Capital Financing Group helps cashstrapped<br />

sports programs through the marketing of<br />

its “sports mortgage.” At the University of Kansas,<br />

Jayhawks fans can sign up to pay $105,000 over<br />

10 years for the right to buy top seats for football<br />

during the next 30 years. In return, the season tickets<br />

will stay locked in at current-year prices. Season<br />

tickets in tier 1 are currently selling for $350.<br />

A fan plans to purchase the sports mortgage along<br />

with season tickets now and each year for the next<br />

30 years (31 seasons). What is the dollar amount of<br />

the savings on the tickets (with no interest considered),<br />

if ticket prices rise at a rate of 3% per year<br />

for the next 30 years?<br />

Present Worth Calculations with Inflation<br />

14.24 There are two ways to account for inflation in present<br />

worth calculations. What are they?<br />

14.25 An environmental testing company needs to purchase<br />

equipment 2 years from now and expects to<br />

pay $50,000 at that time . At a real interest rate of<br />

10% per year and inflation rate of 4% per year, what<br />

is the present worth of the cost of the equipment?<br />

14.26 Carlsbad Gas and Electric is planning to purchase<br />

a degassing tower for removing CO 2 from acidified<br />

saltwater. The supplier quoted a price of<br />

$125,000 if the unit is purchased within the next<br />

3 years. Your supervisor has asked you to calculate<br />

the present worth of the tower when considering<br />

inflation. Assuming the tower will not be purchased<br />

for 3 years, calculate the present worth at<br />

an interest rate of 10% per year and an inflation<br />

rate of 4% per year.<br />

14.27 How much can the manufacturer of superconducting<br />

magnetic energy storage systems afford to<br />

spend now on new equipment in lieu of spending<br />

$75,000 four years from now? The company’s real<br />

MARR is 12% per year, and the inflation rate is<br />

3% per year.<br />

14.28 Find the present worth of the cash flows shown.<br />

Some are expressed as constant-value (CV) dollars,<br />

and others are inflated dollars. Assume a real<br />

interest rate of 8% per year and an inflation rate of<br />

6% per year.<br />

Year 1 2 3 4 5<br />

Cash Flow, $ 3000 6000 8000 4000 5000<br />

Stated as CV Inflated Inflated CV CV


Problems 381<br />

14.29 A doctor is on contract to a medium-sized oil company<br />

to provide medical services at remotely located,<br />

widely separated refineries. The doctor is<br />

considering the purchase of a private plane to reduce<br />

the total travel time between refineries. The doctor<br />

can buy a used Lear jet now or wait for a new very<br />

light jet (VLJ) that will be available 3 years from<br />

now. The cost of the VLJ will be $1.9 million, payable<br />

when the plane is delivered in 3 years. The doctor<br />

has asked you, his friend, to determine the present<br />

worth of the VLJ so that he can decide whether to<br />

buy the used Lear now or wait for the VLJ. If the<br />

MARR is 15% per year and the inflation rate is projected<br />

to be 3% per year, what is the present worth of<br />

the VLJ with inflation considered?<br />

14.30 A regional infrastructure building and maintenance<br />

contractor is trying to decide whether to buy<br />

a new compact horizontal directional drilling<br />

(HDD) machine now or wait to buy it 2 years from<br />

now (when a large pipeline contract will require<br />

the new equipment). The HDD machine will include<br />

an innovative pipe loader design and<br />

maneuverable undercarriage system. The cost of<br />

the system is $68,000 if purchased now or $81,000<br />

if purchased 2 years from now. At a real MARR of<br />

10% per year and an inflation rate of 5% per year,<br />

determine if the company should buy now or later<br />

( a ) without any adjustment for inflation and<br />

( b ) with inflation considered.<br />

14.31 An engineer must recommend one of two rapidprototyping<br />

machines for integration into an upgraded<br />

manufacturing line. She obtained estimates<br />

from salespeople from two companies. Salesman<br />

A gave her the estimates in constant-value<br />

(today’s) dollars, while saleswoman B provided<br />

the estimates in future (then-current) dollars. The<br />

company’s MARR is equal to the real rate of return<br />

of 20% per year, and inflation is estimated at<br />

4% per year. Use PW analysis to determine which<br />

machine the engineer should recommend.<br />

A<br />

(in CV Dollars)<br />

B<br />

(in Future Dollars)<br />

First cost, $ 140,000 155,000<br />

AOC, $ per year 25,000 40,000<br />

Life, years 10 10<br />

14.32 A salesman from Industrial Water Services (IWS),<br />

who is trying to get his foot in the door for a large<br />

account in Fremont, offered water chlorination<br />

equipment for $2.1 million. This is $400,000 more<br />

than the price offered by a competing saleswoman<br />

from AG Enterprises. However, IWS said the company<br />

would not have to pay for the equipment until<br />

the warranty runs out. If the equipment has<br />

a 2-year warranty, determine which offer is better.<br />

The company’s real MARR is 12% per year, and<br />

the inflation rate is 4% per year.<br />

14.33 A chemical engineer is considering two sizes of<br />

pipes, small (S) and large (L), for moving distillate<br />

from a refinery to the tank farm. A small pipeline<br />

will cost less to purchase (including valves and<br />

other appurtenances) but will have a high head loss<br />

and, therefore, a higher pumping cost. In writing<br />

the report, the engineer compared the alternatives<br />

based on future worth values, but the company<br />

president wants the costs expressed as present dollars.<br />

Determine present worth values if future<br />

worth values are FW S $2.3 million and FW L <br />

$2.5 million. The company uses a real interest rate<br />

of 1% per month and an inflation rate of 0.4% per<br />

month. Assume the future worth values were for a<br />

10-year project period.<br />

14.34 As an innovative way to pay for various software<br />

packages, a new high-tech service company has<br />

offered to pay your company, Custom Computer<br />

Services (CCS), in one of three ways: (1) pay<br />

$480,000 now, (2) pay $1.1 million 5 years from<br />

now, or (3) pay an amount of money 5 years from<br />

now that will have the same purchasing power as<br />

$850,000 now. If you, as president of CCS, want to<br />

earn a real interest rate of 10% per year when the<br />

inflation rate is 6% per year, which offer should<br />

you accept?<br />

Future Worth and Other Calculations with Inflation<br />

14.35 If the inflation rate is 6% per year and a person<br />

wants to earn a true (real) interest rate of 10% per<br />

year, determine the number of future dollars she<br />

has to receive 10 years from now if the present investment<br />

is $10,000.<br />

14.36 How many future dollars would you need 5 years<br />

from now just to have the same buying power as<br />

$50,000 now, if the deflation rate is 3% per year?<br />

14.37 If a company deposits $100,000 into an account<br />

that earns a market interest rate of 10% per year at<br />

a time when the defl ation rate is 1% per year, what<br />

will be the purchasing power of the accumulated<br />

amount (with respect to today’s dollars) at the end<br />

of 15 years?<br />

14.38 Harmony Corporation plans to set aside $60,000<br />

per year beginning 1 year from now for replacing<br />

equipment 5 years from now. What will be the purchasing<br />

power (in terms of current-value dollars)<br />

of the amount accumulated, if the investment<br />

grows by 10% per year, but inflation averages<br />

4% per year?


382 Chapter 14 Effects of Inflation<br />

14.39 The strategic plan of a solar energy company that<br />

manufactures high-efficiency solar cells includes<br />

an expansion of its physical plant in 4 years. The<br />

engineer in charge of planning estimates the expenditure<br />

required now to be $8 million, but in<br />

4 years, the cost will be higher by an amount equal<br />

to the inflation rate. If the company sets aside<br />

$7,000,000 now into an account that earns interest<br />

at 7% per year, what will the inflation rate have to<br />

be in order for the company to have exactly the<br />

right amount of money for the expansion?<br />

14.40 Five years ago, an industrial engineer deposited<br />

$10,000 into an account and left it undisturbed<br />

until now. The account is now worth $25,000.<br />

(a) What was the overall ROR during the<br />

5 years?<br />

(b) If the inflation over that period was 4% per<br />

year, what was the real ROR?<br />

(c) What is the buying power of the $25,000 now<br />

compared to the buying power 5 years ago?<br />

14.41 A Toyota Tundra can be purchased today for<br />

$32,350. A civil engineering firm is going to need<br />

three more trucks in 2 years because of a land development<br />

contract it just won. If the price of the<br />

truck increases exactly in accordance with an estimated<br />

inflation rate of 3.5% per year, determine<br />

how much the three trucks will cost in 2 years.<br />

14.42 The Nobel Prize is administered by The Nobel<br />

Foundation, a private institution that was founded<br />

in 1900 based on the will of Alfred Nobel, the inventor<br />

of dynamite. In part, his will stated: “The<br />

capital shall be invested by my executors in safe<br />

securities and shall constitute a fund, the interest<br />

on which shall be annually distributed in the form<br />

of prizes to those who, during the preceding year,<br />

shall have conferred the greatest benefit on mankind.”<br />

The will further stated that the prizes were<br />

to be awarded in physics, chemistry, peace, physiology<br />

or medicine, and literature. In addition to a<br />

gold medal and a diploma, each recipient receives<br />

a substantial sum of money that depends on the<br />

Foundation’s income that year. The first Nobel<br />

Prize was awarded in 1901 in the amount of<br />

$150,000. In 1996, the award was $653,000; it was<br />

$1.4 million in 2009.<br />

(a) If the increase between 1996 and 2009 was<br />

strictly due to inflation, what was the average<br />

inflation rate per year during that 13-year<br />

period?<br />

(b) If the Foundation expects to invest money<br />

with a return of 5% above the inflation rate,<br />

how much will a laureate receive in 2020,<br />

provided the inflation rate averages 3% per<br />

year between 2009 and 2020?<br />

14.43 Timken Roller Bearing is a manufacturer of<br />

seamless tubes for drill bit collars. The company<br />

is planning to add larger-capacity robotic arms to<br />

one of its assembly lines 3 years from now. If it<br />

is done now, the cost of the equipment with installation<br />

is $2.4 million. If the company’s real<br />

MARR is 15% per year, determine the equivalent<br />

amount the company can spend 3 years from<br />

now in then-current dollars if the inflation rate is<br />

2.8% per year.<br />

14.44 The data below show two patterns of inflation that<br />

are exactly the opposite of each other over a<br />

20-year time period.<br />

(a) If each machine costs $10,000 in year 0 and<br />

they both increase in cost exactly in accordance<br />

with the inflation rate, how much will<br />

each machine cost at the end of year 20?<br />

(b) What is the average inflation rate over the<br />

time period for machine A (that is, what single<br />

inflation rate would result in the same<br />

final cost for machine A)?<br />

(c) In which years will machine A cost more<br />

than machine B?<br />

Year Machine A, % Machine B, %<br />

1 10 2<br />

2 10 2<br />

3 2 10<br />

4 2 10<br />

5 10 2<br />

6 10 2<br />

7 2 10<br />

8 2 10<br />

19 2 10<br />

20 2 10<br />

14.45 Factors that increase costs and prices—especially<br />

for materials and manufacturing costs sensitive to<br />

market, technology, and labor availability—can be<br />

considered separately using the real interest rate i ,<br />

the inflation rate f , and additional increases that<br />

grow at a geometric rate g . The future amount is<br />

calculated based on a current estimate by using the<br />

relation<br />

F P (1 i ) n (1 f ) n (1 g ) n<br />

P [(1 i ) (1 f ) (1 g )]<br />

n<br />

The product of the first two terms enclosed in parentheses<br />

results in the inflated interest rate i f . The<br />

geometric rate is the same one used in the geometric<br />

series (Chapter 2). It commonly applies to<br />

maintenance and repair cost increases as machinery<br />

ages. This is over and above the inflation rate.<br />

If the current cost to manufacture an electronic<br />

subcomponent is $300,000 per year, what is the


Problems 383<br />

equivalent cost in 3 years, provided the average<br />

annual rates for i , f , and g are 10%, 3%, and 2%,<br />

respectively?<br />

Capital Recovery with Inflation<br />

14.46 An electric utility is considering two alternatives<br />

for satisfying state regulations regarding pollution<br />

control for one of its generating stations.<br />

This particular station is located at the outskirts<br />

of a major U.S. city and a short distance from a<br />

large city in a neighboring country. The station is<br />

currently producing excess VOCs and oxides of<br />

nitrogen. Two plans have been proposed for satisfying<br />

the regulators. Plan A involves replacing<br />

the burners and switching from fuel oil to natural<br />

gas. The cost of the option will be $300,000 initially<br />

and an extra $900,000 per year in fuel<br />

costs. Plan B involves going to the foreign city<br />

and running gas lines to many of the “backyard”<br />

brick-making sites that now use wood, tires, and<br />

other combustible waste materials for firing the<br />

bricks. The idea behind plan B is that by reducing<br />

the particulate pollution responsible for smog<br />

in the neighboring city, there would be greater<br />

benefit to U.S. citizens than would be achieved<br />

through plan A. The initial cost of plan B will be<br />

$1.2 million for installation of the lines. Additionally,<br />

the electric company would subsidize<br />

the cost of gas for the brick makers to the extent<br />

of $200,000 per year. Extra air monitoring associated<br />

with this plan will cost an additional<br />

$150,000 per year. For a 10-year project period<br />

and no salvage value for either plan, which one<br />

should be selected on the basis of an annual<br />

worth analysis at a real interest rate of 7% per<br />

year and an inflation rate of 4% per year?<br />

14.47 A chemical engineer who is smitten with wanderlust<br />

wants to build a reserve fund that will be sufficient<br />

to permit her to take time off from work to<br />

travel around the world. Her goal is to save enough<br />

money over the next 3 years so that when she begins<br />

her trip, the amount she has accumulated will<br />

have the same buying power as $72,000 today. If<br />

she expects to earn a market rate of 12% per year<br />

on her investments and inflation averages 5% per<br />

year over the 3 years, how much must she save<br />

each year to reach her goal?<br />

14.48 An entrepreneur engaged in wildcat oil well drilling<br />

is seeking investors who will put up $500,000<br />

for an opportunity to reap high returns if the venture<br />

is successful. The prospectus states that a<br />

return of at least 22% per year for 5 years is<br />

likely. How much will the investors have to<br />

receive each year to recover their money if an<br />

inflation rate of 5% per year is to be included in<br />

the calculation?<br />

14.49 Veri-Trol, Inc. manufactures in situ calibration<br />

verification systems that confirm flow measurement<br />

accuracies without removing the meters. The<br />

company has decided to modify the main assembly<br />

line with one of the enhancement processes<br />

shown below. If the company’s real MARR is 15%<br />

per year, which process has the lower annual cost<br />

when inflation of 5% per year is considered?<br />

Process X<br />

Process Y<br />

First cost, $ 65,000 90,000<br />

Operating cost, $ per year 40,000 34,000<br />

Salvage value, $ 0 10,000<br />

Life, years 5 5<br />

14.50 Johnson Thermal Products used austenitic<br />

nickel-chromium alloys to manufacture resistance<br />

heating wire. The company is considering a new<br />

annealing-drawing process to reduce costs. If the<br />

new process will cost $3.7 million now, how much<br />

must be saved each year to recover the investment<br />

in 5 years if the company’s MARR is a real 12%<br />

per year and the inflation rate is 3% per year?<br />

14.51 The costs associated with a small X-ray inspection<br />

system are $40,000 now and $24,000 per year,<br />

with a $6000 salvage value after 3 years. Determine<br />

the equivalent cost of the system if the real<br />

interest rate is 10% per year and the inflation rate<br />

is 4% per year.<br />

14.52 Maintenance costs for pollution control equipment<br />

on a pulverized coal cyclone furnace are expected<br />

to be $180,000 now and another $70,000 three<br />

years from now. The CFO of Monongahela Power<br />

wants to know the equivalent annual cost of the<br />

equipment in years 1 through 5. If the company<br />

uses a real interest rate of 9% per year and the inflation<br />

rate averages 3% per year, what is the<br />

equivalent annual cost of the equipment?<br />

14.53 MetroKlean LLC, a hazardous waste soil cleaning<br />

company, borrowed $2.5 million for 5 years to finance<br />

start-up costs for a new project involving<br />

site reclamation. The company expects to earn a<br />

real rate of return of 20% per year. The average<br />

inflation rate is 5% per year.<br />

(a)<br />

Determine the capital recovery required each<br />

year with inflation considered.<br />

(b) Determine the capital recovery if the company<br />

is satisfied with accumulating $2.5 million<br />

at the end of the 5 years with inflation<br />

considered.<br />

(c) Determine the capital recovery in part ( b )<br />

without considering inflation.


384 Chapter 14 Effects of Inflation<br />

ADDITIONAL PROBLEMS AND FE EXAM REVIEW QUESTIONS<br />

14.54 Inflation occurs when:<br />

(a) Productivity increases<br />

(b) The value of the currency decreases<br />

(c) The value of the currency increases<br />

(d) The price of gold decreases<br />

14.55 “Then-current” (future) dollars can be converted<br />

into constant-value dollars by:<br />

(a) Multiplying them by (1 f ) n<br />

(b) Multiplying them by (1 f ) n n<br />

(1 i )<br />

(c) Dividing them by (1 f ) n<br />

(d)<br />

n<br />

Dividing them by (1 i f )<br />

14.56 To calculate how much something will cost if you<br />

expect its cost to increase by exactly the inflation<br />

rate, you should:<br />

(a) Multiply by (1 f ) n<br />

(b) Multiply by [(1 f ) n n<br />

(1 i ) ]<br />

(c) Divide by (1 f ) n<br />

(d) Multiply by (1 i f ) n<br />

14.57 If the market interest rate is 16% per year when the<br />

inflation rate is 9% per year, the real interest rate is<br />

closest to:<br />

( a ) 6.4%<br />

( b ) 7%<br />

( c ) 9%<br />

( d ) 15.6%<br />

14.58 The market interest rate i f is 12% per year, compounded<br />

semiannually. For an inflation rate of 2%<br />

per 6 months, the effective semiannual interest rate<br />

is closest to:<br />

( a ) 2%<br />

( b ) 3%<br />

( c ) 4%<br />

( d ) 6%<br />

14.59 Construction equipment has a cost today of<br />

$40,000. If its cost has increased only by the inflation<br />

rate of 6% per year, when the market interest<br />

rate was 10% per year, its cost 10 years ago was<br />

closest to:<br />

(a) $15,420<br />

(b) $22,335<br />

(c) $27,405<br />

(d) $71,630<br />

14.60 The amount of money that would be accumulated<br />

now from an investment of $1000 25 years ago at<br />

a market rate of 5% per year and an inflation rate<br />

averaging 2% per year over that time period is<br />

closest to:<br />

(a) $1640<br />

( b) $3385<br />

( c) $5430<br />

( d) Over $5500<br />

14.61 If $1000 is invested now, the number of future dollars<br />

required 10 years from now to earn a real interest<br />

rate of 6% per year, when the inflation rate is<br />

4% per year, is closest to:<br />

(a) $1480<br />

(b) $1790<br />

(c) $2650<br />

(d) Over $2700<br />

14.62 If you expect to receive a gift of $50,000 six years<br />

from now, the present worth of the gift at a real<br />

interest rate of 4% per year and an inflation rate of<br />

3% per year is closest to:<br />

(a) $27,600<br />

(b) $29,800<br />

(c) $33,100<br />

(d) $37,200<br />

14.63 New state-mandated emission testing equipment<br />

for annual inspection of automobiles at Charlie’s<br />

Garage has a first cost of $30,000, an annual operating<br />

cost of $7000, and a $5000 salvage value<br />

after its 20-year life. For a real interest rate of 5%<br />

per year and an inflation rate of 4% per year, the<br />

annual capital recovery requirement for the equipment<br />

(in future dollars) is determined by:<br />

(a) AW 30,000( A P ,4%,20) 7000<br />

5000( A F ,4%,20)<br />

(b) AW 30,000( A P ,5%,20) 7000<br />

5000( A F ,5%,20)<br />

(c) AW 30,000( A P ,9%,20) 7000<br />

5000( A F ,9%,20)<br />

(d) AW 30,000( A P ,9.2%,20) 7000<br />

5000( A F ,9.2%,20)<br />

14.64 Temporary price deflation may occur in specific<br />

sectors of the economy for all of the following reasons<br />

except:<br />

(a) Improved technology<br />

(b) Excessive demand<br />

(c) Dumping<br />

(d) Increased productivity


Case Study 385<br />

CASE STUDY<br />

INFLATION VERSUS STOCK AND BOND INVESTMENTS<br />

Background<br />

The savings and investments that an individual maintains<br />

should have some balance between equity (corporate stocks<br />

that rely on market growth and dividend income) and fixedincome<br />

investments (bonds that pay dividends to the<br />

purchaser and a guaranteed amount upon maturity). When<br />

inflation is moderately high, bonds offer a low return relative<br />

to stocks, because the potential for market growth is not present<br />

with bonds. Additionally, the forces of inflation make the<br />

dividends worth less in future years, because there is no inflation<br />

adjustment made in the amount the dividend pays as time<br />

passes. However, bonds do offer a steady income that may be<br />

important to an individual, and they serve to preserve the<br />

principal invested in the bond, because the face value is returned<br />

at maturity.<br />

Information<br />

Earl is an engineer who wants a predictable flow of money<br />

for travel and vacations. He has a collection of stocks in his<br />

retirement portfolio, but no bonds. He has accumulated a<br />

total of $50,000 of his own funds in low-yielding savings accounts<br />

and wants to improve his long-term return from this<br />

nonretirement program “nest egg.” He can choose additional<br />

stocks or bonds, but has decided to not split the $50,000 between<br />

the two forms of investments. There are two choices he<br />

has outlined, with the best estimates he can make at this time.<br />

He assumes the effects of federal and state income taxes will<br />

be the same for both forms of investment.<br />

Stock purchase: Stocks purchased through a mutual fund<br />

would pay an estimated 2% per year dividend and appreciate<br />

in value at 5% per year.<br />

Bond purchase: If he purchased a bond, he would have a<br />

predictable income of 5% per year and the $50,000 face<br />

value after the 12-year maturity period.<br />

Case Study Questions<br />

The analysis that Earl has laid out has the following questions.<br />

Can you answer them for him for both choices?<br />

1. What is the overall rate of return after 12 years?<br />

2. If he decided to sell the stock or bond immediately after<br />

the fifth annual dividend, what is his minimum selling<br />

price to realize a 7% real return? Include an adjustment<br />

of 4% per year for inflation.<br />

3. If Earl needed some money in the future, say, immediately<br />

after the fifth dividend payment, what would be<br />

the minimum selling price in future dollars, if he were<br />

only interested in recovering an amount that maintained<br />

the purchasing power of the original price?<br />

4. As a follow-on to question 3, what happens to the<br />

selling price (in future dollars) 5 years after purchase,<br />

if Earl is willing to remove (net out) the future purchasing<br />

power of each of the dividends in the computation<br />

to determine the required selling price 5 years<br />

hence?<br />

5. Earl plans to keep the stocks or bonds for 12 years, that<br />

is, until the bond matures. However, he wants to make<br />

the 7% per year real return and make up for the expected<br />

4% per year inflation. For what amount must he<br />

sell the stocks after 12 years, or buy the bonds now to<br />

ensure he realizes this return? Do these amounts seem<br />

reasonable to you, given your knowledge of the way<br />

that stocks and bonds are bought and sold?


CHAPTER 15<br />

Cost<br />

Estimation and<br />

Indirect Cost<br />

Allocation<br />

LEARNING OUTCOMES<br />

Purpose: Make cost estimates using different methods; demonstrate the allocation of indirect costs using traditional and<br />

activity-based costing rates.<br />

SECTION TOPIC LEARNING OUTCOME<br />

15.1 Approach • Explain the bottom-up and design-to-cost (topdown)<br />

approaches to cost estimation.<br />

15.2 Unit method • Use the unit method to make a preliminary cost<br />

estimate.<br />

15.3 Cost index • Use a cost index to estimate a present cost based<br />

on historical data.<br />

15.4 Cost capacity • Use a cost-capacity equation to estimate<br />

component, system, or plant costs.<br />

15.5 Factor method • Estimate total plant cost using the factor<br />

method.<br />

15.6 Indirect cost rates • Allocate indirect costs using traditional indirect<br />

cost rates.<br />

15.7 ABC allocation • Use the Activity-Based Costing (ABC) method to<br />

allocate indirect costs.<br />

15.8 Ethics and profit • Describe how biased estimation can become an<br />

ethical dilemma.


U<br />

p to this point, cost and revenue cash flow values have been stated or assumed<br />

as known. In reality, they are not; they must be estimated. This chapter explains<br />

what cost estimation involves and applies cost estimation techniques. Cost estimation<br />

is important in all aspects of a project, but especially in the stages of project conception,<br />

preliminary design, detailed design, and economic analysis. When a project is developed<br />

in the private or the public sector, questions about costs and revenues will be posed by<br />

individuals representing many different functions: management, engineering, construction,<br />

production, quality, finance, safety, environmental, legal, and marketing, to name some. In<br />

engineering practice, the estimation of costs receives much more attention than revenue<br />

estimation; costs are the topic of this chapter.<br />

Unlike direct costs for labor and materials, indirect costs are not easily traced to a specific<br />

department, machine, or processing line. Therefore, allocation of indirect costs for functions<br />

such as utilities, safety, management and administration, purchasing, and quality is made<br />

using some rational basis. Both the traditional method of allocation and the Activity-Based<br />

Costing (ABC) method are covered in this chapter.<br />

15.1 Understanding How Cost Estimation<br />

Is Accomplished<br />

Cost estimation is a major activity performed in the initial stages of virtually every effort in<br />

industry, business, and government. In general, most cost estimates are developed for either a<br />

project or a system; however, combinations of these are very common. A project usually involves<br />

physical items, such as a building, bridge, manufacturing plant, or offshore drilling<br />

platform, to name just a few. A system is usually an operational design that involves processes,<br />

services, software, and other nonphysical items. Examples might be a purchase order<br />

system, a software package, an Internet-based remote-control system, or a health care delivery<br />

system. Of course, many projects will have major elements that are not physical, so estimates<br />

of both types must be developed. For example, consider a computer network system.<br />

There would be no operational system if only the costs of computer hardware plus wire and<br />

wireless connectors were estimated; it is equally important to estimate the software, personnel,<br />

and maintenance costs.<br />

Thus far virtually all cash flow estimates in the examples, problems, progressive examples,<br />

and case studies were stated or assumed to be known. In real-world practice, the cash flows for<br />

costs and revenues must be estimated prior to the evaluation of a project or comparison of alternatives.<br />

We concentrate on cost estimation because costs are the primary values estimated for the<br />

economic analysis. Revenue estimates utilized by engineers are usually developed in marketing,<br />

sales, and other departments.<br />

Costs are comprised of direct costs and indirect costs . Normally direct costs are estimated<br />

with some detail, then the indirect costs are added using standard rates and factors.<br />

However, direct costs in many industries, including manufacturing and assembly settings,<br />

have become a small percentage of overall product cost, while indirect costs have become<br />

much larger. Accordingly, many industrial settings require some estimating for indirect costs<br />

as well. Indirect cost allocation is discussed in detail in later sections of this chapter. Primarily,<br />

direct costs are discussed here.<br />

Because cost estimation is a complex activity, the following questions form a structure for the<br />

discussion that follows.<br />

DirectIndirect costs<br />

• What cost components must be estimated?<br />

• What approach to cost estimation will be applied?<br />

• How accurate should the estimates be?<br />

• What estimation techniques will be utilized?<br />

Costs to Estimate If a project revolves around a single piece of equipment, for example, an<br />

industrial robot, the cost components will be significantly simpler and fewer than the components<br />

for a complete system such as the manufacturing and testing line for a new product. Therefore, it<br />

is important to know up front how much the cost estimation task will involve. Examples of cost<br />

components are the first cost P and the annual operating cost (AOC), also called the M&O costs


388 Chapter 15 Cost Estimation and Indirect Cost Allocation<br />

(maintenance and operating) of equipment. Each component will have several cost elements .<br />

Listed below are sample elements of the first cost and AOC components.<br />

First cost component P:<br />

Elements:<br />

Equipment cost<br />

Delivery charges<br />

Installation cost<br />

Insurance coverage<br />

Initial training of personnel for equipment use<br />

Delivered-equipment cost is the sum of the first two elements in the list above; installedequipment<br />

cost adds the third element.<br />

AOC component (part of the equivalent annual cost A):<br />

Elements: Direct labor cost for operating personnel<br />

Direct materials<br />

Maintenance costs (daily, periodic, repairs, etc.)<br />

Rework and rebuild<br />

Some of these elements, such as equipment cost, can be determined with high accuracy; others,<br />

such as maintenance costs, are harder to estimate. When costs for an entire system must be estimated,<br />

the number of cost components and elements is likely to be in the hundreds. It is then<br />

necessary to prioritize the estimation tasks.<br />

For familiar projects (houses, office buildings, highways, and some chemical plants) there are<br />

standard cost estimation software packages available. For example, state highway departments<br />

utilize software that prompts for the cost components (bridges, pavement, cut-and-fill profiles,<br />

etc.) and estimates costs with time-proven, built-in relations. Once these components are estimated,<br />

exceptions for the specific project are added. However, there are no “canned” software<br />

packages for a large percentage of industrial, business, service and public sector projects.<br />

Cost Estimation Approach Historically in industry, business, and the public sector, a<br />

bottom-up approach to cost estimation was applied. For a simple rendition of this approach, see<br />

Figure 15–1 (left side). The progression is as follows: cost components and their elements are<br />

identified, cost elements are estimated, and estimates are summed to obtain total direct cost. The<br />

price is then determined by adding indirect costs and the profit margin, which is usually a percentage<br />

of the total cost.<br />

The bottom-up approach treats the required price as an output variable and the cost estimates<br />

as input variables. This approach works well when competition is not a dominant factor in pricing<br />

the product or service.<br />

Figure 15–1 (right side) shows a simplistic progression for the design-to-cost , or top-down, approach.<br />

The competitive price establishes the target cost.<br />

The design-to-cost, or top-down, approach treats the competitive price as an input variable<br />

and the cost estimates as output variables. This approach is useful in encouraging innovation,<br />

new design, manufacturing process improvement, and efficiency. These are some of the essentials<br />

of value engineering and value-added systems engineering.<br />

This approach places greater emphasis on the accuracy of the price estimation activity. The target<br />

cost must be realistic, or else it can be a disincentive to design and engineering staff. The designto-cost<br />

approach is best applied in the early stages of a new or enhanced product design. The<br />

detailed design and specific equipment options are not yet known, but the price estimates assist<br />

in establishing target costs for different components.<br />

Usually, the resulting approach is some combination of these two philosophies. However, it is<br />

helpful to understand up front what approach is to be emphasized. Historically, the bottom-up<br />

approach was more predominant in Western engineering cultures, especially in North America.<br />

The design-to-cost approach is considered routine in Eastern engineering cultures; however,


15.1 Understanding How Cost Estimation Is Accomplished 389<br />

Required<br />

price<br />

Desired<br />

profit<br />

<br />

Top-down approach Figure 15–1<br />

Simplified cost estimation<br />

<br />

Allowed<br />

profit<br />

Competitive<br />

price<br />

processes for bottom-up<br />

and top-down approaches.<br />

Total cost <br />

Target cost<br />

Indirect<br />

costs<br />

Maintenance<br />

and operations<br />

<br />

<br />

Direct<br />

costs<br />

<br />

<br />

Indirect<br />

costs<br />

Maintenance<br />

and operations<br />

Cost<br />

component<br />

estimates<br />

Direct<br />

labor<br />

Direct<br />

materials<br />

<br />

<br />

<br />

<br />

Direct<br />

labor<br />

Direct<br />

materials<br />

Cost<br />

component<br />

estimates<br />

Equipment and<br />

capital recovery<br />

<br />

<br />

Equipment<br />

capital recovery<br />

At design stage<br />

Bottom-up approach<br />

Before design stage<br />

Design-to-cost approach<br />

globalization of engineering design has speeded the adoption of the design-to-cost approach<br />

worldwide.<br />

Accuracy of the Estimates No cost estimates are expected to be exact; however, they are<br />

expected to be reasonable and accurate enough to support economic scrutiny. The accuracy required<br />

increases as the project progresses from preliminary design to detailed design and on to<br />

economic evaluation. Cost estimates made before and during the preliminary design stage are<br />

expected to be good “first-cut” estimates that serve as input to the project budget.<br />

When utilized at early and conceptual design stages, estimates are referred to as order-ofmagnitude<br />

estimates and generally range within 20% of actual cost. At the detailed design<br />

stage, cost estimates are expected to be accurate enough to support economic evaluation for a<br />

go–no go decision. Every project setting has its own characteristics, but a range of 5% of actual<br />

costs is expected at the detailed design stage. Figure 15–2 shows the general range of estimate<br />

accuracy for the construction cost of a building versus time spent in preparing the estimate. Obviously,<br />

the desire for better accuracy has to be balanced against the cost of obtaining it.<br />

Accuracy of Estimate<br />

30%<br />

25%<br />

20%<br />

15%<br />

10%<br />

5%<br />

}<br />

10 minutes<br />

1 hour<br />

1 day<br />

Scoping/feasibility<br />

Order of magnitude estimate<br />

Partially designed<br />

Design 60–100% complete<br />

Detailed estimate<br />

1 week<br />

Time Spent on Estimate<br />

3 weeks<br />

Figure 15–2<br />

Characteristic curve of<br />

estimate accuracy versus<br />

time spent to estimate<br />

construction cost of a<br />

building.


390 Chapter 15 Cost Estimation and Indirect Cost Allocation<br />

Cost Estimation Techniques Methods such as expert opinion and comparison with comparable<br />

installations serve as excellent estimators. The use of the unit method and cost indexes base<br />

the present estimate on past cost experiences, with inflation considered. Models such as costcapacity<br />

equations and the factor method are simple mathematical techniques applied at the<br />

preliminary design stage. They are called cost estimating relationships (CERs). There are many<br />

additional methods discussed in the handbooks and publications of different industries.<br />

Most cost estimates made in a professional setting are accomplished in part or wholly using<br />

software packages linked to updated databases that contain cost indexes and rates for the locations,<br />

products, or processes being studied. There are a wide variety of estimators, cost trackers,<br />

and cost compliance software systems, most of them developed for specific industries. Corporations<br />

usually standardize on one or two packages to ensure consistency over time and projects.<br />

15.2 Unit Method<br />

The unit method is a popular preliminary estimation technique applicable to virtually all professions.<br />

The total estimated cost C T is obtained by multiplying the number of units N by a per unit<br />

cost factor u .<br />

C T u N [15.1]<br />

Unit cost factors must be updated frequently to remain current with changing costs, areas,<br />

and inflation. Some sample unit cost factors (and values) are<br />

Total average cost of operating an automobile (52¢ per mile)<br />

Cost to bury fiber cable in a suburban area ($30,000 per mile)<br />

Cost to construct a parking space in a parking garage ($4500 per space)<br />

Cost of constructing interstate highway ($6.2 million per mile)<br />

Cost of house construction per livable area ($225 per square foot)<br />

Applications of the unit method to estimate costs are easily found. If house construction costs<br />

average $225 per square foot, a preliminary cost estimate for an 1800-square-foot house, using<br />

Equation [15.1], is $405,000. Similarly, a 200-mile trip should cost about $104 for the car only<br />

at 52¢ per mile.<br />

When there are several components to a project or system, the unit cost factors for each component<br />

are multiplied by the amount of resources needed, and the results are summed to obtain<br />

the total cost C T . This is illustrated in Example 15.1.<br />

EXAMPLE 15.1<br />

Justin, an ME with Dynamic Castings, has been asked to make a preliminary estimate of the<br />

total cost to manufacture 1500 sections of high-pressure gas pipe using an advanced centrifugal<br />

casting method. Since a 20% estimate is acceptable at this preliminary stage, a unit<br />

method estimate is sufficient. Use the following resource and unit cost factor estimates to<br />

help Justin.<br />

Materials: 3000 tons at $45.90 per ton<br />

Machinery and tooling: 1500 hours at $120 per hour<br />

Direct labor in plant:<br />

Casting and treating: 3000 hours at $55 per hour<br />

Finishing and shipping: 1200 hours at $45 per hour<br />

Indirect labor: 400 hours at $75 per hour<br />

Solution<br />

Apply Equation [15.1] to each of the five areas and sum the results to obtain the total cost<br />

estimate of $566,700. Table 15–1 provides the details.


15.3 Cost Indexes 391<br />

TABLE 15–1<br />

Total Cost Estimate Using Unit Cost Factors for Several Resource Areas,<br />

Example 15.1<br />

Resource Amount N Unit Cost Factor u, $ Cost Estimate, u N, $<br />

Materials 3000 tons 45.90 per ton 137,700<br />

Machinery, tooling 1500 hours 120 per hour 180,000<br />

Labor, casting 3000 hours 55 per hour 165,000<br />

Labor, finishing 1200 hours 45 per hour 54,000<br />

Labor, indirect 400 hours 75 per hour 30,000<br />

Total cost estimate 566,700<br />

15.3 Cost Indexes<br />

This section explains indexes and their use in cost estimation. An index is a ratio or other number<br />

based on observation and used as an indicator or measure. A preliminary cost estimate is often<br />

based on a cost index.<br />

A cost index is a ratio of the cost of something today to its cost sometime in the past. The index is<br />

dimensionless and measures relative cost change over time . Because these indexes are sensitive to<br />

technological change, the predefined quantity and quality of elements used to define the index may be<br />

hard to retain over time, thus causing “index creep.” Timely updating of the index is very important.<br />

One such index that most people are familiar with is the Consumer Price Index (CPI), which<br />

shows the relationship between present and past costs for many of the things that “typical” consumers<br />

must buy. This index includes such items as rent, food, transportation, and certain services.<br />

Other indexes track the costs of equipment, and goods and services that are more pertinent<br />

to the engineering disciplines. Table 15–2 is a listing of some of the more common indexes.<br />

TABLE 15–2<br />

Types and Sources of Various Cost Indexes<br />

Type of Index<br />

Overall prices<br />

Consumer (CPI)<br />

Producer (wholesale)<br />

Construction<br />

Chemical plant overall<br />

Equipment, machinery, and supports<br />

Construction labor<br />

Buildings<br />

<strong>Engineering</strong> and supervision<br />

<strong>Engineering</strong> News Record overall<br />

Construction<br />

Building<br />

Common labor<br />

Skilled labor<br />

Materials<br />

EPA treatment plant indexes<br />

Large-city advanced treatment (LCAT)<br />

Small-city conventional treatment (SCCT)<br />

Federal highway<br />

Contractor cost<br />

Equipment<br />

Marshall and Swift (M&S) overall<br />

M&S specific industries<br />

Labor<br />

Output per worker-hour by industry<br />

Source<br />

Bureau of Labor Statistics<br />

U.S. Department of Labor<br />

Chemical <strong>Engineering</strong><br />

<strong>Engineering</strong> News Record (ENR)<br />

Environmental Protection Agency<br />

and ENR<br />

Marshall & Swift<br />

U.S. Department of Labor


392 Chapter 15 Cost Estimation and Indirect Cost Allocation<br />

TABLE 15–3<br />

Values for Selected Indexes<br />

Year<br />

CE Plant<br />

Cost Index<br />

ENR Construction<br />

Cost Index<br />

M&S Equipment<br />

Cost Index<br />

1995 381.1 5471 1027.5<br />

1996 381.8 5620 1039.2<br />

1997 386.5 5826 1056.8<br />

1998 389.5 5920 1061.9<br />

1999 390.6 6059 1068.3<br />

2000 394.1 6221 1089.0<br />

2001 394.3 6343 1093.9<br />

2002 395.6 6538 1104.2<br />

2003 401.7 6694 1123.6<br />

2004 444.2 7115 1178.5<br />

2005 468.2 7446 1244.5<br />

2006 499.6 7751 1302.3<br />

2007 525.4 7967 1373.3<br />

2008 575.4 8310 1449.3<br />

2009 521.9 8570 1468.6<br />

2010 (midyear) 555.3 8837 1461.3<br />

The general equation for updating costs through the use of a cost index over a period from<br />

time t 0 (base) to another time t is<br />

EXAMPLE 15.2<br />

where C t estimated cost at present time t<br />

C 0 cost at previous time t 0<br />

I t index value at time t<br />

I 0 index value at time t 0<br />

C t C 0 ( I t<br />

—<br />

I 0<br />

) [15.2]<br />

Generally, the indexes for equipment and materials are made up of a mix of components that<br />

are assigned certain weights, with the components sometimes further subdivided into more basic<br />

items. For example, the equipment, machinery, and support component of the chemical plant cost<br />

index is subdivided into process machinery, pipes, valves and fittings, pumps and compressors,<br />

and so forth. These subcomponents, in turn, are built up from even more basic items such as pressure<br />

pipe, black pipe, and galvanized pipe. Table 15–3 shows the Chemical <strong>Engineering</strong> plant<br />

cost index (CEPCI), the <strong>Engineering</strong> News Record (ENR) construction cost index, and the<br />

Marshall and Swift (M&S) equipment cost index for several years. The base period of 1957 to<br />

1959 is assigned a value of 100 for the CEPCI, 1913 100 for the ENR index, and 1926 100<br />

for the M&S equipment cost index.<br />

Current and past values of several of the indexes may be obtained from the Internet (usually<br />

for a fee). For example, the CE plant cost index is available at www.che.compci. The ENR construction<br />

cost index is found at www.construction.com. This latter site offers a comprehensive<br />

series of construction-related resources, including several ENR cost indexes and cost estimation<br />

systems. A website used by many engineering professionals in the form of a “technical chat<br />

room” for all types of topics, including estimation, is www.eng-tips.com.<br />

In evaluating the feasibility of a major construction project, an engineer is interested in estimating<br />

the cost of skilled labor for the job. The engineer finds that a project of similar complexity<br />

and magnitude was completed 5 years ago at a skilled labor cost of $360,000. The ENR<br />

skilled labor index was 3496 then and is now 5127. What is the estimated skilled labor cost for<br />

the new project?


15.3 Cost Indexes 393<br />

Solution<br />

The base time t 0 is 5 years ago. Using Equation [15.2], the present cost estimate is<br />

C r 360,000 (<br />

——— 5127<br />

3496 )<br />

$527,952<br />

In the manufacturing and service industries, tabulated cost indexes are not readily available.<br />

The cost index will vary, perhaps with the region of the country, the type of product or service,<br />

and many other factors. When estimating costs for a manufacturing system, for example, it is<br />

often necessary to develop the cost index for high-priority items such as subcontracted components,<br />

selected materials, and labor costs. The development of the cost index requires the actual<br />

cost at different times for a prescribed quantity and quality of the item. The base period is a selected<br />

time when the index is defined with a basis value of 100 (or 1). The index each year (period)<br />

is determined as the cost divided by the base-year cost and multiplied by 100 (or 1). Future<br />

index values may be forecast using simple extrapolation or more refined mathematical techniques,<br />

such as time-series analysis.<br />

EXAMPLE 15.3<br />

Sean, owner of Alamo Pictures, makes fact-based documentaries about the Old West and<br />

sells them in a variety of outlets, by mail, and online. He has decided to expand into new<br />

areas and wants to make cost estimates for three of the more significant labor costs involved<br />

in making these types of films. The Director of Finance of Alamo Pictures generated the<br />

annual average hourly costs (Table 15–4).<br />

(a) Make 2008 the base year, and determine the cost indexes using a basis of 1.00. Comment<br />

on the trend of each index over the years.<br />

(b) Sean expects to utilize a lot of graphics services in 2014 for a planned documentary; however,<br />

it is the most rapidly increasing cost component. The cost in 2010 was $78 per hour;<br />

assume a worst-case scenario is that the graphics index continues the same arithmetic trend<br />

it had from 2010 to 2011. Determine the hourly cost that he should budget for in 2014.<br />

Solution<br />

(a) For each type of service, calculate I t I 0 where t 2005, 2006, . . . with 2008 as the base<br />

year 0. Table 15–5 presents the indexes. Observations about trend are as follows:<br />

Graphics labor cost: Constantly increasing over all years.<br />

Stuntmen labor cost: Rising until 2009, then stable.<br />

Actors labor cost: Higher in 2007 and 2008; comparatively lower and stable in other years.<br />

TABLE 15–4 Average Hourly Costs for Three Services, Example 15.3<br />

Cost of Service, Average $ per Hour<br />

Type of Service 2005 2006 2007 2008 2009 2010 2011<br />

Graphics 50 56 65 67 70 78 90<br />

Stuntmen 50 55 60 70 87 83 85<br />

Actors 80 80 90 90 80 75 85<br />

TABLE 15–5 Index Values with 2008 as Base Year, Example 15.3<br />

Index I t I 0<br />

Type of Service 2005 2006 2007 2008 2009 2010 2011<br />

Graphics 0.75 0.84 0.97 1.00 1.04 1.16 1.34<br />

Stuntmen 0.71 0.79 0.86 1.00 1.24 1.19 1.21<br />

Actors 0.89 0.89 1.00 1.00 0.89 0.83 0.94


394 Chapter 15 Cost Estimation and Indirect Cost Allocation<br />

(b) The index in 2010 is 1.16. The increase to 2011 is 0.18; the index value in 2014 will be<br />

1.34 3(0.18) 1.88. Equation [15.2] finds the expected, worst-case cost in 2014.<br />

C 2014 C 2010 (I 2014 I 2010 ) 78(1.881.16)<br />

78(1.62)<br />

$126 per hour<br />

15.4 Cost-Estimating Relationships:<br />

Cost-Capacity Equations<br />

Design variables (speed, weight, thrust, physical size, etc.) for plants, equipment, and construction<br />

are determined in the early design stages. Cost-estimating relationships (CERs) use these<br />

design variables to predict costs. Thus, a CER is generically different from the cost index method,<br />

because the index is based on the cost history of a defined quantity and quality of a variable.<br />

One of the most widely used CER models is a cost-capacity equation . As the name implies,<br />

an equation relates the cost of a component, system, or plant to its capacity. This is also known<br />

as the power law and sizing model . Since many cost-capacity equations plot as a straight line on<br />

log-log paper, a common form is<br />

C 2 C 1 ( Q x<br />

2<br />

——<br />

Q 1<br />

)<br />

[15.3]<br />

where C 1 cost at capacity Q 1<br />

C 2 cost at capacity Q 2<br />

x correlating exponent<br />

The value of the exponent for various components, systems, or entire plants can be obtained or<br />

derived from a number of sources, including Plant Design and Economics for Chemical Engineers,<br />

Preliminary Plant Design in Chemical <strong>Engineering</strong>, Chemical Engineers’ Handbook,<br />

technical journals (especially Chemical <strong>Engineering</strong> ), the U.S. Environmental Protection Agency,<br />

professional or trade organizations, consulting firms, handbooks, and equipment companies.<br />

Table 15–6 is a partial listing of typical values of the exponent for various units. When an exponent<br />

value for a particular unit is not known, it is common practice to use the value of x 0.6. In<br />

fact, in the chemical processing industry, Equation [15.3] is referred to as the six-tenths model.<br />

The exponent x in the cost-capacity equation is commonly in the range 0 x 1.<br />

If x 1, economies of scale provide a cost advantage for larger sizes.<br />

If x 1, a linear relationship is present.<br />

If x 1, there are diseconomies of scale present in that a larger size is more costly than that<br />

of a linear relation.<br />

It is especially powerful to combine the time adjustment of the cost index ( I t I 0 ) from Equation<br />

[15.2] with a cost-capacity equation to estimate costs that change over time. If the index is<br />

embedded into the cost-capacity computation in Equation [15.3], the cost at time t and capacity<br />

level 2 may be written as the product of two independent terms.<br />

C 2, t (cost at time 0 of level 2) (time adjustment cost index)<br />

<br />

[<br />

C 1,0 ( Q x<br />

2<br />

——<br />

Q 1<br />

) I 0<br />

)<br />

] ( I t<br />

—<br />

This is commonly expressed without the time subscripts. Thus,<br />

C 2 C 1 ( Q x<br />

2<br />

——<br />

Q 1<br />

)<br />

The following example illustrates the use of this relation.<br />

( I t<br />

—<br />

I 0<br />

) [15.4]


15.5 Cost-Estimating Relationships: Factor Method 395<br />

TABLE 15–6<br />

Sample Exponent Values for Cost-Capacity Equations<br />

ComponentSystemPlant Size Range Exponent<br />

Activated sludge plant 1–100 MGD 0.84<br />

Aerobic digester 0.2–40 MGD 0.14<br />

Blower 1000–7000 ftmin 0.46<br />

Centrifuge 40–60 in 0.71<br />

Chlorine plant 3000–350,000 tonsyear 0.44<br />

Clarifier 0.1–100 MGD 0.98<br />

Compressor, reciprocating (air service) 5–300 hp 0.90<br />

Compressor 200–2100 hp 0.32<br />

Cyclone separator 20–8000 ft 3 min 0.64<br />

Dryer 15–400 ft 2 0.71<br />

Filter, sand 0.5–200 MGD 0.82<br />

Heat exchanger 500–3000 ft 2 0.55<br />

Hydrogen plant 500–20,000 scfd 0.56<br />

Laboratory 0.05–50 MGD 1.02<br />

Lagoon, aerated 0.05–20 MGD 1.13<br />

Pump, centrifugal 10–200 hp 0.69<br />

Reactor 50–4000 gal 0.74<br />

Sludge drying beds 0.04–5 MGD 1.35<br />

Stabilization pond 0.01–0.2 MGD 0.14<br />

Tank, stainless 100–2000 gal 0.67<br />

Note: MGD million gallons per day; hp horsepower; scfd standard cubic feet per day.<br />

EXAMPLE 15.4<br />

The total design and construction cost for a digester to handle a flow rate of 0.5 million gallons<br />

per day (MGD) was $1.7 million in 2010. Estimate the cost today for a flow rate of 2.0 MGD.<br />

The exponent from Table 15–6 for the MGD range of 0.2 to 40 is 0.14. The cost index in 2010<br />

of 131 has been updated to 225 for this year.<br />

Solution<br />

Equation [15.3] can estimate the cost of the larger system in 2010, but it must be updated by<br />

the cost index to today’s dollars. Equation [15.4] performs both operations at once. The estimated<br />

cost is<br />

C 2 1,700,000 (<br />

—— 2.0<br />

0.5 ) 0.14 (<br />

—— 225<br />

131 )<br />

1,700,000(1.214)(1.718) $3.546 million<br />

15.5 Cost-Estimating Relationships:<br />

Factor Method<br />

Another widely used model for preliminary cost estimates of process plants is called the factor<br />

method . While the methods discussed above can be used to estimate the costs of major items of<br />

equipment, processes, and the total plant costs, the factor method was developed specifically for<br />

total plant costs . The method is based on the premise that fairly reliable total plant costs can be<br />

obtained by multiplying the cost of the major equipment by certain factors. Since major equipment<br />

costs are readily available, rapid plant estimates are possible if the appropriate factors are<br />

known. These factors are commonly referred to as Lang factors after Hans J. Lang, who first<br />

proposed the method.


396 Chapter 15 Cost Estimation and Indirect Cost Allocation<br />

In its simplest form, the factor method is expressed in the same form as the unit method<br />

C T hC E [15.5]<br />

DirectIndirect costs<br />

where C T total plant cost<br />

h overall cost factor or sum of individual cost factors<br />

C E total cost of major equipment<br />

The h may be one overall cost factor or, more realistically, the sum of individual cost components<br />

such as construction, maintenance, direct labor, materials, and indirect cost elements. This follows<br />

the cost estimation approaches presented in Figure 15–1.<br />

In his original work, Lang showed that direct cost factors and indirect cost factors can be<br />

combined into one overall factor for some types of plants as follows: solid process plants, 3.10;<br />

solid-fluid process plants, 3.63; and fluid process plants, 4.74. These factors reveal that the total<br />

installed-plant cost is many times the first cost of the major equipment.<br />

EXAMPLE 15.5<br />

An engineer with Valero Petroleum has learned that an expansion of the solid-fluid process<br />

plant is expected to have a delivered equipment cost of $2.08 million. If the overall cost factor<br />

for this type of plant is 3.63, estimate the plant’s total cost.<br />

Solution<br />

The total plant cost is estimated by Equation [15.5].<br />

C T 3.63(2,080,000)<br />

$7,550,400<br />

Subsequent refinements of the factor method have led to the development of separate factors<br />

for direct and indirect cost components. Direct costs as discussed in Section 15.1 are specifically<br />

identifiable with a product, function, or process. Indirect costs are not directly attributable to a<br />

single function, but are shared by several because they are necessary to perform the overall objective.<br />

Examples of indirect costs are general administration, computer services, quality, safety,<br />

taxes, security, and a variety of support functions. The factors for both direct and indirect costs<br />

are sometimes developed for use with delivered-equipment costs and other times for installedequipment<br />

costs, as defined in Section 15.1. In this text, we assume that all factors apply to delivered-equipment<br />

costs, unless otherwise specified.<br />

For indirect costs, some of the factors apply to equipment costs only, while others apply to the<br />

total direct cost. In the former case, the simplest procedure is to add the direct and indirect cost<br />

factors before multiplying by the delivered-equipment cost. The overall cost factor h can be<br />

written as<br />

n<br />

h 1 f i [15.6]<br />

where f i factor for each cost component<br />

i 1 to n components, including indirect cost<br />

If the indirect cost factor is applied to the total direct cost, only the direct cost factors are added<br />

to obtain h . Therefore, Equation [15.5] is rewritten as<br />

C T <br />

[<br />

C E (<br />

where f I indirect cost factor<br />

f i factors for direct cost components<br />

Examples 15.6 and 15.7 illustrate these equations.<br />

i1<br />

n<br />

1 f i ) ] (1 f I ) [15.7]<br />

i1


15.6 Traditional Indirect Cost Rates and Allocation 397<br />

EXAMPLE 15.6<br />

The delivered-equipment cost for a small chemical process plant is expected to be $2 million.<br />

If the direct cost factor is 1.61 and the indirect cost factor is 0.25, determine the total plant cost.<br />

Solution<br />

Since all factors apply to the delivered-equipment cost, they are added to obtain h, the total cost<br />

factor in Equation [15.6].<br />

The total plant cost is<br />

h 1 1.61 0.25 2.86<br />

C T 2.86(2,000,000) $5,720,000<br />

EXAMPLE 15.7<br />

A new container-handling crane at the Port of Singapore is expected to have a deliveredequipment<br />

cost of $875,000. The cost factor for the installation of tracks, concrete, steel, noise<br />

abatement, supports, etc., is 0.49. The construction factor is 0.53, and the indirect cost factor is<br />

0.21. Determine the total cost if (a) all cost factors are applied to the cost of the delivered<br />

equipment and (b) the indirect cost factor is applied to the total direct cost.<br />

Solution<br />

(a) Total equipment cost is $875,000. Since both the direct and indirect cost factors are applied<br />

to only the equipment cost, the overall cost factor from Equation [15.6] is<br />

The total cost is<br />

h 1 0.49 0.53 0.21 2.23<br />

C T 2.23(875,000) $1,951,250<br />

(b) Now the total direct cost is calculated first, and Equation [15.7] is used to estimate total<br />

cost.<br />

n<br />

h 1 f i 1 0.49 0.53 2.02<br />

i1<br />

C T [875,000(2.02)](1.21) $2,138,675<br />

Comment<br />

Note the decrease in estimated cost when the indirect cost is applied to the equipment cost only<br />

in part (a). This illustrates the importance of determining exactly what the factors apply to<br />

before they are used.<br />

15.6 Traditional Indirect Cost Rates and Allocation<br />

Costs incurred in the production of an item or delivery of a service are tracked and assigned by a<br />

cost accounting system . For the manufacturing environment, it can be stated generally that the<br />

statement of cost of goods sold (discussed in Appendix B) is one end product of this system. The<br />

cost accounting system accumulates material costs, labor costs, and indirect costs (also called<br />

overhead costs or factory expenses) by using cost centers. All costs incurred in one department<br />

or process line are collected under a cost center title, for example, Department 3X. Since direct<br />

materials and direct labor are usually directly assignable to a cost center, the system need only<br />

identify and track these costs. Of course, this in itself is no easy chore, and the cost of the tracking<br />

system may prohibit collection of all direct cost data to the level of detail desired.<br />

One of the primary and more difficult tasks of cost accounting is the allocation of indirect costs<br />

when it is necessary to allocate them separately to departments, processes, and product lines.


398 Chapter 15 Cost Estimation and Indirect Cost Allocation<br />

TABLE 15–7<br />

Sample Indirect Cost Allocation Bases<br />

Cost Category<br />

Taxes<br />

Heat, light<br />

Power<br />

Receiving, purchasing<br />

Personnel, machine shop<br />

Building maintenance<br />

Software<br />

Quality control<br />

Possible Allocation Basis<br />

Space occupied<br />

Space, usage, number of outlets<br />

Space, direct labor hours, horsepower-hours, machine hours<br />

Cost of materials, number of orders, number of items<br />

Direct labor hours, direct labor cost<br />

Space occupied, direct labor cost<br />

Number of accesses<br />

Number of inspections<br />

Indirect costs<br />

Indirect costs are costs associated with property taxes, service and maintenance departments,<br />

personnel, legal, quality, supervision, purchasing, utilities, software development,<br />

etc. They must all be allocated to the using cost center. Detailed collection of these data is<br />

cost- prohibitive and often impossible; thus, allocation schemes are utilized to distribute the<br />

expenses on a reasonable basis.<br />

A listing of possible allocation bases is included in Table 15–7. Historically, common bases have<br />

been direct labor cost, direct labor hours, machine-hours, number of employees, space, and direct<br />

materials.<br />

Most allocation is accomplished utilizing a predetermined indirect cost rate, computed by<br />

using the general relation.<br />

Indirect cost rate —————————————<br />

estimated total indirect costs<br />

estimated basis level<br />

[15.8]<br />

EXAMPLE 15.8<br />

The estimated indirect cost is the amount allocated to a cost center. For example, if a division has<br />

two producing departments, the total indirect cost allocated to a department is used as the numerator<br />

in Equation [15.8] to determine the department rate. Example 15.8 illustrates allocation<br />

when the cost center is a machine.<br />

The manager of beauty products at BestWay wants to determine allocation rates for $150,000<br />

of indirect costs for the three machines used to process beauty lotions. The following information<br />

was obtained from last year’s budget for the three machines. Determine rates for each<br />

machine if the amount is equally distributed.<br />

Cost Source Allocation Basis Estimated Activity Level<br />

Machine 1 Direct labor cost $100,000<br />

Machine 2 Direct labor hours 2000 hours<br />

Machine 3 Direct material cost $250,000<br />

Solution<br />

Applying Equation [15.8] for each machine, the annual rates are<br />

indirect budget<br />

Machine 1 rate ———————<br />

direct labor cost ————<br />

50,000<br />

100,000<br />

$0.50 per direct labor dollar<br />

indirect budget<br />

Machine 2 rate ————————<br />

direct labor hours ———<br />

50,000<br />

2000<br />

$25 per direct labor hour<br />

indirect budget<br />

Machine 3 rate ——————— ————<br />

50,000<br />

material cost 250,000<br />

$0.20 per direct material dollar


15.6 Traditional Indirect Cost Rates and Allocation 399<br />

Now the actual direct labor costs and hours and material costs are determined for this<br />

year, and each dollar of direct labor cost spent on machine 1 implies that $0.50 in indirect<br />

cost will be added to the cost of the product. Similarly, indirect costs are added for machines<br />

2 and 3.<br />

When the same allocation basis is used to distribute indirect costs to several cost centers, a<br />

blanket rate may be determined. For example, if direct materials are the basis for allocation to<br />

four separate processing lines, the blanket rate is<br />

Indirect cost rate ———————————<br />

total indirect costs<br />

total direct materials cost<br />

If $500,000 in indirect costs and $3 million in materials are estimated for the four lines, the blanket<br />

indirect rate is 500,0003,000,000 $0.167 per materials cost dollar. Blanket rates are easier<br />

to calculate and apply, but they do not account for differences in the type of activities accomplished<br />

in each cost center.<br />

In most cases, machinery or processes add value to the end product at different rates per unit<br />

or hour of use. For example, light machinery may contribute less per hour than heavy, more expensive<br />

machinery. This is especially true when advanced technology processing, for example,<br />

an automated manufacturing cell, is used along with traditional methods, for example, nonautomated<br />

finishing equipment. The use of blanket rates in these cases is not recommended, as the<br />

indirect cost will be incorrectly allocated. The lower-value-contribution machinery will accumulate<br />

too much of the indirect cost. The approach should be the application of different bases for<br />

different machines, activities, etc., as discussed earlier and illustrated in Example 15.8. The use<br />

of different, appropriate bases is often called the productive hour rate method since the cost<br />

rate is determined based on the value added, not a uniform or blanket rate. Realization that more<br />

than one basis should be normally used in allocating indirect costs has led to the use of activitybased<br />

costing methods, as discussed in the next section.<br />

Once a period of time (month, quarter, or year) has passed, the indirect cost rates are applied<br />

to determine the indirect cost charge, which is then added to direct costs. This results in the total<br />

cost of production, which is called the cost of goods sold, or factory cost . These costs are all accumulated<br />

by cost center .<br />

If the total indirect cost budget is correct, the indirect costs charged to all cost centers for the<br />

period of time should equal this budget amount. However, since some error in budgeting always<br />

exists, there will be overallocation or underallocation relative to actual charges, which is termed<br />

allocation variance . Experience in indirect cost estimation assists in reducing the variance at the<br />

end of the accounting period.<br />

EXAMPLE 15.9<br />

Since the manager determined indirect cost rates for BestWay (Example 15.8), she can now<br />

compute the total factory cost for a month. Perform the computations using the data in<br />

Table 15–8. Also calculate the variance for indirect cost allocation for the month.<br />

Solution<br />

Start with the cost of goods sold (factory cost) relation given by Equation [B.1] in Appendix B,<br />

which is<br />

Cost of goods sold direct materials direct labor indirect costs<br />

TABLE 15–8<br />

Actual Monthly Data Used for Indirect Cost Allocation<br />

Cost Source Machine Number Actual Cost, $ Actual Hours<br />

Material 1 3,800<br />

3 19,550<br />

Labor 1 2,500 650<br />

2 3,200 750<br />

3 2,800 720


400 Chapter 15 Cost Estimation and Indirect Cost Allocation<br />

To determine indirect cost, the rates from Example 15.8 are applied:<br />

Machine 1 indirect (labor cost)(rate) 2500(0.50) $1250<br />

Machine 2 indirect (labor hours)(rate) 750(25.00) $18,750<br />

Machine 3 indirect (material cost)(rate) 19,550(0.20) $3910<br />

Total charged indirect cost $23,910<br />

Factory cost is the sum of actual material and labor costs from Table 15–8 of $31,850 and the<br />

indirect cost charge for a total of $55,760.<br />

Based on the annual indirect cost budget of $150,000, one month represents 112 of the<br />

total or<br />

Monthly budget ————<br />

150,000<br />

12<br />

$12,500<br />

The allocation variance for total indirect cost is<br />

Variance 12,500 23,910 $11,410<br />

This is a large budget underallocation, since much more was actually charged than allocated.<br />

The $12,500 budgeted for the three machines represents a 91.3% underallocation of indirect<br />

costs. This analysis for only one month of a year will most likely prompt a rapid review of the<br />

rates and the indirect cost budget.<br />

EXAMPLE 15.10<br />

Once estimates of indirect costs are determined, it is possible to perform an economic analysis of<br />

the present operation versus a proposed operation. Such a study is described in Example 15.10.<br />

For several years, Cuisinart Corporation has purchased the carafe assembly of its major coffeemaker<br />

line at an annual cost of $2.2 million. The suggestion to make the component in-house<br />

has been made. For the three departments involved, the annual indirect cost rates, estimated<br />

material, labor, and hours are found in Table 15–9. The allocated hours column is the time<br />

necessary to produce the carafes for a year.<br />

Equipment must be purchased with the following estimates: first cost of $2 million, salvage<br />

value of $50,000, and life of 10 years. Perform an economic analysis for the make alternative,<br />

assuming that a market rate of 15% per year is the MARR.<br />

Solution<br />

For making the components in-house, the AOC is comprised of direct labor, direct material,<br />

and indirect costs. Use the data of Table 15–9 to calculate the indirect cost allocation.<br />

Department<br />

Department A: 25,000(10) $250,000<br />

Department B: 25,000(5) 125,000<br />

Department C: 10,000(15) 150,000<br />

$525,000<br />

AOC 500,000 300,000 525,000 $1,325,000<br />

TABLE 15–9 Production Cost Estimates for Example 15.10<br />

Basis,<br />

Hours<br />

Rate<br />

per Hour, $<br />

Indirect Costs<br />

Allocated<br />

Hours<br />

Material<br />

Cost, $<br />

Direct<br />

Labor<br />

Cost, $<br />

A Labor 10 25,000 200,000 200,000<br />

B Machine 5 25,000 50,000 200,000<br />

C Labor 15 10,000 50,000 100,000<br />

300,000 500,000


15.7 Activity-Based Costing (ABC) for Indirect Costs 401<br />

The make alternative annual worth is<br />

AW make P(AP,i,n) S(AF,i,n) AOC<br />

2,000,000(AP,15%,10) 50,000(AF,15%,10) 1,325,000<br />

$1,721,037<br />

Currently, the carafes are purchased with an AW of<br />

AW buy $2,200,000<br />

It is cheaper to make, because the AW of costs is less.<br />

15.7 Activity-Based Costing (ABC) for Indirect Costs<br />

As automation, software, and manufacturing technologies have advanced, the number of<br />

direct labor hours necessary to manufacture a product has decreased substantially. Where<br />

once as much as 35% to 45% of the final product cost was represented in labor, now the<br />

labor component is commonly 5% to 15% of total manufacturing cost. However, the indirect<br />

cost may represent as much as 35% to 45% of the total manufacturing cost. The use of<br />

bases, such as direct labor hours, to allocate indirect cost is not accurate enough for automated<br />

and technologically advanced environments. This has led to the development of<br />

methods that replace or supplement traditional cost allocations that rely upon one form or<br />

another of Equation [15.8]. Also, allocation bases different from traditional ones are commonly<br />

utilized.<br />

A product that by traditional methods may have contributed a large portion to profit may actually<br />

be a loser when indirect costs are allocated more correctly. Companies that have a wide variety<br />

of products and produce some in small lots may find that traditional allocation methods have<br />

a tendency to underallocate the indirect cost to small-lot products. This may indicate that they are<br />

profitable, when in actuality they are losing money.<br />

The best allocation method for high-overhead industries is Activity-Based Costing (ABC). It<br />

is designed to identify cost centers, activities, and cost drivers. Descriptions of each follow.<br />

Cost centers: The final products or services of the corporation are called cost centers or cost<br />

pools. They receive the allocated indirect costs.<br />

Activities: These are usually support departments (purchasing, quality, IT, maintenance, engineering,<br />

supervision) that generate the indirect costs which are then distributed to the cost<br />

centers.<br />

Cost drivers: Commonly expressed in volumes, these drive the consumption of a shared resource.<br />

Examples are the number of purchase orders, cost of engineering change orders,<br />

number of machine setups, number of safety violations, and the like.<br />

Implementing ABC involves several steps.<br />

1. Identify each activity and its total cost.<br />

2. Identify the cost drivers and their usage volumes.<br />

3. Calculate the indirect cost rate for each activity.<br />

ABC indirect cost rate <br />

total cost of activity<br />

————————————<br />

total volume of cost driver<br />

[15.9]<br />

4. Use the rate to allocate indirect cost to cost centers for each activity.<br />

As an illustration, assume a company that produces two types of industrial lasers (cost centers)<br />

has three primary support departments (activities; step 1 in the procedure). The allocation<br />

for costs generated by the purchasing department, for example, is based on the number of purchase<br />

orders (step 2) to support laser production. The ABC rate (step 3) in dollars per purchase<br />

order is used to allocate indirect costs to the two laser products (step 4).


402 Chapter 15 Cost Estimation and Indirect Cost Allocation<br />

EXAMPLE 15.11<br />

A multinational aerospace firm uses traditional methods to allocate manufacturing and management<br />

support costs for its European division. However, accounts such as business travel<br />

have historically been allocated on the basis of the number of employees at the plants in France,<br />

Italy, Germany, and Spain.<br />

The president recently stated that some product lines are likely generating much more travel<br />

than others. The ABC system is chosen to augment the traditional method to more precisely<br />

allocate travel costs to major product lines at each plant.<br />

(a) First, assume that allocation of total observed travel expenses of $500,000 to the plants<br />

using a traditional basis of workforce size is sufficient. If total employment of 29,100 is<br />

distributed as follows, allocate the $500,000.<br />

Paris, France plant<br />

12,500 employees<br />

Florence, Italy plant 8,600 employees<br />

Hamburg, Germany plant 4,200 employees<br />

Barcelona, Spain plant 3,800 employees<br />

(b) Now, assume that corporate management wants to know more about travel expenses based<br />

on product line, not merely plant location and workforce size. The ABC method will be<br />

applied to allocate travel costs to major product lines. Annual plant support budgets indicate<br />

that the following percentages are expended for travel:<br />

Paris 5% of $2 million<br />

Florence 15% of $500,000<br />

Hamburg 17.5% of $1 million<br />

Barcelona 30% of $500,000<br />

Further, the study indicates that in 1 year a total of 500 travel vouchers were processed by<br />

the management of the major five product lines produced at the four plants. The distribution<br />

is as follows:<br />

Paris Product lines—1 and 2; number of vouchers—50 for line 1, 25 for 2.<br />

Florence Product lines—1, 3, and 5; vouchers—80 for line 1, 30 for 3, 30 for 5.<br />

Hamburg Product lines—1, 2 and 4; vouchers—100 for line 1, 25 for 2, 20 for 4.<br />

Barcelona Product line—5; vouchers—140 for line 5.<br />

Use the ABC method to determine how the product lines drive travel costs at the plants.<br />

Solution<br />

(a) In this case, Equation [15.8] takes the form of a blanket rate per employee.<br />

travel budget<br />

Indirect cost rate ———————<br />

total workforce<br />

————<br />

$500,000 $17.1821 per employee<br />

29,100<br />

Using this traditional basis of rate times workforce size results in a plant-by-plant allocation.<br />

Paris $17.1821(12,500) $214,777<br />

Florence $147,766<br />

Hamburg $72,165<br />

Barcelona $65,292<br />

(b) The ABC method is more involved to apply, and the by-plant amounts will be different<br />

from those in part (a) since completely different bases are applied. Use the 4-step procedure<br />

to allocate travel costs to the five products.<br />

Step 1. The total amount to be allocated is determined from the percentages of each<br />

plant’s support budget devoted to travel. The number is determined from the<br />

percent-of-budget data as follows:<br />

0.05(2,000,000) . . . 0.30(500,000) $500,000


15.8 Making Estimates and Maintaining Ethical Practices 403<br />

TABLE 15–10 ABC Allocation of Travel Cost ($ in Thousands), Example 15.11<br />

Product Line<br />

1 2 3 4 5 Total<br />

Paris 50 25 75<br />

Florence 80 30 30 140<br />

Hamburg 100 25 20 145<br />

Barcelona 140 140<br />

Total $230 $50 $30 $20 $170 $500<br />

Step 2.<br />

The cost driver is the number of travel vouchers submitted by the management<br />

unit responsible for each product line at each plant. The allocation will be to<br />

the products directly, not to the plants. However, the travel allocation to the<br />

plants can be determined afterward since we know what product lines are<br />

produced at each plant.<br />

Step 3. Equation [15.9] determines an ABC allocation rate.<br />

ABC allocation rate ———————————<br />

total cost of travel<br />

total number of vouchers<br />

————<br />

$500,000<br />

500<br />

$1000 per voucher<br />

Step 4.<br />

Table 15–10 summarizes the vouchers and allocation by product line and by city.<br />

Product 1 ($230,000) and product 5 ($170,000) drive the travel costs based on<br />

the ABC analysis. Comparison of the by-plant totals in Table 15–10 (far right<br />

column) with the respective totals in part (a) indicates a substantial difference in<br />

the amounts allocated, especially to Paris, Hamburg, and Barcelona. This comparison<br />

verifies the president’s statement that product lines, not plants, drive<br />

travel requirements.<br />

Comment<br />

Let’s assume that product 1 has been produced in small lots at the Hamburg plant for a number<br />

of years. This analysis, when compared to the traditional cost allocation method in part (a), reveals<br />

a very interesting fact. In the ABC analysis, Hamburg has a total of $145,000 travel dollars<br />

allocated, $100,000 from product 1. In the traditional analysis based on workforce size, Hamburg<br />

was allocated only $72,165—about 50% of the more precise ABC analysis amount. This indicates<br />

to management the need to examine the manufacturing lot size practices at Hamburg and<br />

possibly other plants, especially when a product is currently manufactured at more than one plant.<br />

Some proponents of the ABC method recommend discarding the traditional cost accounting<br />

methods of a company and utilizing ABC exclusively. This is not a good approach, since ABC is<br />

not a complete cost system. The ABC method provides information that assists in cost control,<br />

while the traditional method emphasizes cost allocation and cost estimation. The two systems<br />

work well together with the traditional methods allocating costs that have identifiable direct<br />

bases, for example, direct labor. ABC analysis is usually more expensive and time-consuming<br />

than a traditional cost allocation system, but in many cases it can assist in understanding the<br />

economic impact of management decisions and in controlling certain types of indirect costs.<br />

15.8 Making Estimates and Maintaining<br />

Ethical Practices<br />

Making estimates about the future of costs, revenues, cash flows, rates of return, and many other<br />

parameters is routine when one is engaged in any type of economic analysis. Public agencies,<br />

private corporations, and not-for-profit businesses all make economic decisions based on these


404 Chapter 15 Cost Estimation and Indirect Cost Allocation<br />

estimates, most of which are made by employees of the organizations or by outside consultants<br />

hired to perform specific activities under contract. The opportunities for bias, poor accuracy,<br />

deception, and profit-driven or other motives are always present. The personal morals and adherence<br />

to codes of professional ethics discussed in Section 1.3 guide individuals in their work to<br />

make fair and believable estimates for the analyses and decisions that follow.<br />

The NSPE Code of Ethics for Engineers (Appendix C) referenced previously starts with a list<br />

of six Fundamental Canons. One very relevant to estimation integrity is “Avoid deceptive acts.”<br />

Acts that bias the results from experimental samples, previous cost data, or survey results for the<br />

purposes of personal gain, increased profits, or favoritism are examples of unethical behavior.<br />

Estimates of all types should be founded on practices such as the following:<br />

Base estimates on sound information gathered over a range of situations representative of the<br />

current one.<br />

Use accepted theory and techniques in taking statistical samples, building budget elements,<br />

and drawing conclusions that are included in proposals, applications, and recommendations.<br />

As a consultant or contractor, keep personal and working relationships separate when making<br />

estimates and delivering the final documentation to a client or sponsor.<br />

The second case study at the end of this chapter presents an example of some ethical challenges<br />

present when preparing estimates and proposals for contract work.<br />

CHAPTER SUMMARY<br />

Cost estimates are not expected to be exact, but they should be accurate enough to support a<br />

thorough economic analysis using an engineering economy approach. There are bottom-up and<br />

top-down approaches; each treats price and cost estimates differently.<br />

Costs can be updated via a cost index, which is a ratio of costs for the same item at two separate<br />

times. The Consumer Price Index (CPI) is an often-quoted example of cost indexing. Cost<br />

estimating may also be accomplished with a variety of models called cost-estimating relationships.<br />

Two of them are<br />

Cost-capacity equation— good for estimating costs from design variables for equipment, materials,<br />

and construction<br />

Factor method— good for estimating total plant cost<br />

Traditional cost allocation uses an indirect cost rate determined for a machine, department, product<br />

line, etc. Bases such as direct labor cost, direct material cost, and direct labor hours are used.<br />

With increased automation and information technology, different techniques of indirect cost allocation<br />

have been developed. The Activity-Based Costing method is an excellent technique to<br />

augment the traditional allocation method.<br />

The ABC method allocates indirect costs on the rationale that purchase orders, inspections,<br />

machine setups, reworks, etc. drive the costs accumulated in departments or functions, such as<br />

quality, purchasing, accounting, and maintenance. Improved understanding of how the company<br />

or plant accumulates indirect costs is a major by-product of implementing the ABC method.<br />

PROBLEMS<br />

Understanding Cost Estimation<br />

15.1 Rank the following estimate types in terms of time<br />

spent to carry out the estimate (most time to least<br />

time): partially designed, design 60% to 100%<br />

complete, order of magnitude, scopingfeasibility,<br />

detailed estimate.<br />

15.2 Classify the following cost elements as first cost<br />

(FC) components or annual operating cost (AOC)<br />

components for a piece of equipment on the<br />

shop floor: supplies, insurance, equipment cost,<br />

utility cost, installation, delivery charges, labor<br />

cost.<br />

15.3 State whether actual (A) or estimated (E) costs are<br />

more likely to be used to carry out the following<br />

activities: calculate taxes, make bids, pay bonuses,<br />

determine profit or loss, predict sales, set prices,


Problems 405<br />

evaluate proposals, distribute resources, plan production,<br />

and set goals.<br />

15.4 Identify the output and input variables in both the<br />

bottom-up and top-down approaches to cost estimating.<br />

15.5 Classify the following costs as typically direct (D)<br />

or indirect (I):<br />

Project staff<br />

Utilities<br />

Raw materials<br />

Project supplies<br />

Administrative staff<br />

Audit and legal<br />

Rent<br />

Equipment training<br />

Labor<br />

Miscellaneous office supplies<br />

15.6 Identify each of the following costs associated<br />

with owning an automobile as direct or indirect<br />

(assume a direct cost is one that is directly attributable<br />

to the number of miles driven): license plate,<br />

driver’s license, gasoline, highway toll fee, oil<br />

change, repairs after collision, gasoline tax,<br />

monthly loan payment, annual inspection fee, and<br />

garage rental.<br />

15.7 In the early and conceptual design stages of a project,<br />

what are the cost estimates called? Approximately<br />

how close should they be to the actual cost?<br />

Unit Costs<br />

15.8 Use the unit cost method to determine the preliminary<br />

cost of a guardrail for a small bridge if a total<br />

of 120 linear feet will be required at a cost of<br />

$58.19 per linear foot for material, equipment, and<br />

labor.<br />

15.9 Sun-Metro Bus is planning to construct a 600-car<br />

parking garage on the outskirts of the city to encourage<br />

people to use the “Park ’n Ride” to work<br />

or for shopping downtown. Prepare a preliminary<br />

cost estimate for the garage if the cost per parking<br />

space is $4700.<br />

15.10 The Office of the Undersecretary of Defense periodically<br />

releases unit cost data for use in military<br />

construction programs. If the unit cost for a satellite<br />

communications center (with shielding and<br />

generator) is $496 per square foot, what would be<br />

the estimated cost of a 6000-square-foot building?<br />

15.11 The Department of Defense uses area cost factors<br />

(ACFs) to adjust for differences in construction costs<br />

in different parts of the country (and world). The<br />

area cost factor for Andros Island in the Bahamas is<br />

1.70 while the ACF for Rapid City, South Dakota, is<br />

0.93. If a cold storage processing warehouse costs<br />

$1,350,000 in Rapid City, estimate the cost for<br />

Andros Island.<br />

15.12 Preliminary cost estimates for jails can be made<br />

using costs based on either unit area (square feet)<br />

or unit volume (cubic feet). If the unit area cost is<br />

$185 per square foot and the average height of the<br />

ceilings is 10 feet, what is the unit volume cost?<br />

15.13 The unit area and unit volume total project costs<br />

for a library are $114 per square foot and $7.55 per<br />

cubic foot, respectively. Based on these numbers,<br />

what is the average height of library rooms?<br />

15.14 The equipment required to place 160 cubic yards of<br />

concrete by experienced workers is two gasoline engine<br />

vibrators and one concrete pump. If the vibrators<br />

cost $76 per day and the concrete pump costs<br />

$580 per day, estimate ( a ) the cost of the equipment<br />

per cubic yard of concrete and ( b ) the equipment<br />

cost for placing 56 cubic yards of concrete.<br />

15.15 A labor crew for placing concrete consists of<br />

1 labor foreman at $25.85 per hour, 1 cement finisher<br />

at $28.60 per hour, 5 laborers at $23.25 per<br />

hour each, and 1 equipment operator at $31.45 per<br />

hour. Such a crew, called a C20 crew, can place 160<br />

cubic yards of concrete per 8-hour day. Determine<br />

( a ) the cost per day of labor for the C20 crew, ( b ) the<br />

cost of the C20 crew per cubic yard of concrete, and<br />

( c ) the cost to place 250 cubic yards of concrete.<br />

15.16 Site work activities associated with constructing a<br />

small bridge are shown in the table below. The<br />

table includes the quantity of each activity, the unit<br />

of measurement associated with each activity, and<br />

the unit cost of each activity. Use the data to determine<br />

( a ) the total cost for structural excavation, ( b )<br />

the total cost for the pile-driving rig, and ( c ) the<br />

total labor cost for the site work.<br />

Activity<br />

Excavation,<br />

unclassified<br />

Excavation,<br />

structural<br />

Backfill,<br />

compacted<br />

Quantity<br />

Unit of<br />

Measurement<br />

Labor<br />

Unit Cost, $<br />

Equipment<br />

Unit Cost, $<br />

Material<br />

Unit Cost, $<br />

1667 cy 1.35 1.43 0<br />

120 cy 21.31 5.00 0<br />

340 cy 7.78 1.72 0<br />

Pile-driving rig job ls 5688 6420 300<br />

Piling, steel, 2240 lf 3.13 2.93 16.57<br />

driving<br />

Legend: cy cubic yard; ls lump sum; lf linear foot<br />

Cost Indexes<br />

15.17 From historical data, you discover that the national<br />

average construction cost of middle schools is<br />

$10,500 per student. If the state index for Texas is<br />

76.9 and for California it is 108.5, estimate the<br />

total construction cost of a middle school for 800<br />

students in each state.


406 Chapter 15 Cost Estimation and Indirect Cost Allocation<br />

15.18 A consulting engineering firm is preparing a preliminary<br />

cost estimate for a design-construct project<br />

of a coal processing plant. The firm, which<br />

completed a similar project in 2001 with a construction<br />

cost of $30 million, wants to use the ENR<br />

construction cost index to update the cost. Use the<br />

values in Table 15–3 to estimate the cost of construction<br />

for a similar-size plant in mid-2010.<br />

15.19 If the editors at ENR decide to redo the construction<br />

cost index so that the year 2000 has a base<br />

value of 100, determine the value for the year ( a )<br />

1995 and ( b ) 2009.<br />

15.20 (a) Estimate the value of the ENR construction<br />

cost index by using the average (compounded)<br />

percentage change in its value between 1995<br />

and 2005 to predict the value in 2009.<br />

(b) How much difference (numerical) is there between<br />

the estimated and actual 2009 values?<br />

Is this an underestimate or overestimate?<br />

15.21 An engineer who owns a construction company<br />

that specializes in large commercial projects noticed<br />

that material costs increased at a rate of 1%<br />

per month over the past 12 months. If a material<br />

cost index were created for each month of this year<br />

with the value of the index set at 100 for the beginning<br />

of the year, what would be the value of the<br />

index at the end of the year? Express your answer<br />

to two decimal places.<br />

15.22 Using 1913 as the base year with a value of 100,<br />

the ENR construction cost index (CCI) for August<br />

2009 was 8563.35. For August 2010, the CCI<br />

value was 8837.38. ( a ) What was the inflation rate<br />

for construction for that 1-year period? ( b ) What<br />

will be the CCI value in August 2013, provided the<br />

inflation rate remains the same as it was in the<br />

2009–2010 period?<br />

15.23 Electropneumatic general-purpose pressure transducers<br />

convert supply pressure to regulated output<br />

pressure in direct proportion to an electrical input<br />

signal. The cost of a certain transducer was $194 in<br />

1985 when the M&S equipment index was at<br />

789.6. If the price increased exactly in proportion<br />

to the M&S equipment index, what would its cost<br />

be in 2010 (midyear) when the index value was<br />

1461.3?<br />

15.24 The ENR construction cost index for New York<br />

City had a value of 12,381.40 in February 2007.<br />

For Pittsburgh and Atlanta, the values were<br />

7341.32 and 4874.06, respectively. If a general<br />

contractor in Atlanta won construction jobs totaling<br />

$54.3 million, determine their equivalent total<br />

value in New York City.<br />

15.25 The ENR construction cost index (CCI) for August<br />

2010 had a value of 8837.37 when the base year<br />

was 1913 with a value of 100. If the base year is<br />

1967 with a value of 1.0, the CCI for August 2010<br />

would be 8.2272. In this case, what is the CCI<br />

value for 1967 when the base year is 1913?<br />

15.26 The ENR materials cost index (MCI) had a value<br />

of 2708.51 in August 2010. In the same month, the<br />

cost for cement was $96.55 per ton. If cement increased<br />

in price exactly in accordance with the<br />

MCI, what was the cost of cement per ton in 1913<br />

when the MCI index value was 100?<br />

15.27 A contractor purchased equipment costing $40,000<br />

in 2010 when the M&S equipment cost index was<br />

at 1461.3. He remembers purchasing the same<br />

equipment for $21,771 many years ago, but he does<br />

not remember the year that he did so. If the M&S<br />

equipment cost index increased by 2.68% per year<br />

over that time period and the equipment increased<br />

in price exactly in proportion to the index, ( a ) in<br />

what year did he purchase the equipment and ( b )<br />

what was the value of the index in that year?<br />

Cost-Capacity and Factor Methods<br />

15.28 If the materials cost index decreased by 2% over<br />

the same time period that the construction cost<br />

index increased by 2%, what does that suggest<br />

about the labor cost index, if labor represents 35%<br />

of the cost of construction?<br />

15.29 Use the exponent values in Table 15–6 to estimate<br />

the cost for the following equipment to be placed<br />

on an offshore drilling platform.<br />

(a) The cost of a 125-hp centrifugal pump if a<br />

200-hp pump costs $28,000.<br />

(b) The cost of a 1700-gallon stainless steel tank<br />

if a 900-gallon tank costs $4100.<br />

15.30 A high-pressure stainless steel pump (1000 psi)<br />

with a variable-frequency drive (VFD) is installed<br />

in a seawater reverse-osmosis pilot plant that is recovering<br />

water from membrane concentrate at a<br />

rate of 4 gallons per minute (gpm). The cost of the<br />

pump was $13,000. Because of favorable results<br />

from the pilot study, the city utility wants to go with<br />

a full-scale system that will produce 500 gpm.<br />

Determine the estimated cost of the larger pump, if<br />

the exponent in the cost-capacity equation has a<br />

value of 0.37.<br />

15.31 A 0.75 million gallon per day (MGD) induceddraft<br />

packed tower for air-stripping trihalomethanes<br />

from drinking water costs $58,890. Estimate<br />

the cost of a 2-MGD tower if the exponent in the<br />

cost-capacity equation is 0.58.


Problems 407<br />

15.32 The on-site manager told you that the variablefrequency<br />

drive (VFD) for a 300-hp motor costs<br />

$20,000. Make the best cost estimate possible for<br />

the VFD for a 100-hp motor. The exponent in the<br />

cost-capacity equation is not available currently<br />

where you are located.<br />

15.33 The cost of a 68 m 2 falling-film evaporator was<br />

1.52 times the cost of the 30 m 2 unit. What exponent<br />

value in the cost-capacity equation yielded<br />

these results?<br />

15.34 Reinforced concrete pipe (RCP) that is 12 inches<br />

in diameter had a cost of $12.54 per linear foot in<br />

Dallas, Texas in 2007. The cost for 24-inch RCP<br />

was $27.23 per foot. If the cross-sectional area of<br />

the pipe is considered the “ capacity ” in the costcapacity<br />

equation, determine the value of the exponent<br />

in the cost-capacity equation that exactly<br />

relates the two pipe sizes.<br />

15.35 A 100,000 barrel per day (bpd) fractionation tower<br />

cost $1.2 million in 2001 when the Chemical <strong>Engineering</strong><br />

plant cost index value was 394.3. How<br />

much would a 450,000 bpd plant cost when the<br />

index value is 575.8, provided the exponent in the<br />

cost-capacity equation is 0.67?<br />

15.36 A mini wind tunnel for calibrating vane or hotwire<br />

anemometers cost $3750 in 2002 when the<br />

M&S equipment index value was 1104.2. If the<br />

index value is now 1620.6, estimate the cost of a<br />

tunnel twice as large. The cost-capacity equation<br />

exponent is 0.89.<br />

15.37 In 2008, a military engineer estimated the cost for<br />

a classified laser-guided device to be $376,900.<br />

The engineer used the M&S equipment index for<br />

the years 1998 and 2008 and the cost-capacity<br />

equation with an exponent value of 0.61. If the<br />

original equipment had only one-fourth the capacity<br />

of the new equipment, what was the cost of the<br />

original equipment in 1998?<br />

15.38 Instead of using a cost-capacity equation for relating<br />

project size and construction cost, the Department<br />

of Defense uses Size Adjustment Factors<br />

(SAFs) that are based on Size Relationship Ratios<br />

(SRRs). For example, a SRR of 2.00 has a SAF of<br />

0.942. Determine the required exponent value in<br />

the cost-capacity equation in order for C 2 to be<br />

0.942 C 1 when Q 2 Q 1 2.00.<br />

15.39 The equipment cost for removing arsenic from a<br />

well that delivers 800 gallons per minute (gpm) is<br />

$1.8 million. If the overall cost factor for this type<br />

of treatment system is 2.25, what is the total plant<br />

cost expected to be?<br />

15.40 A closed-loop filtration system for waterjet cutting<br />

industries eliminates the cost of makeup water<br />

treatments (water softeners, reverse osmosis, etc.)<br />

while maximizing orifice life and machine performance.<br />

If the equipment cost is $225,000 and the<br />

total plant cost is $1.32 million, what is the overall<br />

cost factor for the system?<br />

15.41 The equipment cost for a laboratory that plans to<br />

specialize in analyzing for endocrine disrupters,<br />

pharmaceuticals, and personal care products is<br />

$870,000. If the direct cost factor is 1.32 and the<br />

indirect cost factor is 0.45 (applies to equipment<br />

only), determine the expected cost of the laboratory.<br />

15.42 The equipment cost for a 10 gallon per minute<br />

farm-scale ethanol fuel production plant is<br />

$243,000. The direct cost factor for construction is<br />

1.28 and for installation is 0.23. The indirect cost<br />

factor for licenses, insurance, etc. is 0.84 (applied<br />

to total direct cost). Determine the estimated total<br />

plant cost.<br />

15.43 A chemical engineer at Western Refining has estimated<br />

that the total cost for a diesel fuel desulfurization<br />

system will be $2.3 million. If the direct<br />

cost factor is 1.35 and the indirect cost factor is<br />

0.41, what is the total equipment cost? Both factors<br />

apply to delivered-equipment cost.<br />

15.44 A mechanical engineer estimated that the equipment<br />

cost for a multitube cyclone system with a<br />

capacity of 60,000 cfm would be $400,000. If the<br />

direct cost factor is 3.1 and the indirect cost factor<br />

is 0.38, what is the estimated total plant cost? The<br />

indirect cost factor applies to the total direct cost.<br />

15.45 Nicole is an engineer on temporary assignment at<br />

a refinery operation in Seaside. She has reviewed a<br />

cost estimate for $430,000, which covers some<br />

new processing equipment for the ethylene line.<br />

The equipment itself is estimated at $250,000 with<br />

a construction cost factor of 0.30 and an installation<br />

cost factor of 0.30. No indirect cost factor is<br />

listed, but she knows from other sites that indirect<br />

cost is a sizable amount that increases the cost of<br />

the line’s equipment. ( a ) If the indirect cost factor<br />

should be 0.40, determine whether the current estimate<br />

includes a factor comparable to this value<br />

( b ) Determine the cost estimate if the 0.40 indirect<br />

cost factor is used.<br />

Indirect Cost Allocation<br />

15.46 The company you work for currently allocates insurance<br />

costs on the basis of cost per direct labor<br />

hour. This indirect cost component for the year is<br />

budgeted at $36,000. If the direct labor hours for


408 Chapter 15 Cost Estimation and Indirect Cost Allocation<br />

departments A, B, and C are expected to be 2000,<br />

8000, and 5000, respectively, this year, determine<br />

the allocation to each department.<br />

15.47 The director of public works needs to distribute the<br />

indirect cost allocation of $1.2 million to the three<br />

branches around the city. The recorded amounts<br />

for this year are as follows:<br />

Branch<br />

Miles Driven<br />

Records for This Year<br />

Direct Labor<br />

Hours<br />

North 275,000 38,000<br />

South 247,000 31,000<br />

Midtown 395,000 55,500<br />

The director plans to use the allocation and information<br />

from last year to determine the rates for<br />

this year. This information follows:<br />

Branch<br />

Miles<br />

Driven<br />

Direct<br />

Labor<br />

Hours<br />

Basis<br />

Indirect Cost<br />

Allocation<br />

Last Year, $<br />

North 350,000 40,000 Miles 300,000<br />

South 200,000 20,000 Labor 200,000<br />

Midtown 500,000 64,000 Labor 450,000<br />

(a) Determine the rates for this year for each<br />

branch.<br />

(b) Use the rate to distribute this year’s total indirect<br />

cost. What percentage of this year’s<br />

budget is now distributed?<br />

15.48 A company has a processing department with 10<br />

stations. Because of the nature and use of three of<br />

these stations, each is considered a separate cost<br />

center for indirect cost allocation. The remaining<br />

seven are grouped as one center, CC190. Machine<br />

operating hours are used as the allocation basis for<br />

all machines. A total of $250,000 is allocated to the<br />

department for next year. Use the data collected<br />

this year to determine the indirect cost rate for<br />

each center.<br />

Cost Center<br />

Indirect<br />

Cost Allocated, $<br />

Estimated<br />

Machine Hours<br />

CC100 25,000 800<br />

CC110 50,000 200<br />

CC120 75,000 1200<br />

CC190 100,000 1600<br />

15.49 All the indirect costs are allocated by accounting<br />

for a department. The manager has obtained records<br />

of allocation rates and actual charges for<br />

the prior 3 months and estimates for this month<br />

(May) and next month (see the table). The basis<br />

of allocation is not indicated, and the company<br />

accountant has no record of the basis used. However,<br />

the accountant advises the manager to not<br />

be concerned because the allocation rates have<br />

decreased each month.<br />

Indirect Cost, $<br />

Month Rate Allocated Charged<br />

February 1.40 2800 2600<br />

March 1.33 3400 3800<br />

April 1.37 3500 3500<br />

May 1.03 3600<br />

June 0.92 6000<br />

During the evaluation, the following additional<br />

information from departmental and accounting<br />

records is obtained.<br />

Direct Labor<br />

Month Hours Cost, $<br />

Material<br />

Cost, $<br />

Departmental<br />

Space, ft 2<br />

February 640 2560 5400 2000<br />

March 640 2560 4600 2000<br />

April 640 2560 5700 3500<br />

May 640 2720 6300 3500<br />

June 800 3320 6500 3500<br />

(a) With this information determine the allocation<br />

basis used each month.<br />

(b) Comment on the accountant’s statement<br />

about decreasing allocation rates.<br />

15.50 The mechanical components division manager<br />

asks you to recommend a make/buy decision on a<br />

major automotive subassembly that is currently<br />

purchased externally for a total of $3.9 million<br />

this year. This cost is expected to continue rising<br />

at a rate of $300,000 per year. Your manager asks<br />

that both direct and indirect costs be included<br />

when in-house manufacturing (make alternative)<br />

is evaluated. New equipment will cost $3 million<br />

and will have a salvage of $0.5 million and a life<br />

of 6 years. Estimates of materials, labor costs, and<br />

other direct costs are $1.5 million per year. Typical<br />

indirect rates, bases, and expected usage are<br />

shown below. Perform the AW evaluation at<br />

MARR 12% per year over a 6-year study period.<br />

Show both hand and spreadsheet solutions,<br />

as directed.<br />

Department Basis Rate<br />

Expected<br />

Usage<br />

X Direct labor cost $2.40 per $ $450,000<br />

Y Materials cost $0.50 per $ $850,000<br />

Z Number of<br />

inspections<br />

$20 per<br />

inspection<br />

4,500


Problems 409<br />

ABC Method<br />

15.51 The municipal water and desalinization utility in a<br />

California city currently allocates some costs for<br />

maintenance shop workers to pumping stations<br />

based on the number of pumps at each station. At<br />

the last director’s semiannual meeting, a suggestion<br />

was made to change the allocation basis to the<br />

number of trips that pump service personnel make<br />

to each station, because some stations have old<br />

pumps that require more maintenance. Information<br />

about the stations is below. The indirect cost<br />

budget is $20,000 per pump.<br />

( a) Allocate the budget to each station based on<br />

the number of service trips.<br />

( b ) Determine the old allocation on the basis of<br />

the number of pumps, and comment on any<br />

significant differences in the amounts allocated<br />

to the stations.<br />

Station ID No. of Pumps Service Trips per Year<br />

Sylvester 5 190<br />

Laurel 7 55<br />

7 th St 3 38<br />

Spicewood 4 104<br />

15.52 Factory Direct manufactures and sells manufactured<br />

homes. Traditionally, it has distributed indirect costs<br />

to its three construction plants based on materials<br />

cost. Each plant builds different models and floor<br />

plans. Advances in weight and shape of plastic and<br />

wood composite components have decreased cost<br />

and time to produce a unit. Because of these advances,<br />

the CFO plans to use build-time per unit as<br />

the new basis. However, he initially wants to determine<br />

what the allocation would have been this year<br />

had build-time been the basis prior to incorporation<br />

of the new materials. The data shown represents average<br />

costs and times. Use this data and the three<br />

bases indicated to determine the allocation rates and<br />

indirect cost distribution of $900,000 for this year.<br />

Plant Texas Oklahoma Kansas<br />

Direct material cost,<br />

$ per unit<br />

Previous build-time<br />

per unit, work-hours<br />

New build-time per<br />

unit, work-hours<br />

20,000 12,700 18,600<br />

400 415 580<br />

425 355 480<br />

Problems 15.53 through 15.55 use the following<br />

information.<br />

Jet Green Airways historically distributes the indirect<br />

costs of lost and damaged baggage to its three major hubs<br />

using a basis of annual number of flights in and out of<br />

each hub. Last year $667,500 was distributed as follows:<br />

Hub Airport Flights Rate, $ per Flight Allocation, $<br />

DFW 55,000 6 330,000<br />

YYZ 20,833 9 187,500<br />

MEX 15,000 10 150,000<br />

The airline’s baggage management director suggests that<br />

an allocation on the basis of baggage traffic, not flights,<br />

will better represent the distribution, primarily based on<br />

the fact that the high fees now charged to passengers to<br />

check luggage have significantly changed the number of<br />

bags handled at the major hubs. Total number of bags<br />

handled during the year are 2,490,000 at DFW, 1,582,400<br />

at YYZ, and 763,500 at MEX.<br />

15.53 What are the activity and the cost driver for the<br />

suggested baggage-traffic basis?<br />

15.54 Using the baggage-traffic basis, determine the allocation<br />

rate using last year’s total of $667,500,<br />

and distribute this amount to the hubs this year.<br />

15.55 What are the percentage changes in allocation at<br />

each hub using the two different bases?<br />

15.56 On-line Vacation distributes advertising costs to its<br />

four resort sites in the Caribbean on the basis of<br />

the size of the resort budget. For this year, in round<br />

numbers, the budgets and allocation of $1 million<br />

advertising indirect costs are as follows:<br />

Site<br />

A B C D<br />

Budget, $ 2 million 3 million 4 million 1 million<br />

Allocation, $ 200,000 300,000 400,000 100,000<br />

(a)<br />

(b)<br />

Determine the allocation if the ABC method is<br />

used with a new basis. Define the activity as the<br />

advertising department at each resort. The cost<br />

driver is the number of guests during the year.<br />

Site<br />

A B C D<br />

Guests 3500 4000 8000 1000<br />

Again use the ABC method, but now make the<br />

cost driver the total number of guest nights at<br />

each resort. The average number of lodging<br />

nights for guests at each site is as follows:<br />

Site<br />

A B C D<br />

Length of stay, nights 3.0 2.5 1.25 4.75<br />

(c) Comment on the distribution of advertising<br />

costs using the two methods. Identify any<br />

other cost drivers that might be considered<br />

for the ABC approach that may reflect a<br />

realis tic allocation of the costs.


410 Chapter 15 Cost Estimation and Indirect Cost Allocation<br />

ADDITIONAL PROBLEMS AND FE EXAM REVIEW QUESTIONS<br />

15.57 In the bottom-up approach to cost estimating:<br />

(a) Required price is an input variable.<br />

(b) Cost estimates are an output variable.<br />

(c) Required price is an output variable.<br />

(d) Both ( a ) and ( b ) are correct.<br />

15.58 A ratio of the cost of something today to its cost at<br />

some time in the past is called a:<br />

(a) Cost-capacity index<br />

(b) Cost index<br />

(c) Buyer’s guide<br />

(d) Bluebook index<br />

15.59 Index values could probably be obtained from all<br />

of the following places except:<br />

(a) Trade organizations<br />

(b) <strong>Engineering</strong> News Record magazine<br />

(c) Government organizations<br />

(d) Home Depot or Lowe’s home improvement<br />

stores<br />

15.60 A 50-hp turbine pump was purchased for $2100. If<br />

the exponent in the cost-capacity equation has a<br />

value of 0.76, a 200-hp turbine pump could be expected<br />

to cost about:<br />

( a ) $6020<br />

( b ) $5320<br />

( c ) $4890<br />

( d ) $4260<br />

100,000 units per day was $3 million, the value of<br />

the exponent in the cost-capacity equation is:<br />

( a ) 0.26<br />

( b ) 0.39<br />

( c ) 0.45<br />

( d ) 0.60<br />

15.64 The equipment for applying specialty coatings<br />

that provide a high angle of skid for the paperboard<br />

and corrugated box industries has a delivered<br />

cost of $390,000. If the overall cost factor<br />

for the complete system is 2.96, the total plant<br />

cost is approximately:<br />

( a ) $954,400<br />

( b ) $1,054,400<br />

( c ) $1,154,400<br />

( d ) $1,544,400<br />

15.65 The delivered-equipment cost for setting up a<br />

production and assembly line for high-sensitivity,<br />

gas-damped accelerometers is $650,000. If the direct<br />

cost and indirect cost factors are 1.82 and<br />

0.31, respectively, and both factors apply to<br />

delivered-equipment cost, the total plant cost estimate<br />

is approximately:<br />

( a ) $2,034,500<br />

( b ) $1,734,500<br />

( c ) $1,384,500<br />

( d ) $1,183,000<br />

15.61 The city built a recreation park in 1980 for<br />

$500,000. The ENR construction cost index had<br />

a value of 3378.17 at that time. If the city is<br />

planning to construct a similar recreation park<br />

when the index value is 5542.16, the estimated<br />

cost is closest to:<br />

( a ) $695,800<br />

( b ) $750,700<br />

( c ) $820,300<br />

( d ) $910,500<br />

15.62 A small company bought a 250-hp compressor in<br />

1998 for $3000 when the M&S equipment cost<br />

index had a value of 1061.9. If the exponent in the<br />

cost-capacity equation is 0.32 and the M&S index<br />

value was 1449.3 in 2008, the cost of a 500-hp<br />

compressor in 2008 was closest to:<br />

( a ) $3744<br />

( b ) $4094<br />

( c ) $4627<br />

( d ) $5110<br />

15.63 The cost for implementing a manufacturing process<br />

that has a capacity of 6000 units per day was<br />

$550,000. If the cost for a plant with a capacity of<br />

15.66 A police department wants to allocate the indirect<br />

cost of speed monitoring to the three toll roads<br />

around the city. An allocation basis that may not be<br />

reasonable is:<br />

(a) Miles of toll road monitored<br />

(b) Average number of cars patrolling per hour<br />

( c) Amount of car traffic per section of toll<br />

road<br />

(d) Cost to operate a patrol car<br />

15.67 The IT department allocates indirect costs to user<br />

departments on the basis of CPU time at the rate of<br />

$2000 per second. For the first quarter, the two<br />

heaviest-use departments logged 900 and 1300 seconds,<br />

respectively. If the IT indirect budget for the<br />

year is $8.0 million, the percentage of this year’s<br />

allocation consumed by these departments is<br />

closest to:<br />

( a ) 32%<br />

( b ) 22.5%<br />

( c ) 55%<br />

( d ) Not enough information to determine<br />

15.68 If the engineering department is the activity to<br />

receive indirect cost allocation for the year, cost


Case Study 411<br />

drivers for the ABC method that seem reasonable<br />

may be:<br />

1. Cost of engineering changes processed<br />

2. Size of the workforce<br />

3. Administrative cost to process a change<br />

order<br />

( a ) 1<br />

( b ) 2<br />

( c ) 3<br />

( d ) 1 and 3<br />

15.69 Advantages of the ABC method are:<br />

1. It is an excellent replacement for a traditional<br />

cost accounting system.<br />

2. It is always cheaper to operate than a traditional<br />

allocation system.<br />

3. It can help explain the economic impact of<br />

management decisions.<br />

( a ) 1<br />

( b ) 2<br />

( c ) 3<br />

( d ) 1 and 3<br />

CASE STUDY<br />

INDIRECT COST ANALYSIS OF MEDICAL EQUIPMENT MANUFACTURING COSTS<br />

Background<br />

all its other product lines. The same was applied when Quik-<br />

Sterz was priced. However, Arnie, the person who performed<br />

Three years ago Medical Dynamics, a medical equipment<br />

the indirect cost analysis and set the sales price, is no longer<br />

unit of Johnson and Sons, Inc., initiated the manufacture and<br />

at the company, and the detailed analysis is no longer available.<br />

Through e-mail and telephone conversations, Arnie said<br />

sales of a portable sterilization unit (Quik-Sterz) that can be<br />

placed in the hospital room of a patient. This unit sterilizes<br />

the current price was set at about 10% above the total manufacturing<br />

cost determined 2 years ago, and that some records<br />

and makes available at the bedside some of the reusable instruments<br />

that nurses and doctors usually obtain by walking<br />

were available in the design department files. A search of<br />

to or receiving delivery from a centralized area. This new unit<br />

these files revealed the manufacturing and cost information in<br />

makes the instruments available at the point and time of use<br />

Table 15–11. It is clear from these and other records that<br />

for burn and severe wound patients who are in a regular patient<br />

room.<br />

Arnie used traditional indirect cost analysis based on direct<br />

labor hours to estimate the total manufacturing costs of<br />

There are two models of Quik-Sterz sold. The standard<br />

$9.73 per unit for the standard model and $27.07 per unit for<br />

version sells for $10.75, and a premium version with customized<br />

trays and a battery backup system sells for $29.75. The<br />

the premium model.<br />

Last year management decided to place the entire plant on<br />

product has sold well to hospitals, convalescent units, and<br />

the ABC system of indirect cost allocation. The costs and<br />

nursing homes at the level of about 1 million units per year.<br />

sales figures collected for Quik-Sterz the year before were<br />

still accurate. Five activities and their cost drivers were identified<br />

for the Medical Dynamics manufacturing operations<br />

Information<br />

Medical Dynamics has historically used an indirect cost allocation<br />

system based upon direct hours to manufacture for marized in this<br />

(Table 15–12). Also, the volumes for each model are sum-<br />

table.<br />

TABLE 15–11<br />

Historical Records of Direct and Indirect Cost Analyses for Quik-Sterz<br />

Quik-Sterz Direct Cost (DC) Evaluation<br />

Direct Material,<br />

$Unit<br />

Model<br />

Direct Labor,<br />

$Unit 1<br />

Direct Labor,<br />

HoursUnit<br />

Total Direct<br />

Labor Hours<br />

Standard 5.00 2.50 0.25 187,500<br />

Premium 10.00 3.75 0.50 125,000<br />

Quik-Sterz Indirect Cost (IDC) Evaluation<br />

Fraction IDC<br />

Allocated<br />

Model<br />

Direct Labor,<br />

HoursUnit<br />

Allocated<br />

IDC, $<br />

Sales,<br />

Units/Year<br />

Standard 0.25 —<br />

3<br />

1<br />

Premium 0.50 — 2 3<br />

1.67 million 750,000<br />

3.33 million 250,000<br />

1 Average direct labor rate is $20 per hour.


412 Chapter 15 Cost Estimation and Indirect Cost Allocation<br />

TABLE 15–12<br />

Quik-Sterz Activities, Cost Drivers, and Volume Levels for ABC-Based<br />

Indirect Cost Allocation<br />

Activity Cost Driver VolumeYear<br />

Actual<br />

Cost, $/Year<br />

Quality Inspections 20,000 inspections 800,000<br />

Purchasing Purchase orders 40,000 orders 1,200,000<br />

Scheduling Change orders 1,000 orders 800,000<br />

Production setup Setups 5,000 setups 1,000,000<br />

Machine operations Machine hours 10,000 hours 1,200,000<br />

Volume Level for the Year<br />

Cost Driver Standard Premium<br />

Quality inspections 8,000 12,000<br />

Purchase orders 30,000 10,000<br />

Number of change orders 400 600<br />

Production setup 1,500 3,500<br />

Machine hours 7,000 3,000<br />

The ABC method will be used henceforth, with the intention<br />

of determining the total cost and price based on its results. The<br />

first impression of the production manager is that the new system<br />

will show that indirect costs for Quik-Sterz are about the<br />

same as they have been for other products over the last several<br />

years when a standard model and an upgrade (premium) model<br />

were sold. Predictably, they state, the standard model will receive<br />

about 13 of the indirect cost, and the premium will receive<br />

the remaining 23. Fundamentally, there are two reasons<br />

why the production manager does not like to produce premium<br />

versions: They are less profitable for the company, and they require<br />

significantly more time and operations to manufacture.<br />

Case Study Exercises<br />

1. Use traditional indirect cost allocation to verify Arnie’s<br />

cost and price estimates.<br />

2. Use the ABC method to estimate the indirect cost allocation<br />

and total cost for each model.<br />

3. If the prices and number of units sold are the same<br />

next year (750,000 standard and 250,000 premium),<br />

and all other costs remain constant, compare the<br />

profit from Quik-Sterz under the ABC method with<br />

the profit using the traditional indirect cost allocation<br />

method.<br />

4. What prices should Medical Dynamics charge next year<br />

based on the ABC method and a 10% markup over cost?<br />

What is the total profit from Quik-Sterz predicted to be<br />

if sales hold steady?<br />

5. Using the results above, comment on the production<br />

manager’s prediction of indirect costs using ABC (13<br />

standard; 23 premium) and the two reasons given to<br />

not produce the premium version of Quik-Sterz.<br />

CASE STUDY<br />

DECEPTIVE ACTS CAN GET YOU IN TROUBLE<br />

Contributed by Dr. Paul Askenasy, Agronomist, Texas Commission<br />

on Environmental Quality<br />

Background<br />

Surface mining of coal is the removal of soil and sediments<br />

from underlying strata that lie above the material to be mined.<br />

The law requires that land disturbed by these types of mining<br />

activities be returned to a productive capacity that is as good<br />

as or better than its productive capacity before mining.<br />

The productive capacity of soils is directly correlated to<br />

the textural (sand, silt, and clay content) and chemical<br />

characteristics of the soil (for example, pH). To this end,<br />

mining companies must sample the different soils found in<br />

the areas to be disturbed by mining activities. The purpose<br />

of the sampling is to establish a baseline characterizing the<br />

textural and chemical makeup of the soils prior to mining.<br />

Soils in a low pH range (pH values 5) are indicative of<br />

low fertility. Once the natural resource, such as coal, is removed,<br />

the pit is backfilled with sediments and the terrain<br />

surface is contoured to reestablish the premine drainages.<br />

To meet the baseline for pH, the acreage of the mine soils<br />

with low pH should not exceed the acreage of the unmined<br />

soils with low pH.


Case Study 413<br />

Information<br />

Yucatan Mining Company (not the actual name) planned to<br />

disturb 600 acres due to mining activities. The different soils<br />

within the 600 acres were depicted in the County Soil Survey<br />

where the mining activities were to take place. Prior to mining,<br />

the company obtained soil samples from 10 different locations<br />

within each soil type and had them analyzed for a<br />

number of parameters including pH. Assessment of the data<br />

indicated that 30% of the area (180 acres) occupied by the<br />

soils in the area to be disturbed had pH values between 4.0<br />

and 4.9. The application for mining was approved by the<br />

State Department of Mining and Reclamation.<br />

Six years later, 450 acres had been mined and the terrain<br />

surface had been leveled to reestablish premine slopes. Of the<br />

450 acres leveled, 175 acres had pH values between 4.0 and<br />

4.9. The president of Yucatan indicated that the company<br />

would submit a revised soil baseline based on new sampling<br />

in the remaining 150 acres of unmined soils because, in his<br />

opinion, the first soil baseline was biased.<br />

The request to do more soil analyses to augment their existing<br />

soil baseline was approved. The company quickly hired<br />

a consultant to develop the new baseline, and subsequently<br />

Yucatan submitted the final report from its consultant to the<br />

State Department of Mining and Reclamation. This revised<br />

premine soil baseline indicated that 45% of the premine soils<br />

had pH values between 4.0 and 4.9. Comparative results between<br />

the old and new samples can be expressed as follows:<br />

Percent and Acreage of Area<br />

Soil Baselines Old Soil Baseline Revised Soil Baseline<br />

pH: 4.0 4.9 30% 45%<br />

180 acres 270 acres<br />

A rough statistical check between the old and revised soil<br />

baselines indicated that the results were mixed. Based on this<br />

preliminary result and the fact that there was a significant increase<br />

in the percent of area with low-productivity soil, an<br />

in-depth analysis of the revised baseline sample study was<br />

performed. Contained in the submitted new-sample package<br />

was a letter from the Yucatan consultant. It indicated to<br />

Yucatan’s management that 100 separate soil samples had<br />

been obtained and analyzed and that the revised premine soil<br />

baseline had been developed using the data from the 30 samples<br />

with the lowest pH values .<br />

The State Department of Mining and Reclamation staff<br />

concluded that the revised soil pH sample data had been carefully<br />

“screened” to reduce the amount of remediation work<br />

that Yucatan Mining would have to complete . Within a week,<br />

Yucatan was notified that further review of the revised soil<br />

baseline could not be pursued, because it appeared the revised<br />

soil baseline was developed using a technique that<br />

skewed results in favor of lower pH values. It was also noted<br />

that should Yucatan Mining disagree with this response, the<br />

case would be filed with the legal staff as a contested case.<br />

Within several days, the Yucatan president responded indicating<br />

that the company was withdrawing the new application<br />

from consideration by the department.<br />

Case Study Questions<br />

1. Assume you are the director of the State Department of<br />

Mining and Reclamation and were informed of the findings<br />

on the new samples versus the old samples. What<br />

actions would you direct your staff to take concerning<br />

this situation?<br />

2. Suppose there had been several cases of deceptive acts<br />

similar to this one over the last few years. What type of<br />

“audit” procedures might you want implemented to<br />

identify these possibly unethical activities?<br />

3. Yucatan clearly would state that both the old and new<br />

samples were randomly located about the entire mining<br />

area. When the 30 lowest pH samples were used to establish<br />

the new baseline, were the samples still random,<br />

according to experimental design standards? If so, why?<br />

If not, why not?<br />

4. You and the president of Yucatan Mining have been acquaintances<br />

for some years. You have golfed together<br />

several times, your and his children are on the same soccer<br />

team at school, and your families are members of<br />

the same community swimming pool club. What effect<br />

would this event have upon your and your family’s relationships<br />

with the family of the Yucatan president? How<br />

would you handle this situation?<br />

5. As a matter of principle and practice, do you believe<br />

there is some amount of data-altering or bias-making<br />

that is allowed before an application (such as the one<br />

described here) should be considered the result of professionally<br />

unethical acts? How would you define such<br />

a threshold limit?


CHAPTER 16<br />

Depreciation<br />

Methods<br />

LEARNING OUTCOMES<br />

Purpose: Use depreciation or depletion methods to reduce the book value of a capital investment in an asset and natural<br />

resource.<br />

SECTION TOPIC LEARNING OUTCOME<br />

16.1 Terminology • Define and use the basic terms of asset<br />

depreciation.<br />

16.2 Straight line • Apply the straight line (SL) method of<br />

depreciation.<br />

16.3 Declining balance • Apply the declining balance (DB) and double<br />

declining balance (DDB) methods of<br />

depreciation.<br />

16.4 MACRS • Apply the modified accelerated cost recovery<br />

system for tax depreciation purposes for U.S.-<br />

based corporations.<br />

16.5 Recovery period • Select the asset recovery period for MACRS<br />

depreciation.<br />

16.6 Depletion • Explain depletion; apply cost depletion and<br />

percentage depletion methods.<br />

Chapter 16 Appendix<br />

16A.1 Historical methods • Apply the sum-of-years-digits (SYD) and unitof-production<br />

(UOP) methods of depreciation.<br />

16A.2 Switching • Switch between classical depreciation methods;<br />

explain how MACRS provides for switching.<br />

16A.3 MACRS and switching • Calculate MACRS rates using switching between<br />

classical methods and MACRS rules.


T<br />

he capital investments of a corporation in tangible assets—equipment, computers,<br />

vehicles, buildings, and machinery—are commonly recovered on the books<br />

of the corporation through depreciation . Although the depreciation amount is<br />

not an actual cash flow, the process of depreciating an asset on the books of the corporation<br />

accounts for the decrease in an asset’s value because of age, wear, and obsolescence.<br />

Even though an asset may be in excellent working condition, the fact that it is worth less<br />

through time is taken into account in after-tax economic evaluation studies. An introduction<br />

to depreciation types, terminology, and classical methods is followed by a discussion<br />

of the Modified Accelerated Cost Recovery System (MACRS) , which is the standard in the<br />

United States for tax purposes. Other countries commonly use the classical methods for tax<br />

computations.<br />

Why is depreciation important to engineering economy? Depreciation is a tax-allowed<br />

deduction included in tax calculations in virtually all industrialized countries. Depreciation<br />

lowers income taxes via the general relation<br />

Taxes (income deductions)(tax rate)<br />

Income taxes are discussed further in Chapter 17.<br />

This chapter concludes with an introduction to two methods of depletion, which are used<br />

to recover capital investments in deposits of natural resources such as oil, gas, minerals, ores,<br />

and timber.<br />

The chapter appendix describes two historically useful methods of depreciation—<br />

sum-of-years-digits and unit-of-production. Additionally, the appendix includes an in-depth<br />

derivation of the MACRS depreciation rates from the straight line and declining balance<br />

rates. This is accomplished using a procedure called switching between classical depreciation<br />

methods.<br />

16.1 Depreciation Terminology<br />

The concept and types of depreciation are defined here. Most descriptions are applicable to corporations<br />

as well as individuals who own depreciable assets.<br />

Depreciation is a book method (noncash) to represent the reduction in value of a tangible asset.<br />

The method used to depreciate an asset is a way to account for the decreasing value of the asset<br />

to the owner and to represent the diminishing value (amount) of the capital funds invested in it.<br />

The annual depreciation amount is not an actual cash flow , nor does it necessarily reflect the<br />

actual usage pattern of the asset during ownership.<br />

Though the term amortization is sometimes used interchangeably with the term depreciation,<br />

they are different. Depreciation is applied to tangible assets, while amortization is used to reflect<br />

the decreasing value of intangibles, such as loans, mortgages, patents, trademarks, and goodwill.<br />

In addition, the term capital recovery is sometimes used to identify depreciation. This is clearly<br />

a different use of the term than what we learned in Chapter 5. The term depreciation is used<br />

throughout this book.<br />

There are two different purposes for using the depreciation methods we will cover in this<br />

chapter:<br />

Book depreciation Used by a corporation or business for internal financial accounting to<br />

track the value of an asset or property over its life.<br />

Tax depreciation Used by a corporation or business to determine taxes due based on current<br />

tax laws of the government entity (country, state, province, etc.). Even though depreciation<br />

itself is not a cash flow, it can result in actual cash flow changes because the amount of<br />

tax depreciation is a deductible item when calculating annual income taxes for the corporation<br />

or business.<br />

The methods applied for these two purposes may or may not utilize the same formulas. Book<br />

depreciation indicates the reduced investment in an asset based upon the usage pattern and<br />

expected useful life of the asset. There are classical, internationally accepted depreciation methods<br />

used to determine book depreciation: straight line, declining balance, and the historical


416 Chapter 16 Depreciation Methods<br />

sum-of-years-digits method. The amount of tax depreciation is important in an after-tax engineering<br />

economy study and will vary among nations.<br />

In most industrialized countries, the annual tax depreciation is tax-deductible ; that is, it is subtracted<br />

from income when calculating the amount of taxes due each year. However, the tax<br />

depreciation amount must be calculated using a government-approved method.<br />

Tax depreciation may be calculated and referred to differently in countries outside the United<br />

States. For example, in Canada the equivalent is CCA (capital cost allowance), which is calculated<br />

based on the undepreciated value of all corporate properties that form a particular<br />

class of assets, whereas in the United States depreciation may be determined for each asset<br />

separately.<br />

Where allowed, tax depreciation is usually based on an accelerated method , whereby the<br />

depreciation for the first years of use is larger than that for later years. In the United States this<br />

method is called MACRS, as covered in later sections. In effect, accelerated methods defer<br />

some of the income tax burden to later in the asset’s life; they do not reduce the total tax<br />

burden.<br />

Common terms used in depreciation are explained here.<br />

First cost P or unadjusted basis B is the delivered and installed cost of the asset including<br />

purchase price, delivery and installation fees, and other depreciable direct costs incurred to<br />

prepare the asset for use. The term unadjusted basis , or simply basis , is used when the asset is<br />

new, with the term adjusted basis used after some depreciation has been charged. When the<br />

first cost has no added, depreciable costs, the basis is the first cost, that is, P B .<br />

Book value BV t represents the remaining, undepreciated capital investment on the books after<br />

the total amount of depreciation charges to date has been subtracted from the basis. The book<br />

value is determined at the end of each year t ( t 1, 2, . . . , n ), which is consistent with the<br />

end-of-year convention.<br />

Recovery period n is the depreciable life of the asset in years. Often there are different<br />

n values for book and tax depreciation. Both of these values may be different from the asset’s<br />

estimated productive life.<br />

Market value MV , a term also used in replacement analysis, is the estimated amount realizable<br />

if the asset were sold on the open market. Because of the structure of depreciation laws,<br />

the book value and market value may be substantially different. For example, a commercial<br />

building tends to increase in market value, but the book value will decrease as depreciation<br />

charges are taken. However, a computer workstation may have a market value much lower<br />

than its book value due to rapidly changing technology.<br />

Salvage value S is the estimated trade-in or market value at the end of the asset’s useful life.<br />

The salvage value, expressed as an estimated dollar amount or as a percentage of the first cost,<br />

may be positive, zero, or negative due to dismantling and carry-away costs.<br />

Depreciation rate or recovery rate d t is the fraction of the first cost removed by depreciation<br />

each year t. This rate may be the same each year, which is called the straight line rate d , or<br />

different for each year of the recovery period.<br />

Personal property, one of the two types of property for which depreciation is allowed is<br />

the income-producing, tangible possessions of a corporation used to conduct business.<br />

Included is most manufacturing and service industry property—vehicles, manufacturing<br />

equipment, materials handling devices, computers and networking equipment, communications<br />

equipment, office furniture, refining process equipment, construction assets, and<br />

much more.<br />

Real property includes real estate and all improvements—office buildings, manufacturing<br />

structures, test facilities, warehouses, apartments, and other structures. Land itself is considered<br />

real property, but it is not depreciable.<br />

Half-year convention assumes that assets are placed in service or disposed of in midyear,<br />

regardless of when these events actually occur during the year. This convention is utilized in<br />

this text and in most U.S.-approved tax depreciation methods. There are also midquarter and<br />

midmonth conventions.


16.1 Depreciation Terminology 417<br />

Figure 16–1<br />

General shape of book<br />

value curves for different<br />

depreciation methods.<br />

SL<br />

Book value, $<br />

MACRS<br />

DB<br />

Estimated<br />

salvage<br />

value<br />

Time, years<br />

Recovery period<br />

As mentioned before, there are several models for depreciating assets. The straight line (SL)<br />

method is used historically and internationally. Accelerated models, such as the declining bal -<br />

ance (DB) method, decrease the book value to zero (or to the salvage value) more rapidly than<br />

the straight line method, as shown by the general book value curves in Figure 16–1.<br />

For each of the methods—straight line, declining balance, MACRS, and sum-of-yearsdigits—there<br />

are spreadsheet functions available to determine annual depreciation. Each function<br />

is introduced and illustrated as the method is explained.<br />

As expected, there are many rules and exceptions to the depreciation laws of a country. One<br />

that may be of interest to a U.S.-based small or medium-sized business performing an economic<br />

analysis is the Section 179 Deduction. This is an economic incentive that changes over the years<br />

and encourages businesses to invest capital in equipment directly used in the company. Up to a<br />

specified amount, the entire basis of an asset is treated as a business expense in the year of purchase.<br />

This tax treatment reduces federal income taxes, just as depreciation does, but it is allowed<br />

in lieu of depreciating the first cost over several years. The limit changes with time; it was<br />

$24,000 in 2002; $102,000 in 2004; $125,000 in 2007; and $250,000 in 2008–2010. The economic<br />

stimulus efforts in the United States and around the world during the latter part of the decade<br />

made many attempts to put investment capital to work in small and medium-sized businesses.<br />

Investments above these limits must be depreciated using MACRS.<br />

In the 1980s the U.S. government standardized accelerated methods for federal tax depreciation<br />

purposes. In 1981, all classical methods, including straight line, declining balance, and sum-of-yearsdigits<br />

depreciation, were disallowed as tax deductible and replaced by the Accelerated Cost Recovery<br />

System (ACRS). In a second round of standardization, MACRS (Modified ACRS) was made the<br />

required tax depreciation method in 1986. To this date, the following is the law in the United States.<br />

Tax depreciation must be calculated using MACRS; book depreciation may be calculated<br />

using any classical method or MACRS.<br />

MACRS has the DB and SL methods, in slightly different forms, embedded in it, but these two<br />

methods cannot be used directly if the annual depreciation is to be tax deductible. Many U.S.<br />

companies still apply the classical methods for keeping their own books, because these methods<br />

are more representative of how the usage patterns of the asset reflect the remaining capital invested<br />

in it. Most other countries still recognize the classical methods of straight line and declining<br />

balance for tax or book purposes. Because of the continuing importance of the SL and DB<br />

methods, they are explained in the next two sections prior to MACRS. Appendix Section 16A.1<br />

discusses two historical methods of depreciation.


418 Chapter 16 Depreciation Methods<br />

Tax law revisions occur often, and depreciation rules are changed from time to time in the<br />

United States and other countries. For more depreciation and tax law information, consult the<br />

U.S. Department of the Treasury, Internal Revenue Service (IRS), website at www.irs.gov. Pertinent<br />

publications can be downloaded. Publication 946, How to Depreciate Property, is especially<br />

applicable to this chapter. MACRS and most corporate tax depreciation laws are discussed in it.<br />

16.2 Straight Line (SL) Depreciation<br />

Straight line depreciation derives its name from the fact that the book value decreases linearly<br />

with time. The depreciation rate is the same (1 n ) each year of the recovery period n .<br />

Straight line depreciation is considered the standard against which any depreciation model is<br />

compared. For book depreciation purposes, it offers an excellent representation of book value for<br />

any asset that is used regularly over an estimated number of years. For tax depreciation, as mentioned<br />

earlier, it is not used directly in the United States, but it is commonly used in most countries<br />

for tax purposes. However, the U.S. MACRS method includes a version of SL depreciation<br />

with a larger n value than that prescribed by regular MACRS (see Section 16.5).<br />

The annual SL depreciation is determined by multiplying the first cost minus the salvage value<br />

by d t . In equation form,<br />

D t ( B S ) d t<br />

B S ———<br />

n [16.1]<br />

Book Value<br />

B<br />

S<br />

Time<br />

1<br />

where t year ( t 1, 2, . . . , n )<br />

D t annual depreciation charge<br />

B first cost or unadjusted basis<br />

S estimated salvage value<br />

n recovery period<br />

d t depreciation rate 1 n<br />

Since the asset is depreciated by the same amount each year, the book value after t years of service,<br />

denoted by BV t , will be equal to the first cost B minus the annual depreciation times t .<br />

BV t B tD t [16.2]<br />

Earlier we defined d t as a depreciation rate for a specific year t . However, the SL model has the<br />

same rate for all years, that is,<br />

d d t — 1 n [16.3]<br />

The format for the spreadsheet function to display the annual depreciation D t in a single-cell<br />

operation is<br />

SLN( B , S , n) [16.4]<br />

EXAMPLE 16.1<br />

If an asset has a first cost of $50,000 with a $10,000 estimated salvage value after 5 years,<br />

( a ) calculate the annual depreciation and ( b ) calculate and plot the book value of the asset after<br />

each year, using straight line depreciation.<br />

Solution<br />

(a) The depreciation each year for 5 years can be found by Equation [16.1].<br />

D t B S ———<br />

n<br />

50,000 10,000<br />

——————— $8000<br />

5<br />

Enter the function SLN(50000,10000,5) in any cell to display the D t of $8000.


16.3 Declining Balance (DB) and Double Declining Balance (DDB) Depreciation 419<br />

Book value BV t $1000<br />

50<br />

40<br />

30<br />

20<br />

10<br />

D t = $8000<br />

Figure 16–2<br />

Book value of an asset<br />

using straight line depreciation,<br />

Example 16.1.<br />

S = $10,000<br />

0 1 2 3<br />

Year t<br />

4 5<br />

t<br />

(b) The book value after each year t is computed using Equation [16.2]. The BV t values are<br />

plotted in Figure 16–2. For years 1 and 5, for example,<br />

BV 1 50,000 1(8000) $42,000<br />

BV 5 50,000 5(8000) $10,000 S<br />

16.3 Declining Balance (DB) and Double Declining<br />

Balance (DDB) Depreciation<br />

The declining balance method is commonly applied as the book depreciation method. Like the<br />

SL method, DB is embedded in the MACRS method, but the DB method itself cannot be used to<br />

determine the annual tax-deductible depreciation in the United States. This method is used routinely<br />

in most other countries for tax and book depreciation purposes.<br />

Declining balance is also known as the fixed percentage or uniform percentage method.<br />

DB depreciation accelerates the write-off of asset value because the annual depreciation is determined<br />

by multiplying the book value at the beginning of a year by a fixed (uniform) percentage d ,<br />

expressed in decimal form. If d 0.1, then 10% of the book value is removed each year. Therefore,<br />

the depreciation amount decreases each year.<br />

The maximum annual depreciation rate for the DB method is twice the straight line rate, that is,<br />

d max 2 n [16.5]<br />

Book Value<br />

B<br />

Time<br />

In this case the method is called double declining balance (DDB) . If n 10 years, the DDB rate<br />

is 210 0.2; so 20% of the book value is removed annually. Another commonly used percentage<br />

for the DB method is 150% of the SL rate, where d 1.5 n .<br />

The depreciation for year t is the fixed rate d times the book value at the end of the previous year.<br />

D t ( d )BV t 1 [16.6]<br />

The actual depreciation rate for each year t , relative to the basis B , is<br />

d t d (1 d ) t 1 [16.7]<br />

If BV t 1 is not known, the depreciation in year t can be calculated using B and d .<br />

D t dB (1 d ) t1 [16.8]<br />

Book value in year t is determined in one of two ways: by using the rate d and basis B or by<br />

subtracting the current depreciation charge from the previous book value. The equations are<br />

BV t B (1 d ) t [16.9]<br />

BV t BV t 1 D t [16.10]


420 Chapter 16 Depreciation Methods<br />

It is important to understand that the book value for the DB method never goes to zero, because<br />

the book value is always decreased by a fixed percentage. The implied salvage value after n<br />

years is the BV n amount, that is,<br />

Implied S BV n B (1 d ) n [16.11]<br />

If a salvage value is estimated for the asset, this estimated S value is not used in the DB or<br />

DDB method to calculate annual depreciation. However, if the implied S estimated S , it is<br />

necessary to stop charging further depreciation when the book value is at or below the estimated<br />

salvage value. In most cases, the estimated S is in the range of zero to the implied S value. (This<br />

guideline is important when the DB method can be used directly for tax depreciation purposes.)<br />

If the fixed percentage d is not stated, it is possible to determine an implied fixed rate using the<br />

estimated S value, if S 0. The range for d is 0 d 2 n .<br />

Implied d 1 ( S —<br />

B ) 1n [16.12]<br />

The spreadsheet functions DDB and DB are used to display depreciation amounts for specific<br />

years. The function is repeated in consecutive spreadsheet cells because the depreciation amount<br />

D t changes with t . For the double declining balance method, the format is<br />

DDB( B , S , n , t , d ) [16.13]<br />

The entry d is the fixed rate expressed as a number between 1 and 2. If omitted, this optional<br />

entry is assumed to be 2 for DDB. An entry of d 1.5 makes the DDB function display 150%<br />

declining balance method amounts. The DDB function automatically checks to determine when<br />

the book value equals the estimated S value. No further depreciation is charged when this<br />

occurs. (To allow full depreciation charges to be made, ensure that the S entered is between zero<br />

and the implied S from Equation [16.11].) Note that d 1 is the same as the straight line rate<br />

1 n , but D t will not be the SL amount because declining balance depreciation is determined as a<br />

fixed percentage of the previous year’s book value, which is completely different from the SL<br />

calculation in Equation [16.1].<br />

The format for the DB function is DB( B , S , n , t ). Caution is needed when using this function.<br />

The fixed rate d is not entered in the DB function; d is an embedded calculation using<br />

a spreadsheet equivalent of Equation [16.12]. Also, only three significant digits are maintained<br />

for d , so the book value may go below the estimated salvage value due to round-off<br />

errors. Therefore, if the depreciation rate is known, always use the DDB function to ensure<br />

correct results. Examples 16.2 and 16.3 illustrate DB and DDB depreciation and their spreadsheet<br />

functions.<br />

EXAMPLE 16.2<br />

Underwater electroacoustic transducers were purchased for use in SONAR applications. The<br />

equipment will be DDB depreciated over an expected life of 12 years. There is a first cost of<br />

$25,000 and an estimated salvage of $2500. ( a ) Calculate the depreciation and book value for<br />

years 1 and 4. Write the spreadsheet functions to display depreciation for years 1 and 4.<br />

( b ) Calculate the implied salvage value after 12 years.<br />

Solution<br />

(a) The DDB fixed depreciation rate is d 2 n 212 0.1667 per year. Use Equations [16.8]<br />

and [16.9].<br />

Year 1: D 1 (0.1667)(25,000)(1 0.1667) 11 $4167<br />

BV 1 25,000(1 0.1667) 1 $20,833<br />

Year 4: D 4 (0.1667)(25,000)(1 0.1667) 41 $2411<br />

BV 4 25,000(1 0.1667) 4 $12,054<br />

The DDB functions for D 1 and D 4 are, respectively, DDB(25000,2500,12,1) and<br />

DDB(25000,2500,12,4).


16.3 Declining Balance (DB) and Double Declining Balance (DDB) Depreciation 421<br />

(b) From Equation [16.11], the implied salvage value after 12 years is<br />

Implied S 25,000(1 − 0.1667) 12 $2803<br />

Since the estimated S $2500 is less than $2803, the asset is not fully depreciated when<br />

its 12-year expected life is reached.<br />

EXAMPLE 16.3<br />

Freeport-McMoRan Copper and Gold has purchased a new ore grading unit for $80,000. The<br />

unit has an anticipated life of 10 years and a salvage value of $10,000. Use the DB and DDB<br />

methods to compare the schedule of depreciation and book values for each year. Solve by hand<br />

and by spreadsheet.<br />

Solution by Hand<br />

An implied DB depreciation rate is determined by Equation [16.12].<br />

d 1 ( 10,000 ———<br />

80,000 ) 110 0.1877<br />

Note that 0.1877 2 n 0.2, so this DB model does not exceed twice the straight line rate.<br />

Table 16–1 presents the D t values using Equation [16.6] and the BV t values from Equation<br />

[16.10] rounded to the nearest dollar. For example, in year t 2, the DB results are<br />

D 2 d (BV 1 ) 0.1877(64,984) $12,197<br />

BV 2 64,984 12,197 $52,787<br />

Because we round off to even dollars, $2312 is calculated for depreciation in year 10, but<br />

$2318 is deducted to make BV 10 S $10,000 exactly. Similar calculations for DDB with<br />

d 0.2 result in the depreciation and book value series in Table 16–1.<br />

TABLE 16–1 D t and BV t Values for DB and DDB Depreciation, Example 16.3<br />

Declining Balance, $ Double Declining Balance, $<br />

Year t D t BV t D t BV t<br />

0 — 80,000 — 80,000<br />

1 15,016 64,984 16,000 64,000<br />

2 12,197 52,787 12,800 51,200<br />

3 9,908 42,879 10,240 40,960<br />

4 8,048 34,831 8,192 32,768<br />

5 6,538 28,293 6,554 26,214<br />

6 5,311 22,982 5,243 20,972<br />

7 4,314 18,668 4,194 16,777<br />

8 3,504 15,164 3,355 13,422<br />

9 2,846 12,318 2,684 10,737<br />

10 2,318 10,000 737 10,000<br />

Solution by Spreadsheet<br />

The spreadsheet in Figure 16–3 displays the results for the DB and DDB methods. The chart<br />

plots book values for each year. Since the fixed rates are close—0.1877 for DB and 0.2 for<br />

DDB—the annual depreciation and book value series are approximately the same for the two<br />

methods.<br />

The depreciation rate (cell B5) is calculated by Equation [16.12], but note in the cell tags<br />

that the DDB function is used in both columns B and D to determine annual depreciation.<br />

As mentioned earlier, the DB function automatically calculates the implied rate by Equation<br />

[16.12] and maintains it to only three significant digits. Therefore, if the DB function


422 Chapter 16 Depreciation Methods<br />

1(B3/B2)^0.1<br />

DDB(B$2,B$3,B$4,$A18,10*$B$5)<br />

DDB(B$2,B$3,B$4,$A18)<br />

Figure 16–3<br />

Annual depreciation and book value using DB and DDB methods, Example 16.3.<br />

were used in column B (Figure 16–3), the fixed rate applied would be 0.188. The resulting<br />

D t and BV t values for years 8, 9, and 10 would be as follows:<br />

t D t , $ BV t , $<br />

8 3,501 15,120<br />

9 2,842 12,277<br />

10 2,308 9,969<br />

Also noteworthy is the fact that the DB function uses the implied rate without a check to halt<br />

the book value at the estimated salvage value. Thus, BV 10 will go slightly below S $10,000,<br />

as shown above. However, the DDB function uses a relation different from that of the DB function<br />

to determine annual depreciation—one that correctly stops depreciating at the estimated<br />

salvage value, as shown in Figure 16–3, cells E17–E18.<br />

16.4 Modified Accelerated Cost Recovery<br />

System (MACRS)<br />

In the 1980s, the United States introduced MACRS as the required tax depreciation method for<br />

all depreciable assets. Through MACRS, the 1986 Tax Reform Act defined statutory depreciation<br />

rates that take advantage of the accelerated DB and DDB methods. Corporations are free to apply<br />

any of the classical methods for book depreciation. When developed, MACRS and its predecessor<br />

ACRS were intended to create economic growth through the investment of new capital and<br />

the tax advantages that accelerated depreciation methods offer corporations and businesses. 1<br />

Many aspects of MACRS deal with the specific depreciation accounting aspects of tax law.<br />

This section covers only the elements that materially affect after-tax economic analysis. Additional<br />

information on how the DDB, DB, and SL methods are embedded into MACRS and how<br />

to derive the MACRS depreciation rates is presented and illustrated in the chapter appendix,<br />

Sections 16A.2 and 16A.3.<br />

MACRS determines annual depreciation amounts using the relation<br />

D t d t B [16.14]<br />

1 R. Lundquist, “The Pedagogy of Taxes and Tax Purpose Depreciation,” Proceedings, ASEE Annual<br />

Conference, Austin, TX, June 2009.


16.4 Modified Accelerated Cost Recovery System (MACRS) 423<br />

where the depreciation rate d t is provided in tabulated form. As for other methods, the book value<br />

in year t is determined by subtracting the depreciation amount from the previous year’s book<br />

value<br />

BV t BV t 1 D t [16.15]<br />

or by subtracting the total depreciation from the first cost.<br />

BV t first cost sum of accumulated depreciation<br />

jt<br />

B D j [16.16]<br />

j1<br />

MACRS has standardized and simplified many of the decisions and calculations of depreciation.<br />

The basis B (or first cost P ) is completely depreciated; salvage is always assumed to be zero,<br />

or S $0.<br />

Recovery periods are standardized to specific values:<br />

n 3, 5, 7, 10, 15, or 20 years<br />

n 27.5 or 39 years<br />

for personal property (e.g., equipment or vehicles)<br />

for real property (e.g., rental property or structures)<br />

Depreciation rates provide accelerated write-off by incorporating switching between classical<br />

methods.<br />

Section 16.5 explains how to determine an allowable MACRS recovery period. The MACRS<br />

personal property depreciation rates ( d t values) for n 3, 5, 7, 10, 15, and 20 for use in Equation<br />

[16.14] are included in Table 16–2.<br />

MACRS depreciation rates incorporate the DDB method ( d 2 n ) and switch to SL depreciation<br />

during the recovery period as an inherent component for personal property depreciation.<br />

The MACRS rates start with the DDB rate or the 150% DB rate and switch when the SL method<br />

offers faster write-off.<br />

TABLE 16–2<br />

Depreciation Rates d t Applied to the Basis B for the MACRS Method<br />

Depreciation Rate (%) for Each MACRS Recovery Period in Years<br />

Year n 3 n 5 n 7 n 10 n 15 n 20<br />

1 33.33 20.00 14.29 10.00 5.00 3.75<br />

2 44.45 32.00 24.49 18.00 9.50 7.22<br />

3 14.81 19.20 17.49 14.40 8.55 6.68<br />

4 7.41 11.52 12.49 11.52 7.70 6.18<br />

5 11.52 8.93 9.22 6.93 5.71<br />

6 5.76 8.92 7.37 6.23 5.29<br />

7 8.93 6.55 5.90 4.89<br />

8 4.46 6.55 5.90 4.52<br />

9 6.56 5.91 4.46<br />

10 6.55 5.90 4.46<br />

11 3.28 5.91 4.46<br />

12 5.90 4.46<br />

13 5.91 4.46<br />

14 5.90 4.46<br />

15 5.91 4.46<br />

16 2.95 4.46<br />

17–20 4.46<br />

21 2.23


424 Chapter 16 Depreciation Methods<br />

For real property, MACRS utilizes the SL method for n 39 throughout the recovery period.<br />

The annual percentage depreciation rate is d 139 0.02564. However, MACRS forces<br />

partial-year recovery in years 1 and 40. The MACRS real property rates in percentage amounts are<br />

Year 1 100 d 1 1.391%<br />

Years 2–39 100 d t 2.564%<br />

Year 40 100 d 40 1.177%<br />

The real property recovery period of 27.5 years, which applies only to residential rental property,<br />

uses the SL method in a similar fashion.<br />

Note that all MACRS depreciation rates in Table 16–2 are presented for 1 year longer than the<br />

stated recovery period. Also note that the extra-year rate is one-half of the previous year’s rate.<br />

This is so because a built-in half-year convention is imposed by MACRS. This convention assumes<br />

that all property is placed in service at the midpoint of the tax year of installation. Therefore,<br />

only 50% of the first-year DB depreciation applies for tax purposes. This removes some of<br />

the accelerated depreciation advantage and requires that one-half year of depreciation be taken in<br />

year n 1.<br />

No specially designed spreadsheet function is present for MACRS depreciation. However, the<br />

variable declining balance (VDB) function, which is used to determine when to switch between<br />

classical methods, can be adapted to display MACRS deprecation for each year. (The VDB function<br />

is explained in detail in Section 16A.2 of this chapter and Appendix A of the text.) The<br />

MACRS depreciation format of the VDB function requires embedded MAX and MIN functions,<br />

as follows:<br />

where<br />

VDB( B ,0, n , MAX(0, t 1.5), MIN( n , t 0.5),d) [16.17]<br />

B first cost<br />

0 salvage value of S 0<br />

n recovery period<br />

d <br />

2 if MACRS n 3, 5, 7, or 10<br />

{ 1.5 if MACRS n 15 or 20<br />

The MAX and MIN functions ensure that the MACRS half-year conventions are followed; that<br />

is, only one-half of the first year’s depreciation is charged in year 1, and one-half of the last year’s<br />

charge is carried over to year n 1.<br />

EXAMPLE 16.4<br />

Chevron Phillips Chemical Company in Baytown, Texas, acquired new equipment for its<br />

polyethylene processing line. This chemical is a resin used in plastic pipe, retail bags, blow<br />

molding, and injection molding. The equipment has an unadjusted basis of B $400,000, a<br />

life of only 3 years, and a salvage value of 5% of B. The chief engineer asked the finance director<br />

to provide an analysis of the difference between (1) the DDB method, which is the internal<br />

book depreciation and book value method used at the plant, and (2) the required<br />

MACRS tax depreciation and its book value. He is especially curious about the differences<br />

after 2 years of service for this short-lived, but expensive asset. Use hand and spreadsheet<br />

solutions to do the following:<br />

(a) Determine which method offers the larger total depreciation after 2 years.<br />

(b) Determine the book value for each method after 2 years and at the end of the recovery<br />

period.<br />

Solution by Hand<br />

The basis is B $400,000 and the estimated S 0.05(400,000) $20,000. The MACRS rates<br />

for n 3 are taken from Table 16–2, and the depreciation rate for DDB is d max 23 <br />

0.6667. Table 16–3 presents the depreciation and book values. Year 3 depreciation for DDB<br />

would be $44,444(0.6667) $29,629, except this would make BV 3 $20,000. Only the<br />

remaining amount of $24,444 is removed.


16.4 Modified Accelerated Cost Recovery System (MACRS) 425<br />

TABLE 16–3 Comparing MACRS and DDB Depreciation, Example 16.4<br />

Year<br />

Rate<br />

MACRS<br />

Tax<br />

Depreciation, $<br />

Book<br />

Value, $<br />

DDB<br />

Book<br />

Depreciation, $<br />

Book<br />

Value, $<br />

0 400,000 400,000<br />

1 0.3333 133,320 266,680 266,667 133,333<br />

2 0.4445 177,800 88,880 88,889 44,444<br />

3 0.1481 59,240 29,640 24,444 20,000<br />

4 0.0741 29,640 0<br />

(a) The 2-year accumulated depreciation values from Table 16–3 are<br />

MACRS: D 1 D 2 $133,320 177,800 $311,120<br />

DDB: D 1 D 2 $266,667 88,889 $355,556<br />

The DDB depreciation is larger. (Remember that for tax purposes, the company does not<br />

have the choice in the United States of DDB as applied here.)<br />

(b) After 2 years the book value for DDB at $44,444 is 50% of the MACRS book value of $88,880.<br />

At the end of recovery (4 years for MACRS due to the built-in half-year convention, and<br />

3 years for DDB), the MACRS book value is BV 4 0 and for DDB, BV 3 $20,000. This<br />

occurs because MACRS always removes the entire first cost, regardless of the estimated salvage<br />

value. This is a tax depreciation advantage of the MACRS method (unless the asset is<br />

disposed of for more than the MACRS-depreciated book value, as discussed in Section 17.4).<br />

Solution by Spreadsheet<br />

Figure 16–4 presents the spreadsheet solution using the VDB function (column B) for MACRS<br />

depreciation (in lieu of the MACRS rates) and applying the DDB function in column D.<br />

(a) The 2-year accumulated depreciation values are<br />

MACRS (add cells B6 B7): $133,333 177,778 $311,111<br />

DDB (add cells D6 D7): $266,667 88,889 $355,556<br />

(b) Book values after 2 years are<br />

MACRS (cell C7): $88,889<br />

DDB (cell E7): $44,444<br />

The book values are plotted in Figure 16–4. Observe that MACRS goes to zero in year 4, while<br />

DDB stops at $20,000 in year 3.<br />

Comment<br />

It is advisable to set up a spreadsheet template for use with depreciation problems in this and<br />

future chapters. The format and functions of Figure 16–4 are a good template for MACRS and<br />

DDB methods.<br />

Figure 16–4<br />

Spreadsheet screen shot of MACRS and DDB depreciation and book value, Example 16.4.


426 Chapter 16 Depreciation Methods<br />

MACRS simplifies depreciation computations, but it removes much of the flexibility of<br />

method selection for a business or corporation. In general, an economic comparison that includes<br />

depreciation may be performed more rapidly and usually without altering the final decision by<br />

applying the classical straight line method in lieu of MACRS.<br />

16.5 Determining the MACRS Recovery Period<br />

The expected useful life of property is estimated in years and used as the n value in alternative<br />

evaluation and in depreciation computations. For book depreciation the n value should be the<br />

expected useful life. However, when the depreciation will be claimed as tax deductible, the<br />

n value should be lower. There are tables that assist in determining the life and recovery period<br />

for tax purposes.<br />

The advantage of a recovery period shorter than the anticipated useful life is leveraged by the<br />

accelerated depreciation methods that write off more of the basis B in the initial years.<br />

The U.S. government requires that all depreciable property be classified into a property class<br />

which identifies its MACRS-allowed recovery period. Table 16–4, a summary of material from<br />

IRS Publication 946, gives examples of assets and the MACRS n values. Virtually any property<br />

considered in an economic analysis has a MACRS n value of 3, 5, 7, 10, 15, or 20 years.<br />

Table 16–4 provides two MACRS n values for each property. The first is the general depreciation<br />

system (GDS) value, which we use in examples and problems. The depreciation rates in Table<br />

16–2 correspond to the n values for the GDS column and provide the fastest write-off allowed. The<br />

rates utilize the DDB method or the 150% DB method with a switch to SL depreciation. Note that<br />

any asset not in a stated class is automatically assigned a 7-year recovery period under GDS.<br />

The far right column of Table 16–4 lists the alternative depreciation system ( ADS ) recovery<br />

period range. This alternative method allows the use of SL depreciation over a longer<br />

TABLE 16–4<br />

Example MACRS Recovery Periods for Various Asset Descriptions<br />

MACRS<br />

n Value, Years<br />

Asset Description (Personal and Real Property) GDS ADS Range<br />

Special manufacturing and handling devices, tractors,<br />

3 3–5<br />

racehorses<br />

Computers and peripherals, oil and gas drilling<br />

5 6–9.5<br />

equipment, construction assets, autos, trucks, buses,<br />

cargo containers, some manufacturing equipment<br />

Office furniture; some manufacturing equipment;<br />

7 10–15<br />

railroad cars, engines, tracks; agricultural machinery;<br />

petroleum and natural gas equipment; all<br />

property not in another class<br />

Equipment for water transportation, petroleum refining,<br />

10 15–19<br />

agriculture product processing, durable-goods<br />

manufacturing, shipbuilding<br />

Land improvements, docks, roads, drainage, bridges,<br />

15 20–24<br />

landscaping, pipelines, nuclear power production<br />

equipment, telephone distribution<br />

Municipal sewers, farm buildings, telephone switching<br />

20 25–50<br />

buildings, power production equipment (steam<br />

and hydraulic), water utilities<br />

Residential rental property (house, mobile home) 27.5 40<br />

Nonresidential real property attached to the land, but<br />

not the land itself<br />

39 40


16.6 Depletion Methods 427<br />

recovery period than the GDS. The half-year convention applies, and any salvage value is neglected,<br />

as it is in regular MACRS. The use of ADS is generally a choice left to a company, but it is required<br />

for some special asset situations. Since it takes longer to depreciate the asset, and since the SL model<br />

is required (thus removing the advantage of accelerated depreciation), ADS is usually not considered<br />

an option for the economic analysis. This electable SL option is, however, sometimes chosen<br />

by businesses that are young and do not need the tax benefit of accelerated depreciation during the<br />

first years of operation and asset ownership. If ADS is selected, tables of d t rates are available.<br />

16.6 Depletion Methods<br />

Previously, for all assets, facilities, and equipment that can be replaced we have applied depreciation.<br />

We now turn to irreplaceable natural resources and the equivalent of depreciation, which<br />

is called depletion.<br />

Depletion is a book method (noncash) to represent the decreasing value of a natural resource<br />

as it is recovered, removed, or felled. The two methods of depletion for book or tax purposes are<br />

used to write off the first cost, or value of the estimated quantity, of resources in mines, wells,<br />

quarries, geothermal deposits, forests, and the like.<br />

The two methods of depletion are cost and percentage depletion , as described below. Details for<br />

U.S. taxes on depletion are found in IRS Publication 535, Business Expenses .<br />

Cost depletion Sometimes referred to as factor depletion, cost depletion is based on the level<br />

of activity or usage, not time, as in depreciation. Cost depletion may be applied to most types of<br />

natural resources and must be applied to timber production. The cost depletion factor for year t , denoted<br />

by CD t , is the ratio of the first cost of the resource to the estimated number of units recoverable.<br />

CD t ————————<br />

first cost<br />

resource capacity<br />

[16.18]<br />

The annual depletion charge is CD t times the year’s usage or volume. The total cost depletion<br />

cannot exceed the fi rst cost of the resource. If the capacity of the property is reestimated some<br />

year in the future, a new cost depletion factor is determined based upon the undepleted amount<br />

and the new capacity estimate.<br />

Percentage depletion This is a special consideration given for natural resources. A constant,<br />

stated percentage of the resource’s gross income may be depleted each year provided it does not<br />

exceed 50% of the company’s taxable income. The depletion amount for year t is calculated as<br />

Percentage depletion t percentage depletion rate<br />

gross income from property<br />

PD GI t [16.19]<br />

Using percentage depletion, total depletion charges may exceed first cost with no limitation. The<br />

U.S. government does not generally allow percentage depletion to be applied to oil and gas wells<br />

(except small independent producers).<br />

The annual percentage depletion rates for some common natural deposits are listed below per<br />

U.S. tax law.<br />

Deposit<br />

Percentage of<br />

Gross Income, PD<br />

Sulfur, uranium, lead, nickel, zinc,<br />

22<br />

and some other ores and minerals<br />

Gold, silver, copper, iron ore, and<br />

15<br />

some oil shale<br />

Oil and natural gas wells (varies) 15–22<br />

Coal, lignite, sodium chloride 10<br />

Gravel, sand, peat, some stones 5<br />

Most other minerals, metallic ores 14


428 Chapter 16 Depreciation Methods<br />

EXAMPLE 16.5<br />

Temple-Inland Corporation has negotiated the rights to cut timber on privately held forest acreage<br />

for $700,000. An estimated 350 million board feet of lumber is harvestable.<br />

(a) Determine the depletion amount for the first 2 years if 15 million and 22 million board feet<br />

are removed.<br />

(b) After 2 years the total recoverable board feet was reestimated upward to be 450 million from<br />

the time the rights were purchased. Compute the new cost depletion factor for years 3 and later.<br />

Solution<br />

(a) Use Equation [16.18] for CD t in dollars per million board feet.<br />

700,000<br />

CD t ———— $2000 per million board feet<br />

350<br />

Multiply CD t by the annual harvest to obtain depletion of $30,000 in year 1 and $44,000<br />

in year 2. Continue until a total of $700,000 is written off.<br />

(b) After 2 years, a total of $74,000 has been depleted. A new CD t value must be calculated<br />

based on the remaining 700,000 74,000 $626,000 investment. Additionally, with the<br />

new estimate of 450 million board feet, a total of 450 15 22 413 million board feet<br />

remains. For years t 3, 4, . . . , the cost depletion factor is<br />

CD t ————<br />

626,000 $1516 per million board feet<br />

413<br />

EXAMPLE 16.6<br />

A gold mine was purchased for $10 million. It has an anticipated gross income of $5.0 million<br />

per year for years 1 to 5 and $3.0 million per year after year 5. Assume that depletion charges<br />

do not exceed 50% of taxable income. Compute annual depletion amounts for the mine. How<br />

long will it take to recover the initial investment at i 0%?<br />

Solution<br />

The rate for gold is PD 0.15. Depletion amounts are<br />

Years 1 to 5: 0.15(5.0 million) $750,000<br />

Years thereafter: 0.15(3.0 million) $450,000<br />

A total of $3.75 million is written off in 5 years, and the remaining $6.25 million is written off<br />

at $450,000 per year. The total number of years is<br />

$6.25 million<br />

5 —————— 5 13.9 18.9<br />

$450,000<br />

In 19 years, the initial investment could be fully depleted.<br />

In many of the natural resource depletion situations, the tax law allows the larger of the two<br />

depletion amounts to be claimed each year. This is allowed provided the percentage depletion<br />

amount does not exceed 50% of taxable income. Therefore, it is wise to calculate both depletion<br />

amounts and select the larger. Use the following terminology for year t ( t 1, 2, . . .).<br />

CDA t cost depletion amount<br />

PDA t percentage depletion amount<br />

TI t taxable income<br />

The guideline for the tax-allowed depletion amount for year t is<br />

Depletion { max[CDA t , PDA t ]<br />

max[CDA t , 50% of TI t ]<br />

if PDA t 50% of TI t<br />

if PDA t 50% of TI t


For example, assume a medium-sized quarry owner calculates the following for 1 year.<br />

TI $500,000 CDA $275,000 PDA $280,000<br />

Since 50% of TI is $250,000, the PDA is too large and, therefore, is not allowed. For tax purposes,<br />

apply the guideline above and use the cost depletion of $275,000, since it is larger than<br />

50% of TI.<br />

Chapter Summary 429<br />

CHAPTER SUMMARY<br />

Depreciation may be determined for internal company records (book depreciation) or for income<br />

tax purposes (tax depreciation). In the United States, the MACRS method is the only<br />

one allowed for tax depreciation. In many other countries, straight line and declining balance<br />

methods are applied for both tax and book depreciation. Depreciation does not result in cash<br />

flow directly. It is a book method by which the capital investment in tangible property is recovered.<br />

The annual depreciation amount is tax deductible, which can result in actual cash<br />

flow changes.<br />

Some important points about the straight line, declining balance, and MACRS methods are<br />

presented below. Common relations for each method are summarized in Table 16–5.<br />

Straight Line (SL)<br />

• It writes off capital investment linearly over n years.<br />

• The estimated salvage value is always considered.<br />

• This is the classical, nonaccelerated depreciation model.<br />

Declining Balance (DB)<br />

• The method accelerates depreciation compared to the straight line method.<br />

• The book value is reduced each year by a fixed percentage.<br />

• The most used rate is twice the SL rate, which is called double declining balance (DDB).<br />

• It has an implied salvage that may be lower than the estimated salvage.<br />

• It is not an approved tax depreciation method in the United States. It is frequently used for<br />

book depreciation purposes.<br />

Modified Accelerated Cost Recovery System (MACRS)<br />

• It is the only approved tax depreciation system in the United States.<br />

• It automatically switches from DDB or DB to SL depreciation.<br />

• It always depreciates to zero; that is, it assumes S 0.<br />

• Recovery periods are specified by property classes.<br />

• Depreciation rates are tabulated.<br />

• The actual recovery period is 1 year longer due to the imposed half-year convention.<br />

• MACRS straight line depreciation is an option, but recovery periods are longer than those for<br />

regular MACRS.<br />

Cost and percentage depletion methods recover investment in natural resources. The annual<br />

cost depletion factor is applied to the amount of resource removed. No more than the initial investment<br />

can be recovered with cost depletion. Percentage depletion, which can recover more<br />

than the initial investment, reduces the investment value by a constant percentage of gross income<br />

each year.<br />

TABLE 16–5<br />

Summary of Common Depreciation Method Relations<br />

Method MACRS SL DDB<br />

Fixed depreciation rate d Not defined — 1 2<br />

n —<br />

n<br />

Annual rate d t Table 16–2 —<br />

n<br />

1<br />

Annual depreciation D t d tB ——— B n<br />

S<br />

d(1 d)t1<br />

d(BVt1)<br />

Book value BV t BV t1 D t B tD t B(1 d) t


430 Chapter 16 Depreciation Methods<br />

CHAPTER 16 APPENDIX<br />

16A.1 Sum-of-Years-Digits (SYD) and Unit-of-<br />

Production (UOP) Depreciation<br />

The SYD method is a historical accelerated depreciation technique that removes much of the<br />

basis in the first one-third of the recovery period; however, write-off is not as rapid as for DDB<br />

or MACRS. This technique may be used in an engineering economy analysis in the book depreciation<br />

of multiple-asset accounts (group and composite depreciation).<br />

The mechanics of the method involve the sum of the year’s digits from 1 through the recovery<br />

period n . The depreciation charge for any given year is obtained by multiplying the basis of the<br />

asset, less any salvage value, by the ratio of the number of years remaining in the recovery period<br />

to the sum of the year’s digits SUM.<br />

depreciable years remaining<br />

D t ————————————— (basis salvage value)<br />

sum of years digits<br />

D t ————— n t 1 (B S) [16A.1]<br />

SUM<br />

where SUM is the sum of the digits 1 through n .<br />

The book value for any year t is calculated as<br />

BV t B <br />

jn<br />

n(n 1)<br />

SUM j ————<br />

2<br />

j1<br />

t(n t2 0.5)<br />

——————— (B S) [16A.2]<br />

SUM<br />

The rate of depreciation decreases each year and equals the multiplier in Equation [16A.1].<br />

d t ———— n t 1<br />

[16A.3]<br />

SUM<br />

The SYD spreadsheet function displays the depreciation for the year t . The function format is<br />

SYD( B , S , n , t )<br />

EXAMPLE 16A.1<br />

Calculate the SYD depreciation charges for year 2 for electro-optics equipment with<br />

B $25,000, S $4000, and an 8-year recovery period.<br />

Solution<br />

The sum of the year’s digits is 36, and the depreciation amount for the second year by Equation<br />

[16A.1] is<br />

D 2 —— 7 (21,000) $4083<br />

36<br />

The SYD function is SYD(25000,4000,8,2).<br />

Figure 16A–1 is a plot of the book values for an $80,000 asset with S $10,000 and n <br />

10 years using the four depreciation methods that we have learned. The MACRS, DDB, and SYD<br />

curves track closely except for year 1 and years 9 through 11.


16A.1 Sum-of-Years-Digits (SYD) and Unit-of-Production (UOP) Depreciation 431<br />

80<br />

70<br />

MACRS<br />

Book value BV t $1000<br />

60<br />

50<br />

40<br />

30<br />

20<br />

DDB<br />

SYD<br />

SL<br />

10<br />

S = $10,000<br />

0 1 2 3 4 5 6 7 8 9 10<br />

Year<br />

Figure 16A–1<br />

Comparison of book values using SL, SYD, DDB, and MACRS depreciation.<br />

11<br />

A second depreciation method that is not allowed for tax purposes, but useful in some situations<br />

is the unit-of-production (UOP) method . When the decreasing value of equipment is<br />

based on usage, not time , the UOP method is quite applicable. Suppose a highway contractor<br />

has a series of state highway department contracts that will last several years and that earth moving<br />

equipment is purchased for use on all contracts. If the equipment usage goes up and down<br />

significantly over the years, the UOP method is ideal for book depreciation. For year t , UOP<br />

deprecation is calculated as<br />

actual usage for year t<br />

D t —————————— (basis salvage)<br />

total lifetime usage<br />

[16A.4]<br />

EXAMPLE 16A.2<br />

Zachry Contractors purchased an $80,000 mixer for use during the next 10 years for contract<br />

work on IH-10 in San Antonio. The mixer will have a negligible salvage value after 10 years,<br />

and the total amount of material to process is estimated at 2 million m 3 . Use the actual usage per<br />

year shown in Table 16A–1 and the unit-of-production method to determine annual depreciation.<br />

Solution<br />

The actual usage each year is placed in the numerator of Equation [16A.4] to determine the annual<br />

depreciation based on the estimated total lifetime amount of material, 2 million m 3 in this<br />

case. Table 16A–1 shows the annual and cumulative depreciation over the 10 years. If the mixer<br />

is continued in service after the 2 million m 3 is processed, no further depreciation is allowed.<br />

TABLE 16A–1 Unit-of-Production Method of Depreciation, Example 16A.2<br />

Year t<br />

Actual<br />

Usage, 1000 m 3<br />

Annual<br />

Depreciation D t , $<br />

Cumulative<br />

Depreciation, $<br />

1 400 16,000 16,000<br />

2–8 200 8,000 72,000<br />

9–10 100 4,000 80,000<br />

Total 2000 80,000


432 Chapter 16 Depreciation Methods<br />

16A.2 Switching between Depreciation Methods<br />

Switching between depreciation methods may assist in accelerated reduction of the book value.<br />

It also maximizes the present value of accumulated and total depreciation over the recovery period.<br />

Therefore, switching usually increases the tax advantage in years where the depreciation is<br />

larger. The approach below is an inherent part of MACRS.<br />

Switching from a DB method to the SL method is the most common switch because it usually<br />

offers a real advantage, especially if the DB method is DDB. General rules of switching are summarized<br />

here.<br />

1. Switching is recommended when the depreciation for year t by the currently used method is<br />

less than that for a new method. The selected depreciation D t is the larger amount.<br />

2. Only one switch can take place during the recovery period.<br />

3. Regardless of the (classical) depreciation methods, the book value cannot go below the<br />

estimated salvage value. When switching from a DB method, the estimated salvage value,<br />

not the DB-implied salvage value, is used to compute the depreciation for the new method;<br />

we assume S 0 in all cases. (This does not apply to MACRS, since it already includes<br />

switching.)<br />

4. The undepreciated amount, that is, BV t , is used as the new adjusted basis to select the larger<br />

D t for the next switching decision.<br />

In all situations, the criterion is to maximize the present worth of the total depreciation PW D .<br />

The combination of depreciation methods that results in the largest present worth is the best<br />

switching strategy.<br />

tn<br />

PW D D t (PF, i, t)<br />

t1<br />

[16A.5]<br />

This logic minimizes tax liability in the early part of an asset’s recovery period.<br />

Switching is most advantageous from a rapid write-off method such as DDB to the SL model.<br />

This switch is predictably advantageous if the implied salvage value computed by Equation<br />

[16.11] exceeds the salvage value estimated at purchase time; that is, switch if<br />

BV n B (1 d ) n estimated S<br />

[16A.6]<br />

Since we assume that S will be zero per rule 3 above, and since BV n will be greater than zero, for<br />

a DB method a switch to SL is always advantageous. Depending upon the values of d and n , the<br />

switch may be best in the later years or last year of the recovery period, which removes the implied<br />

S inherent to the DDB model.<br />

The procedure to switch from DDB to SL depreciation is as follows:<br />

1. For each year t , compute the two depreciation charges.<br />

For DDB: D DDB d (BV t 1) [16A.7]<br />

For SL:<br />

BV t1<br />

D SL ————<br />

n t 1<br />

2. Select the larger depreciation value. The depreciation for each year is<br />

D t max[ D DDB , D SL ]<br />

[16A.8]<br />

[16A.9]<br />

3. If needed, determine the present worth of total depreciation, using Equation [16A.5].<br />

It is acceptable, though not usually financially advantageous, to state that a switch will take<br />

place in a particular year, for example, a mandated switch from DDB to SL in year 7 of a 10-year<br />

recovery period. This approach is usually not taken, but the switching technique will work<br />

correctly for all depreciation methods.<br />

To use a spreadsheet for switching, first understand the depreciation model switching rules<br />

and practice the switching procedure from declining balance to straight line. Once these are understood,<br />

the mechanics of the switching can be speeded up by applying the spreadsheet function


16A.2 Switching between Depreciation Methods 433<br />

VDB (variable declining balance). This is a quite powerful function that determines the depreciation<br />

for 1 year or the total over several years for the DB-to-SL switch. The function format is<br />

VDB(B, S, n,start_t,end_t, d,no_switch)<br />

[16A.10]<br />

Appendix A explains all the fields in detail, but for simple applications, where the DDB and SL<br />

annual D t values are needed, the following are correct entries:<br />

start_t is the year (t1)<br />

end_t is year t<br />

d is optional; 2 for DDB is assumed, the same as in the DDB function<br />

no_switch is an optional logical value:<br />

FALSE or omitted—switch to SL occurs, if advantageous<br />

TRUE—DDB or DB method is applied with no switching to SL depreciation considered.<br />

Entering TRUE for the no_switch option obviously causes the VDB function to display the same<br />

depreciation amounts as the DDB function. This is discussed in Example 16A.3 d . You may notice<br />

that the VDB function is the same one used to calculate annual MACRS depreciation.<br />

EXAMPLE 16A.3<br />

The Outback Steakhouse main office has purchased a $100,000 online document imaging system<br />

with an estimated useful life of 8 years and a tax depreciation recovery period of 5 years.<br />

Compare the present worth of total depreciation for (a) the SL method, (b) the DDB method,<br />

and (c) DDB-to-SL switching. (d) Perform the DDB-to-SL switch using a spreadsheet and plot<br />

the book values. Use a rate of i 15% per year.<br />

Solution by Hand<br />

The MACRS method is not involved in this solution.<br />

(a) Equation [16.1] determines the annual SL depreciation.<br />

D t ——————<br />

100,000 0 $20,000<br />

5<br />

Since D t is the same for all years, the PA factor replaces PF to compute PW D .<br />

PW D 20,000(PA,15%,5) 20,000(3.3522) $67,044<br />

(b) For DDB, d 25 0.40. The results are shown in Table 16A–2. The value PW D <br />

$69,915 exceeds $67,044 for SL depreciation. As is predictable, the accelerated depreciation<br />

of DDB increases PW D.<br />

(c) Use the DDB-to-SL switching procedure.<br />

1. The DDB values for D t in Table 16A–2 are repeated in Table 16A–3 for comparison<br />

with the D SL values from Equation [16A.8]. The D SL values change each year because<br />

BV t1 is different. Only in year 1 is D SL $20,000, the same as computed in part (a).<br />

For illustration, compute D SL values for years 2 and 4. For t 2, BV 1 $60,000 by the<br />

DDB method and<br />

D SL —————<br />

60,000 0<br />

5 2 1 $15,000<br />

For t 4, BV 3 $21,600 by the DDB method and<br />

D SL —————<br />

21,600 0<br />

5 4 1 $10,800<br />

2. The column “Larger D t” indicates a switch in year 4 with D 4 $10,800. The D SL <br />

$12,960 in year 5 would apply only if the switch occurred in year 5. Total depreciation<br />

with switching is $100,000 compared to the DDB amount of $92,224.<br />

3. With switching, PW D $73,943, which is an increase over both the SL and DDB<br />

methods.


434 Chapter 16 Depreciation Methods<br />

TABLE 16A–2<br />

DDB Model Depreciation and Present Worth Computations,<br />

Example 16A.3b<br />

Year<br />

t D t , $ BV t , $ (PF,15%,t)<br />

Present Worth<br />

of D t , $<br />

0 100,000<br />

1 40,000 60,000 0.8696 34,784<br />

2 24,000 36,000 0.7561 18,146<br />

3 14,400 21,600 0.6575 9,468<br />

4 8,640 12,960 0.5718 4,940<br />

5 5,184 7,776 0.4972 2,577<br />

Totals 92,224 69,915<br />

TABLE 16A–3<br />

Depreciation and Present Worth for DDB-to-SL Switching,<br />

Example 16A.3c<br />

Year<br />

t<br />

DDB Method, $<br />

BV t<br />

D DDB<br />

SL Method<br />

D SL, $<br />

Larger<br />

D t , $<br />

PF<br />

Factor<br />

Present<br />

Worth of<br />

D t, $<br />

0 — 100,000<br />

1 40,000 60,000 20,000 40,000 0.8696 34,784<br />

2 24,000 36,000 15,000 24,000 0.7561 18,146<br />

3 14,400 21,600 12,000 14,400 0.6575 9,468<br />

4* 8,640 12,960 10,800 10,800 0.5718 6,175<br />

5 5,184 7,776 12,960 10,800 0.4972 5,370<br />

Totals 92,224 100,000 73,943<br />

*Indicates year of switch from DDB to SL depreciation.<br />

Solution by Spreadsheet<br />

(d) In Figure 16A–2, column D entries are the VDB functions to determine that the DDB-to-<br />

SL switch should take place in year 4. The entries “2,FALSE” at the end of the VDB function<br />

are optional (see the VDB function description). If TRUE were entered, the declining<br />

balance model would be maintained throughout the recovery period, and the annual depreciation<br />

amounts would be equal to those in column B. The plot in Figure 16A–2 indicates<br />

another difference in depreciation methods. The terminal book value in year 5 for the DDB<br />

method is BV 5 $7776, while the DDB-to-SL switch reduces the book value to zero.<br />

The NPV function determines the PW of depreciation (row 9). The results here are the<br />

same as in parts (b) and (c) above. The DDB-to-SL switch has the larger PW D value.<br />

DDB($C$3,0,5,$A7)<br />

VDB($E$3,0,5,$A6,$A7,2,FALSE)<br />

Figure 16A–2<br />

Depreciation for DDB-to-SL switch using the VDB function, Example 16A.3.


16A.3 Determination of MACRS Rates 435<br />

In MACRS, recovery periods of 3, 5, 7, and 10 years apply DDB depreciation with half-year<br />

convention switching to SL. When the switch to SL takes place, which is usually in the last 1 to<br />

3 years of the recovery period, any remaining basis is charged off in year n 1 so that the book<br />

value reaches zero. Usually 50% of the applicable SL amount remains after the switch has occurred.<br />

For recovery periods of 15 and 20 years, 150% DB with the half-year convention and the<br />

switch to SL apply.<br />

The present worth of depreciation PW D will always indicate which method is the most advantageous.<br />

Only the MACRS rates for the GDS recovery periods (Table 16–4) utilize the DDBto-SL<br />

switch. The MACRS rates for the alternative depreciation system (ADS) have longer<br />

recovery periods and impose the SL model for the entire recovery period.<br />

EXAMPLE 16A.4<br />

In Example 16A.3, parts (c) and (d), the DDB-to-SL switching method was applied to a<br />

$100,000, n 5 years asset resulting in PW D $73,943 at i 15%. Use MACRS to depreciate<br />

the same asset for a 5-year recovery period, and compare PW D values.<br />

Solution<br />

Table 16A–4 summarizes the computations for depreciation (using Table 16–2 rates), book<br />

value, and present worth of depreciation. The PW D values for all four methods are as follows:<br />

DDB-to-SL switching $73,943<br />

Double declining balance $69,916<br />

MACRS $69,016<br />

Straight line $67,044<br />

MACRS provides a slightly less accelerated write-off. This is so, in part, because the half-year<br />

convention disallows 50% of the first-year DDB depreciation (which amounts to 20% of the<br />

basis). Also the MACRS recovery period extends to year 6, further reducing PW D .<br />

TABLE 16A–4<br />

Depreciation and Book Value Using MACRS,<br />

Example 16A.4<br />

t d t D t , $ BV t, $<br />

0 — — 100,000<br />

1 0.20 20,000 80,000<br />

2 0.32 32,000 48,000<br />

3 0.192 19,200 28,800<br />

4 0.1152 11,520 17,280<br />

5 0.1152 11,520 5,760<br />

6 0.0576 5,760 0<br />

1.000 100,000<br />

t6<br />

PW D D t(PF,15%,t) $69,016<br />

t1<br />

16A.3 Determination of MACRS Rates<br />

The depreciation rates for MACRS incorporate the DB-to-SL switching for all GDS recovery<br />

periods from 3 to 20 years. In the first year, some adjustments have been made to compute the<br />

MACRS rate. The adjustments vary and are not usually considered in detail in economic analyses.<br />

The half-year convention is always imposed, and any remaining book value in year n is removed<br />

in year n 1. The value S 0 is assumed for all MACRS schedules.<br />

Since different DB depreciation rates apply for different n values, the following summary may<br />

be used to determine D t and BV t values. The symbols D DB and D SL are used to identify DB and<br />

SL depreciation, respectively.


436 Chapter 16 Depreciation Methods<br />

For n 3, 5, 7, and 10 Use DDB depreciation with the half-year convention, switching to SL<br />

depreciation in year t when D SL D DB . Use the switching rules of Section 16A.2, and add onehalf<br />

year when computing D SL to account for the half-year convention. The yearly depreciation<br />

rates are<br />

{<br />

— 1 n t 1<br />

d t <br />

2—<br />

n t 2, 3, . . .<br />

[16A.11]<br />

Annual depreciation values for each year t applied to the adjusted basis, allowing for the halfyear<br />

convention, are<br />

D DB d t (BV t 1 )<br />

{<br />

— 1<br />

2 (<br />

— 1 n ) B t 1<br />

D SL <br />

BV t1<br />

————— t 2, 3, . . . , n<br />

n t 1.5<br />

[16A.12]<br />

[16A.13]<br />

After the switch to SL depreciation takes place—usually in the last 1 to 3 years of the recovery<br />

period—any remaining book value in year n is removed in year n 1.<br />

For n 15 and 20 Use 150% DB with the half-year convention and the switch to SL when<br />

D SL D DB . Until SL depreciation is more advantageous, the annual DB depreciation is computed<br />

using a form of Equation [16A.7]<br />

D DB d t (BV t 1 )<br />

where<br />

{<br />

—— 0.75 n t 1<br />

d t <br />

—— 1.50<br />

n t 2, 3, . . .<br />

[16A.14]<br />

EXAMPLE 16A.5<br />

A wireless tracking system for shop floor control with a MACRS 5-year recovery period has<br />

been purchased for $10,000. (a) Use Equations [16A.11] through [16A.13] to obtain the annual<br />

depreciation and book value. (b) Determine the resulting annual depreciation rates and compare<br />

them with the MACRS rates in Table 16–2 for n 5.<br />

Solution<br />

(a) With n 5 and the half-year convention, use the DDB-to-SL switching procedure to obtain<br />

the results in Table 16A–5. The switch to SL depreciation, which occurs in year 4<br />

when both depreciation values are equal, is indicated by<br />

D DB 0.4(2880) $1152<br />

D SL ————— 2880<br />

5 4 1.5 $1152<br />

The SL depreciation of $1000 in year 1 results from applying the half-year convention included<br />

in the first relation of Equation [16A.13]. Also, the SL depreciation of $576 in<br />

year 6 is the result of the half-year convention.<br />

(b) The actual rates are computed by dividing the “Larger D t ” column values by the first cost<br />

of $10,000. The rates below are the same as the Table 16–2 rates.<br />

t 1 2 3 4 5 6<br />

d t 0.20 0.32 0.192 0.1152 0.1152 0.0576


16A.3 Determination of MACRS Rates 437<br />

TABLE 16A–5<br />

Depreciation Amounts Used to Determine MACRS Rates<br />

for n 5, Example 16A.5<br />

Years<br />

DDB<br />

SL Depreciation Larger<br />

t d t D DB, $ D SL, $<br />

D t, $ BV t, $<br />

0 — — — — 10,000<br />

1 0.2 2,000 1,000 2,000 8,000<br />

2 0.4 3,200 1,777 3,200 4,800<br />

3 0.4 1,920 1,371 1,920 2,880<br />

4 0.4 1,152 1,152 1,152 1,728<br />

5 0.4 691 1,152 1,152 576<br />

6 — — 576 576 0<br />

10,000<br />

It is clearly easier to use the rates in Table 16–2 or the VDB spreadsheet function than to determine<br />

each MACRS rate using the switching logic above. But the logic behind the MACRS<br />

rates is described here for those interested. The annual MACRS rates may be derived by using<br />

the applicable rate for the DB method. The subscripts DB and SL have been inserted along with<br />

the year t . For the first year t 1,<br />

d DB,1 1 —<br />

n<br />

or d SL,1 1 —<br />

2 ( 1 —<br />

n )<br />

For summation purposes only, we introduce the subscript i ( i 1, 2, . . . , t ) on d . Then the depreciation<br />

rates for years t 2, 3, . . . , n are<br />

it1<br />

d DB,t d ( 1 <br />

i1<br />

it1<br />

( 1 <br />

i1<br />

d SL,t <br />

d i<br />

)<br />

d i<br />

)<br />

—————<br />

n t 1.5<br />

Also, for year n 1, the MACRS rate is one-half the SL rate of the previous year n.<br />

[16A.15]<br />

[16A.16]<br />

d SL,n1 0.5(d SL,n )<br />

[16A.17]<br />

The DB and SL rates are compared each year to determine which is larger and when the switch<br />

to SL depreciation should occur.<br />

EXAMPLE 16A.6<br />

Verify the MACRS rates in Table 16–2 for a 3-year recovery period. The rates in percent are<br />

33.33, 44.45, 14.81, and 7.41.<br />

Solution<br />

The fixed rate for DDB with n 3 is d 23 0.6667. Using the half-year convention in year 1<br />

and Equations [16A.15] through [16A.17], the results are as follows:<br />

d 1 : d DB,1 0.5d 0.5(0.6667) 0.3333<br />

d 2 : Cumulative depreciation rate is 0.3333.<br />

d DB,2 0.6667(1 0.3333) 0.4445<br />

d SL,2 ————— 1 0.3333<br />

3 2 1.5 0.2267<br />

(larger value)


438 Chapter 16 Depreciation Methods<br />

d 3 : Cumulative depreciation rate is 0.3333 0.4445 0.7778.<br />

d DB,3 0.6667(1 0.7778) 0.1481<br />

d SL,2 1 0.7778 —————<br />

3 3 1.5 0.1481<br />

Both values are the same; switch to straight line depreciation.<br />

d 4: This rate is 50% of the last SL rate.<br />

d 4 0.5(d SL,3) 0.5(0.1481) 0.0741<br />

PROBLEMS<br />

Fundamentals of Depreciation<br />

16.1 How does depreciation affect a company’s cash<br />

flow?<br />

16.2 What is the difference between book value and<br />

market value?<br />

16.3 State the difference between book depreciation<br />

and tax depreciation.<br />

16.4 State the difference between unadjusted and adjusted<br />

basis.<br />

16.5 Explain why the recovery period used for tax depreciation<br />

purposes may be different from the estimated<br />

n value in an engineering economy study.<br />

16.6 Visit the U.S. Internal Revenue Service website at<br />

www.irs.gov and answer the following questions<br />

about depreciation and MACRS by consulting<br />

Publication 946, How to Depreciate Property .<br />

(a) What is the definition of depreciation according<br />

to the IRS?<br />

(b) What is the description of the term salvage<br />

value ?<br />

(c) What are the two depreciation systems within<br />

MACRS, and what are the major differences<br />

between them?<br />

(d) What are the properties listed that cannot be<br />

depreciated under MACRS?<br />

(e) When does depreciation begin and end?<br />

(f) What is a Section 179 deduction?<br />

16.7 A major energy production company has the following<br />

information regarding the acquisition of<br />

new-generation equipment.<br />

Purchase price $580,000<br />

Transoceanic shipping and delivery cost $4300<br />

Installation cost (1 technician at $1600 per day<br />

for 4 days) $6400<br />

Tax recovery period 15 years<br />

Book depreciation recovery period 10 years<br />

Salvage value 10% of purchase price<br />

Operating cost (with technician) $185,000<br />

per year<br />

The manager of the department asked the newest<br />

hire to enter the appropriate data in the tax accounting<br />

program. For the MACRS method, what<br />

are the values of B , n , and S in depreciating the<br />

asset for tax purposes?<br />

16.8 Stahmann Products paid $350,000 for a numerical<br />

controller during the last month of 2007 and had it<br />

installed at a cost of $50,000. The recovery period<br />

was 7 years with an estimated salvage value of<br />

10% of the original purchase price. Stahmann sold<br />

the system at the end of 2011 for $45,000.<br />

(a) What numerical values are needed to develop<br />

a depreciation schedule at purchase time?<br />

(b) State the numerical values for the following:<br />

remaining life at sale time, market value in<br />

2011, book value at sale time if 65% of the<br />

basis had been depreciated.<br />

16.9 An asset with an unadjusted basis of $50,000 was<br />

depreciated over n tax 10 years for tax depreciation<br />

purposes and n book 5 years for book depreciation<br />

purposes. The annual depreciation was<br />

1 n using the relevant life value. Use a spreadsheet<br />

to plot on one graph the annual book value<br />

for both methods of depreciation.<br />

Straight Line Depreciation<br />

16.10 What is the depreciation rate d t per year for an<br />

asset that has an 8-year useful life and is straight<br />

line depreciated?<br />

16.11 Pneumatics <strong>Engineering</strong> purchased a machine that<br />

had a first cost of $40,000, an expected useful life<br />

of 8 years, a recovery period of 10 years, and a<br />

salvage value of $10,000. The operating cost of the<br />

machine is expected to be $15,000 per year. The<br />

inflation rate is 6% per year and the company’s


Problems 439<br />

MARR is 11% per year. Determine ( a ) the depreciation<br />

charge for year 3, ( b ) the present worth of<br />

the third-year depreciation charge in year 0, the<br />

time of asset purchase, and ( c ) the book value for<br />

year 3 according to the straight line method.<br />

16.12 An asset that is book-depreciated over a 5-year period<br />

by the straight line method has BV 3 $62,000<br />

with a depreciation charge of $26,000 per year.<br />

Determine ( a ) the first cost of the asset and ( b ) the<br />

assumed salvage value.<br />

16.13 Lee Company of Westbrook, Connecticut, manufactures<br />

pressure relief inserts for thermal relief<br />

and low-flow hydraulic pressure relief applications<br />

where zero leakage is required. A machine purchased<br />

3 years ago has been book-depreciated by<br />

the straight line method using a 5-year useful life.<br />

If the book value at the end of year 3 is $30,000 and<br />

the company assumed that the machine would be<br />

worthless at the end of its 5-year useful life,<br />

( a ) what is the book depreciation charge each year<br />

and ( b ) what was the first cost of the machine?<br />

16.14 An asset has an unadjusted basis of $200,000, a<br />

salvage value of $10,000, and a recovery period of<br />

7 years. Write a single-cell spreadsheet function to<br />

display the book value after 5 years of straight line<br />

depreciation. Use your function to determine the<br />

book value.<br />

16.15 Bristol Myers Squibb purchased a tablet-forming<br />

machine in 2008 for $750,000. The company<br />

planned to use the machine for 10 years; however,<br />

due to rapid obsolescence it will be retired after<br />

only 4 years in <strong>2012</strong>. Develop a spreadsheet for<br />

depreciation and book value amounts necessary to<br />

answer the following.<br />

(a) What is the amount of capital investment remaining<br />

when the asset is prematurely retired?<br />

(b) If the asset is sold at the end of 4 years for<br />

$175,000, what is the amount of capital investment<br />

lost based on straight line depreciation?<br />

( c) If the new-technology machine has an estimated<br />

cost of $300,000, how many more<br />

years should the company retain and depreciate<br />

the currently owned machine to make its<br />

book value and the first cost of the new machine<br />

equal to each other?<br />

16.16 A special-purpose graphics workstation acquired<br />

by Busbee Consultants has B $50,000 with a<br />

4-year recovery period. Tabulate the values for SL<br />

depreciation, accumulated depreciation, and book<br />

value for each year if ( a ) S 0 and ( b ) S $16,000.<br />

( c ) Use a spreadsheet to plot the book value over<br />

the 4 years on one chart for both salvage value<br />

estimates.<br />

16.17 A company owns the same asset in a U.S. plant<br />

(New York) and in a EU plant (Paris). It has B <br />

$2,000,000 and a salvage value of 20% of B . For<br />

tax depreciation purposes, the United States allows<br />

a straight line write-off over 5 years, while<br />

the EU allows SL write-off over 8 years. The general<br />

managers of the two plants want to know the<br />

difference in ( a ) the depreciation amount for year<br />

5 and ( b ) the book value after 5 years. Using a<br />

spreadsheet, write cell functions in only two cells<br />

to answer both questions.<br />

Declining Balance Depreciation<br />

16.18 When declining balance (DB) depreciation is applied,<br />

there can be three different depreciation<br />

rates involved— d , d max , and d t . Explain the differences<br />

between these rates.<br />

16.19 Equipment for immersion cooling of electronic<br />

components has an installed value of $182,000<br />

with an estimated trade-in value of $40,000 after<br />

15 years. For years 2 and 10, use DDB book depreciation<br />

to determine ( a ) the depreciation charge<br />

and ( b ) the book value.<br />

16.20 A cooling-water pumping station at the LCRA<br />

plant costs $600,000 to construct, and it is projected<br />

to have a 25-year life with an estimated<br />

salvage value of 15% of the construction cost.<br />

However, the station will be book-depreciated to<br />

zero over a recovery period of 30 years. Calculate<br />

the annual depreciation charge for years 4, 10,<br />

and 25, using ( a ) straight line depreciation and<br />

( b ) DDB depreciation. ( c ) What is the implied<br />

salvage value for DDB? ( d ) Use a spreadsheet to<br />

build the depreciation and book value schedules<br />

for both methods to verify your answers.<br />

16.21 A video recording system was purchased 3 years<br />

ago at a cost of $30,000. A 5-year recovery period<br />

and DDB depreciation have been used to write off<br />

the basis. The system is to be replaced this year with<br />

a trade-in value of $5000. What is the difference between<br />

the book value and the trade-in value?<br />

16.22 An engineer with Accenture Middle East BV in<br />

Dubai was asked by her client to help him understand<br />

the difference between 150% DB and DDB depreciation.<br />

Answer these questions if B $180,000,<br />

n 12 years, and S $30,000.<br />

( a) What are the book values after 12 years for<br />

both methods?<br />

( b) How do the estimated salvage and these book<br />

values compare in value after 12 years?<br />

( c) Which of the two methods, when calculated<br />

correctly considering S $30,000, writes off<br />

more of the first cost over 12 years?


440 Chapter 16 Depreciation Methods<br />

16.23 Exactly 10 years ago, Boyditch Professional Associates<br />

purchased $100,000 in depreciable assets<br />

with an estimated salvage of $10,000. For tax<br />

depreciation, the SL method with n 10 years<br />

was used, but for book depreciation, Boyditch applied<br />

the DDB method with n 7 years and neglected<br />

the salvage estimate. The company sold<br />

the assets today for $12,500.<br />

(a) Compare the sales price today with the book<br />

values using the SL and DDB methods.<br />

(b) If the salvage of $12,500 had been estimated<br />

exactly 10 years ago, determine the depreciation<br />

for each method in year 10.<br />

16.24 Shirley is studying depreciation in her engineering<br />

management course. The instructor asked her to<br />

graphically compare the total percent of first cost<br />

depreciated for an asset costing B dollars over a<br />

life of n 5 years for DDB and 125% DB depreciation.<br />

Help her by developing the plots of percent<br />

of B depreciated versus years. Use a spreadsheet<br />

unless otherwise instructed.<br />

MACRS Depreciation<br />

16.25 Explain the difference between an accelerated depreciation<br />

method and one that is not accelerated.<br />

Give an example of each.<br />

16.26 What was one of the prime reasons that MACRS<br />

depreciation was initiated in the mid-1980s?<br />

16.27 A company just purchased an intelligent robot,<br />

which has a first cost of $80,000. Since the robot is<br />

unique in its capabilities, the company expects to<br />

be able to sell it in 4 years for $95,000.<br />

(a) If the company spends $10,000 per year in<br />

maintenance and operation of the robot, what<br />

will the company’s MACRS depreciation<br />

charge be in year 2? Assume the recovery period<br />

for robots is 5 years and the company’s<br />

MARR is 16% per year when the inflation<br />

rate is 9% per year.<br />

(b) Determine the book value of the robot at the<br />

end of year 2.<br />

16.28 Animatics Corp. of Santa Clara, California, makes<br />

small servo systems with built-in controllers, amplifiers,<br />

and encoders so that they can control entire<br />

machines. The company purchased an asset 2 years<br />

ago that has a 5-year recovery period. The depreciation<br />

charge by the MACRS method for year 2 is $24,320.<br />

(a) What was the first cost of the asset?<br />

(b) How much was the depreciation charge in<br />

year 1?<br />

(c) Develop the complete MACRS depreciation<br />

and book value schedule using the VDB<br />

function.<br />

16.29 A plant manager for a large cable company knows<br />

that the remaining invested value of certain types<br />

of manufacturing equipment is more closely approximated<br />

when the equipment is depreciated<br />

linearly by the SL method compared to a rapid<br />

write-off method such as MACRS. Therefore, he<br />

keeps two sets of books, one for tax purposes<br />

(MACRS) and one for equipment management<br />

purposes (SL). For an asset that has a first cost of<br />

$80,000, a depreciable life of 5 years, and a salvage<br />

value equal to 25% of the first cost, determine<br />

the difference in the book values shown in<br />

the two sets of books at the end of year 4.<br />

16.30 The manager of a Glidden Paint manufacturing<br />

plant is aware that MACRS and DDB are both accelerated<br />

depreciation methods; however, out of curiosity,<br />

she wants to determine which one provides<br />

the faster write-off in the first 3 years for a recently<br />

purchased mixer that has a first cost of $300,000, a<br />

5-year recovery period, and a $60,000 salvage<br />

value. Determine which method yields the lower<br />

book value and by how much after 3 years. The annual<br />

MACRS depreciation rates are 20%, 32%, and<br />

19.2% for years 1, 2, and 3, respectively.<br />

16.31 Railroad cars used to transport coal from Wyoming<br />

mines to Texas power plants cost $1.2 million and<br />

have an estimated salvage value of $300,000. Develop<br />

the depreciation and book value schedules for<br />

the GDS MACRS method by using two methods on<br />

a spreadsheet—the VDB function and the MACRS<br />

rates. Are the book value series the same?<br />

16.32 A 120-metric-ton telescoping crane that cost<br />

$320,000 is owned by Upper State Power. Salvage<br />

is estimated at $75,000. ( a ) Compare book values<br />

for MACRS and standard SL depreciation over a<br />

7-year recovery period. ( b ) Explain how the estimated<br />

salvage is treated using MACRS.<br />

16.33 Youngblood Shipbuilding Yard just purchased<br />

$800,000 in capital equipment for ship repairing<br />

functions on dry-docked ships. Estimated salvage<br />

is $150,000 for any year after 5 years of use. Compare<br />

the depreciation and book value for year 3 for<br />

each of the following depreciation methods.<br />

(a) GDS MACRS where a recovery period of<br />

10 years is allowed<br />

(b) Double declining balance with a recovery<br />

period of 15 years<br />

(c) ADS straight line as an alternative to MACRS,<br />

with a recovery period of 15 years<br />

16.34 Basketball.com has installed $100,000 worth of<br />

depreciable software and equipment that represents<br />

the latest in Internet teaming and basket<br />

competition, intended to allow anyone to enjoy


Problems 441<br />

the sport on the Web or in the alley. No salvage<br />

value is estimated. The company can depreciate<br />

using MACRS for a 5-year recovery period or opt<br />

for the ADS alternate system over 10 years using<br />

the straight line method. The SL rates require the<br />

half-year convention; that is, only 50% of the regular<br />

annual rate applies for years 1 and 11.<br />

( a) Construct the book value curves for both<br />

methods on one graph. Show hand or spreadsheet<br />

computations as instructed.<br />

( b) After 3 years of use, what percentage of the<br />

$100,000 basis is removed for each method?<br />

Compare the two percentages.<br />

16.35 A company has purchased special-purpose equipment<br />

for the manufacture of rubber products<br />

(asset class 30.11 in IRS Publication 946) and expects<br />

to use it predominately outside the United<br />

States. In this case, the ADS alternative to<br />

MACRS is required for tax depreciation purposes.<br />

The manager wants to understand the difference<br />

in yearly recovery rates for classical SL, MACRS,<br />

and the ADS alternative to MACRS. Using a recovery<br />

period of 3 years, except for the ADS alternative,<br />

which requires a 4-year recovery with<br />

half-year convention included, prepare a single<br />

graph showing the annual recovery rates (in<br />

percent) for the three methods.<br />

Depletion<br />

16.36 A coal mine purchased 3 years ago for $7 million<br />

was estimated to contain 4,000,000 tons of coal.<br />

During the past 3 years the amount of coal removed<br />

was 21,000, 18,000, and 20,000 tons,<br />

respectively. The gross income obtained in these 3<br />

years was $257,000 for the first year, $320,000 for<br />

the second year, and $340,000 for the third year.<br />

Determine ( a ) the cost depletion allowance for<br />

each year and ( b ) the percentage of the purchase<br />

price depleted thus far.<br />

16.37 NA Forest Resources purchased forest acreage<br />

for $500,000 from which an estimated 200 million<br />

board feet of lumber is recoverable. The<br />

company will sell the lumber for $0.10 per board<br />

foot. No lumber will be sold for the next 2 years<br />

because an environmental impact statement<br />

must be completed before harvesting can begin.<br />

In years 3 to 10, however, the company expects<br />

to remove 20 million board feet per year. The<br />

inflation rate is 8%, and the company’s MARR<br />

is 10%.<br />

( a) Determine the depletion amount in year 2 by<br />

the cost depletion method.<br />

( b) Determine the depletion amount in year 5 by<br />

the percentage depletion method.<br />

16.38 A sand and gravel pit purchased for $900,000 is<br />

expected to yield 50,000 tons of gravel and 80,000<br />

tons of sand per year. The gravel will sell for<br />

$6 per ton and the sand for $9 per ton.<br />

(a) Determine the depletion charge according to<br />

the percentage depletion method. The percentage<br />

depletion rate for sand and gravel<br />

is 5%.<br />

(b) If taxable income is $100,000 for the year, is<br />

this depletion charge allowed? If not, how<br />

much is allowed?<br />

16.39 Vesco Mineral Resources purchased mineral rights<br />

to land in the foothills of the Santa Cristo mountains.<br />

The cost of the purchase was $9 million.<br />

Vesco originally thought that it would be able to<br />

extract 200,000 tons of lignite from the land, but<br />

further exploration revealed that 280,000 tons<br />

could be economically removed. If the company<br />

sold 20,000 tons in year 1 and 30,000 tons in year<br />

2, what would the depletion charges be each year<br />

according to the cost depletion method?<br />

16.40 A company owns gold mining operations in the<br />

United States, Australia, and South Africa. The<br />

Colorado mine has the taxable income and sales<br />

results summarized below. Determine the annual<br />

percentage depletion amount for the gold mine.<br />

Year<br />

Taxable<br />

Income, $<br />

Sales,<br />

Ounces<br />

Average<br />

Sales Price,<br />

$ per Ounce<br />

1 1,500,000 1800 1115<br />

2 2,000,000 5500 1221<br />

3 800,000 2300 1246<br />

16.41 A highway construction company operates a<br />

quarry. During the last 5 years, the amount extracted<br />

each year was 60,000, 50,000, 58,000<br />

60,000, and 65,000 tons. The mine is estimated to<br />

contain a total of 2.5 million tons of usable stones<br />

and gravel. The quarry land had an initial cost of<br />

$3.2 million. The company had a per-ton gross<br />

income of $30 for the first year, $25 for the second<br />

year, $35 for the next 2 years, and $40 for the<br />

last year.<br />

(a) Determine the depletion charge each year,<br />

using the larger of the values for the two depletion<br />

methods. Assume all depletion amounts<br />

are less than 50% of taxable income.<br />

(b) Compute the percent of the initial cost that<br />

has been written off in these 5 years, using<br />

the depletion charges in part ( a ).<br />

(c) If the quarry operation is reevaluated after<br />

the first 3 years of operation and estimated to<br />

contain a total of 1.5 million tons remaining,<br />

rework parts ( a ) and ( b ).


442 Chapter 16 Depreciation Methods<br />

ADDITIONAL PROBLEMS AND FE EXAM REVIEW QUESTIONS<br />

16.42 All of the following types of real property are depreciable<br />

except:<br />

(a) Warehouses<br />

(b) Land<br />

(c) Office buildings<br />

(d) Test facilities<br />

16.43 According to information on the IRS website, a<br />

taxpayer can take a depreciation deduction as long<br />

as the property meets all of the following requirements<br />

except:<br />

(a) The taxpayer must own the property.<br />

(b) The taxpayer must use the property in an<br />

(c)<br />

income-producing activity.<br />

The taxpayer must use the property for personal<br />

purposes.<br />

(d) The property must have a determinable useful<br />

life of more than 1 year.<br />

16.44 A machine with a 5-year life has a first cost of<br />

$20,000 and a $2000 salvage value. Its annual operating<br />

cost is $8000 per year. According to the<br />

classical straight line method, the depreciation<br />

charge in year 2 is nearest to:<br />

( a ) $2800 ( b ) $3600<br />

( c ) $4500 ( d ) $5300<br />

16.45 A machine with a 10-year life is MACRSdepreciated.<br />

The machine has a first cost of $40,000<br />

with a $5000 salvage value. Its annual operating<br />

cost is $7000 per year, and d t for years 1, 2, and 3 is<br />

10.00%, 18.00%, and 14.40%, respectively. The<br />

depreciation charge in year 3 is nearest to:<br />

( a ) $5800 ( b ) $7200<br />

( c ) $8500 ( d ) $9300<br />

16.46 A coal mine purchased for $5 million has enough<br />

coal to operate for 10 years. The annual cost is expected<br />

to be $200,000 per year. The coal is<br />

expected to sell for $150 per ton, with annual production<br />

expected to be 10,000 tons. Coal has a<br />

depletion percentage rate of 10%. The depletion<br />

charge for year 6 according to the percentage depletion<br />

method would be closest to:<br />

( a ) $75,000<br />

( b ) $100,000<br />

( c ) $125,000<br />

( d ) $150,000<br />

16.47 The depreciation charge for a 5-year, straight line<br />

depreciated vehicle is $3000 in year 4. If the first<br />

cost was $20,000, the salvage value used in the depreciation<br />

calculation was closest to:<br />

( a ) $0 ( b ) $2500<br />

( c ) $5000 ( d ) $7500<br />

16.48 The book value of an asset that was DDBdepreciated<br />

over a 10-year period was $5832 at the<br />

end of year 4. If the first cost of the asset was<br />

$80,000, the salvage value that was used in the depreciation<br />

calculation was closest to:<br />

( a ) $0 ( b ) $2000<br />

( c ) $5000 ( d ) $8000<br />

16.49 For an asset that has B $100,000, S $40,000,<br />

and a 10-year depreciable life, the book value at<br />

the end of year 4 according to the MACRS method<br />

would be closest to ( d t values for years 1, 2, 3, 4,<br />

and 5 are 10.00%, 18.00%, 14.40%, 11.52%, and<br />

9.22%, respectively):<br />

( a ) $58,700 ( b ) $62,400<br />

( c ) $53,900 ( d ) $46,100<br />

16.50 An asset that was depreciated over a 5-year period<br />

by the MACRS method had a BV of $33,025 at the<br />

end of year 3. If the MACRS depreciation rates for<br />

years 1, 2, and 3, were 0.20, 0.32, and 0.192, respectively,<br />

the basis of the asset was closest to:<br />

( a ) $158,000 ( b ) $172,000<br />

( c ) $185,000 ( d ) $193,000<br />

16.51 A lumber company purchased a tract of land for<br />

$70,000 that contained an estimated 25,000 usable<br />

trees. The value of the land was estimated at<br />

$20,000. In the first year of operation, the lumber<br />

company cut down 5000 trees. According to the<br />

cost depletion method, the depletion deduction for<br />

year 1 is closest to:<br />

( a ) $2000 ( b ) $7000<br />

( c ) $10,000 ( d ) $14,000<br />

16.52 An asset with a first cost of $50,000 and an estimated<br />

salvage value of $10,000 is depreciated by<br />

the MACRS method. If its book value at the end of<br />

year 3 is $21,850 and its market value is $25,850,<br />

the total amount of depreciation charged against<br />

the asset up to this time is closest to:<br />

( a ) $18,850 ( b ) $21,850<br />

( c ) $25,850 ( d ) $28,150<br />

16.53 Under the General Depreciation System (GDS) of<br />

asset classification, any asset that is not in a stated<br />

class is automatically assigned a recovery period of:<br />

( a ) 5 years (b) 7 years<br />

( c ) 10 years ( d ) 15 years<br />

16.54 All of the following statements about the Alternative<br />

Depreciation System (ADS) are true except:<br />

(a) The half-year convention applies.<br />

(b) Salvage value is neglected.<br />

(c) The recovery periods are shorter than in GDS.<br />

( d) The straight line method is required.


Appendix Problems 443<br />

APPENDIX PROBLEMS<br />

Sum-of-Years-Digits Depreciation<br />

16A.1 A European manufacturing company has new<br />

equipment with a first cost of 12,000 euros, an<br />

estimated salvage value of 2000 euros, and a<br />

recovery period of 8 years. Use the SYD method<br />

to tabulate annual depreciation and book value.<br />

16A.2 Earthmoving equipment with a first cost of<br />

$150,000 is expected to have a life of 10 years.<br />

The salvage value is expected to be 10% of the<br />

first cost. Calculate ( a ) by hand and ( b ) by<br />

spreadsheet the depreciation charge and book<br />

value for years 2 and 7 using the SYD method.<br />

16A.3 If B $12,000, n 6 years, and S is estimated<br />

at 15% of B , use the SYD method to determine<br />

( a ) the book value after 3 years and ( b ) the rate<br />

of depreciation and the depreciation amount in<br />

year 4.<br />

Unit-of-Production Depreciation<br />

16A.4 A robot used in simulated car crashes cost<br />

$70,000, has no salvage value, and has an<br />

expected capacity of tests not to exceed 10,000<br />

according to the manufacturer. Volvo Motors decided<br />

to use the unit-of-production depreciation<br />

method because the number of test crashes per<br />

year in which the robot would be involved was<br />

not estimable. Determine the annual depreciation<br />

and book value for the first 3 years if the number<br />

of tests were 3810, 2720, and 5390 per year.<br />

16A.5 A new hybrid car was purchased by Pedernales<br />

Electric Cooperative as a courier vehicle to<br />

transport items between its 12 city offices. The<br />

car cost $30,000 and was retained for 5 years.<br />

Alternatively, it could have been retained for<br />

100,000 miles. Salvage value is nil. Five-year<br />

DDB depreciation was applied. The car pool<br />

manager stated that he prefers UOP depreciation<br />

on vehicles because it writes off the first<br />

cost faster. Use the actual annual miles driven,<br />

listed below, to plot the book values for both<br />

methods. Determine which method would have<br />

removed the $30,000 faster. Show hand or<br />

spreadsheet solution, as instructed.<br />

Year 1 2 3 4 5<br />

Miles Driven, 1000 15 22 16 18 24<br />

Switching Methods<br />

16A.6 An asset has a first cost of $45,000, a recovery<br />

period of 5 years, and a $3000 salvage value.<br />

Use the switching procedure from DDB to SL<br />

depreciation, and calculate the present worth of<br />

depreciation at i 18% per year.<br />

16A.7 If B $45,000, S $3000, and n 5-year<br />

recovery period, use a spreadsheet and i <br />

18% per year to maximize the present worth of<br />

depreciation, using the following methods:<br />

DDB-to-SL switching (this was determined in<br />

Problem 16A.6) and MACRS. Given that<br />

MACRS is the required depreciation system in<br />

the United States, comment on the results.<br />

16A.8 Hempstead Industries has a new milling machine<br />

with B $110,000, n 10 years, and S $10,000.<br />

Determine the depreciation schedule and present<br />

worth of depreciation at i 12% per year, using<br />

the 175% DB method for the first 5 years and<br />

switching to the classical SL method for the last 5<br />

years. Use a spreadsheet to solve this problem.<br />

16A.9 Reliant Electric Company has erected a large<br />

portable building with a first cost of $155,000<br />

and an anticipated salvage of $50,000 after<br />

25 years. ( a ) Should the switch from DDB to<br />

SL depreciation be made? ( b ) For what values<br />

of the uniform depreciation rate in the DB<br />

method would it be advantageous to switch<br />

from DB to SL depreciation at some point in<br />

the life of the building?<br />

MACRS Rates<br />

16A.10 Verify the 5-year recovery period rates for<br />

MACRS given in Table 16–2. Start with the<br />

DDB method in year 1, and switch to SL depreciation<br />

when it offers a larger recovery rate.<br />

16A.11 A video recording system was purchased<br />

3 years ago at a cost of $30,000. A 5-year recovery<br />

period and MACRS depreciation have been used<br />

to write off the basis. The system is to be prematurely<br />

replaced with a trade-in value of $5000.<br />

Determine the MACRS depreciation, using the<br />

switching rules to find the difference between the<br />

book value and the trade-in value after 3 years.<br />

16A.12 Use the computations in Equations [16A.11]<br />

through [16A.13] to determine the MACRS annual<br />

depreciation for the following asset data:<br />

B $50,000 and a recovery period of 7 years.<br />

16A.13 The 3-year MACRS recovery rates are 33.33%,<br />

44.45%, 14.81%, and 7.41%, respectively.<br />

( a ) What are the corresponding rates for the alternative<br />

MACRS straight line ADS method<br />

with the half-year convention imposed?<br />

( b ) Compare the PW D values for these two methods<br />

if B $80,000 and i 15% per year.


CHAPTER 17<br />

After-Tax<br />

Economic<br />

Analysis<br />

LEARNING OUTCOMES<br />

Purpose: Perform an after-tax economic evaluation considering the impact of pertinent tax regulations, income taxes,<br />

and depreciation.<br />

SECTION TOPIC LEARNING OUTCOME<br />

17.1 Terminology and rates • Know the fundamental terms and relations of after-tax<br />

analysis; use a marginal tax rate table.<br />

17.2 CFBT and CFAT • Determine cash flow series before taxes and after taxes.<br />

17.3 Taxes and depreciation • Demonstrate the tax advantage of accelerated<br />

depreciation and shortened recovery periods.<br />

17.4 Depreciation recapture • Calculate the tax impact of DR; explain the use of<br />

capital gains and capital losses.<br />

17.5 After-tax analysis • Evaluate one project or multiple alternatives using<br />

after-tax PW, AW, and ROR analysis.<br />

17.6 After-tax replacement • Evaluate a defender and challenger in an after-tax<br />

replacement study.<br />

17.7 EVA analysis • Evaluate an alternative using after-tax economic valueadded<br />

analysis; compare to CFAT analysis.<br />

17.8 Taxes outside the United States • Understand the fundamental practices for depreciation<br />

and tax rates in international settings.<br />

17.9 VAT • Demonstrate the use and computation of a valueadded<br />

tax on manufactured products.


T<br />

his chapter provides an overview of tax terminology, income tax rates, and tax<br />

equations pertinent to an after-tax economic analysis. The change from estimating<br />

cash flow before taxes (CFBT) to cash flow after taxes (CFAT) involves a consideration<br />

of significant tax effects that may alter the final decision, and estimates the magnitude<br />

of the effect on cash flow over the life of the alternative that taxes may have.<br />

Mutually exclusive alternative comparisons using after-tax PW, AW, and ROR methods<br />

are explained with major tax implications considered. Replacement studies are discussed<br />

with tax effects that occur at the time that a defender is replaced. Also, the after-tax economic<br />

value added by an alternative is discussed in the context of annual worth analysis. All<br />

these methods use the procedures learned in earlier chapters, except now with tax effects<br />

considered.<br />

An after-tax evaluation using any method requires more computations than in previous<br />

chapters. Templates for tabulation of cash flow after taxes by hand and by spreadsheet<br />

are developed. Additional information on U.S. federal taxes—tax law and annually updated<br />

tax rates—is available through Internal Revenue Service publications and, more readily,<br />

on the IRS website www.irs.gov. Publications 542, Corporations , and 544, Sales and Other<br />

Dispositions of Assets , are especially applicable to this chapter. Some differences in tax considerations<br />

outside the United States are summarized.<br />

17.1 Income Tax Terminology and Basic Relations<br />

The perspective taken in engineering economy when performing an after-tax evaluation is that of<br />

the project and how relevant tax rules and allowances influence the economic decision. The perspective<br />

of a financial study is that of the corporation and how the tax structure and laws affect<br />

profitability. We take the engineering economy viewpoint in the sections that follow.<br />

There are many types of taxes levied upon corporations and individuals in all countries, including<br />

the United States. Some are sales tax, valued-added tax, import tax, income tax, highway<br />

tax, gasoline tax, and property (real estate) tax. Federal governments rely on income taxes for a<br />

significant portion of their annual revenue. States, provinces, and municipal-level governments<br />

rely on sales, value-added, and property taxes to maintain services, schools, etc. for the citizenry.<br />

For a starting point on corporate income taxes and how they are used when performing an aftertax<br />

economic evaluation of a project or multiple alternatives, this section covers basic definitions,<br />

terms, and relations.<br />

Income tax is the amount of the payment (taxes) on income or profit that must be delivered to a<br />

federal (or lower-level) government unit. Taxes are real cash flows ; however, for corporations<br />

tax computation requires some noncash elements, such as depreciation. Corporate income taxes<br />

are usually submitted quarterly, and the last payment of the year is submitted with the annual tax<br />

return.<br />

The Internal Revenue Service (IRS), a part of the U.S. Department of the Treasury, collects the<br />

taxes and enforces tax laws. The website www.irs.gov provides information on tax laws, rates,<br />

publications, etc. that are referenced in this chapter.<br />

Though the formulas are much more complex when applied to a specific situation, two fundamental<br />

relations form the basis for income tax computations. The first involves only actual cash<br />

flows:<br />

Net operating income revenue operating expenses<br />

The second involves actual cash flows and noncash deductibles, such as depreciation.<br />

Taxable income revenue operating expenses depreciation<br />

These terms and relations for corporations are now described. Since each term is calculated for<br />

1 year, there can be a subscript t ( t 1, 2, . . .) added; t is omitted here for simplicity.<br />

Operating revenue R , also commonly called gross income GI , is the total income realized<br />

from all revenue-producing sources. These incomes are listed in the income statement. (See<br />

Appendix B on accounting reports.) Other, nonoperating revenues such as sale of assets,<br />

license fee income, and royalties are considered separately for tax purposes.


446 Chapter 17 After-Tax Economic Analysis<br />

Operating expenses OE include all costs incurred in the transaction of business. These expenses<br />

are tax-deductible for corporations. For after-tax economic evaluations, the AOC (annual<br />

operating costs) and M&O (maintenance and operating) costs are applicable here. Depreciation<br />

is not included here since it is not an operating expense .<br />

Net operating income NOI, often called EBIT (earnings before interest and income taxes),<br />

is the difference between gross income and operating expenses.<br />

NOI EBIT GI OE [17.1]<br />

Taxable income TI is the amount of income upon which taxes are based. A corporation is<br />

allowed to remove depreciation , depletion and amortization, and some other deductibles<br />

from net operating income in determining the taxable income for a year. For our evaluations,<br />

we define taxable income as<br />

TI gross income operating expenses depreciation<br />

GI OE D [17.2]<br />

Though there may be subtleties and varying interpretations over time, in essence, the differences<br />

between NOI and TI are tax-law-allowed deductibles, such as depreciation. (In keeping<br />

with the project view of engineering economics, we will primarily use the TI relation when<br />

conducting an after-tax evaluation.)<br />

Tax rate T is a percentage, or decimal equivalent, of TI that is owed in taxes. The tax rates<br />

in many countries (including the United States) are graduated (or progressive) by level of<br />

TI; that is, higher rates apply as the TI increases. The marginal tax rate is the percentage<br />

paid on the last dollar of income . The average tax rate paid is calculated separately from the<br />

highest marginal rate used, as shown later. The general tax computation relation is<br />

Income taxes applicable tax rate taxable income<br />

( T )(TI) [17.3]<br />

Net operating profit after taxes NOPAT is the amount remaining each year after taxes are<br />

subtracted from taxable income.<br />

NOPAT TI taxes TI ( T )(TI)<br />

TI(1 T) [17.4]<br />

Basically, NOPAT represents the money remaining in the corporation as a result of the capital<br />

invested during the year. It is also called net profi t after taxes (NPAT).<br />

The graduated tax rate schedule for corporations is presented in Table 17–1 as taken from IRS<br />

Publication 542, Corporations . These are rates for the entire corporation, not for an individual<br />

project, though they are often applied in the after-tax analysis of a single project. The rates can<br />

change annually based upon government legislation; however, the corporate tax rate schedule has<br />

remained the same for some years. To illustrate the use of the graduated tax rate, assume a company<br />

is expected to generate a taxable income of $500,000 in 1 year. From Table 17–1 , the marginal<br />

tax rate for the last dollar of TI is 34%, but the graduated rates become progressively larger<br />

as TI increases. In this case, for TI $500,000,<br />

Taxes 113,900 0.34(500,000 335,000)<br />

113,900 56,100<br />

$170,000<br />

Alternatively, the rates for each TI level can be used to calculate taxes the longer way.<br />

Taxes 0.15(50,000) 0.25(75,000 50,000) 0.34(100,000 75,000)<br />

0.39(335,000 100,000) 0.34(500,000 335,000)<br />

7500 6250 8500 91,650 56,100<br />

$170,000


17.1 Income Tax Terminology and Basic Relations 447<br />

TABLE 17–1 U.S. Corporate Income Tax Rate Schedule (2010)<br />

Over<br />

But Not<br />

over<br />

If Taxable Income ($) Is:<br />

Tax Is<br />

Of the<br />

Amount over<br />

0 50,000 15% 0<br />

50,000 75,000 7,500 25% 50,000<br />

75,000 100,000 13,750 34% 75,000<br />

100,000 335,000 22,250 39% 100,000<br />

335,000 10,000,000 113,900 34% 335,000<br />

10,000,000 15,000,000 3,400,000 35% 10,000,000<br />

15,000,000 18,333,333 5,150,000 38% 15,000,000<br />

18,333,333 — 35% 0<br />

Smaller businesses (with TI $335,000) receive a slight tax advantage compared to large corporations.<br />

Once the TI exceeds $335,000, an effective federal tax rate of 34% applies, and when<br />

TI $18.33 million, there is a flat tax rate of 35%.<br />

As we move forward with after-tax analysis, it is important to keep the following in mind.<br />

The corporate tax rates apply to a corporation as a whole, not to a specific project, unless the<br />

project is the company. Tax rates are usually graduated by level of taxable income. Therefore, the<br />

last dollar of TI is taxed at a marginal rate. The average federal tax rate actually paid is lower than<br />

the highest marginal rate paid because the rates, in general, graduate to higher percentages as<br />

TI increases.<br />

Because the marginal tax rates change with TI, it is not possible to quote directly the percent<br />

of TI paid in income taxes. Alternatively, a single-value number, the average tax rate, is<br />

calculated as<br />

total taxes paid<br />

Average tax rate ———————<br />

taxable income ——— taxes<br />

[17.5]<br />

TI<br />

Referring to Table 17–1 , for a small business with TI $100,000, the federal income tax<br />

burden averages $22,250100,000 22.25%. If TI $15 million, the average tax rate is<br />

$5.15 million15 million 34.33%.<br />

As mentioned earlier, there are federal, state, and local taxes imposed. For the sake of simplicity,<br />

the tax rate used in an economy study is often a single-figure effective tax rate T e , which<br />

accounts for all taxes. Effective tax rates are in the range of 35% to 50%. One reason to use the<br />

effective tax rate is that state taxes are deductible for federal tax computation. The effective tax<br />

rate and taxes are calculated as<br />

EXAMPLE 17.1<br />

T e state rate (1 state rate)(federal rate) [17.6]<br />

Taxes ( T e )(TI) [17.7]<br />

REI (Recreational Equipment Incorporated) sells outdoor equipment and sporting goods through<br />

retail outlets, the Internet, and catalogs. Assume that for 1 year REI has the following financial<br />

results in the state of Kentucky, which has a flat tax rate of 6% on corporate taxable income.<br />

Total revenue<br />

$19.9 million<br />

Operating expenses<br />

$8.6 million<br />

Depreciation and other allowed deductions $1.8 million<br />

(a) Determine the state taxes and federal taxes due using Table 17–1 rates.<br />

(b) Find the average federal tax rate paid for the year.<br />

(c) Determine a single-value tax rate useful in economic evaluations using the average federal<br />

tax rate determined in part (b).<br />

(d) Estimate federal and state taxes using the single-value rate, and compare their total with the<br />

total in part (a).


448 Chapter 17 After-Tax Economic Analysis<br />

Solution<br />

(a) Calculate TI by Equation [17.2] and use Table 17–1 rates for federal taxes due.<br />

Kentucky state TI GI OE D 19.9 million 8.6 million 1.8 million<br />

$9.5 million<br />

Kentucky state taxes 0.06(TI) 0.06(9,500,000) $570,000<br />

Federal TI GI OE D state taxes 9,500,000 570,000<br />

$8,930,000<br />

Federal taxes 113,900 0.34(8,930,000 335,000) $3,036,200<br />

Total federal and state taxes 3,036,200 570,000 $3,606,200 [17.8]<br />

(b) From Equation [17.5], the average tax rate paid is approximately 32% of TI.<br />

Average federal tax rate 3,036,2009,500,000 0.3196<br />

(c) By Equation [17.6], T e is slightly over 36% per year for combined state and federal taxes.<br />

T e 0.06 (1 0.06)(0.3196) 0.3604 (36.04%)<br />

(d) Use the effective tax rate and TI $9.5 million from part (a) in Equation [17.7] to<br />

approximate total taxes.<br />

Taxes 0.3604(9,500,000) $3,423,800<br />

Compared to Equation [17.8], this approximation is $182,400 low, a 5.06% underestimate.<br />

It is interesting to understand how corporate tax and individual tax computations differ. Gross<br />

income for an individual taxpayer is comparable if revenue is replaced by salaries and wages.<br />

However, for an individual’s taxable income, most of the expenses for living and working are not<br />

tax deductible to the same degree as operating expenses are for corporations. For individual taxpayers,<br />

GI salaries wages interest and dividends other income<br />

TI GI personal exemption standard or itemized deductions<br />

Taxes ( T )(TI)<br />

For TI, operating expenses are replaced by individual exemptions and specific deductions. Exemptions<br />

are yourself, your spouse, your children, and your other dependents. Each exemption<br />

reduces TI by $3500 to $4000 per year, depending upon current exemption allowances.<br />

In the United States, the tax rates for individuals, like those for corporations, are graduated by<br />

level of TI. In 2010, the marginal rates ranged from 10% to 35%; however, the top marginal rates<br />

are increasing for individuals with larger TI amounts. These rates and TI levels are the subject of<br />

ongoing political debates at the U.S. national level depending upon the balance of power in the<br />

congressional bodies and the economic condition of the country. Once the marginal rates are set,<br />

the TI levels are adjusted each year to account for inflation and other factors. This process is<br />

called indexing . Clearly, tax rates for individuals change much more frequently than the rates for<br />

corporations change. Current information is available on the IRS website www.irs.gov through<br />

Publication 17, Your Federal Income Tax. The current rate schedule is published in the back of<br />

this document for four filing status categories:<br />

Unmarried individuals (single)<br />

Married filing jointly<br />

Married filing separately<br />

Head of household<br />

17.2 Calculation of Cash Flow after Taxes<br />

Early in the text, the term net cash fl ow ( NCF ) was identified as the best estimate of actual cash<br />

flow each year. The NCF is calculated as cash inflows minus cash outflows. Since then, the annual<br />

NCF amounts have been used many times to perform alternative evaluations via the PW,<br />

AW, ROR, and B/C methods. Now that the impact on cash flow of depreciation and related taxes


17.2 Calculation of Cash Flow after Taxes 449<br />

will be considered, it is time to expand our terminology. NCF is replaced by the term cash flow<br />

before taxes (CFBT) , and we introduce the new term cash flow after taxes (CFAT) .<br />

CFBT and CFAT are actual cash flows ; that is, they represent the estimated actual flow of<br />

money into and out of the corporation that will result from the alternative. The remainder of<br />

this section explains how to transition from before-tax to after-tax cash flows for solutions by<br />

hand and by spreadsheet, using tax regulations described in the next few sections. Once the<br />

CFAT estimates are developed, the economic evaluation is performed using the same methods<br />

and selection guidelines applied previously. However, the analysis is performed on the CFAT<br />

estimates.<br />

We learned that net operating income (NOI) does not include the purchase or sale of capital<br />

assets. However, the annual CFBT estimate must include the initial capital investment and salvage<br />

value for the years in which they occur. Incorporating the definitions of gross income and<br />

operating expenses from NOI, CFBT for any year is defined as<br />

CFBT gross income operating expenses initial investment salvage value<br />

GI OE P S [17.9]<br />

As in previous chapters, P is the initial investment (year 0) and S is the estimated salvage value<br />

in year n. Therefore, only in year 0 will the CFBT include P , and only in year n will an S value<br />

be present. Once all taxes are estimated, the annual after-tax cash flow is simply<br />

CFAT CFBT taxes [17.10]<br />

where taxes are estimated using the relation ( T )(TI) or ( T e )(TI).<br />

We know from Equation [17.2] that depreciation D is considered when calculating TI. It is<br />

very important to understand the different roles of depreciation for income tax computations and<br />

in CFAT estimation.<br />

Depreciation is not an operating expense and is a noncash flow. Depreciation is tax deductible for<br />

determining the amount of income taxes only, but it does not represent a direct, after-tax cash flow.<br />

Therefore, the after-tax engineering economy study must be based on actual cash flow estimates,<br />

that is, annual CFAT estimates that do not include depreciation as an expense (negative cash flow).<br />

Accordingly, if the CFAT expression is determined using the TI relation, depreciation must not<br />

be included outside of the TI component. Equations [17.9] and [17.10] are now combined.<br />

CFAT GI OE P S (GI OE D )( T e ) [17.11]<br />

Suggested table column headings for CFBT and CFAT calculations by hand or by spreadsheet are<br />

shown in Table 17–2 . The equations are shown in column numbers, with the effective tax rate T e<br />

used for income tax estimation. Expenses OE and initial investment P carry negative signs in all<br />

tables and spreadsheets.<br />

A negative TI value may occur in some years due to a depreciation amount that is larger than<br />

(GI − OE). It is possible to account for this in a detailed after-tax analysis using carry-forward<br />

and carry-back rules for operating losses. It is the exception that the engineering economy study<br />

will consider this level of detail. Rather, the associated negative income tax is considered as a tax<br />

savings for the year . The assumption is that the negative tax will offset taxes for the same year<br />

in other income-producing areas of the corporation.<br />

TABLE 17–2<br />

Suggested Column Headings for Calculation of CFAT<br />

Year<br />

Gross<br />

Income<br />

GI<br />

Operating<br />

Expenses<br />

OE<br />

Investment<br />

and<br />

Salvage<br />

P and S<br />

CFBT<br />

(1) (2) (3) (4) <br />

(1) (2) (3)<br />

Depreciation<br />

D<br />

(5) (6) <br />

(1) (2) (5)<br />

Taxable<br />

Income<br />

TI Taxes CFAT<br />

(7) <br />

T e (6)<br />

(8) <br />

(4) (7)


450 Chapter 17 After-Tax Economic Analysis<br />

EXAMPLE 17.2<br />

Wilson Security has received a contract to provide additional security for corporate and government<br />

personnel along the Arizona-Mexico border. Wilson plans to purchase listening and detection<br />

equipment for use in the 6-year contract. The equipment is expected to cost $550,000 and have a<br />

resale value of $150,000 after 6 years. Based on the incentive clause in the contract, Wilson estimates<br />

that the equipment will increase contract revenue by $200,000 per year and require an additional<br />

M&O expense of $90,000 per year. MACRS depreciation allows recovery in 5 years, and<br />

the effective corporate tax rate is 35% per year. Tabulate and plot the CFBT and CFAT series.<br />

Solution<br />

The spreadsheet in Figure 17–1 presents before-tax and after-tax cash flows using the format<br />

of Table 17–2. The functions for year 6 are detailed in row 11. Discussion and sample calculations<br />

follow.<br />

CFBT: The operating expenses OE and initial investment P are shown as negative cash flows.<br />

The $150,000 salvage (resale) is a positive cash flow in year 6. CFBT is calculated by Equation<br />

[17.9]. In year 6, for example, when the equipment is sold, the function in row 11 indicates that<br />

CFBT 6 200,000 90,000 150,000 $260,000<br />

CFAT: Column F for MACRS depreciation, which is determined using the VDB function<br />

over the 6-year period, writes off the entire $550,000 investment. Taxable income, taxes,<br />

and CFAT are calculated, using year 4 as an example.<br />

TI 4 GI OE D 200,000 90,000 63,360 $46,640<br />

Taxes 4 (0.35) (TI) (0.35) (46,640) $16,324<br />

CFAT 4 GI OE taxes 200,000 − 90,000 16,324 $93,676<br />

In year 2, MACRS depreciation is large enough to cause TI to be negative ($−66,000). As<br />

mentioned above, the negative tax ($−23,100) is considered a tax savings in year 2, thus<br />

increasing CFAT.<br />

Comment<br />

MACRS depreciates to a salvage value of S 0. Later we will learn about a tax implication<br />

due to “recapturing of depreciation” when an asset is sold for an amount larger than zero and<br />

MACRS was applied to fully depreciate the asset to zero.<br />

Figure 17–1<br />

Computation of CFBT and CFAT using MACRS depreciation and T e 35%, Example 17.2.<br />

17.3 Effect on Taxes of Different Depreciation<br />

Methods and Recovery Periods<br />

It is important to understand why accelerated depreciation rates give the corporation a tax advantage<br />

relative to the straight line method with the same recovery period. Larger rates in earlier<br />

years of the recovery period require less taxes due to the larger reductions in taxable income. The<br />

criterion of minimizing the present worth of taxes is used to demonstrate the tax effect. For the


17.3 Effect on Taxes of Different Depreciation Methods and Recovery Periods 451<br />

recovery period n , choose the depreciation rates that result in the minimum present worth value<br />

for taxes.<br />

tn<br />

PW tax (taxes in year t)(PF, i, t) [17.12]<br />

t1<br />

This is equivalent to maximizing the present worth of total depreciation PW D .<br />

Compare any two depreciation methods. Assume the following: (1) There is a constant singlevalue<br />

tax rate, (2) CFBT exceeds the annual depreciation amount, (3) both methods reduce book<br />

value to the same salvage value, and (4) the same recovery period is used. On the basis of these<br />

assumptions, the following statements are correct:<br />

The total taxes paid are equal for all depreciation methods.<br />

The present worth of taxes is less for accelerated depreciation methods.<br />

As we learned in Chapter 16, MACRS is the prescribed tax depreciation method in the United<br />

States, and the only alternative is MACRS straight line depreciation with an extended recovery<br />

period. The accelerated write-off of MACRS always provides a smaller PW tax compared to less<br />

accelerated methods. If the DDB method were still allowed directly, rather than embedded in<br />

MACRS, DDB would not fare as well as MACRS. This is so because DDB does not reduce the<br />

book value to zero. This is illustrated in Example 17.3 .<br />

EXAMPLE 17.3<br />

An after-tax analysis for a new $50,000 machine proposed for a fiber optics manufacturing line<br />

is in process. The CFBT for the machine is estimated at $20,000. If a recovery period of 5 years<br />

applies, use the present worth of taxes criterion, an effective tax rate of 35%, and a return of<br />

8% per year to compare the following: classical straight line, classical DDB, and required<br />

MACRS depreciation. Use a 6-year period for the comparison to accommodate the half-year<br />

convention imposed by MACRS.<br />

Solution<br />

Table 17–3 presents a summary of annual depreciation, taxable income, and taxes for each<br />

method. For classical straight line depreciation with n 5, D t $10,000 for 5 years and D 6 <br />

0 (column 3). The CFBT of $20,000 is fully taxed at 35% in year 6.<br />

The classical DDB percentage of d 2n 0.40 is applied for 5 years. The implied salvage<br />

value is $50,000 − 46,112 $3888, so not all $50,000 is tax-deductible. The taxes using classical<br />

DDB will be $3888(0.35) $1361 larger than for the classical SL method.<br />

TABLE 17–3<br />

Comparison of Taxes and Present Worth of Taxes for Different Depreciation Methods<br />

(1)<br />

Year t<br />

(2)<br />

CFBT, $<br />

(3)<br />

D t , $<br />

Classical Straight Line<br />

(4)<br />

TI, $<br />

(5) 0.35(4)<br />

Taxes, $<br />

Classical Double Declining<br />

Balance<br />

(6)<br />

D t , $<br />

(7)<br />

TI, $<br />

(8) 0.35(7)<br />

Taxes, $<br />

(9)<br />

D t , $<br />

(10)<br />

TI, $<br />

MACRS<br />

(11) 0.35(10)<br />

Taxes, $<br />

1 20,000 10,000 10,000 3,500 20,000 0 0 10,000 10,000 3,500<br />

2 20,000 10,000 10,000 3,500 12,000 8,000 2,800 16,000 4,000 1,400<br />

3 20,000 10,000 10,000 3,500 7,200 12,800 4,480 9,600 10,400 3,640<br />

4 20,000 10,000 10,000 3,500 4,320 15,680 5,488 5,760 14,240 4,984<br />

5 20,000 10,000 10,000 3,500 2,592 17,408 6,093 5,760 14,240 4,984<br />

6 20,000 0 20,000 7,000 0 20,000 7,000 2,880 17,120 5,992<br />

Totals 50,000 24,500 46,112 25,861* 50,000 24,500<br />

PW tax 18,386 18,549 18,162<br />

*Larger than other values since there is an implied salvage value of $3888 not recovered.


452 Chapter 17 After-Tax Economic Analysis<br />

25,000<br />

20,000<br />

Accumulated taxes, $<br />

15,000<br />

10,000<br />

MACRS<br />

method<br />

SL<br />

method<br />

DDB method<br />

5,000<br />

0 1<br />

2 3<br />

Year, t<br />

4 5 6<br />

Figure 17–2<br />

Cumulative taxes incurred by different depreciation rates for a<br />

6-year comparison period, Example 17.3.<br />

MACRS writes off $50,000 in 6 years using the rates of Table 16–2. Total taxes are $24,500,<br />

the same as for classical SL depreciation.<br />

The annual taxes (columns 5, 8, and 11) are accumulated year by year in Figure 17–2. Note<br />

the pattern of the curves, especially the lower total taxes relative to the SL model after year 1<br />

for MACRS and in years 1 through 4 for DDB. These higher tax values for SL cause PW tax for<br />

SL depreciation to be larger. The PW tax values at the bottom of Table 17–3 are calculated using<br />

Equation [17.12]. The MACRS PW tax value is the smallest at $18,162.<br />

To compare taxes for different recovery periods , change only assumption 4 at the beginning<br />

of this section to read: The same depreciation method is applied. It can be shown that a shorter<br />

recovery period will offer a tax advantage over a longer period using the criterion to minimize<br />

PW tax . Comparison will indicate that<br />

The total taxes paid are equal for all n values.<br />

The present worth of taxes is less for smaller n values.<br />

This is why corporations want to use the shortest MACRS recovery period allowed for income<br />

tax purposes. Example 17.4 demonstrates these conclusions for classical straight line depreciation,<br />

but the conclusions are correct for MACRS or any other tax depreciation method.<br />

EXAMPLE 17.4<br />

Grupo Grande Maquinaría, a diversified manufacturing corporation based in Mexico, maintains<br />

parallel records for depreciable assets in its European operations in Germany. This is<br />

common for multinational corporations. One set is for corporate use that reflects the estimated<br />

useful life of assets. The second set is for foreign government purposes, such as<br />

depreciation and taxes.<br />

The company just purchased an asset for $90,000 with an estimated useful life of 9 years;<br />

however, a shorter recovery period of 5 years is allowed by German tax law. Demonstrate the<br />

tax advantage for the smaller n if net operating income (NOI) is $30,000 per year, an effective<br />

tax rate of 35% applies, invested money is returning 5% per year after taxes, and classical SL<br />

depreciation is allowed. Neglect the effect of any salvage value.<br />

Solution<br />

Determine the annual TI and taxes by Equations [17.2] through [17.3] and the present worth of<br />

taxes using Equation [17.12] for both n values.


17.4 Depreciation Recapture and Capital Gains (Losses) 453<br />

Useful life n 9 years:<br />

D ———<br />

90,000 10,000<br />

9<br />

TI 30,000 10,000 $20,000 per year<br />

Taxes (0.35)(20,000) $7000 per year<br />

PW tax 7000(PA,5%,9) $49,755<br />

Total taxes (7000)(9) $63,000<br />

Recovery period n 5 years:<br />

Use the same comparison period of 9 years, but depreciation occurs only during the first 5 years.<br />

———<br />

D t <br />

{ 90,000 $18,000 t 1 to 5<br />

5<br />

0 t 6 to 9<br />

18,000) $4200 t 1 to 5<br />

Taxes {(0.35)(30,000) $10,500 t 6 to 9<br />

PW tax 4200(PA,5%,5) 10,500(PA,5%,4)(PF,5%,5)<br />

$47,356<br />

Total taxes 4200(5) 10,500(4) $63,000<br />

A total of $63,000 in taxes is paid in both cases. However, the more rapid write-off for<br />

n 5 results in a present worth tax savings of nearly $2400 (49,755 47,356).<br />

17.4 Depreciation Recapture and Capital<br />

Gains (Losses)<br />

All the tax implications discussed here are the result of disposing of a depreciable asset before, at,<br />

or after its recovery period. In an after-tax economic analysis of large investment assets, these tax<br />

effects should be considered. The key is the size of the selling price (or salvage or market value)<br />

relative to the current book value at disposal time and relative to the first cost, which is called the<br />

unadjusted basis B in depreciation terminology. There are three relevant terms.<br />

Depreciation recapture DR , also called ordinary gain , occurs when a depreciable asset is<br />

sold for more than the current book value BV t . As shown in Figure 17–3 ,<br />

Depreciation recapture selling price book value<br />

DR SP 2 BV t [17.13]<br />

If selling price<br />

(SP) at time of<br />

sale is:<br />

The capital gain,<br />

depreciation recapture,<br />

or capital loss is:<br />

For an after-tax<br />

study, the tax<br />

effect is:<br />

First cost P<br />

or basis B<br />

SP 1<br />

CG<br />

plus<br />

CG: Taxed at T e<br />

(after CL offset)<br />

DR<br />

DR: Taxed at T e<br />

Book<br />

value BV t<br />

SP 2<br />

SP 3<br />

CL<br />

DR<br />

CL: Can only offset<br />

CG<br />

Zero, $0<br />

Figure 17–3<br />

Summary of calculations and tax treatment for depreciation recapture and capital gains (losses).


454 Chapter 17 After-Tax Economic Analysis<br />

Depreciation recapture is often present in the after-tax study . In the United States, an amount<br />

equal to the estimated salvage value can always be anticipated as DR when the asset is disposed<br />

of after the MACRS recovery period. This is correct simply because MACRS depreciates<br />

every asset to zero in n 1 years. The amount of DR is treated as ordinary taxable income<br />

in the year of asset disposal.<br />

Capital gain CG is an amount incurred when the selling price exceeds its (unadjusted) basis<br />

B . See Figure 17–3 .<br />

Capital gain selling price basis<br />

CG SP 1 B [17.14]<br />

Since future capital gains are difficult to predict, they are usually not detailed in an after-tax economy<br />

study. An exception is for assets that historically increase in value, such as buildings and land.<br />

When the selling price exceeds B , the TI due to the sale is the capital gain plus the depreciation<br />

recapture, as shown in Figure 17–3 . The DR is now the total amount of depreciation<br />

taken thus far, that is, B BV .<br />

Capital loss CL occurs when a depreciable asset is disposed of for less than its current book<br />

value. In Figure 17–3 ,<br />

Capital loss book value selling price<br />

CL BV t SP 3 [17.15]<br />

An economic analysis does not commonly account for capital loss, simply because it is not<br />

estimable for a specific alternative. However, an after-tax replacement study should account<br />

for any capital loss if the defender must be traded at a “sacrifice” price. For the purposes of the<br />

economic study, this provides a tax savings in the year of replacement. Use the effective tax<br />

rate to estimate the tax savings. These savings are assumed to be offset elsewhere in the corporation<br />

by other income-producing assets that generate taxes.<br />

There are several additional points worth mentioning about capital gains and capital losses for<br />

a corporation, apart from their presence in an economic evaluation.<br />

• U.S. tax law defines capital gains as long-term (items retained for more than 1 year) or short-term.<br />

• Capital gains actually take place for property that is not depreciated or amortized. The term<br />

capital gain correctly applies at sale time to property such as investments (stocks and bonds),<br />

art, jewelry, land, and the like. When a depreciable asset’s selling price is higher than the<br />

original cost (its basis), it is correctly termed an ordinary gain . Corporate tax treatment at this<br />

time is the same for both: taxed as ordinary income. All said, it is common to classify an<br />

expected ordinary gain on a depreciable asset as a capital gain, without altering the economic<br />

decision.<br />

• Capital gains are taxed as ordinary taxable income at the corporation’s regular tax rates.<br />

• Capital losses do not directly reduce annual income taxes because they can only be netted<br />

against capital gains to the maximum extent of the capital gains for the year. The terms used<br />

then are net capital gains (losses).<br />

• When capital losses exceed capital gains, the corporation can take advantage of carry-back<br />

and carry-forward tax laws for the excess, something of value to a finance or tax officer, not<br />

an engineer doing an economic analysis.<br />

• When an asset is disposed of, the tax treatment is referred to as a Section 1231 transaction ,<br />

which is the IRS rule section of the same number.<br />

• IRS Publication 544, Sales and Other Dispositions of Assets , may be helpful if gains and<br />

losses are present in a study.<br />

• All these rules apply to corporations. Individual tax rules and rates are different concerning<br />

asset disposition.<br />

If the three additional income and tax elements covered here are incorporated into Equation<br />

[17.2], taxable income is defined as<br />

TI gross income operating expenses depreciation<br />

depreciation recapture net capital gain net capital loss<br />

GI OE D DR CG CL [17.16]


17.4 Depreciation Recapture and Capital Gains (Losses) 455<br />

In keeping with our perspective of an engineering economy study rather than that of a financial<br />

study, deprecation recapture (i.e., ordinary gain) will be the primary element considered in aftertax<br />

evaluations. Only when a capital gain or loss must be included due to the nature of the problem<br />

will the calculations involve it.<br />

EXAMPLE 17.5<br />

Biotech, a medical imaging and modeling company, must purchase a bone cell analysis system for<br />

use by a team of bioengineers and mechanical engineers studying bone density in athletes. This<br />

particular part of a 3-year contract with the NBA will provide additional gross income of $100,000<br />

per year. The effective tax rate is 35%. Estimates for two alternatives are summarized below.<br />

Analyzer 1 Analyzer 2<br />

Basis B, $ 150,000 225,000<br />

Operating expenses, $ per year 30,000 10,000<br />

MACRS recovery, years 5 5<br />

Answer the following questions, solving by hand and spreadsheet.<br />

(a) The Biotech president, who is very tax conscious, wishes to use a criterion of minimizing<br />

total taxes incurred over the 3 years of the contract. Which analyzer should be purchased?<br />

(b) Assume that 3 years have now passed, and the company is about to sell the analyzer. Using<br />

the same total tax criterion, did either analyzer have an advantage? Assume the selling<br />

price is $130,000 for analyzer 1, or $225,000 for analyzer 2.<br />

Solution by Hand<br />

(a) Table 17–4 details the tax computations. First, the yearly MACRS depreciation is determined.<br />

Equation [17.2], TI GI − OE − D, is used to calculate TI, after which the 35% tax<br />

rate is applied each year. Taxes for the 3-year period are summed, with no consideration of<br />

the time value of money.<br />

Analyzer 1 tax total: $36,120 Analyzer 2 tax total: $38,430<br />

The two analyzers are very close, but analyzer 1 wins with $2310 less in total taxes.<br />

(b) When the analyzer is sold after 3 years of service, there is a depreciation recapture (DR)<br />

that is taxed at the 35% rate. This tax is in addition to the third-year tax. For each analyzer,<br />

account for the DR by Equation [17.13], SP BV 3 ; then determine the TI, using<br />

Equa tion [17.16], TI GI − OE − D DR. Again, find the total taxes over 3 years, and<br />

select the analyzer with the smaller total.<br />

TABLE 17–4 Comparison of Total Taxes for Two Alternatives, Example 17.5a<br />

Year<br />

Gross<br />

Income<br />

GI, $<br />

Operating<br />

Expenses<br />

OE, $<br />

Basis<br />

B, $<br />

MACRS<br />

Depreciation<br />

D, $<br />

Analyzer 1<br />

Book<br />

Value<br />

BV, $<br />

Taxable<br />

Income<br />

TI, $<br />

Taxes<br />

at<br />

0.35TI, $<br />

0 150,000 150,000<br />

1 100,000 30,000 30,000 120,000 40,000 14,000<br />

2 100,000 30,000 48,000 72,000 22,000 7,700<br />

3 100,000 30,000 28,800 43,200 41,200 14,420<br />

36,120<br />

Analyzer 2<br />

0 225,000 225,000<br />

1 100,000 10,000 45,000 180,000 45,000 15,750<br />

2 100,000 10,000 72,000 108,000 18,000 6,300<br />

3 100,000 10,000 43,200 64,800 46,800 16,380<br />

38,430


456 Chapter 17 After-Tax Economic Analysis<br />

Analyzer 1: DR 130,000 43,200 $86,800<br />

Year 3 TI 100,000 30,000 28,800 86,800 $128,000<br />

Year 3 taxes (0.35)(128,000) $44,800<br />

Total taxes 14,000 7700 44,800 $66,500<br />

Analyzer 2: DR 225,000 64,800 $160,200<br />

Year 3 TI 100,000 10,000 43,200 160,200 $207,000<br />

Year 3 taxes (0.35)(207,000) $72,450<br />

Total taxes 15,750 6300 72,450 $94,500<br />

Now, analyzer 1 has a considerable advantage in total taxes ($94,500 versus $66,500).<br />

Solution by Spreadsheet<br />

(a) Rows 5 through 9 of Figure 17–4 perform the same computations as the hand solution for<br />

analyzer 1 with total taxes of $36,120. Similar analysis in rows 14 to 18 results in total<br />

taxes of $38,430 for analyzer 2, indicating that the company should select analyzer 1, based<br />

on taxes only.<br />

(b) Revised year 3 entries for analyzer 1 in row 10 show the sales price of $130,000, the updated<br />

TI of $128,000, and a 3-year tax total of $66,500. The new TI in year 3 has the depreciation<br />

recapture incorporated as DR selling price book value SP BV 3 , which<br />

is shown in the cell tag as the last term (D10 F10). With a similar updating for analyzer<br />

2 (row 19), total taxes of $94,500 now show a significantly larger tax advantage for analyzer<br />

1 over 3 years.<br />

Comment<br />

Note that no time value of money is considered in these analyses, as we have used in previous<br />

alternative evaluations. In Section 17.5 below we will rely upon PW, AW, and ROR analyses<br />

at an established MARR to make an after-tax decision based upon CFAT values.<br />

B6C6E6<br />

B10C10E10(D10F10)<br />

Last term calculates the<br />

depreciation recapture<br />

Total tax advantage<br />

of analyzer 1 over 2<br />

without sale and<br />

with sale in year 3<br />

Figure 17–4<br />

Impact of depreciation recapture on total taxes, Example 17.5.<br />

17.5 After-Tax Evaluation<br />

The required after-tax MARR is established using the market interest rate, the corporation’s effective<br />

tax rate, and its weighted average cost of capital. The CFAT estimates are used to compute<br />

the PW or AW at the after-tax MARR. When positive and negative CFAT values are present, a<br />

PW or AW 0 indicates the MARR is not met. For a single project or mutually exclusive alternatives,<br />

apply the same logic as in Chapters 5 and 6. The guidelines are:


17.5 After-Tax Evaluation 457<br />

One project. PW or AW 0, the project is financially viable because the after-tax MARR is<br />

met or exceeded.<br />

Two or more alternatives. Select the alternative with the best (numerically largest) PW or AW<br />

value.<br />

If only cost CFAT amounts are estimated, calculate the after-tax savings generated by the operating<br />

expenses and depreciation. Assign a plus sign to each saving and apply the guidelines above.<br />

Remember, the equal-service assumption requires that the PW analysis be performed over the<br />

least common multiple (LCM) of alternative lives. This requirement must be met for every<br />

analysis—before or after taxes.<br />

Since the CFAT estimates usually vary from year to year in an after-tax evaluation, the spreadsheet<br />

offers a much speedier analysis than solution by hand.<br />

For AW analysis: Use the PMT function with an embedded NPV function over one life cycle .<br />

The general format is as follows, with the NPV function in italics for the CFAT series.<br />

ME alternative selection<br />

Equal service<br />

PMT(MARR,n, NPV(MARR,year_1: year_n) year_0) [17.17]<br />

For PW analysis: Obtain the PMT function results first, followed by the PV function taken<br />

over the LCM. (There is an LCM function in Excel.) The cell containing the PMT function result<br />

is entered as the A value. The general format is<br />

EXAMPLE 17.6<br />

PV(MARR,LCM_years, PMT_result_cell ) [17.18]<br />

Paul is designing the interior walls of an industrial building. In some places, it is important<br />

to reduce noise transmission across the wall. Two construction options—stucco on metal<br />

lath (S) and bricks (B)—each have about the same transmission loss, approximately<br />

33 decibels. This will reduce noise attenuation costs in adjacent office areas. Paul has estimated<br />

the first costs and after-tax savings each year for both designs. (a) Use the CFAT<br />

values and an after-tax MARR of 7% per year to determine which is economically better.<br />

(b) Use a spreadsheet to select the alternative and determine the required first cost for the<br />

plans to break even.<br />

Plan S<br />

Plan B<br />

Year CFAT, $ Year CFAT, $<br />

0 28,800 0 50,000<br />

1–6 5,400 1 14,200<br />

7–10 2,040 2 13,300<br />

10 2,792 3 12,400<br />

4 11,500<br />

5 10,600<br />

Solution by Hand<br />

(a) In this example, both AW and PW analyses are shown. Develop the AW relations using<br />

the CFAT values over each plan’s life. Select the larger value.<br />

AW S [−28,800 5400(PA,7%,6) 2040(PA,7%,4)(PF,7%,6)<br />

2792(PF,7%,10)](AP,7%,10)<br />

$422<br />

AW B [−50,000 14,200(PF,7%,1) · · · 10,600(PF,7%,5)](AP,7%,5)<br />

$327<br />

Both plans are financially viable; select plan S because AW S is larger.<br />

For the PW analysis, the LCM is 10 years. Use the AW values and the PA factor for the<br />

LCM of 10 years to select stucco on metal lath, plan S.<br />

PW S AW S (PA,7%,10) 422(7.0236) $2964<br />

PW B AW B (PA,7%,10) 327(7.0236) $2297


458 Chapter 17 After-Tax Economic Analysis<br />

PMT(7%,5,NPV(7%,C4:C8)+C3)<br />

PV(7%,10,$C$14)<br />

(a)<br />

Figure 17–5<br />

(a) After-tax AW and PW analysis, and (b) breakeven first cost using Goal Seek, Example 17.6.<br />

Solution by Spreadsheet<br />

(b) Figure 17–5a, row 14, displays the AW value calculated using the PMT function defined by<br />

Equation [17.17], and row 15 shows the 10-year PW that results from the PV function in<br />

Equation [17.18]. Plan S is chosen by a relatively small margin.<br />

Figure 17–5b shows the Goal Seek template used to equate the AW values and determine<br />

the plan B first cost of $49,611 that causes the plans to break even. This is a small<br />

reduction from the $−50,000 first cost initially estimated.<br />

Comment<br />

It is important to remember the minus signs in PMT and PV functions when utilizing them to<br />

obtain the corresponding PW and AW values. If the minus is omitted, the AW and PW values<br />

have the wrong sign and it appears that the plans are not financially viable in that they do not<br />

return at least the after-tax MARR. That would happen in this example.<br />

(b)<br />

To utilize the ROR method, apply exactly the same procedures as in Chapter 7 (single project)<br />

and Chapter 8 (two or more alternatives) to the CFAT series. A PW or AW relation is developed<br />

to estimate the rate of return i * for a project, or i * for the incremental CFAT between two<br />

alternatives. Multiple roots may exist in the CFAT series, as they can for any cash flow series. For<br />

a single project, set the PW or AW equal to zero and solve for i *.<br />

Project evaluation<br />

Present worth:<br />

Annual worth:<br />

tn<br />

0 CFAT t (PF,i*,t) [17.19]<br />

t1<br />

tn<br />

0 CFAT t (PF,i*,t)(AP,i*,n) [17.20]<br />

t1<br />

If i * after-tax MARR, the project is economically justified.


17.5 After-Tax Evaluation 459<br />

Spreadsheet solution for i * is faster for most CFAT series. It is performed using the IRR function<br />

with the general format<br />

IRR(year_0_CFAT:year_n_CFAT) [17.21]<br />

If the after-tax ROR is important to the analysis, but the details of an after-tax study are not of<br />

interest, the before-tax ROR (or MARR) can be adjusted with the effective tax rate T e by using<br />

the approximating relation<br />

Before-tax ROR ———————<br />

after-tax ROR<br />

1 T e<br />

[17.22]<br />

For example, assume a company has an effective tax rate of 40% and normally uses an after-tax<br />

MARR of 12% per year for economic analyses that consider taxes explicitly. To approximate the<br />

effect of taxes without performing the details of an after-tax study, the before-tax MARR can be<br />

estimated as<br />

Before-tax MARR ———— 0.12<br />

20% per year<br />

1 0.40<br />

If the decision concerns the economic viability of a project and the resulting PW or AW value is<br />

close to zero, the details of an after-tax analysis should be developed.<br />

EXAMPLE 17.7<br />

A fiber optics manufacturing company operating in Hong Kong has spent $50,000 for a<br />

5- year-life machine that has a projected $20,000 annual NOI and annual depreciation of<br />

$10,000 for years 1 through 5. The company has a T e of 40%. (a) Determine the after-tax rate<br />

of return. (b) Approximate the before-tax return.<br />

Solution<br />

(a) The CFAT in year 0 is $50,000. For years 1 through 5 there is no capital purchase or sale,<br />

so NOI CFBT. (See Equations [17.1] and [17.9].) Determine CFAT.<br />

TI NOI D 20,000 10,000 $10,000<br />

Taxes T e (TI) 0.4(10,000) $4000<br />

CFAT CFBT taxes 20,000 4000 $16,000<br />

Since the CFAT for years 1 through 5 has the same value, use the PA factor in Equation<br />

[17.19].<br />

0 50,000 16,000(PA,i*,5)<br />

(PA,i*,5) 3.125<br />

Solution gives i* 18.03% as the after-tax rate of return.<br />

(b) Use Equation [17.22] for the before-tax return estimate.<br />

Before-tax ROR ———— 0.1803 0.3005 (30.05%)<br />

1 0.40<br />

The actual before-tax i* using CFBT $20,000 for 5 years is 28.65% from the relation<br />

0 50,000 20,000(PA,i*,5)<br />

The tax effect will be slightly overestimated if a MARR of 30.05% is used in a before-tax<br />

analysis.<br />

A rate of return evaluation performed by hand on two or more alternatives utilizes a PW or<br />

AW relation to determine the incremental return i * of the incremental CFAT series between<br />

two alternatives. Solution by spreadsheet is accomplished using the incremental CFAT values<br />

and the IRR function. The equations and procedures applied are the same as in Chapter 8<br />

(Sections 8.4 through 8.6) for selection from mutually exclusive alternatives using the ROR<br />

method. You should review and understand these sections before proceeding with this section.


460 Chapter 17 After-Tax Economic Analysis<br />

From this review, several important facts can be recalled:<br />

Selection guideline: The fundamental rule of incremental ROR evaluation at a stated MARR<br />

is as follows:<br />

Select the one alternative that requires the largest initial investment, provided the extra investment<br />

is justified relative to another justified alternative.<br />

Incremental ROR: Incremental analysis must be performed. Overall i * values cannot be<br />

depended upon to select the correct alternative, unlike the PW or AW method at the MARR,<br />

which will always indicate the correct alternative.<br />

Equal-service requirement: Incremental ROR analysis requires that the alternatives be evaluated<br />

over equal time periods. The LCM of the two alternative lives must be used to find the<br />

PW or AW of incremental cash flows. (The only exception, mentioned in Section 8.5, occurs<br />

when the AW analysis is performed on actual cash fl ows, not the increments; then onelife-cycle<br />

analysis is acceptable over the respective alternative lives.)<br />

Revenue and cost alternatives: Revenue alternatives (positive and negative cash flows) may<br />

be treated differently from cost alternatives (cost-only cash flow estimates). For revenue alternatives,<br />

the overall i * may be used to perform an initial screening. Alternatives with<br />

i * MARR can be removed from further evaluation. An i * for cost-only alternatives cannot<br />

be determined, so incremental analysis is required with all alternatives included.<br />

Breakeven ROR<br />

Once the CFAT series are developed, the breakeven ROR can be obtained using a plot of PW<br />

versus i * by solving the PW relation for each alternative over the LCM at several interest rates.<br />

For any after-tax MARR greater than the breakeven ROR, the extra investment is not justified.<br />

The next examples solve CFAT problems using incremental ROR analysis and the breakeven<br />

ROR plot of PW versus i .<br />

EXAMPLE 17.8<br />

In Example 17.6, Paul estimated the CFAT for interior wall materials to reduce sound transmission;<br />

plan S is to construct with stucco on metal lath, and plan B is to construct using brick.<br />

Figure 17–5a presented both a PW analysis over 10 years and an AW analysis over the respective<br />

lives. Plan S was selected. After reviewing this earlier solution, (a) perform an ROR<br />

evaluation at the after-tax MARR of 7% per year and (b) plot the PW versus i graph to determine<br />

the breakeven ROR.<br />

Solution by Spreadsheet<br />

(a) The LCM is 10 years for the incremental ROR analysis, and plan B requires the extra investment<br />

that must be justified. Apply the procedure in Section 8.6 for incremental ROR<br />

analysis. Figure 17–6 shows the estimated CFAT for each alternative and the incremental<br />

CFAT series. Since these are revenue alternatives, the overall i* is calculated first to ensure<br />

that they both make at least the MARR of 7%. Row 14 indicates they do. The IRR<br />

function (cell E14) is applied to the incremental CFAT, indicating that i* 6.35%. Since<br />

this is lower than the MARR, the extra investment in brick walls is not justified. Plan S is<br />

selected, the same as with the PW and AW methods.<br />

(b) The NPV function is used to find the PW of the incremental CFAT series at various i values.<br />

The graph indicates that the breakeven i* occurs at 6.35%—the same that the IRR function<br />

found. Whenever the after-tax MARR is above 6.35%, as is the case here with MARR <br />

7%, the extra investment in plan B is not justified.<br />

Comment<br />

Note that the incremental CFAT series has three sign changes. The cumulative series also has<br />

three sign changes (Norstrom’s criterion). Accordingly, there may be multiple i* values. The<br />

application of the IRR function using the “guess” option finds no other real number roots in the<br />

normal rate of return range.


17.5 After-Tax Evaluation 461<br />

Breakeven ROR<br />

6.35%<br />

Figure 17–6<br />

Incremental evaluation of CFAT and determination of breakeven ROR, Example 17.8.<br />

EXAMPLE 17.9<br />

In Example 17.5 an after-tax analysis of two bone cell analyzers was initiated due to a new<br />

3-year NBA contract. The criterion used to select analyzer 1 was the total taxes for the 3 years.<br />

The complete solution is in Table 17–4 (hand) and Figure 17–4 (spreadsheet).<br />

Continue the spreadsheet analysis by performing an after-tax ROR evaluation, assuming the<br />

analyzers are sold after 3 years for the amounts estimated in Example 17.5: $130,000 for analyzer<br />

1 and $225,000 for analyzer 2. The after-tax MARR is 10% per year.<br />

Solution<br />

A spreadsheet solution is presented here, but a hand solution is equivalent, just slower.<br />

Fig ure 17–7 is an updated version of the spreadsheet in Figure 17–4 to include the sale of the<br />

analyzers in year 3. The CFAT series (column I) are determined by the relation CFAT CFBT <br />

taxes, with the taxable income determined using Equation [17.16], where DR is included. For<br />

example, in year 3 when analyzer 2 is sold for S $225,000, the CFAT calculation is<br />

CFAT 3 CFBT (TI)(T e ) GI OE P S (GI OE D DR)(T e )<br />

The depreciation recapture DR is the amount above the year 3 book value received at sale time.<br />

Using the book value after 3 years (F14),<br />

DR selling price BV 3 225,000 64,800 $160,200<br />

Now the CFAT in year 3 for analyzer 2 can be determined.<br />

CFAT 3 100,000 10,000 0 225,000<br />

(100,000 10,000 43,200 160,200)(0.35)<br />

315,000 207,000(0.35) $242,550<br />

The cell tags in row 14 of Figure 17–7 follow this same progression. The incremental CFAT is<br />

calculated in column J, ready for the after-tax incremental ROR analysis.<br />

These are revenue alternatives, so the overall i* values indicate that both CFAT series are<br />

acceptable. The value i* 23.6% (cell J17) also exceeds MARR 10%, so analyzer 2 is<br />

selected. This decision applies the ROR method guideline: Select the alternative that requires<br />

the largest, incrementally justified investment.


462 Chapter 17 After-Tax Economic Analysis<br />

Overall i *<br />

B14C14E14(D14F14)<br />

This term<br />

calculates<br />

DR $160,200<br />

CFAT calculation<br />

B14C14D14H14<br />

Incremental i *<br />

IRR(J11:J14)<br />

Figure 17–7<br />

Incremental ROR analysis of CFAT with depreciation recapture, Example 17.9.<br />

Comment<br />

In Section 8.4, we demonstrated the fallacy of selecting an alternative based solely on the<br />

overall i*, because of the ranking inconsistency problem of the ROR method. The incremental<br />

ROR must be used. The same fact is demonstrated in this example. If the larger i* alternative<br />

is chosen, analyzer 1 is incorrectly selected. When i* exceeds the MARR, the larger investment<br />

is correctly chosen—analyzer 2 in this case. For verification, the PW at 10% is calculated<br />

for each analyzer (column I). Again, analyzer 2 is the winner, based on its larger PW of $93,905.<br />

EXAMPLE 17.10<br />

17.6 After-Tax Replacement Study<br />

When a currently installed asset (the defender) is challenged with possible replacement, the effect<br />

of taxes can have an impact upon the decision of the replacement study. The final decision<br />

may not be reversed by taxes, but the difference between before-tax AW values may be significantly<br />

different from the after-tax difference. Tax considerations in the year of the replacement<br />

are as follows:<br />

Depreciation recapture or tax savings due to a sizable capital loss are possible, if it is necessary<br />

to trade the defender at a sacrifice price. Additionally, the after-tax replacement study considers<br />

tax-deductible depreciation and operating expenses not accounted for in a before-tax analysis.<br />

The effective tax rate T e is used to estimate the amount of annual taxes (or tax savings) from TI.<br />

The same procedure as the before-tax replacement study in Chapter 11 is applied here, but for<br />

CFAT estimates. The procedure should be thoroughly understood before proceeding. Special attention<br />

to Sections 11.3 and 11.5 is recommended.<br />

Example 17.10 presents a solution by hand of an after-tax replacement study using a simplifying<br />

assumption of classical SL (straight line) depreciation. Example 17.11 solves the same problem<br />

by spreadsheet, but includes the detail of MACRS depreciation. This provides an opportunity<br />

to observe the difference in the AW values between the two depreciation methods.<br />

Midcontinent Power Authority purchased emission control equipment 3 years ago for $600,000.<br />

Management has discovered that it is technologically and legally outdated now. New equipment<br />

has been identified. If a market value of $400,000 is offered as the trade-in for the current<br />

equipment, perform a replacement study using (a) a before-tax MARR of 10% per year and


17.6 After-Tax Replacement Study 463<br />

(b) a 7% per year after-tax MARR. Assume an effective tax rate of 34%. As a simplifying assumption,<br />

use classical straight line depreciation with S 0 for both alternatives.<br />

Defender<br />

Challenger<br />

Market value, $ 400,000<br />

First cost, $<br />

1,000,000<br />

Annual cost, $year 100,000 15,000<br />

Recovery period, years 8 (originally) 5<br />

Solution<br />

Assume that an ESL (economic service life) analysis has determined the best life values to be<br />

5 more years for the defender and 5 years total for the challenger.<br />

(a) For the before-tax replacement study, find the AW values. The defender AW uses the market<br />

value as the first cost, P D $400,000.<br />

AW D 400,000(AP,10%,5) 100,000 $205,520<br />

AW C 1,000,000(AP,10%,5) 15,000 $278,800<br />

Applying step 1 of the replacement study procedure (Section 11.3), we select the better AW<br />

value. The defender is retained now with a plan to keep it for the 5 remaining years. The<br />

defender has a $73,280 lower equivalent annual cost compared to the challenger. This complete<br />

solution is included in Table 17–5 (left half) for comparison with the after-tax study.<br />

(b) For the after-tax replacement study, there are no tax effects other than income tax for the<br />

defender. The annual SL depreciation is $75,000, determined when the equipment was<br />

purchased 3 years ago.<br />

D t 600,0008 $75,000<br />

t 1 to 8 years<br />

Table 17–5 shows the TI and taxes at 34%. The taxes are actually tax savings of $59,500<br />

per year, as indicated by the minus sign. (Remember that for tax savings in an economic<br />

TABLE 17–5 Before-Tax and After-Tax Replacement Analyses, Example 17.10<br />

Defender<br />

Age<br />

Year<br />

Before Taxes<br />

Expenses<br />

OE, $ P and S, $ CFBT, $<br />

Defender<br />

Depreciation<br />

D, $<br />

Taxable<br />

Income<br />

TI, $<br />

After Taxes<br />

Taxes*<br />

at<br />

0.34TI, $ CFAT, $<br />

3 0 400,000 400,000 400,000<br />

4 1 100,000 100,000 75,000 175,000 59,500 40,500<br />

5 2 100,000 100,000 75,000 175,000 59,500 40,500<br />

6 3 100,000 100,000 75,000 175,000 59,500 40,500<br />

7 4 100,000 100,000 75,000 175,000 59,500 40,500<br />

8 5 100,000 0 100,000 75,000 175,000 59,500 40,500<br />

AW at 10% 205,520 AW at 7% 138,056<br />

Challenger<br />

0 1,000,000 1,000,000 25,000 † 8,500 1,008,500<br />

1 15,000 15,000 200,000 215,000 73,100 58,100<br />

2 15,000 15,000 200,000 215,000 73,100 58,100<br />

3 15,000 15,000 200,000 215,000 73,100 58,100<br />

4 15,000 15,000 200,000 −215,000 73,100 58,100<br />

5 15,000 0 15,000 200,000 215,000 ‡ 73,100 58,100<br />

AW at 10% 278,800 AW at 7% 187,863<br />

*<br />

Minus sign indicates a tax savings for the year.<br />

†<br />

Depreciation recapture on defender trade-in.<br />

‡<br />

Assumes challenger’s salvage actually realized is S 0; no tax.


464 Chapter 17 After-Tax Economic Analysis<br />

analysis it is assumed that there is positive taxable income elsewhere in the corporation to<br />

offset the saving.) Since only costs are estimated, the annual CFAT is negative, but the<br />

$59,500 tax savings has reduced it. The CFAT and AW at 7% per year are<br />

CFAT CFBT taxes 100,000 (59,500) $40,500<br />

AW D 400,000(AP,7%,5) 40,500 $138,056<br />

For the challenger, depreciation recapture on the defender occurs when it is replaced<br />

because the trade-in amount of $400,000 is larger than the current book value. In year<br />

0 for the challenger, Table 17–5 includes the following computations to arrive at a tax<br />

of $8500.<br />

Defender book value, year 3: BV 3 600,000 3(75,000) $375,000<br />

Depreciation recapture: DR 3 TI 400,000 375,000 $25,000<br />

Taxes on trade-in, year 0: Taxes 0.34(25,000) $8500<br />

The SL depreciation is $1,000,0005 $200,000 per year. This results in tax saving and<br />

CFAT as follows:<br />

Taxes (15,000 200,000)(0.34) $73,100<br />

CFAT CFBT taxes 15,000 (73,100) $58,100<br />

In year 5, it is assumed the challenger is sold for $0; there is no depreciation recapture. The<br />

AW for the challenger at the 7% after-tax MARR is<br />

AW C 1,008,500(AP,7%,5) 58,100 $187,863<br />

The defender is again selected; however, the equivalent annual advantage has decreased<br />

from $73,280 before taxes to $49,807 after taxes.<br />

Conclusion: By either analysis, retain the defender now and plan to keep it for 5 more<br />

years. Additionally, plan to evaluate the estimates for both alternatives 1 year hence. If and<br />

when cash flow estimates change significantly, perform another replacement analysis.<br />

Comment<br />

If the market value (trade-in) had been less than the current defender book value of $375,000, a<br />

capital loss, rather than depreciation recapture, would occur in year 0. The resulting tax savings<br />

would decrease the CFAT (which is to reduce costs if CFAT is negative) of the challenger. For<br />

example, a trade-in amount of $350,000 would result in a TI of $350,000 375,000 $25,000<br />

and a tax savings of $8500 in year 0. The CFAT is then $1,000,000 (8500) <br />

$991,500.<br />

EXAMPLE 17.11<br />

Repeat the after-tax replacement study of Example 17.10b using 7-year MACRS depreciation<br />

for the defender and 5-year MACRS depreciation for the challenger. Assume either<br />

asset is sold after 5 years for exactly its book value. Determine if the answers are significantly<br />

different from those obtained when the simplifying assumption of classical SL depreciation<br />

was made.<br />

Solution<br />

Figure 17–8 shows the complete analysis. MACRS requires more computation than SL depreciation,<br />

but this effort is easily reduced by the use of a spreadsheet. Again the defender is selected<br />

for retention, but now by an advantage of $44,142 annually. This compares to the<br />

$49,807 advantage using classical SL depreciation and the $73,280 before-tax advantage of the<br />

defender. Therefore, taxes and MACRS have reduced the defender’s economic advantage, but<br />

not enough to reverse the decision to retain it.<br />

Several other differences in the results between SL and MACRS depreciation are worth<br />

noting. There is depreciation recapture in year 0 of the challenger due to trade-in of the defender<br />

at $400,000, a value larger than the book value of the 3-year-old defender. This amount,


17.7 After-Tax Value-Added Analysis 465<br />

Depreciation recapture<br />

C5 F11<br />

Challenger sales price<br />

B15 F24<br />

Figure 17–8<br />

After-tax replacement study with MACRS depreciation and depreciation recapture, Example 17.11.<br />

$137,620 (cell G18), is treated as ordinary taxable income. The calculations for the DR and<br />

associated tax, by hand, are as follows:<br />

BV 3 first cost MACRS depreciation for 3 years<br />

total MACRS depreciation for years 4 through 8<br />

$262,380 (cell F11)<br />

DR TI 0 trade-in BV 3<br />

400,000 − 262,380 $137,620 (cell G18)<br />

Taxes (0.34)(137,620) $46,791 (cell H18)<br />

See the cell tags and table notes that duplicate this logic.<br />

The assumption that the challenger is sold after 5 years at its book value implies a positive<br />

cash flow in year 5. The entry $57,600 (C23) reflects this assumption, since the forgone<br />

MACRS depreciation in year 6 would be 1,000,000(0.0576) $57,600. The spreadsheet<br />

relation B15 F24 determines this value using the accumulated depreciation in F24.<br />

[Note: If the salvage S 0 is anticipated after 5 years, then a capital loss of $57,600 will be<br />

incurred. This implies an additional tax saving of 57,600(0.34) $19,584 in year 5. Conversely,<br />

if the salvage value exceeds the book value, a depreciation recapture and associated<br />

tax should be estimated.]<br />

17.7 After-Tax Value-Added Analysis<br />

When a person or company is willing to pay more for an item, it is likely that some processing<br />

has been performed on an earlier version of the item to make it more valuable now to the purchaser.<br />

This is value added.<br />

Value added is a term used to indicate that a product or service has added worth from the<br />

perspective of a consumer, owner, investor, or purchaser. It is common to leverage valueadding<br />

activities on a product or service.<br />

Value added


466 Chapter 17 After-Tax Economic Analysis<br />

For an example of highly leveraged value-added activities, consider onions that are grown and<br />

sold at the farm level for cents per pound. They may be purchased by the shopper in a store at<br />

50 cents to $1.25 per pound. But when onions are cut and coated with a special batter, they may<br />

be fried in hot oil and sold as onion rings for several dollars per pound. Thus, from the perspective<br />

of the consumer, there has been a large amount of value added by the processing from raw<br />

onions in the ground into onion rings sold at a restaurant or fast-food shop.<br />

The value-added measure was briefly introduced in conjunction with AW analysis before<br />

taxes. When value-added analysis is performed after taxes, the approach is somewhat different<br />

from that of CFAT analysis developed previously in this chapter. However, as shown below,<br />

The decision about an alternative will be the same for both the value-added and CFAT methods,<br />

because the AW of economic value-added estimates is the same as the AW of CFAT estimates.<br />

Value-added analysis starts with Equation [17.4], net operating profit after taxes (NOPAT), which<br />

includes the depreciation for year 1 through year n. Depreciation D is included in that TI GI <br />

OE − D . This is different from CFAT, where the depreciation has been specifically removed so that<br />

only actual cash flow estimates are used for year 0 through year n .<br />

The term economic value added (EVA) indicates the monetary worth added by an alternative<br />

to the corporation’s bottom line. (The term EVA is a trademark of Stern Stewart & Co.) The technique<br />

discussed below was first publicized in several articles 1 in the mid-1990s, and it has since<br />

become very popular as a means to evaluate the ability of a corporation to increase its economic<br />

worth, especially from the shareholders’ viewpoint.<br />

The annual EVA is the amount of NOPAT remaining on corporate books after removing the cost<br />

of invested capital during the year. That is, EVA indicates the project’s contribution to the net<br />

profit of the corporation after taxes.<br />

The cost of invested capital is the after-tax rate of return (usually the MARR value) multiplied<br />

by the book value of the asset during the year. This is the interest incurred by the current<br />

level of capital invested in the asset. (If different tax and book depreciation methods are used, the<br />

book depreciation value is used here, because it more closely represents the remaining capital<br />

invested in the asset from the corporation’s perspective.) Computationally,<br />

EVA NOPAT cost of invested capital<br />

NOPAT (after-tax interest rate)(book value in year t 1)<br />

TI(1 T e ) ( i )(BV t 1 ) [17.23]<br />

Since both TI and the book value consider depreciation, EVA is a measure of worth that mingles<br />

actual cash flow with noncash flows to determine the estimated financial worth contribution to<br />

the corporation. This financial worth is the amount used in public documents of the corporation<br />

(balance sheet, income statement, stock reports, etc.). Because corporations want to present the<br />

largest value possible to the stockholders and other owners, the EVA method is often more appealing<br />

than the AW method from the financial perspective.<br />

The result of an EVA analysis is a series of annual EVA estimates. Two or more alternatives<br />

are compared by calculating the AW of EVA estimates and selecting the alternative with the<br />

larger AW value. If only one project is evaluated, AW 0 means the after-tax MARR is exceeded,<br />

thus making the project value-adding.<br />

Sullivan and Needy 2 have demonstrated that the AW of EVA and the AW of CFAT are identical<br />

in amount. Thus, either method can be used to make a decision. The annual EVA estimates indicate<br />

added worth to the corporation generated by the alternative, while the annual CFAT estimates<br />

describe how cash will flow. This comparison is made in Example 17.12 .<br />

1 A. Blair, “EVA Fever,” Management Today, Jan. 1997, pp. 42–45; W. Freedman, “How Do You Add Up?”<br />

Chemical Week, Oct. 9, 1996, pp. 31–34.<br />

2 W. G. Sullivan and K. L. Needy, “Determination of Economic Value Added for a Proposed Investment in<br />

New Manufacturing.” The <strong>Engineering</strong> Economist, vol. 45, no. 2 (2000), pp. 166–181.


17.7 After-Tax Value-Added Analysis 467<br />

EXAMPLE 17.12<br />

Biotechnics <strong>Engineering</strong> has developed two mutually exclusive plans for investing in new<br />

capital equipment with the expectation of increased revenue from its medical diagnostic services<br />

to cancer patients. The estimates are summarized below. (a) Use classical straight line<br />

depreciation, an after-tax MARR of 12%, and an effective tax rate of 40% to perform two annual<br />

worth after-tax analyses: EVA and CFAT. (b) Explain the fundamental difference between<br />

the results of the two analyses.<br />

Plan A<br />

Plan B<br />

Initial investment, $ 500,000 1,200,000<br />

Gross income expenses, $ 170,000 per year 600,000 in year 1, decreasing by 100,000<br />

per year thereafter<br />

Estimated life, years 4 4<br />

Salvage value None None<br />

Solution by Spreadsheet<br />

(a) Refer to the spreadsheet and function cells (row 22) in Figure 17–9.<br />

EVA evaluation: All the necessary information for EVA estimation is determined in<br />

columns B through G. The net operating profit after taxes (NOPAT) in column H is calculated<br />

by Equation [17.4], TI taxes. The book values (column E) are used to determine the<br />

cost of invested capital in column I, using the second term in Equa tion [17.23], that is,<br />

i(BV t1 ), where i is the 12% after-tax MARR. This represents the amount of interest at<br />

12% per year, after taxes, for the currently invested capital as reflected by the book value<br />

at the beginning of the year. The EVA estimate is the sum of columns H and I for years 1<br />

through 4. Notice there is no EVA estimate for year 0, since NOPAT and the cost of invested<br />

capital are estimated for years 1 through n. Finally, the larger AW of the EVA value is selected,<br />

which indicates that plan B is better and that plan A does not make the 12% return.<br />

CFAT evaluation: As shown in function row 22 (plan B for year 3), CFAT estimates<br />

(column K) are calculated as (GI OE) P taxes. The AW of CFAT again concludes<br />

that plan B is better and that plan A does not return the after-tax MARR of 12% (K10).<br />

(b) What is the fundamental difference between the EVA and CFAT series in columns J and K?<br />

They are clearly equivalent from the time value of money perspective since the AW values<br />

are numerically the same. To answer the question, consider plan A, which has a constant<br />

CFAT estimate of $152,000 per year. To obtain the AW of EVA estimate of $12,617 for<br />

Figure 17–9<br />

Comparison of two plans using EVA and CFAT analyses, Example 17.12.


468 Chapter 17 After-Tax Economic Analysis<br />

years 1 through 4, the initial investment of $500,000 is distributed over the 4-year life using<br />

the AP factor at 12%. That is, an equivalent amount of $500,000(AP,12%,4) $164,617<br />

is “charged” against the cash inflows in each of years 1 through 4. In effect, the yearly<br />

CFAT is reduced by this charge.<br />

CFAT (initial investment)(AP,12%,4) 152,000 − 500,000(AP,12%,4)<br />

152,000 164,617 $12,617<br />

AW of EVA<br />

This is the AW value for both series, demonstrating that the two methods are economically<br />

equivalent. However, the EVA method indicates an alternative’s yearly estimated contribution<br />

to the value of the corporation, whereas the CFAT method estimates the actual cash<br />

flows to the corporation. This is why the EVA method is often more popular than the cash<br />

flow method with corporate executives.<br />

Comment<br />

The calculation P(AP,i,n) $500,000(AP,12%,4) is exactly the same as the capital recovery<br />

in Equation [6.3], assuming an estimated salvage value of zero. Thus, the cost of invested<br />

capital for EVA is the same as the capital recovery discussed in Chapter 6. This further demonstrates<br />

why the AW method is economically equivalent to the EVA evaluation.<br />

17.8 After-Tax Analysis for International Projects<br />

Primary questions to be answered prior to performing a corporate-based after-tax analysis for<br />

international settings revolve around tax-deductible allowances—depreciation, business expenses,<br />

capital asset evaluation—and the effective tax rate needed for Equation [17.7], taxes <br />

( T e )(TI). As discussed in Chapter 16, most governments of the world recognize and use the<br />

straight line (SL) and declining balance (DB) methods of depreciation with some variations to<br />

determine the annual tax-deductible allowance. Expense deductions vary widely from country to<br />

country. By way of example, some of these are summarized here.<br />

Canada<br />

Depreciation: This is deductible and is normally based on DB calculations, although SL may be<br />

used. An equivalent of the half-year convention is applied in the first year of ownership. The<br />

annual tax-deductible allowance is termed capital cost allowance (CCA). As in the U.S. system,<br />

recovery rates are standardized, so the depreciation amount does not necessarily reflect<br />

the useful life of an asset.<br />

Class and CCA rate: Asset classes are defined and annual depreciation rates are specified by<br />

class. No specific recovery period (life) is identified, in part because assets of a particular class<br />

are grouped together and the annual CCA is determined for the entire class, not individual assets.<br />

There are some 44 classes, and CCA rates vary from 4% per year (the equivalent of a<br />

25-year-life asset) for buildings (class 1) to 100% (1-year life) for applications software, chinaware,<br />

dies, etc. (class 12). Most rates are in the range of 10% to 30% per year.<br />

Expenses: Business expenses are deductible in calculating TI. Expenses related to capital investments<br />

are not deductible, since they are accommodated through the CCA.<br />

Internet: Further details are available on the Revenue Canada website at www.cra.gc.ca in the<br />

Forms and Publications section.<br />

China (PRC)<br />

Depreciation: Officially, SL is the primary method of tax-deductible depreciation; however, assets<br />

employed in selected industries or types of assets can utilize accelerated DB or SYD<br />

(sum-of-years-digits) depreciation, when approved by the government. The selected industries<br />

and assets can change over time; currently favored industries serve areas such as technology<br />

and oil exploration, and equipment subjected to large vibrations during normal usage is<br />

allowed accelerated depreciation.


17.8 After-Tax Analysis for International Projects 469<br />

Recovery period: Standardized recovery periods are published that vary from 3 years (electronic<br />

equipment) to 10 years (aircraft, machinery, and other production equipment) to 20 years (buildings).<br />

Shortened periods can be approved, but the minimum recovery period cannot be less<br />

than 60% for the normal period defined by current tax law.<br />

Expenses: Business expenses are deductible with some limitations and some special incentives.<br />

Limitations are placed, for example, on advertising expense deductions (15% of sales for the<br />

year). Incentives are generous in some cases; for example, 150% of actual expenses is deductible<br />

for new technology and new product R&D activities.<br />

Internet: Summary information for China and several other countries is available at<br />

www.worldwide-tax.com.<br />

Mexico<br />

Depreciation: This is a fully deductible allowance for calculating TI. The SL method is applied<br />

with an index for inflation considered each year. For some asset types, an immediate deduction<br />

of a percentage of the first cost is allowed. (This is a close equivalent to the Section 179<br />

Deduction in the United States.)<br />

Class and rates: Asset types are identified, though not as specifically defined as in some countries.<br />

Major classes are identified, and annual recovery rates vary from 5% for buildings (the<br />

equivalent of a 20-year life) to 100% for environmental machinery. Most rates range from<br />

10% to 30% per year.<br />

Profi t tax: The income tax is levied on profits on income earned from carrying on business in<br />

Mexico. Most business expenses are deductible. Corporate income is taxed only once, at the<br />

federal level; no state-level taxes are imposed.<br />

Tax on Net Assets (TNA): Under some conditions, a tax of 1.8% of the average value of assets<br />

located in Mexico is paid annually in addition to income taxes.<br />

Internet: The best information is via websites for companies that assist international corporations<br />

located in Mexico. One example is PriceWaterhouseCoopers at www.pwcglobal<br />

.com/mx/eng.<br />

Japan<br />

Depreciation: This is fully deductible and based on classical SL and DB methods. To encourage<br />

capital investment for long-term economic growth, in 2007 Japan allowed assets to be depreciated<br />

using a 250% DB method ; that is, a significantly increased accelerated rate is allowed<br />

compared to historical Japanese allowances. Switching to classical SL depreciation must take<br />

place in the year that the accelerated rate amount falls below the corresponding SL amount.<br />

Class and life: A statutory useful life ranging from 4 to 24 years, with a 50-year life for reinforced<br />

concrete buildings, is specified.<br />

Expenses: Business expenses are deductible in calculating TI.<br />

Internet: Further details are available on the Japanese Ministry of Finance website at<br />

www.mof.go.jp.<br />

The effective tax rate varies considerably among countries. Some countries levy taxes only<br />

at the federal level, while others impose taxes at several levels of government (federal, state or<br />

provincial, prefecture, county, and city). A summary of international corporate average tax rates<br />

is presented in Table 17–6 for a wide range of industrialized countries. These include income<br />

taxes at all reported levels of government within each country; however, other types of taxes<br />

may be imposed by a particular government. Although these average rates of taxation will vary<br />

from year to year, especially as tax reform is enacted, it can be surmised that most corporations<br />

face effective rates of about 20% to 40% of taxable income. A close examination of international<br />

rates shows that they have decreased significantly over the last decade. In fact, the KPMG<br />

report noted in Table 17–6 indicates that the global average corporate tax rate on TI has decreased<br />

from 32.7% (1999) to 25.5% (2009). This has encouraged corporate investment and<br />

business expansion within country borders and helped soften the massive economic downturn<br />

experienced in recent years.


470 Chapter 17 After-Tax Economic Analysis<br />

TABLE 17–6<br />

Summary of International Corporate Average Tax Rates<br />

Tax Rate Levied on<br />

Taxable Income, %<br />

For These Countries<br />

40<br />

United States, Japan<br />

35 to 40 South Africa, Pakistan<br />

32 to 35 Canada, France, India<br />

28 to 32 Australia, United Kingdom, New Zealand, Spain, Germany,<br />

Mexico, Indonesia<br />

24 to 28 China, Taiwan, Republic of Korea<br />

20 to 24 Russia, Turkey, Saudi Arabia<br />

20<br />

Singapore, Hong Kong, Chile, Ireland, Iceland, Hungary<br />

Sources: Extracted from KPMG’s Corporate and Indirect Tax Survey 2009 (www.kpmg.<br />

comGlobalenIssuesAndInsightsPagesdefault.aspx) and from country websites on corporate taxation.<br />

One of the prime ways that governments have been able to reduce corporate tax rates is by<br />

shifting to indirect taxes on goods and services for additional tax revenue. These taxes are usually<br />

in the form of a value-added tax (VAT), goods and services tax (GST), and taxes on products<br />

imported from outside its borders. As corporate tax rates have declined, in general the indirect<br />

tax rates have increased. This has been especially true during the first decade of the 21st<br />

century. However, the worldwide economic slump experienced in recent years has made it necessary<br />

for governments around the world to be more cautious of how they tax corporations and<br />

maintain a reasonable balance between regular tax rates (as listed in Table 17–6 ) and indirect tax<br />

rates. The VAT system is explained now.<br />

17.9 Value-Added Tax<br />

A value-added tax (VAT) has facetiously been called a sales tax on steroids, because the VAT<br />

rates on some items in some countries that impose a VAT can be as high as 90%. It is also called<br />

a GST (goods and service tax).<br />

A value-added tax is an indirect tax; that is, it is a tax on goods and services rather than on<br />

people or corporations. It differs from a sales tax in two ways: (1) when it is charged and (2) who<br />

pays (as explained below). A specific percentage, say 10%, is a charge added to the price of the<br />

item and paid by the buyer. The seller then sends this 10% VAT to the taxing entity, usually a<br />

government unit. This process of 10% VAT continues every time the item is resold—as purchased<br />

or in a modified form—thus the term value-added .<br />

A value-added tax is commonly used throughout the world. In some countries, VAT is used in<br />

lieu of business or individual income taxes. There is no VAT system in the United States yet. In<br />

fact, the United States is the only major industrialized country in the world that does not have a<br />

VAT system, though other forms of indirect taxation are used liberally. There is mounting evidence,<br />

however, that a VATGST system will be necessary in the United States in the near future,<br />

but not without a great amount of political discord.<br />

A sales tax is used by the U.S. government, by nearly all of the states, and by many local entities.<br />

A sales tax is charged on goods and services at the time the goods and services reach the end<br />

user or consumer . That is, businesses do not pay a sales tax on raw material, unfinished goods,<br />

or items they purchase that will ultimately be sold to an end user; only the end user pays the sales<br />

tax. Businesses do pay a sales tax on items for which they are the end user. Total sales tax percentages<br />

imposed by multiple government levels can range from 5% to 11%, sometimes larger<br />

on specific items. For example, when Home Depot (HD) purchases microwave ovens from Jenn-<br />

Air, Inc., Home Depot does not pay a sales tax on the microwave ovens because they will be sold<br />

to HD customers who will pay the sales tax. On the other hand, if Home Depot purchases a forklift<br />

from Caterpillar for loading and unloading merchandise in one of its stores, HD will pay the<br />

sales tax on the forklift, since HD is the end user. Thus, a sales tax is paid only one time , and that


17.9 Value-Added Tax 471<br />

time is when the goods or services are purchased by the end user . The sales tax is the responsibility<br />

of the merchant to collect and remand to the taxing entity.<br />

A value-added tax (VAT) , on the other hand, is charged to the buyer at purchase time, whether<br />

the buyer is a business or end user. The seller sends the collected VAT to the taxing entity. If the<br />

buyer subsequently resells the goods to another buyer (as is or a modification of it), another VAT<br />

is collected by the seller. Now, this second seller will send to the taxing entity an amount that is<br />

equal to the total tax collected minus the amount of VAT already paid.<br />

As an illustration, assume the U.S. government charged a 10% VAT. Here is how the VAT<br />

might work.<br />

Northshore Mining Corporation of Babbitt, Minnesota, sells $100,000 worth of iron ore to<br />

Westfall Steel. As part of the price, Northshore collects $110,000, that is, $100,000 0.1 <br />

100,000, from Westfall Steel. Northshore remits the $10,000 VAT to the U.S. Treasury.<br />

Westfall Steel sells all of the steel it made from the iron ore at a price of $300,000 to General<br />

Electric (GE). Westfall collects $330,000 from GE and then sends $20,000 to the U.S. Treasury,<br />

that is, $30,000 it collected in VAT from GE minus $10,000 it paid in taxes to Northshore<br />

Mining.<br />

GE uses the steel to make refrigerators that it sells for $700,000 to retailers, such as Home<br />

Depot, Lowe’s, and others. GE collects $770,000 and then remits $40,000 to the U.S. Treasury,<br />

that is, $70,000 it collected in taxes from retailers minus $30,000 it paid in VAT to<br />

Westfall Steel.<br />

If GE purchased machines, tools, or other items to make the refrigerators during this accounting<br />

period, and it paid taxes on those items, the taxes paid would also be deducted from the<br />

VAT that GE collected before sending the money to the U.S. Treasury. For example, if GE<br />

paid $5000 in taxes on motors it purchased for the refrigerators, the amount GE would remit<br />

to the U.S. Treasury would be $35,000 (that is, $70,000 it collected from retailers minus<br />

$30,000 it paid in taxes to Westfall Steel minus $5000 it paid in taxes on the motors).<br />

The retailers sell the refrigerators for $950,000 and collect $95,000 in taxes from end users—<br />

consumers. The retailers remit $25,000 to the U.S. Treasury (that is, $95,000 they collected<br />

minus $70,000 they paid previously).<br />

Through this process, the U.S Treasury has received $10,000 from Northshore, $20,000<br />

from Westfall Steel, $35,000 from GE, $5000 from the supplier of the motors, and $25,000<br />

from the retailers, for a total of $95,000. This is 10% of the final sales price of $950,000. The<br />

VAT money was deposited into the Treasury at several different times from several different<br />

companies.<br />

The taxes that a company pays for materials or items it purchases in order to produce goods<br />

or services that will subsequently be sold to another business or end user are called input taxes ,<br />

and the company is able to recover them when it collects the VAT from the sale of its products.<br />

The taxes that a company collects are called output taxes , and these are forwarded to the taxing<br />

entity, less the amount of input taxes the company paid. Hence, the businesses incur no taxes<br />

themselves, the same as with a sales tax.<br />

Several dimensions of a VAT distinguish it from a sales tax or corporate income taxes. Some<br />

are as follow:<br />

• Value-added taxes are taxes on consumption, not production or taxable income.<br />

• The end user pays all of the value-added taxes, but VATs are not as obvious as a sales tax that<br />

is added to the price of the item at the time of purchase (and displayed on the receipt). Therefore,<br />

VAT taxing entities encounter less resistance from consumers.<br />

• Value-added taxes are generally considerably higher than sales taxes, with the average<br />

European VAT rate at 20% and the worldwide average at 15.25%.<br />

• The VAT is essentially a “sales tax,” but it is charged at each stage of the product development<br />

process instead of when the product is sold.<br />

• With VAT, there is less evasion of taxes because it is harder for multiple entities to evade collecting<br />

and paying the taxes than it is for one entity to do so.<br />

• VAT rates vary from country to country and from category to category. For example, in some<br />

countries, food has a 0% VAT rate, while aviation fuel is taxed at 32%.


472 Chapter 17 After-Tax Economic Analysis<br />

EXAMPLE 17.13<br />

Tata Motors is a major player in automobile manufacturing in India. It has three different<br />

manufacturing units that specialize in manufacturing different transportation-related products,<br />

such as trucks, engines and axles, commercial vehicles, utility vehicles, and passenger<br />

cars. The company buys products that fall under different sections of the Indian government<br />

VAT tax code, and therefore the products have different VAT rates. In one<br />

particular accounting period, Tata had invoices from four different suppliers (vendors A,<br />

B, C, and D) in the respective amounts of $1.5 million, $3.8 million, $1.1 million, and<br />

$900,000. The products Tata purchased were subject to VAT rates of 4%, 4%, 12.5%, and<br />

22%, respectively.<br />

(a) How much total VAT did Tata pay to its vendors?<br />

(b) Assume that Tata’s products have a VAT rate of 12.5%. If Tata’s sales during the period<br />

were $9.2 million, how much VAT did the Indian Treasury receive from Tata?<br />

Solution<br />

(a) Let X equal the product price before the VAT is added. Solve for X and then subtract it<br />

from the purchase amount to determine the VAT charged by each vendor. Table 17–7<br />

shows the VAT that Tata paid its four vendors. An example computation for vendor A is<br />

as follows:<br />

X 0.04X 1,500,000<br />

1.04X 1,500,000<br />

X $1,442,308<br />

VAT A 1,500,000 1,442,308<br />

$57,692<br />

Total VAT paid 57,692 146,154 122,222 162,295<br />

$488,363<br />

(b) Total from Tata total VAT VAT paid by vendors<br />

9,200,000(0.125) 488,363<br />

$661,637<br />

TABLE 17–7 VAT Computation, Example 17.13<br />

Vendor Purchases, $ VAT Rate, % Price before VAT, X, $ VAT, $<br />

A 1,500,000 4.0 1,442,308 57,692<br />

B 3,800,000 4.0 3,653,846 146,154<br />

C 1,100,000 12.5 977,778 122,222<br />

D 900,000 22.0 737,705 162,295<br />

Total 488,363<br />

CHAPTER SUMMARY<br />

After-tax analysis does not usually change the decision to select one alternative over another;<br />

however, it does offer a much clearer estimate of the monetary impact of taxes. After-tax PW,<br />

AW, and ROR evaluations of one or more alternatives are performed on the CFAT series using<br />

exactly the same procedures as in previous chapters.<br />

Income tax rates for U.S. corporations and individual taxpayers are graduated or progressive—<br />

higher taxable incomes pay higher income taxes. A single-value, effective tax rate T e is usually<br />

applied in an after-tax economic analysis. Taxes are reduced because of tax deductible items,<br />

such as depreciation and operating expenses. Because depreciation is a noncash flow, it is important<br />

to consider depreciation only in TI computations, and not directly in the CFBT and


CFAT calculations. Accordingly, key general cash flow after-tax relations for each year are as<br />

follows:<br />

NOI gross income expenses<br />

TI gross income operating expenses depreciation depreciation recapture<br />

CFBT gross income operating expenses initial investment salvage value<br />

CFAT CFBT taxes CFBT ( T e )(TI)<br />

If an alternative’s estimated contribution to corporate financial worth is the economic measure,<br />

the economic value added (EVA) should be determined. Unlike CFAT, the EVA includes the effect<br />

of depreciation. The equivalent annual worths of CFAT and EVA estimates are the same<br />

numerically, because they interpret the annual cost of the capital investment in different, but<br />

equivalent manners when the time value of money is taken into account.<br />

In a replacement study, the tax impact of depreciation recapture, which may occur when the<br />

defender is traded for the challenger, is accounted for in an after-tax analysis. The replacement<br />

study procedure of Chapter 11 is applied. The tax analysis may not reverse the decision to replace<br />

or retain the defender, but the effect of taxes will likely reduce (possibly by a significant amount)<br />

the economic advantage of one alternative over the other.<br />

International corporate tax rates have steadily decreased, but indirect taxes, such as valueadded<br />

tax (VAT), have increased. The mechanism of a VAT is explained and compared to a sales<br />

tax. The United States currently has no VAT.<br />

Problems 473<br />

PROBLEMS<br />

Basic Tax Computations<br />

17.1 (a) Define the following tax terms: graduated tax<br />

rates , marginal tax rate , and indexing .<br />

( b) Describe a fundamental difference between<br />

each of the following terms: net operating income<br />

(NOI), taxable income (TI), and net<br />

operating profit after taxes (NOPAT).<br />

17.2 Determine the average tax rate for a corporation<br />

that has taxable income of ( a ) $150,000 and<br />

( b ) $12,000,000.<br />

17.3 Determine the single-value effective tax rate for a<br />

corporation that has a federal tax rate of 35% and a<br />

state tax rate of 5%.<br />

17.4 Identify the primary term described by each event<br />

below: gross income , depreciation , operating expense<br />

, taxable income , income tax , or net operating<br />

profit after taxes .<br />

( a) A new machine had a first-year write-off of<br />

$10,500.<br />

( b) A public corporation estimates that it will report<br />

a $−750,000 net profit on its annual income<br />

statement.<br />

( c) An asset with a book value of $8000 was retired<br />

and sold for $8450.<br />

( d) An over-the-counter software system will<br />

generate $420,000 in revenue this quarter.<br />

( e) An asset with a MACRS recovery period of 7<br />

years has been owned for 10 years. It was<br />

just sold for $2750.<br />

(f ) The cost of goods sold in the past year was<br />

$3,680,200.<br />

( g) A convenience store collected $33,550 in lottery<br />

ticket sales last month. Based on winners<br />

holding these tickets, a rebate of $350<br />

was sent to the manager.<br />

( h) An asset with a first cost of $65,000 was utilized<br />

on a new product line to increase sales<br />

by $150,000.<br />

( i) The cost to maintain equipment during the<br />

past year was $641,000.<br />

17.5 Two companies have the following values on their<br />

annual tax returns.<br />

Company 1 Company 2<br />

Sales revenue, $ 1,500,000 820,000<br />

Interest revenue, $ 31,000 25,000<br />

Expenses, $ 754,000 591,000<br />

Depreciation, $ 48,000 18,000<br />

(a) Calculate the federal income tax for the year<br />

for each company.<br />

(b) Determine the percent of sales revenue each<br />

company will pay in federal income tax.<br />

(c) Estimate the taxes using an effective rate of<br />

34% of the entire TI. Determine the percentage<br />

error made relative to the exact taxes in<br />

part ( a ).<br />

17.6 Last year, one division of Hagauer.com, a dot-com<br />

sports industry service firm that provides real-time


474 Chapter 17 After-Tax Economic Analysis<br />

analysis of mechanical stress due to athlete injury,<br />

had $250,000 in taxable income. This year, TI is<br />

estimated to be $600,000. Calculate the federal income<br />

taxes and answer the following.<br />

(a) What was the average federal tax rate paid<br />

last year?<br />

(b) What is the marginal federal tax rate on the<br />

additional TI this year?<br />

(c)<br />

What will be the average federal tax rate this<br />

year?<br />

(d) What will be the NOPAT on just the additional<br />

$350,000 in taxable income?<br />

17.7 Yamachi and Nadler of Hawaii has a gross income<br />

of $7.5 million for the year. Depreciation and operating<br />

expenses total $4.3 million. The combined<br />

state and local tax rate is 7.2%. If an effective federal<br />

rate of 35% applies, estimate the income taxes<br />

using the effective tax rate equation.<br />

17.8 Workman Tools reported a TI of $90,000 last year.<br />

If the state income tax rate is 7%, determine the<br />

( a ) average federal tax rate, ( b ) overall effective<br />

tax rate, ( c ) total taxes to be paid based on the effective<br />

tax rate, and ( d ) total taxes paid to the state<br />

and paid to the federal government.<br />

17.9 The taxable income for a motorcycle sales and repair<br />

business is estimated to be $150,000 this year.<br />

A single-value tax rate of 39% is used. A new engine<br />

diagnostics system will cost $40,000 and<br />

have an average annual depreciation of $8000. The<br />

equipment will increase gross income by an estimated<br />

$9000 and expenses by $2000 for the year.<br />

Compute the expected change in income taxes for<br />

the year if the new system is purchased.<br />

17.10 C.F. Jordon Construction Services has operated for<br />

the last 26 years in a northern U.S. state where the<br />

state income tax on corporate revenue is 6% per<br />

year. C.F. Jordon pays an average federal tax of<br />

23% and reports taxable income of $7 million. Because<br />

of pressing labor cost increases, the president<br />

wants to move to another state to reduce the<br />

total tax burden. The new state may have to be<br />

willing to offer tax allowances or an interest-free<br />

grant for the first couple of years in order to attract<br />

the company. You are an engineer with the company<br />

and are asked to do the following.<br />

(a) Determine the effective tax rate for C.F.<br />

Jordon.<br />

(b) Estimate the state tax rate that would be necessary<br />

to reduce the overall effective tax rate<br />

by 10% per year.<br />

(c) Determine what the new state would have to<br />

do financially for C.F. Jordon to move there<br />

and to reduce its effective tax rate to 22% per<br />

year.<br />

CFBT and CFAT<br />

17.11 Identify which of the following items are included<br />

in the calculation of cash flow before taxes<br />

(CFBT): operating expenses, salvage value, depreciation,<br />

initial investment, gross income, tax rate.<br />

17.12 What is the basic difference between cash flow<br />

after taxes (CFAT) and net operating profit after<br />

taxes (NOPAT)?<br />

17.13 For a year in which there is no initial investment P<br />

or salvage value S , derive an equation for CFBT<br />

that contains only the following terms: CFAT,<br />

CFBT, D , and T e .<br />

17.14 Determine the cash flow before taxes for Anderson<br />

Consultants when the cash flow after taxes was<br />

$600,000, asset depreciation was $350,000 and the<br />

company’s effective tax rate was 36%.<br />

17.15 Four years ago Sierra Instruments of Monterey,<br />

California spent $200,000 for equipment to manufacture<br />

standard gas flow calibrators. The equipment<br />

was depreciated by MACRS using a 3-year<br />

recovery period. The gross income for year 4 was<br />

$100,000, with operating expenses of $50,000.<br />

Use an effective tax rate of 40% to determine the<br />

CFAT in year 4 if the asset was ( a ) discarded with<br />

no salvage value in year 4 and ( b ) sold for $20,000<br />

at the end of year 4 (neglect any taxes that may be<br />

incurred on the sale of the equipment). The<br />

MACRS depreciation rate for year 4 is 7.41%.<br />

17.16 Four years ago a division of Harcourt-Banks purchased<br />

an asset that was depreciated by the MACRS<br />

method using a 3-year recovery period. The total<br />

revenue for year 2 was $48 million, depreciation<br />

was $8.2 million, and operating expenses were<br />

$28 million. Use a federal tax rate of 35% and a<br />

state tax rate of 6.5% to determine ( a ) CFAT, ( b )<br />

percentage of total revenue expended on taxes, and<br />

( c ) net profit after taxes for the year.<br />

17.17 Advanced Anatomists, Inc., researchers in medical<br />

science, is contemplating a commercial venture<br />

concentrating on proteins based on the new<br />

X-ray technology of free- electron lasers. To recover<br />

the huge investment needed, an annual<br />

$2.5 million CFAT is needed. A favored average<br />

federal tax rate of 20% is expected; however, state<br />

taxing authorities will levy an 8% tax on TI. Over<br />

a 3-year period, the deductible expenses and depreciation<br />

are estimated to total $1.3 million the<br />

first year, increasing by $500,000 per year thereafter.<br />

Of this, 50% is expenses and 50% is depreciation.<br />

What is the required gross income each<br />

year?


Problems 475<br />

The following information is used in Problems 17.18<br />

through 17.21. (Show hand and spreadsheet solutions, as<br />

instructed)<br />

After 4 years of use, Procter and Gamble has decided to<br />

replace capital equipment used on its Zest bath soap line.<br />

The equipment was MACRS-depreciated over a 3-year<br />

recovery period. After-tax MARR is 10% per year, and T e<br />

is 35% in the United States. The cash flow data is tabulated<br />

in $1000 units.<br />

Year<br />

0 1 2 3 4<br />

Purchase, $<br />

1900<br />

Gross income, $ 800 950 600 300<br />

Operating expenses, $ 100 150 200 250<br />

Salvage, $ 700<br />

17.18 Utilize the CFBT series and AW value to determine<br />

whether the equipment investment exceeded<br />

the MARR.<br />

17.19 Calculate MACRS depreciation and estimate the<br />

CFAT series over 4 years. Neglect any tax impact<br />

caused by the $700,000 salvage received in year 4.<br />

17.20 Utilize the CFAT series and AW value to determine<br />

whether the investment exceeded the MARR.<br />

17.21 Compare the after-tax ROR values using both<br />

methods—CFAT series and approximation from<br />

the CFBT values using the before-tax ROR and T e .<br />

17.22 A Wal-Mart Distribution Center has put into service<br />

forklifts and conveyors purchased for $250,000.<br />

Use a spreadsheet to tabulate CFBT, CFAT, NOPAT,<br />

and i * before and after taxes for 6 years of ownership.<br />

The effective tax rate is 40%, and the estimated<br />

cash flow and depreciation amounts are<br />

shown. Salvage is expected to be zero.<br />

Year<br />

Gross<br />

Income, $<br />

Operating<br />

Expenses, $<br />

MACRS<br />

Depreciation, $<br />

1 210,000 120,000 50,000<br />

2 210,000 120,000 80,000<br />

3 160,000 122,000 48,000<br />

4 160,000 124,000 28,800<br />

5 160,000 126,000 28,800<br />

6 140,000 128,000 14,400<br />

Taxes and Depreciation<br />

17.23 An asset purchased by Stratasys, Inc. had a first<br />

cost of $70,000 with an expected salvage value of<br />

$10,000 at the end of its 5-year life. In year 2, the<br />

revenue was $490,000 with operating expenses of<br />

$140,000. If the company’s effective tax rate was<br />

36%, determine the difference in taxes paid in<br />

year 2 if the depreciation method had been straight<br />

line instead of MACRS.<br />

17.24 Cheryl, an electrical engineering student who is<br />

working on a business minor, is studying depreciation<br />

and finance in her engineering management<br />

course. The assignment is to demonstrate that<br />

shorter recovery periods require the same total<br />

taxes, but they offer a time value of taxes advantage<br />

for depreciable assets. Help her, using asset<br />

estimates made for a 6-year study period: P <br />

$65,000, S $5000, GI $32,000 per year, AOC<br />

is $10,000 per year, SL depreciation, i 12% per<br />

year, T e 31%. Make the comparison using recovery<br />

periods of 3 and 6 years.<br />

17.25 Complete the last four columns of the table below<br />

using an effective tax rate of 40% for an asset that<br />

has a first cost of $20,000 and a 3-year recovery<br />

period with no salvage value, using ( a ) straight<br />

line depreciation and ( b ) MACRS depreciation.<br />

All cash flows are in $1000 units.<br />

Estimates, $<br />

Year GI P OE D TI Taxes CFAT<br />

0 — 20 — — — — 20<br />

1 8 2<br />

2 15 4<br />

3 12 3<br />

4 10 5<br />

17.26 Use an effective tax rate of 32% to determine the<br />

CFAT and NOPAT associated with the asset shown<br />

below under two different scenarios: ( a ) with depreciation<br />

at $6000 per year and ( b ) with depreciation<br />

at $6000, $9600, $5760, and $3456 in years 1<br />

through 4, respectively. All monetary amounts are<br />

in $1000.<br />

Estimates, $<br />

Year GI OE P D TI Taxes CFAT NOPAT<br />

0 — — 30 — — — 30 —<br />

1 8 2<br />

2 15 4<br />

3 12 3<br />

4 10 5<br />

17.27 J. B. Hunt, Inc., an overland freight company, has<br />

purchased new trailers for $150,000 and expects to<br />

realize a net $80,000 in gross income over operating<br />

expenses for each of the next 3 years. The trailers<br />

have a recovery period of 3 years. Assume an<br />

effective tax rate of 35% and an interest rate of<br />

15% per year.<br />

(a) Show the advantage of accelerated depreciation<br />

by calculating the present worth of taxes<br />

for the MACRS method versus the classical


476 Chapter 17 After-Tax Economic Analysis<br />

SL method. Since MACRS takes an additional<br />

year to fully depreciate the basis, assume<br />

no CFBT beyond year 3, but include<br />

any negative tax as a tax savings.<br />

(b) Show that the total taxes are the same for<br />

both methods.<br />

17.28 A bioengineer is evaluating methods used to<br />

apply adhesive onto microporous paper tape that<br />

is commonly used after surgery. The machinery<br />

costs $200,000, has no salvage value, and the<br />

CFBT estimate is $75,000 per year for up to 10<br />

years. The T e 38% and i 8% per year. The<br />

two depreciation methods to consider are:<br />

MACRS with n 5 years and SL with n <br />

8 years (neglect the half-year convention effect).<br />

For a study period of 8 years, ( a ) determine<br />

which depreciation method offers the better tax<br />

advantage, and ( b ) demonstrate that the same<br />

total taxes are paid for MACRS and SL.<br />

17.29 An asset with a first cost of $9000 is depreciated<br />

by MACRS over a 5-year recovery period. The<br />

CFBT is estimated at $10,000 for the first 4 years<br />

and $5000 thereafter as long as the asset is retained.<br />

The effective tax rate is 40%, and money is<br />

worth 10% per year. In present worth dollars, how<br />

much of the cash flow generated by the asset over<br />

its recovery period is lost to taxes?<br />

Depreciation Recapture and Capital Gains (Losses)<br />

17.30 Last month, a company specializing in wind power<br />

plant design and construction made a capital investment<br />

of $400,000 in physical simulation<br />

equipment that will be used for at least 5 years,<br />

after which it is expected to be sold for approximately<br />

25% of its first cost. According to tax law,<br />

the simulation is MACRS-depreciated using a<br />

3-year recovery period.<br />

(a) Explain why there is a predictable tax implication<br />

when the simulator is sold.<br />

(b) Determine by how much the sale will cause<br />

TI and taxes to change in year 5 if T e 35%.<br />

The following information is used in Problems 17.31<br />

through 17.34.<br />

Open Access, Inc. is an international provider of computer<br />

network communications gear. Different depreciation,<br />

recovery period, and tax law practices in the three<br />

countries where depreciable assets are located are summarized<br />

in the table. Also, information is provided about<br />

assets purchased 5 years ago at each location and sold this<br />

year. After-tax MARR 9% per year and T e 30% can<br />

be used for all countries.<br />

Practice or Estimate Country 1 Country 2 Country 3<br />

Depreciation<br />

method<br />

SL with<br />

n 5<br />

MACRS<br />

with n 3<br />

DDB with<br />

n 5<br />

Depreciation recapture Not taxed Taxed as TI Taxed as TI<br />

First cost, $ 100,000 100,000 100,000<br />

GI OE, $ per year 25,000 25,000 25,000<br />

Estimated<br />

salvage, $<br />

0 in year 5 0 in year 5 20,000 in<br />

year 5<br />

Life, years 5 5 5<br />

Actual selling<br />

price, $<br />

20,000 in<br />

year 5<br />

20,000 in<br />

year 5<br />

20,000 in<br />

year 5<br />

17.31 For country 1, SL depreciation is $20,000 per year.<br />

Determine the ( a ) CFAT series and ( b ) PW of depreciation,<br />

taxes, and CFAT series.<br />

17.32 For country 2, MACRS depreciation for 4 years is<br />

$33,333, $44,444, $14,815, and $7407, respectively.<br />

Determine the ( a ) CFAT series and ( b ) PW<br />

of depreciation, taxes, and CFAT series.<br />

17.33 For country 3, DDB depreciation for 5 years is<br />

$40,000, $24,000, $14,400, $1600, and 0, respectively.<br />

Determine the ( a ) CFAT series and ( b ) PW<br />

of depreciation, taxes, and CFAT series.<br />

17.34 If you worked Problems 17.31 through 17.33, develop<br />

a table that summarizes, for each country,<br />

the total taxes paid and the PW values of the depreciation,<br />

taxes, and CFAT series. For each criterion,<br />

select the country that provides the best PW<br />

value. Explain why the same country is not selected<br />

for all three criteria. ( Hint : The PW should<br />

be minimized for some criteria and maximized for<br />

other criteria. Review previous material first to be<br />

sure you choose correctly.)<br />

17.35 An asset with a first cost of $350,000 three years<br />

ago is sold for $385,000. The asset was depreciated<br />

by the MACRS method and has a book value<br />

of $100,800 at the time of sale. Determine the capital<br />

gain and depreciation recapture, if any.<br />

17.36 Sun-Tex Truck Stop is located in the desert southwest<br />

and is 5 miles from the nearest municipal water<br />

system. In order to have fresh water at the site, the<br />

company purchased a turnkey reverse osmosis (RO)<br />

system for $355,000. The company depreciated the<br />

RO system using the MACRS method with a<br />

10-year recovery period. Four years after the system<br />

was purchased, water lines from a local water system<br />

were extended to the truck stop, so Sun-Tex<br />

sold the RO system for $190,000. Determine which<br />

of the following apply and the amount, if any, to<br />

include in an after-tax analysis of the project: depreciation<br />

recapture, capital gain, capital loss. The<br />

MACRS depreciation rates are 10%, 18%, 14.4%,<br />

and 11.52% for years 1 through 4, respectively.


Problems 477<br />

17.37 Freeman <strong>Engineering</strong> paid $28,500 for specialized<br />

equipment for use with its new GPSGIS system.<br />

The equipment was depreciated over a 3-year recovery<br />

period using MACRS depreciation. The<br />

company sold the equipment after 2 years for<br />

$5000 when it purchased an upgraded system.<br />

( a ) Determine the amount of the depreciation recapture<br />

or capital loss involved in selling the asset.<br />

( b ) What tax effect will this amount have?<br />

17.38 Determine any depreciation recapture, capital gain,<br />

or capital loss generated by each event described<br />

below. Use them to determine the amount of income<br />

tax effect, if the effective tax rate is 35%.<br />

( a) A MACRS-depreciated asset with a 7-year<br />

recovery period has been sold prematurely<br />

after 4 years at an amount equal to 40% of its<br />

first cost, which was $150,000.<br />

( b) A hi-tech machine was sold internationally<br />

for $10,000 more than its purchase price just<br />

after it was in service 1 year. The asset had<br />

P $100,000, S $1000, and n 3 years<br />

and was depreciated by the MACRS method<br />

for the 1 year.<br />

( c) Land purchased 4 years ago for $1.8 million<br />

was sold at a 10% profit.<br />

( d) A 21-year-old asset was removed from<br />

service and sold for $500. When purchased,<br />

the asset was entered on the books with a<br />

basis of P $180,000, S $5000, and n <br />

18 years. Classical straight line depreciation<br />

was used for the entire recovery period.<br />

(e) A corporate car was depreciated using MACRS<br />

over a 3-year recovery period. It was sold in<br />

the fourth year of use for $2000.<br />

17.39 Sunnen Products Co. of St. Louis, Missouri, makes<br />

actuator hones for gas meter tubes where light-duty<br />

metal removal is needed. The company purchased<br />

land, a building, and two depreciable assets from<br />

MPG Automation Systems Corporation, all of which<br />

have recently been disposed of. Use the information<br />

shown to determine the presence and amount of any<br />

depreciation recapture, capital gain, or capital loss.<br />

Asset<br />

Purchase<br />

Price, $<br />

Recovery<br />

Period,<br />

Years<br />

Current<br />

Book<br />

Value, $<br />

Sales<br />

Price, $<br />

Land 220,000 — 295,000<br />

Building 900,000 27.5 300,000 275,000<br />

Asset 1 50,500 3 15,500 19,500<br />

Asset 2 20,000 3 5,000 12,500<br />

After-Tax Economic Analysis<br />

17.40 Compute the required before-tax return if an aftertax<br />

return of 7% per year is expected and the state<br />

and local tax rates total 4.2%. The effective federal<br />

tax rate is 34%.<br />

17.41 Estimate the approximate after-tax rate of return<br />

(ROR) for a project that has a before-tax ROR of<br />

24%. Assume that the company has an effective<br />

tax rate of 35% and it uses MACRS depreciation<br />

for an asset that has a $27,000 salvage value.<br />

17.42 Estimate the approximate after-tax rate of return<br />

for a project that has a first cost of $500,000, a<br />

salvage value of 20% of the first cost after 3 years,<br />

and annual gross income less operating expenses<br />

of (GIOE) $230,000. Assume the company<br />

has a 35% effective tax rate.<br />

17.43 An engineer is making an annual return of 12%<br />

before taxes on the retirement investments placed<br />

through her employer. No taxes are paid on retirement<br />

earnings until they are withdrawn; however,<br />

she was told by her brother, an accountant, that this<br />

is the equivalent of an 8% per year after-tax return.<br />

What percent of taxable income is her brother assuming<br />

to be taken by income taxes?<br />

17.44 Bart is an economic consultant to the textile industry.<br />

In both a small business and a large corporation<br />

he performed economic evaluations that have<br />

an average 18% per year before-tax return. If the<br />

stated MARR in both companies is 12% per year<br />

after taxes, determine if management at both companies<br />

should accept the projects. The before-tax<br />

return is used to approximate the after-tax return.<br />

Effective tax rates are 34% for the larger corporation<br />

and 28% for the small company.<br />

17.45 Elias wants to perform an after-tax evaluation of<br />

equivalent methods A and B to electrostatically remove<br />

airborne particulate matter from clean rooms<br />

used to package liquid pharmaceutical products.<br />

Using the information shown, MACRS depreciation<br />

with n 3 years, a 5-year study period, after-tax<br />

MARR 7% per year, and T e 34% and a spreadsheet,<br />

he obtained the results AW A $−2176 and<br />

AW B $3545. Any tax effects when the equipment<br />

is salvaged were neglected. Thus, with MACRS depreciation,<br />

method B is the better method. Now, use<br />

classical SL depreciation with n 5 years to evaluate<br />

the alternatives. Is the decision different from<br />

that reached using MACRS?<br />

Method A<br />

Method B<br />

First cost, $ 100,000 150,000<br />

Salvage value, $ 10,000 20,000<br />

Savings, $ per year 35,000 45,000<br />

AOC, $ per year 15,000 6,000<br />

Expected life, years 5 5


478 Chapter 17 After-Tax Economic Analysis<br />

17.46 A corporation uses the following: before-tax<br />

MARR of 14% per year, after-tax MARR of<br />

7% per year, and T e of 50%. Two new machines<br />

have the following estimates.<br />

Machine A<br />

Machine B<br />

First cost, $ 15,000 22,000<br />

Salvage value, $ 3,000 5,000<br />

AOC, $ per year 3,000 1,500<br />

Life, years 10 10<br />

The machine is retained in use for a total of 10 years,<br />

then sold for the estimated salvage value. Select one<br />

machine under the following conditions:<br />

(a) Before-tax PW analysis.<br />

(b) After-tax PW analysis, using classical SL depreciation<br />

over the 10-year life.<br />

(c) After-tax PW analysis, using MACRS depreciation<br />

with a 5-year recovery period.<br />

17.47 Choose between alternatives A and B below if the<br />

after-tax MARR is 8% per year, MACRS depreciation<br />

is used, and T e 40%. The GIOE estimate<br />

is made for only 3 years; it is zero when each asset<br />

is sold in year 4.<br />

Alternative<br />

A<br />

Alternative<br />

B<br />

First cost, $ 8,000 13,000<br />

Actual salvage value, $ 0 2,000<br />

Gross income expenses, $ 3,500 5,000<br />

Recovery period, years 3 3<br />

17.48 Offshore platform safety equipment, designed for<br />

special jobs, will cost $2,500,000, will have no<br />

salvage value, and will be kept in service for exactly<br />

5 years, according to company policy. Operating<br />

revenue minus expenses is estimated to be<br />

$1,500,000 in year 1 and only $300,000 each additional<br />

year. The effective tax rate for the multinational<br />

oil company is 30%. Show hand and<br />

spreadsheet solutions, as instructed, to find the<br />

after-tax ROR using ( a ) classical SL depreciation<br />

and ( b ) MACRS 5-year depreciation, neglecting<br />

any tax effects at the end of 5 years of service.<br />

17.49 Automatic inspection equipment purchased for<br />

$78,000 by Stimson <strong>Engineering</strong> generated an average<br />

of $26,080 annually in before-tax cash flow<br />

during its 5-year estimated life. This represents a<br />

return of 20%. However, the corporate tax expert<br />

said the CFAT was only $15,000 per year. If the<br />

corporation wants to realize an after-tax return of<br />

10% per year, for how many more years must the<br />

equipment remain in service?<br />

After-Tax Replacement<br />

17.50 In a replacement study between a defender and a<br />

challenger, there may be a capital gain or loss<br />

when the defender asset is sold. ( a ) How is the<br />

gain or loss calculated, and ( b ) how does it affect<br />

the AW values in the study?<br />

17.51 In an after-tax replacement study of cost alternatives<br />

involving one challenger and one defender,<br />

how will a capital loss when selling the defender<br />

affect the AW of each alternative?<br />

17.52 An asset that was purchased 2 years ago was expected<br />

to be kept in service for its projected life of 5 years, but<br />

a new version (the challenger) of this asset promises<br />

to be more efficient and have lower operating costs.<br />

You have been asked to figure out if it would be more<br />

economically attractive to replace the defender now<br />

or keep it for 3 more years as originally planned. The<br />

defender had a first cost of $300,000, but its market<br />

value now is only $150,000. It has operating expenses<br />

of $120,000 per year and no estimated salvage value<br />

after 3 more years. To simplify calculations for this<br />

problem only, assume that SL depreciation was<br />

charged at $60,000 per year and that it will continue<br />

at that rate for the next 3 years.<br />

The challenger will cost $420,000, will have no<br />

salvage value after its 3-year life, will have chargeable<br />

expenses of $30,000 per year, and will be depreciated<br />

at $140,000 per year (again, using SL<br />

depreciation for simplicity in this case). Assume<br />

the company’s effective tax rate is 35%, and its<br />

after-tax MARR is 15% per year.<br />

(a) Determine the CFAT in year 0 for the challenger<br />

and defender.<br />

(b)<br />

Determine the CFAT in years 1 through 3 for<br />

the challenger and defender.<br />

(c) Conduct an AW evaluation to determine if<br />

the defender should be kept for 3 more years<br />

or replaced now.<br />

17.53 The defender in a catalytic oxidizer manufacturing<br />

plant has a market value of $130,000 and expected<br />

annual operating costs of $70,000 with no salvage<br />

value after its remaining life of 3 years. The depreciation<br />

charges for the next 3 years will be $69,960,<br />

$49,960, and $35,720. Using an effective tax rate<br />

of 35%, determine the CFAT for next year only<br />

that should be used in a present worth equation to<br />

compare this defender against a challenger that<br />

also has a 3-year life. Assume the company’s aftertax<br />

MARR is 12%.<br />

17.54 Perform a PW replacement study (hand and<br />

spreadsheet solutions, if instructed) from the information<br />

shown using an after-tax MARR 12%<br />

per year, T e 35%, and a study period of 4 years.


Problems 479<br />

(Assume that the assets will be salvaged at their<br />

original salvage estimates. Since no revenues are<br />

estimated, all taxes are negative and considered<br />

“savings” to the alternative.) All monetary values<br />

are in $1000 units.<br />

Defender Challenger<br />

First cost, $1000 45 24<br />

Estimated S at purchase, $1000 5 0<br />

Market value now, $1000 35 —<br />

AOC, $1000 per year 7 8<br />

Depreciation method SL MACRS<br />

Recovery period, years 8 3<br />

Useful life, years 8 5<br />

Years owned 3 —<br />

17.55 After 8 years of use, the heavy truck engine overhaul<br />

equipment at Pete’s Truck Repair was evaluated<br />

for replacement. Pete’s accountant used an<br />

after-tax MARR of 8% per year, T e 30%, and a<br />

current market value of $25,000 to determine<br />

AW $2100. The new equipment costs $75,000,<br />

uses SL depreciation over a 10-year recovery period,<br />

and has a $15,000 salvage estimate. Estimated<br />

CFBT is $15,000 per year. Pete asked his<br />

engineer son Ramon to determine if the new equipment<br />

should replace what is owned currently.<br />

From the accountant, Ramon learned the current<br />

equipment cost $20,000 when purchased and<br />

reached a zero book value several years ago. Help<br />

Ramon answer his father’s question.<br />

17.56 Apple Crisp Foods signed a contract some years<br />

ago for maintenance services on its fleet of<br />

trucks and cars. The contract is up for renewal<br />

now for a period of 1 year or 2 years only. The<br />

contract quote is $300,000 per year if taken for<br />

1 year and $240,000 per year if taken for 2 years.<br />

The finance vice president wants to renew the<br />

contract for 2 years without further analysis, but<br />

the vice president for engineering believes it is<br />

more economical to perform the maintenance<br />

in-house. Since much of the fleet is aging and<br />

must be replaced in the near future, a fixed<br />

3-year study period has been agreed upon. The<br />

estimates for the in-house (challenger) alternative<br />

are as follows:<br />

First cost, $ −800,000<br />

AOC, $ per year −120,000<br />

Life, years 4<br />

Estimated salvage Loses 25% of P annually:<br />

End year 1, S $600,000<br />

End year 2, S $400,000<br />

End year 3, S $200,000<br />

End year 4, S $0<br />

MACRS depreciation 3-year recovery period<br />

The effective tax rate is 35%, and the after-tax<br />

MARR is 10% per year. Perform an after-tax<br />

AW analysis, and determine which vice president<br />

has the better economic strategy over the<br />

next 3 years.<br />

17.57 Nuclear safety devices installed several years<br />

ago have been depreciated from a first cost of<br />

$200,000 to zero using MACRS. The devices<br />

can be sold on the used equipment market for an<br />

estimated $15,000. Or they can be retained in<br />

service for 5 more years with a $9000 upgrade<br />

now and an AOC of $6000 per year. The upgrade<br />

investment will be depreciated over 3 years with<br />

no salvage value. The challenger is a replacement<br />

with newer technology at a first cost of<br />

$40,000, n 5 years, and S 0. The new units<br />

will have operating expenses of $7000 per year.<br />

( a ) Use a 5-year study period, an effective tax<br />

rate of 40%, an after-tax MARR of 12% per<br />

year, and an assumption of classical straight line<br />

depreciation (no half-year convention) to perform<br />

an after-tax replacement study. ( b ) If the<br />

challenger is known to be salable after 5 years<br />

for an amount between $2000 and $4000, will<br />

the challenger AW value become more or less<br />

costly? Why?<br />

17.58 Three years ago, Silver House Steel purchased a<br />

new quenching system for $550,000. The<br />

salvage value after 10 years at that time was estimated<br />

to be $50,000. Currently the expected<br />

remaining life is 7 years with an AOC of $27,000<br />

per year. The new president has recommended<br />

early replacement of the system with one that<br />

costs $400,000 and has a 12-year life, a $35,000<br />

salvage value, and an estimated AOC of $50,000<br />

per year. The MARR for the corporation is 12%<br />

per year. The president wishes to know the<br />

replacement value that will make the recommendation<br />

to replace now economically advantageous.<br />

Use a spreadsheet and Solver to find the<br />

minimum trade-in value ( a ) before taxes and<br />

( b ) after taxes, using an effective tax rate of<br />

30%. For solution purposes, use classical SL depreciation<br />

for both systems. Comment on the<br />

difference in replacement value made by the<br />

consideration of taxes.<br />

Economic Value Added<br />

17.59 While an engineering manager may prefer to use<br />

CFAT estimates to evaluate the AW of a project, a<br />

financial manager may select AW of EVA estimates.<br />

Why are these preferences predictable?<br />

17.60 Cardenas and Moreno <strong>Engineering</strong> is evaluating<br />

a very large flood control program for several


480 Chapter 17 After-Tax Economic Analysis<br />

southern U.S. cities. One component is a 4-year<br />

project for a special-purpose transport ship-crane<br />

for use in building permanent storm surge protection<br />

against hurricanes on the New Orleans coastline.<br />

The estimates are P $300,000, S 0, and<br />

n 3 years. MACRS depreciation with a 3-year<br />

recovery is indicated. Gross income and operating<br />

expenses are estimated at $200,000 and<br />

$80,000, respectively, for each of 4 years. The<br />

CFAT is shown below. Calculate the AW values<br />

of the CFAT and EVA series. They should have<br />

the same value. The after-tax MARR is 9.75%<br />

and T e 35%.<br />

Year GI, $ OE, $ P and S , $ D, $ TI, $ Taxes, $ CFAT, $<br />

0 300,000 300,000<br />

1 200,000 80,000 99,990 20,010 7,003 112,997<br />

2 200,000 80,000 133,350 13,350 4,673 124,673<br />

3 200,000 80,000 0 44,430 75,570 26,450 93,551<br />

4 200,000 80,000 22,230 97,770 34,220 85,781<br />

17.61 Triple Play Innovators Corporation (TPIC) plans<br />

to offer IPTV (Internet Protocol TV) service to<br />

North American customers starting soon. Perform<br />

an AW analysis of the EVA series for the two alternative<br />

suppliers available for the hardware and<br />

software. Let T e 30% and after-tax MARR <br />

8%; use SL depreciation (neglect half-year convention<br />

and MACRS, for simplicity) and a study<br />

period of 8 years.<br />

Hong Kong<br />

Vendor<br />

Japan<br />

First cost, $ 4.2 million 3.6 million<br />

Recovery period, years 8 5<br />

Salvage value, $ 0 0<br />

GI − OE, $ per year 1,500,000 in year 1; increasing<br />

by 300,000 per year up to 8 years<br />

17.62 Sun Microsystems has developed partnerships<br />

with several large manufacturing corporations to<br />

use Java software in their consumer and industrial<br />

products. A new corporation will be formed to<br />

manage these applications. One major project involves<br />

using Java in commercial and industrial<br />

appliances that store and cook food. The gross<br />

income and operating expenses are expected to<br />

follow the relations shown for the estimated life of<br />

6 years. For t 1 to 6 years,<br />

Annual gross income, GI 2,800,000 100,000 t<br />

Annual operating expenses, OE 950,000 50,000 t<br />

The effective tax rate is 30%, the interest rate is<br />

12% per year, and the depreciation method chosen<br />

for the $3,000,000 capital investment is a 5-year<br />

MACRS ADS alternative that allows straight line<br />

write-off with the half-year convention in years 1<br />

and 6. Using a spreadsheet, estimate ( a ) the annual<br />

economic contribution of the project to the new<br />

corporation and ( b ) the equivalent annual worth of<br />

these contributions.<br />

Value-Added Tax<br />

17.63 What is the primary difference between a sales tax<br />

and a value-added tax?<br />

17.64 In Denmark, VAT is applied at a rate of 25%, with<br />

few exceptions. Vendor A sells raw materials to<br />

vendor B for $60,000 plus VAT, vendor B sells a<br />

product to vendor C for $130,000 plus VAT, and<br />

vendor C sells an improved, value-added product<br />

to an end user for $250,000 plus VAT. Determine<br />

the following.<br />

(a) The amount of VAT collected by vendor B.<br />

(b) The amount of tax vendor B sends to<br />

Denmark’s Treasury.<br />

(c) The total amount of tax collected by the<br />

Treasury department.<br />

The following information is used in Problems 17.65<br />

through 17.70<br />

Ajinkya Electronic Systems, a company in India that<br />

manufactures many different electronic products, has to<br />

purchase goods and services from a variety of suppliers<br />

(wire, diodes, LED displays, plastic components, etc.).<br />

The table below shows several suppliers and the VAT<br />

rates associated with each. It also shows the purchases in<br />

$1000 units that Ajinkya made (before taxes) from each<br />

supplier in the previous accounting period. Ajinkya’s<br />

sales to end users was $9.2 million, and Ajinkya’s products<br />

carry a VAT of 12.5%.<br />

Supplier VAT Rate, %<br />

Purchases by<br />

Ajinkya, $1000<br />

A 4.0 350<br />

B 12.5 870<br />

C 12.5 620<br />

D 21.3 90<br />

E 32.6 50<br />

17.65 How much VAT did supplier C collect?<br />

17.66 How much tax did Ajinkya keep from the tax it<br />

collected based on the purchases it made from<br />

supplier A?


Additional Problems and FE Exam Review Questions 481<br />

17.67 What was the total amount of taxes paid by Ajinkya<br />

to the suppliers?<br />

17.68 What was the average VAT rate paid by Ajinkya in<br />

purchasing goods and services?<br />

17.69 What was the amount of taxes Ajinkya sent to the<br />

Treasury of India?<br />

17.70 What was the total amount of taxes collected by<br />

the Treasury of India from Ajinkya and Ajinkya’s<br />

suppliers?<br />

ADDITIONAL PROBLEMS AND FE EXAM REVIEW QUESTIONS<br />

17.71 A graduated income tax system means:<br />

( a) Only taxable incomes above a certain level<br />

pay any taxes.<br />

( b) A higher flat rate goes with all of the taxable<br />

income.<br />

( c) Higher tax rates go with higher taxable<br />

incomes.<br />

( d) Rates are indexed each year to keep up with<br />

inflation.<br />

17.72 All of the following are characteristics of a valueadded<br />

tax system except:<br />

( a) Value-added taxes are taxes on consumption.<br />

( b) The end user pays value-added taxes.<br />

( c) Value-added taxes are charged at each stage<br />

of product development.<br />

( d) Value-added taxes are charged only on the<br />

raw materials for product development.<br />

17.73 A small company has a taxable income that places<br />

it in the 35% tax bracket. The amount of taxes that<br />

a depreciation charge of $16,000 would save is<br />

closest to:<br />

( a ) $0<br />

( b ) $3200<br />

( c ) $5600<br />

( d ) $10,400<br />

17.74 A company that has a 50% effective tax rate had<br />

income of $200 million in each of the last 2 years.<br />

In one of those years, the company had deductions<br />

of $100 million. In the other year, the company<br />

had deductions of only $80 million. The difference<br />

in income taxes paid by the company in those<br />

2 years was closest to:<br />

( a) $10 million<br />

(b) $20 million<br />

(c) $50 million<br />

(d) $60 million<br />

17.75 Taxable income (TI) is defined as:<br />

( a) TI revenue operating expenses<br />

depreciation<br />

( b) TI revenue operating expenses<br />

depreciation<br />

(c)<br />

(d)<br />

TI revenue operating expenses<br />

depreciation amortization<br />

TI revenue operating expenses<br />

depreciation<br />

17.76 The marginal tax rate is defined as:<br />

( a) The percentage paid on the last dollar of<br />

i ncome<br />

(b) The tax rate that applies to a questionable<br />

investment<br />

(c)<br />

The tax rate that includes federal, state, and<br />

local taxes<br />

(d) The percentage paid on the first dollar of<br />

income<br />

17.77 A subcontractor with an effective tax rate of 25%<br />

has gross income of $55,000, other income of<br />

$4000, operating expenses of $13,000, and other<br />

deductions and exemptions of $11,000. The income<br />

tax due is closest to:<br />

( a ) $11,750<br />

( b ) $8,750<br />

( c ) $10,750<br />

( d ) $13,750<br />

17.78 When a depreciable asset is disposed of for less than<br />

its current book value, the transaction is known as:<br />

( a) An after-tax expense<br />

(b) Capital loss<br />

(c) Capital gain<br />

(d) Depreciation recapture<br />

17.79 The after-tax analysis for a $60,000 investment<br />

with associated gross income minus expenses<br />

(GIOE) is shown below for the first 2 years only.<br />

If the effective tax rate is 40%, the values for depreciation<br />

( D ), taxable income (TI), and taxes for<br />

year 1 are closest to:<br />

Year<br />

Investment,<br />

$<br />

GI − OE,<br />

$ D, $ TI, $<br />

Taxes,<br />

$<br />

CFAT,<br />

$<br />

0 60,000 60,000<br />

1 30,000 26,000<br />

2 35.000 15,000 6000 29,000


482 Chapter 17 After-Tax Economic Analysis<br />

(a) D $5,000, TI $25,000, taxes $10,000<br />

(b) D $30,000, TI $30,000, taxes $4,000<br />

(c) D $20,000, TI $50,000, taxes $20,000<br />

(d) D $20,000, TI $10,000, taxes $4,000<br />

17.80 If the after-tax rate of return for a cash flow series<br />

is 11.9% and the corporate effective tax rate is<br />

34%, the approximated before-tax rate of return is<br />

closest to:<br />

( a ) 6.8%<br />

( b ) 5.4%<br />

( c ) 18.0%<br />

( d ) 28.7%<br />

17.81 An asset purchased for $100,000 with S $20,000<br />

after 5 years was depreciated using the 5-year<br />

MACRS rates. Expenses averaged $18,000 per<br />

year, and the effective tax rate is 30%. The asset<br />

was actually sold after 5 years of service for<br />

$22,000. MACRS rates in years 5 and 6 are 11.53%<br />

and 5.76%, respectively. The after-tax cash flow<br />

from the sale is closest to:<br />

( a ) $27,760<br />

( b ) $17,130<br />

( c ) $26,870<br />

( d ) $20,585<br />

17.82 When accelerated depreciation methods or<br />

shortened recovery periods are applied, there<br />

are impacts on the income taxes due. Of the following,<br />

the statements that are commonly incorrect<br />

are:<br />

1. Total taxes paid are the same for all depreciation<br />

methods.<br />

2. Present worth of taxes is lower for shorter<br />

recovery periods.<br />

3. Accelerated depreciation imposes more taxes<br />

in the later years of the recovery period.<br />

4. Present worth of taxes is higher for shorter<br />

recovery periods.<br />

( a ) 1, 2, and 3<br />

( b ) 1 and 4<br />

( c ) 2<br />

( d ) 4<br />

CASE STUDY<br />

AFTER-TAX ANALYSIS FOR BUSINESS EXPANSION<br />

Background<br />

Charles was always a hands-on type of person. Within a couple<br />

of years of graduating from college, he started his own business.<br />

After some 20 years, it has grown significantly. He owns and<br />

operates Pro-Fence, Inc. in the Metroplex, specializing in custom-made<br />

metal and stone fencing for commercial and residential<br />

sites. For some time, Charles has thought he should expand<br />

into a new geographic region, with the target area being another<br />

large metropolitan area about 500 miles north, called Victoria.<br />

Pro-Fence is privately owned by Charles; therefore, the question<br />

of how to finance such an expansion has been, and still is,<br />

the major challenge. Debt financing would not be a problem in<br />

that the Victoria Bank has already offered a loan of up to $2 million.<br />

Taking capital from the retained earnings of Pro-Fence is a<br />

second possibility, but taking too much will jeopardize the current<br />

business, especially if the expansion were not an economic<br />

success and Pro-Fence were stuck with a large loan to repay.<br />

This is where you come in as a long-time friend of<br />

Charles. He knows you are quite economically oriented and<br />

that you understand the rudiments of debt and equity financing<br />

and economic analysis. He wants you to advise him on<br />

the balance between using Pro-Fence funds and borrowed<br />

funds. You have agreed to help him, as much as you can.<br />

Information<br />

Charles has collected some information that he shares with<br />

you. Between his accountant and a small market survey of the<br />

business opportunities in Victoria, the following generalized<br />

estimates seem reasonable.<br />

Initial capital investment $1.5 million<br />

Annual gross income $700,000<br />

Annual operating expenses $100,000<br />

Effective income tax rate for Pro-Fence 35%<br />

Five-year MACRS depreciation for all $1.5 million investment<br />

The terms of the Victoria Bank loan would be 6% per year<br />

simple interest based on the initial loan principal. Repayment<br />

would be in 5 equal payments of interest and principal.<br />

Charles comments that this is not the best loan arrangement<br />

he hopes to get, but it is a good worst-case<br />

scenario upon which to base the debt portion of the analysis.<br />

A range of D-E mixes should be analyzed. Between<br />

Charles and yourself, you have developed the following<br />

viable options.<br />

Debt<br />

Loan<br />

Percentage Amount, $<br />

Equity<br />

Investment<br />

Percentage Amount, $<br />

0 100 1,500,000<br />

50 750,000 50 750,000<br />

70 1,050,000 30 450,000<br />

90 1,350,000 10 150,000


Case Study 483<br />

Case Study Exercises<br />

1. For each funding option, perform a spreadsheet analysis<br />

that shows the total CFAT and its present worth over a<br />

6-year period, the time it will take to realize the full<br />

advantage of MACRS depreciation. An after-tax return<br />

of 10% is expected. Which funding option is best for<br />

Pro-Fence? ( Hint: For the spreadsheet, sample column<br />

headings are: Year, GI − OE, Loan interest, Loan principal,<br />

Equity investment, Depreciation rate, Depreciation,<br />

Book value, TI, Taxes, and CFAT.)<br />

2. Observe the changes in the total 6-year CFAT as the<br />

D-E percentages change. If the time value of money<br />

is neglected, what is the constant amount by which<br />

this sum changes for every 10% increase in equity<br />

funding?<br />

3. Charles noticed that the CFAT total and PW values go in<br />

opposite directions as the equity percentage increases.<br />

He wants to know why this phenomenon occurs. How<br />

should you explain this to Charles?<br />

4. After deciding on the 50-50 split of debt and equity financing,<br />

Charles wants to know what additional bottomline<br />

contributions to the economic worth of the company<br />

may be added by the new Victoria site. What are the best<br />

estimates at this time?


CHAPTER 18<br />

Sensitivity<br />

Analysis and<br />

Staged<br />

Decisions<br />

LEARNING OUTCOMES<br />

Purpose : Perform a sensitivity analysis of parameters; use expected values to evaluate staged funding options.<br />

SECTION TOPIC LEARNING OUTCOME<br />

18.1 Sensitivity to variation • Use a measure of worth to explain sensitivity to<br />

variation in one or more parameters.<br />

18.2 Three-estimate variation • Choose the better alternative using three<br />

estimates of variation in selected parameters.<br />

18.3 E ( X ) • Calculate the expected value of a variable.<br />

18.4 E ( X ) of cash flows • Evaluate a project or alternatives using expected<br />

values of cash flows.<br />

18.5 Decision trees • Use a decision tree to evaluate alternatives stage<br />

by stage.<br />

18.6 Real options • Explain a real option in engineering economics<br />

and evaluate a staged decision using real<br />

options analysis.


T<br />

his chapter includes several related topics about alternative evaluation. Initially,<br />

we expand our capability to perform a sensitivity analysis of one or<br />

more parameters and of an entire alternative. Then the determination and use<br />

of the expected value of a cash flow series are treated. The techniques of decision trees<br />

help make a series of economic decisions for alternatives that have different, but closely<br />

connected stages.<br />

Economic decisions that involve staged funding are very common in professional and<br />

everyday life. The last topic of real options analysis introduces a method useful in these<br />

circumstances.<br />

18.1 Determining Sensitivity to Parameter Variation<br />

The term parameter is used in this chapter to represent any variable or factor for which an estimate<br />

or stated value is necessary. Example parameters are first cost, salvage value, AOC, estimated<br />

life, production rate, and materials costs. Estimates such as the loan interest rate and the<br />

inflation rate can also be parameters of the analysis.<br />

Economic analysis uses estimates of a parameter’s future value to assist decision makers.<br />

Since future estimates are always incorrect to some degree, inaccuracy is present in the economic<br />

projections. The effect of variation may be determined by using sensitivity analysis.<br />

Sensitivity analysis determines how a measure of worth—PW, AW, FW, ROR, BC, or<br />

CER—is altered when one or more parameters vary over a selected range of values. Usually<br />

one parameter at a time is varied, and independence with other parameters is assumed. Though<br />

this approach is an oversimplification in real-world situations, since the dependencies are difficult<br />

to accurately model, the end results are usually correct.<br />

Sensitivity analysis<br />

In reality, we have applied this approach (informally) throughout previous chapters to determine<br />

the response to variation in a variety of parameters. Variation in a parameter such as MARR will<br />

not alter the decision to select an alternative when all compared alternatives return considerably<br />

more than the MARR; thus, the decision is relatively insensitive to the MARR. However, variation<br />

in the n or AOC value may indicate that the alternative’s measure of worth is very sensitive<br />

to the estimated life or annual operating costs.<br />

Usually the variations in life, annual costs, and revenues result from variations in selling price,<br />

operation at different levels of capacity, inflation, etc. For example, if an operating level of 90%<br />

of airline seating capacity for a domestic route is compared with 70% for a proposed international<br />

route, the operating cost and revenue per passenger-mile will increase, but anticipated aircraft<br />

life will probably decrease only slightly. Usually several important parameters are studied to<br />

learn how the uncertainty of estimates affects the economic analysis.<br />

Sensitivity analysis routinely concentrates on the variation expected in estimates of P , AOC,<br />

S , n , unit costs, unit revenues, and similar parameters. These parameters are often the result of<br />

design questions and their answers, as discussed in Chapter 15. Parameters that are interest rate–<br />

based are not treated in the same manner.<br />

Parameters such as MARR and other interest rates (loan rates, inflation rate) are more stable from<br />

project to project. If performed, sensitivity analysis on them is for specific values or over a<br />

narrow range of values. This point is important to remember if simulation is used for decision<br />

making under risk (Chapter 19).<br />

Plotting the sensitivity of PW, AW, or ROR versus the parameter(s) studied is very helpful.<br />

Two alternatives can be compared with respect to a given parameter and the breakeven point.<br />

This is the value at which the two alternatives are economically equivalent. However, the breakeven<br />

chart commonly represents only one parameter per chart. Thus, several charts are constructed,<br />

and independence of each parameter is assumed. In previous uses of breakeven analysis,<br />

we often computed the measure of worth at only two values of a parameter and connected the<br />

points with a straight line. However, if the results are sensitive to the parameter value, several<br />

intermediate points should be used to better evaluate the sensitivity, especially if the relationships<br />

are not linear.<br />

When several parameters are studied, sensitivity analysis can become quite complex. It may<br />

be performed one parameter at a time using a spreadsheet or computations by hand or calculator.<br />

PW<br />

Low<br />

Parameter<br />

High


486 Chapter 18 Sensitivity Analysis and Staged Decisions<br />

EXAMPLE 18.1<br />

The computer facilitates comparison of multiple parameters and multiple measures of worth, and<br />

the software can rapidly plot the results.<br />

Here is a general procedure to follow when conducting a thorough sensitivity analysis.<br />

1. Determine which parameter(s) of interest might vary from the most likely estimated value.<br />

2. Select the probable range and an increment of variation for each parameter.<br />

3. Select the measure of worth.<br />

4. Compute the results for each parameter, using the measure of worth as a basis.<br />

5. To better interpret the sensitivity, graphically display the parameter versus the measure of<br />

worth.<br />

This sensitivity analysis procedure should indicate the parameters that warrant closer study<br />

or require additional information. When there are two or more alternatives, it is better to use<br />

the PW or AW measure of worth in step 3. If ROR is used, it requires the extra efforts of<br />

incremental analysis between alternatives. Example 18.1 illustrates sensitivity analysis for<br />

one project.<br />

Wild Rice, Inc. expects to purchase a new asset for automated rice handling. Most likely estimates<br />

are a first cost of $80,000, zero salvage value, and a cash flow before taxes (CFBT) per<br />

year t that follows the relation $27,000 − 2000t. The MARR for the company varies over a<br />

wide range from 10% to 25% per year for different types of investments. The economic life of<br />

similar machinery varies from 8 to 12 years. Evaluate the sensitivity of PW by varying<br />

(a) MARR, while assuming a constant n value of 10 years, and (b) n, while MARR is constant<br />

at 15% per year. Perform the analysis by hand and by spreadsheet.<br />

Solution by Hand<br />

(a) Follow the procedure above to understand the sensitivity of PW to MARR variation.<br />

1. MARR is the parameter of interest.<br />

2. Select 5% increments to evaluate sensitivity to MARR; the range is 10% to 25%.<br />

3. The measure of worth is PW.<br />

4. Set up the PW relation for 10 years. When MARR 10%,<br />

PW 80,000 25,000(PA,10%,10) 2000(PG,10%,10)<br />

$27,830<br />

The PW for all MARR values at 5% intervals is as follows:<br />

MARR, % PW, $<br />

10 27,830<br />

15 11,512<br />

20 962<br />

25 10,711<br />

5. A plot of MARR versus PW is shown in Figure 18–1. The steep negative slope indicates<br />

that the decision to accept the proposal based on PW is quite sensitive to<br />

variations in the MARR. If the MARR is established at the upper end of the range, the<br />

investment is not attractive.<br />

(b) 1. Asset life n is the parameter.<br />

2. Select 2-year increments to evaluate PW sensitivity over the range 8 to 12 years.<br />

3. The measure of worth is PW.<br />

4. Set up the same PW relation as in part (a) at i 15%. The PW results are<br />

n PW, $<br />

8 7,221<br />

10 11,511<br />

12 13,145


18.1 Determining Sensitivity to Parameter Variation 487<br />

30,000<br />

20,000<br />

MARR<br />

Figure 18–1<br />

Plot of PW versus MARR<br />

and n for sensitivity<br />

analysis, Example 18.1.<br />

n<br />

Present worth, $<br />

10,000<br />

0<br />

10<br />

6<br />

15<br />

8<br />

20<br />

10<br />

25<br />

12<br />

MARR %<br />

Life n<br />

– 10,000<br />

– 20,000<br />

5. Figure 18–1 presents the plot of PW versus n. Since the PW measure is positive for all<br />

values of n, the decision to invest is not materially affected by the estimated life. The PW<br />

curve levels out above n 10. This insensitivity to changes in cash flow in the distant<br />

future is a predictable observation, because the PF factor gets smaller as n increases.<br />

Solution by Spreadsheet<br />

Figure 18–2 presents two spreadsheets and accompanying plots of PW versus MARR (fixed n)<br />

and PW versus n (fixed MARR). The NPV function calculates PW for i values from 10% to<br />

25% and n values from 8 to 12 years. As the solution by hand indicated, so do the charts; PW<br />

is sensitive to changes in MARR values, but not very sensitive to variations in n.<br />

Figure 18–2<br />

Sensitivity analysis of<br />

PW to variation in<br />

(a) MARR values and<br />

(b) life estimates,<br />

Example 18.1.<br />

PW computation<br />

NPV(C6,B$4:B$13)<br />

B$3<br />

(a)<br />

PW computation<br />

NPV(15%,B$4:B$15)<br />

B$3<br />

(b)


488 Chapter 18 Sensitivity Analysis and Staged Decisions<br />

Figure 18–3<br />

Sensitivity analysis graph<br />

of percent variation from<br />

the most likely estimate.<br />

Rate of return on capital<br />

50%<br />

40%<br />

30%<br />

20%<br />

10%<br />

+<br />

+<br />

+<br />

+<br />

+<br />

+<br />

+<br />

+<br />

+<br />

0%<br />

– 10%<br />

+<br />

– 20%<br />

– 50% – 40% – 30% – 20% – 10% 0% 10% 20% 30% 40% 50%<br />

Parameter Legend<br />

Sales price<br />

Direct material<br />

Sales volume<br />

Indirect cost<br />

Direct labor<br />

Capital<br />

% Change in individual parameter<br />

When the sensitivity of several parameters is considered for one alternative using a single<br />

measure of worth, it is helpful to graph percentage change for each parameter versus the measure<br />

of worth. This is sometimes called a spider graph . Figure 18–3 illustrates ROR versus six<br />

different parameters for one alternative. The variation in each parameter is indicated as a percentage<br />

deviation from the most likely estimate on the horizontal axis. If the ROR response curve is<br />

flat and approaches horizontal over the range of total variation graphed for a parameter, there is<br />

little sensitivity of ROR to changes in the parameter’s value. This is the conclusion for indirect<br />

cost in Figure 18–3 . On the other hand, ROR is very sensitive to sales price. A reduction of 30%<br />

from the expected sales price reduces the ROR from approximately 20% to −10%, whereas a<br />

10% increase in price raises the ROR to about 30%.<br />

If two alternatives are compared and the sensitivity to one parameter is sought, the graph<br />

may show quite nonlinear results. Observe the general shape of the sample sensitivity graphs in<br />

Figure 18–4 . The plots are shown as linear segments between specific computation points. The<br />

graph indicates that the PW of each plan is a nonlinear function of hours of operation. Plan A is<br />

Figure 18–4<br />

Sample PW sensitivity to<br />

hours of operation for two<br />

alternatives.<br />

150<br />

Plan B<br />

PW, $ 1000<br />

100<br />

Plan A<br />

50<br />

0 1000 2000 3000<br />

Hours of operation per year


18.1 Determining Sensitivity to Parameter Variation 489<br />

very sensitive in the range of 0 to 2000 hours, but it is comparatively insensitive above<br />

2000 hours. Plan B is more attractive due to its relative insensitivity. The breakeven point is at<br />

about 1750 hours per year. It may be necessary to plot the measure of worth at intermediate<br />

points to better understand the nature of the sensitivity.<br />

EXAMPLE 18.2<br />

Columbus, Ohio needs to resurface a 3-kilometer stretch of highway. Knobel Construction has<br />

proposed two methods of resurfacing. The first method is a concrete surface for a cost of<br />

$1.5 million and an annual maintenance cost of $10,000. The second method is an asphalt<br />

covering with a first cost of $1 million and a yearly maintenance of $50,000. However, Knobel<br />

requests that every third year the asphalt highway be touched up at a cost of $75,000.<br />

The city uses the interest rate on bonds, 6% on its last bond issue, as the discount rate.<br />

(a) Determine the breakeven number of years of the two methods. If the city expects an interstate<br />

to replace this stretch of highway in 10 years, which method should be selected?<br />

(b) If the touch-up cost increases by $5000 per kilometer every 3 years, is the decision sensitive<br />

to this increase?<br />

Solution<br />

(a) Use PW analysis to determine the breakeven n value.<br />

PW of concrete PW of asphalt<br />

1,500,000 10,000(PA,6%,n) 1,000,000 50,000(PA,6%,n)<br />

75,000 [ (PF,6%, j )<br />

j<br />

]<br />

where j 3, 6, 9, . . . , n. The relation can be rewritten to reflect the incremental cash flows.<br />

500,000 40,000(PA,6%,n) 75,000 [ (PF,6%, j )<br />

j<br />

] 0 [18.1]<br />

The breakeven n value can be determined by hand solution by increasing n until Equation<br />

[18.1] switches from negative to positive PW values. Alternatively, a spreadsheet solution<br />

using the NPV function can find the breakeven n value (Figure 18–5). The NPV functions<br />

in column C are the same each year, except that the cash flows are extended 1 year for each<br />

Figure 18–5<br />

Sensitivity of the breakeven<br />

life between two alternatives,<br />

Example 18.2.<br />

PW for 11 years<br />

NPV(6%,$B$5:$B15)$B$4<br />

PW for 12 years<br />

NPV(6%,$B$5:$B16)$B$4<br />

Year counter advances by 1


490 Chapter 18 Sensitivity Analysis and Staged Decisions<br />

present worth calculation. At approximately n 11.4 years, concrete and asphalt resurfacing<br />

break even economically. Since the road is needed for 10 more years, the extra cost of<br />

concrete is not justified; select the asphalt alternative.<br />

(b) The total touch-up cost will increase by $15,000 every 3 years. Equation [18.1] is now<br />

500,000 40,000(PA,6%,n) [ 75,000 15,000 ( ———<br />

j 3<br />

3 ) ] [ <br />

j<br />

( PF,6%, j )<br />

] 0<br />

Now the breakeven n value is between 10 and 11 years—10.8 years using linear interpolation<br />

(Figure 18–5, column E). The decision has become marginal for asphalt, since the<br />

interstate is planned for 10 years hence.<br />

Noneconomic considerations may be used to determine if asphalt is still the better alternative.<br />

One conclusion is that the asphalt decision becomes more questionable as the<br />

asphalt alternative maintenance costs increase; that is, the PW value is sensitive to increasing<br />

touch-up costs.<br />

EXAMPLE 18.3<br />

18.2 Sensitivity Analysis Using Three Estimates<br />

We can thoroughly examine the economic advantages and disadvantages among two or more<br />

alternatives by borrowing from the field of project scheduling the concept of making three estimates<br />

for each parameter: a pessimistic, a most likely, and an optimistic estimate. Depending<br />

upon the nature of a parameter, the pessimistic estimate may be the lowest value (alternative life<br />

is an example) or the largest value (such as asset first cost).<br />

This approach allows us to study measure of worth and alternative selection sensitivity within a<br />

predicted range of variation for each parameter. Usually the most likely estimate is used for all other<br />

parameters when the measure of worth is calculated for one particular parameter or one alternative.<br />

An engineer is evaluating three alternatives for new equipment at Emerson Electronics. She<br />

has made three estimates for the salvage value, annual operating cost, and life. The estimates<br />

are presented on an alternative-by-alternative basis in Table 18–1. For example, alternative B<br />

has pessimistic estimates of S $500, AOC $4000, and n 2 years. The first costs are<br />

known, so they have the same value. Perform a sensitivity analysis and determine the most<br />

economical alternative, using AW analysis at a MARR of 12% per year.<br />

TABLE 18–1<br />

Competing Alternatives with Three Estimates Made for Salvage Value,<br />

AOC, and Life Parameters<br />

Strategy<br />

First Cost,<br />

$<br />

Salvage Value S,<br />

$<br />

AOC,<br />

$ per Year<br />

Alternative A<br />

P 20,000 0 11,000 3<br />

Estimates ML 20,000 0 9,000 5<br />

O 20,000 0 5,000 8<br />

Alternative B<br />

P 15,000 500 4,000 2<br />

Estimates ML 15,000 1,000 3,500 4<br />

O 15,000 2,000 2,000 7<br />

Alternative C<br />

P 30,000 3,000 8,000 3<br />

Estimates ML 30,000 3,000 7,000 7<br />

O 30,000 3,000 3,500 9<br />

P pessimistic; ML most likely; O optimistic.<br />

Life n,<br />

Years


18.3 Estimate Variability and the Expected Value 491<br />

TABLE 18–2 Annual Worth Values, Example 18.3<br />

Alternative AW Values, $<br />

Estimates A B C<br />

P 19,327 12,640 19,601<br />

ML 14,548 8,229 13,276<br />

O 9,026 5,089 8,927<br />

20<br />

18<br />

16<br />

AW of costs, $1000<br />

14<br />

12<br />

10<br />

8<br />

6<br />

Alternative A<br />

Alternative B<br />

Alternative C<br />

4<br />

2<br />

0<br />

1<br />

2<br />

3<br />

4<br />

5<br />

6<br />

7<br />

8<br />

9<br />

Life n, years<br />

Figure 18–6<br />

Plot of AW of costs for different-life estimates, Example 18.3.<br />

Solution<br />

For each alternative in Table 18–1, calculate the AW value of costs. For example, the AW relation<br />

for alternative A, pessimistic estimates, is<br />

AW 20,000(AP,12%,3) 11,000 $19,327<br />

Table 18–2 presents all AW values. Figure 18–6 is a plot of AW versus the three estimates of<br />

life for each alternative. Since the AW calculated using the ML estimates for alternative B<br />

($−8229) is economically better than even the optimistic AW value for alternatives A and C,<br />

alternative B is clearly favored.<br />

Comment<br />

While the alternative that should be selected here is quite obvious, this is not normally the case.<br />

For example, in Table 18–2, if the pessimistic alternative B equivalent AW were much higher,<br />

say, $21,000 per year (rather than $12,640), and the optimistic AW values for alternatives A<br />

and C were less than that for B ($−5089), the choice of B would not be apparent or correct. In<br />

this case, it would be necessary to select one set of estimates (P, ML, or O) upon which to base<br />

the decision. Alternatively, the different estimates can be used in an expected value analysis,<br />

which is introduced next.<br />

18.3 Estimate Variability and the Expected Value<br />

Engineers and economic analysts usually deal with estimate variation and risk about an uncertain<br />

future by placing appropriate reliance on past data, if any exist. This means that probability and<br />

samples are used. Actually the use of probabilistic analysis is not as common as might be expected.<br />

The reason is not that the computations are difficult to perform or understand, but that<br />

realistic probabilities associated with cash flow estimates are difficult to assign. Experience and


492 Chapter 18 Sensitivity Analysis and Staged Decisions<br />

judgment can often be used in conjunction with probabilities and expected values to evaluate the<br />

desirability of an alternative.<br />

The expected value can be interpreted as a long-run average observable if the project is repeated<br />

many times. Since a particular alternative is evaluated or implemented only once, the expected<br />

value results in a point estimate. However, even for a single occurrence, the expected value is a<br />

meaningful number.<br />

The expected value E ( X ) is computed using the relation<br />

im<br />

E(X ) X i P(X i ) [18.2]<br />

i1<br />

where<br />

X i value of the variable X for i from 1 to m different values<br />

P ( X i ) probability that a specific value of X will occur<br />

Probabilities are always correctly stated in decimal form, but they are routinely spoken of in<br />

percentages and often referred to as chance , such as the chances are about 10%. When placing<br />

the probability value in Equation [18.2] or any other relation, use the decimal equivalent of 10%,<br />

that is, 0.1. In all probability statements the P ( X i ) values for a variable X must total to 1.0.<br />

im<br />

<br />

i1<br />

P(X i ) 1.0<br />

We may frequently omit the subscript i on X for simplicity.<br />

If X represents the estimated cash flows, some will be positive and others will be negative. If<br />

a cash flow sequence includes revenues and costs, and the measure of worth is present worth<br />

calculated at the MARR, the result is the expected value of the discounted cash flows E (PW). If<br />

the expected value is negative, the overall outcome is expected to be a cash outflow. For example,<br />

if E (PW) $1500, this indicates that the proposal is not expected to return the MARR.<br />

EXAMPLE 18.4<br />

ANA airlines plans to offer several new electronic services on flights between Tokyo and selected<br />

European destinations. The marketing director estimates that for a typical 24-hour period<br />

there is a 50% chance of having a net cash flow of $5000 and a 35% chance of $10,000.<br />

He also estimates there is a small 5% chance of no cash flow and a 10% chance of a loss of<br />

$1000, which is the estimated extra personnel and utility costs to offer the services. Determine<br />

the expected net cash flow.<br />

Solution<br />

Let NCF be the net cash flow in dollars, and let P (NCF) represent the associated probabilities.<br />

Using Equation [18.2],<br />

E (NCF) 5000(0.5) 10,000(0.35) 0(0.05) 1000(0.1) $5900<br />

Although the “no cash flow” possibility does not increase or decrease E (NCF), it is included<br />

because it makes the probability values sum to 1.0 and it makes the computation complete.<br />

18.4 Expected Value Computations for Alternatives<br />

The expected value computation E ( X ) is utilized in a variety of ways. Two prime ways are to:<br />

• Prepare information for use in an economic analysis.<br />

• Evaluate the expected viability of a fully formulated alternative.<br />

Example 18.5 illustrates the first situation, and Example 18.6 determines the expected PW when<br />

the entire cash flow series and its probabilities are estimated.


18.4 Expected Value Computations for Alternatives 493<br />

EXAMPLE 18.5<br />

There are many government incentives to become more energy-efficient. Installing solar<br />

panels on homes, business buildings, and multiple-family dwellings is one of them. The<br />

owner pays a portion of the total installation costs, and the government agency pays the<br />

rest. Nichole works for the Department of Energy and is responsible for approving solar<br />

panel incentive payouts. She has exceeded the annual budgeted amount of $50 million per<br />

year in each of the previous 2 years. Disappointed with this situation, Nichole and her boss<br />

decided to collect data to determine what size increase in annual budget the incentive program<br />

needs in the future. Over the last 36 months, the amount of average monthly payout<br />

and number of months are shown in Table 18–3 . She categorized by level the monthly<br />

averages according to her experience with the program. Provided the same pattern continues,<br />

what is the expected value of the dollar increase in annual budget that is needed to<br />

meet the requests?<br />

TABLE 18–3 Solar Panel Incentive Payouts, Example 18.5<br />

Level<br />

Average Payout,<br />

$ Million per Month<br />

Months over<br />

Past 3 Years<br />

Very high 6.5 15<br />

High 4.7 10<br />

Moderate 3.2 7<br />

Low 2.9 4<br />

Solution<br />

Use the 36 months of payouts PO j ( j low, . . . ,very high) to estimate the probability P (PO j )<br />

for each level, and make sure the total is 1.0.<br />

Level, j Probability of Payout Level, P (PO j )<br />

Very high<br />

High<br />

P (PO 1 ) 1536 0.417<br />

P (PO 2 ) 1036 0.278<br />

Moderate P (PO 3 ) 736 0.194<br />

Low P (PO 4 ) 436 0.111<br />

1.000<br />

The expected monthly payout is calculated using Equation [18.2]. In $ million units,<br />

E [PO] 6.5(0.417) 4.7(0.278) 3.2(0.194) 2.9(0.111)<br />

2.711 1.307 0.621 0.322<br />

$4.961 ($4,961,000)<br />

The annual expected budget need is 12 4.961 million $59.532 million. The current budget<br />

of $50 million should be increased by an average of $9.532 million per year.<br />

EXAMPLE 18.6<br />

Lite-Weight Wheelchair Company has a substantial investment in tubular steel bending equipment.<br />

A new piece of equipment costs $5000 and has a life of 3 years. Estimated cash flows<br />

( Table 18–4 ) depend on economic conditions classified as receding, stable, or expanding. A<br />

probability is estimated that each of the economic conditions will prevail during the 3-year<br />

period. Apply expected value and PW analysis to determine if the equipment should be purchased.<br />

Use a MARR of 15% per year.


494 Chapter 18 Sensitivity Analysis and Staged Decisions<br />

TABLE 18–4 Equipment Cash Flow and Probabilities, Example 18.6<br />

Receding<br />

(Prob. 0.4)<br />

Solution<br />

First determine the PW of the cash flows in Table 18–4 for each economic condition, and then<br />

calculate E (PW) using Equation [18.2]. Define subscripts R for receding economy, S for stable,<br />

and E for expanding. The PW values for the three scenarios are<br />

PW R 5000 2500( PF ,15%,1) 2000( PF ,15%,2) 1000( PF ,15%,3)<br />

5000 4344 $656<br />

PW S 5000 5708 $708<br />

PW E 5000 6309 $1309<br />

Economic Condition<br />

Stable<br />

(Prob. 0.4)<br />

Year Annual Cash Flow Estimates, $<br />

Only in a receding economy will the cash flows not return the 15% to justify the investment.<br />

The expected present worth is<br />

E(PW) PW j [P(j)]<br />

jR,S,E<br />

656(0.4) 708(0.4) 1309(0.2)<br />

$283<br />

At 15%, E (PW) 0; the equipment is justified, using an expected value analysis.<br />

Comment<br />

It is also correct to calculate the E (cash flow) for each year and then determine PW of the<br />

E (cash flow) series, because the PW computation is a linear function of cash flows. Computing<br />

E (cash flow) first may be easier in that it reduces the number of PW computations. In this<br />

example, calculate E (CF t ) for each year, then determine E (PW).<br />

E (CF 0 ) $5000<br />

E (CF 1 ) 2500(0.4) 2500(0.4) 2000(0.2) $2400<br />

E (CF 2 ) $2400<br />

E (CF 3 ) $2100<br />

Expanding<br />

(Prob. 0.2)<br />

0 5000 5000 5000<br />

1 2500 2500 2000<br />

2 2000 2500 3000<br />

3 1000 2500 3500<br />

E (PW) 5000 2400( PF ,15%,1) 2400( PF ,15%,2) 2100( PF ,15%,3)<br />

$283<br />

18.5 Staged Evaluation of Alternatives<br />

Using a Decision Tree<br />

Alternative evaluation may require a series of decisions in which the outcome from one stage is important<br />

to the next stage of decision making. When each alternative is clearly defined and probability estimates<br />

can be made to account for risk , it is helpful to perform the evaluation using a decision tree .<br />

A decision tree includes:<br />

• More than one stage of alternative selection.<br />

• Selection of an alternative at one stage that leads to another stage.<br />

• Expected results from a decision at each stage.<br />

• Probability estimates for each outcome.<br />

• Estimates of economic value (cost or revenue) for each outcome.<br />

• Measure of worth as the selection criterion, such as E(PW).


18.5 Staged Evaluation of Alternatives Using a Decision Tree 495<br />

Decision<br />

node<br />

Alternatives<br />

D<br />

(a) Decision node<br />

Probability<br />

node<br />

Outcomes<br />

0.5<br />

0.2<br />

0.3<br />

Probabilities<br />

(b) Probability node with outcomes<br />

D<br />

D<br />

D<br />

Final outcomes<br />

(c) Tree structure<br />

Figure 18–7<br />

Decision and probability nodes used to construct a decision tree.<br />

The decision tree is constructed left to right and includes each possible decision and outcome.<br />

• A square represents a decision node with the possible alternatives indicated on the branches<br />

from the decision node ( Figure 18–7 a ).<br />

• A circle represents a probability node with the possible outcomes and estimated probabilities<br />

on the branches ( Figure 18–7 b ).<br />

• The treelike structure in Figure 18–7 c results, with outcomes following a decision.<br />

Usually each branch of a decision tree has some estimated economic value (often referred to<br />

as payoff ) in cost, revenue, saving, or benefit. These cash flows are expressed in terms of PW,<br />

AW, or FW values and are shown to the right of each final outcome branch. The cash flow and<br />

probability estimates on each outcome branch are used in calculating the expected economic<br />

value of each decision branch. This process, called solving the tree or rollback , is explained<br />

after Example 18.7 , which illustrates the construction of a decision tree.<br />

EXAMPLE 18.7<br />

Jerry Hill is president and CEO of a U.S.-based food processing company, Hill Products and<br />

Services. He was recently approached by an international supermarket chain that wants to<br />

market in-country its own brand of frozen microwaveable dinners. The offer made to Jerry by<br />

the supermarket corporation requires that a series of two decisions be made, now and 2 years<br />

hence. The current decision involves two alternatives: (1) Lease a facility in the United Arab<br />

Emirates (UAE) from the supermarket chain, which has agreed to convert a current processing<br />

facility for immediate use by Jerry’s company; or (2) build and own a processing and packaging<br />

facility in the UAE. Possible outcomes of this first decision stage are good market or poor<br />

market depending upon the public’s response.<br />

The decision choices 2 years hence are dependent upon the lease-or-own decision made<br />

now. If Hill decides to lease, good market response means that the future decision alternatives<br />

are to produce at twice, equal to, or one-half of the original volume. This will be a mutual decision<br />

between the supermarket chain and Jerry’s company. A poor market response will indicate


496 Chapter 18 Sensitivity Analysis and Staged Decisions<br />

a one-half level of production, or complete removal from the UAE market. Outcomes for the<br />

future decisions are, again, good and poor market responses.<br />

As agreed by the supermarket company, the current decision for Jerry to own the facility<br />

will allow him to set the production level 2 years hence. If market response is good, the decision<br />

alternatives are four or two times original levels. The reaction to poor market response<br />

will be production at the same level or no production at all.<br />

Construct the tree of decisions and outcomes for Hill Products and Services.<br />

Solution<br />

This is a two-stage decision tree that has alternatives now and 2 years hence. Identify the decision<br />

nodes and branches, and then develop the tree using the branches and the outcomes of<br />

good and poor market for each decision. Figure 18–8 details the decision stages and outcome<br />

branches.<br />

Stage 1 (decision now):<br />

Label it D1.<br />

Alternatives: lease (L) and own (O).<br />

Outcomes: good and poor markets.<br />

Good<br />

Good<br />

D2<br />

2<br />

1<br />

0.5<br />

Poor<br />

Good<br />

Poor<br />

Good<br />

Poor<br />

Poor<br />

D3<br />

0.5<br />

0<br />

Good<br />

Poor<br />

Out of<br />

business<br />

Lease<br />

(L)<br />

D1<br />

Own<br />

(O)<br />

Good<br />

D4<br />

4<br />

2<br />

Good<br />

Poor<br />

Good<br />

Poor<br />

Poor<br />

Figure 18–8<br />

A two-stage decision tree identifying alternatives and possible outcomes.<br />

D5<br />

1<br />

0<br />

Good<br />

Poor<br />

Out of<br />

business


18.5 Staged Evaluation of Alternatives Using a Decision Tree 497<br />

Stage 2 (Decisions 2 years hence):<br />

Label them D2 through D5.<br />

Outcomes: good market, poor market, and out of business.<br />

Choice of production levels for D2 through D5:<br />

Quadruple production (4); double production (2); level production<br />

(1 ); one-half production (0.5); stop production (0)<br />

The alternatives for future production levels (D2 through D5) are added to the tree and followed<br />

by the market responses of good and poor. If the stop-production (0) decision is made<br />

at D3 or D5, the only outcome is out of business.<br />

To utilize the decision tree for alternative evaluation and selection, the following additional<br />

information is necessary for each branch:<br />

• The estimated probability that each outcome may occur. These probabilities must sum to 1.0<br />

for each set of outcomes (branches) that result from a decision.<br />

• Economic information for each decision alternative and possible outcome, such as initial<br />

investment and estimated cash flows.<br />

Decisions are made using the probability estimate and economic value estimate for each outcome<br />

branch. Commonly the present worth at the MARR is used in an expected value computation<br />

of the type in Equation [18.2]. This is the general procedure to solve the tree using PW<br />

analysis:<br />

1. Start at the top right of the tree. Determine the PW value for each outcome branch considering<br />

the time value of money.<br />

2. Calculate the expected value for each decision alternative.<br />

E(decision) (outcome estimate)P(outcome) [18.3]<br />

where the summation is taken over all possible outcomes for each decision alternative.<br />

3. At each decision node, select the best E (decision) value—minimum cost or maximum value<br />

(if both costs and revenues are estimated).<br />

4. Continue moving to the left of the tree to the root decision in order to select the best alternative.<br />

5. Trace the best decision path through the tree.<br />

EXAMPLE 18.8<br />

A decision is needed to either market or sell a new invention. If the product is marketed, the<br />

next decision is to take it international or national. Assume the details of the outcome branches<br />

result in the decision tree of Figure 18–9 . The probabilities for each outcome and PW of CFBT<br />

(cash flow before taxes) are indicated. These payoffs are in millions of dollars. Determine the<br />

best decision at the decision node D1.<br />

Solution<br />

Use the procedure above to determine that the D1 decision alternative to sell the invention<br />

should maximize E (PW of CFBT).<br />

1. Present worth of CFBT is supplied.<br />

2. Calculate the expected PW for alternatives from nodes D2 and D3, using Equation [18.3].<br />

In Figure 18–9 , to the right of decision node D2, the expected values of 14 and 0.2 in ovals<br />

are determined as<br />

E (international decision) 12(0.5) 16(0.5) 14<br />

E (national decision) 4(0.4) 3(0.4) 1(0.2) 0.2<br />

The expected PW values of 4.2 and 2 for D3 are calculated in a similar fashion.


498 Chapter 18 Sensitivity Analysis and Staged Decisions<br />

Market<br />

9<br />

D1<br />

High<br />

6.16 0.2<br />

0.8<br />

Low<br />

14<br />

D2<br />

4.2<br />

D3<br />

International<br />

National<br />

International<br />

National<br />

14<br />

0.2<br />

4.2<br />

2<br />

0.5<br />

0.5<br />

0.4<br />

0.4<br />

0.2<br />

0.8<br />

0.2<br />

0.4<br />

0.4<br />

0.2<br />

PW of<br />

CFBT<br />

($ million)<br />

12<br />

16<br />

4<br />

–3<br />

–1<br />

6<br />

–3<br />

6<br />

–2<br />

2<br />

Sell<br />

9<br />

1.0<br />

9<br />

Expected values for each<br />

decision alternative<br />

Figure 18–9<br />

Solution of a decision tree with present worth of estimated CFBT values,<br />

Example 18.8 .<br />

3. Select the larger expected value at each decision node. These are 14 (international) at D2<br />

and 4.2 (international) at D3.<br />

4. Calculate the expected PW for the two D1 branches.<br />

E (market decision) 14(0.2) 4.2(0.8) 6.16<br />

E (sell decision) 9(1.0) 9<br />

The expected value for the sell decision is simple since the one outcome has a payoff of 9.<br />

The sell decision yields the larger expected PW of 9.<br />

5. The largest expected PW of CFBT path is to select the sell branch at D1 for a guaranteed<br />

$9,000,000.<br />

18.6 Real Options in <strong>Engineering</strong> Economics<br />

As we learned with decision trees, many of the problems in engineering economy can be viewed<br />

as staged decisions. When the decision to invest more or less can be delayed into the future, the<br />

problem is called staged funding. As an illustration, assume a large company expects to sell an<br />

energy-saving, window-mounted residential air conditioning unit at the rate of 100,000 per<br />

month by the end of 2 years on the market. Decision makers may opt to (1) build the capacity to<br />

supply100,000 per month to market immediately or (2) build capacity to supply 25,000 per<br />

month now and test the market’s receptivity. If positive, they can stage the increase by 25,000<br />

additional units each 6 months to meet current demand. Of course, if an aggressive competitor<br />

enters the scene, or the economy falters, the staged funding decision will change as warranted.<br />

These alternatives provide time-based options to the company. Before we go further, some definitions<br />

are needed.<br />

An option is a purchase or investment that contractually provides the privilege to take a<br />

stated action by some stated time in the future, or the right to not accept the offer and forfeit<br />

the option.


18.6 Real Options in <strong>Engineering</strong> Economics 499<br />

A real option, in engineering economy terms, is the investment (cost) in a project, process, or<br />

system. The options usually involve physical (real) assets, buildings, equipment, materials, and<br />

the like, thus, the word real. Options may also be leases, subcontracts, or franchises. The investment<br />

alternatives present varying amounts of risk, which is estimated by probabilities of occurrence<br />

for predictable future events.<br />

Real options analysis is the application of techniques to determine the economic consequences<br />

of delaying the funding decisions as allowed by the option. The estimated cash flows and other<br />

consequences of these delays are analyzed with risk taken into account to the degree possible. A<br />

measure of worth, e.g., PW or AW, is the criterion used to make the staged funding decisions. A<br />

decision may be to expand, continue as is, contract, abandon, or replicate the alternative at the<br />

time the option must be exercised.<br />

An inherent part of real options analysis is the uncertainty of future estimates, as it is for most<br />

economic analyses. After some illustrations of real options, we will discuss the probabilistic<br />

dimensions. Samples from industry and everyday personal life that can be formulated as real<br />

o ptions follow.<br />

Industrial Setting<br />

New markets—Purchase equipment and staff to enter an expanding international market over<br />

the next 5 years.<br />

New planes—Purchase commercial airplanes now with an option to buy an additional 5 planes<br />

over the next 3 years at the same price as that paid for the current order.<br />

Removing car models—Ford Motor Company can decide to maintain production on an established<br />

car model with dwindling sales for the next 3 years or can opt to discontinue the model<br />

in stages over a 1- or 2-year period.<br />

Drilling lease—Buy a drilling option contract from landowners to drill for oil and gas at some<br />

time in the next 10 years. The drilling may not be justified at this time, but the contract offers<br />

the option to drill were it to become economically advantageous based on events such as increased<br />

oil prices or improved recovery technology.<br />

Personal Decision Making<br />

Extended car warranty—When purchasing a new car, the option to buy an extended-coverage<br />

warranty beyond the manufacturer’s warranty is always an option. The price of the option is<br />

the cost of the extended warranty. The uncertainties and risks are the future unknown costs for<br />

repairs and failed components.<br />

House insurance—When a homeowner has no mortgage to pay, maintaining house insurance<br />

is an option. Deductibles are high enough, e.g., 1% to 5% of the fully appraised value, that<br />

insurance primarily covers only catastrophic damage to the structure. Self-insurance, where<br />

money is set aside for potential damages while accepting the risk that a major event will take<br />

place, is an option for the homeowner.<br />

Some of the primary characteristics (with an example) of a real options analysis performed within<br />

the context of engineering economics are as follows:<br />

• Cost to obtain the option to delay a decision (PW of initial investment, lease cost, or future<br />

investment amount).<br />

• Anticipated future options and cash flow estimates (double production with annual net cash<br />

flows estimated).<br />

• Time period for follow-on decisions (staged decision time, such as 1 year or a 3-year test period).<br />

• Market and risk-free interest rates (expected market MARR of 12% per year and inflation<br />

rate estimate of 4% per year).<br />

• Estimates of risk and future uncertainty for each option (probability that an estimated cash<br />

flow series will actually occur, if a specific option is selected).<br />

• Economic criterion used to make a decision (PW, ROR, or other measure of worth).


500 Chapter 18 Sensitivity Analysis and Staged Decisions<br />

It is common to use a decision tree to record and understand the options prior to performing a real<br />

options analysis with risk included. Example 18.9 demonstrates the use of a decision tree and PW<br />

analysis.<br />

EXAMPLE 18.9<br />

A start-up company in the solar energy production business, SolarScale Energy, Inc., has<br />

developed and field-tested a modularized, scalar solar thermal electric (STE) generation<br />

system that is relatively inexpensive to purchase and has an efficiency considerably better<br />

than traditional photovoltaic (PV) panels. The technology is promising enough that Capital<br />

Investor Funds (CIF) has provided $10 million for manufacturing. Additionally, a contract<br />

with a consortium of sunbelt states has been offered, but not accepted thus far, for a total of<br />

$1.5 million per year for a 2-year test period. By contract, the units will be marketed through<br />

the state energy departments with all revenue going to the state treasuries. The lead engineer<br />

at SolarScale, the manager of CIF, and a conservation representative for the state<br />

consortium have developed the following staged-funding options, based on the delayed<br />

decision to increase manufacturing production level until preliminary results of the 2-year<br />

contract are in hand.<br />

Condition Option CIF Funding Consortium Contract<br />

Sales are excellent<br />

( 5000 unitsyear)<br />

2 production<br />

level<br />

Additional $10 million in<br />

year 2<br />

Additional 8 years;<br />

$4 million in years 3–10<br />

Sales are excellent<br />

(3000–5000 unitsyear)<br />

1 production<br />

level<br />

Nothing; no salvage after<br />

10 years<br />

Additional 8 years;<br />

$1.5 million in<br />

years 3–10<br />

Additional 3 years;<br />

$1.5 million in<br />

years 3–5<br />

Nothing<br />

Sales are poor<br />

(2000–3000 unitsyear)<br />

Reduce to ½ <br />

production<br />

Nothing; sell for<br />

$2.5 million after 5 years<br />

Sales are poor<br />

( 2000 unitsyear)<br />

Stop after<br />

2 years<br />

Nothing; sell for<br />

$5 million after 2 years<br />

(a) Develop the two-stage decision tree for the options described.<br />

(b) The base case is the 1 production level with the 8-year follow-on contract from the consortium.<br />

If the estimates for this option are considered the most likely (expected value)<br />

estimates, determine the present worth at a MARR of 10% per year.<br />

(c) Determine the PW values for each possible final outcome at 10% per year, and identify the<br />

best economic option when the stage 2 funding decision must be made.<br />

Solution<br />

( a) Figure 18–10 details the options with the year shown at the bottom. There are 2 outcome<br />

branches initially (accept option; decline option) and four final branches for the<br />

accept decision at D1, based upon sales level. The decline option has a $0 outcome.<br />

(SolarScale has other ways to pursue revenue that are not represented in this abbreviated<br />

example.)<br />

(b) Perform a PW evaluation as we have in all previous chapters, assuming the estimates are<br />

point estimates over the 10-year life of the project. The resulting PW 1 0 as shown<br />

below indicates the contract is not justified economically. In $ millions,<br />

PW 1 10 1.5( PA ,10%,10)<br />

$0.78 ($780,000)<br />

(c) Figure 18–11 is a spreadsheet screen shot that calculates the PW for each option using the<br />

NPV function. The i * values are also shown using the IRR function. Note that the sale of<br />

production assets after 5 years for $2.5 million (½ level) or after 2 years for $5 million<br />

(stop) is included. Also, the extra $10 million investment in year 2 for the 2 level option<br />

is i ncluded.


18.6 Real Options in <strong>Engineering</strong> Economics 501<br />

> 5000<br />

2<br />

Sales<br />

excellent<br />

D2<br />

3000–5000<br />

1<br />

D1<br />

Accept<br />

option<br />

Sales<br />

poor<br />

D3<br />

1/2<br />

2000–3000<br />

< 2000<br />

Stop<br />

now<br />

Decline<br />

option<br />

No<br />

sales<br />

Now<br />

Year 2<br />

Year 5 Year 10<br />

Time<br />

Figure 18–10<br />

Decision tree showing real options over 10-year period, Example 18.9 .<br />

Sale of assets in this year<br />

Figure 18–11<br />

PW analysis of real options without risk considered, Example 18.9 .<br />

Only the 2 production-level option is justified at MARR 10% per year. If SolarScale<br />

and CIF, the financial backers, are not convinced that the sales level will exceed<br />

5000 units per year, the contract option should be declined. Some marketing survey information<br />

and risk analysis may be very helpful before this important decision is made.<br />

Of the characteristics listed above for real option situations, the primary one absent in<br />

Example 18.9 is that of estimate variation and some measure of risk. Decision making under<br />

risk is covered more extensively in the next chapter; however, we can use the following definition<br />

for this discussion of real options analysis.<br />

When a parameter can take on more than one value and there is any estimate of chance or<br />

probability about the opportunity that each value may be observed, risk is present.<br />

Risk


502 Chapter 18 Sensitivity Analysis and Staged Decisions<br />

EXAMPLE 18.10<br />

A coin has two sides. If it is a perfectly balanced coin, a flip of the coin should result in heads 50%<br />

of the time and tails 50% of the time. If the coin is intentionally biased in weight, such that 58% of<br />

the time it lands heads up, then the long-run probability of heads is P (heads) 0.58. Since the sum<br />

of probabilities across all possible values must add to 1, the biased coin has P (tails) 0.42.<br />

There are a couple of points worth mentioning about risk and the calculated PW values for real<br />

options analysis.<br />

• When the risk is higher and the stakes are larger, the real options analysis is often more valuable.<br />

(We shall see this in Example 18.10 .)<br />

• Since one of the objectives of real options analysis is to evaluate the economic consequences<br />

of delaying a decision, a PW value of the base case that is moderately positive means that the<br />

project is justified and should be accepted immediately, without the decision delay. On the<br />

other hand, if the PW is largely negative , the delay is likely not worthwhile, as it would take<br />

a very large positive PW to result in E (PW) 0. Thus, the project should be rejected now, not<br />

delayed for a future decision.<br />

We return to the previous example with some risk assessment added to determine if the contract<br />

option should be declined, as indicated by the base case.<br />

Before the stage 1 decision is made in Example 18.9 about SolarScale’s state-consortium contract<br />

offer, some expected sales information was collected. Once the results were reviewed by<br />

the three individuals in attendance at the final decision-making meeting—one representative<br />

each for SolarScale, CIF, and the consortium—each person recorded her or his estimated probability<br />

as a measure of risk that the sales would be excellent or poor, which represents the two<br />

outcome possibilities were the option accepted. The results are as follows:<br />

Probability of Outcome<br />

Excellent<br />

Poor<br />

SolarScale 0.5 0.5<br />

CIF 0.8 0.2<br />

Consortium 0.6 0.4<br />

Use these probability estimates to determine the expected PW value, provided equal weighting<br />

is given to each representative’s input.<br />

Solution<br />

For each outcome (excellent and poor), select the best PW value from Figure 18–11 , and then<br />

find E (PW) for each representative.<br />

Excellent: From 2 level and 1 level, select 2 with PW $1.97 million.<br />

Poor: From ½ level and stop now, select ½ with PW $2.76 million.<br />

In $ million, E (PW) for each organization is<br />

E (PW for SolarScale) 1.97(0.5) 2.76(0.5) $0.40<br />

E (PW for CIF) 1.97(0.8) 2.76(0.2) $1.02<br />

E (PW for consortium) 1.97(0.6) 2.76(0.4) $0.08<br />

With a 1/3 chance assigned to each representative, the overall E (PW of stage 2 decision) is<br />

E (PW of stage 2 decision) 0.33(0.40 1.02 0.08)<br />

$0.23 ($230,000)<br />

The base case of 1 level production in part ( b ) of Example 18.9 resulted in E (PW) <br />

$−780,000. When compared with the positive E (PW) result here, we see that with consideration<br />

of the different options of production level and probabilities for sales level, the expected<br />

PW has increased to a positive value. All other things being equal, the state consortium offer<br />

should be accepted; that is, accept the real option of the contract.


There are many other examples and dimensions of real options analysis in engineering economics<br />

and in the area of financial analysis, where options analysis got its start some years ago.<br />

If you are interested in this new and interesting area of analysis, consult more advanced texts and<br />

journal articles on the topic of real options.<br />

Problems 503<br />

CHAPTER SUMMARY<br />

In this chapter the emphasis is on sensitivity to variation in one or more parameters using a<br />

specific measure of worth. When two alternatives are compared, compute and graph the<br />

measure of worth for different values of the parameter to determine when each alternative is<br />

better.<br />

When several parameters are expected to vary over a predictable range, the measure of worth<br />

is plotted and calculated using three estimates for a parameter—most likely, pessimistic, and<br />

optimistic. This approach can help determine which alternative is best among several. Independence<br />

between parameters is assumed in all these analyses.<br />

The combination of parameter and probability estimates results in the expected value relation<br />

E ( X ) XP ( X )<br />

This expression is also used to calculate E (revenue), E (cost), E (cash flow), and E (PW) for the<br />

entire cash flow sequence of an alternative.<br />

Decision trees are used to make a series of alternative selections. This is a way to explicitly<br />

take risk into account. It is necessary to make several types of estimates for a decision tree: outcomes<br />

for each possible decision, cash flows, and probabilities. Expected value computations are<br />

coupled with those for the measure of worth to solve the tree and find the best alternatives stage<br />

by stage.<br />

Staged funding over time can be approached using the evolving area of real options. Delaying<br />

an investment decision and considering the risks of the future can improve the overall E (PW) of<br />

a project, process, or system.<br />

PROBLEMS<br />

Sensitivity to Parameter Variation<br />

18.1 Kahn Instruments is considering an investment of<br />

$500,000 in a new product line. The company will<br />

make the investment only if it will result in a rate<br />

of return of 15% per year or higher. If the revenue<br />

is expected to be between $135,000 and $165,000<br />

per year for 5 years, determine if the decision to<br />

invest is sensitive to the projected range of income<br />

using a present worth analysis.<br />

18.2 A young couple planning ahead for their retirement<br />

has decided that $2,600,000 is the amount<br />

they will need to retire comfortably 20 years<br />

from now. For the past 5 years they have been<br />

able to invest one of their salaries ($50,000 per<br />

year, which includes employer contributions)<br />

while living off the other one. They plan to start<br />

a family sometime in the next 10 years, and<br />

when they have their first child, one of the parents<br />

will quit working, causing the savings to<br />

decrease to $15,000 per year thereafter. If they<br />

have gotten a rate of return of 10% per year on<br />

their investments and expect to continue at this<br />

ROR, is reaching their goal of $2.6 million<br />

20 years from now sensitive to when they have<br />

their first child (i.e., between now and 10 years<br />

from now)? Use an FW analysis.<br />

18.3 A company that manufactures high-speed submersible<br />

rotary indexing spindles is considering<br />

upgrading the production equipment to reduce<br />

costs over a 6-year planning horizon. The company<br />

can invest $80,000 now, 1 year from now,<br />

or 2 years from now. Depending on when the investment<br />

is made, the savings will vary. That is,<br />

the savings will be $25,000, $26,000, or $29,000<br />

per year if the investment is made now (year 0),<br />

in 1 year, or in 2 years, respectively. Will the<br />

timing of the investment affect the request to<br />

make at least a 20% per year return? Use future<br />

worth analysis.<br />

18.4 Different membrane systems are under consideration<br />

for treating 3 million gallons per day (MGD)<br />

of cooling tower blowdown water to reduce its


504 Chapter 18 Sensitivity Analysis and Staged Decisions<br />

volume. Option 1 is a low-pressure seawater reverse<br />

osmosis (SWRO) system that will operate at<br />

500 psi with a fixed cost of $465 per day and an<br />

operating cost of $0.67 per 1000 gallons. A second<br />

option is a higher-pressure SWRO system offered<br />

by vendor X that operates at 800 psi and will have<br />

a lower fixed cost of $328 per day (because of<br />

fewer membranes); however, its operating cost<br />

will be $1.35 per 1000 gallons. A third option is<br />

also a high-pressure SWRO system from vendor Y,<br />

who claims that its system will have a lower operating<br />

cost of $1.28 per 1000 gallons and the same<br />

fixed cost as that of vendor X. Determine if the<br />

selection of a low- or high- pressure system is dependent<br />

on the lower operating cost offered by<br />

vendor Y.<br />

18.5 A machine that is currently used in manufacturing<br />

circuit board card locks has AW $–63,000 per<br />

year. A possible replacement is under consideration<br />

with a first cost of $64,000 and an operating<br />

cost of $38,000 per year for the next 3 years. Three<br />

different engineers have given their opinion, about<br />

what the salvage value of the new machine will be<br />

3 years from now: $10,000, $13,000, and $18,000.<br />

Is the decision to replace the machine sensitive to<br />

the salvage value estimates at the company’s<br />

MARR of 15% per year?<br />

18.6 An equipment alternative is being economically<br />

evaluated separately by three engineers at<br />

Raytheon. The first cost will be $77,000, and the<br />

life is estimated at 6 years with a salvage value of<br />

$10,000. The engineers disagree, however, on the<br />

estimated revenue the equipment will generate.<br />

Joe has made an estimate of $10,000 per year. Jane<br />

states that this is too low and estimates $14,000,<br />

while Carlos estimates $18,000 per year. If the<br />

before-tax MARR is 8% per year, use PW to determine<br />

if these different estimates will change the<br />

decision to purchase the equipment.<br />

18.7 The owner of a small construction company is<br />

planning to purchase specialized equipment to<br />

complete a contract he just received. The first<br />

cost of the equipment is $250,000, and it will<br />

likely have a salvage value of $90,000 in 3 years,<br />

at which time he will not need the equipment<br />

anymore. The operating cost is expected to be<br />

$75,000 per year. Alternatively, the owner can<br />

subcontract the work for $175,000 per year. Because<br />

the equipment is specialized, the owner is<br />

not sure about the salvage value. He thinks it<br />

might be worth as little as $10,000 in 3 years<br />

(a scrap value). If his minimum attractive rate of<br />

return is 15% per year, determine if the decision<br />

to buy the equipment is sensitive to the salvage<br />

value.<br />

18.8 A company planning to borrow $10.5 million for a<br />

plant expansion is not sure what the interest rate<br />

will be when it applies for the loan. The rate could<br />

be as low as 10% per year or as high as 12% per<br />

year for a 5-year loan. The company will only<br />

move forward with the project if the annual worth<br />

of the expansion is below $5.7 million. The M&O<br />

cost is fixed at $3.1 million per year. The salvage<br />

could be $2 million if the interest rate is 10% or<br />

$2.5 million if it is 12% per year. Is the decision to<br />

move forward with the project sensitive to the interest<br />

rate and salvage value estimates?<br />

18.9 A company that manufactures clear PVC pipe is<br />

investigating the production options of batch and<br />

continuous processes. Estimated cash flows are as<br />

follows:<br />

Batch<br />

Continuous<br />

First cost, $ 80,000 130,000<br />

Annual cost, $ per year 55,000 30,000<br />

Salvage value for any year, $ 10,000 40,000<br />

Life, years 3–10 5<br />

The chief operating officer (COO) has asked you<br />

to determine if the batch option would ever have a<br />

lower annual worth than the continuous flow system,<br />

using interest rates over a range of 5% to 15%<br />

for the batch option but only 15% for the continuous<br />

flow system. ( Note : The continuous flow process<br />

was previously determined to have its lowest<br />

cost over a 5-year life cycle; the batch process can<br />

be used from 3 to 10 years.)<br />

18.10 An engineer collected average cost and revenue data<br />

for Arenson’s FC1 handheld financial calculator.<br />

Fixed cost $300,000 per year<br />

Cost per unit $40<br />

Revenue per unit $70<br />

( a) What is the range in breakeven quantity if<br />

there is possible variation in the fixed cost<br />

from $200,000 to $400,000 per year? (Use<br />

$50,000 increments.)<br />

( b) What is the incremental change in the breakeven<br />

quantity for each $50,000 change in<br />

fixed cost?<br />

The following information is used for Problems 18.11<br />

through 18.14.<br />

A new online patient diagnostics system for surgeons will<br />

cost $200,000 to install, cost $5000 annually to maintain<br />

and will have an expected life of 5 years. The added revenue<br />

is estimated to be $60,000 per year, and the MARR<br />

is 10% per year. Examine the sensitivity of present worth<br />

to variation in selected parameter estimates, while others<br />

remain constant.


Problems 505<br />

18.11 Sensitivity to first cost variation: $150,000 to<br />

$250,000 (−25% to 25%).<br />

18.12 Sensitivity to revenue variation: $45,000 to $75,000<br />

(−25% to 25%).<br />

18.13 Sensitivity to life variation: 4 years to 7 years (−20%<br />

to 40%).<br />

18.14 Plot the results on a graph similar to Figure 18–3<br />

and comment on the relative sensitivity of each<br />

parameter.<br />

18.15 Charlene plans to place an annual savings<br />

amount of A $27,185 into a retirement program<br />

at the end of each year for 20 years starting<br />

next year. She expects to retire and start to draw<br />

a total of R $60,000 per year 1 year after the<br />

20th deposit. Assume an effective earning rate<br />

of i 6% per year on the retirement investments<br />

and an infinite life. Determine and comment on<br />

the sensitivity of the size of the annual withdrawal<br />

R for variations in A and i. Show hand<br />

and spreadsheet solutions.<br />

( a) Variation of 5% in the annual deposit A.<br />

(b) Variation of 1% in the effective earning rate<br />

i , that is, ranging from 5% to 7% per year.<br />

18.16 Ned Thompson Labs performs tests on super<br />

alloys, titanium, aluminum, and most metals. Tests<br />

on metal composites that rely upon scanning electron<br />

microscope results can be subcontracted, or<br />

the labs can purchase new equipment. Evaluate the<br />

sensitivity of the economic decision to purchase<br />

the equipment over a range of 20% (in 10% increments)<br />

of the estimates for P , AOC, R , n , and<br />

MARR (range on MARR is 12% to 16%). Use the<br />

AW method and plot the results on a sensitivity<br />

graph (like Figure 18–3 ). For which parameter(s)<br />

is the AW most sensitive? Least sensitive?<br />

First cost P $−220,000<br />

Salvage S $20,000<br />

Life n 10 years<br />

Annual operating cost AOC $−30,000/year<br />

Annual revenue R $70,000 per year<br />

MARR i 15% per year<br />

18.17 Titan manufactures and sells gas-powered electricity<br />

generators. It can purchase a new line of fuel<br />

injectors from either of two companies. Cost and<br />

savings estimates are made, but the savings estimate<br />

is unreliable at this time. Use an AW analysis<br />

at 10% per year to determine if the selection between<br />

company A and company B changes when<br />

the savings per year may vary as much as 40%<br />

from the best estimates made thus far.<br />

Company A<br />

Company B<br />

First cost, $ 50,000 37,500<br />

AOC, $ per year 7,500 8,000<br />

Savings best estimate, $ per year 15,000 13,000<br />

Salvage, $ 5,000 3,700<br />

Life, years 5 5<br />

18.18 ( a) Graph the sensitivity of what a person<br />

should be willing to pay now for a 9%,<br />

$10,000 bond due in 10 years if there is a<br />

30% change in (1) face value, (2) dividend<br />

rate, or (3) required nominal rate of return,<br />

which is expected to be 8% per year, compounded<br />

semiannually. The bond pays dividends<br />

semiannually.<br />

( b) If the investor did purchase the $10,000 face<br />

value bond at a premium of 5% (i.e., 5%<br />

above face value) and all your other estimates<br />

were correct, that is, 0% change, did<br />

he pay too much or too little? How much?<br />

Three Estimates<br />

18.19 DVH Technologies purchases several parts for<br />

the instruments it makes via a fixed-price contract<br />

of $190,000 per year from a local supplier.<br />

The company is considering making the parts<br />

in-house through the purchase of equipment that<br />

will have a first cost of $240,000 with an estimated<br />

salvage value of $30,000 after 5 years. The<br />

operating cost is difficult to estimate, but company<br />

engineers have made optimistic, most likely,<br />

and pessimistic estimates of $60,000, $85,000,<br />

and $120,000 per year, respectively. Determine if<br />

the company should purchase the equipment<br />

under any of the operating cost scenarios. The<br />

MARR is 20% per year.<br />

18.20 Astor <strong>Engineering</strong> recently merged with another<br />

firm and could lease additional office space or<br />

purchase its own building. The $30,000 per year<br />

lease agreement will be a net, net, net lease,<br />

which means that the lessee (Astor) will pay the<br />

real estate taxes on the leased space, the building<br />

insurance on the leased space, and the common<br />

area maintenance. Since these costs are about the<br />

same if Astor owned the building, they do not<br />

need to be considered in the analysis. A new<br />

building will cost $880,000 to purchase, but there<br />

is considerable uncertainty about what it will be<br />

worth in 20 years, which is the planning period<br />

selected. The individuals involved in the discussion<br />

made optimistic, most likely, and pessimistic<br />

estimates of $2,400,000, $1,400,000, and<br />

$900,000, respectively. Determine if Astor should<br />

purchase the building under any of the estimated<br />

resale values at i 10% per year.


506 Chapter 18 Sensitivity Analysis and Staged Decisions<br />

18.21 Holly Farms is considering two environmental<br />

chambers to accomplish detailed laboratory confirmations<br />

of online bacteria tests in chicken<br />

meat for the presence of E. coli 0157:H7 and Listeria<br />

monocytogenes . There is some uncertainty<br />

about how long the D103 chamber will be useful.<br />

A realistic estimate is 3 years, but pessimistic<br />

and optimistic estimates of 2 years and 6 years,<br />

respectively, are also possible. The estimated<br />

salvage value will remain the same. Using an interest<br />

rate of 10% per year, determine if any of<br />

the D103 estimates would result in a lower cost<br />

than that of the 490G chamber for a 6-year planning<br />

period.<br />

Chamber D103 Chamber 490G<br />

Installed cost, $ 400,000 250,000<br />

AOC, $ per year 4,000 3,000<br />

Salvage value at 10% of P , $ 40,000 25,000<br />

Life, years 2, 3, or 6 2<br />

18.22 When the country’s economy is expanding, AB Investment<br />

Company is optimistic and expects a<br />

MARR of 15% for new investments. However, in<br />

a receding economy the expected return is 8%.<br />

Normally a 10% return is required. An expanding<br />

economy causes the estimates of asset life to go<br />

down about 20%, and a receding economy makes<br />

the n values increase about 10%. Calculate and observe<br />

or plot the sensitivity of PW values versus<br />

( a ) the MARR and ( b ) the life values for the two<br />

plans detailed below, using the most likely estimates<br />

for the other factors. ( c ) Considering all the<br />

analyses, under which scenario, if any, should plan<br />

M or Q be rejected?<br />

Plan M<br />

Plan Q<br />

Initial investment, $ 100,000 110,000<br />

Cash flow, $ per year 15,000 19,000<br />

Life, years 20 20<br />

Expected Value<br />

18.23 Determine the expected net operating income<br />

(NOI) from sales of micro turbine components.<br />

The probabilities are 20%, 50%, and 30% for revenues<br />

of $800,000, $1,000,000, and $1,100,000<br />

per year, respectively, and operating expenses are<br />

constant at $200,000 per year.<br />

18.24 A company that manufactures amplified pressure<br />

transducers is trying to decide between a dualspeed<br />

and a variable-speed machine. The engineers<br />

are not sure about the salvage value of the<br />

variable-speed machine, so they have asked several<br />

different used-equipment dealers for<br />

estimates. The results can be summarized as follows:<br />

there is a 32% chance of getting $20,000, a<br />

45% chance of getting $28,000, and a 13%<br />

chance of getting $34,000. Also, there is a 10%<br />

chance that the company may have to pay $5000<br />

to dispose of the equipment. Calculate the<br />

expected salvage value.<br />

18.25 The average success probability for a wildcat oil<br />

well drilled in the Wind River basin 7 miles from<br />

the nearest existing production well is estimated<br />

to be 13%. If the value of the oil has equal<br />

chances of being $1.5 million, $1.9 million, and<br />

$2.4 million, what is the expected income from<br />

the well?<br />

18.26 Nationwide income from monthly sales data<br />

(rounded to the nearest $100,000) of Stay Flat<br />

vacuum hold-down tables for last year is shown<br />

below. Determine the expected value of the<br />

monthly income, if economic conditions remain<br />

the same.<br />

Income, $ per Month<br />

Number<br />

of Months<br />

500,000 4<br />

600,000 2<br />

700,000 1<br />

800,000 2<br />

900,000 3<br />

18.27 Determine the expected maximum rainfall intensity<br />

in El Paso, Texas for the month of July using<br />

the estimated probabilities shown.<br />

Rainfall Rate, inches per hour 3 4 5 6<br />

Probability 0.4 0.3 0.2 0.1<br />

18.28 There are four estimates made for the anticipated<br />

cycle time to produce a subcomponent. The estimates,<br />

in seconds, are 10, 20, 30, and 50. ( a ) If<br />

equal weight is placed on each estimate, what is<br />

the expected cycle time? ( b ) If the largest time is<br />

disregarded, what is the percent reduction in the<br />

expected time?<br />

18.29 The PW value for an alternative is expected to be<br />

one of two values based on bids from two vendors.<br />

Your office partner told you that the low bid is<br />

$3200 per year. If she indicates a chance of 70% of<br />

accepting the high bid and that her expected PW is<br />

$5875, what is the PW of the high bid?<br />

18.30 A total of 40 different proposals were evaluated by<br />

the IRAD (Industrial Research and Development)<br />

committee during the past year. Twenty were<br />

funded. Their rate of return estimates are summarized<br />

with the i * values rounded to the nearest


Problems 507<br />

integer. For the accepted proposals, calculate the<br />

expected rate of return E ( i ).<br />

Proposal<br />

ROR, i *%<br />

Number of<br />

Proposals<br />

8 1<br />

5 1<br />

0 5<br />

5 5<br />

8 2<br />

10 3<br />

15 3<br />

20<br />

18.31 Beckman Electronics has performed an economic<br />

analysis of proposed service in a new region of the<br />

country. The three-estimate approach to sensitivity<br />

analysis has been applied. The optimistic and pessimistic<br />

values each have an estimated 20% chance<br />

of occurring. Use the FW values shown to determine<br />

the expected FW.<br />

Optimistic Most Likely Pessimistic<br />

FW value, $ 300,000 50,000 25,000<br />

18.32 A very successful health and recreation club<br />

wants to construct a mock mountain for climbing<br />

and exercise outside for its customers’ use.<br />

Because of its location, there is a 30% chance of<br />

a 120-day season of good outdoor weather, a<br />

50% chance of a 150-day season, and a 20%<br />

chance of a 165-day season. The mountain will<br />

be used by an estimated 350 persons each day of<br />

the 4-month (120-day) season, but by only 100<br />

per day for each extra day the season lasts. The<br />

feature will cost $375,000 to construct and require<br />

a $25,000 rework each 4 years; and the annual<br />

maintenance and insurance costs will be<br />

$56,000. The climbing fee will be $5 per person.<br />

If a life of 10 years is anticipated and a 12% per<br />

year return is expected, determine if the addition<br />

is economically justified.<br />

18.33 The owner of Ace Roofing may invest $200,000 in<br />

new equipment. A life of 6 years and a salvage<br />

value of 12% of first cost are anticipated. The annual<br />

extra revenue will depend upon the state of<br />

the housing and construction industry. The extra<br />

revenue is expected to be only $20,000 per year if<br />

the current slump in the industry continues. Real<br />

estate economists estimate a 50% chance of the<br />

slump lasting 3 years and they give it a 20% chance<br />

of continuing for 3 additional years. However, if<br />

the depressed market does improve, during either<br />

the first or second 3-year period, the revenue of the<br />

investment is expected to increase by a total of<br />

$35,000 per year. Can the company expect to make<br />

a return of 8% per year on its investment? Use<br />

present worth analysis.<br />

18.34 A flagship hotel in Cedar Falls must construct a<br />

retaining wall next to its parking lot due to the<br />

widening of the city’s main thoroughfare located<br />

in front of the hotel. The amount of rainfall experienced<br />

in a short time may cause damage in varying<br />

amounts, and the wall increases in cost in<br />

order to protect against larger and faster rainfalls.<br />

The probabilities of a specific amount of rainfall<br />

in a 30-minute period and wall cost estimates are<br />

as follows:<br />

Rainfall, Inches<br />

per 30 Minutes<br />

Probability of<br />

Greater Rainfall<br />

First Cost<br />

of Wall, $<br />

2.0 0.3 200,000<br />

2.25 0.1 225,000<br />

2.5 0.05 300,000<br />

3.0 0.01 400,000<br />

3.25 0.005 450,000<br />

The wall will be financed through a 6% per year<br />

loan. The principal and interest will be repaid over<br />

a 10-year period. Records indicate an average<br />

damage of $50,000 has occurred with heavy rains,<br />

due to the relatively poor cohesive properties of<br />

the soil along the thoroughfare. A discount rate of<br />

6% per year is applicable. Find the amount of rainfall<br />

to protect against by choosing the retaining<br />

wall with the smallest AW value over the 10-year<br />

period.<br />

Decision Trees<br />

18.35 For the decision tree branch shown, determine the<br />

expected values of the two outcomes if decision<br />

D3 is already selected and the maximum outcome<br />

value is sought. (This decision branch is part of a<br />

larger tree.)<br />

D3<br />

Probability<br />

0.4<br />

0.3<br />

0.3<br />

0.6<br />

0.4<br />

Value, $<br />

55<br />

–30<br />

10<br />

–17<br />

18.36 A large decision tree has an outcome branch detailed<br />

(next page). If decisions D1, D2, and D3 are<br />

all options in a 1-year period, find the decision<br />

0


508 Chapter 18 Sensitivity Analysis and Staged Decisions<br />

path that maximizes the outcome value. There are<br />

specific investments necessary for decision nodes<br />

D1, D2, and D3, as indicated on each branch.<br />

Investment<br />

D1<br />

$50<br />

$25<br />

0.9<br />

0.5<br />

D2 $30 0.5<br />

$20<br />

$80<br />

0.1<br />

0.3<br />

0.3<br />

0.4<br />

D3<br />

Value $<br />

150<br />

–30<br />

75<br />

200<br />

– 100<br />

50<br />

0.4<br />

Value, $<br />

30<br />

100<br />

0.6 –50<br />

500<br />

18.37 Decision D4, which has three possible alternatives—<br />

x , y , or z —must be made in year 3 of a 6-year<br />

study period in order to maximize the expected<br />

value of present worth. Using a rate of return of 15%<br />

per year, the investment required in year 3 and the<br />

estimated cash flows for years 4 through 6, determine<br />

which decision should be made in year 3.<br />

D4<br />

x<br />

y<br />

z<br />

High<br />

Low<br />

High<br />

Low<br />

High<br />

Low<br />

Investment Cash flow, $1000<br />

Required, $<br />

Years<br />

3 4 5 6<br />

– 200,000<br />

– 75,000<br />

50<br />

40<br />

30<br />

30<br />

– 350,000 190<br />

–30<br />

50<br />

30<br />

40<br />

30<br />

170<br />

–30<br />

50<br />

20<br />

50<br />

30<br />

150<br />

–30<br />

90<br />

Outcome<br />

probability<br />

0.7<br />

0.3<br />

0.45<br />

0.55<br />

0.7<br />

0.3<br />

18.38 A total of 5000 mechanical subassemblies are<br />

needed annually on a final assembly line. The subassemblies<br />

can be obtained in one of three ways:<br />

(1) Make them in one of three plants owned by the<br />

company; (2) buy them off the shelf from the one<br />

and only manufacturer; or (3) contract to have them<br />

made to specifications by a vendor. The estimated<br />

annual equivalent cost for each alternative is dependent<br />

upon specific circumstances of the plant, producer,<br />

or contractor. The information shown details<br />

the circumstance, a probability of occurrence, and<br />

the estimated annual cost. Construct and solve a decision<br />

tree to determine the least-cost alternative to<br />

provide the subassemblies.<br />

Decision<br />

Alternative Outcomes Probability<br />

Annual Cost for<br />

5000 Units, $<br />

per Year<br />

1. Make Plant:<br />

A 0.3 250,000<br />

B 0.5 400,000<br />

C 0.2 350,000<br />

2. Buy off<br />

the shelf<br />

Quantity:<br />

5000, pay premium 0.2 550,000<br />

5000 available 0.7 250,000<br />

5000, forced to buy 0.1 290,000<br />

3. Contract Delivery:<br />

Timely delivery 0.5 175,000<br />

Late delivery, then<br />

buy some off shelf<br />

0.5 450,000<br />

18.39 The president of ChemTech is trying to decide<br />

whether to start a new product line or purchase a<br />

small company. It is not financially possible to do<br />

both. To make the product for a 3-year period will<br />

require an initial investment of $250,000. The expected<br />

annual cash flows with probabilities in<br />

parentheses are: $75,000 (0.5), $90,000 (0.4), and<br />

$150,000 (0.1). To purchase the small company<br />

will cost $450,000 now. Market surveys indicate a<br />

55% chance of increased sales for the company and<br />

a 45% chance of severe decreases with an annual<br />

cash flow of $25,000. If decreases are experienced<br />

in the first year, the company will be sold immediately<br />

(during year 1) at a price of $200,000. Increased<br />

sales could be $100,000 the first 2 years. If<br />

this occurs, a decision to expand after 2 years at an<br />

additional investment of $100,000 will be considered.<br />

This expansion could generate cash flows<br />

with indicated probabilities as follows: $120,000<br />

(0.3), $140,000 (0.3), and $175,000 (0.4). If expansion<br />

is not chosen, the current size will be maintained<br />

with anticipated sales to continue. Assume<br />

there are no salvage values on any investments. Use<br />

the description given and a 15% per year return to<br />

do the following.<br />

(a) Construct a decision tree with all values and<br />

probabilities shown.<br />

(b) Determine the expected PW values at the<br />

“ expansion/no expansion” decision node after<br />

2 years, provided sales are up.<br />

(c) Determine what decision should be made<br />

now to offer the greatest return possible for<br />

ChemTech.


Additional Problems and FE Exam Review Questions 509<br />

( d) Explain in words what would happen to the<br />

expected values at each decision node if the<br />

planning horizon were extended beyond<br />

3 years and all cash flow values continued as<br />

forecasted in the description.<br />

Real Options<br />

18.40 A privately held company that makes chips that are<br />

essential for high-volume data storage is valued at<br />

$3 billion. A computer company that wants to get<br />

into cloud computing is considering purchasing<br />

the company, but because of the uncertain economy,<br />

it would prefer to purchase an option that will<br />

allow it to buy the company for up to 1 year from<br />

now at a cost of $3.1 billion. What is the maximum<br />

amount the company should be willing to pay for<br />

the option, if its MARR is 12% per year?<br />

18.41 A company that is considering adding a new product<br />

line has determined that the first cost would be<br />

$80 million. The company is not sure about how<br />

the product will be received, so it has projected revenues<br />

using optimistic, most likely, and pessimistic<br />

estimates of $35 million, $25 million, and $10 million,<br />

respectively, with equal probability for each.<br />

Instead of expanding now, the company could implement<br />

a test program for 1 year in a limited area<br />

that will cost $4 million. (The full-scale project will<br />

still cost $80 million if implemented after the test<br />

program is over.) This will provide the company<br />

with the option to move forward or cancel the project.<br />

The criterion identified to move ahead with<br />

full-scale implementation is that revenues must exceed<br />

$900,000. In this case, the pessimistic estimate<br />

will be eliminated, and equal probability will<br />

be placed on the remaining revenue projections. If<br />

the company uses a 5-year planning horizon and a<br />

MARR of 12% per year, should the company go<br />

ahead with the full-scale project now or take the<br />

option to implement the test program for 1 year?<br />

18.42 Dow Chemical is considering licensing a low liquid<br />

discharge (LLD) water treatment system from a<br />

small company that developed the process. Dow<br />

can purchase a 1-year option for $150,000 that will<br />

give it time to pilot-test the LLD process, or Dow<br />

can acquire the license now at a cost of $1.8 million<br />

plus 25% of sales. If Dow waits 1 year, the cost<br />

will increase to $1.9 million plus 30% of sales. If<br />

Dow projects the sales to be $1,000,000 per year<br />

over the 5-year license period, should the company<br />

license the process now or purchase the option to<br />

license it after the 1-year test period? Assume the<br />

MARR is 15% per year.<br />

18.43 Abby has just negotiated a $15,000 price on a<br />

2-year-old car and is with the salesman closing the<br />

deal. There is a 1-year sales warranty with the purchase;<br />

however, an extended warranty is available<br />

for $2500 that will cover the same repairs and component<br />

failures as the 1-year warranty for 3 additional<br />

years. Abby understands this to be a real options<br />

situation with the price of the option ($2500)<br />

paid to avoid future, unknown costs. To help with<br />

her decision, the salesman provided three typical<br />

sets of historical data on estimated repair costs for<br />

used cars. The first-year costs are shown as zero<br />

because they will be covered by the sales warranty.<br />

Year<br />

1 2 3 4<br />

Repair cost, $ per year:<br />

A 0 500 1200 850<br />

B 0 1000 1400 400<br />

C 0 0 500 2000<br />

The salesman said case C is the base case, since it<br />

shows that the extended warranty is not needed because<br />

the cost of repairs equals the warranty cost.<br />

Abby immediately recognized this to be the case<br />

only when i 0%.<br />

(a)<br />

If Abby assumes that each repair cost scenario<br />

has equal probability of occurring with her<br />

car, and money is worth 5% per year to her,<br />

how much should she be willing to pay for the<br />

extended warranty that is offered at $2500?<br />

(b) If the base case actually occurs for her car<br />

and she does not purchase the warranty, what<br />

is the PW value of the expected future costs<br />

at i 5% per year?<br />

ADDITIONAL PROBLEMS AND FE EXAM REVIEW QUESTIONS<br />

18.44 In conducting a sensitivity analysis, all of the<br />

following could be used as a measure of worth<br />

except:<br />

( a) Present worth<br />

( b) Cost-capacity equations<br />

( c) Annual worth<br />

( d) Benefit-cost ratio<br />

18.45 When the measure of worth is plotted versus percent<br />

change for several parameters, the parameter that is<br />

the most sensitive in the economic analysis is the one:<br />

(a) That has the steepest curve<br />

(b) That has the flattest curve<br />

(c) With the largest present worth<br />

(d) With the shortest life


510 Chapter 18 Sensitivity Analysis and Staged Decisions<br />

18.46 In conducting a formalized sensitivity analysis<br />

using three estimates, the three estimates should be:<br />

(a) Strategic, pessimistic, real likely:<br />

(b) Deterministic, realistic, optimistic<br />

(c) Optimistic, pessimistic, most likely<br />

(d) Authentic, realistic, probabilistic<br />

18.47 For annual worth values of $30,000, $40,000, and<br />

$50,000 with chances of 20%, 20%, and 60%, respectively,<br />

the expected AW is closest to:<br />

(a) $34,000<br />

(b) $40,000<br />

(c) $44,000<br />

(d) $48,000<br />

18.48 The AW of a 3-year-old machine that is used in the<br />

manufacture of modular magnetic encoders, which<br />

provide data on the speed and positioning of rotating<br />

motor shafts, is $−48,000. A challenger will have a<br />

first cost of $90,000, an operating cost of $29,000,<br />

and a salvage value after 5 years that may vary considerably.<br />

For an interest rate of 10% per year and<br />

optimistic, most likely, and pessimistic salvage values<br />

of $15,000, $10,000, and $2000, respectively,<br />

the salvage value(s) for which the challenger AW<br />

will be lower than that of the defender are:<br />

(a) All of them<br />

(b) Only the optimistic one<br />

(c)<br />

(d)<br />

Only the optimistic and pessimistic ones<br />

None of them<br />

18.49 A decision tree includes all of the following except:<br />

(a) Probability estimates for each outcome<br />

(b) Measure of worth as the selection criterion<br />

(c) Expected results from a decision at each<br />

stage<br />

(d) The MARR<br />

18.50 A real options analysis is most valuable when:<br />

(a) The risk is low and stakes are high<br />

(b) The stakes are low and risk is high<br />

(c) The stakes are high and risk is high<br />

(d) The stakes are low and risk is low<br />

18.51 A small manufacturing company needs to purchase<br />

a machine that will have a first cost of<br />

$70,000. The company wants to buy an option that<br />

will allow it to purchase the machine for the same<br />

price of $70,000 for up to 1 year from now. If the<br />

company’s MARR is 10% per year, the maximum<br />

amount the company should pay for the option is<br />

closest to:<br />

(a) $5850<br />

(b) $6365<br />

(c) $6845<br />

(d) $7295<br />

CASE STUDY<br />

SENSITIVITY TO THE ECONOMIC ENVIRONMENT<br />

Background and Information<br />

Case Study Questions<br />

Berkshire Controllers usually finances its engineering projects<br />

with a combination of debt and equity capital. The resulting<br />

MARR ranges from a low of 4% per year, if business is<br />

slow, to a high of 10% per year. Normally, a 7% per year return<br />

is expected. Also the life estimates for assets tend to go<br />

down about 20% from normal in a vigorous business environment<br />

and up about 10% in a receding economy. The following<br />

estimates are the most likely values for two expansion<br />

plans currently being evaluated. Plan A will be executed at<br />

one location; Plan B will require two locations. All monetary<br />

estimates are in $1000 units.<br />

Plan B<br />

Plan A Location 1 Location 2<br />

First cost, $ 10,000 30,000 5,000<br />

AOC, $ per year 500 100 200<br />

Salvage value, $ 1,000 5,000 200<br />

Estimated life, years 40 40 20<br />

At the weekly meeting, you were asked to examine the following<br />

questions from Berkshire’s president.<br />

1. Are the PW values for plans A and B sensitive to<br />

changes in the MARR?<br />

2. Are the PW values sensitive to varying life estimates?<br />

3. Is the breakeven point for the first cost of plan A sensitive<br />

to the changes in MARR as business goes from vigorous<br />

to receding?


Case Study 511<br />

CASE STUDY<br />

SENSITIVITY ANALYSIS OF PUBLIC SECTOR PROJECTS—WATER SUPPLY PLANS<br />

Background<br />

Information<br />

One of the most basic services provided by municipal governments<br />

is the delivery of a safe, reliable water supply. As<br />

cities grow and extend their boundaries to outlying areas,<br />

they often inherit water systems that were not constructed<br />

according to city codes. The upgrading of these systems is<br />

sometimes more expensive than installing one correctly in<br />

the first place. To avoid these problems, city officials sometimes<br />

install water systems beyond the existing city limits<br />

in anticipation of future growth. This case study was extracted<br />

from such a countywide water and wastewater management<br />

plan and is limited to only some of the water supply<br />

alternatives.<br />

From about a dozen suggested plans, five methods were<br />

developed by an executive committee as alternative ways<br />

of providing water to the study area. These methods were<br />

then subjected to a preliminary evaluation to identify the<br />

most promising alternatives. Six attributes or factors were<br />

used in the initial rating: ability to serve the area, relative<br />

cost, engineering feasibility, institutional issues, environmental<br />

considerations, and lead time requirement. Each<br />

factor carried the same weighting and had values ranging<br />

from 1 to 5, with 5 being best. After the top three alternatives<br />

were identified, each was subjected to a detailed economic<br />

evaluation for selection of the best alternative.<br />

These detailed evaluations included an estimate of the capital<br />

investment of each alternative amortized over 20 years<br />

at 8% per year interest and the annual maintenance and operation<br />

(M&O) costs. The annual cost (an AW value) was<br />

then divided by the population served to arrive at a monthly<br />

cost per household.<br />

Table 18–5 presents the results of the screening using the six<br />

factors rated on a scale of 1 to 5. Alternatives 1A, 3, and 4<br />

were determined to be the three best and were chosen for further<br />

evaluation.<br />

Detailed Cost Estimates<br />

All amounts are cost estimates.<br />

Alternative 1A<br />

Capital cost<br />

Land with water rights: 1720 hectares<br />

@ $5000 per hectare $8,600,000<br />

Primary treatment plant 2,560,000<br />

Booster station at plant 221,425<br />

Reservoir at booster station 50,325<br />

Site cost 40,260<br />

Transmission line from river 3,020,000<br />

Transmission line right-of-way 23,350<br />

Percolation beds 2,093,500<br />

Percolation bed piping 60,400<br />

Production wells 510,000<br />

Well field gathering system 77,000<br />

Distribution system 1,450,000<br />

Additional distribution system 3,784,800<br />

Reservoirs 250,000<br />

Reservoir site, land, and development 17,000<br />

Subtotal 22,758,060<br />

<strong>Engineering</strong> and contingencies 5,641,940<br />

Total capital investment $28,400,000<br />

TABLE 18–5 Results of Rating Six Factors for Each Alternative, Case Study<br />

Alternative<br />

Description<br />

1A Receive city<br />

water and<br />

recharge wells<br />

3 Joint city and<br />

county plant<br />

4 County treatment<br />

plant<br />

8 Desalt<br />

groundwater<br />

12 Develop military<br />

water<br />

Ability<br />

to<br />

Supply<br />

Area<br />

Relative<br />

Cost<br />

<strong>Engineering</strong><br />

Feasibility<br />

Factors<br />

Institutional<br />

Issues<br />

Environmental<br />

Considerations<br />

Lead Time<br />

Requirement Total<br />

5 4 3 4 5 3 24<br />

5 4 4 3 4 3 23<br />

4 4 3 3 4 3 21<br />

1 2 1 1 3 4 12<br />

5 5 4 1 3 1 19


512 Chapter 18 Sensitivity Analysis and Staged Decisions<br />

Maintenance and operation costs (annual)<br />

Pumping 9,812,610 kWh per year<br />

@ $0.08 per kWh $ 785,009<br />

Fixed operating cost 180,520<br />

Variable operating cost 46,730<br />

Taxes for water rights 48,160<br />

Total annual M&O cost $1,060,419<br />

Total annual cost equivalent capital<br />

investment M&O cost<br />

28,400,000( AP ,8%,20)<br />

1,060,419<br />

2,892,540 1,060,419<br />

$3,952,959<br />

Average monthly household cost to serve 95% of 4980<br />

households is<br />

Household cost (3,952,959) ( — 1 ——<br />

$69.63 per month<br />

——<br />

0.95 )<br />

12 ) (<br />

1<br />

4980 ) ( 1<br />

Alternative 3<br />

Total capital investment $29,600,000<br />

Total annual M&O cost $867,119<br />

Total annual cost 29,600,000( AP ,8%,20)<br />

867,119<br />

3,014,760 867,119<br />

$3,881,879<br />

Household cost $68.38 per month<br />

Alternative 4<br />

Total capital investment $29,000,000<br />

Total annual M&O cost $1,063,449<br />

Total annual cost 29,000,000( AP ,8%,20)<br />

1,063,449<br />

2,953,650 1,063,449<br />

$4,017,099<br />

Household cost $70.76 per month<br />

On the basis of the lowest monthly household cost, alternative<br />

3 (joint city and county plant) is the most economically<br />

attractive.<br />

Case Study Exercises<br />

1. If the environmental considerations factor is to have a<br />

weighting of twice as much as any of the other five factors,<br />

what is its percentage weighting?<br />

2. If the ability to supply area and relative cost factors were<br />

each weighted 20% and the other four factors 15% each,<br />

which alternatives would be ranked in the top three?<br />

3. By how much would the capital investment of alternative<br />

4 have to decrease to make it more attractive than<br />

alternative 3?<br />

4. If alternative 1A served 100% of the households instead<br />

of 95%, by how much would the monthly household<br />

cost decrease?<br />

5. (a) Perform a sensitivity analysis on the two parameters<br />

of M&O costs and number of households to<br />

determine if alternative 3 remains the best economic<br />

choice. Three estimates are made for each<br />

parameter in Table 18–6 . M&O costs may vary up<br />

TABLE 18–6 Pessimistic, Most Likely, and Optimistic<br />

Estimates for Two Parameters<br />

Annual<br />

M&O<br />

Costs<br />

Number<br />

of<br />

Households<br />

Alternative 1A<br />

Pessimistic 1% 4980<br />

Most likely $1,060,419 2%<br />

Optimistic −1% 5%<br />

Alternative 3<br />

Pessimistic 5% 4980<br />

Most likely $867,119 2%<br />

Optimistic 0% 5%<br />

Alternative 4<br />

Pessimistic 2% 4980<br />

Most likely $1,063,449 2%<br />

Optimistic −10% 5%


Case Study 513<br />

(b)<br />

(pessimistic) or down (optimistic) from the most<br />

likely estimates presented in the case statement.<br />

The estimated number of households (4980) is determined<br />

to be the pessimistic estimate. Growth of<br />

2% up to 5% (optimistic) will tend to lower the<br />

monthly cost per household.<br />

Consider the monthly cost per household for alternative<br />

4, the optimistic estimate. The number of<br />

households is 5% above 4980, or 5230. What is<br />

the number of households that would have to be<br />

available in order for this option to have exactly<br />

the same monthly household cost as that for alternative<br />

3 at the optimistic estimate of 5230 households?


CHAPTER 19<br />

More on<br />

Variation and<br />

Decision<br />

Making under<br />

Risk<br />

LEARNING OUTCOMES<br />

Purpose: Incorporate decision making under risk into an engineering economy evaluation using probability, sampling,<br />

and simulation.<br />

SECTION TOPIC LEARNING OUTCOME<br />

19.1 Risk versus certainty • Understand the approaches to decision making<br />

under risk and certainty.<br />

19.2 Probability and distributions • Construct a probability distribution and<br />

cumulative distribution for one variable.<br />

19.3 Random sample • Obtain a random sample from a cumulative<br />

distribution using a random number table.<br />

19.4 , , and 2 • Estimate the population expected value, standard<br />

deviation, and variance from a random sample.<br />

19.5 Simulation • Use Monte Carlo sampling and spreadsheetbased<br />

simulation for alternative evaluation.


T<br />

his chapter further expands our ability to analyze variation in estimates, to consider<br />

probability, and to make decisions under risk . Fundamentals discussed include<br />

variables; probability distributions, especially their graphs and properties<br />

of expected value and dispersion; random sampling; and the use of simulation to<br />

account for estimate variation in engineering economy studies.<br />

Through coverage of variation and probability, this chapter complements topics in the first<br />

sections of Chapter 1: the role of engineering economy in decision making and economic<br />

analysis in the problem-solving process. These techniques are more time-consuming than using<br />

estimates made with certainty, so they should be used primarily for critical parameters.<br />

19.1 Interpretation of Certainty,<br />

Risk, and Uncertainty<br />

All things in the world vary—one from another, over time, and with different environments. We<br />

are guaranteed that variation will occur in engineering economy due to its emphasis on decision<br />

making for the future. Except for the use of breakeven analysis, sensitivity analysis, and a very<br />

brief introduction to expected values, virtually all our estimates have been certain; that is, no<br />

variation in the amount has entered into the computations of PW, AW, ROR, or any relations<br />

used. For example, the estimate that cash flow next year will be $4500 is one of certainty. Decision<br />

making under certainty is, of course, not present in the real world now and surely not in the<br />

future. We can observe outcomes with a high degree of certainty, but even this depends upon the<br />

accuracy and precision of the scale or measuring instrument.<br />

To allow a parameter of an engineering economy study to vary implies that risk , and possibly<br />

uncertainty, is introduced.<br />

When there may be two or more observable values for a parameter and it is possible to estimate<br />

the chance that each value may occur, risk is present. Virtually all decision making is<br />

performed under risk .<br />

Risk<br />

As an illustration, decision making under risk is introduced when an annual cash flow estimate<br />

has a 50-50 chance of being either $−1000 or $500.<br />

Decision making under uncertainty means there are two or more values observable, but the<br />

chances of their occurring cannot be estimated or no one is willing to assign the chances. The<br />

observable values in uncertainty analysis are often referred to as states of nature.<br />

For example, consider the states of nature to be the rate of national inflation in a particular<br />

country during the next 2 to 4 years: remain low, increase 2% to 6% annually, or increase 6%<br />

to 8% annually. If there is absolutely no indication that the three values are equally likely, or<br />

that one is more likely than the others, this is a statement that indicates decision making under<br />

uncertainty.<br />

Example 19.1 explains how a parameter can be described and graphed to prepare for decision<br />

making under risk.<br />

EXAMPLE 19.1<br />

CMS in Fairfield, Virginia received three bids each from vendors for two different pieces of<br />

large equipment, A and B. One of each piece of equipment must be purchased. Tom, an engineer<br />

at CMS, performed an evaluation of each bid and assigned it a rating between 0 and 100, with<br />

100 points being the best of the three. The total for each piece of equipment is 100%. The bid<br />

amounts and ratings are shown at the top of Figure 19–1 .<br />

(a) Consider the ratings as the chance out of 100 that the bid will be chosen, and plot cost<br />

versus chance for each vendor.<br />

(b) Since one each of A and B must be purchased, the total cost will vary somewhere between the<br />

sum of the lowest bids ($11 million) and the sum of the highest bids ($25 million). Plot this<br />

range with an equal chance of 1 in 14 that any amount in between these limits is possible.<br />

(c) Discuss the significant difference between the values of the cost ( x axis values) in the<br />

graphs in (a) and (b) above and how the chances are stated ( y axis values).


516 Chapter 19 More on Variation and Decision Making under Risk<br />

Figure 19–1<br />

Plot of cost estimates<br />

versus chance for ( a )<br />

each piece of equipment<br />

and ( b ) total cost<br />

range, Example 19.1.<br />

Equipment A<br />

Equipment B<br />

Bid, $1000 Rating, % Bid, $1000 Rating, %<br />

3,000 65 8,000 33.3<br />

5,000 25 10,000 33.3<br />

10,000 10 15,000 33.3<br />

80<br />

60<br />

65<br />

Equipment A<br />

Chance, %<br />

40<br />

25<br />

20<br />

10<br />

0<br />

1 2 3<br />

4 5<br />

6 7 8 9 10<br />

11 12<br />

Cost, $ million<br />

40<br />

Equipment B<br />

33.3 33.3 33.3<br />

Chance, %<br />

20<br />

0<br />

1 2 3 4 5 6 7 8 9 10 11 12 13 14 15 16<br />

Cost, $ million<br />

(a) Specific values<br />

—<br />

14 1<br />

Total cost<br />

Chance<br />

0<br />

7 9 11 13 15 17 19 21 23 25 27 29<br />

Cost, $ million<br />

(b) Continuous range<br />

Solution<br />

(a) Figure 19–1 a plots the specific bids for equipment A and B. The chances (ratings) for A and<br />

for B add to 100%. No values between the specific bids have any chance of occurring, according<br />

to the single-estimate bids from the three vendors.<br />

(b) The range of total cost is between $11 million and $25 million, as shown in Figure 19–1 b .<br />

Tom decided to make his estimate of total cost continuous between these two extremes.<br />

This means that the discrete sums of bids ($11 million, $15 million, and $25 million) are<br />

no longer used. Rather the entire range from $11 million to $25 million with a chance for<br />

every total cost in between is included. Every value has a chance of 1 in 14 of being observed.<br />

Now, the sum is a continuous value.<br />

(c) In the graph for bid values (Figure 19–1 a ), only specific or discrete estimates are included<br />

on the x axis. In the graph for the sum of the cost for equipment A and B (Figure 19–1 b ),<br />

the y axis values are continuous over a specific range.<br />

In the next section, the term variable is defined and two types of variables are explained—<br />

discrete and continuous —as illustrated here in an elementary form.


19.1 Interpretation of Certainty, Risk, and Uncertainty 517<br />

Before initiating an engineering economy study, it is important to decide if the analysis will<br />

be conducted with certainty for all parameters or if risk will be introduced. A summary of the<br />

meaning and use for each type of analysis follows.<br />

Decision Making under Certainty This is what we have done in most analyses thus far.<br />

Deterministic estimates are made and entered into measure of worth relations—PW, AW, FW,<br />

ROR, BC—and decision making is based on the results. The values estimated can be considered<br />

the most likely to occur with all chance placed on the single-value estimate. A typical example is<br />

an asset’s first cost estimate made with certainty, say, P $50,000. A plot of P versus chance has<br />

the general form of Figure 19–1 a with one vertical bar at $50,000 and 100% chance placed on it.<br />

The term deterministic , in lieu of certainty , is often used when single-value or single-point<br />

estimates are used exclusively.<br />

In fact, sensitivity analysis using different values of an estimate is simply another form of<br />

analysis with certainty, except that the analysis is repeated with different values, each estimated<br />

with certainty. The resulting measure of worth values are calculated and graphically<br />

portrayed to determine the decision’s sensitivity to different estimates for one or more<br />

parameters.<br />

Decision Making under Risk Now the element of chance is formally taken into account.<br />

However, it is more difficult to make a clear decision because the analysis attempts to accommodate<br />

variation . One or more parameters in an alternative will be allowed to vary. The estimates<br />

will be expressed as in Example 19.1 or in slightly more complex forms. Fundamentally,<br />

there are two ways to consider risk in an analysis:<br />

Expected value analysis. Use the chance and parameter estimates to calculate expected<br />

values E (parameter) via formulas such as Equation [18.2]. Analysis results in E (cash flow),<br />

E (AOC), and the like; and the final result is the expected value for a measure of worth, such<br />

as E (PW), E (AW), E (ROR), E (BC). To select the alternative, choose the most favorable<br />

expected value of the measure of worth. In an elementary form, this is what we learned about<br />

expected values in Chapter 18. The computations may become more elaborate, but the principle<br />

is fundamentally the same.<br />

Simulation analysis. Use the chance and parameter estimates to generate repeated computations<br />

of the measure of worth relation by randomly sampling from a plot for each varying<br />

parameter similar to those in Figure 19–1 . When a representative and random sample is complete,<br />

an alternative is selected utilizing a table or plot of the results. Usually, graphics are an<br />

important part of decision making via simulation analysis. Basically, this is the approach discussed<br />

in the rest of this chapter.<br />

Decision Making under Uncertainty When chances are not known for the identified<br />

states of nature (or values) of the uncertain parameters, the use of expected value–based decision<br />

making under risk as outlined above is not an option . In fact, it is difficult to determine<br />

what criterion to use to even make the decision. If it is possible to agree that each state is<br />

equally likely, then all states have the same chance, and the situation reduces to one of decision<br />

making under risk, because expected values can be determined. Because of the relatively<br />

inconclusive approaches necessary to incorporate decision making under uncertainty into an<br />

engineering economy study, the techniques can be quite useful but are beyond the intended<br />

scope of this text.<br />

In an engineering economy study, observed parameter values will vary from the value estimated<br />

at the time of the study. However, when performing the analysis, not all parameters should<br />

be considered as probabilistic (or at risk). Those that are estimable with a relatively high degree<br />

of certainty should be fixed for the study. Accordingly, the methods of sampling, simulation, and<br />

statistical data analysis are selectively used on parameters deemed important to the decisionmaking<br />

process. Parameters such as P , AOC, material and unit costs, sales price, revenues, etc.,<br />

are the targets of decision making under risk. Anticipated variation in interest rates is more commonly<br />

addressed by sensitivity analysis.<br />

The remainder of this chapter concentrates on decision making under risk as applied in an<br />

engineering economy study. Sections 19.2 to 19.4 provide foundation material necessary to design<br />

and correctly conduct a simulation analysis (Section 19.5).


518 Chapter 19 More on Variation and Decision Making under Risk<br />

Figure 19–2<br />

( a ) Discrete and continuous<br />

variable scales and<br />

( b) scales for a variable<br />

versus its probability.<br />

Discrete<br />

variable<br />

Continuous<br />

variable<br />

1.0<br />

3 5 10 15 Years<br />

Estimated life, n<br />

…<br />

–100%<br />

∞<br />

Rate of return, i<br />

(a)<br />

P(n)<br />

P(i)<br />

0.8<br />

0.6<br />

0.4<br />

0.2<br />

0<br />

1.0<br />

0.8<br />

0.6<br />

0.4<br />

0.2<br />

Discrete variable<br />

Estimated life,<br />

probability vs. years<br />

3 5 10 15 n, years<br />

Continuous variable<br />

Rate of return,<br />

probability vs. percent<br />

10 5 0 5 10 15 20 25 30 35 … i, %<br />

(b)<br />

19.2 Elements Important to Decision<br />

Making under Risk<br />

Some basics of probability and statistics are essential to correctly perform decision making under<br />

risk via expected value or simulation analysis. They are the random variable, probability, probability<br />

distribution , and cumulative distribution , as defined here. (If you are already familiar with<br />

them, this section will provide a review.)<br />

A random variable or variable is a characteristic or parameter that can take on any one of several<br />

values. Variables are classified as discrete or continuous. Discrete variables have several<br />

specific, isolated values, while continuous variables can assume any value between two stated<br />

limits, called the range of the variable.<br />

The estimated life of an asset is a discrete variable. For example, n may be expected to<br />

have values of n 3, 5, 10, or 15 years, and no others. The rate of return is an example of a<br />

continuous variable; i can vary from −100% to , that is, −100% i . The ranges of possible<br />

values for n (discrete) and i (continuous) are shown as the x axes in Figure 19–2 a . (In<br />

probability texts, capital letters symbolize a variable, say X , and small letters x identify a<br />

specific value of the variable. Though correct, this level of rigor in terminology is not applied<br />

in this chapter.)<br />

Probability is a number between 0 and 1.0 that expresses the chance in decimal form that a random<br />

variable (discrete or continuous) will take on any value from those identified for it. Probability<br />

is simply the amount of chance, divided by 100.<br />

Probabilities are commonly identified by P ( X i ) or P ( X X i ), which is read as the probability that<br />

the variable X takes on the value X i . (Actually, for a continuous variable, the probability at a<br />

single value is zero, as shown in a later example.) The sum of all P ( X i ) for a variable must be 1.0,


19.2 Elements Important to Decision Making under Risk 519<br />

a requirement already discussed. The probability scale, like the percentage scale for chance in<br />

Figure 19–1 , is indicated on the ordinate ( y axis) of a graph. Figure 19–2 b shows the 0 to 1.0<br />

range of probability for the variables n and i.<br />

A probability distribution describes how probability is distributed over the different values of<br />

a variable. Discrete variable distributions look significantly different from continuous variable<br />

distributions, as indicated by the inset at the right.<br />

The individual probability values are stated as<br />

P ( X i ) probability that X equals X i [19.1]<br />

The distribution may be developed in one of two ways: by listing each probability value for each<br />

possible variable value (see Example 19.2) or by a mathematical description or expression that<br />

states probability in terms of the possible variable values (Example 19.3).<br />

Cumulative distribution, also called the cumulative probability distribution , is the accumulation<br />

of probability over all values of a variable up to and including a specified value.<br />

Identified by F ( X i ), each cumulative value is calculated as<br />

P(X i )<br />

P(X i )<br />

F(X i )<br />

Discrete<br />

X i<br />

Continuous<br />

X i<br />

Discrete<br />

F ( X i ) sum of all probabilities through the value X i<br />

P ( X X i ) [19.2]<br />

As with a probability distribution, cumulative distributions appear differently for discrete (stairstepped)<br />

and continuous variables (smooth curve). Examples 19.2 and 19.3 illustrate cumulative<br />

distributions that correspond to specific probability distributions. These fundamentals about<br />

F ( X i ) are applied in the next section to develop a random sample.<br />

EXAMPLE 19.2<br />

Alvin is a medical doctor and biomedical engineering graduate who practices at Medical Center<br />

Hospital. He is planning to start prescribing an antibiotic that may reduce infection in patients<br />

with flesh wounds. Tests indicate the drug has been applied up to 6 times per day without<br />

harmful side effects. If no drug is used, there is always a positive probability that the infection<br />

will be reduced by a person’s own immune system.<br />

Published drug test results provide good probability estimates of positive reaction (i.e., reduction<br />

in the infection count) within 48 hours for increased treatments per day. Use the probabilities<br />

listed below to construct a probability distribution and a cumulative distribution for<br />

the total number of treatments per day.<br />

Number of Added<br />

Treatments per Day<br />

Probability of Infection Reduction<br />

for Each Added Treatment<br />

0 0.07<br />

1 0.08<br />

2 0.10<br />

3 0.12<br />

4 0.13<br />

5 0.25<br />

6 0.25<br />

Solution<br />

Define the random variable T as the number of added treatments per day. Since T can take on<br />

only seven different values, it is a discrete variable . The probability of infection reduction is<br />

listed for each value in column 2 of Table 19–1 . The cumulative probability F ( T i ) is determined<br />

using Equation [19.2] by adding all P ( T i ) values through T i , as indicated in column 3.<br />

Figure 19–3 a and b shows plots of the probability distribution and cumulative distribution,<br />

respectively. The summing of probabilities to obtain F ( T i ) gives the cumulative distribution the<br />

stair-stepped appearance, and in all cases the final F ( T i ) 1.0, since the total of all P ( T i ) values<br />

must equal 1.0.<br />

F(X i )<br />

Continuous<br />

X i<br />

X i


520 Chapter 19 More on Variation and Decision Making under Risk<br />

TABLE 19–1 Probability Distribution and Cumulative<br />

Distribution for Example 19.2<br />

(1) (2) (3)<br />

Number per Day<br />

T i<br />

Probability<br />

P ( T i )<br />

Cumulative<br />

Probability<br />

F ( T i )<br />

0 0.07 0.07<br />

1 0.08 0.15<br />

2 0.10 0.25<br />

3 0.12 0.37<br />

4 0.13 0.50<br />

5 0.25 0.75<br />

6 0.25 1.00<br />

Figure 19–3<br />

( a ) Probability distribution<br />

P ( T i ) and<br />

( b ) cumulative distribution<br />

F ( T i ) for<br />

Example 19.2.<br />

P(T i )<br />

0.3<br />

0.25 0.25<br />

0.2<br />

0.1<br />

0.07 0.08 0.10 0.12 0.13<br />

F(T i )<br />

1.0<br />

0.8<br />

0.6<br />

0.3<br />

0.15<br />

0.07<br />

0.25<br />

0.37<br />

0.50<br />

0.75<br />

1.00<br />

0<br />

0 1 2 3 4 5 6 T i<br />

(a)<br />

0<br />

0 1 2 3 4 5 6 T i<br />

(b)<br />

Comment<br />

Rather than use a tabular form as in Table 19–1 to state P ( T i ) and F ( T i ) values, it is possible to<br />

express them for each value of the variable.<br />

{<br />

0.07 T 1 0<br />

0.08 T 2 1<br />

0.10 T 3<br />

{<br />

0.07 T 1 0<br />

0.15 T 2 1<br />

2<br />

0.25 T 3 2<br />

P(T i ) 0.12 T 4 3 F(T i ) 0.37 T 4 3<br />

0.13 T 5 4<br />

0.50 T 5 4<br />

0.25 T 6 5<br />

0.75 T 6 5<br />

0.25 T 7 6<br />

1.00 T 7 6<br />

In basic engineering economy situations, the probability distribution for a continuous variable<br />

is commonly expressed as a mathematical function, such as a uniform distribution, a triangular<br />

distribution (both discussed in Example 19.3 in terms of cash flow), or the more complex, but<br />

commonly used, normal distribution. For continuous variable distributions, the symbol f (X ) is<br />

routinely used instead of P (X i ), and F (X ) is used instead of F (X i ), simply because the point probability<br />

for a continuous variable is zero. Thus, f (X ) and F (X ) are continuous lines and curves.<br />

EXAMPLE 19.3<br />

As president of a manufacturing systems consultancy, Sallie has observed the monthly cash<br />

flows that have occurred over the last 3 years into company accounts from two longstanding<br />

clients. Sallie has concluded the following about the distribution of these monthly cash flows:


19.2 Elements Important to Decision Making under Risk 521<br />

Client 1 Client 2<br />

Estimated low cash flow: $10,000 Estimated low cash flow: $20,000<br />

Estimated high cash flow: $15,000 Estimated high cash flow: $30,000<br />

Most likely cash flow: same for all values Most likely cash flow: $28,000<br />

Distribution of probability: uniform Distribution of probability: mode at $28,000<br />

The mode is the most frequently observed value for a variable. Sallie assumes cash flow to be<br />

a continuous variable referred to as C . ( a ) Write and graph the two probability distributions and<br />

cumulative distributions for monthly cash flow, and ( b ) determine the probability that monthly<br />

cash flow is no more than $12,000 for client 1 and at least $25,000 for client 2.<br />

Solution<br />

All cash flow values are expressed in $1000 units.<br />

Client 1: monthly cash fl ow distribution<br />

(a) The distribution of cash flows for client 1, identified by the variable C 1 , follows the uniform<br />

distribution. Probability and cumulative probability take the following general forms.<br />

f ( C 1 ) ————— 1<br />

high low<br />

low value C 1 high value<br />

f ( C 1 ) ——— 1<br />

H L<br />

L C 1 H [19.3]<br />

F ( C 1 ) ——————<br />

value low<br />

low value C<br />

high low<br />

1 high value<br />

F ( C 1 ) C 1<br />

———<br />

L<br />

H L<br />

L C 1 H [19.4]<br />

For client 1, monthly cash flow is uniformly distributed with L $10, H $15, and $10 <br />

C 1 $15. Figure 19–4 is a plot of f ( C 1 ) and F ( C 1 ) from Equations [19.3] and [19.4].<br />

f ( C 1 ) 1 —<br />

5 0.2 $10 C 1 $15<br />

F ( C 1 ) C 1 10 ————<br />

5<br />

$10 C 1 $15<br />

(b) The probability that client 1 has a monthly cash flow of no more than $12 is easily determined<br />

from the F ( C 1 ) plot as 0.4, or a 40% chance. If the F ( C 1 ) relation is used directly, the<br />

computation is<br />

F ($12) P ( C 1 $12) ———— 12 10<br />

0.4<br />

5<br />

1.0<br />

F(C 1 )<br />

Figure 19–4<br />

Uniform distribution for<br />

client 1 monthly cash<br />

flow, Example 19.3.<br />

0.8<br />

0.6<br />

0.4<br />

f (C 1 )<br />

0.2<br />

0.2<br />

0<br />

10 15<br />

0<br />

10<br />

12 15<br />

C 1 C 1


522 Chapter 19 More on Variation and Decision Making under Risk<br />

Client 2: monthly cash fl ow distribution<br />

(a) The distribution of cash flows for client 2, identified by the variable C 2 , follows the triangular<br />

distribution. This probability distribution has the shape of an upward-pointing triangle<br />

with the peak at the mode M , and downward-sloping lines joining the x axis on either<br />

side at the low ( L ) and high ( H ) values. The mode of the triangular distribution has the<br />

maximum probability value.<br />

f (mode) f ( M ) ——— 2<br />

[19.5]<br />

H L<br />

The cumulative distribution is comprised of two curved line segments from 0 to 1 with a<br />

break point at the mode, where<br />

F (mode) F ( M ) ——— M H L<br />

L<br />

[19.6]<br />

For C 2 , the low value is L $20, the high is H $30, and the most likely cash flow is the<br />

mode M $28. The probability at M from Equation [19.5] is<br />

f (28) ———— 2<br />

30 20 ——<br />

10 2 0.2<br />

The break point in the cumulative distribution occurs at C 2 28. Using Equation [19.6],<br />

F (28) ———— 28 20<br />

30 20 0.8<br />

Figure 19–5 presents the plots for f ( C 2 ) and F ( C 2 ). Note that f ( C 2 ) is skewed, since the<br />

mode is not at the midpoint of the range H L , and F ( C 2 ) is a smooth S-shaped curve with<br />

an inflection point at the mode.<br />

(b) From the cumulative distribution in Figure 19–5 , there is an estimated 31.25% chance that<br />

cash flow is $25 or less. Therefore,<br />

F ($30) F ($25) P ( C 2 $25) 1 0.3125 0.6875<br />

Comment<br />

The general relations f ( C 2 ) and F ( C 2 ) are not developed here. The variable C 2 is not a uniform<br />

distribution; it is triangular. Therefore, it requires the use of an integral to find cumulative probability<br />

values from the probability distribution f ( C 2 ).<br />

F(C 2 )<br />

1.0<br />

Mode<br />

0.8<br />

0.6<br />

0.2<br />

f (C 2 )<br />

Mode<br />

0.4<br />

0.3125<br />

0.2<br />

0<br />

20 28 30<br />

0<br />

20 25 28 30<br />

C 2 C 2<br />

Figure 19–5<br />

Triangular distribution for client 2 monthly cash flow, Example 19.3.


19.3 Random Samples 523<br />

19.3 Random Samples<br />

Estimating a parameter with a single value in previous chapters is the equivalent of taking a random<br />

sample of size 1 from an entire population of possible values. As an illustration, assume that<br />

estimates of first cost, annual operating cost, interest rate, and other parameters are used to compute<br />

one PW value in order to accept or reject an alternative. Each estimate is a sample of size 1<br />

from an entire population of possible values for each parameter. Now, if a second estimate is<br />

made for each parameter and a second PW value is determined, a sample of size 2 has been taken.<br />

If all values in the population were known, the probability distribution and cumulative distribution<br />

would be known. Then a sample would not be necessary.<br />

When we perform an engineering economy study and utilize decision making under certainty,<br />

we use one estimate for each parameter to calculate a measure of worth (i.e., a sample of size 1<br />

for each parameter). The estimate is the most likely value, that is, one estimate of the expected<br />

value. We know that all parameters will vary somewhat; yet some are important enough, or will<br />

vary enough, that a probability distribution should be determined or assumed for it and the parameter<br />

treated as a random variable. This is using risk, and a sample from the parameter’s probability<br />

distribution— P ( X ) for discrete or f ( X ) for continuous—helps formulate probability statements<br />

about the estimates. This approach complicates the analysis somewhat; however, it also<br />

provides a sense of confidence (or possibly a lack of confidence in some cases) about the decision<br />

made concerning the economic viability of the alternative based on the varying parameter. (We<br />

will further discuss this aspect later, after we learn how to correctly take a random sample from<br />

any probability distribution.)<br />

A random sample of size n is the selection in a random fashion of n values from a population<br />

with an assumed or known probability distribution, such that the values of the variable have the<br />

same chance of occurring in the sample as they are expected to occur in the population.<br />

Suppose Yvon is an engineer with 20 years of experience working for the Aircraft Safety<br />

Commission. For a two-crew aircraft, there are three parachutes on board. The safety standard<br />

states that 99% of the time, all three chutes must be “fully ready for emergency deployment.”<br />

Yvon is relatively sure that nationwide the probability distribution of N , the specific number of<br />

chutes fully ready, may be described by the probability distribution<br />

{<br />

0.005 N 0 chutes ready<br />

0.015 N 1 chute ready<br />

P(N N i ) <br />

0.060 N 2 chutes ready<br />

0.920 N 3 chutes ready<br />

This means that the safety standard is clearly not met nationwide. Yvon is in the process of<br />

sampling 200 (randomly selected) corporate and private aircraft across the nation to determine<br />

how many chutes are classified as fully ready. If the sample is truly random and Yvon’s<br />

probability distribution is a correct representation of actual parachute readiness, the observed<br />

N values in the 200 aircraft will approximate the same proportions as the population<br />

probabilities, that is, 1 aircraft with 0 chutes ready, etc. Since this is a sample, it is likely that<br />

the results won’t track the population exactly. However, if the results are relatively close, the<br />

study indicates that the sample results may be useful in predicting parachute safety across<br />

the nation.<br />

To develop a random sample, use random numbers ( RN ) generated from a uniform probability<br />

distribution for the discrete numbers 0 through 9, that is,<br />

P ( X i ) 0.1 for X i 0, 1, 2, … , 9<br />

In tabular form, the random digits so generated are commonly clustered in groups of two digits,<br />

three digits, or more. Table 19–2 is a sample of 264 random digits clustered into two-digit numbers.<br />

This format is very useful because the numbers 00 to 99 conveniently relate to the cumulative<br />

distribution values 0.01 to 1.00. This makes it easy to select a two-digit RN and enter F ( X )<br />

to determine a value of the variable with the same proportions as it occurs in the probability distribution.<br />

To apply this logic manually and develop a random sample of size n from a known


524 Chapter 19 More on Variation and Decision Making under Risk<br />

TABLE 19–2<br />

Random Digits Clustered into Two-Digit Numbers<br />

51 82 88 18 19 81 03 88 91 46 39 19 28 94 70 76 33 15 64 20 14 52<br />

73 48 28 59 78 38 54 54 93 32 70 60 78 64 92 40 72 71 77 56 39 27<br />

10 42 18 31 23 80 80 26 74 71 03 90 55 61 61 28 41 49 00 79 96 78<br />

45 44 79 29 81 58 66 70 24 82 91 94 42 10 61 60 79 30 01 26 31 42<br />

68 65 26 71 44 37 93 94 93 72 84 39 77 01 97 74 17 19 46 61 49 67<br />

75 52 14 99 67 74 06 50 97 46 27 88 10 10 70 66 22 56 18 32 06 24<br />

discrete probability distribution P ( X ) or a continuous variable distribution f ( X ), the following<br />

procedure may be used.<br />

1. Develop the cumulative distribution F ( X ) from the probability distribution. Plot F ( X ).<br />

2. Assign the RN values from 00 to 99 to the F ( X ) scale (the y axis) in the same proportion as<br />

the probabilities. For the parachute safety example, the probabilities from 0.0 to 0.15 are<br />

represented by the random numbers 00 to 14. Indicate the RNs on the graph.<br />

3. To use a table of random numbers, determine the scheme or sequence of selecting RN<br />

values—down, up, across, diagonally. Any direction and pattern is acceptable, but the<br />

scheme should be used consistently for one entire sample.<br />

4. Select the first number from the RN table, enter the F ( X ) scale, and observe and record the<br />

corresponding variable value. Repeat this step until there are n values of the variable that<br />

constitute the random sample.<br />

5. Use the n sample values for analysis and decision making under risk. These may include<br />

• Plotting the sample probability distribution.<br />

• Developing probability statements about the parameter.<br />

• Comparing sample results with the assumed population distribution.<br />

• Determining sample statistics (Section 19.4).<br />

• Performing a simulation analysis (Section 19.5).<br />

EXAMPLE 19.4<br />

Develop a random sample of size 10 for the variable N , number of months, as described by the<br />

probability distribution<br />

0.20 N 24<br />

P(N N i ) 0.50 N 30<br />

[19.7]<br />

{<br />

0.30 N 36<br />

Solution<br />

Apply the procedure above, using the P ( N N i ) values in Equation [19.7].<br />

1. The cumulative distribution, Figure 19–6 , is for the discrete variable N , which can assume<br />

three different values.<br />

2. Assign 20 numbers (00 through 19) to N 1 24 months, where P ( N 24) 0.2; 50 numbers<br />

to N 2 30; and 30 numbers to N 3 36.<br />

3. Initially select any position in Table 19–2 , and go across the row to the right and onto the<br />

row below toward the left. (Any routine can be developed, and a different sequence for<br />

each random sample may be used.)<br />

4. Select the initial number 45 (4th row, 1st column), and enter Figure 19–6 in the RN range<br />

of 20 to 69 to obtain N 30 months.<br />

5. Select and record the remaining nine values from Table 19–2 as shown below.<br />

RN 45 44 79 29 81 58 66 70 24 82<br />

N 30 30 36 30 36 30 30 36 30 36


19.3 Random Samples 525<br />

Cumulative<br />

probability<br />

1.0<br />

0.7<br />

Assigned<br />

random<br />

numbers<br />

70–99<br />

Figure 19–6<br />

Cumulative distribution<br />

with random number values<br />

assigned in proportion<br />

to probabilities,<br />

Example 19.4.<br />

F(N i )<br />

20–69<br />

45<br />

0.2<br />

00–19<br />

24 30 36<br />

N i , months<br />

Now, using the 10 values, develop the sample probabilities.<br />

Months<br />

N<br />

Times in<br />

Sample<br />

Sample<br />

Probability<br />

Equation [19.7]<br />

Probability<br />

24 0 0.00 0.2<br />

30 6 0.60 0.5<br />

36 4 0.40 0.3<br />

With only 10 values, we can expect the sample probability estimates to be different from the<br />

values in Equation [19.7]. Only the value N 24 months is significantly different, since no RN<br />

of 19 or less occurred. A larger sample will definitely make the probabilities closer to the<br />

original data.<br />

To take a random sample of size n for a continuous variable, the procedure above is applied,<br />

except the random number values are assigned to the cumulative distribution on a continuous<br />

scale of 00 to 99 corresponding to the F ( X ) values. As an illustration, consider Figure 19–4 ,<br />

where C 1 is the uniformly distributed cash flow variable for client 1 in Example 19.3. Here L <br />

$10, H $15, and f ( C 1 ) 0.2 for all values between L and H (all values are divided by $1000).<br />

The F ( C 1 ) is repeated as Figure 19–7 with the assigned random number values shown on the right<br />

scale. If the two-digit RN of 45 is chosen, the corresponding C 1 is graphically estimated to be<br />

$12.25. It can also be linearly interpolated as $12.25 10 (45100)(15 – 10).<br />

Cumulative<br />

probability<br />

1.0<br />

0.8<br />

Assigned<br />

random<br />

numbers<br />

99<br />

79<br />

Figure 19–7<br />

Random numbers<br />

assigned to the<br />

continuous variable of<br />

client 1 cash flows in<br />

Example 19.3.<br />

F(C 1 )<br />

0.6<br />

0.4<br />

59<br />

39<br />

45<br />

0.2<br />

19<br />

00<br />

10 11 12 13 14 15<br />

12.25<br />

C 1 , $1000


526 Chapter 19 More on Variation and Decision Making under Risk<br />

For greater accuracy when developing a random sample, especially for a continuous variable,<br />

it is possible to use 3-, 4-, or 5-digit RNs. These can be developed from Table 19–2 simply by<br />

combining digits in the columns and rows or obtained from tables with RNs printed in larger<br />

clusters of digits. In computer-based sampling, most simulation software packages have an RN<br />

generator built in that will generate values in the range of 0 to 1 from a continuous variable uniform<br />

distribution, usually identified by the symbol U (0, 1). The RN values, usually between<br />

0.00000 and 0.99999, are used to sample directly from the cumulative distribution employing<br />

essentially the same procedure explained here. The Excel functions RAND and RANDBE-<br />

TWEEN are described in Appendix A, Section A.3.<br />

An initial question in random sampling usually concerns the minimum size of n required to<br />

ensure confidence in the results. Without detailing the mathematical logic, sampling theory,<br />

which is based upon the law of large numbers and the central limit theorem (check a basic statistics<br />

book to learn about these), indicates that an n of 30 is sufficient. However, since reality does<br />

not follow theory exactly, and since engineering economy often deals with sketchy estimates,<br />

samples in the range of 100 to 200 are the common practice. But samples as small as 10 to 25<br />

provide a much better foundation for decision making under risk than the single-point estimate<br />

for a parameter that is known to vary widely.<br />

19.4 Expected Value and Standard Deviation<br />

Two very important measures or properties of a random variable are the expected value and standard<br />

deviation. If the entire population for a variable were known, these properties would be<br />

calculated directly. Since they are usually not known, random samples are commonly used to<br />

estimate them via the sample mean and the sample standard deviation, respectively. The following<br />

is a brief introduction to the interpretation and calculation of these properties using a random<br />

sample of size n from the population.<br />

The usual symbols are Greek letters for the true population measures and English letters for<br />

the sample estimates.<br />

True Population Measure<br />

Sample Estimate<br />

Symbol Name Symbol Name<br />

Expected value or E ( X ) Mu or true mean<br />

__<br />

X Sample mean<br />

Standard deviation<br />

______<br />

__<br />

or √Var(X)<br />

___ Sigma or true s or √s Sample standard<br />

or √ standard deviation<br />

deviation<br />

The expected value E(X) is the long-run expected average if the variable is sampled many times.<br />

The population expected value is not known exactly, since the population itself is not known<br />

completely, so µ is estimated either by E ( X ) from a distribution or by X __<br />

, the sample mean. Equation<br />

[18.2], repeated here as Equation [19.8], is used to compute the E ( X ) of a probability distribution,<br />

and Equation [19.9] is the sample mean, also called the sample average .<br />

Population:<br />

<br />

Probability distribution: E(X) X i P(X i ) [19.8]<br />

Sample:<br />

__ sum of sample values<br />

X ——————————<br />

sample size<br />

X i<br />

——<br />

n f i X i<br />

———<br />

n [19.9]<br />

The f i in the second form of Equation [19.9] is the frequency of X i , that is, the number of times<br />

each value occurs in the sample. The resulting X __<br />

is not necessarily an observed value of the variable;<br />

it is the long-run average value and can take on any value within the range of the variable.<br />

(We omit the subscript i on X and f when there is no confusion introduced.)


19.4 Expected Value and Standard Deviation 527<br />

EXAMPLE 19.5<br />

Kayeu, an engineer with Pacific NW Utilities, is planning to test several hypotheses about<br />

residential electricity bills in North American and Asian countries. The variable of interest is<br />

X , the monthly residential bill in U.S. dollars (rounded to the nearest dollar). Two small samples<br />

have been collected from different countries of North America and Asia. Estimate the<br />

population expected value. Do the samples (from a nonstatistical viewpoint) appear to be<br />

drawn from one population of electricity bills or from two different populations?<br />

North American, Sample 1, $ 40 66 75 92 107 159 275<br />

Asian, Sample 2, $ 84 90 104 187 190<br />

Solution<br />

Use Equation [19.9] for the sample mean.<br />

Sample 1: n 7 X i 814<br />

Sample 2: n 5 X i 655<br />

__<br />

X __ $116.29<br />

X $131.00<br />

Based solely on the small sample averages, the approximate $15 difference, which is<br />

only11% of the larger average bill, does not seem sufficiently large to conclude that the two<br />

populations are different. There are several statistical tests available to determine if samples<br />

come from the same or different populations. (Check a basic statistics text to learn<br />

about them.)<br />

Comment<br />

There are three commonly used measures of central tendency for data. The sample average<br />

is the most popular, but the mode and the median are also good measures. The mode, which<br />

is the most frequently observed value, was utilized in Example 19.3 for a triangular distribution.<br />

There is no specific mode in Kayeu’s two samples, since all values are different. The<br />

median is the middle value of the sample. It is not biased by extreme sample values, as is the<br />

mean. The two medians in the samples are $92 and $104. Based solely on the medians, the<br />

conclusion is still that the samples do not necessarily come from two different populations<br />

of electricity bills.<br />

The standard deviation s or s(X ) is the dispersion or spread of values about the expected value<br />

E(X ) or sample average X __<br />

.<br />

The sample standard deviation s estimates the property , which is the population measure of<br />

dispersion about the expected value of the variable. A probability distribution for data with strong<br />

central tendency is more closely clustered about the center of the data, and has a smaller s , than<br />

a wider, more dispersed distribution. In Figure 19–8 , the samples with larger s values— s 1 and<br />

s 4 —have a flatter, wider probability distribution.<br />

f(X)<br />

P(X)<br />

s 2<br />

s 1<br />

s 3<br />

s 4<br />

X 1<br />

X 2<br />

s 1 s 2 s 4 s 3<br />

Figure 19–8<br />

Sketches of distributions with different separate lines standard deviation values.


528 Chapter 19 More on Variation and Decision Making under Risk<br />

EXAMPLE 19.6<br />

Actually, the variance s 2 is often quoted as the measure of dispersion. The standard deviation<br />

is simply the square root of the variance, so either measure can be used. The s value is what we<br />

use routinely in making computations about risk and probability. Mathematically, the formulas<br />

and symbols for variance and standard deviation of a discrete variable and a random sample of<br />

size n are as follows:<br />

___<br />

Population: 2 Var ( X ) and √ 2 √ _______<br />

Var(X)<br />

Probability distribution: Var ( X ) [X i E(X)] 2 P(X i ) [19.10]<br />

Sample: s 2 sum of (sample value sample average)2<br />

——————————————————<br />

sample size 1<br />

(X i X __<br />

) 2<br />

—————<br />

[19.11]<br />

n 1<br />

__<br />

s √s 2<br />

Equation [19.11] for sample variance is usually applied in a more computationally convenient<br />

form.<br />

s 2 X i 2<br />

———<br />

n 1 <br />

__<br />

——— n 2<br />

X f i X i 2<br />

———<br />

n 1 n 1 <br />

__<br />

——— n 2<br />

X [19.12]<br />

n 1<br />

The standard deviation uses the sample average as a basis about which to measure the spread or<br />

dispersion of data via the calculation ( X X __<br />

), which can have a minus or plus sign. To accurately<br />

measure the dispersion in both directions from the average, the quantity ( X X __<br />

) is squared. To<br />

return to the dimension of the variable itself, the square root of Equation [19.11] is extracted. The<br />

term ( X X __<br />

) 2 is called the mean-squared deviation, and s has historically also been referred to<br />

as the root-mean-square deviation. The f i in the second form of Equation [19.12] uses the frequency<br />

of each X i value to calculate s 2 .<br />

One simple way to combine the average and standard deviation is to determine the percentage<br />

or fraction of the sample that is within 1, 2, or 3 standard deviations of the average, that is,<br />

__<br />

X ts for t 1, 2, or 3 [19.13]<br />

In probability terms, this is stated as<br />

P ( __<br />

X ts X __<br />

X ts ) [19.14]<br />

Virtually all the sample values will always be within the 3 s range of __<br />

__ X , but the percent within<br />

1 s will vary depending on how the data points are distributed about X . Example 19.6 illustrates<br />

the calculation of s to estimate and incorporates s with the sample average using X __<br />

ts.<br />

( a ) Use the two samples of Example 19.5 to estimate population variance and standard deviation<br />

for electricity bills. ( b ) Determine the percentages of each sample that are inside the ranges<br />

of 1 and 2 standard deviations from the mean.<br />

Solution<br />

(a) For illustration purposes only, apply the two different relations to calculate s for the two<br />

samples. For sample 1 (North American) with n 7, use X to identify the values. Table 19–3<br />

presents the computation of (X X __<br />

) 2 for Equation [19.11], with X __<br />

$116.29. The resulting<br />

s 2 and s values are<br />

s 2 —————<br />

37,743.40 6290.57<br />

6<br />

s $79.31


19.4 Expected Value and Standard Deviation 529<br />

TABLE 19–3 Computation of Standard Deviation<br />

Using Equation [19.11] with X __<br />

<br />

$116.29, Example 19.6<br />

X , $<br />

X − X<br />

__<br />

( X − X __<br />

) 2<br />

40 76.29 5,820.16<br />

66 50.29 2,529.08<br />

75 41.29 1,704.86<br />

92 24.29 590.00<br />

107 9.29 86.30<br />

159 42.71 1,824.14<br />

275 158.71 25,188.86<br />

814 37,743.40<br />

TABLE 19–4 Computation of Standard Deviation<br />

Using Equation [19.12] with __<br />

Y $131,<br />

Example 19.6<br />

For sample 2 (Asian), use Y to identify the values. With n 5 and __<br />

Y 131, Table 19–4<br />

shows Y 2 for Equation [19.12]. Then<br />

s 2 ———<br />

97,041<br />

4<br />

s $53<br />

Y , $ Y 2<br />

84 7,056<br />

90 8,100<br />

104 10,816<br />

187 34,969<br />

190 36,100<br />

655 97,041<br />

5 —<br />

4 (131) 2 42,260.25 1.25(17,161) 2809<br />

The dispersion is smaller for the Asian sample ($53) than for the North American sample<br />

($79.31).<br />

(b) Equation [19.13] determines the ranges of X __<br />

1 s and X __<br />

2 s . Count the number of sample<br />

data points between the limits, and calculate the corresponding percentage. See Figure 19–9<br />

for a plot of the data and the standard deviation ranges.<br />

–<br />

X = 116.29<br />

– 80 – 40 0 40 80 120<br />

–<br />

X ± 1s<br />

–<br />

X ± 2s<br />

(a)<br />

–<br />

Y = 131<br />

160 200 240 280 X<br />

0 40 80 120<br />

–<br />

Y ± 1s<br />

–<br />

Y ± 2s<br />

Figure 19–9<br />

Values, averages, and standard deviation ranges for ( a ) North American and<br />

( b ) Asian samples, Example 19.6.<br />

(b)<br />

160 200 240 Y


530 Chapter 19 More on Variation and Decision Making under Risk<br />

North American sample<br />

__<br />

X 1 s 116.29 79.31 for a range of $36.98 to $195.60<br />

Six out of seven values are within this range, so the percentage is 85.7%.<br />

__<br />

X 2 s 116.29 158.62 for a range of $42.33 to $274.91<br />

There are still six of the seven values within the X __<br />

2 s range. The limit $42.33 is meaningful<br />

only from the probabilistic perspective; from the practical viewpoint, use zero, that<br />

is, no amount billed.<br />

Asian sample<br />

__<br />

Y 1 s 131 53 for a range of $78 to $184<br />

There are three of five values, or 60%, within the range.<br />

__<br />

Y 2 s 131 106 for a range of $25 to $237<br />

All five of the values are within the Y __<br />

2 s range.<br />

Comment<br />

A second common measure of dispersion is the range, which is simply the largest minus the<br />

smallest sample values. In the two samples here, the range estimates are $235 and $106.<br />

Only the hand computations for E ( X ), s , and s 2 have been demonstrated here. Calculators and<br />

spreadsheets all have functions to determine these values by simply entering the data.<br />

Before we perform simulation analysis in engineering economy, it may be of use to summarize<br />

the expected value and standard deviation relations for a continuous variable, since Equations<br />

[19.8] through [19.12] address only discrete variables. The primary differences are that the<br />

summation symbol is replaced by the integral over the defined range of the variable, which we<br />

identify as R , and that P ( X ) is replaced by the differential element f ( X ) dX . For a stated continuous<br />

probability distribution f ( X ), the formulas are<br />

Expected value: E ( X ) R<br />

Xf ( X ) dX [19.15]<br />

Variance: Var ( X ) R<br />

X 2 f ( X ) dX [ E(X)] 2 [19.16]<br />

For a numerical example, again use the uniform distribution in Example 19.3 ( Figure 19–4 ) over<br />

the range R from $10 to $15. If we identify the variable as X , rather than C 1 , the following are<br />

correct.<br />

f ( X ) — 1 0.2 $10 X $15<br />

5<br />

E ( X ) X(0.2) dX 0.1X2 R | 15 0.1(225 100) $12.5<br />

10<br />

Var( X ) X2 (0.2) dX (12.5) 2 —— 0.2<br />

R 3 | 15 X3 10 (12.5) 2<br />

0.06667(3375 1000) 156.25 2.08<br />

____<br />

√2.08 $1.44<br />

Therefore, the uniform distribution between L $10 and H $15 has an expected value of<br />

$12.5 (the midpoint of the range, as expected) and a standard deviation of $1.44.<br />

EXAMPLE 19.7<br />

Christy is the regional safety engineer for a chain of franchise-based gasoline and food stores.<br />

The home office has had many complaints and several legal actions from employees and customers<br />

about slips and falls due to liquids (water, oil, gas, soda, etc.) on concrete surfaces.


19.4 Expected Value and Standard Deviation 531<br />

Corporate management has authorized each regional engineer to contract locally to apply to all<br />

exterior concrete surfaces a newly marketed product that absorbs up to 100 times its own<br />

weight in liquid and to charge a home office account for the installation. The authorizing letter<br />

to Christy states that, based upon their simulation and random samples that assume a normal<br />

population, the cost of the locally arranged installation should be about $10,000 and almost<br />

always is within the range of $8000 to $12,000.<br />

You have been asked to write a brief but thorough summary about the normal distribution,<br />

explain the $8000 to $12,000 range statement, and explain the phrase “random samples that<br />

assume a normal population.”<br />

Solution<br />

The following summaries about the normal distribution and sampling will help explain the<br />

authorization letter.<br />

Normal distribution, probabilities, and random samples<br />

The normal distribution is also referred to as the bell-shaped curve , the Gaussian distribution<br />

, or the error distribution . It is, by far, the most commonly used probability distribution<br />

in all applications. It places exactly one-half of the probability on either side of the mean or<br />

expected value. It is used for continuous variables over the entire range of numbers. The<br />

normal distribution is found to accurately predict many types of outcomes, such as IQ values;<br />

manufacturing errors about a specified size, volume, weight, etc.; and the distribution of<br />

sales revenues, costs, and many other business parameters around a specified mean, which is<br />

why it may apply in this situation.<br />

The normal distribution, identified by the symbol N ( , 2 ), where is the expected value or<br />

mean and 2 is the variance, or measure of spread, can be described as follows:<br />

• The mean locates the probability distribution ( Figure 19–10 a ), and the spread of the distribution<br />

varies with variance ( Figure 19–10 b ), growing wider and flatter for larger variance<br />

values.<br />

• When a sample is taken, the estimates are identified as sample mean X __<br />

for and sample<br />

standard deviation s for .<br />

• The normal probability distribution f ( X ) for a variable X is quite complicated, because its<br />

formula is<br />

f ( X ) ——— 1___<br />

√2<br />

exp { [<br />

where exp represents the number e 2.71828.<br />

(X )2<br />

————<br />

2 2 ] }<br />

Since f ( X ) is so unwieldy, random samples and probability statements are developed using a<br />

transformation, called the standard normal distribution (SND), which uses and (population)<br />

or X __<br />

and s (sample) to compute values of the variable Z .<br />

Population :<br />

Sample:<br />

Z —————————<br />

deviation from mean<br />

standard deviation<br />

———<br />

X <br />

[19.17]<br />

Z X X<br />

__<br />

———<br />

s [19.18]<br />

The SND for Z ( Figure 19–10 c ) is the same as for X , except that it always has a mean of 0 and<br />

a standard deviation of 1, and it is identified by the symbol N (0, 1). Therefore, the probability<br />

values under the SND curve can be stated exactly. It is always possible to transfer back to the<br />

original values from sample data by solving Equation [19.17] for X :<br />

X Z [19.19]


532 Chapter 19 More on Variation and Decision Making under Risk<br />

Figure 19–10<br />

Normal distribution<br />

showing ( a ) different<br />

mean values µ , ( b )<br />

different standard deviation<br />

values , and<br />

( c ) relation of<br />

normal X to standard<br />

normal Z .<br />

f(X)<br />

1<br />

2 3<br />

1 2 3<br />

(a)<br />

X<br />

• Equal dispersion<br />

( 1 = 2 = 3 )<br />

• Increasing means<br />

f(X)<br />

1<br />

• Increasing<br />

dispersion<br />

( 1 < 2 < 3 )<br />

• Two different<br />

means<br />

3<br />

1 = 3 2<br />

2<br />

(b)<br />

X<br />

f(X)<br />

Normal distribution<br />

– 3<br />

+ 3<br />

f(Z)<br />

<br />

0.9974<br />

0.9546<br />

0.6826<br />

X<br />

0.3413 0.3413<br />

Standard normal<br />

distribution<br />

0.0013 0.0214<br />

0.1360<br />

0.1360<br />

0.0214 0.0013<br />

–3 –2 –1 0 1 2 3 Z<br />

(c)<br />

Several probability statements for Z and X are summarized in the following table and are<br />

shown on the distribution curve for Z in Figure 19–10 c .<br />

Variable X Range Probability Variable Z Range<br />

1 0.3413 0 to 1<br />

1 0.6826 1 to 1<br />

2 0.4773 0 to 2<br />

2 0.9546 2 to 2<br />

3 0.4987 0 to 3<br />

3 0.9974 3 to 3<br />

As an illustration, probability statements from this tabulation and Figure 19–10 c for X and Z<br />

are as follows:<br />

The probability that X is within 2 of its mean is 0.9546.<br />

The probability that Z is within 2 of its mean, which is the same as between the values<br />

2 and 2, is also 0.9546.


19.5 Monte Carlo Sampling and Simulation Analysis 533<br />

In order to take a random sample from a normal N ( µ , 2 ) population, a specially prepared table of<br />

SND random numbers is used. (Tables of SND values are available in many statistics books.) The<br />

numbers are actually values from the Z or N (0, 1) distribution and have values such as 2.10,<br />

1.24, etc. Translation from the Z value back to the sample values for X is via Equation [19.19].<br />

Interpretation of the home office memo<br />

The statement that virtually all the local contract amounts should be between $8000 and<br />

$12,000 may be interpreted as follows: A normal distribution is assumed with a mean of µ <br />

$10,000 and a standard deviation for $667, or a variance of 2 ($667) 2 ; that is, an<br />

N [$10,000, ($667)<br />

2<br />

] distribution is assumed. The value $667 is calculated using the fact<br />

that virtually all the probability (99.74%) is within 3 of the mean, as stated above. Therefore,<br />

3 $2000 and $667 (rounded off)<br />

As an illustration, if six SND random numbers are selected and used to take a sample of size 6<br />

from the normal distribution N [$10,000, ($667)<br />

2<br />

], the results are as follows:<br />

SND Random Number<br />

Z<br />

X Using Equation [19.19]<br />

X Z <br />

2.10 X (2.10)(667) 10,000 $8599<br />

3.12 X (3.12)(667) 10,000 $12,081<br />

0.23 X (0.23)(667) 10,000 $9847<br />

1.24 X (1.24)(667) 10,000 $10,827<br />

2.61 X (2.61)(667) 10,000 $8259<br />

0.99 X (0.99)(667) 10,000 $9340<br />

In this sample of six typical concrete surfacing contract amounts for sites in our region, the<br />

average is $9825 and five of six values are within the range of $8000 to $12,000, with the sixth<br />

being only $81 above the upper limit.<br />

19.5 Monte Carlo Sampling and Simulation Analysis<br />

Up to this point, all alternative selections have been made using estimates with certainty, possibly<br />

followed by some testing of the decision via sensitivity analysis or expected values. In this<br />

section, we will use a simulation approach that incorporates the material of the previous sections<br />

to facilitate the engineering economy decision about one alternative or between two or<br />

more alternatives.<br />

The random sampling technique discussed in Section 19.3 is called Monte Carlo sampling .<br />

The general procedure outlined below uses Monte Carlo sampling to obtain samples of size n for<br />

selected parameters of formulated alternatives. These parameters, expected to vary according to a<br />

stated probability distribution, warrant decision making under risk. All other parameters in an alternative<br />

are considered certain; that is, they are known, or they can be estimated with enough precision<br />

to consider them certain. An important assumption is made, usually without realizing it.<br />

All parameters are independent; that is, one variable’s distribution does not affect the value of<br />

any other variable of the alternative. This is referred to as the property of independent random<br />

variables.<br />

The simulation approach to engineering economy analysis is summarized in the following<br />

basic steps.<br />

Step 1. Formulate alternative(s). Set up each alternative in the form to be considered<br />

using engineering economic analysis, and select the measure of worth upon which<br />

to base the decision. Determine the form of the relation(s) to calculate the measure<br />

of worth.<br />

Step 2. Parameters with variation. Select the parameters in each alternative to be treated<br />

as random variables. Estimate values for all other (certain) parameters for the analysis.


534 Chapter 19 More on Variation and Decision Making under Risk<br />

Step 3. Determine probability distributions. Determine whether each variable is discrete<br />

or continuous, and describe a probability distribution for each variable in each<br />

alternative. Use standard distributions, where possible, to simplify the sampling process<br />

and to prepare for computer-based simulation.<br />

Step 4. Random sampling. Incorporate the random sampling procedure of Section 19.3<br />

(the first four steps) into this procedure. This results in the cumulative distribution,<br />

assignment of RNs, selection of the RNs, and a sample of size n for each variable.<br />

Step 5. Measure of worth calculation. Compute n values of the selected measure of<br />

worth from the relation(s) determined in step 1. Use the estimates made with certainty<br />

and the n sample values for the varying parameters. (This is when the property<br />

of independent random variables is actually applied.)<br />

Step 6. Measure of worth description. Construct the probability distribution of the measure<br />

__ of __ worth, using between 10 and 20 cells of data, and calculate measures such as<br />

X , s , X ts , and relevant probabilities.<br />

Step 7. Conclusions. Draw conclusions about each alternative, and decide which is to be<br />

selected. If the alternative(s) has (have) been previously evaluated under the assumption<br />

of certainty for all parameters, comparison of results may help with the final decision.<br />

Example 19.8 illustrates this procedure using an abbreviated manual simulation analysis, and<br />

Example 19.9 utilizes spreadsheet simulation for the same estimates.<br />

EXAMPLE 19.8<br />

Yvonne Ramos is the CEO of a chain of 50 fitness centers in the United States and Canada. An<br />

equipment salesperson has offered Yvonne two long-term opportunities on new aerobic exercise<br />

systems, for which the usage is charged to customers on a per-use basis on top of the<br />

monthly fees paid by customers. As an enticement, the offer includes a guarantee of annual<br />

revenue for one of the systems for the first 5 years.<br />

Since this is an entirely new and risky concept of revenue generation, Yvonne wants to do a<br />

careful analysis of each alternative. Details for the two systems follow:<br />

System 1. First cost is P $12,000 for a set period of n 7 years with no salvage value.<br />

No guarantee for annual net revenue is offered.<br />

System 2. First cost is P $8000, there is no salvage value, and there is a guaranteed<br />

annual net revenue of $1000 for each of the first 5 years, but after this period, there is<br />

no guarantee. The equipment with updates may be useful up to 15 years, but the exact<br />

number is not known. Cancellation anytime after the initial 5 years is allowed, with no<br />

penalty.<br />

For either system, new versions of the equipment will be installed with no added costs. If the<br />

MARR is 15% per year, use PW analysis to determine if neither, one, or both of the systems<br />

should be installed.<br />

Solution by Hand<br />

Estimates that Yvonne makes to use the simulation analysis procedure are included in the following<br />

steps.<br />

Step 1. Formulate alternatives. Using PW analysis, the relations for system 1 and system<br />

2 are developed. The symbol NCF identifies the net cash flows (revenues), and<br />

NCF G is the guaranteed NCF of $1000 for system 2.<br />

PW 1 P 1 NCF 1 ( P A ,15%, n 1 ) [19.20]<br />

PW 2 P 2 NCF G ( P A ,15%,5)<br />

NCF 2 ( P A ,15%, n 2 5)( P F ,15%,5) [19.21]<br />

Step 2. Parameters with variation. Yvonne summarizes the parameters estimated with<br />

certainty and makes distribution assumptions about three parameters treated as random<br />

variables.


19.5 Monte Carlo Sampling and Simulation Analysis 535<br />

System 1<br />

Certainty. P 1 $12,000; n 1 7 years.<br />

Variable. NCF 1 is a continuous variable, uniformly distributed between<br />

L $−4000 and H $6000 per year, because this is considered a high-risk<br />

venture.<br />

System 2<br />

Certainty. P 2 $8000; NCF G $1000 for first 5 years.<br />

Variable. NCF 2 is a discrete variable, uniformly distributed over the<br />

values L $1000 to H $6000 only in $1000 increments, that is, $1000,<br />

$2000, etc.<br />

Variable. n 2 is a continuous variable that is uniformly distributed between<br />

L 6 and H 15 years.<br />

Now, rewrite Equations [19.20] and [19.21] to reflect the estimates made with certainty.<br />

PW 1 12,000 NCF 1 ( P A ,15%,7)<br />

12,000 NCF 1 (4.1604) [19.22]<br />

PW 2 8000 1000( P A ,15%,5)<br />

NCF 2 ( P A ,15%, n 2 5)( P F ,15%,5)<br />

4648 NCF 2 ( P A ,15%, n 2 5)(0.4972) [19.23]<br />

Step 3. Determine probability distributions. Figure 19–11 (left side) shows the assumed<br />

probability distributions for NCF 1 , NCF 2 , and n 2 .<br />

Step 4. Random sampling. Yvonne decides on a sample of size 30 and applies the first<br />

four of the random sample steps in Section 19.3. Figure 19–11 (right side) shows the<br />

cumulative distributions (step 1) and assigns RNs to each variable (step 2). The RNs<br />

for NCF 2 identify the x axis values so that all net cash flows will be in even $1000<br />

amounts. For the continuous variable n 2 , three-digit RN values are used to make the<br />

numbers come out evenly, and they are shown in cells only as “indexers” for easy<br />

reference when a RN is used to find a variable value. However, we round the number<br />

to the next higher value of n 2 because it is likely the contract may be canceled on an<br />

anniversary date. Also, now the tabulated compound interest factors for ( n 2 5)<br />

years can be used directly (see Table 19–5 ).<br />

Once the first RN is selected randomly from Table 19–2 , the sequence (step 3)<br />

used will be to proceed down the RN table column and then up the column to the left.<br />

Table 19–5 shows only the first five RN values selected for each sample and the corresponding<br />

variable values taken from the cumulative distributions in Figure 19–11<br />

(step 4).<br />

Step 5. Measure of worth calculation. With the five sample values in Table 19–5 , calculate<br />

the PW values using Equations [19.22] and [19.23].<br />

1. PW 1 12,000 (2200)(4.1604) $21,153<br />

2. PW 1 12,000 2000(4.1604) $3679<br />

3. PW 1 12,000 (1100)(4.1604) $16,576<br />

4. PW 1 12,000 (900)(4.1604) $15,744<br />

5. PW 1 12,000 3100(4.1604) $897<br />

1. PW 2 4648 1000( P A ,15%,7)(0.4972) $2579<br />

2. PW 2 4648 1000( P A ,15%,5)(0.4972) $2981<br />

3. PW 2 4648 5000( P A ,15%,8)(0.4972) $6507<br />

4. PW 2 4648 3000( P A ,15%,10)(0.4972) $2838<br />

5. PW 2 4648 4000( P A ,15%,3)(0.4972) $107<br />

Now, 25 more RNs are selected for each variable from Table 19–2 , and the PW values<br />

are calculated.<br />

Step 6. Measure of worth description. Figure 19–12 a and b presents the PW 1 and PW 2<br />

probability distributions for the 30 samples with 14 and 15 cells, respectively, as well<br />

as the range of individual PW values and the X __<br />

and s values.


536 Chapter 19 More on Variation and Decision Making under Risk<br />

Figure 19-11<br />

Distributions used for<br />

random samples,<br />

Example 19.8.<br />

f (NCF 1 )<br />

1<br />

10<br />

P(NCF 2 )<br />

1<br />

6<br />

Continuous variable<br />

–4 –2 0 2 4 6<br />

NCF 1 , $1000<br />

Discrete variable<br />

F(NCF 1 )<br />

1.0<br />

0.8<br />

0.6<br />

0.4<br />

0.2<br />

0<br />

F(NCF 2 )<br />

1.00<br />

–4 –2 0 2 4 6<br />

0<br />

0 1 2 3 4 5 6<br />

1 2 3 4 5 6<br />

NCF 2 , $1000 NCF 2 , $1000<br />

0.83<br />

0.67<br />

0.50<br />

0.33<br />

0.17<br />

NCF 1 , $1000<br />

RN<br />

80–99<br />

60–79<br />

40–59<br />

20–39<br />

00–19<br />

RN<br />

83–99<br />

67–82<br />

50–66<br />

33–49<br />

17–32<br />

00–16<br />

F(n 2 )<br />

1.0<br />

RN<br />

888–999<br />

0.8<br />

666–887<br />

0.6<br />

444–665<br />

1<br />

9<br />

f (n 2 )<br />

Continuous variable<br />

0.4<br />

0.2<br />

222–443<br />

000–221<br />

6 10 15<br />

n 2 , years<br />

0<br />

6<br />

8 10 12 14 15<br />

n 2 , years<br />

TABLE 19–5 Random Numbers and Variable Values for NCF 1 , NCF 2 , and<br />

n 2 , Example 19.8<br />

NCF 1 NCF 2 n 2<br />

RN* Value, $ RN † Value, $ RN ‡ Value, Year Rounded §<br />

18 2200 10 1000 586 11.3 12<br />

59 2000 10 1000 379 9.4 10<br />

31 1100 77 5000 740 12.7 13<br />

29 900 42 3000 967 14.4 15<br />

71 3100 55 4000 144 7.3 8<br />

*Randomly start with row 1, column 4 in Table 19–2.<br />

† Start with row 6, column 14.<br />

‡ Start with row 4, column 6.<br />

§ The n 2 value is rounded up.


19.5 Monte Carlo Sampling and Simulation Analysis 537<br />

X 1 ± 1s 1<br />

n 1 = 30<br />

s 1 = $10,190<br />

Frequency<br />

X 1 = $–7729<br />

Range = $37,443<br />

5<br />

4<br />

PW 0<br />

3<br />

2<br />

1<br />

0<br />

–27 –24 –21 –18 –15 –12 –9 –6 –3 0 3 6 9 12 15<br />

PW 1 , $1000<br />

(a) System 1<br />

Frequency<br />

6<br />

5<br />

4<br />

X 2 ± 1s 2<br />

X 2 = $2724<br />

PW 0<br />

n 2 = 30<br />

s 2 = $4336<br />

Range = $13,355<br />

3<br />

2<br />

1<br />

0<br />

–4 –3 –2 –1 0 1 2 3 4 5 6 7 8 9 10 11<br />

PW 2 , $1000<br />

(b) System 2<br />

Figure 19–12<br />

Probability distributions of simulated PW values for a sample of size 30, Example 19.8.<br />

PW 1 . Sample values range from $−24,481 to $12,962. The calculated measures<br />

of the 30 values are<br />

__<br />

X 1 $7729<br />

s 1 $10,190<br />

PW 2 . Sample values range from $−3031 to $10,324. The sample measures<br />

are<br />

__<br />

X 2 $2724<br />

s 2 $4336<br />

Step 7. Conclusions. Additional sample values will surely make the central tendency of<br />

the PW distributions more evident and may reduce the s values, which are quite<br />

large. Of course, many conclusions are possible once the PW distributions are<br />

known, but the following seem clear at this point.<br />

System 1. Based on this small sample of 30 observations, do not accept this<br />

alternative. The likelihood of making the MARR 15% is relatively small, since<br />

the sample indicates a probability of 0.27 (8 out of 30 values) that the PW will be<br />

positive, and X __<br />

1 is a large negative. Though appearing large, the standard deviation<br />

may be used to determine that about 20 of the 30 sample PW values (twothirds)<br />

are within the limits X __<br />

1 s , which are $17,919 and $2461. A larger<br />

sample may alter this analysis somewhat.<br />

System 2. If Yvonne is willing to accept the longer-term commitment that may<br />

increase the NCF some years out, the sample of 30 observations indicates to accept<br />

this alternative. At a MARR of 15%, the simulation approximates the chance<br />

for a positive PW as 67% (20 of the 30 PW values in Figure 19–12 b are positive).<br />

However, the probability of observing PW within the X __<br />

1 s limits ($1612 and<br />

$7060) is 0.53 (16 of 30 sample values).<br />

Conclusion at this point. Reject system 1; accept system 2; and carefully<br />

watch net cash flow, especially after the initial 5-year period.


538 Chapter 19 More on Variation and Decision Making under Risk<br />

Comment<br />

The estimates in Example 13.5 are very similar to those here, except all estimates were made<br />

with certainty (NCF 1 $3000, NCF 2 $3000, and n 2 14 years). The alternatives were<br />

evaluated by the payback period method at MARR 15%, and the first alternative was selected.<br />

However, the subsequent PW analysis in Example 13.5 selected alternative 2 based, in<br />

part, upon the anticipated larger cash flow in the later years.<br />

EXAMPLE 19.9<br />

Help Yvonne Ramos set up a spreadsheet simulation for the three random variables and PW<br />

analysis in Example 19.8. Does the PW distribution vary appreciably from that developed<br />

using manual simulation? Do the decisions to reject the system 1 proposal and accept the system<br />

2 proposal still seem reasonable?<br />

Solution by Spreadsheet<br />

Figures 19–13 and 19–14 are spreadsheet screen shots that accomplish the simulation portion<br />

of the analysis described above in steps 3 (determine probability distribution) through 6 (measure<br />

of worth description). Most spreadsheet systems are limited in the variety of distributions<br />

they can accept for sampling, but common ones such as uniform and normal are available.<br />

Figure 19–13 shows the results of a small sample of 30 values from the three distributions<br />

using the RAND and IF functions. (See Section A.3 in Appendix A.)<br />

NCF 1 : Continuous uniform from $−4000 to $6000. The spreadsheet relation in column B<br />

translates RN1 values (column A) into NCF1 amounts.<br />

NCF 2 : Discrete uniform in $1000 increments from $1000 to $6000. Column D cells display<br />

NCF2 in the $1000 increments using the logical IF function to translate from the RN2 values.<br />

n 2 : Continuous uniform from 6 to 15 years. The results in column F are integer values obtained<br />

using the INT function operating on the RN3 values.<br />

Figure 19–13<br />

Random sample of 30 values generated for spreadsheet simulation, Example 19.9.


19.5 Monte Carlo Sampling and Simulation Analysis 539<br />

AVERAGE(F13:F42)<br />

SUM(G13:G42)<br />

STDEV(F13:F42)<br />

Figure 19–14<br />

Simulation results for 30 PW values, Example 19.9.<br />

Figure 19–14 presents the two alternatives’ estimates in the top section. The PW1 and PW2<br />

computations for the 30 repetitions of NCF1, NCF2, and N2 are the spreadsheet equivalent of<br />

Equations [19.22] and [19.23]. The tabular approach used here tallies the number of PW values<br />

below zero ($0) and equal to or exceeding zero using the IF operator. For example, cell C17<br />

contains a 1, indicating PW1 0 when NCF1 $3100 (in cell B7 of Figure 19–13 ), which was<br />

used to calculate PW1 $897 by Equation [19.22]. Cells in rows 7 and 8 show the number of<br />

times in the 30 samples that system 1 and system 2 may return at least the MARR 15% because<br />

the corresponding PW 0. Sample averages and standard deviations are also indicated.<br />

Comparison between the hand and spreadsheet simulations is presented below.<br />

System 1 PW<br />

__<br />

X , $ s , $<br />

No. of<br />

PW 0<br />

System 2 PW<br />

__<br />

X , $ s , $<br />

No. of<br />

PW 0<br />

Hand 7,729 10,190 8 2,724 4,336 20<br />

Spreadsheet 7,105 13,199 10 1,649 3,871 19<br />

For the spreadsheet simulation, 10 (33%) of the PW1 values exceed zero, while the manual simulation<br />

included 8 (27%) positive values. These comparative results will change every time this<br />

spreadsheet is activated since the RAND function is set up (in this case) to produce a new RN each<br />

time. (It is possible to define RAND to keep the same RN values. See the Excel User’s Guide.)<br />

The conclusion to reject the system 1 proposal and accept system 2 is still appropriate for<br />

the spreadsheet simulation as it was for the hand solution, since there are comparable chances<br />

that PW 0.


540 Chapter 19 More on Variation and Decision Making under Risk<br />

CHAPTER SUMMARY<br />

To perform decision making under risk implies that some parameters of an engineering alternative<br />

are treated as random variables. Assumptions about the shape of the variable’s probability<br />

distribution are used to explain how the estimates of parameter values may vary. Additionally,<br />

measures such as the expected value and standard deviation describe the characteristic shape of<br />

the distribution. In this chapter, we learned several of the simple, but useful, discrete and continuous<br />

population distributions used in engineering economy—uniform and triangular—as well<br />

as specifying our own distribution or assuming the normal distribution.<br />

Since the population’s probability distribution for a parameter is not fully known, a random<br />

sample of size n is usually taken, and its sample average and standard deviation are determined.<br />

The results are used to make probability statements about the parameter, which help make the<br />

final decision with risk considered.<br />

The Monte Carlo sampling method is combined with engineering economy relations for a<br />

measure of worth such as PW to implement a simulation approach to risk analysis. The results of<br />

such an analysis can then be compared with decisions when parameter estimates are made with<br />

certainty.<br />

PROBLEMS<br />

Certainty, Risk, and Uncertainty<br />

19.1 Identify the following variables as either discrete<br />

or continuous.<br />

(a) The interest rates available in the marketplace<br />

for jumbo certificates of deposit<br />

(b)<br />

Optimistic, most likely, and pessimistic estimates<br />

of salvage value<br />

(c) The number of cars that are red in the first<br />

100 that pass through a certain intersection<br />

(d ) The weight of the purse or wallet carried when<br />

a person leaves her or his residence<br />

(e) The gallons of water that evaporate from<br />

Lake Erie in a given day<br />

19.2 For each situation below, determine (1) if the variable<br />

is discrete or continuous and (2) if the information<br />

involves certainty, risk, andor uncertainty.<br />

(a)<br />

The first cost of a new front-end loader is<br />

$34,000 or $38,000 depending on the size<br />

purchased.<br />

(b) The raises for engineers and technical staff<br />

employees will be 3%, or 5%, with one-half<br />

getting 3% and one-half getting 5%.<br />

(c)<br />

Revenue from a new product line is expected<br />

to be between $350,000 and $475,000 per<br />

year.<br />

(d ) The salvage value for an old machine will be<br />

$500 (i.e., its asking price) or $0 (it will be<br />

thrown away).<br />

(e)<br />

Profits are equally likely to be up anywhere<br />

from 25% to 60% this year.<br />

19.3 An engineer learned that production output is between<br />

1000 and 2000 units per week 90% of the<br />

time, and it may fall below 1000 or go above<br />

2000. He wants to use E (output) in the decisionmaking<br />

process. Identify at least two additional<br />

pieces of information that must be obtained or<br />

assumed to finalize the output information for<br />

this use.<br />

Probability and Distributions<br />

19.4 Royalties received by an investor in an oil well<br />

vary according to the price of oil. Data collected<br />

from stripper wells in an established oil field were<br />

used to develop the probability-royalty relationship<br />

shown below.<br />

Royalties, $ per Year 6200 8500 9600 10,300 12,600 15,500<br />

Probability 0.10 0.21 0.32 0.24 0.09 0.04<br />

(a) Calculate the expected value of royalty income<br />

(RI) per year.<br />

(b) Determine the probability that the royalty<br />

income will be at least $12,600 per year.<br />

19.5 Daily revenue from vending machines placed in<br />

various buildings of a major university is as follows:<br />

20, 75, 43, 62, 51, 52, 78, 33, 28, 39, 61, 56, 43, 49, 48, 49,<br />

71, 53, 57, 46, 42, 41, 63, 36, 51, 59, 40, 32, 37, 29, 26<br />

(a) Construct a frequency distribution table with<br />

a cell size of 12 starting with 19.5 (i.e., first<br />

cell is 19.5–31.5, next is 31.5–43.5, etc.).<br />

(b) Determine the probability distribution.<br />

(c) What is the probability of revenue from a<br />

machine being less than $44?<br />

(d) What is the probability that revenue from a<br />

machine will equal or exceed $44?


Problems 541<br />

19.6 A survey of households included a question about<br />

the number of operating automobiles N currently<br />

owned by people living at the residence and the<br />

interest rate i on the lowest-rate loan for the cars.<br />

The results for 100 households are shown.<br />

Number of Cars N<br />

Households<br />

0 12<br />

1 56<br />

2 26<br />

3 3<br />

4 3<br />

Loan Rate i , % Households<br />

0.02 13<br />

2.014 14<br />

4.016 19<br />

6.018 38<br />

8.0110 12<br />

10.0112 4<br />

( a) State whether each variable is discrete or<br />

continuous.<br />

( b) Plot the probability distributions and cumulative<br />

distributions for N and i .<br />

( c) From the data collected, what is the probability<br />

that a household has 1 or 2 cars? Three or<br />

more cars?<br />

( d) Use the data for i to estimate the chances that<br />

the interest rate is between 7% and 11% per<br />

year.<br />

19.7 An officer of the state lottery commission sampled<br />

lottery ticket purchasers over a 1-week period at<br />

one location. The amounts distributed back to the<br />

purchasers and the associated probabilities for<br />

5000 tickets are as follows:<br />

Distribution, $ 0 2 5 10 100<br />

Probability 0.91 0.045 0.025 0.013 0.007<br />

( a) Plot the cumulative distribution of winnings.<br />

( b) Calculate the expected value of the distribution<br />

of dollars per ticket.<br />

( c) If tickets cost $2, what is the expected longterm<br />

income to the state per ticket, based<br />

upon this sample?<br />

19.8 Bob is working on two separate probability-related<br />

projects. The first involves a variable N , which is the<br />

number of consecutively manufactured parts that<br />

weigh in above the weight specification limit. The<br />

variable N is described by the formula (0.5) N because<br />

each unit has a 50-50 chance of being below<br />

or above the limit. The second involves a battery<br />

life L that varies between 2 and 5 months. The probability<br />

distribution is triangular with the mode at<br />

5 months, which is the design life. Some batteries<br />

fail early, but 2 months is the smallest life experienced<br />

thus far. ( a ) Write out and plot the probability<br />

distributions and cumulative distributions for Bob.<br />

( b ) Determine the probability of N being 1, 2, or 3<br />

consecutive units above the weight limit.<br />

19.9 An alternative to buy and an alternative to lease<br />

hydraulic lifting equipment have been formulated.<br />

Use the parameter estimates and assumed distribution<br />

data shown to plot the probability distributions<br />

on one graph for each corresponding parameter.<br />

Label the parameters carefully.<br />

Parameter<br />

Purchase Alternative<br />

Estimated Value<br />

High<br />

Low<br />

Assumed<br />

Distribution<br />

First cost, $ 25,000 20,000 Uniform;<br />

continuous<br />

Salvage value, $ 3,000 2,000 Triangular;<br />

mode at $2500<br />

Life, years 8 4 Triangular;<br />

mode at 6<br />

AOC, $ per year 9,000 5,000 Uniform;<br />

continuous<br />

Parameter<br />

Lease Alternative<br />

Estimated Value<br />

High<br />

Low<br />

Assumed<br />

Distribution<br />

Lease first cost, $ 2000 1800 Uniform;<br />

continuous<br />

AOC, $ per year 9000 5000 Triangular;<br />

mode at $7000<br />

Lease term, years 3 3 Certainty<br />

19.10 Carla is a statistician with a bank. She has collected<br />

debt-to-equity mix data on mature ( M ) and<br />

young ( Y ) companies. The debt percentages vary<br />

from 20% to 80% in her sample. Carla has defined<br />

D M as a variable for the mature companies from<br />

0 to 1, with D M 0 interpreted as the low of 20%<br />

debt and D M 1.0 as the high of 80% debt. The<br />

variable for young corporation debt percentages<br />

D Y is similarly defined. The probability distributions<br />

used to describe D M and D Y are<br />

f ( D M ) 3(1 D M ) 2 0 D M 1<br />

f ( D Y ) 2 D Y 0 D Y 1<br />

(a) Use different values of the debt percentage between<br />

20% and 80% to calculate values for the probability<br />

distributions and then plot them. ( b ) What can<br />

you comment about the probability that a mature<br />

company or a young company will have a low debt<br />

percentage? A high debt percentage?


542 Chapter 19 More on Variation and Decision Making under Risk<br />

19.11 A discrete variable X can take on integer values of<br />

1 to 10. A sample of size 50 results in the following<br />

probability estimates:<br />

X i 1 2 3 6 9 10<br />

P(X i ) 0.2 0.2 0.2 0.1 0.2 0.1<br />

(a) Write out and graph the cumulative<br />

distribution.<br />

(b) Calculate the following probabilities using<br />

the cumulative distribution: X is between 6<br />

and 10, and X has the values 4, 5, or 6.<br />

(c) Use the cumulative distribution to show that<br />

P ( X 7 or 8) 0.0. Even though this probability<br />

is zero, the statement about X is that it<br />

can take on integer values of 1 to 10. How do<br />

you explain the apparent contradiction in<br />

these two statements?<br />

Random Samples<br />

19.12 A discrete variable X can take on integer values of<br />

1 to 5. A sample of size 100 results in the following<br />

probability estimates.<br />

X i 1 2 3 4 5<br />

P(X i ) 0.2 0.3 0.1 03 0.1<br />

(a) Use the following random numbers to estimate<br />

the probabilities for each value of X .<br />

(b) Determine the sample probabilities for X 1<br />

and X 5. Compare the sample results with<br />

the probabilities in the problem statement.<br />

RN: 10, 42, 18, 31, 23, 80, 80, 26, 74, 71, 03, 90,<br />

55, 61, 61, 28, 41, 49, 00, 79, 96, 78, 42, 31, 26<br />

19.13 The percent price increase p on a variety of retail<br />

food prices over a 1-year period varied from 5% to<br />

10% in all cases. Because of the distribution of<br />

p values, the assumed probability distribution for<br />

the next year is<br />

where<br />

f ( X ) 2 X 0 X 1<br />

0 when p 5%<br />

X <br />

{ 1 when p 10%<br />

For a continuous variable the cumulative distribution<br />

F ( X ) is the integral of f ( X ) over the same<br />

range of the variable. In this case<br />

F ( X ) X 2 0 X 1<br />

(a) Graphically assign RNs to the cumulative<br />

distribution, and take a sample of size 30 for<br />

the variable. Transform the X values into interest<br />

rates.<br />

(b) Calculate the average p value for the sample.<br />

19.14 Develop a discrete probability distribution of your<br />

own for the variable G , the expected grade in this<br />

course, where G A, B, C, D, F, or I (incomplete).<br />

Assign random numbers to F ( G ), and take a sample<br />

from it. Now plot the probability values from<br />

the sample for each G value.<br />

19.15 Use the RAND function in Excel to generate<br />

100 values from a U (0,1) distribution.<br />

(a) Calculate the average and compare it to 0.5,<br />

the expected value for a random sample between<br />

0 and 1.<br />

(b) For the RAND function sample, cluster the<br />

results into cells of 0.1 width, that is 0.0–0.1,<br />

0.1–0.2, etc., where the upper-limit value is<br />

excluded from each cell. Determine the probability<br />

for each grouping from the results.<br />

Does your sample come close to having<br />

approximately 10% in each cell?<br />

Sample Estimates<br />

19.16 An engineer was asked to determine whether the<br />

average air quality in a vehicle assembly plant was<br />

within OSHA guidelines. The following air quality<br />

readings were collected:<br />

(a)<br />

(b)<br />

(c)<br />

81, 86, 80, 91, 83, 83, 96, 85, 89<br />

Determine the sample mean.<br />

Calculate the standard deviation.<br />

Determine the number of values and percent<br />

of values that fall within 1 standard deviation<br />

of the mean.<br />

19.17 Carol sampled the monthly maintenance costs for automated<br />

soldering machines a total of 100 times during<br />

1 year. She clustered the costs into $200 cells, for<br />

example, $500 to $700, with cell midpoints of $600,<br />

$800, $1000, etc. She indicated the number of times<br />

(frequency) each cell value was observed. The costs<br />

and frequency data are as follows.<br />

Cell<br />

Midpoint, $<br />

Frequency<br />

600 6<br />

800 10<br />

1000 7<br />

1200 15<br />

1400 28<br />

1600 15<br />

1800 9<br />

2000 10<br />

(a) Estimate the expected value and standard deviation<br />

of the maintenance costs the company<br />

should anticipate based on Carol’s sample.<br />

( b) What is the best estimate of the percentage of<br />

costs that will fall within 2 standard deviations<br />

of the mean?


Additional Problems and FE Exam Review Questions 543<br />

( c) Develop a probability distribution of the<br />

monthly maintenance costs from Carol’s<br />

sample, and indicate the answers to the previous<br />

two questions on it.<br />

(d) Use a spreadsheet to display the sample mean.<br />

19.18 ( a ) Determine the values of sample average and<br />

standard deviation of the data in Problem 19.11.<br />

( b ) Determine the values 1 and 2 standard deviations<br />

from the mean. Of the 50 sample points, how<br />

many fall within these two ranges?<br />

19.19 ( a ) Use the relations in Section 19.4 for continuous<br />

variables to determine the expected value and<br />

standard deviation for the distribution of f ( D Y ) in<br />

Problem 19.10. ( b ) It is possible to calculate the<br />

probability of a continuous variable X between<br />

two points ( a, b ) using the following integral:<br />

b<br />

P ( a X b ) a f ( X ) dx<br />

Determine the probability that D Y is within 2 standard<br />

deviations of the expected value.<br />

19.20 ( a ) Use the relations in Section 19.4 for continuous<br />

variables to determine the expected value<br />

and variance for the distribution of D M in<br />

Problem 19.10.<br />

f ( D M ) 3(1 D M ) 2 0 D M 1<br />

(b) Determine the probability that D M is within 2<br />

standard deviations of the expected value.<br />

Use the relation in Problem 19.19.<br />

19.21 Calculate the expected value for the variable N in<br />

Problem 19.8.<br />

19.22 A newsstand manager is tracking Y , the number of<br />

weekly magazines left on the shelf when the new<br />

edition is delivered. Data collected over a 30-week<br />

period are summarized by the following probability<br />

distribution. Plot the distribution and the estimates<br />

for expected value and 1 standard deviation<br />

on either side of E ( Y ) on the plot.<br />

Simulation<br />

19.23 Carl, an engineering colleague, estimated net cash<br />

flow after taxes (CFAT) for the project he is working<br />

on. The additional CFAT of $2800 in year 10 is<br />

the salvage value of capital assets.<br />

Year CFAT, $<br />

0 28,800<br />

16 5,400<br />

710 2,040<br />

10 2,800<br />

The PW value at the current MARR of 7% per<br />

year is<br />

PW 28,800 5400( P A ,7%,6)<br />

2040( P A ,7%,4)( P F ,7%,6)<br />

2800( P F ,7%,10)<br />

$2966<br />

Carl believes the MARR will vary over a relatively<br />

narrow range, as will the CFAT, especially during<br />

the out years of 7 through 10. He is willing to accept<br />

the other estimates as certain. Use the following<br />

probability distribution assumptions for MARR<br />

and CFAT to perform a simulation—hand- or<br />

spreadsheet-based.<br />

MARR. Uniform distribution over the range<br />

6% to 10%.<br />

CFAT, years 7 through 10. Uniform distribution<br />

over the range $1600 to $2400 for each<br />

year.<br />

Plot the resulting PW distribution. Should the plan<br />

be accepted using decision making under certainty?<br />

Under risk?<br />

19.24 Repeat Problem 19.23, except use the normal distribution<br />

for the CFAT in years 7 through 10 with<br />

an expected value of $2000 and a standard deviation<br />

of $500.<br />

Y copies 3 7 10 12<br />

P ( Y ) 13 14 13 112<br />

ADDITIONAL PROBLEMS AND FE EXAM REVIEW QUESTIONS<br />

19.25 When there are at least two values for a parameter<br />

and it is possible to estimate the chance that each<br />

may occur, this situation is known as:<br />

( a) Uncertainty<br />

( b) Risk<br />

( c) Standard deviation<br />

( d) Cost estimating<br />

19.26 A deterministic economic analysis is one wherein:<br />

(a) Single-value estimates are used exclusively<br />

(b) Risk is taken into account<br />

(c) A range of values for each parameter is included<br />

in the analysis<br />

(d) Taxes and inflation are not considered in the<br />

cash flow estimates


544 Chapter 19 More on Variation and Decision Making under Risk<br />

19.27 Decision making under risk includes all of the following<br />

except:<br />

(a) Expected value analysis<br />

(b) Simulation<br />

(c) Using only single-value estimates<br />

(d) Probability<br />

19.28 All of the following are elements in decision making<br />

under risk except:<br />

(a) Random variable<br />

(b) Cost indexes<br />

(c) Probability<br />

(d) Cumulative distribution<br />

19.29 For the income and probability values shown in<br />

the table, the probability that the income in any<br />

year will be less than $9600 is closest to:<br />

(a) 0.15<br />

(b) 0.23<br />

(c) 0.32<br />

(d) 0.38<br />

Income, $ per Year 6200 8500 9600 10,300 12,600 15,500<br />

Probability 0.15 0.23 0.32 0.24 0.09 0.04<br />

19.30 The symbol that represents the true population<br />

mean is:<br />

(a) <br />

(b) s<br />

(c) __ <br />

(d) X<br />

19.31 The revenue from an oil dispersant product has<br />

averaged $15,000 per month for the past 12 months.<br />

If the value of ( X i − X __<br />

) 2 is $1,600,000, the standard<br />

deviation is closest to:<br />

( a) $381<br />

( b) $652<br />

( c) $958<br />

( d) $1265<br />

19.32 A survey of the types of cars parked at an NFL<br />

football stadium revealed that there were equal<br />

probabilities of finding cars identified as type A, B,<br />

C, and D. Car types were assigned random numbers<br />

as follows.<br />

Car Type<br />

Assigned RN<br />

A 0 through 24<br />

B 25 through 49<br />

C 50 through 74<br />

D 75 through 99<br />

The sample probability of a type B car from the<br />

12 random numbers shown is closest to:<br />

RN: 75, 52, 14, 99, 67, 74, 06, 50, 97, 46, 27, 88<br />

(a) 0.17<br />

(b) 0.25<br />

(c) 0.33<br />

(d ) 0.42<br />

CASE STUDY<br />

USING SIMULATION AND THREE-ESTIMATE SENSITIVITY ANALYSIS<br />

Background<br />

The Knox Brewing company makes specialty-named sodas<br />

and flavored drinks for retail grocery chains throughout<br />

North and Central America. During the past year, it has become<br />

obvious that a new bottle-capping machine is needed to<br />

replace the current 10-year-old system. Dr. Knox, the owner<br />

and president, knows the business quite well. You just handed<br />

him the first cost bids from three vendors for the machine. He<br />

looked carefully at the numbers and asked you to sit down.<br />

You were quite surprised, as this was the first time you had<br />

been in his office, and most other engineers at Knox have a<br />

great fear of “the Old Man.”<br />

Information<br />

As he examined the three bids on first cost of the machine, he<br />

started to write some numbers, which, he explained, were his<br />

estimates of the annual operating cost, useful life, and possible<br />

salvage value for each of the machines sold by the three vendors.<br />

After a few minutes, he told you to take these numbers<br />

and use some of that “new engineering knowledge” you<br />

acquired in college to determine which, if any, of these three<br />

bids made the best economic sense. He also told you to be<br />

innovative and use a computer and some probability to come<br />

up with a robust recommendation by tomorrow at 2 P.M.<br />

You have used the estimates from the president to develop<br />

Table 19–6 of pessimistic (P), most likely (ML), and optimistic<br />

(O) estimates for each vendor’s machine. In addition, you<br />

developed some possible distributions for the parameters that<br />

Dr. Knox estimated, namely, AOC, life, and salvage value.<br />

These are summarized in Table 19–7 . You plan to use a simple<br />

Monte Carlo simulation to help formulate your recommendation<br />

for tomorrow.<br />

Case Study Exercises<br />

First, learn to use the RNG (random number generator) in<br />

Excel, if you have not already done so. It is necessary to


Case Study 545<br />

TABLE 19–6 Parameter Estimates for Bottle-Capping Machine<br />

First Cost, $ AOC, $ per Year Salvage, $ Life, Years<br />

Vendor 1<br />

P 200,000 11,000 0 3<br />

ML 200,000 10,000 0 5<br />

O −200,000 6,000 0 8<br />

Vendor 2<br />

P 150,000 5,000 0 2<br />

ML 150,000 3,500 5,000 4<br />

O 150,000 2,000 8,000 7<br />

Vendor 3<br />

P 300,000 8,000 5,000 5<br />

ML 300,000 6,000 5,000 7<br />

O 300,000 4,500 8,000 9<br />

TABLE 19–7 Distribution Assumptions about AOC, Life, and Salvage<br />

Parameter Vendor 1 Vendor 2 Vendor 3<br />

AOC, $ per year<br />

Salvage, $<br />

Life, years<br />

Normal<br />

Mean: 10,000<br />

Std. dev.: 500<br />

Uniform<br />

0 to 1000<br />

Discrete uniform<br />

3 to 8, equal probability<br />

Normal<br />

Mean: 3500<br />

Std. dev.: 500<br />

Uniform<br />

0 to 8000<br />

Discrete uniform<br />

2 to 7, equal probability<br />

Normal<br />

Mean: 6000<br />

Std. dev.: 500<br />

Uniform<br />

5000 to 8000<br />

Discrete uniform<br />

5 to 9, equal probability<br />

sample from the normal distributions that you have specified<br />

in Table 19–7 . RNG is part of the Analysis Tool-Pak<br />

accessed through the Office Button, Excel Options, Add-<br />

Ins path.<br />

1. Prepare the simulation using a spreadsheet; determine<br />

which of the vendors offers the best machine from an<br />

economic perspective, and take into account the estimates<br />

made by Dr. Knox. Use a sample size of at least<br />

50, and base your conclusions on the AW measure of<br />

worth.<br />

2. Prepare a short presentation for Dr. Knox (and class)<br />

using your analysis.


APPENDIX A<br />

USING SPREADSHEETS<br />

AND MICROSOFT EXCEL ©<br />

This appendix explains the layout of a spreadsheet and the use of Microsoft Excel (hereafter<br />

called Excel) functions in engineering economy. Refer to the Excel help system for your particular<br />

computer and version of Excel. Some specific commands and entries refer to Excel 2007 and<br />

may differ slightly from your version.<br />

A.1 Introduction to Using Excel<br />

Enter a Formula or Use an Excel Function<br />

The sign is required to perform any formula or function computation in a cell. The formulas<br />

and functions on the worksheet can be displayed by simultaneously pressing Ctrl and `. The symbol<br />

` is usually in the upper left of the keyboard with the ~ (tilde) symbol. Pressing Ctrl+` a second<br />

time hides the formulas and functions.<br />

1. Run Excel.<br />

2. Move to cell C3. (Move the pointer to C3 and left-click.)<br />

3. Type PV(5%,12,10) and Enter. This function will calculate the present value of<br />

12 payments of $10 at a 5% per year interest rate.<br />

Another example: To calculate the future value of 12 payments of $10 at 6% per year interest, do<br />

the following:<br />

1. Move to cell B3, and type INTEREST.<br />

2. Move to cell C3, and type 6% or 6100.<br />

3. Move to cell B4, and type PAYMENT.<br />

4. Move to cell C4, and type 10 (to represent the size of each payment).<br />

5. Move to cell B5, and type NUMBER OF PAYMENTS.<br />

6. Move to cell C5, and type 12 (to represent the number of payments).<br />

7. Move to cell B7, and type FUTURE VALUE.<br />

8. Move to cell C7, and type FV(C3,C5,C4) and hit Enter. The answer will appear<br />

in cell C7.<br />

To edit the values in cells<br />

1. Move to cell C3 and type 5100 (the previous value will be replaced).<br />

2. The value in cell C7 will update.<br />

Cell References in Formulas and Functions<br />

If a cell reference is used in lieu of a specific number, it is possible to change the number once<br />

and perform sensitivity analysis on any variable that is referenced by the cell number, such as C5.<br />

This approach defines the referenced cell as a global variable for the worksheet. There are two<br />

types of cell references—relative and absolute.<br />

Relative References If a cell reference is entered, for example, A1, into a formula or function<br />

that is copied or dragged into another cell, the reference is changed relative to the movement of<br />

the original cell. If the formula in C5 is A1 and it is copied into cell C6, the formula is changed<br />

to A2. This feature is used when dragging a function through several cells, and the source entries<br />

must change with the column or row.<br />

Absolute References If adjusting cell references is not desired, place a $ sign in front of<br />

the part of the cell reference that is not to be adjusted—the column, row, or both. For example,


548 Appendix A Using Spreadsheets and Microsoft Excel ©<br />

$A$1 will retain the formula when it is moved anywhere on the worksheet. Similarly, $A1<br />

will retain the column A, but the relative reference on 1 will adjust the row number upon movement<br />

around the worksheet.<br />

Absolute references are used in engineering economy for sensitivity analysis of parameters<br />

such as MARR, first cost, and annual cash flows. In these cases, a change in the absolutereference<br />

cell entry can help determine the sensitivity of a result, such as PW or AW.<br />

Print the Spreadsheet<br />

First define the portion (or all) of the spreadsheet to be printed.<br />

1. Move the pointer to the top left cell of your spreadsheet.<br />

2. Hold down the left-click button. (Do not release the left-click button.)<br />

3. Drag the mouse to the lower right corner of your spreadsheet or to wherever you want to stop<br />

printing.<br />

4. Release the left-click button. (It is ready to print.)<br />

5. Left-click the Office button (see Figure A–1 ).<br />

6. Move the pointer down to select Print and left-click.<br />

7. In the dialog box, left-click the Print option (or similar command).<br />

Depending on your computer environment, you may have to select a network printer and queue<br />

your printout through a server.<br />

Save the Spreadsheet<br />

You can save your spreadsheet at any time during or after completing your work. It is recommended<br />

that you save your work regularly.<br />

1. Left-click the Office button.<br />

2. To save the spreadsheet the first time, left-click the Save As . . . option.<br />

3. Type the file name, e.g., Prob 7.9, and left-click the Save button.<br />

To save the spreadsheet after it has been saved the first time, i.e., a file name has been assigned<br />

to it, left-click the Office button, move the pointer down, and left-click on Save.<br />

Create an xy (Scatter) Chart<br />

This chart is one of the most commonly used in scientific analysis, including engineering economy.<br />

It plots pairs of data and can place multiple series of entries on the Y axis. The xy scatter<br />

chart is especially useful for results such as the PW versus i graph, where i is the X axis and the<br />

Y axis displays the results of the NPV function for several alternatives.<br />

1. Run Excel.<br />

2. Enter the following numbers in columns A, B, and C, respectively.<br />

Column A, cell A1 through A6: Rate i %, 4, 6, 8, 9, 10<br />

Column B, cell B1 through B6: $ for A, 40, 55, 60, 45, 10<br />

Column C, cell C1 through C6: $ for B, 100, 70, 65, 50, 30.<br />

3. Move the mouse to A1, left-click, and hold while dragging to cell C6. All cells will be highlighted,<br />

including the title cell for each column.<br />

4. If not all the columns for the chart are adjacent to one another, first press and hold the Control<br />

key on the keyboard during the entirety of step 3. After dragging over one column of<br />

data, momentarily release the left click, then move to the top of the next (nonadjacent) column<br />

of the chart. Do not release the Control key until all columns to be plotted have been<br />

highlighted.<br />

5. Left-click on the Insert button on the toolbar.<br />

6. Select the Scatter option and choose a subtype of scatter chart. The graph appears with a<br />

legend ( Figure A–1 ).<br />

Now a large number of styling effects can be introduced for axis titles, legend, data series, etc.<br />

Note that only the bottom row of the title can be highlighted. If titles are not highlighted, the data<br />

sets are generically identified as series 1, series 2, etc. on the legend.


A.2 Organization (Layout) of the Spreadsheet 549<br />

Office button<br />

Insert for<br />

scatter chart<br />

Figure A–1<br />

Scatter chart for data entries and location of commonly used buttons.<br />

Obtain Help While Using Excel<br />

1. To get general help information, left-click on the “?” (upper right).<br />

2. Enter the topic or phrase. For example, if you want to know more about how to save a file,<br />

type the word Save.<br />

3. Select the appropriate matching words. You can browse through the options by left-clicking<br />

on any item.<br />

A.2 Organization (Layout) of the Spreadsheet<br />

A spreadsheet can be used in several ways to obtain answers to numerical questions. The first is<br />

as a rapid solution tool, often with the entry of only a few numbers or one predefined function.<br />

For example, to find the future worth in a single-cell operation, move the pointer to any cell and<br />

enter FV(8%,5,-2500). The display of $14,666.50 is the 8% future worth at the end of year 5<br />

of five equal payments of $2500 each.<br />

A second use is more formal; it presents data, solutions, graphs, and tables developed on the<br />

spreadsheet and ready for presentation to others. Some fundamental guidelines in spreadsheet<br />

organization are presented here. A sample layout is presented in Figure A–2 . As solutions become<br />

more complex, organization of the spreadsheet becomes increasingly important, especially for<br />

presentation to an audience via PowerPoint or similar software.<br />

Cluster the data and the answers. It is advisable to organize the given or estimated data in the<br />

top left of the spreadsheet. A very brief label should be used to identify the data, for example,<br />

MARR in cell A1 and the value, 12%, in cell B1. Then B1 can be the referenced cell for all<br />

entries requiring the MARR. Additionally, it may be worthwhile to cluster the answers into one<br />

area and frame it. Often, the answers are best placed at the bottom or top of the column of entries<br />

used in the formula or predefined function.<br />

Enter titles for columns and rows. Each column or row should be labeled so its entries are<br />

clear to the reader. It is very easy to select from the wrong column or row when no brief title is<br />

present at the head of the data.<br />

Enter income and cost cash fl ows separately. When there are both income and cost cash flows<br />

involved, it is strongly recommended that the cash flow estimates for revenue (usually positive)


550 Appendix A Using Spreadsheets and Microsoft Excel ©<br />

Function used to find FW<br />

Figure A–2<br />

Spreadsheet layout with cash flow estimates, results of functions, function formula detailed, and a scatter chart.<br />

and first cost, salvage value, and annual costs (usually negative, with salvage a positive number)<br />

be entered into two adjacent columns. Then a formula combining them in a third column displays<br />

the net cash flow. There are two immediate advantages to this practice: fewer errors are made<br />

when performing the summation and subtraction mentally, and changes for sensitivity analysis<br />

are more easily made.<br />

Use cell references. The use of absolute and relative cell references is a must when any changes<br />

in entries are expected. For example, suppose the MARR is entered in cell B1 and three separate<br />

references are made to the MARR in functions on the spreadsheet. The absolute cell reference<br />

entry $B$1 in the three functions allows the MARR to be changed one time, not three.<br />

Obtain a fi nal answer through summing and embedding. When the formulas and functions<br />

are kept relatively simple, the final answer can be obtained using the SUM function. For example,<br />

if the present worth values (PW) of two columns of cash flows are determined separately,<br />

then the total PW is the SUM of the subtotals. This practice is especially useful when the cash<br />

flow series are complex.<br />

Prepare for a chart. If a chart (graph) will be developed, plan ahead by leaving sufficient<br />

room on the right of the data and answers. Charts can be placed on the same worksheet or on a<br />

separate worksheet. Placement on the same worksheet is recommended, especially when the<br />

results of sensitivity analysis are plotted.<br />

A.3 Excel Functions Important to <strong>Engineering</strong><br />

<strong>Economy</strong> (alphabetical order)<br />

DB (Declining Balance)<br />

Calculates the depreciation amount for an asset for a specified period n using the declining balance<br />

method. The depreciation rate d used in the computation is determined from asset values<br />

S (salvage value) and B (basis or first cost) as d 1 ( S B )<br />

1 n<br />

. This is Equation [16.12]. Threedecimal-place<br />

accuracy is used for d .<br />

cost<br />

salvage<br />

life<br />

period<br />

month<br />

DB(cost, salvage, life, period, month)<br />

First cost or basis of the asset.<br />

Salvage value.<br />

Depreciation life (recovery period).<br />

The period, year, for which the depreciation is to be calculated.<br />

(optional entry) If this entry is omitted, a full year is assumed for the<br />

first year.


A.3 Excel Functions Important to <strong>Engineering</strong> <strong>Economy</strong> 551<br />

Example A new machine costs $100,000 and is expected to last 10 years. At the end of<br />

10 years, the salvage value of the machine is $50,000. What is the depreciation of the machine in<br />

the first year and the fifth year?<br />

Depreciation for the first year: DB(100000,50000,10,1)<br />

Depreciation for the fifth year: DB(100000,50000,10,5)<br />

Because of the manner in which the DB function determines the fixed percentage d and the accuracy<br />

of the computations, it is recommended that the DDB function (below) be used for all declining<br />

balance depreciation rates. Simply use the optional factor entry for rates other than d 2 n .<br />

DDB (Double Declining Balance)<br />

Calculates the depreciation of an asset for a specified period n using the double declining balance<br />

method. A factor can also be entered for some other declining balance depreciation method by<br />

specifying a factor in the function.<br />

cost<br />

salvage<br />

life<br />

period<br />

factor<br />

DDB(cost, salvage, life, period, factor)<br />

First cost or basis of the asset.<br />

Salvage value of the asset.<br />

Depreciation life.<br />

The period, a year, for which the depreciation is to be calculated.<br />

(optional entry) If this entry is omitted, the function will use a double<br />

declining method with 2 times the straight line rate. If, for example,<br />

the entry is 1.5, the 150% declining balance method will be used.<br />

Example A new machine costs $200,000 and is expected to last 10 years. The salvage value is<br />

$10,000. Calculate the depreciation of the machine for the first and the eighth years. Finally,<br />

calculate the depreciation for the fifth year using the 175% declining balance method.<br />

Depreciation for the first year: DDB(200000,10000,10,1)<br />

Depreciation for the eighth year: DDB(200000,10000,10,8)<br />

Depreciation for the fifth year using 175% DB: DDB(200000,10000,10,5,1.75)<br />

EFFECT (Effective Interest Rate)<br />

Calculates the effective annual interest rate for a stated nominal annual rate and a given number<br />

of compounding periods per year. Excel uses Equation [4.7] to calculate the effective rate.<br />

EFFECT(nominal, npery)<br />

nominal Nominal interest rate for the year .<br />

npery Number of times interest is compounded per year.<br />

Example Claude has applied for a $10,000 loan. The bank officer told him that the interest rate<br />

is 8% per year and that interest is compounded monthly to conveniently match his monthly payments.<br />

What effective annual rate will Claude pay?<br />

Effective annual rate: EFFECT(8%,12)<br />

EFFECT can also be used to find effective rates other than annually. Enter the nominal rate for<br />

the time period of the required effective rate; npery is the number of times compounding occurs<br />

during the time period of the effective rate.<br />

Example Interest is stated as 3.5% per quarter with quarterly compounding. Find the effective<br />

semiannual rate.<br />

The 6-month nominal rate is 7%, and compounding is 2 times per 6 months.<br />

Effective semiannual rate: EFFECT(7%,2)


552 Appendix A Using Spreadsheets and Microsoft Excel ©<br />

FV (Future Value)<br />

Calculates the future value (worth) based on periodic payments at a specific interest rate.<br />

FV(rate, nper, pmt, pv, type)<br />

rate<br />

nper<br />

pmt<br />

pv<br />

type<br />

Interest rate per compounding period.<br />

Number of compounding periods.<br />

Constant payment amount.<br />

The present value amount. If pv is not specified, the function will<br />

assume it to be 0.<br />

(optional entry) Either 0 or 1. A 0 represents payments made at the<br />

end of the period, and 1 represents payments at the beginning of the<br />

period. If omitted, 0 is assumed.<br />

Example Jack wants to start a savings account that can be increased as desired. He will deposit<br />

$12,000 to start the account and plans to add $500 to the account at the beginning of each month<br />

for the next 24 months. The bank pays 0.25% per month. How much will be in Jack’s account at<br />

the end of 24 months?<br />

Future value in 24 months: FV(0.25%,24,500,12000,1)<br />

IF (IF Logical Function)<br />

Determines which of two entries is entered into a cell based on the outcome of a logical check on<br />

the outcome of another cell. The logical test can be a function or a simple value check, but it must<br />

use an equality or inequality sense. If the response is a text string, place it between quote marks<br />

(“ ”). The responses can themselves be IF functions. Up to seven IF functions can be nested for<br />

very complex logical tests.<br />

IF(logical_test,value_if_true,value_if_false)<br />

logical_test<br />

value_if_true<br />

value_if_false<br />

Any worksheet function can be used here, including a<br />

mathematical operation.<br />

Result if the logical_test argument is true.<br />

Result if the logical_test argument is false.<br />

Example The entry in cell B4 should be “selected” if the PW value in cell B3 is greater than<br />

or equal to zero and “rejected” if PW 0.<br />

Entry in cell B4: IF(B3 0,“selected”,“rejected”)<br />

Example The entry in cell C5 should be “selected” if the PW value in cell C4 is greater than<br />

or equal to zero, “rejected” if PW 0, and “fantastic” if PW 200.<br />

Entry in cell C5:<br />

IF(C40,“rejected”,IF(C4200,“fantastic”,“selected”))<br />

IPMT (Interest Payment)<br />

Calculates the interest accrued for a given period n based on constant periodic payments and<br />

interest rate.<br />

IPMT(rate, per, nper, pv, fv, type)<br />

rate Interest rate per compounding period.<br />

per Period for which interest is to be calculated.<br />

nper Number of compounding periods.<br />

pv Present value. If pv is not specified, the function will assume it to be 0.<br />

fv Future value. If fv is omitted, the function will assume it to be 0. The fv<br />

can also be considered a cash balance after the last payment is made.<br />

type (optional entry) Either 0 or 1. A 0 represents payments made at the<br />

end of the period, and 1 represents payments made at the beginning of<br />

the period. If omitted, 0 is assumed.


A.3 Excel Functions Important to <strong>Engineering</strong> <strong>Economy</strong> 553<br />

Example Calculate the interest due in the 10th month for a 48-month, $20,000 loan. The interest<br />

rate is 0.25% per month.<br />

Interest due: IPMT(0.25%,10,48,20000)<br />

IRR (Internal Rate of Return)<br />

Calculates the internal rate of return between 100% and infinity for a series of cash flows at<br />

regular periods.<br />

IRR(values, guess)<br />

values<br />

guess<br />

A set of numbers in a spreadsheet column (or row) for which the rate of<br />

return will be calculated. The set of numbers must consist of at least one<br />

positive and one negative number. Negative numbers denote a payment<br />

made or cash outflow, and positive numbers denote income or cash<br />

inflow.<br />

(optional entry) To reduce the number of iterations, a guessed rate of<br />

return can be entered. In most cases, a guess is not required, and a<br />

10% rate of return is initially assumed. If the #NUM! error appears, try<br />

using different values for guess. Inputting different guess values makes<br />

it possible to determine the multiple roots for the rate of return equation<br />

of a nonconventional cash flow series.<br />

Example John wants to start a printing business. He will need $25,000 in capital and anticipates<br />

that the business will generate the following incomes during the first 5 years. Calculate his<br />

rate of return after 3 years and after 5 years.<br />

Year 1 $5,000<br />

Year 2 $7,500<br />

Year 3 $8,000<br />

Year 4 $10,000<br />

Year 5 $15,000<br />

Set up an array in the spreadsheet.<br />

In cell A1, type 25000 (negative for payment).<br />

In cell A2, type 5000 (positive for income).<br />

In cell A3, type 7500.<br />

In cell A4, type 8000.<br />

In cell A5, type 10000.<br />

In cell A6, type 15000.<br />

Therefore, cells A1 through A6 contain the array of cash flows for the first 5 years, including the<br />

capital outlay. Note that any years with a zero cash fl ow must have a zero entered to ensure that<br />

the year value is correctly maintained for computation purposes.<br />

To calculate the internal rate of return after 3 years, move to cell A7, and type IRR(A1:A4).<br />

To calculate the internal rate of return after 5 years and specify a guess value of 5%, move to<br />

cell A8, and type IRR(A1:A6,5%).<br />

MIRR (Modified Internal Rate of Return)<br />

Calculates the modified internal rate of return for a series of cash flows and reinvestment of income<br />

and interest at a stated rate.<br />

MIRR(values, finance_rate, reinvest_rate)<br />

values<br />

Refers to an array of cells in the spreadsheet. Negative<br />

numbers represent payments, and positive numbers represent


554 Appendix A Using Spreadsheets and Microsoft Excel ©<br />

fi nance_rate<br />

reinvest_rate<br />

income. The series of payments and income must occur at<br />

regular periods and must contain at least one positive number<br />

and one negative number.<br />

Interest rate on funds borrowed from external sources<br />

( i b in Equation [7.9]).<br />

Interest rate for reinvestment on positive cash flows ( i i in<br />

Equation [7.9]). (This is not the same reinvestment rate on the<br />

net investments when the cash flow series is nonconventional.<br />

See Section 7.5 for comments.)<br />

Example Jane opened a hobby store 4 years ago. When she started the business, Jane borrowed<br />

$50,000 from a bank at 12% per year. Since then, the business has yielded $10,000 the<br />

first year, $15,000 the second year, $18,000 the third year, and $21,000 the fourth year. Jane<br />

reinvests her profits, earning 8% per year. What is the modified rate of return after 3 years and<br />

after 4 years?<br />

In cell A1, type 50000.<br />

In cell A2, type 10000.<br />

In cell A3, type 15000.<br />

In cell A4, type 18000.<br />

In cell A5, type 21000.<br />

To calculate the modified rate of return after 3 years, move to cell A6, and type<br />

MIRR(A1:A4,12%,8%).<br />

To calculate the modified rate of return after 4 years, move to cell A7, and type<br />

MIRR(A1:A5,12%,8%).<br />

NOMINAL (Nominal Interest Rate)<br />

Calculates the nominal annual interest rate for a stated effective annual rate and a given number<br />

of compounding periods per year. This function is designed to display only nominal annual rates.<br />

NOMINAL(effective, npery)<br />

effective Effective interest rate for the year .<br />

npery Number of times that interest is compounded per year.<br />

Example Last year, a corporate stock earned an effective return of 12.55% per year. Calculate<br />

the nominal annual rate, if interest is compounded quarterly and compounded continuously.<br />

Nominal annual rate, quarterly compounding: NOMINAL(12.55%,4)<br />

Nominal annual rate, continuous compounding: NOMINAL(12.55%,100000)<br />

NPER (Number of Periods)<br />

Calculates the number of periods for the present worth of an investment to equal the future value<br />

specified, based on uniform regular payments and a stated interest rate.<br />

NPER(rate, pmt, pv, fv, type)<br />

rate<br />

pmt<br />

pv<br />

fv<br />

type<br />

Interest rate per compounding period.<br />

Amount paid during each compounding period.<br />

Present value (lump-sum amount).<br />

(optional entry) Future value or cash balance after the last payment.<br />

If fv is omitted, the function will assume a value of 0.<br />

(optional entry) Enter 0 if payments are due at the end of the<br />

compounding period and 1 if payments are due at the beginning of the<br />

period. If omitted, 0 is assumed.


A.3 Excel Functions Important to <strong>Engineering</strong> <strong>Economy</strong> 555<br />

Example Sally plans to open a savings account that pays 0.25% per month. Her initial deposit<br />

is $3000, and she plans to deposit $250 at the beginning of every month. How many payments<br />

does she have to make to accumulate $25,000 to buy a new car?<br />

Number of payments: NPER(0.25%,250,3000,25000,1)<br />

NPV (Net Present Value)<br />

Calculates the net present value of a series of future cash flows at a stated interest rate.<br />

NPV(rate, series)<br />

rate<br />

series<br />

Interest rate per compounding period.<br />

Series of costs and incomes set up in a range of cells in the<br />

spreadsheet.<br />

Example Mark is considering buying a sports store for $100,000 and expects to receive the<br />

following income during the next 6 years of business: $25,000, $40,000, $42,000, $44,000,<br />

$48,000, $50,000. The interest rate is 8% per year.<br />

In cells A1 through A7, enter 100,000, followed by the six annual incomes.<br />

Present value: NPV(8%,A2:A7) A1<br />

The cell A1 value is already a present value. Any year with a zero cash fl ow must have a 0 entered<br />

to ensure a correct result.<br />

PMT (Payments)<br />

Calculates equivalent periodic amounts based on present value and/or future value at a constant<br />

interest rate.<br />

PMT(rate, nper, pv, fv, type)<br />

rate<br />

nper<br />

pv<br />

fv<br />

type<br />

Interest rate per compounding period.<br />

Total number of periods.<br />

Present value.<br />

Future value.<br />

(optional entry) Enter 0 for payments due at the end of the<br />

compounding period and 1 if payment is due at the start of the<br />

compounding period. If omitted, 0 is assumed.<br />

Example Jim plans to take a $15,000 loan to buy a new car. The interest rate is 7% per year.<br />

He wants to pay the loan off in 5 years (60 months). What are his monthly payments?<br />

Monthly payments: PMT(7%12,60,15000)<br />

PPMT (Principal Payment)<br />

Calculates the payment on the principal based on uniform payments at a specified interest<br />

rate.<br />

PPMT(rate, per, nper, pv, fv, type)<br />

rate<br />

per<br />

nper<br />

pv<br />

fv<br />

type<br />

Interest rate per compounding period.<br />

Period for which the payment on the principal is required.<br />

Total number of periods.<br />

Present value.<br />

Future value.<br />

(optional entry) Enter 0 for payments that are due at the end of the<br />

compounding period and 1 if payments are due at the start of the<br />

compounding period. If omitted, 0 is assumed.


556 Appendix A Using Spreadsheets and Microsoft Excel ©<br />

Example Jovita is planning to invest $10,000 in equipment which is expected to last 10 years<br />

with no salvage value. The interest rate is 5%. What is the principal payment at the end of<br />

year 4 and year 8?<br />

At the end of year 4: PPMT(5%,4,10,10000)<br />

At the end of year 8: PPMT(5%,8,10,10000)<br />

PV (Present Value)<br />

Calculates the present value of a future series of equal cash flows and a single lump sum in the<br />

last period at a constant interest rate.<br />

PV(rate, nper, pmt, fv, type)<br />

rate<br />

nper<br />

pmt<br />

fv<br />

type<br />

Interest rate per compounding period.<br />

Total number of periods.<br />

Cash flow at regular intervals. Negative numbers represent payments<br />

(cash outflows), and positive numbers represent income.<br />

Future value or cash balance at the end of the last period.<br />

(optional entry) Enter 0 if payments are due at the end of the<br />

compounding period and 1 if payments are due at the start of each<br />

compounding period. If omitted, 0 is assumed.<br />

There are two primary differences between the PV function and the NPV function: PV allows for<br />

end or beginning of period cash flows, and PV requires that all amounts have the same value,<br />

whereas they may vary for the NPV function.<br />

Example Jose is considering leasing a car for $300 a month for 3 years (36 months). After the<br />

36-month lease, he can purchase the car for $12,000. Using an interest rate of 8% per year, find<br />

the present value of this option.<br />

Present value: PV(8%12,36,300,12000)<br />

Note the minus signs on the pmt and fv amounts.<br />

RAND (Random Number)<br />

Returns an evenly distributed number that is (1) 0 and 1; (2) 0 and 100; or (3) between<br />

two specified numbers.<br />

RAND() for range 0 to 1<br />

RAND()*100 for range 0 to 100<br />

RAND()*(ba)a<br />

a minimum integer to be generated<br />

b maximum integer to be generated<br />

for range a to b<br />

The Excel function RANDBETWEEN(a,b) may also be used to obtain a random number between<br />

two values.<br />

Example Grace needs random numbers between 5 and 10 with 3 digits after the decimal. What<br />

is the Excel function? Here a 5 and b 10.<br />

Random number: RAND()*5 5<br />

Example Randi wants to generate random numbers between the limits of 10 and 25. What<br />

is the Excel function? The minimum and maximum values are a 10 and b 25, so b a <br />

25 (10) 35.<br />

Random number: RAND()*35 10


A.3 Excel Functions Important to <strong>Engineering</strong> <strong>Economy</strong> 557<br />

RATE (Interest Rate)<br />

Calculates the interest rate per compounding period for a series of payments or incomes.<br />

RATE(nper, pmt, pv, fv, type, guess)<br />

nper<br />

pmt<br />

pv<br />

fv<br />

type<br />

guess<br />

Total number of periods.<br />

Payment amount made each compounding period.<br />

Present value.<br />

Future value (not including the pmt amount).<br />

(optional entry) Enter 0 for payments due at the end of the<br />

compounding period and 1 if payments are due at the start of each<br />

compounding period. If omitted, 0 is assumed.<br />

(optional entry) To minimize computing time, include a guessed<br />

interest rate. If a value of guess is not specified, the function will<br />

assume a rate of 10%. This function usually converges to a solution if<br />

the rate is between 0% and 100%.<br />

Example Alysha wants to start a savings account at a bank. She will make an initial deposit of<br />

$1000 to open the account and plans to deposit $100 at the beginning of each month. She plans<br />

to do this for the next 3 years (36 months). At the end of 3 years, she wants to have at least $5000.<br />

What is the minimum interest required to achieve this result?<br />

Interest rate: RATE(36,100,1000,5000,1)<br />

SLN (Straight Line Depreciation)<br />

Calculates the straight line depreciation of an asset for a given year.<br />

SLN(cost, salvage, life)<br />

cost<br />

salvage<br />

life<br />

First cost or basis of the asset.<br />

Salvage value.<br />

Depreciation life.<br />

Example Maria purchased a printing machine for $100,000. The machine has an allowed depreciation<br />

life of 8 years and an estimated salvage value of $15,000. What is the depreciation<br />

each year?<br />

Depreciation: SLN(100000,15000,8)<br />

SYD (Sum-of-Years-Digits Depreciation)<br />

Calculates the sum-of-years-digits depreciation of an asset for a given year.<br />

SYD(cost, salvage, life, period)<br />

cost<br />

salvage<br />

life<br />

period<br />

First cost or basis of the asset.<br />

Salvage value.<br />

Depreciation life.<br />

The year for which the depreciation is sought.<br />

Example Jack bought equipment for $100,000 that has a depreciation life of 10 years. The<br />

salvage value is $10,000. What is the depreciation for year 1 and year 9?<br />

Depreciation for year 1: SYD(100000,10000,10,1)<br />

Depreciation for year 9: SYD(100000,10000,10,9)<br />

VDB (Variable Declining Balance)<br />

Calculates the depreciation using the declining balance method with a switch to straight line<br />

depreciation in the year in which straight line has a larger depreciation amount. This function


558 Appendix A Using Spreadsheets and Microsoft Excel ©<br />

automatically implements the switch from DB to SL depreciation, unless specifically instructed<br />

to not switch.<br />

VDB (cost, salvage, life, start_period, end_period, factor, no_switch)<br />

cost<br />

salvage<br />

life<br />

start_period<br />

end_period<br />

factor<br />

no_switch<br />

First cost of the asset.<br />

Salvage value.<br />

Depreciation life.<br />

First period for depreciation to be calculated.<br />

Last period for depreciation to be calculated.<br />

(optional entry) If omitted, the function will use the double<br />

declining rate of 2 n , or twice the straight line rate. Other entries<br />

define the declining balance method, for example, 1.5 for 150%<br />

declining balance.<br />

(optional entry) If omitted or entered as FALSE, the function will<br />

switch from declining balance to straight line depreciation when<br />

the latter is greater than DB depreciation. If entered as TRUE, the<br />

function will not switch to SL depreciation at any time during the<br />

depreciation life.<br />

Example Newly purchased equipment with a first cost of $300,000 has a depreciable life of<br />

10 years with no salvage value. Calculate the 175% declining balance depreciation for the first year<br />

and the ninth year if switching to SL depreciation is acceptable and if switching is not permitted.<br />

Depreciation for first year, with switching: VDB(300000,0,10,0,1,1.75)<br />

Depreciation for ninth year, with switching: VDB(300000,0,10,8,9,1.75)<br />

Depreciation for first year, no switching: VDB(300000,0,10,0,1,1.75,TRUE)<br />

Depreciation for ninth year, no switching: VDB(300000,0,10,8,9,1.75,TRUE)<br />

VDB (for MACRS Depreciation)<br />

The VDB function can be adapted to generate the MACRS annual depreciation amount, when the<br />

start_period and end_period are replaced with the MAX and MIN functions, respectively. As<br />

above, the factor option should be entered if other than DDB rates start the MACRS depreciation.<br />

The VDB format is<br />

VDB(cost,0,life,MAX(0,t1.5),MIN(life,t0.5),factor)<br />

Example Determine the MACRS depreciation for year 4 for a $350,000 asset that has a 20%<br />

salvage value and a MACRS recovery period of 3 years. D 4 $25,926 is the display.<br />

Depreciation for year 4: VDB(350000,0,3,MAX(0,41.5),MIN(3,40.5),2)<br />

Example Find the MACRS depreciation in year 16 for a $350,000-asset with a recovery<br />

period of n 15 years. The optional factor 1.5 is required here, since MACRS starts with<br />

150% DB for n 15-year and 20-year recovery periods. D 16 $10,334.<br />

Depreciation for year 16: VDB(350000,0,15,MAX(0,161.5),MIN(15,160.5),1.5)<br />

Other Functions<br />

There are numerous additional financial functions available on Excel, as well as engineering,<br />

mathematics, trigonometry, statistics, data and time, logical, and information functions. These<br />

can be viewed by clicking the Formulas tab on the Excel toolbar.<br />

A.4 Goal Seek—A Tool for Breakeven and<br />

Sensitivity Analysis<br />

Goal Seek is found on the Excel toolbar labeled Data, followed by What-if Analysis. This tool<br />

changes the value in a specific cell based on a numerical value in another (changing) cell as input<br />

by the user. It is a good tool for sensitivity analysis, breakeven analysis, and “what if?” questions


A.5 Solver—An Optimizing Tool for Capital Budgeting, Breakeven, and Sensitivity Analysis 559<br />

Figure A–3<br />

Goal Seek template used<br />

to specify a cell, a value,<br />

and the changing cell.<br />

B$5<br />

B$5500<br />

Figure A–4<br />

Use of Goal Seek to determine an annual cash flow to increase the rate of return.<br />

when no constraint relations or inequalities are needed. The initial Goal Seek template is pictured<br />

in Figure A–3 . One of the cells (set or changing cell) must contain an equation or spreadsheet<br />

function that uses the other cell to determine a numeric value. Only a single cell can be identified<br />

as the changing cell; however, this limitation can be avoided by using equations rather than specific<br />

numerical inputs in any additional cells also to be changed. This is demonstrated below.<br />

Example A new asset will cost $25,000, generate an annual cash flow of $6000 over its 5-year<br />

life, and have an estimated $500 salvage value. The rate of return using the IRR function is<br />

6.94%. Determine the annual cash flow necessary to raise the return to 10% per year.<br />

Figure A–4 (top left) shows the cash flows and return displayed using the function<br />

IRR(B4:B9) prior to the use of Goal Seek. Note that the initial $6000 is input in cell B5, but<br />

other years’ cash flows are input as equations that refer to B5. The $500 salvage is added for the<br />

last year. This format allows Goal Seek to change only cell B5 while making the other cash flows<br />

have the same value. The tool finds the required cash flow of $6506 to approximate the 10% per<br />

year return. The Goal Seek Status inset indicates that a solution is found. Clicking OK saves all<br />

changed cells; clicking Cancel returns to the original values.<br />

A.5 Solver—An Optimizing Tool for Capital Budgeting,<br />

Breakeven, and Sensitivity Analysis<br />

Solver is a powerful spreadsheet tool to change the value in multiple (one or more) cells based on<br />

the value in a specific (target) cell. It is excellent when solving a capital budgeting problem to<br />

select from independent projects where budget constraints are present. (Section 12.4 details this<br />

application.) The initial Solver template is shown in Figure A–5 .


560 Appendix A Using Spreadsheets and Microsoft Excel ©<br />

Figure A–5<br />

Solver template used to<br />

specify optimization in a<br />

target cell, multiple<br />

changing cells, and<br />

constraint relations.<br />

Set Target Cell box. Enter a cell reference or name. The target cell itself must contain a formula<br />

or function. The value in the cell can be maximized (Max), minimized (Min), or restricted<br />

to a specified value (Value of).<br />

By Changing Cells box. Enter the cell reference for each cell to be adjusted, using commas<br />

between nonadjacent cells. Each cell must be directly or indirectly related to the target cell.<br />

Solver proposes a value for the changing cell based on input provided about the target cell. The<br />

Guess button will list all possible changing cells related to the target cell.<br />

Subject to the Constraints box. Enter any constraints that may apply, for example,<br />

$C$1 $50,000. Integer and binary variables are determined in this box.<br />

Options box. Choices here allow the user to specify various parameters of the solution: maximum<br />

time and number of iterations allowed, the precision and tolerance of the values determined,<br />

and the convergence requirements as the final solution is determined. Also, linear and<br />

nonlinear model assumptions can be set here. If integer or binary variables are involved, the<br />

tolerance option must be set to a small number, say, 0.0001. This is especially important for the<br />

binary variables when selecting from independent projects (Chapter 12). If tolerance remains at<br />

the default value of 5%, a project may be incorrectly included in the solution set at a very low<br />

level.<br />

Solver Results box. This appears after Solve is clicked and a solution appears. It is possible,<br />

of course, that no solution can be found for the scenario described. It is possible to update the<br />

spreadsheet by clicking Keep Solver Solution, or return to the original entries using Restore<br />

Original Values.<br />

A.6 Error Messages<br />

If Excel is unable to complete a formula or function computation, an error message is displayed.<br />

Some of the common messages are:<br />

#DIV0! Requires division by zero.<br />

#NA Refers to a value that is not available.<br />

#NAME? Uses a name that Excel doesn’t recognize.<br />

#NULL! Specifies an invalid intersection of two areas.<br />

#NUM! Uses a number incorrectly.<br />

#REF! Refers to a cell that is not valid.<br />

#VALUE! Uses an invalid argument or operand.<br />

##### Produces a result, or includes a constant numeric value,<br />

that is too long to fit in the cell. (Widen the column.)


APPENDIX B<br />

BASICS OF ACCOUNTING<br />

REPORTS AND BUSINESS RATIOS<br />

This appendix provides a fundamental description of financial statements. The documents discussed<br />

here will assist in reviewing or understanding basic financial statements and in gathering<br />

information useful in an engineering economy study.<br />

B.1 The Balance Sheet<br />

The fiscal year and the tax year are defined identically for a corporation or an individual—<br />

12 months in length. The fiscal year (FY) is commonly not the calendar year (CY) for a corporation.<br />

The U.S. government uses October through September as its FY. For example, October<br />

2011 through September <strong>2012</strong> is FY<strong>2012</strong>. The fiscal or tax year is always the calendar year for<br />

an individual citizen.<br />

At the end of each fiscal year, a company publishes a balance sheet. A sample balance sheet<br />

for JAGBA Corporation is presented in Table B–1 . This is a yearly presentation of the state of the<br />

firm at a particular time, for example, May 31, <strong>2012</strong>; however, a balance sheet is also usually<br />

prepared quarterly and monthly. Three main categories are used.<br />

Assets. This section is a summary of all resources owned by or owed to the company. There<br />

are two main classes of assets. Current assets represent shorter-lived working capital (cash,<br />

accounts receivable, etc.), which is more easily converted to cash, usually within 1 year. Longerlived<br />

assets are referred to as fi xed assets (land, equipment, etc.). Conversion of these holdings<br />

to cash in a short time would require a major corporate reorientation.<br />

Liabilities. This section is a summary of all fi nancial obligations (debts, mortgages, loans,<br />

etc.) of a corporation. Bond indebtedness is included here.<br />

Net worth. Also called owner’s equity, this section provides a summary of the financial<br />

value of ownership, including stocks issued and earnings retained by the corporation.<br />

TABLE B–1<br />

Sample Balance Sheet<br />

Assets<br />

JAGBA CORPORATION<br />

Balance Sheet<br />

May 31, <strong>2012</strong><br />

Liabilities<br />

Current<br />

Cash $10,500 Accounts payable $19,700<br />

Accounts receivable 18,700 Dividends payable 7,000<br />

Interest accrued receivable 500 Long-term notes payable 16,000<br />

Inventories 52,000 Bonds payable 20,000<br />

Total current assets $81,700 Total liabilities $62,700<br />

Fixed<br />

Net Worth<br />

Land $25,000 Common stock $275,000<br />

Building and equipment 438,000 Preferred stock 100,000<br />

Less: Depreciation Retained earnings 25,000<br />

allowance $82,000 356,000<br />

Total fixed assets 381,000 Total net worth 400,000<br />

Total assets $462,700 Total liabilities and net worth $462,700


562 Appendix B Basics of Accounting Reports and Business Ratios<br />

The balance sheet is constructed using the relation<br />

Assets liabilities net worth<br />

In Table B–1 each major category is further divided into standard subcategories. For example,<br />

current assets is comprised of cash, accounts receivable, etc. Each subdivision has a specific interpretation,<br />

such as accounts receivable, which represents all money owed to the company by its<br />

customers.<br />

B.2 Income Statement and Cost of Goods<br />

Sold Statement<br />

A second important financial statement is the income statement ( Table B–2 ). The income statement<br />

summarizes the profits or losses of the corporation for a stated period of time. Income statements<br />

always accompany balance sheets. The major categories of an income statement are<br />

Revenues . This includes all sales and interest revenue that the company has received in the<br />

past accounting period.<br />

Expenses. This is a summary of all expenses (operating and others, including taxes) for<br />

the period. Some expense amounts are itemized in other statements, for example, cost of<br />

goods sold.<br />

The final result of an income statement is the net profit after taxes (NPAT), or NOPAT (O for<br />

operating), the amount used in Chapter 17, Sections 17.1 and 17.7. The income statement, published<br />

at the same time as the balance sheet, uses the basic equation<br />

Revenues expenses profit (or loss)<br />

The cost of goods sold is an important accounting term. It represents the net cost of producing<br />

the product marketed by the firm. Cost of goods sold may also be called factory cost. A statement<br />

of the cost of goods sold, such as that shown in Table B–3 , is useful in determining exactly how<br />

much it costs to make a particular product over a stated time period, usually a year. The total of<br />

the cost of goods sold statement is entered as an expense item on the income statement. This total<br />

is determined using the relations<br />

Cost of goods sold prime cost indirect cost<br />

Prime cost direct materials direct labor<br />

[B.1]<br />

TABLE B–2<br />

Sample Income Statement<br />

JAGBA CORPORATION<br />

Income Statement<br />

Year Ended May 31, <strong>2012</strong><br />

Revenues<br />

Sales $505,000<br />

Interest revenue 3,500<br />

Total revenues $508,500<br />

Expenses<br />

Cost of goods sold (from Table B–3) $290,000<br />

Selling 28,000<br />

Administrative 35,000<br />

Other 12,000<br />

Total expenses 365,000<br />

Income before taxes 143,500<br />

Taxes for year 64,575<br />

Net profit after taxes (NPAT) $ 78,925


B.3 Business Ratios 563<br />

TABLE B–3<br />

Sample Cost of Goods Sold Statement<br />

JAGBA CORPORATION<br />

Statement of Cost of Goods Sold<br />

Year Ended May 31, <strong>2012</strong><br />

Materials<br />

Inventory June 1, 2011 $ 54,000<br />

Purchases during year 174,500<br />

Total $228,500<br />

Less: Inventory May 31, <strong>2012</strong> 50,000<br />

Cost of materials $178,500<br />

Direct labor 110,000<br />

Prime cost 288,500<br />

Indirect costs 7,000<br />

Factory cost 295,500<br />

Less: Increase in finished goods inventory during year 5,500<br />

Cost of goods sold (into Table B–2) $290,000<br />

Indirect costs include all indirect and overhead charges made to a product, process, or cost center.<br />

Indirect cost allocation methods are discussed in Chapter 15.<br />

B.3 Business Ratios<br />

Accountants, financial analysts, and engineering economists frequently utilize business ratio<br />

analysis to evaluate the financial health (status) of a company over time and in relation to industry<br />

norms. Because the engineering economist must continually communicate with others, she or<br />

he should have a basic understanding of several ratios. For comparison purposes, it is necessary<br />

to compute the ratios for several companies in the same industry. Industrywide median ratio values<br />

are published annually by firms such as Dun and Bradstreet in Industry Norms and Key Business<br />

Ratios. The ratios are classified according to their role in measuring the corporation.<br />

Solvency ratios. Assess ability to meet short-term and long-term financial obligations.<br />

Efficiency ratios. Measure management’s ability to use and control assets.<br />

Profitability ratios. Evaluate the ability to earn a return for the owners of the corporation.<br />

Numerical data for several important ratios are discussed here and are extracted from the JAGBA<br />

balance sheet and income statement, Tables B–1 and B–2.<br />

Current Ratio This ratio is utilized to analyze the company’s working capital condition. It is<br />

defined as<br />

Current ratio ———————<br />

current assets<br />

current liabilities<br />

Current liabilities include all short-term debts, such as accounts and dividends payable. Note that<br />

only balance sheet data are utilized in the current ratio; that is, no association with revenues or<br />

expenses is made. For the balance sheet of Table B–1 , current liabilities amount to $19,700 <br />

$7000 $26,700 and<br />

Current ratio ———<br />

81,700<br />

26,700 3.06<br />

Since current liabilities are those debts payable in the next year, the current ratio value of 3.06<br />

means that the current assets would cover short-term debts approximately 3 times. Current ratio<br />

values of 2 to 3 are common.<br />

The current ratio assumes that the working capital invested in inventory can be converted to<br />

cash quite rapidly. Often, however, a better idea of a company’s immediate financial position can<br />

be obtained by using the acid test ratio.


564 Appendix B Basics of Accounting Reports and Business Ratios<br />

Acid Test Ratio (Quick Ratio) This ratio is<br />

quick assets<br />

Acid-test ratio ———————<br />

current liabilities<br />

————————————<br />

current assets inventories<br />

current liabilities<br />

It is meaningful for the emergency situation when the firm must cover short-term debts using its<br />

readily convertible assets. For JAGBA Corporation,<br />

81,700 52,000<br />

Acid test ratio ——————— 1.11<br />

26,700<br />

Comparison of this and the current ratio shows that approximately 2 times the current debt of the<br />

company is invested in inventories. However, an acid test ratio of approximately 1.0 is generally<br />

regarded as a strong current position, regardless of the amount of assets in inventories.<br />

Debt Ratio This ratio is a measure of financial strength since it is defined as<br />

Debt ratio ——————<br />

total liabilities<br />

total assets<br />

For JAGBA Corporation,<br />

Debt ratio ————<br />

62,700<br />

462,700 0.136<br />

JAGBA is 13.6% creditor-owned and 86.4% stockholder-owned. A debt ratio in the range of 20%<br />

or less usually indicates a sound financial condition, with little fear of forced reorganization because<br />

of unpaid liabilities. However, a company with virtually no debts, that is, one with a very low debt<br />

ratio, may not have a promising future, because of its inexperience in dealing with short-term and<br />

long-term debt financing. The debt-equity (D-E) mix is another measure of financial strength.<br />

Return on Sales Ratio This often quoted ratio indicates the profit margin for the company. It<br />

is defined as<br />

net profit<br />

Return on sales ————<br />

net sales (100%)<br />

Net profit is the after-tax value from the income statement. This ratio measures profit earned per<br />

sales dollar and indicates how well the corporation can sustain adverse conditions over time, such<br />

as falling prices, rising costs, and declining sales. For JAGBA Corporation,<br />

Return on sales ————<br />

78,925 (100%) 15.6%<br />

505,000<br />

Corporations may point to small return on sales ratios, say, 2.5% to 4.0%, as indications of sagging<br />

economic conditions. In truth, for a relatively large-volume, high-turnover business, an income<br />

ratio of 3% is quite healthy. Of course, a steadily decreasing ratio indicates rising company<br />

expenses, which absorb net profit after taxes.<br />

Return on Assets Ratio This is the key indicator of profitability since it evaluates the ability<br />

of the corporation to transfer assets into operating profit. The definition and value for<br />

JAGBA are<br />

net profit<br />

Return on assets —————<br />

total assets (100%)<br />

————<br />

78,925 (100%) 17.1%<br />

462,700<br />

Efficient use of assets indicates that the company should earn a high return, while low returns<br />

usually accompany lower values of this ratio compared to the industry group ratios.<br />

Inventory Turnover Ratio Two different ratios are used here. They both indicate the number<br />

of times the average inventory value passes through the operations of the company. If turnover of<br />

inventory to net sales is desired, the formula is<br />

Net sales to inventory ————————<br />

net sales<br />

average inventory


B.3 Business Ratios 565<br />

where average inventory is the figure recorded in the balance sheet. For JAGBA Corporation this<br />

ratio is<br />

Net sales to inventory ————<br />

505,000<br />

52,000 9.71<br />

This means that the average value of the inventory has been sold 9.71 times during the year.<br />

Values of this ratio vary greatly from one industry to another.<br />

If inventory turnover is related to cost of goods sold, the ratio to use is<br />

cost of goods sold<br />

Cost of goods sold to inventory ————————<br />

average inventory<br />

Now, average inventory is computed as the average of the beginning and ending inventory values<br />

in the statement of cost of goods sold. This ratio is commonly used as a measure of the inventory<br />

turnover rate in manufacturing companies. It varies with industries, but management likes to see<br />

it remain relatively constant as business increases. For JAGBA, using the values in Table B–3 ,<br />

290,000<br />

Cost of goods sold to inventory ————————— 5.58<br />

1_ (54,000 50,000)<br />

2<br />

There are, of course, many other ratios to use in various circumstances; however, the ones<br />

presented here are commonly used by both accountants and economic analysts.<br />

EXAMPLE B.1<br />

Sample values for financial ratios or percentages of four industry sectors are presented below.<br />

Compare the corresponding JAGBA Corporation values with these norms, and comment on<br />

differences and similarities.<br />

Ratio or<br />

Percentage<br />

Motor Vehicles<br />

and Parts<br />

Manufacturing<br />

336105*<br />

Air<br />

Transportation<br />

(Medium-Sized)<br />

481000*<br />

Industrial<br />

Machinery<br />

Manufacturing<br />

333200*<br />

Home<br />

Furnishings<br />

442000*<br />

Current ratio 2.4 0.4 2.2 2.6<br />

Quick ratio 1.6 0.3 1.5 1.2<br />

Debt ratio 59.3% 96.8% 49.1% 52.4%<br />

Return on<br />

assets<br />

40.9% 8.1% 8.0% 5.1%<br />

*North American Industry Classification System (NAICS) code for this industry sector.<br />

SOURCE: L. Troy, Almanac of Business and Industrial Financial Ratios, CCH, Wolters Kluwer, USA.<br />

Solution<br />

It is not correct to compare ratios for one company with indexes in different industries, that is,<br />

with indexes for different NAICS codes. So the comparison below is for illustration purposes<br />

only. The corresponding values for JAGBA are<br />

Current ratio 3.06<br />

Quick ratio 1.11<br />

Debt ratio 13.5%<br />

Return on assets 17.1%<br />

JAGBA has a current ratio larger than all four of these industries, since 3.06 indicates it can<br />

cover current liabilities 3 times compared with 2.6 and much less in the case of the “average”<br />

air transportation corporation. JAGBA has a significantly lower debt ratio than that of any of<br />

the sample industries, so it is likely more financially sound. Return on assets, which is a measure<br />

of ability to turn assets into profitability, is not as high at JAGBA as motor vehicles, but<br />

JAGBA competes well with the other industry sectors.<br />

To make a fair comparison of JAGBA ratios with other values, it is necessary to have norm<br />

values for its industry type as well as ratio values for other corporations in the same NAICS<br />

category and about the same size in total assets. Corporate assets are classified in categories by<br />

$100,000 units, such as 100 to 250, 1001 to 5000, over 250,000, etc.


APPENDIX C<br />

CODE OF ETHICS FOR ENGINEERS<br />

Source: National Society of Professional Engineers (www.nspe.org).


Appendix C Code of Ethics for Engineers 567<br />

Code of Ethics for Engineers<br />

Preamble<br />

<strong>Engineering</strong> is an important and learned profession. As members of this<br />

profession, engineers are expected to exhibit the highest standards of honesty<br />

and integrity. <strong>Engineering</strong> has a direct and vital impact on the quality of life for<br />

all people. Accordingly, the services provided by engineers require honesty,<br />

impartiality, fairness, and equity, and must be dedicated to the protection of the<br />

public health, safety, and welfare. Engineers must perform under a standard of<br />

professional behavior that requires adherence to the highest principles of ethical<br />

conduct.<br />

I. Fundamental Canons<br />

Engineers, in the fulfillment of their professional duties, shall:<br />

1. Hold paramount the safety, health, and welfare of the public.<br />

2. Perform services only in areas of their competence.<br />

3. Issue public statements only in an objective and truthful manner.<br />

4. Act for each employer or client as faithful agents or trustees.<br />

5. Avoid deceptive acts.<br />

6. Conduct themselves honorably, responsibly, ethically, and<br />

lawfully so as to enhance the honor, reputation, and usefulness<br />

of the profession.<br />

II. Rules of Practice<br />

1. Engineers shall hold paramount the safety, health, and welfare<br />

of the public.<br />

a. If engineers’ judgment is overruled under circumstances that<br />

endanger life or property, they shall notify their employer or client<br />

and such other authority as may be appropriate.<br />

b. Engineers shall approve only those engineering documents that are<br />

in conformity with applicable standards.<br />

c. Engineers shall not reveal facts, data, or information without the<br />

prior consent of the client or employer except as authorized or<br />

required by law or this Code.<br />

d. Engineers shall not permit the use of their name or associate in<br />

business ventures with any person or firm that they believe is<br />

engaged in fraudulent or dishonest enterprise.<br />

e. Engineers shall not aid or abet the unlawful practice of engineering<br />

by a person or firm.<br />

f. Engineers having knowledge of any alleged violation of this Code<br />

shall report thereon to appropriate professional bodies and, when<br />

relevant, also to public authorities, and cooperate with the proper<br />

authorities in furnishing such information or assistance as may be<br />

required.<br />

2. Engineers shall perform services only in the areas of their<br />

competence.<br />

a. Engineers shall undertake assignments only when qualified by<br />

education or experience in the specific technical fields involved.<br />

b. Engineers shall not affix their signatures to any plans or documents<br />

dealing with subject matter in which they lack competence, nor to<br />

any plan or document not prepared under their direction and<br />

control.<br />

c. Engineers may accept assignments and assume responsibility for<br />

coordination of an entire project and sign and seal the engineering<br />

documents for the entire project, provided that each technical<br />

segment is signed and sealed only by the qualified engineers who<br />

prepared the segment.<br />

3. Engineers shall issue public statements only in an objective and<br />

truthful manner.<br />

a. Engineers shall be objective and truthful in professional reports,<br />

statements, or testimony. They shall include all relevant and<br />

pertinent information in such reports, statements, or testimony,<br />

which should bear the date indicating when it was current.<br />

b. Engineers may express publicly technical opinions that are founded<br />

upon knowledge of the facts and competence in the subject matter.<br />

c. Engineers shall issue no statements, criticisms, or arguments on<br />

technical matters that are inspired or paid for by interested parties,<br />

unless they have prefaced their comments by explicitly identifying<br />

the interested parties on whose behalf they are speaking, and by<br />

revealing the existence of any interest the engineers may have in the<br />

matters.<br />

4. Engineers shall act for each employer or client as faithful agents or<br />

trustees.<br />

a. Engineers shall disclose all known or potential conflicts of interest<br />

that could influence or appear to influence their judgment or the<br />

quality of their services.<br />

b. Engineers shall not accept compensation, financial or otherwise,<br />

from more than one party for services on the same project, or for<br />

services pertaining to the same project, unless the circumstances are<br />

fully disclosed and agreed to by all interested parties.<br />

c. Engineers shall not solicit or accept financial or other valuable<br />

consideration, directly or indirectly, from outside agents in<br />

connection with the work for which they are responsible.<br />

d. Engineers in public service as members, advisors, or employees<br />

of a governmental or quasi-governmental body or department shall<br />

not participate in decisions with respect to services solicited or<br />

provided by them or their organizations in private or public<br />

engineering practice.<br />

e. Engineers shall not solicit or accept a contract from a governmental<br />

body on which a principal or officer of their organization serves as<br />

a member.<br />

5. Engineers shall avoid deceptive acts.<br />

a. Engineers shall not falsify their qualifications or permit<br />

misrepresentation of their or their associates’ qualifications. They<br />

shall not misrepresent or exaggerate their responsibility in or for the<br />

subject matter of prior assignments. Brochures or other<br />

presentations incident to the solicitation of employment shall not<br />

misrepresent pertinent facts concerning employers, employees,<br />

associates, joint venturers, or past accomplishments.<br />

b. Engineers shall not offer, give, solicit, or receive, either directly or<br />

indirectly, any contribution to influence the award of a contract by<br />

public authority, or which may be reasonably construed by the<br />

public as having the effect or intent of influencing the awarding of a<br />

contract. They shall not offer any gift or other valuable<br />

consideration in order to secure work. They shall not pay a<br />

commission, percentage, or brokerage fee in order to secure work,<br />

except to a bona fide employee or bona fide established commercial<br />

or marketing agencies retained by them.<br />

III. Professional Obligations<br />

1. Engineers shall be guided in all their relations by the highest standards<br />

of honesty and integrity.<br />

a. Engineers shall acknowledge their errors and shall not distort or<br />

alter the facts.<br />

b. Engineers shall advise their clients or employers when they believe<br />

a project will not be successful.<br />

c. Engineers shall not accept outside employment to the detriment of<br />

their regular work or interest. Before accepting any outside<br />

engineering employment, they will notify their employers.<br />

d. Engineers shall not attempt to attract an engineer from another<br />

employer by false or misleading pretenses.<br />

e. Engineers shall not promote their own interest at the expense of the<br />

dignity and integrity of the profession.<br />

2. Engineers shall at all times strive to serve the public interest.<br />

a. Engineers are encouraged to participate in civic affairs; career<br />

guidance for youths; and work for the advancement of the safety,<br />

health, and well-being of their community.<br />

b. Engineers shall not complete, sign, or seal plans and/or<br />

specifications that are not in conformity with applicable engineering<br />

standards. If the client or employer insists on such unprofessional<br />

conduct, they shall notify the proper authorities and withdraw from<br />

further service on the project.<br />

c. Engineers are encouraged to extend public knowledge and<br />

appreciation of engineering and its achievements.<br />

d. Engineers are encouraged to adhere to the principles of sustainable<br />

development 1 in order to protect the environment for future<br />

generations.


568 Appendix C Code of Ethics for Engineers<br />

3. Engineers shall avoid all conduct or practice that deceives the public.<br />

a. Engineers shall avoid the use of statements containing a material<br />

misrepresentation of fact or omitting a material fact.<br />

b. Consistent with the foregoing, engineers may advertise for<br />

recruitment of personnel.<br />

c. Consistent with the foregoing, engineers may prepare articles for<br />

the lay or technical press, but such articles shall not imply credit to<br />

the author for work performed by others.<br />

4. Engineers shall not disclose, without consent, confidential information<br />

concerning the business affairs or technical processes of any present or<br />

former client or employer, or public body on which they serve.<br />

a. Engineers shall not, without the consent of all interested parties,<br />

promote or arrange for new employment or practice in connection<br />

with a specific project for which the engineer has gained particular<br />

and specialized knowledge.<br />

b. Engineers shall not, without the consent of all interested parties,<br />

participate in or represent an adversary interest in connection with a<br />

specific project or proceeding in which the engineer has gained<br />

particular specialized knowledge on behalf of a former client or<br />

employer.<br />

5. Engineers shall not be influenced in their professional duties by<br />

conflicting interests.<br />

a. Engineers shall not accept financial or other considerations,<br />

including free engineering designs, from material or equipment<br />

suppliers for specifying their product.<br />

b. Engineers shall not accept commissions or allowances, directly or<br />

indirectly, from contractors or other parties dealing with clients or<br />

employers of the engineer in connection with work for which the<br />

engineer is responsible.<br />

6. Engineers shall not attempt to obtain employment or advancement or<br />

professional engagements by untruthfully criticizing other engineers,<br />

or by other improper or questionable methods.<br />

a. Engineers shall not request, propose, or accept a commission on a<br />

contingent basis under circumstances in which their judgment may<br />

be compromised.<br />

b. Engineers in salaried positions shall accept part-time engineering<br />

work only to the extent consistent with policies of the employer and<br />

in accordance with ethical considerations.<br />

c. Engineers shall not, without consent, use equipment, supplies,<br />

laboratory, or office facilities of an employer to carry on outside<br />

private practice.<br />

7. Engineers shall not attempt to injure, maliciously or falsely, directly<br />

or indirectly, the professional reputation, prospects, practice, or<br />

employment of other engineers. Engineers who believe others are<br />

guilty of unethical or illegal practice shall present such information<br />

to the proper authority for action.<br />

a. Engineers in private practice shall not review the work of another<br />

engineer for the same client, except with the knowledge of such<br />

engineer, or unless the connection of such engineer with the work<br />

has been terminated.<br />

b. Engineers in governmental, industrial, or educational employ are<br />

entitled to review and evaluate the work of other engineers when so<br />

required by their employment duties.<br />

c. Engineers in sales or industrial employ are entitled to make<br />

engineering comparisons of represented products with products of<br />

other suppliers.<br />

8. Engineers shall accept personal responsibility for their professional<br />

activities, provided, however, that engineers may seek indemnification<br />

for services arising out of their practice for other than gross<br />

negligence, where the engineer’s interests cannot otherwise be<br />

protected.<br />

a. Engineers shall conform with state registration laws in the practice<br />

of engineering.<br />

b. Engineers shall not use association with a nonengineer, a<br />

corporation, or partnership as a “cloak” for unethical acts.<br />

9. Engineers shall give credit for engineering work to those to whom<br />

credit is due, and will recognize the proprietary interests of others.<br />

a. Engineers shall, whenever possible, name the person or persons<br />

who may be individually responsible for designs, inventions,<br />

writings, or other accomplishments.<br />

b. Engineers using designs supplied by a client recognize that the<br />

designs remain the property of the client and may not be duplicated<br />

by the engineer for others without express permission.<br />

c. Engineers, before undertaking work for others in connection with<br />

which the engineer may make improvements, plans, designs,<br />

inventions, or other records that may justify copyrights or patents,<br />

should enter into a positive agreement regarding ownership.<br />

d. Engineers’ designs, data, records, and notes referring exclusively to<br />

an employer’s work are the employer’s property. The employer<br />

should indemnify the engineer for use of the information for any<br />

purpose other than the original purpose.<br />

e. Engineers shall continue their professional development throughout<br />

their careers and should keep current in their specialty fields by<br />

engaging in professional practice, participating in continuing<br />

education courses, reading in the technical literature, and attending<br />

professional meetings and seminars.<br />

Footnote 1 “Sustainable development” is the challenge of meeting human<br />

needs for natural resources, industrial products, energy, food,<br />

transportation, shelter, and effective waste management while<br />

conserving and protecting environmental quality and the natural<br />

resource base essential for future development.<br />

As Revised July 2007<br />

“By order of the United States District Court for the District of Columbia,<br />

former Section 11(c) of the NSPE Code of Ethics prohibiting competitive<br />

bidding, and all policy statements, opinions, rulings or other guidelines<br />

interpreting its scope, have been rescinded as unlawfully interfering with the<br />

legal right of engineers, protected under the antitrust laws, to provide price<br />

information to prospective clients; accordingly, nothing contained in the NSPE<br />

Code of Ethics, policy statements, opinions, rulings or other guidelines prohibits<br />

the submission of price quotations or competitive bids for engineering services<br />

at any time or in any amount.”<br />

Statement by NSPE Executive Committee<br />

In order to correct misunderstandings which have been indicated in some<br />

instances since the issuance of the Supreme Court decision and the entry of the<br />

Final Judgment, it is noted that in its decision of April 25, 1978, the Supreme<br />

Court of the United States declared: “The Sherman Act does not require<br />

competitive bidding.”<br />

It is further noted that as made clear in the Supreme Court decision:<br />

1. Engineers and firms may individually refuse to bid for engineering services.<br />

2. Clients are not required to seek bids for engineering services.<br />

3. Federal, state, and local laws governing procedures to procure engineering<br />

services are not affected, and remain in full force and effect.<br />

4. State societies and local chapters are free to actively and aggressively seek<br />

legislation for professional selection and negotiation procedures by public<br />

agencies.<br />

5. State registration board rules of professional conduct, including rules<br />

prohibiting competitive bidding for engineering services, are not affected and<br />

remain in full force and effect. State registration boards with authority to<br />

adopt rules of professional conduct may adopt rules governing procedures to<br />

obtain engineering services.<br />

6. As noted by the Supreme Court, “nothing in the judgment prevents NSPE and<br />

its members from attempting to influence governmental action . . .”<br />

Note: In regard to the question of application of the Code to corporations vis-a-vis real persons, business form or type should not negate nor<br />

influence conformance of individuals to the Code. The Code deals with professional services, which services must be performed by real<br />

persons. Real persons in turn establish and implement policies within business structures. The Code is clearly written to apply to the Engineer,<br />

and it is incumbent on members of NSPE to endeavor to live up to its provisions. This applies to all pertinent sections of the Code.<br />

1420 King Street<br />

Alexandria, Virginia 22314-2794<br />

703/684-2800 • Fax:703/836-4875<br />

www.nspe.org<br />

Publication date as revised: July 2007 • Publication #1102


APPENDIX D<br />

ALTERNATE METHODS FOR<br />

EQUIVALENCE CALCULATIONS<br />

Throughout the text, engineering economy factor formulas, tabulated factor values, or built-in<br />

spreadsheet functions have been used to obtain a value of P, F, A, i , or n . Because of advances in<br />

programmable and scientific calculators, many of the equivalence computations can be performed<br />

without the use of tables or spreadsheets, but rather with a handheld calculator. An overview<br />

of the possibilities is presented here.<br />

Alternatively, the recognition that all equivalence calculations involve geometric series can<br />

likewise remove the need for tabulated values or spreadsheet functions. From the summation of<br />

the series, it is possible to perform calculator-based computations to obtain P, F , or A values. A<br />

brief introduction to this technique is presented in Section D.2.<br />

D.1 Using Programmable Calculators<br />

A basic way to calculate one parameter, given the other four, is to use a calculator that allows a<br />

present worth relation to be encoded. The software can then solve for any one of the parameters,<br />

when the remaining four are entered. For example, consider a PW relation in which all five parameters<br />

are included.<br />

A ( P/A,i,n ) F ( P/F,i,n ) P 0<br />

The A, P, and F values can be positive (cash inflow) or negative (cash outflow) or zero, as long<br />

as there is at least one value with each sign. The interest rate i can be coded for entry as a percent<br />

or decimal. When the unknown variable is identified, the calculator’s software can solve the<br />

equation for zero, thus providing the answer.<br />

This is the approach taken by relatively simple scientific calculators, such as the Hewlett-<br />

Packard (HP) scientific series, for example, HP 33s. By substituting the formulas for the factors,<br />

the actual relation entered into the calculator is<br />

n<br />

A [<br />

1 (1 i/100)<br />

————————<br />

i/100 ] F [1 (i/100)] n P 0<br />

The HP calculator uses a slightly different symbol set than we have used thus far. The initial investment<br />

is called B rather than P , and the equal uniform amount is termed P rather than A . Once<br />

entered, the relation can be solved for any one variable, given values for the other four.<br />

Another example that offers freedom from the spreadsheet and tables is an engineering calculator<br />

that has the same functions built in as those on a spreadsheet to determine P, F, A, i , or n . An<br />

example of this higher level is Texas Instrument’s TI-Nspire series. The functions are basically<br />

the same as those on a spreadsheet. They are clustered under the heading of tvm (time value of<br />

money) functions. For example, the tvmPV function format is<br />

where<br />

tvmPV(n,i,Pmt,FV,PpY,CpY,PmtAt)<br />

n number of periods<br />

i annual interest rate as a percent<br />

Pmt equal uniform periodic amount A<br />

FV future amount F<br />

PpY payments per year (optional; default is 1)<br />

CpY compounding periods per year (optional; default is 1)<br />

PmtAt beginning- or end-of-period payments (optional; default is 0 end)


570 Appendix D Alternate Methods for Equivalence Calculations<br />

It is easy to understand why it is possible to do a lot on a calculator with relatively wellbehaved<br />

cash flow series. As the series become more complex, it is necessary to move to a spreadsheet<br />

for speed and versatility. However, the use of tables or factor formulas is not necessary.<br />

D.2 Using the Summation of a Geometric Series<br />

A geometric progression is a series of n terms with a common ratio or base r . If c is a constant for<br />

each term, the series is written in the form<br />

cr a cr a 1 ⋅ ⋅ ⋅ cr n c<br />

r j<br />

ja<br />

The sum S of a geometric series adds the terms using the closed-end form<br />

jn<br />

S r n1 r a<br />

————<br />

r 1<br />

[D.1]<br />

Ristroph<br />

1<br />

and others have explained how the recognition that equivalence computations are<br />

simple applications of geometric series can be used to determine F, P , and A using Equation<br />

[D.1] and a simple handheld calculator with exponentiation capability.<br />

Before explaining how to apply this approach, we define the base r as follows when a future<br />

worth F or present worth P is sought.<br />

To find F :<br />

To find P :<br />

r 1 i<br />

r (1 i) 1<br />

A very familiar application of geometric series is the determination of the equivalent future worth<br />

F in year n for a single present worth amount P in year 0. This is the same as using a geometric<br />

series of only one term. As shown in Figure D–1 , if P $100, n 10 years, and i 10% per<br />

year, when r 1 i ,<br />

F P (1 i )<br />

n<br />

P ( r )<br />

n<br />

100(1.1) 10<br />

100(2.5937)<br />

$259.37<br />

This is identical to using the tabulated value (or formula) for the F/P factor.<br />

F P ( F/P,i,n ) 100( F/P ,10%,10) 100(2.5937)<br />

$259.37<br />

Figure D–2 shows a uniform annual series A $100 for 10 years. To calculate F , we can move<br />

each A value forward to year 10. Place a subscript j on each A value to indicate the year of occurrence,<br />

and determine F for each A j value.<br />

F A 1 (1 i ) 9 A 2 (1 i ) 8 ⋅ ⋅ ⋅ A 9 (1 i )<br />

1 A 10 (1 i ) 0<br />

Figure D–1<br />

Future worth of a single<br />

amount in year 0.<br />

0<br />

F = ?<br />

i = 10%<br />

1 2 3 4 5 6 7 8 9 10 Year<br />

1 2 3 4 5 6 7 8 9 10 Times<br />

compounded<br />

P = $100<br />

1<br />

J. H. Ristroph, “<strong>Engineering</strong> Economics: Time for New Directions?” Proceedings , ASEE<br />

Annual Conference, Austin, TX, June 2009.


D.2 Using the Summation of a Geometric Series 571<br />

F = ?<br />

A $100<br />

i = 10%<br />

0 1 2 3 4 5 6 7 8 9 10<br />

Year<br />

Times<br />

9 8 7 6 5 4 3 2 1<br />

compounded<br />

0<br />

Figure D–2<br />

Future worth of an A series from year 1 through year 10.<br />

P = ?<br />

i = 10%<br />

0 1 2 3 4 5 6 7 8 9 10<br />

Year<br />

A $100<br />

5 6 7 8 9 10<br />

Figure D–3<br />

Present worth of a shifted uniform series.<br />

Times<br />

discounted<br />

Note that the first value A 1 is compounded 9 times, not 10. With r 1 i , the geometric series<br />

and its sum S (from Equation [D.1]) can be developed. Removing the subscript on A ,<br />

j9<br />

F = A [ r 9 r 8 ⋅ ⋅ ⋅ r 1 r 0 j<br />

] A r<br />

j0<br />

S ————<br />

r10 r 0<br />

r 1 (1.1)10 1<br />

————— 15.9374<br />

0.1<br />

The F value is the same whether using the geometric series sum or the tabulated F/A factor.<br />

F 100( S ) 100(15.9374) $1593.74<br />

F 100( F/A ,10%,10) 100(15.9374) $1593.74<br />

As a final demonstration of the geometric series approach to equivalence computations, consider<br />

the shifted cash flow series in Figure D–3 . The A series is present from years 5 through 10,<br />

and the P value is sought. If the tables are used, the solution is<br />

P A ( P/A ,10%,6)( P/F ,10%,4) = 100(4.3553)(0.6830)<br />

$297.47


572 Appendix D Alternate Methods for Equivalence Calculations<br />

As shown in the figure, this is a geometric series with the first A value discounted 5 years, therefore,<br />

a 5. The last A term is discounted 10 years, making n 10. Since P is sought, the geometric<br />

series base is r (1 i )<br />

1<br />

. Again using Equation [D.1] for the summation, we have<br />

j10<br />

P A<br />

[ r<br />

] j j5<br />

100 [ r 11 r 5<br />

————<br />

r 1 ] 100 [ (1/1.1) 11 (1/1.1) 5<br />

————————<br />

100 [ (0.9091) 11 (0.9091) 5<br />

——————————<br />

0.9091 1 ] 100(2.9747)<br />

$297.47<br />

(1/1.1) 1 ]<br />

It is possible to develop similar relations to handle arithmetic and geometric series, conversions<br />

from P to A , and vice versa. Proponents of this approach point out the use of standard<br />

mathematical notation; the removal of a need to derive any factors; no need to remember the<br />

placement of P, F, and A values based on factor formula development; and the easy use of a calculator<br />

to determine the equivalence relations. As with the use of programmable calculators, the<br />

technique is excellent for well-behaved and reasonably complex series. When the series become<br />

quite involved, or when sensitivity analysis is required to reach an economic decision, it may<br />

be beneficial to use a spreadsheet. This often helps in performing sidebar and ancillary calculations<br />

that assist in the understanding of the problem, not just the math computations necessary to<br />

obtain an answer. However, once again, the use of tables and factors is not necessary.


APPENDIX E<br />

GLOSSARY OF CONCEPTS<br />

AND TERMS<br />

E.1 Important Concepts and Guidelines<br />

The following elements of engineering economy are identified throughout the text in the margin<br />

by this checkmark and a title below it. The numbers in parentheses indicate chapters where the<br />

concept or guideline is introduced or essential to obtaining a correct solution.<br />

Title<br />

Time Value of Money It is a fact that money makes money. This concept explains the change<br />

in the amount of money over time for both owned and borrowed funds. (1)<br />

Economic Equivalence A combination of time value of money and interest rate that makes<br />

different sums of money at different times have equal economic value . (1)<br />

Cash Flow The flow of money into and out of a company, project, or activity. Revenues are<br />

cash infl ows and carry a positive (+) sign; expenses are outfl ows and carry a negative (−) sign.<br />

If only costs are involved, the − sign may be omitted, e.g., benefit/cost (B/C) analysis. (1, 9)<br />

End-of-Period Convention To simplify calculations, cash flows (revenues and costs) are<br />

assumed to occur at the end of a time period . An interest period or fiscal period is commonly<br />

1 year . A half-year convention is often used in depreciation calculations. (1)<br />

Cost of Capital The interest rate incurred to obtain capital investment funds. COC is usually<br />

a weighted average that involves the cost of debt capital (loans, bonds, and mortgages) and equity<br />

capital (stocks and retained earnings). (1, 10)<br />

Minimum Attractive Rate of Return (MARR) A reasonable rate of return established for<br />

the evaluation of an economic alternative. Also called the hurdle rate, MARR is based on cost of<br />

capital, market trend, risk, etc. The inequality ROR ≥ MARR > COC is correct for an economically<br />

viable project. (1, 10)<br />

Opportunity Cost A forgone opportunity caused by the inability to pursue a project. Numerically,<br />

it is the largest rate of return of all the projects not funded due to the lack of capital<br />

funds. Stated differently, it is the ROR of the first project rejected because of unavailability of<br />

funds. (1, 10)<br />

Nominal or Effective Interest Rate ( r or i ) A nominal interest rate does not include any<br />

compounding ; for example, 1% per month is the same as nominal 12% per year. Effective interest<br />

rate is the actual rate over a period of time because compounding is imputed ; for example, 1% per<br />

month, compounded monthly, is an effective 12.683% per year. Inflation or deflation is not considered.<br />

(4)<br />

Placement of Present Worth ( P ; PW) In applying the ( P / A , i %, n ) factor, P or PW is always<br />

located one interest period (year) prior to the fi rst A amount . The A or AW is a series of equal,<br />

end-of-period cash flows for n consecutive periods, expressed as money per time (say,<br />

$/year; /year). (2, 3)<br />

Placement of Future Worth ( F ; FW) In applying the ( F / A , i %, n ) factor, F or FW is always<br />

located at the end of the last interest period (year) of the A series . (2, 3)


574 Appendix E Glossary of Concepts and Terms<br />

Placement of Gradient Present Worth ( P G ; P g ) The ( P / G , i %, n ) factor for an arithmetic<br />

gradient finds the P G of only the gradient series 2 years prior to the first appearance of the constant<br />

gradient G. The base amount A is treated separately from the gradient series.<br />

The ( P /A,g,i,n) factor for a geometric gradient determines P g for the gradient and initial amount<br />

A 1 two years prior to the appearance of the first gradient amount. The initial amount A 1 is included<br />

in the value of P g . (2, 3)<br />

Equal-Service Requirement Identical capacity of all alternatives operating over the<br />

same amount of time is mandated by the equal-service requirement. Estimated costs and revenues<br />

for equal service must be evaluated. PW analysis requires evaluation over the same<br />

number of years (periods) using the LCM (least common multiple) of lives; AW analysis is<br />

performed over one life cycle. Further, equal service assumes that all costs and revenues rise<br />

and fall in accordance with the overall rate of inflation or deflation over the total time period<br />

of the evaluation. (5, 6, 8)<br />

LCM or Study Period To select from mutually exclusive alternatives under the equal-service<br />

requirement for PW computations, use the LCM of lives with repurchase(s) as necessary. For a<br />

stated study period (planning horizon), evaluate cash flows only over this period , neglecting any<br />

beyond this time; estimated market values at termination of the study period are the salvage<br />

v alues. (5, 6, 11)<br />

Salvage/Market Value Expected trade-in, market, or scrap value at the end of the estimated<br />

life or the study period . In a replacement study, the defender’s estimated market value at the end<br />

of a year is considered its “first cost” at the beginning of the next year. MACRS depreciation always<br />

reduces the book value to a salvage of zero. (6, 11)<br />

Do Nothing The DN alternative is always an option, unless one of the defined alternatives<br />

must be selected. DN is status quo; it generates no new costs, revenues, or savings . (5)<br />

Revenue or Cost Alternative Revenue alternatives have costs and revenues estimated; savings<br />

are considered negative costs and carry a + sign. Incremental evaluation requires comparison<br />

with DN for revenue alternatives. Cost (or service) alternatives have only costs estimated;<br />

revenues and savings are assumed equal between alternatives. (5)<br />

Rate of Return An interest rate that equates a PW or AW relation to zero . Also defined as the<br />

rate on the unpaid balance of borrowed money, or rate earned on the unrecovered balance of an<br />

investment such that the last cash fl ow brings the balance exactly to zero. (7, 8)<br />

Project Evaluation For a specifi ed MARR, determine a measure of worth for net cash flow<br />

series over the life or study period. Guidelines for a single project to be economically justified at<br />

the MARR (or discount rate) follow. (5, 6, 7, 9, 17)<br />

Present worth: If PW ≥ 0 Annual worth: If AW ≥ 0<br />

Future worth: If FW ≥ 0 Rate of return: If i * ≥ MARR<br />

Benefit/cost: If B/C ≥ 1.0 Profitability index: If PI ≥ 1.0<br />

ME Alternative Selection For mutually exclusive (select only one) alternatives, compare<br />

two alternatives at a time by determining a measure of worth for the incremental (∆) cash flow<br />

series over the life or study period, adhering to the equal-service requirement. (5, 6, 8, 9, 10, 17)<br />

Present worth or annual worth: Find PW or AW values at MARR; select numerically l argest<br />

(least negative or most positive).<br />

Rate of return: Order by initial cost , perform pairwise ∆ i * comparison; if ∆ i * ≥ MARR,<br />

select larger cost alternative; continue until one remains.<br />

Benefit/cost: Order by total equivalent cost , perform pairwise ∆B/C comparison; if<br />

∆B/C ≥ 1.0, select larger cost alternative; continue until one remains.


E.1 Important Concepts and Guidelines 575<br />

Cost-effectiveness ratio: For service sector alternatives; order by effectiveness measure ;<br />

perform pairwise ∆C/E comparison using dominance ; select from nondominated alternatives<br />

without exceeding budget.<br />

Independent Project Selection No comparison between projects; only against DN. Calculate<br />

a measure of worth and select using the guidelines below. (5, 6, 8, 9, 12)<br />

Present worth or annual worth: Find PW or AW at MARR; select all projects with PW or<br />

AW ≥ 0.<br />

Rate of return: No incremental comparison; select all projects with overall i * ≥ MARR.<br />

Benefit/cost: No incremental comparison; select all projects with overall B/C ≥ 1.0.<br />

Cost-effectiveness ratio: For service sector projects; no incremental comparison; order by<br />

CER and select projects to not exceed budget.<br />

When a capital budget limit is defined, independent projects are selected using the capital budgeting<br />

process based on PW values . The Solver spreadsheet tool is useful here.<br />

Capital Recovery CR is the equivalent annual amount an asset or system must earn to recover<br />

the initial investment plus a stated rate of return. Numerically, it is the AW value of the initial<br />

investment at a stated rate of return. The salvage value is considered in CR calculations. (6)<br />

Economic Service Life The ESL is the number of years n at which the total AW of costs ,<br />

including salvage and AOC, is at its minimum, considering all the years the asset may provide<br />

service. (11)<br />

Sunk Cost Capital (money) that is lost and cannot be recovered. Sunk costs are not included<br />

when making decisions about the future. They should be handled using tax laws and write-off<br />

allowances, not the economic study. (11)<br />

Inflation Expressed as a percentage per time (% per year), it is an increase in the amount<br />

of money required to purchase the same amount of goods or services over time . Inflation<br />

occurs when the value of a currency decreases. Economic evaluations are performed using<br />

either a market (inflation-adjusted) interest rate or an inflation-free rate (constant-value<br />

terms). (1, 14)<br />

Breakeven For a single project, the value of a parameter that makes two elements equal,<br />

e.g., sales necessary to equate revenues and costs. For two alternatives, breakeven is the value<br />

of a common variable at which the two are equally acceptable. Breakeven analysis is fundamental<br />

to make-buy decisions, replacement studies, payback analysis, sensitivity analysis,<br />

breakeven ROR analysis, and many others. The Goal Seek spreadsheet tool is useful in breakeven<br />

analysis. (8, 13)<br />

Payback Period Amount of time n before recovery of the initial capital investment is expected.<br />

Payback with i > 0 or simple payback at i = 0 is useful for preliminary or screening analysis to<br />

determine if a full PW, AW, or ROR analysis is needed. (13)<br />

Direct / Indirect Costs Direct costs are primarily human labor, machines, and materials associated<br />

with a product, process, system, or service. Indirect costs, which include support functions,<br />

utilities, management, legal, taxes, and the like, are more difficult to associate with a<br />

specific product or process. (15)<br />

Value Added Activities have added worth to a product or service from the perspective of a<br />

consumer, owner, or investor who is willing to pay more for an enhanced value. (17)<br />

Sensitivity Analysis Determination of how a measure of worth is affected by changes in<br />

estimated values of a parameter over a stated range. Parameters may be any cost factor, revenue,<br />

life, salvage value, inflation rate, etc. (18)


576 Appendix E Glossary of Concepts and Terms<br />

Risk Variation from an expected, desirable, or predicted value that may be detrimental to the<br />

product, process, or system. Risk represents an absence of or deviation from certainty . Probability<br />

estimates of variation (values) help evaluate risk and uncertainty using statistics and simulation.<br />

(10, 18, 19, 20)<br />

E.2 Symbols and Terms<br />

This section identifies and defines the common terms and their symbols used throughout the text.<br />

The numbers in parentheses indicate sections where the term is introduced and used in various<br />

applications.<br />

Term Symbol Description<br />

Annual amount or<br />

worth<br />

A or AW Equivalent uniform annual worth of all cash inflows and<br />

outflows over estimated life (1.5, 6.l).<br />

Annual operating cost AOC Estimated annual costs to maintain and support an<br />

alternative (1.3).<br />

Benefit/cost ratio B/C Ratio of a project’s benefits to costs expressed in PW, AW,<br />

or FW terms (9.2).<br />

Book value BV Remaining capital investment in an asset after depreciation<br />

is accounted for (16.1).<br />

Breakeven point Q BE Quantity at which revenues and costs are equal, or two<br />

alternatives are equivalent (13.1).<br />

Capital budget b Amount of money available for capital investment projects<br />

(12.1).<br />

Capital recovery CR or A Equivalent annual cost of owning an asset plus the required<br />

return on the initial investment (6.2).<br />

Capitalized cost CC or P Present worth of an alternative that will last forever (or a<br />

long time) (5.5).<br />

Cash flow CF Actual cash amounts that are receipts (inflow) and disbursements<br />

(outflow) (1.6).<br />

Cash flow before or<br />

after taxes<br />

Compounding<br />

frequency<br />

CFBT or<br />

CFAT<br />

m<br />

Cash flow amount before relevant taxes or after taxes are<br />

applied (17.2).<br />

Number of times interest is compounded per period (year)<br />

(4.1).<br />

Cost-effectiveness ratio CER Ratio of equivalent cost to effectiveness measure to evaluate<br />

service sector projects (9.5).<br />

Cost estimating relationships<br />

C 2 or C T<br />

Relations that use design variables and changing costs over<br />

time to estimate current and future costs (15.3–4).<br />

Cost of capital WACC Interest rate paid for the use of capital funds; includes both<br />

debt and equity funds. For debt and equity considered, it is<br />

weighted average cost of capital (1.9, 10.2).<br />

Debt-equity mix D-E Percentages of debt and equity investment capital used by a<br />

corporation (10.2).<br />

Depreciation D Reduction in the value of assets using specific models and<br />

rules; there are book and tax depreciation methods (16.1).<br />

Depreciation rate d t Annual rate for reducing the value of assets using different<br />

depreciation methods (16.1).<br />

Economic service life ESL or n Number of years at which the AW of costs is a minimum<br />

(11.2).<br />

Effectiveness measure E A nonmonetary measure used in the cost-effectiveness ratio<br />

for service sector projects (9.5).


E.2 Symbols and Terms 577<br />

Term Symbol Description<br />

Expected value<br />

( average)<br />

X – , , or E(X) Long-run expected average if a random variable is sampled<br />

many times (18.3, 19.4).<br />

Expenses, operating OE All corporate costs incurred in transacting business (17.1).<br />

First cost P Total initial cost—purchase, construction, setup, etc.<br />

(1.3, 16.1).<br />

Future amount or worth F or FW Amount at some future date considering time value of<br />

money (1.5, 5.4).<br />

Gradient, arithmetic G Uniform change (+ or –) in cash flow each time period<br />

(2.5).<br />

Gradient, geometric g Constant rate of change (+ or –) each time period (2.6).<br />

Gross income GI Income from all sources for corporations or individuals<br />

(17.1).<br />

Inflation rate f Rate that reflects changes in the value of a currency over<br />

time (14.1).<br />

Interest I Amount earned or paid over time based on an initial<br />

amount and interest rate (1.4).<br />

Interest rate i or r Interest expressed as a percentage of the original amount per<br />

time period; nominal (r) and effective (i) rates (1.4, 4.1).<br />

Interest rate, inflationadjusted<br />

i f Interest rate adjusted to take inflation into account (14.1).<br />

Life (estimated) n Number of years or periods over which an alternative or<br />

asset will be used; the evaluation time (1.5).<br />

Life-cycle cost LCC Evaluation of costs for a system over all stages: feasibility<br />

to design to phaseout (6.5).<br />

Measure of worth Varies Value, such as PW, AW, i*, used to judge economic<br />

viability (1.1).<br />

Minimum attractive<br />

rate of return<br />

MARR<br />

Minimum value of the rate of return for an alternative to be<br />

financially viable (1.9, 10.1).<br />

Modified ROR i' or MIRR Unique ROR when a reinvestment rate i i and external borrowing<br />

rate i b are applied to multiple-rate cash flows (7.5).<br />

Net cash flow NCF Resulting, actual amount of cash that flows in or out during<br />

a time period (1.6).<br />

Net operating income NOI Difference between gross income and operating expenses<br />

(17.1).<br />

Net operating profit<br />

after taxes<br />

NOPAT or<br />

NPAT<br />

Amount remaining after taxes are removed from taxable<br />

income (17.1).<br />

Net present value NPV Another name for the present worth, PW.<br />

Payback period n p Number of years to recover the initial investment and a<br />

stated rate of return (13.3).<br />

Present amount<br />

or worth<br />

P or PW Amount of money at the current time or a time denoted as<br />

present (1.5, 5.2).<br />

Probability distribution P(X) Distribution of probability over different values of a<br />

v ariable (19.2).<br />

Profitability index PI Ratio of PW of net cash flows to initial investment used for<br />

revenue projects; rewritten modified B/C ratio (9.2, 12.5).<br />

Random variable X Parameter or characteristic that can take on any one of<br />

several values; discrete and continuous (19.2).<br />

Rate of return i* or ROR Compound interest rate on unpaid or unrecovered balances<br />

such that the final amount results in a zero balance (7.1).


578 Appendix E Glossary of Concepts and Terms<br />

Term Symbol Description<br />

Recovery period n Number of years to completely depreciate an asset (16.1).<br />

Return on invested<br />

capital<br />

i'' or ROIC Unique ROR when a reinvestment rate i i is applied to<br />

multiple-rate cash flows (7.5).<br />

Salvage/market value S or MV Expected trade-in or market value when an asset is traded<br />

or disposed of (6.2, 11.1, 16.1).<br />

Standard deviation s or σ Measure of dispersion or spread about the expected value<br />

or average (19.4).<br />

Study period n Specified number of years over which an evaluation takes<br />

place (5.3, 11.5).<br />

Taxable income TI Amount upon which income taxes are based (17.1).<br />

Tax rate T Decimal rate, usually graduated, used to calculate corporate<br />

or individual taxes (17.1).<br />

Tax rate, effective T e Single-figure tax rate incorporating several rates and bases<br />

(17.1).<br />

Time t Indicator for a time period (1.7).<br />

Unadjusted basis B Depreciable amount of first cost, delivery, and installation<br />

costs of an asset (18.1).<br />

Value added EVA Economic value added reflects net profit after taxes (NPAT)<br />

after removing cost of invested capital during the year<br />

(17.7).<br />

Value-added tax VAT An indirect consumption tax collected at each stage of<br />

production/distribution process; different from a sales tax<br />

paid by end user at purchase time (17.9).


REFERENCE MATERIALS<br />

Textbooks on Related Topics<br />

Blank, L. T., and A. Tarquin: Basics of <strong>Engineering</strong> <strong>Economy</strong>, McGraw-Hill, New York, 2008.<br />

Bowman, M. S.: Applied Economic Analysis for Technologists, Engineers, and Managers, 2d ed., Pearson<br />

Prentice Hall, Upper Saddle River, NJ, 2003.<br />

Bussey, L. E., and T. G. Eschenbach: The Economic Analysis of Industrial Projects , 2d ed., Pearson<br />

Prentice Hall, Upper Saddle River, NJ, 1992.<br />

Canada, J. R., W. G. Sullivan, D. J. Kulonda, and J. A. White: Capital Investment Analysis for <strong>Engineering</strong><br />

and Management , 3d ed., Pearson Prentice Hall, Upper Saddle River, NJ, 2005.<br />

Collier, C. A., and C. R. Glagola: <strong>Engineering</strong> Economic and Cost Analysis , 3d ed., Pearson Prentice Hall,<br />

Upper Saddle River, NJ, 1999.<br />

Cushman, R. F., and M. Loulakis: Design-Build Contracting Handbook, 2d ed., Aspen Publishers, New<br />

York, 2001.<br />

Eschenbach, T. G.: <strong>Engineering</strong> <strong>Economy</strong>: Applying Theory to Practice , 3d ed., Oxford University Press,<br />

New York, 2010.<br />

Fabrycky, W. J., G. J. Thuesen, and D. Verma: Economic Decision Analysis , 3d ed., Pearson Prentice Hall,<br />

Upper Saddle River, NJ, 1998.<br />

Fraser, N. M., E. M. Jewkes, I. Bernhardt, and M. Tajima: <strong>Engineering</strong> Economics in Canada, Pearson<br />

Prentice Hall, Upper Saddle River, NJ, 2006.<br />

Hartman, J. C.: <strong>Engineering</strong> <strong>Economy</strong> and the Decision Making Process , Pearson Prentice Hall, Upper<br />

Saddle River, NJ, 2007.<br />

Levy, S. M.: Build, Operate, Transfer: Paving the Way for Tomorrow’s Infrastructure , John Wiley & Sons,<br />

New York, 1996.<br />

Newnan, D. G., J. P. Lavelle, and T. G. Eschenbach: <strong>Engineering</strong> Economic Analysis , 10th ed., Oxford<br />

University Press, New York, 2009.<br />

Ostwald, P. F.: Construction Cost Analysis and Estimating , Pearson Prentice Hall, Upper Saddle River,<br />

NJ, 2001.<br />

Ostwald, P. F., and T. S. McLaren: Cost Analysis and Estimating for <strong>Engineering</strong> and Management ,<br />

Pearson Prentice Hall, Upper Saddle River, NJ, 2004.<br />

Park, C. S.: Contemporary <strong>Engineering</strong> Economics , 5th ed., Pearson Prentice Hall, Upper Saddle River,<br />

NJ, 2011.<br />

Park, C. S.: Fundamentals of <strong>Engineering</strong> Economics , 2d ed., Pearson Prentice Hall, Upper Saddle River,<br />

NJ, 2008.<br />

Peurifoy, R. L., and G. D. Oberlender: Estimating Construction Costs, 5th ed., McGraw-Hill, New York,<br />

2002.<br />

Riggs, J. L., D. D. Bedworth, and S. U. Randhawa: <strong>Engineering</strong> Economics , 4th ed., McGraw-Hill,<br />

New York, 1996.<br />

Stewart, R. D., R. M. Wyskida, and J. D. Johannes: Cost Estimator’s Reference Manual , 2d ed., John<br />

Wiley & Sons, New York, 1995.<br />

Sullivan, W. G., E. M. Wicks, and C. P. Koelling: <strong>Engineering</strong> <strong>Economy</strong> , 15th ed., Pearson Prentice Hall,<br />

Upper Saddle River, NJ, 2011.<br />

Thuesen, G. J., and W. J. Fabrycky: <strong>Engineering</strong> <strong>Economy</strong> , 9th ed., Pearson Prentice Hall, Upper Saddle<br />

River, NJ, 2001.<br />

White, J. A., K. E. Case, D. B. Pratt, and M. H. Agee: Principles of <strong>Engineering</strong> Economic Analysis ,<br />

5th ed., John Wiley & Sons, New York, 2010.<br />

Using Excel 2007<br />

Gottfried, B. S.: Spreadsheet Tools for Engineers Using Excel ® 2007 , McGraw-Hill, New York, 2010.<br />

Materials on <strong>Engineering</strong> Ethics<br />

Harris, C. E., M. S. Pritchard, and M. J. Rabins: <strong>Engineering</strong> Ethics: Concepts and Cases , 4th ed.,<br />

Wadsworth Cengage Learning, Belmont, CA, 2009.<br />

Martin, M. W., and R. Schinzinger: Introduction to <strong>Engineering</strong> Ethics , 2d ed., McGraw-Hill,<br />

New York, 2010.


580 Reference Materials<br />

Websites<br />

Construction cost estimation index: www.construction.com<br />

The Economist : www.economist.com<br />

For this textbook: www.mhhe.com/blank<br />

Plant cost estimation index: www.che.com/pci<br />

Revenue Canada: www.cra.gc.ca<br />

U.S. Internal Revenue Service: www.irs.gov<br />

Wall Street Journal : www.online.wsj.com<br />

U.S. Government Publications (available at<br />

www.irs.gov )<br />

Corporations , Publication 544, Internal Revenue Service, GPO, Washington, DC, annually.<br />

Sales and Other Dispositions of Assets , Publication 542, Internal Revenue Service, GPO, Washington,<br />

DC, annually.<br />

Your Federal Income Tax , Publication 17, Internal Revenue Service, GPO, Washington, DC, annually.<br />

Selected Journals and Other Publications<br />

The <strong>Engineering</strong> Economist, joint publication of ASEE and IIE, published by Taylor and Francis,<br />

Philadelphia, quarterly.<br />

Harvard Business Review , Harvard University Press, Boston, bimonthly.<br />

Journal of Finance , American Finance Association, published by John Wiley & Sons, New York,<br />

bimonthly.


Compound Interest Factor Tables 581<br />

0.25% TABLE 1 Discrete Cash Flow: Compound Interest Factors 0.25%<br />

Single Payments Uniform Series Payments Arithmetic Gradients<br />

Compound Present Sinking Compound Capital Present Gradient Gradient<br />

Amount Worth Fund Amount Recovery Worth Present Worth Uniform Series<br />

n FP PF AF FA AP PA PG AG<br />

1 1.0025 0.9975 1.00000 1.0000 1.00250 0.9975<br />

2 1.0050 0.9950 0.49938 2.0025 0.50188 1.9925 0.9950 0.4994<br />

3 1.0075 0.9925 0.33250 3.0075 0.33500 2.9851 2.9801 0.9983<br />

4 1.0100 0.9901 0.24906 4.0150 0.25156 3.9751 5.9503 1.4969<br />

5 1.0126 0.9876 0.19900 5.0251 0.20150 4.9627 9.9007 1.9950<br />

6 1.0151 0.9851 0.16563 6.0376 0.16813 5.9478 14.8263 2.4927<br />

7 1.0176 0.9827 0.14179 7.0527 0.14429 6.9305 20.7223 2.9900<br />

8 1.0202 0.9802 0.12391 8.0704 0.12641 7.9107 27.5839 3.4869<br />

9 1.0227 0.9778 0.11000 9.0905 0.11250 8.8885 35.4061 3.9834<br />

10 1.0253 0.9753 0.09888 10.1133 0.10138 9.8639 44.1842 4.4794<br />

11 1.0278 0.9729 0.08978 11.1385 0.09228 10.8368 53.9133 4.9750<br />

12 1.0304 0.9705 0.08219 12.1664 0.08469 11.8073 64.5886 5.4702<br />

13 1.0330 0.9681 0.07578 13.1968 0.07828 12.7753 76.2053 5.9650<br />

14 1.0356 0.9656 0.07028 14.2298 0.07278 13.7410 88.7587 6.4594<br />

15 1.0382 0.9632 0.06551 15.2654 0.06801 14.7042 102.2441 6.9534<br />

16 1.0408 0.9608 0.06134 16.3035 0.06384 15.6650 116.6567 7.4469<br />

17 1.0434 0.9584 0.05766 17.3443 0.06016 16.6235 131.9917 7.9401<br />

18 1.0460 0.9561 0.05438 18.3876 0.05688 17.5795 148.2446 8.4328<br />

19 1.0486 0.9537 0.05146 19.4336 0.05396 18.5332 165.4106 8.9251<br />

20 1.0512 0.9513 0.04882 20.4822 0.05132 19.4845 183.4851 9.4170<br />

21 1.0538 0.9489 0.04644 21.5334 0.04894 20.4334 202.4634 9.9085<br />

22 1.0565 0.9466 0.04427 22.5872 0.04677 21.3800 222.3410 10.3995<br />

23 1.0591 0.9442 0.04229 23.6437 0.04479 22.3241 243.1131 10.8901<br />

24 1.0618 0.9418 0.04048 24.7028 0.04298 23.2660 264.7753 11.3804<br />

25 1.0644 0.9395 0.03881 25.7646 0.04131 24.2055 287.3230 11.8702<br />

26 1.0671 0.9371 0.03727 26.8290 0.03977 25.1426 310.7516 12.3596<br />

27 1.0697 0.9348 0.03585 27.8961 0.03835 26.0774 335.0566 12.8485<br />

28 1.0724 0.9325 0.03452 28.9658 0.03702 27.0099 360.2334 13.3371<br />

29 1.0751 0.9301 0.03329 30.0382 0.03579 27.9400 386.2776 13.8252<br />

30 1.0778 0.9278 0.03214 31.1133 0.03464 28.8679 413.1847 14.3130<br />

36 1.0941 0.9140 0.02658 37.6206 0.02908 34.3865 592.4988 17.2306<br />

40 1.1050 0.9050 0.02380 42.0132 0.02630 38.0199 728.7399 19.1673<br />

48 1.1273 0.8871 0.01963 50.9312 0.02213 45.1787 1040.06 23.0209<br />

50 1.1330 0.8826 0.01880 53.1887 0.02130 46.9462 1125.78 23.9802<br />

52 1.1386 0.8782 0.01803 55.4575 0.02053 48.7048 1214.59 24.9377<br />

55 1.1472 0.8717 0.01698 58.8819 0.01948 51.3264 1353.53 26.3710<br />

60 1.1616 0.8609 0.01547 64.6467 0.01797 55.6524 1600.08 28.7514<br />

72 1.1969 0.8355 0.01269 78.7794 0.01519 65.8169 2265.56 34.4221<br />

75 1.2059 0.8292 0.01214 82.3792 0.01464 68.3108 2447.61 35.8305<br />

84 1.2334 0.8108 0.01071 93.3419 0.01321 75.6813 3029.76 40.0331<br />

90 1.2520 0.7987 0.00992 100.7885 0.01242 80.5038 3446.87 42.8162<br />

96 1.2709 0.7869 0.00923 108.3474 0.01173 85.2546 3886.28 45.5844<br />

100 1.2836 0.7790 0.00881 113.4500 0.01131 88.3825 4191.24 47.4216<br />

108 1.3095 0.7636 0.00808 123.8093 0.01058 94.5453 4829.01 51.0762<br />

120 1.3494 0.7411 0.00716 139.7414 0.00966 103.5618 5852.11 56.5084<br />

132 1.3904 0.7192 0.00640 156.1582 0.00890 112.3121 6950.01 61.8813<br />

144 1.4327 0.6980 0.00578 173.0743 0.00828 120.8041 8117.41 67.1949<br />

240 1.8208 0.5492 0.00305 328.3020 0.00555 180.3109 19399 107.5863<br />

360 2.4568 0.4070 0.00172 582.7369 0.00422 237.1894 36264 152.8902<br />

480 3.3151 0.3016 0.00108 926.0595 0.00358 279.3418 53821 192.6699


582 Compound Interest Factor Tables<br />

0.5% TABLE 2 Discrete Cash Flow: Compound Interest Factors 0.5%<br />

Single Payments Uniform Series Payments Arithmetic Gradients<br />

Compound Present Sinking Compound Capital Present Gradient Gradient<br />

Amount Worth Fund Amount Recovery Worth Present Worth Uniform Series<br />

n FP PF AF FA AP PA PG AG<br />

1 1.0050 0.9950 1.00000 1.0000 1.00500 0.9950<br />

2 1.0100 0.9901 0.49875 2.0050 0.50375 1.9851 0.9901 0.4988<br />

3 1.0151 0.9851 0.33167 3.0150 0.33667 2.9702 2.9604 0.9967<br />

4 1.0202 0.9802 0.24813 4.0301 0.25313 3.9505 5.9011 1.4938<br />

5 1.0253 0.9754 0.19801 5.0503 0.20301 4.9259 9.8026 1.9900<br />

6 1.0304 0.9705 0.16460 6.0755 0.16960 5.8964 14.6552 2.4855<br />

7 1.0355 0.9657 0.14073 7.1059 0.14573 6.8621 20.4493 2.9801<br />

8 1.0407 0.9609 0.12283 8.1414 0.12783 7.8230 27.1755 3.4738<br />

9 1.0459 0.9561 0.10891 9.1821 0.11391 8.7791 34.8244 3.9668<br />

10 1.0511 0.9513 0.09777 10.2280 0.10277 9.7304 43.3865 4.4589<br />

11 1.0564 0.9466 0.08866 11.2792 0.09366 10.6770 52.8526 4.9501<br />

12 1.0617 0.9419 0.08107 12.3356 0.08607 11.6189 63.2136 5.4406<br />

13 1.0670 0.9372 0.07464 13.3972 0.07964 12.5562 74.4602 5.9302<br />

14 1.0723 0.9326 0.06914 14.4642 0.07414 13.4887 86.5835 6.4190<br />

15 1.0777 0.9279 0.06436 15.5365 0.06936 14.4166 99.5743 6.9069<br />

16 1.0831 0.9233 0.06019 16.6142 0.06519 15.3399 113.4238 7.3940<br />

17 1.0885 0.9187 0.05651 17.6973 0.06151 16.2586 128.1231 7.8803<br />

18 1.0939 0.9141 0.05323 18.7858 0.05823 17.1728 143.6634 8.3658<br />

19 1.0994 0.9096 0.05030 19.8797 0.05530 18.0824 160.0360 8.8504<br />

20 1.1049 0.9051 0.04767 20.9791 0.05267 18.9874 177.2322 9.3342<br />

21 1.1104 0.9006 0.04528 22.0840 0.05028 19.8880 195.2434 9.8172<br />

22 1.1160 0.8961 0.04311 23.1944 0.04811 20.7841 214.0611 10.2993<br />

23 1.1216 0.8916 0.04113 24.3104 0.04613 21.6757 233.6768 10.7806<br />

24 1.1272 0.8872 0.03932 25.4320 0.04432 22.5629 254.0820 11.2611<br />

25 1.1328 0.8828 0.03765 26.5591 0.04265 23.4456 275.2686 11.7407<br />

26 1.1385 0.8784 0.03611 27.6919 0.04111 24.3240 297.2281 12.2195<br />

27 1.1442 0.8740 0.03469 28.8304 0.03969 25.1980 319.9523 12.6975<br />

28 1.1499 0.8697 0.03336 29.9745 0.03836 26.0677 343.4332 13.1747<br />

29 1.1556 0.8653 0.03213 31.1244 0.03713 26.9330 367.6625 13.6510<br />

30 1.1614 0.8610 0.03098 32.2800 0.03598 27.7941 392.6324 14.1265<br />

36 1.1967 0.8356 0.02542 39.3361 0.03042 32.8710 557.5598 16.9621<br />

40 1.2208 0.8191 0.02265 44.1588 0.02765 36.1722 681.3347 18.8359<br />

48 1.2705 0.7871 0.01849 54.0978 0.02349 42.5803 959.9188 22.5437<br />

50 1.2832 0.7793 0.01765 56.6452 0.02265 44.1428 1035.70 23.4624<br />

52 1.2961 0.7716 0.01689 59.2180 0.02189 45.6897 1113.82 24.3778<br />

55 1.3156 0.7601 0.01584 63.1258 0.02084 47.9814 1235.27 25.7447<br />

60 1.3489 0.7414 0.01433 69.7700 0.01933 51.7256 1448.65 28.0064<br />

72 1.4320 0.6983 0.01157 86.4089 0.01657 60.3395 <strong>2012</strong>.35 33.3504<br />

75 1.4536 0.6879 0.01102 90.7265 0.01602 62.4136 2163.75 34.6679<br />

84 1.5204 0.6577 0.00961 104.0739 0.01461 68.4530 2640.66 38.5763<br />

90 1.5666 0.6383 0.00883 113.3109 0.01383 72.3313 2976.08 41.1451<br />

96 1.6141 0.6195 0.00814 122.8285 0.01314 76.0952 3324.18 43.6845<br />

100 1.6467 0.6073 0.00773 129.3337 0.01273 78.5426 3562.79 45.3613<br />

108 1.7137 0.5835 0.00701 142.7399 0.01201 83.2934 4054.37 48.6758<br />

120 1.8194 0.5496 0.00610 163.8793 0.01110 90.0735 4823.51 53.5508<br />

132 1.9316 0.5177 0.00537 186.3226 0.01037 96.4596 5624.59 58.3103<br />

144 2.0508 0.4876 0.00476 210.1502 0.00976 102.4747 6451.31 62.9551<br />

240 3.3102 0.3021 0.00216 462.0409 0.00716 139.5808 13416 96.1131<br />

360 6.0226 0.1660 0.00100 1004.52 0.00600 166.7916 21403 128.3236<br />

480 10.9575 0.0913 0.00050 1991.49 0.00550 181.7476 27588 151.7949


Compound Interest Factor Tables 583<br />

0.75% TABLE 3 Discrete Cash Flow: Compound Interest Factors 0.75%<br />

Single Payments Uniform Series Payments Arithmetic Gradients<br />

Compound Present Sinking Compound Capital Present Gradient Gradient<br />

Amount Worth Fund Amount Recovery Worth Present Worth Uniform Series<br />

n FP PF AF FA AP PA PG AG<br />

1 1.0075 0.9926 1.00000 1.0000 1.00750 0.9926<br />

2 1.0151 0.9852 0.49813 2.0075 0.50563 1.9777 0.9852 0.4981<br />

3 1.0227 0.9778 0.33085 3.0226 0.33835 2.9556 2.9408 0.9950<br />

4 1.0303 0.9706 0.24721 4.0452 0.25471 3.9261 5.8525 1.4907<br />

5 1.0381 0.9633 0.19702 5.0756 0.20452 4.8894 9.7058 1.9851<br />

6 1.0459 0.9562 0.16357 6.1136 0.17107 5.8456 14.4866 2.4782<br />

7 1.0537 0.9490 0.13967 7.1595 0.14717 6.7946 20.1808 2.9701<br />

8 1.0616 0.9420 0.12176 8.2132 0.12926 7.7366 26.7747 3.4608<br />

9 1.0696 0.9350 0.10782 9.2748 0.11532 8.6716 34.2544 3.9502<br />

10 1.0776 0.9280 0.09667 10.3443 0.10417 9.5996 42.6064 4.4384<br />

11 1.0857 0.9211 0.08755 11.4219 0.09505 10.5207 51.8174 4.9253<br />

12 1.0938 0.9142 0.07995 12.5076 0.08745 11.4349 61.8740 5.4110<br />

13 1.1020 0.9074 0.07352 13.6014 0.08102 12.3423 72.7632 5.8954<br />

14 1.1103 0.9007 0.06801 14.7034 0.07551 13.2430 84.4720 6.3786<br />

15 1.1186 0.8940 0.06324 15.8137 0.07074 14.1370 96.9876 6.8606<br />

16 1.1270 0.8873 0.05906 16.9323 0.06656 15.0243 110.2973 7.3413<br />

17 1.1354 0.8807 0.05537 18.0593 0.06287 15.9050 124.3887 7.8207<br />

18 1.1440 0.8742 0.05210 19.1947 0.05960 16.7792 139.2494 8.2989<br />

19 1.1525 0.8676 0.04917 20.3387 0.05667 17.6468 154.8671 8.7759<br />

20 1.1612 0.8612 0.04653 21.4912 0.05403 18.5080 171.2297 9.2516<br />

21 1.1699 0.8548 0.04415 22.6524 0.05165 19.3628 188.3253 9.7261<br />

22 1.1787 0.8484 0.04198 23.8223 0.04948 20.2112 206.1420 10.1994<br />

23 1.1875 0.8421 0.04000 25.0010 0.04750 21.0533 224.6682 10.6714<br />

24 1.1964 0.8358 0.03818 26.1885 0.04568 21.8891 243.8923 11.1422<br />

25 1.2054 0.8296 0.03652 27.3849 0.04402 22.7188 263.8029 11.6117<br />

26 1.2144 0.8234 0.03498 28.5903 0.04248 23.5422 284.3888 12.0800<br />

27 1.2235 0.8173 0.03355 29.8047 0.04105 24.3595 305.6387 12.5470<br />

28 1.2327 0.8112 0.03223 31.0282 0.03973 25.1707 327.5416 13.0128<br />

29 1.2420 0.8052 0.03100 32.2609 0.03850 25.9759 350.0867 13.4774<br />

30 1.2513 0.7992 0.02985 33.5029 0.03735 26.7751 373.2631 13.9407<br />

36 1.3086 0.7641 0.02430 41.1527 0.03180 31.4468 524.9924 16.6946<br />

40 1.3483 0.7416 0.02153 46.4465 0.02903 34.4469 637.4693 18.5058<br />

48 1.4314 0.6986 0.01739 57.5207 0.02489 40.1848 886.8404 22.0691<br />

50 1.4530 0.6883 0.01656 60.3943 0.02406 41.5664 953.8486 22.9476<br />

52 1.4748 0.6780 0.01580 63.3111 0.02330 42.9276 1022.59 23.8211<br />

55 1.5083 0.6630 0.01476 67.7688 0.02226 44.9316 1128.79 25.1223<br />

60 1.5657 0.6387 0.01326 75.4241 0.02076 48.1734 1313.52 27.2665<br />

72 1.7126 0.5839 0.01053 95.0070 0.01803 55.4768 1791.25 32.2882<br />

75 1.7514 0.5710 0.00998 100.1833 0.01748 57.2027 1917.22 33.5163<br />

84 1.8732 0.5338 0.00859 116.4269 0.01609 62.1540 2308.13 37.1357<br />

90 1.9591 0.5104 0.00782 127.8790 0.01532 65.2746 2578.00 39.4946<br />

96 2.0489 0.4881 0.00715 139.8562 0.01465 68.2584 2853.94 41.8107<br />

100 2.1111 0.4737 0.00675 148.1445 0.01425 70.1746 3040.75 43.3311<br />

108 2.2411 0.4462 0.00604 165.4832 0.01354 73.8394 3419.90 46.3154<br />

120 2.4514 0.4079 0.00517 193.5143 0.01267 78.9417 3998.56 50.6521<br />

132 2.6813 0.3730 0.00446 224.1748 0.01196 83.6064 4583.57 54.8232<br />

144 2.9328 0.3410 0.00388 257.7116 0.01138 87.8711 5169.58 58.8314<br />

240 6.0092 0.1664 0.00150 667.8869 0.00900 111.1450 9494.12 85.4210<br />

360 14.7306 0.0679 0.00055 1830.74 0.00805 124.2819 13312 107.1145<br />

480 36.1099 0.0277 0.00021 4681.32 0.00771 129.6409 15513 119.6620


584 Compound Interest Factor Tables<br />

1% TABLE 4 Discrete Cash Flow: Compound Interest Factors 1%<br />

Single Payments Uniform Series Payments Arithmetic Gradients<br />

Compound Present Sinking Compound Capital Present Gradient Gradient<br />

Amount Worth Fund Amount Recovery Worth Present Worth Uniform Series<br />

n FP PF AF FA AP PA PG AG<br />

1 1.0100 0.9901 1.00000 1.0000 1.01000 0.9901<br />

2 1.0201 0.9803 0.49751 2.0100 0.50751 1.9704 0.9803 0.4975<br />

3 1.0303 0.9706 0.33002 3.0301 0.34002 2.9410 2.9215 0.9934<br />

4 1.0406 0.9610 0.24628 4.0604 0.25628 3.9020 5.8044 1.4876<br />

5 1.0510 0.9515 0.19604 5.1010 0.20604 4.8534 9.6103 1.9801<br />

6 1.0615 0.9420 0.16255 6.1520 0.17255 5.7955 14.3205 2.4710<br />

7 1.0721 0.9327 0.13863 7.2135 0.14863 6.7282 19.9168 2.9602<br />

8 1.0829 0.9235 0.12069 8.2857 0.13069 7.6517 26.3812 3.4478<br />

9 1.0937 0.9143 0.10674 9.3685 0.11674 8.5660 33.6959 3.9337<br />

10 1.1046 0.9053 0.09558 10.4622 0.10558 9.4713 41.8435 4.4179<br />

11 1.1157 0.8963 0.08645 11.5668 0.09645 10.3676 50.8067 4.9005<br />

12 1.1268 0.8874 0.07885 12.6825 0.08885 11.2551 60.5687 5.3815<br />

13 1.1381 0.8787 0.07241 13.8093 0.08241 12.1337 71.1126 5.8607<br />

14 1.1495 0.8700 0.06690 14.9474 0.07690 13.0037 82.4221 6.3384<br />

15 1.1610 0.8613 0.06212 16.0969 0.07212 13.8651 94.4810 6.8143<br />

16 1.1726 0.8528 0.05794 17.2579 0.06794 14.7179 107.2734 7.2886<br />

17 1.1843 0.8444 0.05426 18.4304 0.06426 15.5623 120.7834 7.7613<br />

18 1.1961 0.8360 0.05098 19.6147 0.06098 16.3983 134.9957 8.2323<br />

19 1.2081 0.8277 0.04805 20.8109 0.05805 17.2260 149.8950 8.7017<br />

20 1.2202 0.8195 0.04542 22.0190 0.05542 18.0456 165.4664 9.1694<br />

21 1.2324 0.8114 0.04303 23.2392 0.05303 18.8570 181.6950 9.6354<br />

22 1.2447 0.8034 0.04086 24.4716 0.05086 19.6604 198.5663 10.0998<br />

23 1.2572 0.7954 0.03889 25.7163 0.04889 20.4558 216.0660 10.5626<br />

24 1.2697 0.7876 0.03707 26.9735 0.04707 21.2434 234.1800 11.0237<br />

25 1.2824 0.7798 0.03541 28.2432 0.04541 22.0232 252.8945 11.4831<br />

26 1.2953 0.7720 0.03387 29.5256 0.04387 22.7952 272.1957 11.9409<br />

27 1.3082 0.7644 0.03245 30.8209 0.04245 23.5596 292.0702 12.3971<br />

28 1.3213 0.7568 0.03112 32.1291 0.04112 24.3164 312.5047 12.8516<br />

29 1.3345 0.7493 0.02990 33.4504 0.03990 25.0658 333.4863 13.3044<br />

30 1.3478 0.7419 0.02875 34.7849 0.03875 25.8077 355.0021 13.7557<br />

36 1.4308 0.6989 0.02321 43.0769 0.03321 30.1075 494.6207 16.4285<br />

40 1.4889 0.6717 0.02046 48.8864 0.03046 32.8347 596.8561 18.1776<br />

48 1.6122 0.6203 0.01633 61.2226 0.02633 37.9740 820.1460 21.5976<br />

50 1.6446 0.6080 0.01551 64.4632 0.02551 39.1961 879.4176 22.4363<br />

52 1.6777 0.5961 0.01476 67.7689 0.02476 40.3942 939.9175 23.2686<br />

55 1.7285 0.5785 0.01373 72.8525 0.02373 42.1472 1032.81 24.5049<br />

60 1.8167 0.5504 0.01224 81.6697 0.02224 44.9550 1192.81 26.5333<br />

72 2.0471 0.4885 0.00955 104.7099 0.01955 51.1504 1597.87 31.2386<br />

75 2.1091 0.4741 0.00902 110.9128 0.01902 52.5871 1702.73 32.3793<br />

84 2.3067 0.4335 0.00765 130.6723 0.01765 56.6485 2023.32 35.7170<br />

90 2.4486 0.4084 0.00690 144.8633 0.01690 59.1609 2240.57 37.8724<br />

96 2.5993 0.3847 0.00625 159.9273 0.01625 61.5277 2459.43 39.9727<br />

100 2.7048 0.3697 0.00587 170.4814 0.01587 63.0289 2605.78 41.3426<br />

108 2.9289 0.3414 0.00518 192.8926 0.01518 65.8578 2898.42 44.0103<br />

120 3.3004 0.3030 0.00435 230.0387 0.01435 69.7005 3334.11 47.8349<br />

132 3.7190 0.2689 0.00368 271.8959 0.01368 73.1108 3761.69 51.4520<br />

144 4.1906 0.2386 0.00313 319.0616 0.01313 76.1372 4177.47 54.8676<br />

240 10.8926 0.0918 0.00101 989.2554 0.01101 90.8194 6878.60 75.7393<br />

360 35.9496 0.0278 0.00029 3494.96 0.01029 97.2183 8720.43 89.6995<br />

480 118.6477 0.0084 0.00008 11765 0.01008 99.1572 9511.16 95.9200


Compound Interest Factor Tables 585<br />

1.25% TABLE 5 Discrete Cash Flow: Compound Interest Factors 1.25%<br />

Single Payments Uniform Series Payments Arithmetic Gradients<br />

Compound Present Sinking Compound Capital Present Gradient Gradient<br />

Amount Worth Fund Amount Recovery Worth Present Worth Uniform Series<br />

n FP PF AF FA AP PA PG AG<br />

1 1.0125 0.9877 1.00000 1.0000 1.01250 0.9877<br />

2 1.0252 0.9755 0.49680 2.0125 0.50939 1.9631 0.9755 0.4969<br />

3 1.0380 0.9634 0.32920 3.0377 0.34170 2.9265 2.9023 0.9917<br />

4 1.0509 0.9515 0.24536 4.0756 0.25786 3.8781 5.7569 1.4845<br />

5 1.0641 0.9398 0.19506 5.1266 0.20756 4.8178 9.5160 1.9752<br />

6 1.0774 0.9282 0.16153 6.1907 0.17403 5.7460 14.1569 2.4638<br />

7 1.0909 0.9167 0.13759 7.2680 0.15009 6.6627 19.6571 2.9503<br />

8 1.1045 0.9054 0.11963 8.3589 0.13213 7.5681 25.9949 3.4348<br />

9 1.1183 0.8942 0.10567 9.4634 0.11817 8.4623 33.1487 3.9172<br />

10 1.1323 0.8832 0.09450 10.5817 0.10700 9.3455 41.0973 4.3975<br />

11 1.1464 0.8723 0.08537 11.7139 0.09787 10.2178 49.8201 4.8758<br />

12 1.1608 0.8615 0.07776 12.8604 0.09026 11.0793 59.2967 5.3520<br />

13 1.1753 0.8509 0.07132 14.0211 0.08382 11.9302 69.5072 5.8262<br />

14 1.1900 0.8404 0.06581 15.1964 0.07831 12.7706 80.4320 6.2982<br />

15 1.2048 0.8300 0.06103 16.3863 0.07353 13.6005 92.0519 6.7682<br />

16 1.2199 0.8197 0.05685 17.5912 0.06935 14.4203 104.3481 7.2362<br />

17 1.2351 0.8096 0.05316 18.8111 0.06566 15.2299 117.3021 7.7021<br />

18 1.2506 0.7996 0.04988 20.0462 0.06238 16.0295 130.8958 8.1659<br />

19 1.2662 0.7898 0.04696 21.2968 0.05946 16.8193 145.1115 8.6277<br />

20 1.2820 0.7800 0.04432 22.5630 0.05682 17.5993 159.9316 9.0874<br />

21 1.2981 0.7704 0.04194 23.8450 0.05444 18.3697 175.3392 9.5450<br />

22 1.3143 0.7609 0.03977 25.1431 0.05227 19.1306 191.3174 10.0006<br />

23 1.3307 0.7515 0.03780 26.4574 0.05030 19.8820 207.8499 10.4542<br />

24 1.3474 0.7422 0.03599 27.7881 0.04849 20.6242 224.9204 10.9056<br />

25 1.3642 0.7330 0.03432 29.1354 0.04682 21.3573 242.5132 11.3551<br />

26 1.3812 0.7240 0.03279 30.4996 0.04529 22.0813 260.6128 11.8024<br />

27 1.3985 0.7150 0.03137 31.8809 0.04387 22.7963 279.2040 12.2478<br />

28 1.4160 0.7062 0.03005 33.2794 0.04255 23.5025 298.2719 12.6911<br />

29 1.4337 0.6975 0.02882 34.6954 0.04132 24.2000 317.8019 13.1323<br />

30 1.4516 0.6889 0.02768 36.1291 0.04018 24.8889 337.7797 13.5715<br />

36 1.5639 0.6394 0.02217 45.1155 0.03467 28.8473 466.2830 16.1639<br />

40 1.6436 0.6084 0.01942 51.4896 0.03192 31.3269 559.2320 17.8515<br />

48 1.8154 0.5509 0.01533 65.2284 0.02783 35.9315 759.2296 21.1299<br />

50 1.8610 0.5373 0.01452 68.8818 0.02702 37.0129 811.6738 21.9295<br />

52 1.9078 0.5242 0.01377 72.6271 0.02627 38.0677 864.9409 22.7211<br />

55 1.9803 0.5050 0.01275 78.4225 0.02525 39.6017 946.2277 23.8936<br />

60 2.1072 0.4746 0.01129 88.5745 0.02379 42.0346 1084.84 25.8083<br />

72 2.4459 0.4088 0.00865 115.6736 0.02115 47.2925 1428.46 30.2047<br />

75 2.5388 0.3939 0.00812 123.1035 0.02062 48.4890 1515.79 31.2605<br />

84 2.8391 0.3522 0.00680 147.1290 0.01930 51.8222 1778.84 34.3258<br />

90 3.0588 0.3269 0.00607 164.7050 0.01857 53.8461 1953.83 36.2855<br />

96 3.2955 0.3034 0.00545 183.6411 0.01795 55.7246 2127.52 38.1793<br />

100 3.4634 0.2887 0.00507 197.0723 0.01757 56.9013 2242.24 39.4058<br />

108 3.8253 0.2614 0.00442 226.0226 0.01692 59.0865 2468.26 41.7737<br />

120 4.4402 0.2252 0.00363 275.2171 0.01613 61.9828 2796.57 45.1184<br />

132 5.1540 0.1940 0.00301 332.3198 0.01551 64.4781 3109.35 48.2234<br />

144 5.9825 0.1672 0.00251 398.6021 0.01501 66.6277 3404.61 51.0990<br />

240 19.7155 0.0507 0.00067 1497.24 0.01317 75.9423 5101.53 67.1764<br />

360 87.5410 0.0114 0.00014 6923.28 0.01264 79.0861 5997.90 75.8401<br />

480 388.7007 0.0026 0.00003 31016 0.01253 79.7942 6284.74 78.7619


586 Compound Interest Factor Tables<br />

1.5% TABLE 6 Discrete Cash Flow: Compound Interest Factors 1.5%<br />

Single Payments Uniform Series Payments Arithmetic Gradients<br />

Compound Present Sinking Compound Capital Present Gradient Gradient<br />

Amount Worth Fund Amount Recovery Worth Present Worth Uniform Series<br />

n FP PF AF FA AP PA PG AG<br />

1 1.0150 0.9852 1.00000 1.0000 1.01500 0.9852<br />

2 1.0302 0.9707 0.49628 2.0150 0.51128 1.9559 0.9707 0.4963<br />

3 1.0457 0.9563 0.32838 3.0452 0.34338 2.9122 2.8833 0.9901<br />

4 1.0614 0.9422 0.24444 4.0909 0.25944 3.8544 5.7098 1.4814<br />

5 1.0773 0.9283 0.19409 5.1523 0.20909 4.7826 9.4229 1.9702<br />

6 1.0934 0.9145 0.16053 6.2296 0.17553 5.6972 13.9956 2.4566<br />

7 1.1098 0.9010 0.13656 7.3230 0.15156 6.5982 19.4018 2.9405<br />

8 1.1265 0.8877 0.11858 8.4328 0.13358 7.4859 25.6157 3.4219<br />

9 1.1434 0.8746 0.10461 9.5593 0.11961 8.3605 32.6125 3.9008<br />

10 1.1605 0.8617 0.09343 10.7027 0.10843 9.2222 40.3675 4.3772<br />

11 1.1779 0.8489 0.08429 11.8633 0.09929 10.0711 48.8568 4.8512<br />

12 1.1956 0.8364 0.07668 13.0412 0.09168 10.9075 58.0571 5.3227<br />

13 1.2136 0.8240 0.07024 14.2368 0.08524 11.7315 67.9454 5.7917<br />

14 1.2318 0.8118 0.06472 15.4504 0.07972 12.5434 78.4994 6.2582<br />

15 1.2502 0.7999 0.05994 16.6821 0.07494 13.3432 89.6974 6.7223<br />

16 1.2690 0.7880 0.05577 17.9324 0.07077 14.1313 101.5178 7.1839<br />

17 1.2880 0.7764 0.05208 19.2014 0.06708 14.9076 113.9400 7.6431<br />

18 1.3073 0.7649 0.04881 20.4894 0.06381 15.6726 126.9435 8.0997<br />

19 1.3270 0.7536 0.04588 21.7967 0.06088 16.4262 140.5084 8.5539<br />

20 1.3469 0.7425 0.04325 23.1237 0.05825 17.1686 154.6154 9.0057<br />

21 1.3671 0.7315 0.04087 24.4705 0.05587 17.9001 169.2453 9.4550<br />

22 1.3876 0.7207 0.03870 25.8376 0.05370 18.6208 184.3798 9.9018<br />

23 1.4084 0.7100 0.03673 27.2251 0.05173 19.3309 200.0006 10.3462<br />

24 1.4295 0.6995 0.03492 28.6335 0.04992 20.0304 216.0901 10.7881<br />

25 1.4509 0.6892 0.03326 30.0630 0.04826 20.7196 232.6310 11.2276<br />

26 1.4727 0.6790 0.03173 31.5140 0.04673 21.3986 249.6065 11.6646<br />

27 1.4948 0.6690 0.03032 32.9867 0.04532 22.0676 267.0002 12.0992<br />

28 1.5172 0.6591 0.02900 34.4815 0.04400 22.7267 284.7958 12.5313<br />

29 1.5400 0.6494 0.02778 35.9987 0.04278 23.3761 302.9779 12.9610<br />

30 1.5631 0.6398 0.02664 37.5387 0.04164 24.0158 321.5310 13.3883<br />

36 1.7091 0.5851 0.02115 47.2760 0.03615 27.6607 439.8303 15.9009<br />

40 1.8140 0.5513 0.01843 54.2679 0.03343 29.9158 524.3568 17.5277<br />

48 2.0435 0.4894 0.01437 69.5652 0.02937 34.0426 703.5462 20.6667<br />

50 2.1052 0.4750 0.01357 73.6828 0.02857 34.9997 749.9636 21.4277<br />

52 2.1689 0.4611 0.01283 77.9249 0.02783 35.9287 796.8774 22.1794<br />

55 2.2679 0.4409 0.01183 84.5296 0.02683 37.2715 868.0285 23.2894<br />

60 2.4432 0.4093 0.01039 96.2147 0.02539 39.3803 988.1674 25.0930<br />

72 2.9212 0.3423 0.00781 128.0772 0.02281 43.8447 1279.79 29.1893<br />

75 3.0546 0.3274 0.00730 136.9728 0.02230 44.8416 1352.56 30.1631<br />

84 3.4926 0.2863 0.00602 166.1726 0.02102 47.5786 1568.51 32.9668<br />

90 3.8189 0.2619 0.00532 187.9299 0.02032 49.2099 1709.54 34.7399<br />

96 4.1758 0.2395 0.00472 211.7202 0.01972 50.7017 1847.47 36.4381<br />

100 4.4320 0.2256 0.00437 228.8030 0.01937 51.6247 1937.45 37.5295<br />

108 4.9927 0.2003 0.00376 266.1778 0.01876 53.3137 2112.13 39.6171<br />

120 5.9693 0.1675 0.00302 331.2882 0.01802 55.4985 2359.71 42.5185<br />

132 7.1370 0.1401 0.00244 409.1354 0.01744 57.3257 2588.71 45.1579<br />

144 8.5332 0.1172 0.00199 502.2109 0.01699 58.8540 2798.58 47.5512<br />

240 35.6328 0.0281 0.00043 2308.85 0.01543 64.7957 3870.69 59.7368<br />

360 212.7038 0.0047 0.00007 14114 0.01507 66.3532 4310.72 64.9662<br />

480 1269.70 0.0008 0.00001 84580 0.01501 66.6142 4415.74 66.2883


Compound Interest Factor Tables 587<br />

2% TABLE 7 Discrete Cash Flow: Compound Interest Factors 2%<br />

Single Payments Uniform Series Payments Arithmetic Gradients<br />

Compound Present Sinking Compound Capital Present Gradient Gradient<br />

Amount Worth Fund Amount Recovery Worth Present Worth Uniform Series<br />

n FP PF AF FA AP PA PG AG<br />

1 1.0200 0.9804 1.00000 1.0000 1.02000 0.9804<br />

2 1.0404 0.9612 0.49505 2.0200 0.51505 1.9416 0.9612 0.4950<br />

3 1.0612 0.9423 0.32675 3.0604 0.34675 2.8839 2.8458 0.9868<br />

4 1.0824 0.9238 0.24262 4.1216 0.26262 3.8077 5.6173 1.4752<br />

5 1.1041 0.9057 0.19216 5.2040 0.21216 4.7135 9.2403 1.9604<br />

6 1.1262 0.8880 0.15853 6.3081 0.17853 5.6014 13.6801 2.4423<br />

7 1.1487 0.8706 0.13451 7.4343 0.15451 6.4720 18.9035 2.9208<br />

8 1.1717 0.8535 0.11651 8.5830 0.13651 7.3255 24.8779 3.3961<br />

9 1.1951 0.8368 0.10252 9.7546 0.12252 8.1622 31.5720 3.8681<br />

10 1.2190 0.8203 0.09133 10.9497 0.11133 8.9826 38.9551 4.3367<br />

11 1.2434 0.8043 0.08218 12.1687 0.10218 9.7868 46.9977 4.8021<br />

12 1.2682 0.7885 0.07456 13.4121 0.09456 10.5753 55.6712 5.2642<br />

13 1.2936 0.7730 0.06812 14.6803 0.08812 11.3484 64.9475 5.7231<br />

14 1.3195 0.7579 0.06260 15.9739 0.08260 12.1062 74.7999 6.1786<br />

15 1.3459 0.7430 0.05783 17.2934 0.07783 12.8493 85.2021 6.6309<br />

16 1.3728 0.7284 0.05365 18.6393 0.07365 13.5777 96.1288 7.0799<br />

17 1.4002 0.7142 0.04997 20.0121 0.06997 14.2919 107.5554 7.5256<br />

18 1.4282 0.7002 0.04670 21.4123 0.06670 14.9920 119.4581 7.9681<br />

19 1.4568 0.6864 0.04378 22.8406 0.06378 15.6785 131.8139 8.4073<br />

20 1.4859 0.6730 0.04116 24.2974 0.06116 16.3514 144.6003 8.8433<br />

21 1.5157 0.6598 0.03878 25.7833 0.05878 17.0112 157.7959 9.2760<br />

22 1.5460 0.6468 0.03663 27.2990 0.05663 17.6580 171.3795 9.7055<br />

23 1.5769 0.6342 0.03467 28.8450 0.05467 18.2922 185.3309 10.1317<br />

24 1.6084 0.6217 0.03287 30.4219 0.05287 18.9139 199.6305 10.5547<br />

25 1.6406 0.6095 0.03122 32.0303 0.05122 19.5235 214.2592 10.9745<br />

26 1.6734 0.5976 0.02970 33.6709 0.04970 20.1210 229.1987 11.3910<br />

27 1.7069 0.5859 0.02829 35.3443 0.04829 20.7069 244.4311 11.8043<br />

28 1.7410 0.5744 0.02699 37.0512 0.04699 21.2813 259.9392 12.2145<br />

29 1.7758 0.5631 0.02578 38.7922 0.04578 21.8444 275.7064 12.6214<br />

30 1.8114 0.5521 0.02465 40.5681 0.04465 22.3965 291.7164 13.0251<br />

36 2.0399 0.4902 0.01923 51.9944 0.03923 25.4888 392.0405 15.3809<br />

40 2.2080 0.4529 0.01656 60.4020 0.03656 27.3555 461.9931 16.8885<br />

48 2.5871 0.3865 0.01260 79.3535 0.03260 30.6731 605.9657 19.7556<br />

50 2.6916 0.3715 0.01182 84.5794 0.03182 31.4236 642.3606 20.4420<br />

52 2.8003 0.3571 0.01111 90.0164 0.03111 32.1449 678.7849 21.1164<br />

55 2.9717 0.3365 0.01014 98.5865 0.03014 33.1748 733.3527 22.1057<br />

60 3.2810 0.3048 0.00877 114.0515 0.02877 34.7609 823.6975 23.6961<br />

72 4.1611 0.2403 0.00633 158.0570 0.02633 37.9841 1034.06 27.2234<br />

75 4.4158 0.2265 0.00586 170.7918 0.02586 38.6771 1084.64 28.0434<br />

84 5.2773 0.1895 0.00468 213.8666 0.02468 40.5255 1230.42 30.3616<br />

90 5.9431 0.1683 0.00405 247.1567 0.02405 41.5869 1322.17 31.7929<br />

96 6.6929 0.1494 0.00351 284.6467 0.02351 42.5294 1409.30 33.1370<br />

100 7.2446 0.1380 0.00320 312.2323 0.02320 43.0984 1464.75 33.9863<br />

108 8.4883 0.1178 0.00267 374.4129 0.02267 44.1095 1569.30 35.5774<br />

120 10.7652 0.0929 0.00205 488.2582 0.02205 45.3554 1710.42 37.7114<br />

132 13.6528 0.0732 0.00158 632.6415 0.02158 46.3378 1833.47 39.5676<br />

144 17.3151 0.0578 0.00123 815.7545 0.02123 47.1123 1939.79 41.1738<br />

240 115.8887 0.0086 0.00017 5744.44 0.02017 49.5686 2374.88 47.9110<br />

360 1247.56 0.0008 0.00002 62328 0.02002 49.9599 2482.57 49.7112<br />

480 13430 0.0001 0.02000 49.9963 2498.03 49.9643


588 Compound Interest Factor Tables<br />

3% TABLE 8 Discrete Cash Flow: Compound Interest Factors 3%<br />

Single Payments Uniform Series Payments Arithmetic Gradients<br />

Compound Present Sinking Compound Capital Present Gradient Gradient<br />

Amount Worth Fund Amount Recovery Worth Present Worth Uniform Series<br />

n FP PF AF FA AP PA PG AG<br />

1 1.0300 0.9709 1.00000 1.0000 1.03000 0.9709<br />

2 1.0609 0.9426 0.49261 2.0300 0.52261 1.9135 0.9426 0.4926<br />

3 1.0927 0.9151 0.32353 3.0909 0.35353 2.8286 2.7729 0.9803<br />

4 1.1255 0.8885 0.23903 4.1836 0.26903 3.7171 5.4383 1.4631<br />

5 1.1593 0.8626 0.18835 5.3091 0.21835 4.5797 8.8888 1.9409<br />

6 1.1941 0.8375 0.15460 6.4684 0.18460 5.4172 13.0762 2.4138<br />

7 1.2299 0.8131 0.13051 7.6625 0.16051 6.2303 17.9547 2.8819<br />

8 1.2668 0.7894 0.11246 8.8923 0.14246 7.0197 23.4806 3.3450<br />

9 1.3048 0.7664 0.09843 10.1591 0.12843 7.7861 29.6119 3.8032<br />

10 1.3439 0.7441 0.08723 11.4639 0.11723 8.5302 36.3088 4.2565<br />

11 1.3842 0.7224 0.07808 12.8078 0.10808 9.2526 43.5330 4.7049<br />

12 1.4258 0.7014 0.07046 14.1920 0.10046 9.9540 51.2482 5.1485<br />

13 1.4685 0.6810 0.06403 15.6178 0.09403 10.6350 59.4196 5.5872<br />

14 1.5126 0.6611 0.05853 17.0863 0.08853 11.2961 68.0141 6.0210<br />

15 1.5580 0.6419 0.05377 18.5989 0.08377 11.9379 77.0002 6.4500<br />

16 1.6047 0.6232 0.04961 20.1569 0.07961 12.5611 86.3477 6.8742<br />

17 1.6528 0.6050 0.04595 21.7616 0.07595 13.1661 96.0280 7.2936<br />

18 1.7024 0.5874 0.04271 23.4144 0.07271 13.7535 106.0137 7.7081<br />

19 1.7535 0.5703 0.03981 25.1169 0.06981 14.3238 116.2788 8.1179<br />

20 1.8061 0.5537 0.03722 26.8704 0.06722 14.8775 126.7987 8.5229<br />

21 1.8603 0.5375 0.03487 28.6765 0.06487 15.4150 137.5496 8.9231<br />

22 1.9161 0.5219 0.03275 30.5368 0.06275 15.9369 148.5094 9.3186<br />

23 1.9736 0.5067 0.03081 32.4529 0.06081 16.4436 159.6566 9.7093<br />

24 2.0328 0.4919 0.02905 34.4265 0.05905 16.9355 170.9711 10.0954<br />

25 2.0938 0.4776 0.02743 36.4593 0.05743 17.4131 182.4336 10.4768<br />

26 2.1566 0.4637 0.02594 38.5530 0.05594 17.8768 194.0260 10.8535<br />

27 2.2213 0.4502 0.02456 40.7096 0.05456 18.3270 205.7309 11.2255<br />

28 2.2879 0.4371 0.02329 42.9309 0.05329 18.7641 217.5320 11.5930<br />

29 2.3566 0.4243 0.02211 45.2189 0.05211 19.1885 229.4137 11.9558<br />

30 2.4273 0.4120 0.02102 47.5754 0.05102 19.6004 241.3613 12.3141<br />

31 2.5001 0.4000 0.02000 50.0027 0.05000 20.0004 253.3609 12.6678<br />

32 2.5751 0.3883 0.01905 52.5028 0.04905 20.3888 265.3993 13.0169<br />

33 2.6523 0.3770 0.01816 55.0778 0.04816 20.7658 277.4642 13.3616<br />

34 2.7319 0.3660 0.01732 57.7302 0.04732 21.1318 289.5437 13.7018<br />

35 2.8139 0.3554 0.01654 60.4621 0.04654 21.4872 301.6267 14.0375<br />

40 3.2620 0.3066 0.01326 75.4013 0.04326 23.1148 361.7499 15.6502<br />

45 3.7816 0.2644 0.01079 92.7199 0.04079 24.5187 420.6325 17.1556<br />

50 4.3839 0.2281 0.00887 112.7969 0.03887 25.7298 477.4803 18.5575<br />

55 5.0821 0.1968 0.00735 136.0716 0.03735 26.7744 531.7411 19.8600<br />

60 5.8916 0.1697 0.00613 163.0534 0.03613 27.6756 583.0526 21.0674<br />

65 6.8300 0.1464 0.00515 194.3328 0.03515 28.4529 631.2010 22.1841<br />

70 7.9178 0.1263 0.00434 230.5941 0.03434 29.1234 676.0869 23.2145<br />

75 9.1789 0.1089 0.00367 272.6309 0.03367 29.7018 717.6978 24.1634<br />

80 10.6409 0.0940 0.00311 321.3630 0.03311 30.2008 756.0865 25.0353<br />

84 11.9764 0.0835 0.00273 365.8805 0.03273 30.5501 784.5434 25.6806<br />

85 12.3357 0.0811 0.00265 377.8570 0.03265 30.6312 791.3529 25.8349<br />

90 14.3005 0.0699 0.00226 443.3489 0.03226 31.0024 823.6302 26.5667<br />

96 17.0755 0.0586 0.00187 535.8502 0.03187 31.3812 858.6377 27.3615<br />

108 24.3456 0.0411 0.00129 778.1863 0.03129 31.9642 917.6013 28.7072<br />

120 34.7110 0.0288 0.00089 1123.70 0.03089 32.3730 963.8635 29.7737


Compound Interest Factor Tables 589<br />

4% TABLE 9 Discrete Cash Flow: Compound Interest Factors 4%<br />

Single Payments Uniform Series Payments Arithmetic Gradients<br />

Compound Present Sinking Compound Capital Present Gradient Gradient<br />

Amount Worth Fund Amount Recovery Worth Present Worth Uniform Series<br />

n FP PF AF FA AP PA PG AG<br />

1 1.0400 0.9615 1.00000 1.0000 1.04000 0.9615<br />

2 1.0816 0.9246 0.49020 2.0400 0.53020 1.8861 0.9246 0.4902<br />

3 1.1249 0.8890 0.32035 3.1216 0.36035 2.7751 2.7025 0.9739<br />

4 1.1699 0.8548 0.23549 4.2465 0.27549 3.6299 5.2670 1.4510<br />

5 1.2167 0.8219 0.18463 5.4163 0.22463 4.4518 8.5547 1.9216<br />

6 1.2653 0.7903 0.15076 6.6330 0.19076 5.2421 12.5062 2.3857<br />

7 1.3159 0.7599 0.12661 7.8983 0.16661 6.0021 17.0657 2.8433<br />

8 1.3686 0.7307 0.10853 9.2142 0.14853 6.7327 22.1806 3.2944<br />

9 1.4233 0.7026 0.09449 10.5828 0.13449 7.4353 27.8013 3.7391<br />

10 1.4802 0.6756 0.08329 12.0061 0.12329 8.1109 33.8814 4.1773<br />

11 1.5395 0.6496 0.07415 13.4864 0.11415 8.7605 40.3772 4.6090<br />

12 1.6010 0.6246 0.06655 15.0258 0.10655 9.3851 47.2477 5.0343<br />

13 1.6651 0.6006 0.06014 16.6268 0.10014 9.9856 54.4546 5.4533<br />

14 1.7317 0.5775 0.05467 18.2919 0.09467 10.5631 61.9618 5.8659<br />

15 1.8009 0.5553 0.04994 20.0236 0.08994 11.1184 69.7355 6.2721<br />

16 1.8730 0.5339 0.04582 21.8245 0.08582 11.6523 77.7441 6.6720<br />

17 1.9479 0.5134 0.04220 23.6975 0.08220 12.1657 85.9581 7.0656<br />

18 2.0258 0.4936 0.03899 25.6454 0.07899 12.6593 94.3498 7.4530<br />

19 2.1068 0.4746 0.03614 27.6712 0.07614 13.1339 102.8933 7.8342<br />

20 2.1911 0.4564 0.03358 29.7781 0.07358 13.5903 111.5647 8.2091<br />

21 2.2788 0.4388 0.03128 31.9692 0.07128 14.0292 120.3414 8.5779<br />

22 2.3699 0.4220 0.02920 34.2480 0.06920 14.4511 129.2024 8.9407<br />

23 2.4647 0.4057 0.02731 36.6179 0.06731 14.8568 138.1284 9.2973<br />

24 2.5633 0.3901 0.02559 39.0826 0.06559 15.2470 147.1012 9.6479<br />

25 2.6658 0.3751 0.02401 41.6459 0.06401 15.6221 156.1040 9.9925<br />

26 2.7725 0.3607 0.02257 44.3117 0.06257 15.9828 165.1212 10.3312<br />

27 2.8834 0.3468 0.02124 47.0842 0.06124 16.3296 174.1385 10.6640<br />

28 2.9987 0.3335 0.02001 49.9676 0.06001 16.6631 183.1424 10.9909<br />

29 3.1187 0.3207 0.01888 52.9663 0.05888 16.9837 192.1206 11.3120<br />

30 3.2434 0.3083 0.01783 56.0849 0.05783 17.2920 201.0618 11.6274<br />

31 3.3731 0.2965 0.01686 59.3283 0.05686 17.5885 209.9556 11.9371<br />

32 3.5081 0.2851 0.01595 62.7015 0.05595 17.8736 218.7924 12.2411<br />

33 3.6484 0.2741 0.01510 66.2095 0.05510 18.1476 227.5634 12.5396<br />

34 3.7943 0.2636 0.01431 69.8579 0.05431 18.4112 236.2607 12.8324<br />

35 3.9461 0.2534 0.01358 73.6522 0.05358 18.6646 244.8768 13.1198<br />

40 4.8010 0.2083 0.01052 95.0255 0.05052 19.7928 286.5303 14.4765<br />

45 5.8412 0.1712 0.00826 121.0294 0.04826 20.7200 325.4028 15.7047<br />

50 7.1067 0.1407 0.00655 152.6671 0.04655 21.4822 361.1638 16.8122<br />

55 8.6464 0.1157 0.00523 191.1592 0.04523 22.1086 393.6890 17.8070<br />

60 10.5196 0.0951 0.00420 237.9907 0.04420 22.6235 422.9966 18.6972<br />

65 12.7987 0.0781 0.00339 294.9684 0.04339 23.0467 449.2014 19.4909<br />

70 15.5716 0.0642 0.00275 364.2905 0.04275 23.3945 472.4789 20.1961<br />

75 18.9453 0.0528 0.00223 448.6314 0.04223 23.6804 493.0408 20.8206<br />

80 23.0498 0.0434 0.00181 551.2450 0.04181 23.9154 511.1161 21.3718<br />

85 28.0436 0.0357 0.00148 676.0901 0.04148 24.1085 526.9384 21.8569<br />

90 34.1193 0.0293 0.00121 827.9833 0.04121 24.2673 540.7369 22.2826<br />

96 43.1718 0.0232 0.00095 1054.30 0.04095 24.4209 554.9312 22.7236<br />

108 69.1195 0.0145 0.00059 1702.99 0.04059 24.6383 576.8949 23.4146<br />

120 110.6626 0.0090 0.00036 2741.56 0.04036 24.7741 592.2428 23.9057<br />

144 283.6618 0.0035 0.00014 7066.55 0.04014 24.9119 610.1055 24.4906


590 Compound Interest Factor Tables<br />

5% TABLE 10 Discrete Cash Flow: Compound Interest Factors 5%<br />

Single Payments Uniform Series Payments Arithmetic Gradients<br />

Compound Present Sinking Compound Capital Present Gradient Gradient<br />

Amount Worth Fund Amount Recovery Worth Present Worth Uniform Series<br />

n FP PF AF FA AP PA PG AG<br />

1 1.0500 0.9524 1.00000 1.0000 1.05000 0.9524<br />

2 1.1025 0.9070 0.48780 2.0500 0.53780 1.8594 0.9070 0.4878<br />

3 1.1576 0.8638 0.31721 3.1525 0.36721 2.7232 2.6347 0.9675<br />

4 1.2155 0.8227 0.23201 4.3101 0.28201 3.5460 5.1028 1.4391<br />

5 1.2763 0.7835 0.18097 5.5256 0.23097 4.3295 8.2369 1.9025<br />

6 1.3401 0.7462 0.14702 6.8019 0.19702 5.0757 11.9680 2.3579<br />

7 1.4071 0.7107 0.12282 8.1420 0.17282 5.7864 16.2321 2.8052<br />

8 1.4775 0.6768 0.10472 9.5491 0.15472 6.4632 20.9700 3.2445<br />

9 1.5513 0.6446 0.09069 11.0266 0.14069 7.1078 26.1268 3.6758<br />

10 1.6289 0.6139 0.07950 12.5779 0.12950 7.7217 31.6520 4.0991<br />

11 1.7103 0.5847 0.07039 14.2068 0.12039 8.3064 37.4988 4.5144<br />

12 1.7959 0.5568 0.06283 15.9171 0.11283 8.8633 43.6241 4.9219<br />

13 1.8856 0.5303 0.05646 17.7130 0.10646 9.3936 49.9879 5.3215<br />

14 1.9799 0.5051 0.05102 19.5986 0.10102 9.8986 56.5538 5.7133<br />

15 2.0789 0.4810 0.04634 21.5786 0.09634 10.3797 63.2880 6.0973<br />

16 2.1829 0.4581 0.04227 23.6575 0.09227 10.8378 70.1597 6.4736<br />

17 2.2920 0.4363 0.03870 25.8404 0.08870 11.2741 77.1405 6.8423<br />

18 2.4066 0.4155 0.03555 28.1324 0.08555 11.6896 84.2043 7.2034<br />

19 2.5270 0.3957 0.03275 30.5390 0.08275 12.0853 91.3275 7.5569<br />

20 2.6533 0.3769 0.03024 33.0660 0.08024 12.4622 98.4884 7.9030<br />

21 2.7860 0.3589 0.02800 35.7193 0.07800 12.8212 105.6673 8.2416<br />

22 2.9253 0.3418 0.02597 38.5052 0.07597 13.1630 112.8461 8.5730<br />

23 3.0715 0.3256 0.02414 41.4305 0.07414 13.4886 120.0087 8.8971<br />

24 3.2251 0.3101 0.02247 44.5020 0.07247 13.7986 127.1402 9.2140<br />

25 3.3864 0.2953 0.02095 47.7271 0.07095 14.0939 134.2275 9.5238<br />

26 3.5557 0.2812 0.01956 51.1135 0.06956 14.3752 141.2585 9.8266<br />

27 3.7335 0.2678 0.01829 54.6691 0.06829 14.6430 148.2226 10.1224<br />

28 3.9201 0.2551 0.01712 58.4026 0.06712 14.8981 155.1101 10.4114<br />

29 4.1161 0.2429 0.01605 62.3227 0.06605 15.1411 161.9126 10.6936<br />

30 4.3219 0.2314 0.01505 66.4388 0.06505 15.3725 168.6226 10.9691<br />

31 4.5380 0.2204 0.01413 70.7608 0.06413 15.5928 175.2333 11.2381<br />

32 4.7649 0.2099 0.01328 75.2988 0.06328 15.8027 181.7392 11.5005<br />

33 5.0032 0.1999 0.01249 80.0638 0.06249 16.0025 188.1351 11.7566<br />

34 5.2533 0.1904 0.01176 85.0670 0.06176 16.1929 194.4168 12.0063<br />

35 5.5160 0.1813 0.01107 90.3203 0.06107 16.3742 200.5807 12.2498<br />

40 7.0400 0.1420 0.00828 120.7998 0.05828 17.1591 229.5452 13.3775<br />

45 8.9850 0.1113 0.00626 159.7002 0.05626 17.7741 255.3145 14.3644<br />

50 11.4674 0.0872 0.00478 209.3480 0.05478 18.2559 277.9148 15.2233<br />

55 14.6356 0.0683 0.00367 272.7126 0.05367 18.6335 297.5104 15.9664<br />

60 18.6792 0.0535 0.00283 353.5837 0.05283 18.9293 314.3432 16.6062<br />

65 23.8399 0.0419 0.00219 456.7980 0.05219 19.1611 328.6910 17.1541<br />

70 30.4264 0.0329 0.00170 588.5285 0.05170 19.3427 340.8409 17.6212<br />

75 38.8327 0.0258 0.00132 756.6537 0.05132 19.4850 351.0721 18.0176<br />

80 49.5614 0.0202 0.00103 971.2288 0.05103 19.5965 359.6460 18.3526<br />

85 63.2544 0.0158 0.00080 1245.09 0.05080 19.6838 366.8007 18.6346<br />

90 80.7304 0.0124 0.00063 1594.61 0.05063 19.7523 372.7488 18.8712<br />

95 103.0347 0.0097 0.00049 2040.69 0.05049 19.8059 377.6774 19.0689<br />

96 108.1864 0.0092 0.00047 2143.73 0.05047 19.8151 378.5555 19.1044<br />

98 119.2755 0.0084 0.00042 2365.51 0.05042 19.8323 380.2139 19.1714<br />

100 131.5013 0.0076 0.00038 2610.03 0.05038 19.8479 381.7492 19.2337


Compound Interest Factor Tables 591<br />

6% TABLE 11 Discrete Cash Flow: Compound Interest Factors 6%<br />

Single Payments Uniform Series Payments Arithmetic Gradients<br />

Compound Present Sinking Compound Capital Present Gradient Gradient<br />

Amount Worth Fund Amount Recovery Worth Present Worth Uniform Series<br />

n FP PF AF FA AP PA PG AG<br />

1 1.0600 0.9434 1.00000 1.0000 1.06000 0.9434<br />

2 1.1236 0.8900 0.48544 2.0600 0.54544 1.8334 0.8900 0.4854<br />

3 1.1910 0.8396 0.31411 3.1836 0.37411 2.6730 2.5692 0.9612<br />

4 1.2625 0.7921 0.22859 4.3746 0.28859 3.4651 4.9455 1.4272<br />

5 1.3382 0.7473 0.17740 5.6371 0.23740 4.2124 7.9345 1.8836<br />

6 1.4185 0.7050 0.14336 6.9753 0.20336 4.9173 11.4594 2.3304<br />

7 1.5036 0.6651 0.11914 8.3938 0.17914 5.5824 15.4497 2.7676<br />

8 1.5938 0.6274 0.10104 9.8975 0.16104 6.2098 19.8416 3.1952<br />

9 1.6895 0.5919 0.08702 11.4913 0.14702 6.8017 24.5768 3.6133<br />

10 1.7908 0.5584 0.07587 13.1808 0.13587 7.3601 29.6023 4.0220<br />

11 1.8983 0.5268 0.06679 14.9716 0.12679 7.8869 34.8702 4.4213<br />

12 2.0122 0.4970 0.05928 16.8699 0.11928 8.3838 40.3369 4.8113<br />

13 2.1329 0.4688 0.05296 18.8821 0.11296 8.8527 45.9629 5.1920<br />

14 2.2609 0.4423 0.04758 21.0151 0.10758 9.2950 51.7128 5.5635<br />

15 2.3966 0.4173 0.04296 23.2760 0.10296 9.7122 57.5546 5.9260<br />

16 2.5404 0.3936 0.03895 25.6725 0.09895 10.1059 63.4592 6.2794<br />

17 2.6928 0.3714 0.03544 28.2129 0.09544 10.4773 69.4011 6.6240<br />

18 2.8543 0.3503 0.03236 30.9057 0.09236 10.8276 75.3569 6.9597<br />

19 3.0256 0.3305 0.02962 33.7600 0.08962 11.1581 81.3062 7.2867<br />

20 3.2071 0.3118 0.02718 36.7856 0.08718 11.4699 87.2304 7.6051<br />

21 3.3996 0.2942 0.02500 39.9927 0.08500 11.7641 93.1136 7.9151<br />

22 3.6035 0.2775 0.02305 43.3923 0.08305 12.0416 98.9412 8.2166<br />

23 3.8197 0.2618 0.02128 46.9958 0.08128 12.3034 104.7007 8.5099<br />

24 4.0489 0.2470 0.01968 50.8156 0.07968 12.5504 110.3812 8.7951<br />

25 4.2919 0.2330 0.01823 54.8645 0.07823 12.7834 115.9732 9.0722<br />

26 4.5494 0.2198 0.01690 59.1564 0.07690 13.0032 121.4684 9.3414<br />

27 4.8223 0.2074 0.01570 63.7058 0.07570 13.2105 126.8600 9.6029<br />

28 5.1117 0.1956 0.01459 68.5281 0.07459 13.4062 132.1420 9.8568<br />

29 5.4184 0.1846 0.01358 73.6398 0.07358 13.5907 137.3096 10.1032<br />

30 5.7435 0.1741 0.01265 79.0582 0.07265 13.7648 142.3588 10.3422<br />

31 6.0881 0.1643 0.01179 84.8017 0.07179 13.9291 147.2864 10.5740<br />

32 6.4534 0.1550 0.01100 90.8898 0.07100 14.0840 152.0901 10.7988<br />

33 6.8406 0.1462 0.01027 97.3432 0.07027 14.2302 156.7681 11.0166<br />

34 7.2510 0.1379 0.00960 104.1838 0.06960 14.3681 161.3192 11.2276<br />

35 7.6861 0.1301 0.00897 111.4348 0.06897 14.4982 165.7427 11.4319<br />

40 10.2857 0.0972 0.00646 154.7620 0.06646 15.0463 185.9568 12.3590<br />

45 13.7646 0.0727 0.00470 212.7435 0.06470 15.4558 203.1096 13.1413<br />

50 18.4202 0.0543 0.00344 290.3359 0.06344 15.7619 217.4574 13.7964<br />

55 24.6503 0.0406 0.00254 394.1720 0.06254 15.9905 229.3222 14.3411<br />

60 32.9877 0.0303 0.00188 533.1282 0.06188 16.1614 239.0428 14.7909<br />

65 44.1450 0.0227 0.00139 719.0829 0.06139 16.2891 246.9450 15.1601<br />

70 59.0759 0.0169 0.00103 967.9322 0.06103 16.3845 253.3271 15.4613<br />

75 79.0569 0.0126 0.00077 1300.95 0.06077 16.4558 258.4527 15.7058<br />

80 105.7960 0.0095 0.00057 1746.60 0.06057 16.5091 262.5493 15.9033<br />

85 141.5789 0.0071 0.00043 2342.98 0.06043 16.5489 265.8096 16.0620<br />

90 189.4645 0.0053 0.00032 3141.08 0.06032 16.5787 268.3946 16.1891<br />

95 253.5463 0.0039 0.00024 4209.10 0.06024 16.6009 270.4375 16.2905<br />

96 268.7590 0.0037 0.00022 4462.65 0.06022 16.6047 270.7909 16.3081<br />

98 301.9776 0.0033 0.00020 5016.29 0.06020 16.6115 271.4491 16.3411<br />

100 339.3021 0.0029 0.00018 5638.37 0.06018 16.6175 272.0471 16.3711


592 Compound Interest Factor Tables<br />

7% TABLE 12 Discrete Cash Flow: Compound Interest Factors 7%<br />

Single Payments Uniform Series Payments Arithmetic Gradients<br />

Compound Present Sinking Compound Capital Present Gradient Gradient<br />

Amount Worth Fund Amount Recovery Worth Present Worth Uniform Series<br />

n FP PF AF FA AP PA PG AG<br />

1 1.0700 0.9346 1.00000 1.0000 1.07000 0.9346<br />

2 1.1449 0.8734 0.48309 2.0700 0.55309 1.8080 0.8734 0.4831<br />

3 1.2250 0.8163 0.31105 3.2149 0.38105 2.6243 2.5060 0.9549<br />

4 1.3108 0.7629 0.22523 4.4399 0.29523 3.3872 4.7947 1.4155<br />

5 1.4026 0.7130 0.17389 5.7507 0.24389 4.1002 7.6467 1.8650<br />

6 1.5007 0.6663 0.13980 7.1533 0.20980 4.7665 10.9784 2.3032<br />

7 1.6058 0.6227 0.11555 8.6540 0.18555 5.3893 14.7149 2.7304<br />

8 1.7182 0.5820 0.09747 10.2598 0.16747 5.9713 18.7889 3.1465<br />

9 1.8385 0.5439 0.08349 11.9780 0.15349 6.5152 23.1404 3.5517<br />

10 1.9672 0.5083 0.07238 13.8164 0.14238 7.0236 27.7156 3.9461<br />

11 2.1049 0.4751 0.06336 15.7836 0.13336 7.4987 32.4665 4.3296<br />

12 2.2522 0.4440 0.05590 17.8885 0.12590 7.9427 37.3506 4.7025<br />

13 2.4098 0.4150 0.04965 20.1406 0.11965 8.3577 42.3302 5.0648<br />

14 2.5785 0.3878 0.04434 22.5505 0.11434 8.7455 47.3718 5.4167<br />

15 2.7590 0.3624 0.03979 25.1290 0.10979 9.1079 52.4461 5.7583<br />

16 2.9522 0.3387 0.03586 27.8881 0.10586 9.4466 57.5271 6.0897<br />

17 3.1588 0.3166 0.03243 30.8402 0.10243 9.7632 62.5923 6.4110<br />

18 3.3799 0.2959 0.02941 33.9990 0.09941 10.0591 67.6219 6.7225<br />

19 3.6165 0.2765 0.02675 37.3790 0.09675 10.3356 72.5991 7.0242<br />

20 3.8697 0.2584 0.02439 40.9955 0.09439 10.5940 77.5091 7.3163<br />

21 4.1406 0.2415 0.02229 44.8652 0.09229 10.8355 82.3393 7.5990<br />

22 4.4304 0.2257 0.02041 49.0057 0.09041 11.0612 87.0793 7.8725<br />

23 4.7405 0.2109 0.01871 53.4361 0.08871 11.2722 91.7201 8.1369<br />

24 5.0724 0.1971 0.01719 58.1767 0.08719 11.4693 96.2545 8.3923<br />

25 5.4274 0.1842 0.01581 63.2490 0.08581 11.6536 100.6765 8.6391<br />

26 5.8074 0.1722 0.01456 68.6765 0.08456 11.8258 104.9814 8.8773<br />

27 6.2139 0.1609 0.01343 74.4838 0.08343 11.9867 109.1656 9.1072<br />

28 6.6488 0.1504 0.01239 80.6977 0.08239 12.1371 113.2264 9.3289<br />

29 7.1143 0.1406 0.01145 87.3465 0.08145 12.2777 117.1622 9.5427<br />

30 7.6123 0.1314 0.01059 94.4608 0.08059 12.4090 120.9718 9.7487<br />

31 8.1451 0.1228 0.00980 102.0730 0.07980 12.5318 124.6550 9.9471<br />

32 8.7153 0.1147 0.00907 110.2182 0.07907 12.6466 128.2120 10.1381<br />

33 9.3253 0.1072 0.00841 118.9334 0.07841 12.7538 131.6435 10.3219<br />

34 9.9781 0.1002 0.00780 128.2588 0.07780 12.8540 134.9507 10.4987<br />

35 10.6766 0.0937 0.00723 138.2369 0.07723 12.9477 138.1353 10.6687<br />

40 14.9745 0.0668 0.00501 199.6351 0.07501 13.3317 152.2928 11.4233<br />

45 21.0025 0.0476 0.00350 285.7493 0.07350 13.6055 163.7559 12.0360<br />

50 29.4570 0.0339 0.00246 406.5289 0.07246 13.8007 172.9051 12.5287<br />

55 41.3150 0.0242 0.00174 575.9286 0.07174 13.9399 180.1243 12.9215<br />

60 57.9464 0.0173 0.00123 813.5204 0.07123 14.0392 185.7677 13.2321<br />

65 81.2729 0.0123 0.00087 1146.76 0.07087 14.1099 190.1452 13.4760<br />

70 113.9894 0.0088 0.00062 1614.13 0.07062 14.1604 193.5185 13.6662<br />

75 159.8760 0.0063 0.00044 2269.66 0.07044 14.1964 196.1035 13.8136<br />

80 224.2344 0.0045 0.00031 3189.06 0.07031 14.2220 198.0748 13.9273<br />

85 314.5003 0.0032 0.00022 4478.58 0.07022 14.2403 199.5717 14.0146<br />

90 441.1030 0.0023 0.00016 6287.19 0.07016 14.2533 200.7042 14.0812<br />

95 618.6697 0.0016 0.00011 8823.85 0.07011 14.2626 201.5581 14.1319<br />

96 661.9766 0.0015 0.00011 9442.52 0.07011 14.2641 201.7016 14.1405<br />

98 757.8970 0.0013 0.00009 10813 0.07009 14.2669 201.9651 14.1562<br />

100 867.7163 0.0012 0.00008 12382 0.07008 14.2693 202.2001 14.1703


Compound Interest Factor Tables 593<br />

8% TABLE 13 Discrete Cash Flow: Compound Interest Factors 8%<br />

Single Payments Uniform Series Payments Arithmetic Gradients<br />

Compound Present Sinking Compound Capital Present Gradient Gradient<br />

Amount Worth Fund Amount Recovery Worth Present Worth Uniform Series<br />

n FP PF AF FA AP PA PG AG<br />

1 1.0800 0.9259 1.00000 1.0000 1.08000 0.9259<br />

2 1.1664 0.8573 0.48077 2.0800 0.56077 1.7833 0.8573 0.4808<br />

3 1.2597 0.7938 0.30803 3.2464 0.38803 2.5771 2.4450 0.9487<br />

4 1.3605 0.7350 0.22192 4.5061 0.30192 3.3121 4.6501 1.4040<br />

5 1.4693 0.6806 0.17046 5.8666 0.25046 3.9927 7.3724 1.8465<br />

6 1.5869 0.6302 0.13632 7.3359 0.21632 4.6229 10.5233 2.2763<br />

7 1.7138 0.5835 0.11207 8.9228 0.19207 5.2064 14.0242 2.6937<br />

8 1.8509 0.5403 0.09401 10.6366 0.17401 5.7466 17.8061 3.0985<br />

9 1.9990 0.5002 0.08008 12.4876 0.16008 6.2469 21.8081 3.4910<br />

10 2.1589 0.4632 0.06903 14.4866 0.14903 6.7101 25.9768 3.8713<br />

11 2.3316 0.4289 0.06008 16.6455 0.14008 7.1390 30.2657 4.2395<br />

12 2.5182 0.3971 0.05270 18.9771 0.13270 7.5361 34.6339 4.5957<br />

13 2.7196 0.3677 0.04652 21.4953 0.12652 7.9038 39.0463 4.9402<br />

14 2.9372 0.3405 0.04130 24.2149 0.12130 8.2442 43.4723 5.2731<br />

15 3.1722 0.3152 0.03683 27.1521 0.11683 8.5595 47.8857 5.5945<br />

16 3.4259 0.2919 0.03298 30.3243 0.11298 8.8514 52.2640 5.9046<br />

17 3.7000 0.2703 0.02963 33.7502 0.10963 9.1216 56.5883 6.2037<br />

18 3.9960 0.2502 0.02670 37.4502 0.10670 9.3719 60.8426 6.4920<br />

19 4.3157 0.2317 0.02413 41.4463 0.10413 9.6036 65.0134 6.7697<br />

20 4.6610 0.2145 0.02185 45.7620 0.10185 9.8181 69.0898 7.0369<br />

21 5.0338 0.1987 0.01983 50.4229 0.09983 10.0168 73.0629 7.2940<br />

22 5.4365 0.1839 0.01803 55.4568 0.09803 10.2007 76.9257 7.5412<br />

23 5.8715 0.1703 0.01642 60.8933 0.09642 10.3711 80.6726 7.7786<br />

24 6.3412 0.1577 0.01498 66.7648 0.09498 10.5288 84.2997 8.0066<br />

25 6.8485 0.1460 0.01368 73.1059 0.09368 10.6748 87.8041 8.2254<br />

26 7.3964 0.1352 0.01251 79.9544 0.09251 10.8100 91.1842 8.4352<br />

27 7.9881 0.1252 0.01145 87.3508 0.09145 10.9352 94.4390 8.6363<br />

28 8.6271 0.1159 0.01049 95.3388 0.09049 11.0511 97.5687 8.8289<br />

29 9.3173 0.1073 0.00962 103.9659 0.08962 11.1584 100.5738 9.0133<br />

30 10.0627 0.0994 0.00883 113.2832 0.08883 11.2578 103.4558 9.1897<br />

31 10.8677 0.0920 0.00811 123.3459 0.08811 11.3498 106.2163 9.3584<br />

32 11.7371 0.0852 0.00745 134.2135 0.08745 11.4350 108.8575 9.5197<br />

33 12.6760 0.0789 0.00685 145.9506 0.08685 11.5139 111.3819 9.6737<br />

34 13.6901 0.0730 0.00630 158.6267 0.08630 11.5869 113.7924 9.8208<br />

35 14.7853 0.0676 0.00580 172.3168 0.08580 11.6546 116.0920 9.9611<br />

40 21.7245 0.0460 0.00386 259.0565 0.08386 11.9246 126.0422 10.5699<br />

45 31.9204 0.0313 0.00259 386.5056 0.08259 12.1084 133.7331 11.0447<br />

50 46.9016 0.0213 0.00174 573.7702 0.08174 12.2335 139.5928 11.4107<br />

55 68.9139 0.0145 0.00118 848.9232 0.08118 12.3186 144.0065 11.6902<br />

60 101.2571 0.0099 0.00080 1253.21 0.08080 12.3766 147.3000 11.9015<br />

65 148.7798 0.0067 0.00054 1847.25 0.08054 12.4160 149.7387 12.0602<br />

70 218.6064 0.0046 0.00037 2720.08 0.08037 12.4428 151.5326 12.1783<br />

75 321.2045 0.0031 0.00025 4002.56 0.08025 12.4611 152.8448 12.2658<br />

80 471.9548 0.0021 0.00017 5886.94 0.08017 12.4735 153.8001 12.3301<br />

85 693.4565 0.0014 0.00012 8655.71 0.08012 12.4820 154.4925 12.3772<br />

90 1018.92 0.0010 0.00008 12724 0.08008 12.4877 154.9925 12.4116<br />

95 1497.12 0.0007 0.00005 18702 0.08005 12.4917 155.3524 12.4365<br />

96 1616.89 0.0006 0.00005 20199 0.08005 12.4923 155.4112 12.4406<br />

98 1885.94 0.0005 0.00004 23562 0.08004 12.4934 155.5176 12.4480<br />

100 2199.76 0.0005 0.00004 27485 0.08004 12.4943 155.6107 12.4545


594 Compound Interest Factor Tables<br />

9% TABLE 14 Discrete Cash Flow: Compound Interest Factors 9%<br />

Single Payments Uniform Series Payments Arithmetic Gradients<br />

Compound Present Sinking Compound Capital Present Gradient Gradient<br />

Amount Worth Fund Amount Recovery Worth Present Worth Uniform Series<br />

n FP PF AF FA AP PA PG AG<br />

1 1.0900 0.9174 1.00000 1.0000 1.09000 0.9174<br />

2 1.1881 0.8417 0.47847 2.0900 0.56847 1.7591 0.8417 0.4785<br />

3 1.2950 0.7722 0.30505 3.2781 0.39505 2.5313 2.3860 0.9426<br />

4 1.4116 0.7084 0.21867 4.5731 0.30867 3.2397 4.5113 1.3925<br />

5 1.5386 0.6499 0.16709 5.9847 0.25709 3.8897 7.1110 1.8282<br />

6 1.6771 0.5963 0.13292 7.5233 0.22292 4.4859 10.0924 2.2498<br />

7 1.8280 0.5470 0.10869 9.2004 0.19869 5.0330 13.3746 2.6574<br />

8 1.9926 0.5019 0.09067 11.0285 0.18067 5.5348 16.8877 3.0512<br />

9 2.1719 0.4604 0.07680 13.0210 0.16680 5.9952 20.5711 3.4312<br />

10 2.3674 0.4224 0.06582 15.1929 0.15582 6.4177 24.3728 3.7978<br />

11 2.5804 0.3875 0.05695 17.5603 0.14695 6.8052 28.2481 4.1510<br />

12 2.8127 0.3555 0.04965 20.1407 0.13965 7.1607 32.1590 4.4910<br />

13 3.0658 0.3262 0.04357 22.9534 0.13357 7.4869 36.0731 4.8182<br />

14 3.3417 0.2992 0.03843 26.0192 0.12843 7.7862 39.9633 5.1326<br />

15 3.6425 0.2745 0.03406 29.3609 0.12406 8.0607 43.8069 5.4346<br />

16 3.9703 0.2519 0.03030 33.0034 0.12030 8.3126 47.5849 5.7245<br />

17 4.3276 0.2311 0.02705 36.9737 0.11705 8.5436 51.2821 6.0024<br />

18 4.7171 0.2120 0.02421 41.3013 0.11421 8.7556 54.8860 6.2687<br />

19 5.1417 0.1945 0.02173 46.0185 0.11173 8.9501 58.3868 6.5236<br />

20 5.6044 0.1784 0.01955 51.1601 0.10955 9.1285 61.7770 6.7674<br />

21 6.1088 0.1637 0.01762 56.7645 0.10762 9.2922 65.0509 7.0006<br />

22 6.6586 0.1502 0.01590 62.8733 0.10590 9.4424 68.2048 7.2232<br />

23 7.2579 0.1378 0.01438 69.5319 0.10438 9.5802 71.2359 7.4357<br />

24 7.9111 0.1264 0.01302 76.7898 0.10302 9.7066 74.1433 7.6384<br />

25 8.6231 0.1160 0.01181 84.7009 0.10181 9.8226 76.9265 7.8316<br />

26 9.3992 0.1064 0.01072 93.3240 0.10072 9.9290 79.5863 8.0156<br />

27 10.2451 0.0976 0.00973 102.7231 0.09973 10.0266 82.1241 8.1906<br />

28 11.1671 0.0895 0.00885 112.9682 0.09885 10.1161 84.5419 8.3571<br />

29 12.1722 0.0822 0.00806 124.1354 0.09806 10.1983 86.8422 8.5154<br />

30 13.2677 0.0754 0.00734 136.3075 0.09734 10.2737 89.0280 8.6657<br />

31 14.4618 0.0691 0.00669 149.5752 0.09669 10.3428 91.1024 8.8083<br />

32 15.7633 0.0634 0.00610 164.0370 0.09610 10.4062 93.0690 8.9436<br />

33 17.1820 0.0582 0.00556 179.8003 0.09556 10.4644 94.9314 9.0718<br />

34 18.7284 0.0534 0.00508 196.9823 0.09508 10.5178 96.6935 9.1933<br />

35 20.4140 0.0490 0.00464 215.7108 0.09464 10.5668 98.3590 9.3083<br />

40 31.4094 0.0318 0.00296 337.8824 0.09296 10.7574 105.3762 9.7957<br />

45 48.3273 0.0207 0.00190 525.8587 0.09190 10.8812 110.5561 10.1603<br />

50 74.3575 0.0134 0.00123 815.0836 0.09123 10.9617 114.3251 10.4295<br />

55 114.4083 0.0087 0.00079 1260.09 0.09079 11.0140 117.0362 10.6261<br />

60 176.0313 0.0057 0.00051 1944.79 0.09051 11.0480 118.9683 10.7683<br />

65 270.8460 0.0037 0.00033 2998.29 0.09033 11.0701 120.3344 10.8702<br />

70 416.7301 0.0024 0.00022 4619.22 0.09022 11.0844 121.2942 10.9427<br />

75 641.1909 0.0016 0.00014 7113.23 0.09014 11.0938 121.9646 10.9940<br />

80 986.5517 0.0010 0.00009 10951 0.09009 11.0998 122.4306 11.0299<br />

85 1517.93 0.0007 0.00006 16855 0.09006 11.1038 122.7533 11.0551<br />

90 2335.53 0.0004 0.00004 25939 0.09004 11.1064 122.9758 11.0726<br />

95 3593.50 0.0003 0.00003 39917 0.09003 11.1080 123.1287 11.0847<br />

96 3916.91 0.0003 0.00002 43510 0.09002 11.1083 123.1529 11.0866<br />

98 4653.68 0.0002 0.00002 51696 0.09002 11.1087 123.1963 11.0900<br />

100 5529.04 0.0002 0.00002 61423 0.09002 11.1091 123.2335 11.0930


Compound Interest Factor Tables 595<br />

10% TABLE 15 Discrete Cash Flow: Compound Interest Factors 10%<br />

Single Payments Uniform Series Payments Arithmetic Gradients<br />

Compound Present Sinking Compound Capital Present Gradient Gradient<br />

Amount Worth Fund Amount Recovery Worth Present Worth Uniform Series<br />

n FP PF AF FA AP PA PG AG<br />

1 1.1000 0.9091 1.00000 1.0000 1.10000 0.9091<br />

2 1.2100 0.8264 0.47619 2.1000 0.57619 1.7355 0.8264 0.4762<br />

3 1.3310 0.7513 0.30211 3.3100 0.40211 2.4869 2.3291 0.9366<br />

4 1.4641 0.6830 0.21547 4.6410 0.31547 3.1699 4.3781 1.3812<br />

5 1.6105 0.6209 0.16380 6.1051 0.26380 3.7908 6.8618 1.8101<br />

6 1.7716 0.5645 0.12961 7.7156 0.22961 4.3553 9.6842 2.2236<br />

7 1.9487 0.5132 0.10541 9.4872 0.20541 4.8684 12.7631 2.6216<br />

8 2.1436 0.4665 0.08744 11.4359 0.18744 5.3349 16.0287 3.0045<br />

9 2.3579 0.4241 0.07364 13.5795 0.17364 5.7590 19.4215 3.3724<br />

10 2.5937 0.3855 0.06275 15.9374 0.16275 6.1446 22.8913 3.7255<br />

11 2.8531 0.3505 0.05396 18.5312 0.15396 6.4951 26.3963 4.0641<br />

12 3.1384 0.3186 0.04676 21.3843 0.14676 6.8137 29.9012 4.3884<br />

13 3.4523 0.2897 0.04078 24.5227 0.14078 7.1034 33.3772 4.6988<br />

14 3.7975 0.2633 0.03575 27.9750 0.13575 7.3667 36.8005 4.9955<br />

15 4.1772 0.2394 0.03147 31.7725 0.13147 7.6061 40.1520 5.2789<br />

16 4.5950 0.2176 0.02782 35.9497 0.12782 7.8237 43.4164 5.5493<br />

17 5.0545 0.1978 0.02466 40.5447 0.12466 8.0216 46.5819 5.8071<br />

18 5.5599 0.1799 0.02193 45.5992 0.12193 8.2014 49.6395 6.0526<br />

19 6.1159 0.1635 0.01955 51.1591 0.11955 8.3649 52.5827 6.2861<br />

20 6.7275 0.1486 0.01746 57.2750 0.11746 8.5136 55.4069 6.5081<br />

21 7.4002 0.1351 0.01562 64.0025 0.11562 8.6487 58.1095 6.7189<br />

22 8.1403 0.1228 0.01401 71.4027 0.11401 8.7715 60.6893 6.9189<br />

23 8.9543 0.1117 0.01257 79.5430 0.11257 8.8832 63.1462 7.1085<br />

24 9.8497 0.1015 0.01130 88.4973 0.11130 8.9847 65.4813 7.2881<br />

25 10.8347 0.0923 0.01017 98.3471 0.11017 9.0770 67.6964 7.4580<br />

26 11.9182 0.0839 0.00916 109.1818 0.10916 9.1609 69.7940 7.6186<br />

27 13.1100 0.0763 0.00826 121.0999 0.10826 9.2372 71.7773 7.7704<br />

28 14.4210 0.0693 0.00745 134.2099 0.10745 9.3066 73.6495 7.9137<br />

29 15.8631 0.0630 0.00673 148.6309 0.10673 9.3696 75.4146 8.0489<br />

30 17.4494 0.0573 0.00608 164.4940 0.10608 9.4269 77.0766 8.1762<br />

31 19.1943 0.0521 0.00550 181.9434 0.10550 9.4790 78.6395 8.2962<br />

32 21.1138 0.0474 0.00497 201.1378 0.10497 9.5264 80.1078 8.4091<br />

33 23.2252 0.0431 0.00450 222.2515 0.10450 9.5694 81.4856 8.5152<br />

34 25.5477 0.0391 0.00407 245.4767 0.10407 9.6086 82.7773 8.6149<br />

35 28.1024 0.0356 0.00369 271.0244 0.10369 9.6442 83.9872 8.7086<br />

40 45.2593 0.0221 0.00226 442.5926 0.10226 9.7791 88.9525 9.0962<br />

45 72.8905 0.0137 0.00139 718.9048 0.10139 9.8628 92.4544 9.3740<br />

50 117.3909 0.0085 0.00086 1163.91 0.10086 9.9148 94.8889 9.5704<br />

55 189.0591 0.0053 0.00053 1880.59 0.10053 9.9471 96.5619 9.7075<br />

60 304.4816 0.0033 0.00033 3034.82 0.10033 9.9672 97.7010 9.8023<br />

65 490.3707 0.0020 0.00020 4893.71 0.10020 9.9796 98.4705 9.8672<br />

70 789.7470 0.0013 0.00013 7887.47 0.10013 9.9873 98.9870 9.9113<br />

75 1271.90 0.0008 0.00008 12709 0.10008 9.9921 99.3317 9.9410<br />

80 2048.40 0.0005 0.00005 20474 0.10005 9.9951 99.5606 9.9609<br />

85 3298.97 0.0003 0.00003 32980 0.10003 9.9970 99.7120 9.9742<br />

90 5313.02 0.0002 0.00002 53120 0.10002 9.9981 99.8118 9.9831<br />

95 8556.68 0.0001 0.00001 85557 0.10001 9.9988 99.8773 9.9889<br />

96 9412.34 0.0001 0.00001 94113 0.10001 9.9989 99.8874 9.9898<br />

98 11389 0.0001 0.00001 0.10001 9.9991 99.9052 9.9914<br />

100 13781 0.0001 0.00001 0.10001 9.9993 99.9202 9.9927


596 Compound Interest Factor Tables<br />

11% TABLE 16 Discrete Cash Flow: Compound Interest Factors 11%<br />

Single Payments Uniform Series Payments Arithmetic Gradients<br />

Compound Present Sinking Compound Capital Present Gradient Gradient<br />

Amount Worth Fund Amount Recovery Worth Present Worth Uniform Series<br />

n FP PF AF FA AP PA PG AG<br />

1 1.1100 0.9009 1.00000 1.0000 1.11000 0.9009<br />

2 1.2321 0.8116 0.47393 2.1100 0.58393 1.7125 0.8116 0.4739<br />

3 1.3676 0.7312 0.29921 3.3421 0.40921 2.4437 2.2740 0.9306<br />

4 1.5181 0.6587 0.21233 4.7097 0.32233 3.1024 4.2502 1.3700<br />

5 1.6851 0.5935 0.16057 6.2278 0.27057 3.6959 6.6240 1.7923<br />

6 1.8704 0.5346 0.12638 7.9129 0.23638 4.2305 9.2972 2.1976<br />

7 2.0762 0.4817 0.10222 9.7833 0.21222 4.7122 12.1872 2.5863<br />

8 2.3045 0.4339 0.08432 11.8594 0.19432 5.1461 15.2246 2.9585<br />

9 2.5580 0.3909 0.07060 14.1640 0.18060 5.5370 18.3520 3.3144<br />

10 2.8394 0.3522 0.05980 16.7220 0.16980 5.8892 21.5217 3.6544<br />

11 3.1518 0.3173 0.05112 19.5614 0.16112 6.2065 24.6945 3.9788<br />

12 3.4985 0.2858 0.04403 22.7132 0.15403 6.4924 27.8388 4.2879<br />

13 3.8833 0.2575 0.03815 26.2116 0.14815 6.7499 30.9290 4.5822<br />

14 4.3104 0.2320 0.03323 30.0949 0.14323 6.9819 33.9449 4.8619<br />

15 4.7846 0.2090 0.02907 34.4054 0.13907 7.1909 36.8709 5.1275<br />

16 5.3109 0.1883 0.02552 39.1899 0.13552 7.3792 39.6953 5.3794<br />

17 5.8951 0.1696 0.02247 44.5008 0.13247 7.5488 42.4095 5.6180<br />

18 6.5436 0.1528 0.01984 50.3959 0.12984 7.7016 45.0074 5.8439<br />

19 7.2633 0.1377 0.01756 56.9395 0.12756 7.8393 47.4856 6.0574<br />

20 8.0623 0.1240 0.01558 64.2028 0.12558 7.9633 49.8423 6.2590<br />

21 8.9492 0.1117 0.01384 72.2651 0.12384 8.0751 52.0771 6.4491<br />

22 9.9336 0.1007 0.01231 81.2143 0.12231 8.1757 54.1912 6.6283<br />

23 11.0263 0.0907 0.01097 91.1479 0.12097 8.2664 56.1864 6.7969<br />

24 12.2392 0.0817 0.00979 102.1742 0.11979 8.3481 58.0656 6.9555<br />

25 13.5855 0.0736 0.00874 114.4133 0.11874 8.4217 59.8322 7.1045<br />

26 15.0799 0.0663 0.00781 127.9988 0.11781 8.4881 61.4900 7.2443<br />

27 16.7386 0.0597 0.00699 143.0786 0.11699 8.5478 63.0433 7.3754<br />

28 18.5799 0.0538 0.00626 159.8173 0.11626 8.6016 64.4965 7.4982<br />

29 20.6237 0.0485 0.00561 178.3972 0.11561 8.6501 65.8542 7.6131<br />

30 22.8923 0.0437 0.00502 199.0209 0.11502 8.6938 67.1210 7.7206<br />

31 25.4104 0.0394 0.00451 221.9132 0.11451 8.7331 68.3016 7.8210<br />

32 28.2056 0.0355 0.00404 247.3236 0.11404 8.7686 69.4007 7.9147<br />

33 31.3082 0.0319 0.00363 275.5292 0.11363 8.8005 70.4228 8.0021<br />

34 34.7521 0.0288 0.00326 306.8374 0.11326 8.8293 71.3724 8.0836<br />

35 38.5749 0.0259 0.00293 341.5896 0.11293 8.8552 72.2538 8.1594<br />

40 65.0009 0.0154 0.00172 581.8261 0.11172 8.9511 75.7789 8.4659<br />

45 109.5302 0.0091 0.00101 986.6386 0.11101 9.0079 78.1551 8.6763<br />

50 184.5648 0.0054 0.00060 1668.77 0.11060 9.0417 79.7341 8.8185<br />

55 311.0025 0.0032 0.00035 2818.20 0.11035 9.0617 80.7712 8.9135<br />

60 524.0572 0.0019 0.00021 4755.07 0.11021 9.0736 81.4461 8.9762<br />

65 883.0669 0.0011 0.00012 8018.79 0.11012 9.0806 81.8819 9.0172<br />

70 1488.02 0.0007 0.00007 13518 0.11007 9.0848 82.1614 9.0438<br />

75 2507.40 0.0004 0.00004 22785 0.11004 9.0873 82.3397 9.0610<br />

80 4225.11 0.0002 0.00003 38401 0.11003 9.0888 82.4529 9.0720<br />

85 7119.56 0.0001 0.00002 64714 0.11002 9.0896 82.5245 9.0790


Compound Interest Factor Tables 597<br />

12% TABLE 17 Discrete Cash Flow: Compound Interest Factors 12%<br />

Single Payments Uniform Series Payments Arithmetic Gradients<br />

Compound Present Sinking Compound Capital Present Gradient Gradient<br />

Amount Worth Fund Amount Recovery Worth Present Worth Uniform Series<br />

n FP PF AF FA AP PA PG AG<br />

1 1.1200 0.8929 1.00000 1.0000 1.12000 0.8929<br />

2 1.2544 0.7972 0.47170 2.1200 0.59170 1.6901 0.7972 0.4717<br />

3 1.4049 0.7118 0.29635 3.3744 0.41635 2.4018 2.2208 0.9246<br />

4 1.5735 0.6355 0.20923 4.7793 0.32923 3.0373 4.1273 1.3589<br />

5 1.7623 0.5674 0.15741 6.3528 0.27741 3.6048 6.3970 1.7746<br />

6 1.9738 0.5066 0.12323 8.1152 0.24323 4.1114 8.9302 2.1720<br />

7 2.2107 0.4523 0.09912 10.0890 0.21912 4.5638 11.6443 2.5512<br />

8 2.4760 0.4039 0.08130 12.2997 0.20130 4.9676 14.4714 2.9131<br />

9 2.7731 0.3606 0.06768 14.7757 0.18768 5.3282 17.3563 3.2574<br />

10 3.1058 0.3220 0.05698 17.5487 0.17698 5.6502 20.2541 3.5847<br />

11 3.4785 0.2875 0.04842 20.6546 0.16842 5.9377 23.1288 3.8953<br />

12 3.8960 0.2567 0.04144 24.1331 0.16144 6.1944 25.9523 4.1897<br />

13 4.3635 0.2292 0.03568 28.0291 0.15568 6.4235 28.7024 4.4683<br />

14 4.8871 0.2046 0.03087 32.3926 0.15087 6.6282 31.3624 4.7317<br />

15 5.4736 0.1827 0.02682 37.2797 0.14682 6.8109 33.9202 4.9803<br />

16 6.1304 0.1631 0.02339 42.7533 0.14339 6.9740 36.3670 5.2147<br />

17 6.8660 0.1456 0.02046 48.8837 0.14046 7.1196 38.6973 5.4353<br />

18 7.6900 0.1300 0.01794 55.7497 0.13794 7.2497 40.9080 5.6427<br />

19 8.6128 0.1161 0.01576 63.4397 0.13576 7.3658 42.9979 5.8375<br />

20 9.6463 0.1037 0.01388 72.0524 0.13388 7.4694 44.9676 6.0202<br />

21 10.8038 0.0926 0.01224 81.6987 0.13224 7.5620 46.8188 6.1913<br />

22 12.1003 0.0826 0.01081 92.5026 0.13081 7.6446 48.5543 6.3514<br />

23 13.5523 0.0738 0.00956 104.6029 0.12956 7.7184 50.1776 6.5010<br />

24 15.1786 0.0659 0.00846 118.1552 0.12846 7.7843 51.6929 6.6406<br />

25 17.0001 0.0588 0.00750 133.3339 0.12750 7.8431 53.1046 6.7708<br />

26 19.0401 0.0525 0.00665 150.3339 0.12665 7.8957 54.4177 6.8921<br />

27 21.3249 0.0469 0.00590 169.3740 0.12590 7.9426 55.6369 7.0049<br />

28 23.8839 0.0419 0.00524 190.6989 0.12524 7.9844 56.7674 7.1098<br />

29 26.7499 0.0374 0.00466 214.5828 0.12466 8.0218 57.8141 7.2071<br />

30 29.9599 0.0334 0.00414 241.3327 0.12414 8.0552 58.7821 7.2974<br />

31 33.5551 0.0298 0.00369 271.2926 0.12369 8.0850 59.6761 7.3811<br />

32 37.5817 0.0266 0.00328 304.8477 0.12328 8.1116 60.5010 7.4586<br />

33 42.0915 0.0238 0.00292 342.4294 0.12292 8.1354 61.2612 7.5302<br />

34 47.1425 0.0212 0.00260 384.5210 0.12260 8.1566 61.9612 7.5965<br />

35 52.7996 0.0189 0.00232 431.6635 0.12232 8.1755 62.6052 7.6577<br />

40 93.0510 0.0107 0.00130 767.0914 0.12130 8.2438 65.1159 7.8988<br />

45 163.9876 0.0061 0.0074 1358.23 0.12074 8.2825 66.7342 8.0572<br />

50 289.0022 0.0035 0.00042 2400.02 0.12042 8.3045 67.7624 8.1597<br />

55 509.3206 0.0020 0.00024 4236.01 0.12024 8.3170 68.4082 8.2251<br />

60 897.5969 0.0011 0.00013 7471.64 0.12013 8.3240 68.8100 8.2664<br />

65 1581.87 0.0006 0.00008 13174 0.12008 8.3281 69.0581 8.2922<br />

70 2787.80 0.0004 0.00004 23223 0.12004 8.3303 69.2103 8.3082<br />

75 4913.06 0.0002 0.00002 40934 0.12002 8.3316 69.3031 8.3181<br />

80 8658.48 0.0001 0.00001 72146 0.12001 8.3324 69.3594 8.3241<br />

85 15259 0.0001 0.00001 0.12001 8.3328 69.3935 8.3278


598 Compound Interest Factor Tables<br />

14% TABLE 18 Discrete Cash Flow: Compound Interest Factors 14%<br />

Single Payments Uniform Series Payments Arithmetic Gradients<br />

Compound Present Sinking Compound Capital Present Gradient Gradient<br />

Amount Worth Fund Amount Recovery Worth Present Worth Uniform Series<br />

n FP PF AF FA AP PA PG AG<br />

1 1.1400 0.8772 1.00000 1.0000 1.14000 0.8772<br />

2 1.2996 0.7695 0.46729 2.1400 0.60729 1.6467 0.7695 0.4673<br />

3 1.4815 0.6750 0.29073 3.4396 0.43073 2.3216 2.1194 0.9129<br />

4 1.6890 0.5921 0.20320 4.9211 0.34320 2.9137 3.8957 1.3370<br />

5 1.9254 0.5194 0.15128 6.6101 0.29128 3.4331 5.9731 1.7399<br />

6 2.1950 0.4556 0.11716 8.5355 0.25716 3.8887 8.2511 2.1218<br />

7 2.5023 0.3996 0.09319 10.7305 0.23319 4.2883 10.6489 2.4832<br />

8 2.8526 0.3506 0.07557 13.2328 0.21557 4.6389 13.1028 2.8246<br />

9 3.2519 0.3075 0.06217 16.0853 0.20217 4.9464 15.5629 3.1463<br />

10 3.7072 0.2697 0.05171 19.3373 0.19171 5.2161 17.9906 3.4490<br />

11 4.2262 0.2366 0.04339 23.0445 0.18339 5.4527 20.3567 3.7333<br />

12 4.8179 0.2076 0.03667 27.2707 0.17667 5.6603 22.6399 3.9998<br />

13 5.4924 0.1821 0.03116 32.0887 0.17116 5.8424 24.8247 4.2491<br />

14 6.2613 0.1597 0.02661 37.5811 0.16661 6.0021 26.9009 4.4819<br />

15 7.1379 0.1401 0.02281 43.8424 0.16281 6.1422 28.8623 4.6990<br />

16 8.1372 0.1229 0.01962 50.9804 0.15962 6.2651 30.7057 4.9011<br />

17 9.2765 0.1078 0.01692 59.1176 0.15692 6.3729 32.4305 5.0888<br />

18 10.5752 0.0946 0.01462 68.3941 0.15462 6.4674 34.0380 5.2630<br />

19 12.0557 0.0829 0.01266 78.9692 0.15266 6.5504 35.5311 5.4243<br />

20 13.7435 0.0728 0.01099 91.0249 0.15099 6.6231 36.9135 5.5734<br />

21 15.6676 0.0638 0.00954 104.7684 0.14954 6.6870 38.1901 5.7111<br />

22 17.8610 0.0560 0.00830 120.4360 0.14830 6.7429 39.3658 5.8381<br />

23 20.3616 0.0491 0.00723 138.2970 0.14723 6.7921 40.4463 5.9549<br />

24 23.2122 0.0431 0.00630 158.6586 0.14630 6.8351 41.4371 6.0624<br />

25 26.4619 0.0378 0.00550 181.8708 0.14550 6.8729 42.3441 6.1610<br />

26 30.1666 0.0331 0.00480 208.3327 0.14480 6.9061 43.1728 6.2514<br />

27 34.3899 0.0291 0.00419 238.4993 0.14419 6.9352 43.9289 6.3342<br />

28 39.2045 0.0255 0.00366 272.8892 0.14366 6.9607 44.6176 6.4100<br />

29 44.6931 0.0224 0.00320 312.0937 0.14320 6.9830 45.2441 6.4791<br />

30 50.9502 0.0196 0.00280 356.7868 0.14280 7.0027 45.8132 6.5423<br />

31 58.0832 0.0172 0.00245 407.7370 0.14245 7.0199 46.3297 6.5998<br />

32 66.2148 0.0151 0.00215 465.8202 0.14215 7.0350 46.7979 6.6522<br />

33 75.4849 0.0132 0.00188 532.0350 0.14188 7.0482 47.2218 6.6998<br />

34 86.0528 0.0116 0.00165 607.5199 0.14165 7.0599 47.6053 6.7431<br />

35 98.1002 0.0102 0.00144 693.5727 0.14144 7.0700 47.9519 6.7824<br />

40 188.8835 0.0053 0.00075 1342.03 0.14075 7.1050 49.2376 6.9300<br />

45 363.6791 0.0027 0.00039 2590.56 0.14039 7.1232 49.9963 7.0188<br />

50 700.2330 0.0014 0.00020 4994.52 0.14020 7.1327 50.4375 7.0714<br />

55 1348.24 0.0007 0.00010 9623.13 0.14010 7.1376 50.6912 7.1020<br />

60 2595.92 0.0004 0.00005 18535 0.14005 7.1401 50.8357 7.1197<br />

65 4998.22 0.0002 0.00003 35694 0.14003 7.1414 50.9173 7.1298<br />

70 9623.64 0.0001 0.00001 68733 0.14001 7.1421 50.9632 7.1356<br />

75 18530 0.0001 0.00001 0.14001 7.1425 50.9887 7.1388<br />

80 35677 0.14000 7.1427 51.0030 7.1406<br />

85 68693 0.14000 7.1428 51.0108 7.1416


Compound Interest Factor Tables 599<br />

15% TABLE 19 Discrete Cash Flow: Compound Interest Factors 15%<br />

Single Payments Uniform Series Payments Arithmetic Gradients<br />

Compound Present Sinking Compound Capital Present Gradient Gradient<br />

Amount Worth Fund Amount Recovery Worth Present Worth Uniform Series<br />

n FP PF AF FA AP PA PG AG<br />

1 1.1500 0.8696 1.00000 1.0000 1.15000 0.8696<br />

2 1.3225 0.7561 0.46512 2.1500 0.61512 1.6257 0.7561 0.4651<br />

3 1.5209 0.6575 0.28798 3.4725 0.43798 2.2832 2.0712 0.9071<br />

4 1.7490 0.5718 0.20027 4.9934 0.35027 2.8550 3.7864 1.3263<br />

5 2.0114 0.4972 0.14832 6.7424 0.29832 3.3522 5.7751 1.7228<br />

6 2.3131 0.4323 0.11424 8.7537 0.26424 3.7845 7.9368 2.0972<br />

7 2.6600 0.3759 0.09036 11.0668 0.24036 4.1604 10.1924 2.4498<br />

8 3.0590 0.3269 0.07285 13.7268 0.22285 4.4873 12.4807 2.7813<br />

9 3.5179 0.2843 0.05957 16.7858 0.20957 4.7716 14.7548 3.0922<br />

10 4.0456 0.2472 0.04925 20.3037 0.19925 5.0188 16.9795 3.3832<br />

11 4.6524 0.2149 0.04107 24.3493 0.19107 5.2337 19.1289 3.6549<br />

12 5.3503 0.1869 0.03448 29.0017 0.18448 5.4206 21.1849 3.9082<br />

13 6.1528 0.1625 0.02911 34.3519 0.17911 5.5831 23.1352 4.1438<br />

14 7.0757 0.1413 0.02469 40.5047 0.17469 5.7245 24.9725 4.3624<br />

15 8.1371 0.1229 0.02102 47.5804 0.17102 5.8474 26.6930 4.5650<br />

16 9.3576 0.1069 0.01795 55.7175 0.16795 5.9542 28.2960 4.7522<br />

17 10.7613 0.0929 0.01537 65.0751 0.16537 6.0472 29.7828 4.9251<br />

18 12.3755 0.0808 0.01319 75.8364 0.16319 6.1280 31.1565 5.0843<br />

19 14.2318 0.0703 0.01134 88.2118 0.16134 6.1982 32.4213 5.2307<br />

20 16.3665 0.0611 0.00976 102.4436 0.15976 6.2593 33.5822 5.3651<br />

21 18.8215 0.0531 0.00842 118.8101 0.15842 6.3125 34.6448 5.4883<br />

22 21.6447 0.0462 0.00727 137.6316 0.15727 6.3587 35.6150 5.6010<br />

23 24.8915 0.0402 0.00628 159.2764 0.15628 6.3988 36.4988 5.7040<br />

24 28.6252 0.0349 0.00543 184.1678 0.15543 6.4338 37.3023 5.7979<br />

25 32.9190 0.0304 0.00470 212.7930 0.15470 6.4641 38.0314 5.8834<br />

26 37.8568 0.0264 0.00407 245.7120 0.15407 6.4906 38.6918 5.9612<br />

27 43.5353 0.0230 0.00353 283.5688 0.15353 6.5135 39.2890 6.0319<br />

28 50.0656 0.0200 0.00306 327.1041 0.15306 6.5335 39.8283 6.0960<br />

29 57.5755 0.0174 0.00265 377.1697 0.15265 6.5509 40.3146 6.1541<br />

30 66.2118 0.0151 0.00230 434.7451 0.15230 6.5660 40.7526 6.2066<br />

31 76.1435 0.0131 0.00200 500.9569 0.15200 6.5791 41.1466 6.2541<br />

32 87.5651 0.0114 0.00173 577.1005 0.15173 6.5905 41.5006 6.2970<br />

33 100.6998 0.0099 0.00150 664.6655 0.15150 6.6005 41.8184 6.3357<br />

34 115.8048 0.0086 0.00131 765.3654 0.15131 6.6091 42.1033 6.3705<br />

35 133.1755 0.0075 0.00113 881.1702 0.15113 6.6166 42.3586 6.4019<br />

40 267.8635 0.0037 0.00056 1779.09 0.15056 6.6418 43.2830 6.5168<br />

45 538.7693 0.0019 0.00028 3585.13 0.15028 6.6543 43.8051 6.5830<br />

50 1083.66 0.0009 0.00014 7217.72 0.15014 6.6605 44.0958 6.6205<br />

55 2179.62 0.0005 0.00007 14524 0.15007 6.6636 44.2558 6.6414<br />

60 4384.00 0.0002 0.00003 29220 0.15003 6.6651 44.3431 6.6530<br />

65 8817.79 0.0001 0.00002 58779 0.15002 6.6659 44.3903 6.6593<br />

70 17736 0.0001 0.00001 0.15001 6.6663 44.4156 6.6627<br />

75 35673 0.15000 6.6665 44.4292 6.6646<br />

80 71751 0.15000 6.6666 44.4364 6.6656<br />

85 0.15000 6.6666 44.4402 6.6661


600 Compound Interest Factor Tables<br />

16% TABLE 20 Discrete Cash Flow: Compound Interest Factors 16%<br />

Single Payments Uniform Series Payments Arithmetic Gradients<br />

Compound Present Sinking Compound Capital Present Gradient Gradient<br />

Amount Worth Fund Amount Recovery Worth Present Worth Uniform Series<br />

n FP PF AF FA AP PA PG AG<br />

1 1.1600 0.8621 1.00000 1.0000 1.16000 0.8621<br />

2 1.3456 0.7432 0.46296 2.1600 0.62296 1.6052 0.7432 0.4630<br />

3 1.5609 0.6407 0.28526 3.5056 0.44526 2.2459 2.0245 0.9014<br />

4 1.8106 0.5523 0.19738 5.0665 0.35738 2.7982 3.6814 1.3156<br />

5 2.1003 0.4761 0.14541 6.8771 0.30541 3.2743 5.5858 1.7060<br />

6 2.4364 0.4104 0.11139 8.9775 0.27139 3.6847 7.6380 2.0729<br />

7 2.8262 0.3538 0.08761 11.4139 0.24761 4.0386 9.7610 2.4169<br />

8 3.2784 0.3050 0.07022 14.2401 0.23022 4.3436 11.8962 2.7388<br />

9 3.8030 0.2630 0.05708 17.5185 0.21708 4.6065 13.9998 3.0391<br />

10 4.4114 0.2267 0.04690 21.3215 0.20690 4.8332 16.0399 3.3187<br />

11 5.1173 0.1954 0.03886 25.7329 0.19886 5.0286 17.9941 3.5783<br />

12 5.9360 0.1685 0.03241 30.8502 0.19241 5.1971 19.8472 3.8189<br />

13 6.8858 0.1452 0.02718 36.7862 0.18718 5.3423 21.5899 4.0413<br />

14 7.9875 0.1252 0.02290 43.6720 0.18290 5.4675 23.2175 4.2464<br />

15 9.2655 0.1079 0.01936 51.6595 0.17936 5.5755 24.7284 4.4352<br />

16 10.7480 0.0930 0.01641 60.9250 0.17641 5.6685 26.1241 4.6086<br />

17 12.4677 0.0802 0.01395 71.6730 0.17395 5.7487 27.4074 4.7676<br />

18 14.4625 0.0691 0.01188 84.1407 0.17188 5.8178 28.5828 4.9130<br />

19 16.7765 0.0596 0.01014 98.6032 0.17014 5.8775 29.6557 5.0457<br />

20 19.4608 0.0514 0.00867 115.3797 0.16867 5.9288 30.6321 5.1666<br />

22 26.1864 0.0382 0.00635 157.4150 0.16635 6.0113 32.3200 5.3765<br />

24 35.2364 0.0284 0.00467 213.9776 0.16467 6.0726 33.6970 5.5490<br />

26 47.4141 0.0211 0.00345 290.0883 0.16345 6.1182 34.8114 5.6898<br />

28 63.8004 0.0157 0.00255 392.5028 0.16255 6.1520 35.7073 5.8041<br />

30 85.8499 0.0116 0.00189 530.3117 0.16189 6.1772 36.4234 5.8964<br />

32 115.5196 0.0087 0.00140 715.7475 0.16140 6.1959 36.9930 5.9706<br />

34 155.4432 0.0064 0.00104 965.2698 0.16104 6.2098 37.4441 6.0299<br />

35 180.3141 0.0055 0.00089 1120.71 0.16089 6.2153 37.6327 6.0548<br />

36 209.1643 0.0048 0.00077 1301.03 0.16077 6.2201 37.8000 6.0771<br />

38 281.4515 0.0036 0.00057 1752.82 0.16057 6.2278 38.0799 6.1145<br />

40 378.7212 0.0026 0.00042 2360.76 0.16042 6.2335 38.2992 6.1441<br />

45 795.4438 0.0013 0.00020 4965.27 0.16020 6.2421 38.6598 6.1934<br />

50 1670.70 0.0006 0.00010 10436 0.16010 6.2463 38.8521 6.2201<br />

55 3509.05 0.0003 0.00005 21925 0.16005 6.2482 38.9534 6.2343<br />

60 7370.20 0.0001 0.00002 46058 0.16002 6.2492 39.0063 6.2419


Compound Interest Factor Tables 601<br />

18% TABLE 21 Discrete Cash Flow: Compound Interest Factors 18%<br />

Single Payments Uniform Series Payments Arithmetic Gradients<br />

Compound Present Sinking Compound Capital Present Gradient Gradient<br />

Amount Worth Fund Amount Recovery Worth Present Worth Uniform Series<br />

n FP PF AF FA AP PA PG AG<br />

1 1.1800 0.8475 1.00000 1.0000 1.18000 0.8475<br />

2 1.3924 0.7182 0.45872 2.1800 0.63872 1.5656 0.7182 0.4587<br />

3 1.6430 0.6086 0.27992 3.5724 0.45992 2.1743 1.9354 0.8902<br />

4 1.9388 0.5158 0.19174 5.2154 0.37174 2.6901 3.4828 1.2947<br />

5 2.2878 0.4371 0.13978 7.1542 0.31978 3.1272 5.2312 1.6728<br />

6 2.6996 0.3704 0.10591 9.4420 0.28591 3.4976 7.0834 2.0252<br />

7 3.1855 0.3139 0.08236 12.1415 0.26236 3.8115 8.9670 2.3526<br />

8 3.7589 0.2660 0.06524 15.3270 0.24524 4.0776 10.8292 2.6558<br />

9 4.4355 0.2255 0.05239 19.0859 0.23239 4.3030 12.6329 2.9358<br />

10 5.2338 0.1911 0.04251 23.5213 0.22251 4.4941 14.3525 3.1936<br />

11 6.1759 0.1619 0.03478 28.7551 0.21478 4.6560 15.9716 3.4303<br />

12 7.2876 0.1372 0.02863 34.9311 0.20863 4.7932 17.4811 3.6470<br />

13 8.5994 0.1163 0.02369 42.2187 0.20369 4.9095 18.8765 3.8449<br />

14 10.1472 0.0985 0.01968 50.8180 0.19968 5.0081 20.1576 4.0250<br />

15 11.9737 0.0835 0.01640 60.9653 0.19640 5.0916 21.3269 4.1887<br />

16 14.1290 0.0708 0.01371 72.9390 0.19371 5.1624 22.3885 4.3369<br />

17 16.6722 0.0600 0.01149 87.0680 0.19149 5.2223 23.3482 4.4708<br />

18 19.6733 0.0508 0.00964 103.7403 0.18964 5.2732 24.2123 4.5916<br />

19 23.2144 0.0431 0.00810 123.4135 0.18810 5.3162 24.9877 4.7003<br />

20 27.3930 0.0365 0.00682 146.6280 0.18682 5.3527 25.6813 4.7978<br />

22 38.1421 0.0262 0.00485 206.3448 0.18485 5.4099 26.8506 4.9632<br />

24 53.1090 0.0188 0.00345 289.4945 0.18345 5.4509 27.7725 5.0950<br />

26 73.9490 0.0135 0.00247 405.2721 0.18247 5.4804 28.4935 5.1991<br />

28 102.9666 0.0097 0.00177 566.4809 0.18177 5.5016 29.0537 5.2810<br />

30 143.3706 0.0070 0.00126 790.9480 0.18126 5.5168 29.4864 5.3448<br />

32 199.6293 0.0050 0.00091 1103.50 0.18091 5.5277 29.8191 5.3945<br />

34 277.9638 0.0036 0.00065 1538.69 0.18065 5.5356 30.0736 5.4328<br />

35 327.9973 0.0030 0.00055 1816.65 0.18055 5.5386 30.1773 5.4485<br />

36 387.0368 0.0026 0.00047 2144.65 0.18047 5.5412 30.2677 5.4623<br />

38 538.9100 0.0019 0.00033 2988.39 0.18033 5.5452 30.4152 5.4849<br />

40 750.3783 0.0013 0.00024 4163.21 0.18024 5.5482 30.5269 5.5022<br />

45 1716.68 0.0006 0.00010 9531.58 0.18010 5.5523 30.7006 5.5293<br />

50 3927.36 0.0003 0.00005 21813 0.18005 5.5541 30.7856 5.5428<br />

55 8984.84 0.0001 0.00002 49910 0.18002 5.5549 30.8268 5.5494<br />

60 20555 114190 0.18001 5.5553 30.8465 5.5526


602 Compound Interest Factor Tables<br />

20% TABLE 22 Discrete Cash Flow: Compound Interest Factors 20%<br />

Single Payments Uniform Series Payments Arithmetic Gradients<br />

Compound Present Sinking Compound Capital Present Gradient Gradient<br />

Amount Worth Fund Amount Recovery Worth Present Worth Uniform Series<br />

n FP PF AF FA AP PA PG AG<br />

1 1.2000 0.8333 1.00000 1.0000 1.20000 0.8333<br />

2 1.4400 0.6944 0.45455 2.2000 0.65455 1.5278 0.6944 0.4545<br />

3 1.7280 0.5787 0.27473 3.6400 0.47473 2.1065 1.8519 0.8791<br />

4 2.0736 0.4823 0.18629 5.3680 0.38629 2.5887 3.2986 1.2742<br />

5 2.4883 0.4019 0.13438 7.4416 0.33438 2.9906 4.9061 1.6405<br />

6 2.9860 0.3349 0.10071 9.9299 0.30071 3.3255 6.5806 1.9788<br />

7 3.5832 0.2791 0.07742 12.9159 0.27742 3.6046 8.2551 2.2902<br />

8 4.2998 0.2326 0.06061 16.4991 0.26061 3.8372 9.8831 2.5756<br />

9 5.1598 0.1938 0.04808 20.7989 0.24808 4.0310 11.4335 2.8364<br />

10 6.1917 0.1615 0.03852 25.9587 0.23852 4.1925 12.8871 3.0739<br />

11 7.4301 0.1346 0.03110 32.1504 0.23110 4.3271 14.2330 3.2893<br />

12 8.9161 0.1122 0.02526 39.5805 0.22526 4.4392 15.4667 3.4841<br />

13 10.6993 0.0935 0.02062 48.4966 0.22062 4.5327 16.5883 3.6597<br />

14 12.8392 0.0779 0.01689 59.1959 0.21689 4.6106 17.6008 3.8175<br />

15 15.4070 0.0649 0.01388 72.0351 0.21388 4.6755 18.5095 3.9588<br />

16 18.4884 0.0541 0.01144 87.4421 0.21144 4.7296 19.3208 4.0851<br />

17 22.1861 0.0451 0.00944 105.9306 0.20944 4.7746 20.0419 4.1976<br />

18 26.6233 0.0376 0.00781 128.1167 0.20781 4.8122 20.6805 4.2975<br />

19 31.9480 0.0313 0.00646 154.7400 0.20646 4.8435 21.2439 4.3861<br />

20 38.3376 0.0261 0.00536 186.6880 0.20536 4.8696 21.7395 4.4643<br />

22 55.2061 0.0181 0.00369 271.0307 0.20369 4.9094 22.5546 4.5941<br />

24 79.4968 0.0126 0.00255 392.4842 0.20255 4.9371 23.1760 4.6943<br />

26 114.4755 0.0087 0.00176 567.3773 0.20176 4.9563 23.6460 4.7709<br />

28 164.8447 0.0061 0.00122 819.2233 0.<strong>2012</strong>2 4.9697 23.9991 4.8291<br />

30 237.3763 0.0042 0.00085 1181.88 0.20085 4.9789 24.2628 4.8731<br />

32 341.8219 0.0029 0.00059 1704.11 0.20059 4.9854 24.4588 4.9061<br />

34 492.2235 0.0020 0.00041 2456.12 0.20041 4.9898 24.6038 4.9308<br />

35 590.6682 0.0017 0.00034 2948.34 0.20034 4.9915 24.6614 4.9406<br />

36 708.8019 0.0014 0.00028 3539.01 0.20028 4.9929 24.7108 4.9491<br />

38 1020.67 0.0010 0.00020 5098.37 0.20020 4.9951 24.7894 4.9627<br />

40 1469.77 0.0007 0.00014 7343.86 0.20014 4.9966 24.8469 4.9728<br />

45 3657.26 0.0003 0.00005 18281 0.20005 4.9986 24.9316 4.9877<br />

50 9100.44 0.0001 0.00002 45497 0.20002 4.9995 24.9698 4.9945<br />

55 22645 0.00001 0.20001 4.9998 24.9868 4.9976


Compound Interest Factor Tables 603<br />

22% TABLE 23 Discrete Cash Flow: Compound Interest Factors 22%<br />

Single Payments Uniform Series Payments Arithmetic Gradients<br />

Compound Present Sinking Compound Capital Present Gradient Gradient<br />

Amount Worth Fund Amount Recovery Worth Present Worth Uniform Series<br />

n FP PF AF FA AP PA PG AG<br />

1 1.2200 0.8197 1.00000 1.0000 1.22000 0.8197<br />

2 1.4884 0.6719 0.45045 2.2200 0.67045 1.4915 0.6719 0.4505<br />

3 1.8158 0.5507 0.26966 3.7084 0.48966 2.0422 1.7733 0.8683<br />

4 2.2153 0.4514 0.18102 5.5242 0.40102 2.4936 3.1275 1.2542<br />

5 2.7027 0.3700 0.12921 7.7396 0.34921 2.8636 4.6075 1.6090<br />

6 3.2973 0.3033 0.09576 10.4423 0.31576 3.1669 6.1239 1.9337<br />

7 4.0227 0.2486 0.07278 13.7396 0.29278 3.4155 7.6154 2.2297<br />

8 4.9077 0.2038 0.05630 17.7623 0.27630 3.6193 9.0417 2.4982<br />

9 5.9874 0.1670 0.04411 22.6700 0.26411 3.7863 10.3779 2.7409<br />

10 7.3046 0.1369 0.03489 28.6574 0.25489 3.9232 11.6100 2.9593<br />

11 8.9117 0.1122 0.02781 35.9620 0.24781 4.0354 12.7321 3.1551<br />

12 10.8722 0.0920 0.02228 44.8737 0.24228 4.1274 13.7438 3.3299<br />

13 13.2641 0.0754 0.01794 55.7459 0.23794 4.2028 14.6485 3.4855<br />

14 16.1822 0.0618 0.01449 69.0100 0.23449 4.2646 15.4519 3.6233<br />

15 19.7423 0.0507 0.01174 85.1922 0.23174 4.3152 16.1610 3.7451<br />

16 24.0856 0.0415 0.00953 104.9345 0.22953 4.3567 16.7838 3.8524<br />

17 29.3844 0.0340 0.00775 129.0201 0.22775 4.3908 17.3283 3.9465<br />

18 35.8490 0.0279 0.00631 158.4045 0.22631 4.4187 17.8025 4.0289<br />

19 43.7358 0.0229 0.00515 194.2535 0.22515 4.4415 18.2141 4.1009<br />

20 53.3576 0.0187 0.00420 237.9893 0.22420 4.4603 18.5702 4.1635<br />

22 79.4175 0.0126 0.00281 356.4432 0.22281 4.4882 19.1418 4.2649<br />

24 118.2050 0.0085 0.00188 532.7501 0.22188 4.5070 19.5635 4.3407<br />

26 175.9364 0.0057 0.00126 795.1653 0.22126 4.5196 19.8720 4.3968<br />

28 261.8637 0.0038 0.00084 1185.74 0.22084 4.5281 20.0962 4.4381<br />

30 389.7579 0.0026 0.00057 1767.08 0.22057 4.5338 20.2583 4.4683<br />

32 580.1156 0.0017 0.00038 2632.34 0.22038 4.5376 20.3748 4.4902<br />

34 863.4441 0.0012 0.00026 3920.20 0.22026 4.5402 20.4582 4.5060<br />

35 1053.40 0.0009 0.00021 4783.64 0.22021 4.5411 20.4905 4.5122<br />

36 1285.15 0.0008 0.00017 5837.05 0.22017 4.5419 20.5178 4.5174<br />

38 1912.82 0.0005 0.00012 8690.08 0.2<strong>2012</strong> 4.5431 20.5601 4.5256<br />

40 2847.04 0.0004 0.00008 12937 0.22008 4.5439 20.5900 4.5314<br />

45 7694.71 0.0001 0.00003 34971 0.22003 4.5449 20.6319 4.5396<br />

50 20797 0.00001 94525 0.22001 4.5452 20.6492 4.5431<br />

55 56207 0.22000 4.5454 20.6563 4.5445


604 Compound Interest Factor Tables<br />

24% TABLE 24 Discrete Cash Flow: Compound Interest Factors 24%<br />

Single Payments Uniform Series Payments Arithmetic Gradients<br />

Compound Present Sinking Compound Capital Present Gradient Gradient<br />

Amount Worth Fund Amount Recovery Worth Present Worth Uniform Series<br />

n FP PF AF FA AP PA PG AG<br />

1 1.2400 0.8065 1.00000 1.0000 1.24000 0.8065<br />

2 1.5376 0.6504 0.44643 2.2400 0.68643 1.4568 0.6504 0.4464<br />

3 1.9066 0.5245 0.26472 3.7776 0.50472 1.9813 1.6993 0.8577<br />

4 2.3642 0.4230 0.17593 5.6842 0.41593 2.4043 2.9683 1.2346<br />

5 2.9316 0.3411 0.12425 8.0484 0.36425 2.7454 4.3327 1.5782<br />

6 3.6352 0.2751 0.09107 10.9801 0.33107 3.0205 5.7081 1.8898<br />

7 4.5077 0.2218 0.06842 14.6153 0.30842 3.2423 7.0392 2.1710<br />

8 5.5895 0.1789 0.05229 19.1229 0.29229 3.4212 8.2915 2.4236<br />

9 6.9310 0.1443 0.04047 24.7125 0.28047 3.5655 9.4458 2.6492<br />

10 8.5944 0.1164 0.03160 31.6434 0.27160 3.6819 10.4930 2.8499<br />

11 10.6571 0.0938 0.02485 40.2379 0.26485 3.7757 11.4313 3.0276<br />

12 13.2148 0.0757 0.01965 50.8950 0.25965 3.8514 12.2637 3.1843<br />

13 16.3863 0.0610 0.01560 64.1097 0.25560 3.9124 12.9960 3.3218<br />

14 20.3191 0.0492 0.01242 80.4961 0.25242 3.9616 13.6358 3.4420<br />

15 25.1956 0.0397 0.00992 100.8151 0.24992 4.0013 14.1915 3.5467<br />

16 31.2426 0.0320 0.00794 126.0108 0.24794 4.0333 14.6716 3.6376<br />

17 38.7408 0.0258 0.00636 157.2534 0.24636 4.0591 15.0846 3.7162<br />

18 48.0386 0.0208 0.00510 195.9942 0.24510 4.0799 15.4385 3.7840<br />

19 59.5679 0.0168 0.00410 244.0328 0.24410 4.0967 15.7406 3.8423<br />

20 73.8641 0.0135 0.00329 303.6006 0.24329 4.1103 15.9979 3.8922<br />

22 113.5735 0.0088 0.00213 469.0563 0.24213 4.1300 16.4011 3.9712<br />

24 174.6306 0.0057 0.00138 723.4610 0.24138 4.1428 16.6891 4.0284<br />

26 268.5121 0.0037 0.00090 1114.63 0.24090 4.1511 16.8930 4.0695<br />

28 412.8642 0.0024 0.00058 1716.10 0.24058 4.1566 17.0365 4.0987<br />

30 634.8199 0.0016 0.00038 2640.92 0.24038 4.1601 17.1369 4.1193<br />

32 976.0991 0.0010 0.00025 4062.91 0.24025 4.1624 17.2067 4.1338<br />

34 1500.85 0.0007 0.00016 6249.38 0.24016 4.1639 17.2552 4.1440<br />

35 1861.05 0.0005 0.00013 7750.23 0.24013 4.1664 17.2734 4.1479<br />

36 2307.71 0.0004 0.00010 9611.28 0.24010 4.1649 17.2886 4.1511<br />

38 3548.33 0.0003 0.00007 14781 0.24007 4.1655 17.3116 4.1560<br />

40 5455.91 0.0002 0.00004 22729 0.24004 4.1659 17.3274 4.1593<br />

45 15995 0.0001 0.00002 66640 0.24002 4.1664 17.3483 4.1639<br />

50 46890 0.00001 0.24001 4.1666 17.3563 4.1653<br />

55 0.24000 4.1666 17.3593 4.1663


Compound Interest Factor Tables 605<br />

25% TABLE 25 Discrete Cash Flow: Compound Interest Factors 25%<br />

Single Payments Uniform Series Payments Arithmetic Gradients<br />

Compound Present Sinking Compound Capital Present Gradient Gradient<br />

Amount Worth Fund Amount Recovery Worth Present Worth Uniform Series<br />

n FP PF AF FA AP PA PG AG<br />

1 1.2500 0.8000 1.00000 1.0000 1.25000 0.8000<br />

2 1.5625 0.6400 0.44444 2.2500 0.69444 1.4400 0.6400 0.4444<br />

3 1.9531 0.5120 0.26230 3.8125 0.51230 1.9520 1.6640 0.8525<br />

4 2.4414 0.4096 0.17344 5.7656 0.42344 2.3616 2.8928 1.2249<br />

5 3.0518 0.3277 0.12185 8.2070 0.37185 2.6893 4.2035 1.5631<br />

6 3.8147 0.2621 0.08882 11.2588 0.33882 2.9514 5.5142 1.8683<br />

7 4.7684 0.2097 0.06634 15.0735 0.31634 3.1611 6.7725 2.1424<br />

8 5.9605 0.1678 0.05040 19.8419 0.30040 3.3289 7.9469 2.3872<br />

9 7.4506 0.1342 0.03876 25.8023 0.28876 3.4631 9.0207 2.6048<br />

10 9.3132 0.1074 0.03007 33.2529 0.28007 3.5705 9.9870 2.7971<br />

11 11.6415 0.0859 0.02349 42.5661 0.27349 3.6564 10.8460 2.9663<br />

12 14.5519 0.0687 0.01845 54.2077 0.26845 3.7251 11.6020 3.1145<br />

13 18.1899 0.0550 0.01454 68.7596 0.26454 3.7801 12.2617 3.2437<br />

14 22.7374 0.0440 0.01150 86.9495 0.26150 3.8241 12.8334 3.3559<br />

15 28.4217 0.0352 0.00912 109.6868 0.25912 3.8593 13.3260 3.4530<br />

16 35.5271 0.0281 0.00724 138.1085 0.25724 3.8874 13.7482 3.5366<br />

17 44.4089 0.0225 0.00576 173.6357 0.25576 3.9099 14.1085 3.6084<br />

18 55.5112 0.0180 0.00459 218.0446 0.25459 3.9279 14.4147 3.6698<br />

19 69.3889 0.0144 0.00366 273.5558 0.25366 3.9424 14.6741 3.7222<br />

20 86.7362 0.0115 0.00292 342.9447 0.25292 3.9539 14.8932 3.7667<br />

22 135.5253 0.0074 0.00186 538.1011 0.25186 3.9705 15.2326 3.8365<br />

24 211.7582 0.0047 0.00119 843.0329 0.25119 3.9811 15.4711 3.8861<br />

26 330.8722 0.0030 0.00076 1319.49 0.25076 3.9879 15.6373 3.9212<br />

28 516.9879 0.0019 0.00048 2063.95 0.25048 3.9923 15.7524 3.9457<br />

30 807.7936 0.0012 0.00031 3227.17 0.25031 3.9950 15.8316 3.9628<br />

32 1262.18 0.0008 0.00020 5044.71 0.25020 3.9968 15.8859 3.9746<br />

34 1972.15 0.0005 0.00013 7884.61 0.25013 3.9980 15.9229 3.9828<br />

35 2465.19 0.0004 0.00010 9856.76 .025010 3.9984 15.9367 3.9858<br />

36 3081.49 0.0003 0.00008 12322 0.25008 3.9987 15.9481 3.9883<br />

38 4814.82 0.0002 0.00005 19255 0.25005 3.9992 15.9651 3.9921<br />

40 7523.16 0.0001 0.00003 30089 0.25003 3.9995 15.9766 3.9947<br />

45 22959 0.00001 91831 0.25001 3.9998 15.9915 3.9980<br />

50 70065 0.25000 3.9999 15.9969 3.9993<br />

55 0.25000 4.0000 15.9989 3.9997


606 Compound Interest Factor Tables<br />

30% TABLE 26 Discrete Cash Flow: Compound Interest Factors 30%<br />

Single Payments Uniform Series Payments Arithmetic Gradients<br />

Compound Present Sinking Compound Capital Present Gradient Gradient<br />

Amount Worth Fund Amount Recovery Worth Present Worth Uniform Series<br />

n FP PF AF FA AP PA PG AG<br />

1 1.3000 0.7692 1.00000 1.0000 1.30000 0.7692<br />

2 1.6900 0.5917 0.43478 2.3000 0.73478 1.3609 0.5917 0.4348<br />

3 2.1970 0.4552 0.25063 3.9900 0.55063 1.8161 1.5020 0.8271<br />

4 2.8561 0.3501 0.16163 6.1870 0.46163 2.1662 2.5524 1.1783<br />

5 3.7129 0.2693 0.11058 9.0431 0.41058 2.4356 3.6297 1.4903<br />

6 4.8268 0.2072 0.07839 12.7560 0.37839 2.6427 4.6656 1.7654<br />

7 6.2749 0.1594 0.05687 17.5828 0.35687 2.8021 5.6218 2.0063<br />

8 8.1573 0.1226 0.04192 23.8577 0.34192 2.9247 6.4800 2.2156<br />

9 10.6045 0.0943 0.03124 32.0150 0.33124 3.0190 7.2343 2.3963<br />

10 13.7858 0.0725 0.02346 42.6195 0.32346 3.0915 7.8872 2.5512<br />

11 17.9216 0.0558 0.01773 56.4053 0.31773 3.1473 8.4452 2.6833<br />

12 23.2981 0.0429 0.01345 74.3270 0.31345 3.1903 8.9173 2.7952<br />

13 30.2875 0.0330 0.01024 97.6250 0.31024 3.2233 9.3135 2.8895<br />

14 39.3738 0.0254 0.00782 127.9125 0.30782 3.2487 9.6437 2.9685<br />

15 51.1859 0.0195 0.00598 167.2863 0.30598 3.2682 9.9172 3.0344<br />

16 66.5417 0.0150 0.00458 218.4722 0.30458 3.2832 10.1426 3.0892<br />

17 86.5042 0.0116 0.00351 285.0139 0.30351 3.2948 10.3276 3.1345<br />

18 112.4554 0.0089 0.00269 371.5180 0.30269 3.3037 10.4788 3.1718<br />

19 146.1920 0.0068 0.00207 483.9734 0.30207 3.3105 10.6019 3.2025<br />

20 190.0496 0.0053 0.00159 630.1655 0.30159 3.3158 10.7019 3.2275<br />

22 321.1839 0.0031 0.00094 1067.28 0.30094 3.3230 10.8482 3.2646<br />

24 542.8008 0.0018 0.00055 1806.00 0.30055 3.3272 10.9433 3.2890<br />

25 705.6410 0.0014 0.00043 2348.80 0.30043 3.3286 10.9773 3.2979<br />

26 917.3333 0.0011 0.00033 3054.44 0.30033 3.3297 11.0045 3.3050<br />

28 1550.29 0.0006 0.00019 5164.31 0.30019 3.3312 11.0437 3.3153<br />

30 2620.00 0.0004 0.00011 8729.99 0.30011 3.3321 11.0687 3.3219<br />

32 4427.79 0.0002 0.00007 14756 0.30007 3.3326 11.0845 3.3261<br />

34 7482.97 0.0001 0.00004 24940 0.30004 3.3329 11.0945 3.3288<br />

35 9727.86 0.0001 0.00003 32423 0.30003 3.3330 11.0980 3.3297


Compound Interest Factor Tables 607<br />

35% TABLE 27 Discrete Cash Flow: Compound Interest Factors 35%<br />

Single Payments Uniform Series Payments Arithmetic Gradients<br />

Compound Present Sinking Compound Capital Present Gradient Gradient<br />

Amount Worth Fund Amount Recovery Worth Present Worth Uniform Series<br />

n FP PF AF FA AP PA PG AG<br />

1 1.3500 0.7407 1.00000 1.0000 1.35000 0.7407<br />

2 1.8225 0.5487 0.42553 2.3500 0.77553 1.2894 0.5487 0.4255<br />

3 2.4604 0.4064 0.23966 4.1725 0.58966 1.6959 1.3616 0.8029<br />

4 3.3215 0.3011 0.15076 6.6329 0.50076 1.9969 2.2648 1.1341<br />

5 4.4840 0.2230 0.10046 9.9544 0.45046 2.2200 3.1568 1.4220<br />

6 6.0534 0.1652 0.06926 14.4384 0.41926 2.3852 3.9828 1.6698<br />

7 8.1722 0.1224 0.04880 20.4919 0.39880 2.5075 4.7170 1.8811<br />

8 11.0324 0.0906 0.03489 28.6640 0.38489 2.5982 5.3515 2.0597<br />

9 14.8937 0.0671 0.02519 39.6964 0.37519 2.6653 5.8886 2.2094<br />

10 20.1066 0.0497 0.01832 54.5902 0.36832 2.7150 6.3363 2.3338<br />

11 27.1439 0.0368 0.01339 74.6967 0.36339 2.7519 6.7047 2.4364<br />

12 36.6442 0.0273 0.00982 101.8406 0.35982 2.7792 7.0049 2.5205<br />

13 49.4697 0.0202 0.00722 138.4848 0.35722 2.7994 7.2474 2.5889<br />

14 66.7841 0.0150 0.00532 187.9544 0.35532 2.8144 7.4421 2.6443<br />

15 90.1585 0.0111 0.00393 254.7385 0.35393 2.8255 7.5974 2.6889<br />

16 121.7139 0.0082 0.00290 344.8970 0.35290 2.8337 7.7206 2.7246<br />

17 164.3138 0.0061 0.00214 466.6109 0.35214 2.8398 7.8180 2.7530<br />

18 221.8236 0.0045 0.00158 630.9247 0.35158 2.8443 7.8946 2.7756<br />

19 299.4619 0.0033 0.00117 852.7483 0.35117 2.8476 7.9547 2.7935<br />

20 404.2736 0.0025 0.00087 1152.21 0.35087 2.8501 8.0017 2.8075<br />

22 736.7886 0.0014 0.00048 2102.25 0.35048 2.8533 8.0669 2.8272<br />

24 1342.80 0.0007 0.00026 3833.71 0.35026 2.8550 8.1061 2.8393<br />

25 1812.78 0.0006 0.00019 5176.50 0.35019 2.8556 8.1194 2.8433<br />

26 2447.25 0.0004 0.00014 6989.28 0.35014 2.8560 8.1296 2.8465<br />

28 4460.11 0.0002 0.00008 12740 0.35008 2.8565 8.1435 2.8509<br />

30 8128.55 0.0001 0.00004 23222 0.35004 2.8568 8.1517 2.8535<br />

32 14814 0.0001 0.00002 42324 0.35002 2.8569 8.1565 2.8550<br />

34 26999 0.00001 77137 0.35001 2.8570 8.1594 2.8559<br />

35 36449 0.00001 0.35001 2.8571 8.1603 2.8562


608 Compound Interest Factor Tables<br />

40% TABLE 28 Discrete Cash Flow: Compound Interest Factors 40%<br />

Single Payments Uniform Series Payments Arithmetic Gradients<br />

Compound Present Sinking Compound Capital Present Gradient Gradient<br />

Amount Worth Fund Amount Recovery Worth Present Worth Uniform Series<br />

n FP PF AF FA AP PA PG AG<br />

1 1.4000 0.7143 1.00000 1.0000 1.40000 0.7143<br />

2 1.9600 0.5102 0.41667 2.4000 0.81667 1.2245 0.5102 0.4167<br />

3 2.7440 0.3644 0.22936 4.3600 0.62936 1.5889 1.2391 0.7798<br />

4 3.8416 0.2603 0.14077 7.1040 0.54077 1.8492 2.0200 1.0923<br />

5 5.3782 0.1859 0.09136 10.9456 0.49136 2.0352 2.7637 1.3580<br />

6 7.5295 0.1328 0.06126 16.3238 0.46126 2.1680 3.4278 1.5811<br />

7 10.5414 0.0949 0.04192 23.8534 0.44192 2.2628 3.9970 1.7664<br />

8 14.7579 0.0678 0.02907 34.3947 0.42907 2.3306 4.4713 1.9185<br />

9 20.6610 0.0484 0.02034 49.1526 0.42034 2.3790 4.8585 2.0422<br />

10 28.9255 0.0346 0.01432 69.8137 0.41432 2.4136 5.1696 2.1419<br />

11 40.4957 0.0247 0.01013 98.7391 0.41013 2.4383 5.4166 2.2215<br />

12 56.6939 0.0176 0.00718 139.2348 0.40718 2.4559 5.6106 2.2845<br />

13 79.3715 0.0126 0.00510 195.9287 0.40510 2.4685 5.7618 2.3341<br />

14 111.1201 0.0090 0.00363 275.3002 0.40363 2.4775 5.8788 2.3729<br />

15 155.5681 0.0064 0.00259 386.4202 0.40259 2.4839 5.9688 2.4030<br />

16 217.7953 0.0046 0.00185 541.9883 0.40185 2.4885 6.0376 2.4262<br />

17 304.9135 0.0033 0.00132 759.7837 0.40132 2.4918 6.0901 2.4441<br />

18 426.8789 0.0023 0.00094 1064.70 0.40094 2.4941 6.1299 2.4577<br />

19 597.6304 0.0017 0.00067 1491.58 0.40067 2.4958 6.1601 2.4682<br />

20 836.6826 0.0012 0.00048 2089.21 0.40048 2.4970 6.1828 2.4761<br />

22 1639.90 0.0006 0.00024 4097.24 0.40024 2.4985 6.2127 2.4866<br />

24 3214.20 0.0003 0.00012 8033.00 0.40012 2.4992 6.2294 2.4925<br />

25 4499.88 0.0002 0.00009 11247 0.40009 2.4994 6.2347 2.4944<br />

26 6299.83 0.0002 0.00006 15747 0.40006 2.4996 6.2387 2.4959<br />

28 12348 0.0001 0.00003 30867 0.40003 2.4998 6.2438 2.4977<br />

30 24201 0.00002 60501 0.40002 2.4999 6.2466 2.4988<br />

32 47435 0.00001 0.40001 2.4999 6.2482 2.4993<br />

34 92972 0.40000 2.5000 6.2490 2.4996<br />

35 0.40000 2.5000 6.2493 2.4997


Compound Interest Factor Tables 609<br />

50% TABLE 29 Discrete Cash Flow: Compound Interest Factors 50%<br />

Single Payments Uniform Series Payments Arithmetic Gradients<br />

Compound Present Sinking Compound Capital Present Gradient Gradient<br />

Amount Worth Fund Amount Recovery Worth Present Worth Uniform Series<br />

n FP PF AF FA AP PA PG AG<br />

1 1.5000 0.6667 1.00000 1.0000 1.50000 0.6667<br />

2 2.2500 0.4444 0.40000 2.5000 0.90000 1.1111 0.4444 0.4000<br />

3 3.3750 0.2963 0.21053 4.7500 0.71053 1.4074 1.0370 0.7368<br />

4 5.0625 0.1975 0.12308 8.1250 0.62308 1.6049 1.6296 1.0154<br />

5 7.5938 0.1317 0.07583 13.1875 0.57583 1.7366 2.1564 1.2417<br />

6 11.3906 0.0878 0.04812 20.7813 0.54812 1.8244 2.5953 1.4226<br />

7 17.0859 0.0585 0.03108 32.1719 0.53108 1.8829 2.9465 1.5648<br />

8 25.6289 0.0390 0.02030 49.2578 0.52030 1.9220 3.2196 1.6752<br />

9 38.4434 0.0260 0.01335 74.8867 0.51335 1.9480 3.4277 1.7596<br />

10 57.6650 0.0173 0.00882 113.3301 0.50882 1.9653 3.5838 1.8235<br />

11 86.4976 0.0116 0.00585 170.9951 0.50585 1.9769 3.6994 1.8713<br />

12 129.7463 0.0077 0.00388 257.4927 0.50388 1.9846 3.7842 1.9068<br />

13 194.6195 0.0051 0.00258 387.2390 0.50258 1.9897 3.8459 1.9329<br />

14 291.9293 0.0034 0.00172 581.8585 0.50172 1.9931 3.8904 1.9519<br />

15 437.8939 0.0023 0.00114 873.7878 0.50114 1.9954 3.9224 1.9657<br />

16 656.8408 0.0015 0.00076 1311.68 0.50076 1.9970 3.9452 1.9756<br />

17 985.2613 0.0010 0.00051 1968.52 0.50051 1.9980 3.9614 1.9827<br />

18 1477.89 0.0007 0.00034 2953.78 0.50034 1.9986 3.9729 1.9878<br />

19 2216.84 0.0005 0.00023 4431.68 0.50023 1.9991 3.9811 1.9914<br />

20 3325.26 0.0003 0.00015 6648.51 0.50015 1.9994 3.9868 1.9940<br />

22 7481.83 0.0001 0.00007 14962 0.50007 1.9997 3.9936 1.9971<br />

24 16834 0.0001 0.00003 33666 0.50003 1.9999 3.9969 1.9986<br />

25 25251 0.00002 50500 0.50002 1.9999 3.9979 1.9990<br />

26 37877 0.00001 75752 0.50001 1.9999 3.9985 1.9993<br />

28 85223 0.00001 0.50001 2.0000 3.9993 1.9997<br />

30 0.50000 2.0000 3.9997 1.9998<br />

32 0.50000 2.0000 3.9998 1.9999<br />

34 0.50000 2.0000 3.9999 2.0000<br />

35 0.50000 2.0000 3.9999 2.0000


PHOTO CREDITS<br />

Chapter 1<br />

Opener: Royalty-Free/CORBIS.<br />

Chapter 2<br />

Opener: Royalty-Free/CORBIS.<br />

Chapter 3<br />

Opener: Getty Images.<br />

Chapter 4<br />

Opener: Chad Baker/Getty Images.<br />

Chapter 5<br />

Opener: © Digital Vision/PunchStock.<br />

Chapter 6<br />

Opener: Stockdisc.<br />

Chapter 7<br />

Opener: Getty Images.<br />

Chapter 8<br />

Opener: PhotoLink/Getty Images.<br />

Chapter 9<br />

Opener: Royalty-Free/CORBIS.<br />

Chapter 10<br />

Opener: © Digital Vision/PunchStock.<br />

Chapter 11<br />

Opener: (a) Royalty-Free/CORBIS; (b) Royalty-Free/CORBIS.<br />

Chapter 12<br />

Opener: Ariel Skelley/Blend Images/Getty Images.<br />

Chapter 13<br />

Opener: Randy Allbritton/Getty Images.<br />

Chapter 14<br />

Opener: Photodisc/Getty Images.<br />

Chapter 15<br />

Opener: Ryan McVay/Getty Images.<br />

Chapter 16<br />

Opener: Arthur S. Aubry/Getty Images.<br />

Chapter 17<br />

Opener: Photodisc/Getty Images.<br />

Chapter 18<br />

Opener: Rubberball/SuperStock.<br />

Chapter 19<br />

Opener: © Brand X/JupiterImages.


INDEX<br />

A<br />

A , 13, 43<br />

Absolute cell referencing, 9, 547, 550<br />

Accelerated Cost Recovery System (ACRS), 422<br />

Accelerated write-off, 416, 450–52<br />

Accounting<br />

ratios, 563–65<br />

statements, 561–63<br />

Acid-test ratio, 564<br />

Acquisition phase, 161<br />

Activity based costing (ABC), 401–03<br />

Additive weight technique, 282–83<br />

A/F factor, 46<br />

After-tax<br />

and alternative selection, 456–62<br />

cash flow, 448–50<br />

debt versus equity financing, 267<br />

and depreciation, 415, 453<br />

international, 468–70<br />

and MARR, 269, 456<br />

and present worth, 456<br />

rate of return, 458–62<br />

spreadsheet analysis, 457–58<br />

and WACC, 271<br />

After-tax replacement analysis, 462–65<br />

A/G factor, 53. See also Gradient, arithmetic<br />

Alternative depreciation system (ADS), 426–27<br />

Allocation variance, 399<br />

Alternatives<br />

cost, 131<br />

defined, 6<br />

do-nothing, 130–31<br />

independent, 130–31, 132, 157, 203, 207, 244, 247 ( see also<br />

Capital budgeting)<br />

infinite life, 138, 157–60<br />

mutually exclusive, 130–31, 132, 155, 208, 238, 248<br />

revenue, 131<br />

selection, 6<br />

service, 131<br />

in simulation, 533–40<br />

Amortization, 415<br />

Annual interest rate<br />

effective, 99–105<br />

nominal, 99–105<br />

Annual operating costs (AOC), 6, 153, 297–98<br />

and estimation, 388<br />

Annual Percentage Rate (APR), 97<br />

Annual Percentage Yield (APY), 97<br />

Annual worth<br />

advantages, 151<br />

after-tax analysis, 456–58<br />

of annual operating costs, 297–98<br />

and B/C analysis, 235, 238–42<br />

and breakeven analysis, 345–48<br />

and capital-recovery-plus-interest, 153–54<br />

equivalent uniform, 151<br />

evaluation by, 155–60<br />

and EVA, 153, 465–68<br />

and future worth, 151<br />

and incremental rate of return, 213–14<br />

of infinite-life projects, 157–60<br />

and inflation, 151, 377–78<br />

and present worth, 151<br />

and rate of return, 175, 213–14<br />

and replacement analysis, 302–06, 307–10, 462–65<br />

spreadsheet solutions, 156, 159, 458<br />

when to use, 262<br />

AOC. See Annual operating costs<br />

A/P factor, 43–45, 153<br />

Arithmetic gradient. See Gradient, arithmetic<br />

Assets. See also Book value; Depreciation; Life; Salvage value<br />

in balance sheet, 561<br />

capital recovery, 153–54<br />

return on, 564<br />

sunk cost, 295<br />

Attributes<br />

evaluating multiple, 282–83<br />

identifying, 278–79<br />

weighting, 279–81<br />

Average. See Expected value<br />

Average cost per unit, 345<br />

Average tax rate, 447<br />

B<br />

Balance sheet, 267, 561–63<br />

Base amount<br />

defined, 50<br />

and shifted gradients, 80–82<br />

Basis, unadjusted, 416<br />

B/C. See Benefit/cost ratio<br />

Before-tax rate of return<br />

and after-tax, 459<br />

calculation, 173–90<br />

Bell-shaped curve. See Normal distribution<br />

Benefit and cost difference, 236<br />

Benefit/cost ratio<br />

calculation, 235–36<br />

conventional, 235<br />

incremental analysis, 238–39<br />

modified, 235–36<br />

for three or more alternatives, 242–46<br />

for two alternatives, 238–42<br />

when to use, 262<br />

Benefits<br />

direct versus implied, 242<br />

in public projects, 231, 235, 242<br />

(beta), 274<br />

Bonds<br />

and debt financing, 271–72<br />

and inflation, 372, 385<br />

interest computation, 190<br />

payment periods, 190<br />

present worth, 191–92<br />

for public sector projects, 232<br />

rate of return, 190–92<br />

types, 191<br />

Book depreciation, 415, 417, 466<br />

Book value<br />

declining balance method, 419–22<br />

defined, 416<br />

double declining balance method, 419–22<br />

and EVA, 466


612 Index<br />

Book value (continued)<br />

MACRS method, 423<br />

versus market value, 416<br />

straight line method, 418<br />

sum-of-years-digits method, 430, 557<br />

unit-of-production method, 431<br />

Borrowed money, 267<br />

Borrowing rate, 185–87<br />

Bottom-up approach, 388–89<br />

Breakeven analysis. See also PW vs. i graph<br />

average cost per unit, 345<br />

fixed costs, 341<br />

and Goal Seek, 353–54<br />

and make-buy decisions, 341, 347<br />

and payback, 351–54<br />

and rate of return, 210–12, 460<br />

versus sensitivity analysis, 341, 485<br />

single project, 341–45<br />

spreadsheet application, 212, 352–54<br />

three or more alternatives, 348<br />

two alternatives, 345–47<br />

variable costs, 341<br />

Breakeven point, 341, 489<br />

Budgeting. See Capital budgeting<br />

Bundles, 325, 327<br />

Business ratios, 563–65<br />

C<br />

Canada, depreciation and taxes, 468<br />

Capital<br />

cost of ( see Cost of capital)<br />

cost of invested, 466<br />

debt versus equity, 27, 267<br />

description, 3<br />

limited, 268, 323<br />

unrecovered, 295<br />

Capital asset pricing model (CAPM), 274<br />

Capital budgeting<br />

description, 323–25<br />

equal life projects, 325–26<br />

linear programming, 329–31<br />

mutually exclusive bundles, 325, 327<br />

present-worth use, 325–29<br />

reinvestment assumption, 324, 328–29<br />

spreadsheet solution, 330–31<br />

unequal life projects, 327–29<br />

Capital financing. See also Cost of capital<br />

debt, 27, 267, 271–73<br />

equity, 27, 267, 273–75<br />

mixed (debt and equity), 275–77<br />

Capital gains<br />

defined, 454<br />

short-term and long-term, 454<br />

taxes for, 454<br />

Capital losses<br />

defined, 454<br />

taxes for, 454<br />

Capital rationing. See Capital budgeting<br />

Capital recovery, 153–54. See also A/P factor; Depreciation<br />

defined, 153<br />

and economic service life, 297–99<br />

and EVA, 468<br />

and inflation, 377–78<br />

and replacement analysis, 306<br />

Capital recovery factor, 43–45<br />

and equivalent annual worth, 153<br />

Capitalized cost<br />

in alternative evaluation, 138–42<br />

and annual worth, 138<br />

and public projects, 244–46<br />

CAPM. See Capital asset pricing model<br />

Carry-back and carry-forward, 454<br />

Case studies<br />

after-tax analysis, 482–83<br />

alternative description, 38<br />

annual worth, 93, 171<br />

breakeven analysis, 363–64<br />

compound interest, 70–71<br />

debt versus equity financing, 290–91, 482–83<br />

economic service life, 321<br />

energy, 36, 363–64<br />

ethics, 412–13<br />

house financing, 124–25<br />

indirect costs, 411–12<br />

inflation, 385<br />

multiple attributes, 511–13<br />

multiple interest rates, 200–01<br />

public project, 259–60, 511–13<br />

rate of return, 200–01, 385<br />

replacement analysis, 321<br />

sale of business, 227<br />

sensitivity analysis, 226, 510, 544–45<br />

simulation, 544–45<br />

social security, 149<br />

Cash flow<br />

after tax, 448–50<br />

before tax, 449–50<br />

beyond study period, 133–34<br />

continuous, 116<br />

conventional series, 180<br />

defined, 15<br />

diagramming, 16–18<br />

discounted, 129<br />

estimating, 15, 231, 387–90<br />

incremental, 203–06, 213<br />

inflow and outflow, 15<br />

net, 15<br />

and payback period, 349–51<br />

nonconventional, 180–90<br />

and public sector projects, 231<br />

recurring and nonrecurring, 139<br />

and replacement analysis, 306<br />

revenue versus cost, 131, 204<br />

and rule of signs, 180–84, 219<br />

and simulation, 533–38<br />

using actual versus incremental, 213, 240, 460<br />

zero, 78, 553, 555<br />

Cash flow after taxes (CFAT), 448–50, 456–65<br />

and EVA, 465–68<br />

Cash flow before taxes (CFBT), 448–50, 473<br />

Cash flow diagrams, 16–18<br />

partitioned, 55<br />

Cell references, spreadsheet, 29, 547, 550<br />

CFAT. See Cash flow after taxes<br />

CFBT. See Cash flow before taxes<br />

Challenger<br />

in B/C analysis, 242–46<br />

in replacement analysis, 294, 302–12, 462–64<br />

in ROR analysis, 214–19<br />

in service sector projects, 248–49<br />

China, depreciation and taxes, 468–69<br />

Code of ethics, 7–8, 251, 404, 566–68<br />

Common stock, 267, 273–75


Index 613<br />

Compound amount factors<br />

single payment (F/P), 40<br />

uniform series (F/A), 46<br />

Compound interest, 22, 24–25, 28. See also Compounding<br />

Compounding<br />

annual, 99–105<br />

continuous, 114–16<br />

and effective interest rate, 96<br />

frequency, 97–98<br />

interperiod, 113–14<br />

period, 97–98<br />

and simple interest, 21–23<br />

Compounding period<br />

continuous, 114–16<br />

defined, 97<br />

and effective annual rate, 102<br />

and payment period, 106–14<br />

Concepts, fundamental, summary, 573–76<br />

Contingent projects, 323<br />

Continuous compounding, 114–16<br />

Contracts, types, 234<br />

Conventional benefit/cost ratio, 235<br />

Conventional cash flow series, 180<br />

Conventional gradient, 51–57<br />

Corporations<br />

and capital, 4<br />

financial worth, 466–68<br />

leveraged, 275–77<br />

Cost alternative, 131, 204, 216, 457, 460<br />

Cost-effectiveness<br />

analysis, 246–50<br />

ratio, 246<br />

Cost, life-cycle, 160–63<br />

Cost-capacity equations, 394–95<br />

Cost centers, 397, 401<br />

Cost components, 387–88<br />

Cost depletion, 427–29<br />

Cost drivers, 401<br />

Cost-estimating relationships, 394–97<br />

Cost estimation<br />

accuracy, 389<br />

approaches, 388–89<br />

cost-capacity method, 394–95<br />

and cost indexes, 391–94<br />

factor method, 395–97<br />

and inflation, 12, 377<br />

unit method, 390<br />

Cost of capital<br />

and debt-equity mix, 269–71<br />

for debt financing, 271–73<br />

defined, 26, 267<br />

for equity financing, 273–75<br />

versus MARR, 26, 267<br />

weighted average, 27, 270, 275<br />

Cost of goods sold, 397, 562–63, 565<br />

Cost of invested capital, 466–67<br />

Costs. See also Capital; Incremental cash flow; Opportunity cost<br />

annual operating, 6, 153, 297–98<br />

and annual worth, 153<br />

of asset ownership, 153<br />

direct, 387, 390–97<br />

estimating, 388–97<br />

EUAW( see Annual worth)<br />

fixed, 341<br />

indirect, 387, 396, 397–404<br />

life-cycle, 160–63<br />

marginal, 300–01<br />

in public projects, 231, 235<br />

sign convention, 15, 235<br />

sunk, 295<br />

variable, 341<br />

Coupon rate, 190<br />

Cumulative cash flow sign test, 181–84, 219<br />

Cumulative distribution, 519–22<br />

Current assets, 561<br />

Current liabilities, 561–62<br />

Current ratio, 563<br />

D<br />

DB function, 420, 550–51<br />

DDB function, 420, 551<br />

Debt capital, 267<br />

Debt-equity mix, 269–71, 275–77<br />

Debt financing, 269–71<br />

on balance sheet, 267, 561<br />

costs of, 271–73<br />

and inflation, 374, 378<br />

leveraging, 275–77<br />

Debt ratio, 564<br />

Decision making<br />

attributes, 3–4, 278–83<br />

under certainty, 517<br />

and engineering economy role, 3–4<br />

under risk, 517, 518–22<br />

under uncertainty, 517<br />

Decision trees, 494–98<br />

Declining balance depreciation, 419–22<br />

in Excel, 420, 550–51<br />

Decreasing gradients, 51, 56, 83–86<br />

Defender<br />

in B/C analysis, 242–46<br />

in replacement analysis, 294, 302–12, 462–64<br />

in ROR analysis, 214–19<br />

in service sector projects, 248–49<br />

Deflation, 368–69<br />

Delphi method, 278–79<br />

Dependent projects, 323<br />

Depletion<br />

cost, 427–29<br />

percentage, 427–29<br />

Depreciation. See also Depreciation recapture; Rate of<br />

depreciation; Replacement analysis<br />

accelerated, 416, 419<br />

ACRS, 422<br />

alternative system, 426–27<br />

and amortization, 415<br />

basis, 416<br />

book, 415, 417, 466<br />

declining balance, 419–22, 550–51<br />

defined, 415<br />

double declining balance, 419–22, 551<br />

and EVA, 466<br />

general depreciation system (GDS), 426<br />

half-year convention, 416, 424, 427<br />

and income taxes, 415, 417, 445–68<br />

MACRS, 417, 422–27<br />

present worth of, 432–35<br />

property class, 426<br />

recovery period for, 416, 418, 426<br />

rate of, 416<br />

recovery rate, 416


614 Index<br />

Depreciation (continued)<br />

straight line, 418–19, 557<br />

straight line alternative, 426–27<br />

sum-of-years digits, 430, 557<br />

switching methods, 432–38, 557–58<br />

tax, 415, 417<br />

unit-of-production, 431<br />

Depreciation recapture<br />

definition, 453<br />

in replacement studies, 462–65<br />

and taxes, 453, 461<br />

Descartes’ rule, 181–84, 219<br />

Design-build contracts, 234<br />

Design stages, preliminary and detailed, 161, 163, 389<br />

Design-to-cost approach, 388–89<br />

Direct benefits, 242, 244<br />

Direct costs, 387, 390–97<br />

Disbenefits, 231, 235<br />

Disbursements, 15, 177<br />

Discount rate, 129, 232<br />

Discounted cash flow, 129<br />

Discounted payback analysis, 349<br />

Discrete cash flows<br />

compound interest factors (tables), 581–609<br />

discrete versus continuous compounding, 114–16<br />

and end-of-period convention, 15–16<br />

Disposal phase, 161<br />

Distribution. See also Probability distribution<br />

defined, 519<br />

normal, 520, 531–33<br />

standard normal, 531–33<br />

triangular, 520, 522<br />

uniform, 520–21, 535, 538<br />

Dividends<br />

bonds, 190, 271–72<br />

stocks, 273–75<br />

Dominance, 248–49<br />

Do-nothing alternative<br />

and B/C analysis, 239, 242<br />

defined, 6, 131<br />

and independent projects, 132, 207, 244, 325–27<br />

and present worth, 130–31<br />

and rate of return, 207, 214, 216<br />

Double declining balance, 419–22<br />

in Excel, 420–22, 551<br />

in switching, 432–38<br />

and taxes, 451–52<br />

Dumping, 368<br />

E<br />

Economic equivalence. See Equivalence<br />

Economic service life (ESL), 294, 296–302<br />

Economic value added, 153, 465–68<br />

EFFECT function, 103, 551<br />

Effective interest rate<br />

annual, 99–100<br />

for any time period, 105–06<br />

of bonds, 191–92<br />

and compounding periods, 100, 105<br />

for continuous compounding, 114–16<br />

defined, 96<br />

and nominal rate, 96–97<br />

Effective tax rate, 447, 462<br />

Efficiency ratios, 563<br />

End-of-period convention, 15–16<br />

<strong>Engineering</strong> economy<br />

Concepts, summary, 573–76<br />

defined, 3<br />

study approach, 4–7<br />

terminology and symbols, 13<br />

Equal service requirement, 131, 151, 213, 217, 240, 457<br />

Equity financing, 26, 267<br />

cost of, 273–75<br />

Equivalence<br />

calculations without tables, 569–72<br />

compounding period greater than payment period, 112–14<br />

compounding period less than payment period, 107–12<br />

defined, 19–21<br />

Equivalent uniform annual cost. See Annual worth<br />

Equivalent uniform annual worth. See Annual worth<br />

Error distribution. See Normal distribution<br />

Estimation<br />

and alternatives, 6<br />

of cash flow, 6, 15–19<br />

of costs, 388–97<br />

and sensitivity analysis, 485, 490–91<br />

before tax ROR, 459<br />

Ethics<br />

and cost estimation, 403–04<br />

in public sector, 250–51<br />

overview, 7–10<br />

EUAC. See Annual worth<br />

EUAW. See Annual worth<br />

Evaluation criteria, 4, 262<br />

Evaluation method, 261–64<br />

Excel. See also Spreadsheet, usage in examples<br />

absolute cell reference, 9, 547, 550<br />

basics, 547–50<br />

charts, 548–49<br />

displaying functions, 30<br />

embedding functions, 75, 156–57<br />

error messages, 560<br />

functions, in engineering economy, 28, 550–58<br />

Goal Seek tool, 558–59<br />

introduction, 27–30, 547–50<br />

and linear programming, 330–31<br />

random number generation, 556<br />

Solver tool, 559–60<br />

spreadsheet layout, 549–50<br />

Expected value<br />

computation, 492–94, 526–27, 530<br />

and decisions under risk, 517<br />

and decision trees, 497–98<br />

defined, 492, 526<br />

and real options, 501–02<br />

in simulation, 538–39<br />

Expenses, operating, 6, 153, 297–98<br />

External rate of return, 185–90. See also Rate of return<br />

F<br />

F, 13<br />

F/A factor, 46<br />

F/G factor, 54<br />

Face value, of bonds, 190<br />

Factor method estimation, 395–97<br />

Factors, compound interest<br />

annual worth, 44, 46, 53<br />

arithmetic gradient, 52–54


Index 615<br />

capital recovery, 44<br />

continuous compound interest, 114–16<br />

derivations, 39–46, 52–54<br />

equivalence without, 569–72<br />

future worth, 40, 46, 54<br />

geometric gradient, 58–59<br />

interpolation, 48–50<br />

multiple, 73–86<br />

notation, 40, 44, 47, 53, 59<br />

present worth, 40, 43, 52, 58<br />

single payment, 39–42<br />

sinking fund, 46<br />

tables, 581–609<br />

uniform series, 43–48, 53<br />

Factory cost, 397, 562<br />

Financial worth of corporations, 466, 561<br />

First cost<br />

and depreciation, 416<br />

description, 6, 15<br />

and estimation, 388–97<br />

in replacement studies, 294–95, 462<br />

Fiscal year, 561<br />

Fixed assets, 561<br />

Fixed costs, 341<br />

Fixed income investment, 372, 385. See also Bonds<br />

Fixed percentage method. See Declining balance depreciation<br />

F/P factor, 40. See also Single payment factors<br />

Future worth<br />

and annual worth, 151<br />

and effective interest rate, 100<br />

evaluation by, 137<br />

and inflation, 374–77<br />

and multiple rates of return, 186–87<br />

from present worth, 137<br />

of shifted series, 73, 77–78<br />

and spreadsheet solutions, 42, 48<br />

when to use, 262<br />

FV function, 28, 552<br />

and shifted uniform series, 78–79<br />

and single payments, 41<br />

and uniform series, 46<br />

G<br />

Gains and losses, 454<br />

Gaussian distribution. See Normal distribution<br />

General depreciation system (GDS), 426<br />

Geometric gradient<br />

defined, 58<br />

factors, 58–59<br />

shifted, 82–86<br />

Geometric series, and equivalence, 570–72<br />

Goal Seek, 56–57, 86, 156–57, 189, 312, 354, 458, 558–59<br />

Government projects. See Public sector projects<br />

Gradient, arithmetic<br />

base amount, 50<br />

conventional, 51<br />

decreasing, 51, 56<br />

defined, 50<br />

factors, 52–54<br />

increasing, 51–55<br />

shifted, 80–82, 83–86<br />

spreadsheet use, 57<br />

Graduated tax rates, 446–47, 448<br />

Gross income, 445, 448<br />

H<br />

Half-year convention, 416, 424, 427<br />

Highly leveraged corporations, 275–77<br />

Hurdle rate. See Minimum attractive rate of return<br />

Hyperinflation, 368, 377<br />

I<br />

IF function, 189, 245, 552<br />

Implied benefits, 240, 242<br />

Income<br />

estimated annual, 6, 15<br />

gross, 445, 448<br />

net operating, 446, 459<br />

taxable, 446, 454<br />

Income statement, 562<br />

basic equation, 562<br />

ratios, 564<br />

Income tax, 445–70<br />

average tax rate, 447<br />

and capital gains and losses, 454<br />

and cash flow, 448–50<br />

corporate, 447<br />

and debt financing, 271–73<br />

defined, 445<br />

and depreciation, 450–56<br />

effective rates, 447<br />

international, 468–70<br />

negative, 449<br />

present worth of, 451–53<br />

and rate of return, 458–62<br />

rates, 446–48<br />

and replacement studies, 462–65<br />

state, 445, 447<br />

tax savings, 449<br />

and taxable income, 446<br />

terminology, 445–46<br />

Incremental benefit/cost analysis<br />

for three or more alternatives, 242–46<br />

for two alternatives, 238–42<br />

Incremental cash flow<br />

and benefit/cost analysis, 238–39<br />

calculation, 203–06<br />

and rate of return, 207–10, 213–14<br />

Incremental rate of return<br />

for multiple alternatives, 214–18<br />

for two alternatives, 213–14<br />

unequal lives, 207, 213<br />

Independent projects<br />

AW evaluation, 157<br />

B/C evaluation, 244<br />

and capital budgeting, 323–31<br />

definition, 130<br />

and do-nothing alternative, 130, 132, 157, 207, 244, 325, 327<br />

and incremental cash flow, 207, 244<br />

PW evaluation, 132<br />

ROR evaluation, 207<br />

service project evaluation, 247–48<br />

Indexing, income taxes, 448<br />

Indirect costs<br />

and activity-based costing, 401–03<br />

allocation variance, 399<br />

in cost of goods sold statement, 562–63<br />

defined, 398


616 Index<br />

Indirect costs (continued)<br />

and factor method, 396–97<br />

rates, 398–99<br />

Infinite life, 138–40, 157–60, 231, 244–45<br />

Inflation<br />

assumption, PW and AW analysis, 134, 151<br />

and capital recovery, 377–78<br />

constant-value, 367<br />

definition, 12, 367<br />

versus deflation, 368–69<br />

and dumping, 368<br />

and future worth, 374–77<br />

high, 377<br />

impact, 12, 367–68<br />

and interest rates, 368<br />

market adjusted, 368<br />

and MARR, 368, 375–77<br />

and present worth, 369–74<br />

and sensitivity analysis, 485<br />

Initial investment. See First cost<br />

Installment financing, 175<br />

Intangible factors, 6. See also Multiple attribute evaluation<br />

Integer linear programming, 329–31<br />

Interest. See also Interest rate(s)<br />

compound, 22, 24, 29<br />

continuous compounding, 114–16<br />

defined, 10<br />

interperiod, 112–14<br />

rate, 10, 12<br />

simple, 21, 24<br />

Interest period, 10, 12<br />

Interest rate(s). See also Effective interest rate; Rate of return<br />

and breakeven analysis, 345<br />

definition, 10–12<br />

Excel functions, 61, 177, 553, 557<br />

expressions, 98<br />

inflation-adjusted, 368, 370<br />

inflation free (real), 368, 374<br />

interpolation, 61–63<br />

market, 368, 370<br />

multiple, 180–84<br />

nominal versus effective, 96–99<br />

for public sector, 232<br />

and risk, 267, 274<br />

and sensitivity analysis, 485<br />

on unrecovered balance (ROR), 173–75<br />

varying over time, 116–17<br />

Interest tables<br />

discrete compounding, 581–609<br />

interpolation, 48–50<br />

Internal rate of return. See also Rate of return<br />

and annual worth, 175<br />

definition, 173<br />

versus external ROR, 185<br />

and present worth, 175<br />

spreadsheet solution, 177<br />

International aspects<br />

contracts, 234<br />

corporate taxes, 468–70<br />

cost estimation, 388–89<br />

deflation, 368–69<br />

depreciation, 416–17, 470–72<br />

dumping, 368<br />

inflation aspects, 368, 377<br />

value-added tax, 470–72<br />

Interperiod interest, 112–14<br />

Interpolation, in interest rate tables, 48–50<br />

Inventory turnover ratio, 564–65<br />

Invested capital, cost of, 466–67<br />

Investment(s). See also First cost<br />

extra, 206–07, 214–18<br />

fi xed income, 372, 385<br />

net, 188<br />

permanent, 138–41, 157–60<br />

Investment rate, 185–90<br />

IPMT function, 552<br />

IRR function, 61, 63, 177–79, 217, 460, 553<br />

L<br />

Land, 416<br />

Lang factors, 395<br />

Least common multiple<br />

and annual worth, 151–52<br />

assumptions, 134<br />

in evaluation methods, 262–63<br />

and future worth, 137<br />

and incremental cash flow, 204<br />

and incremental rate of return, 207–09, 213–14, 218–19<br />

and independent projects, 324, 329<br />

and present worth, 133–37<br />

in spreadsheet analysis, 205<br />

versus study period, 134, 136<br />

Leveraging, 275–77<br />

Liabilities, 561<br />

Life<br />

finite, 141<br />

and income taxes, 452–53<br />

infinite or very long, 138–41, 157–58<br />

minimum cost, 297<br />

recovery period, 416<br />

in simulation, 534–35, 538<br />

unknown, 61, 63–64<br />

useful, 6, 416<br />

Life cycle, and annual worth, 160–62<br />

Life-cycle costs, 160–63<br />

Likert scale, 281<br />

Linear programming, 329–31<br />

Lives<br />

equal, 131, 155, 204, 240<br />

perpetual, 138, 157<br />

unequal, 133, 155, 204, 207<br />

Loan repayment, 24–25<br />

M<br />

MACRS (Modified Accelerated Cost Recovery System)<br />

in CFAT example, 450<br />

depreciation rates, 423–24<br />

PW of, depreciation, 432<br />

recovery period, 423, 426–27<br />

spreadsheet function, 424, 558<br />

straight line alternative (ADS), 426–27<br />

switching, 432, 435–38<br />

U.S., required, 417, 422<br />

Maintenance and operating (M&O) costs, 6, 153. See also Annual<br />

operating cost<br />

Make-or-buy decisions, 341, 347. See also Breakeven analysis<br />

Marginal costs, 300–01<br />

Marginal tax rates, 446–48


Index 617<br />

Market interest rate, 368<br />

Market value<br />

and depreciation, 416<br />

in ESL analysis, 297, 300–01<br />

estimating, 301–02<br />

in replacement analysis, 294, 302–06<br />

as salvage value, 153, 294<br />

and study period, 134<br />

MARR. See Minimum attractive rate of return<br />

Mean. See Expected value<br />

Mean squared deviation, 528<br />

Measure of worth, 4, 6, 262<br />

Median, 527<br />

Mexico, depreciation and taxes, 469<br />

Minimum attractive rate of return<br />

after-tax, 269, 456–58<br />

and bonds, 192<br />

and capital budgeting, 323–27, 328–29<br />

definition, 26, 267<br />

establishing, 26–27, 267–69, 275<br />

as hurdle rate, 26<br />

inflation-adjusted, 368, 375–76<br />

and rate of return, 203, 206, 208, 213, 214–18<br />

and reinvestment, 324, 328–29<br />

in sensitivity analysis, 485–87<br />

before tax, 269<br />

and WACC, 27, 267, 275<br />

Minimum cost life. See Economic service life<br />

MIRR function, 186, 553–54<br />

M&O costs. See Annual operating costs; Maintenance and<br />

operating costs<br />

Mode, 522, 527<br />

Modified benefit/cost ratio, 235–36<br />

Modified ROR approach, 185–87<br />

Monte Carlo simulation, 533–39<br />

Most likely estimate, 490–91<br />

Multiple alternatives<br />

breakeven analysis, 348<br />

incremental benefit/cost analysis, 242–46<br />

incremental rate of return, 214–18<br />

Multiple attribute evaluation, 278–83<br />

Multiple rate of return<br />

definition, 179–80<br />

determining, 181–84<br />

presence of, 180–81<br />

removing, 184–90<br />

Municipal bonds, 190–92<br />

Mutually exclusive alternatives, 130–31<br />

and annual worth, 155–57<br />

and B/C, 238–44<br />

evaluation method selection, 261–64<br />

and present worth, 132–37<br />

and rate of return, 208–10<br />

and service projects, 247–49<br />

N<br />

Natural resources. See Depletion<br />

Net cash flow, 15<br />

Net operating income (NOI), 446, 459<br />

Net investment procedure, 187–90<br />

Net operating profit after taxes (NOPAT), 446, 466<br />

Net present value. See NPV function; Present worth<br />

Net profit after taxes (NPAT), 446<br />

Net worth, 561<br />

NOMINAL function, 103, 554<br />

Nominal interest rate<br />

annual, 96, 103<br />

of bonds, 190–92<br />

definition, 96<br />

and effective rates, 96–98<br />

Nonconventional cash flow series, 180–84<br />

Noneconomic factors, 6<br />

Nonowner’s viewpoint, 295<br />

Nonrecurring cash flows, 139<br />

Nonsimple cash flow series, 180–84<br />

No-return (simple) payback, 349–50<br />

Normal distribution, 520, 531–33<br />

Norstrom’s criterion, 181–84, 219, 460<br />

Notation for factors, 40, 44, 47, 53, 59<br />

NPER function, 554–55<br />

and payback, 350, 353<br />

and unknown n, 61, 63–64<br />

NPV function, 555<br />

for arithmetic gradient, 57, 86<br />

embedding in PMT, 75, 156<br />

geometric gradients 86<br />

independent projects, 328<br />

and present worth, 135–36<br />

in PW vs. i graphs, 178–79<br />

sensitivity analysis, 486–87, 489<br />

and shifted series, 75, 86<br />

NSPE (National Society of Professional Engineers), 7,<br />

404, 566–68<br />

O<br />

Obsolescence, 294<br />

One-additional-year replacement study, 302–05<br />

Operating costs. See Annual operating costs<br />

Operations phase, 161, 163<br />

Opportunity cost, 27, 267, 324<br />

and replacement analysis, 306<br />

Optimistic estimate, 490–91<br />

Order of magnitude, 389<br />

Overhead rates. See Indirect costs<br />

Owner’s equity, 267, 561<br />

P<br />

P, 13, 40<br />

P/A factor,43, 53. See also Geometric gradient;<br />

Uniform series<br />

Payback analysis<br />

and breakeven analysis, 351–52<br />

calculation, 349–51<br />

definition, 348–49<br />

limitations, 349<br />

spreadsheet analysis, 350–54<br />

Payment period<br />

of bonds, 190<br />

defined, 106<br />

equals compounding period, 106, 107, 109–11<br />

longer than compounding period, 109<br />

shorter than compounding period, 112–14<br />

single amount, 107–09<br />

Payout period. See Payback analysis<br />

Percentage depletion. 427–29<br />

Permanent investments, 138–41, 157–58<br />

Personal property, 416, 423, 426


618 Index<br />

Perspective<br />

for public sector analysis, 232–34<br />

for replacement analysis, 295<br />

Pessimistic estimate, 490–91<br />

P/F factor, 40<br />

P/G factor, 53. See also Gradient, arithmetic<br />

Phaseout phase, 161<br />

Planning horizon. See Study period<br />

PMT function, 28, 555<br />

and after-tax analysis, 457<br />

and annual worth, 44, 155–57<br />

and arithmetic gradient, 54<br />

and capital recovery, 154<br />

and economic service life, 298–99<br />

and embedded NPV, 75, 80, 156<br />

and geometric gradient, 59<br />

and random single amounts, 79–80<br />

and shifted series, 75–76<br />

and sinking fund factor, 44<br />

and uniform series, 75, 79<br />

Point estimates, 15, 517<br />

Power law and sizing model, 394–95<br />

PPMT function, 555–56<br />

Preferred stock, 267, 273<br />

Present value. See Present worth<br />

Present worth<br />

after-tax analysis, 456–58<br />

and annual worth, 151<br />

assumptions, 134<br />

and B/C analysis, 235<br />

of bonds, 191–92<br />

and breakeven analysis, 345–46<br />

and capital budgeting, 325–29<br />

of depreciation, 432<br />

for equal lives, 132–33<br />

evaluation method, 261–62<br />

geometric gradient series, 82–86<br />

income taxes, 451–53<br />

and independent projects, 325–29<br />

index, 332<br />

and inflation, 369–74<br />

and multiple interest rates, 181–84<br />

and profitability index, 237<br />

and rate of return, 175–76, 182–84, 458–62<br />

and sensitivity analysis, 486–90<br />

in shifted series, 73, 76, 80–86<br />

in simulation, 534–39<br />

single-payment factor, 40–41<br />

for unequal lives, 133–37<br />

Present worth factors<br />

gradient, 50–53, 58–60<br />

single payment factor, 40–41<br />

uniform series, 43–45<br />

Probability<br />

in decision trees, 495–98<br />

defined, 492, 518<br />

and expected value, 492–94, 526–27<br />

and standard deviation, 528<br />

Probability distribution<br />

of continuous variables, 519–22<br />

defined, 519<br />

of discrete variables, 519–20<br />

properties, 526–28, 530<br />

and samples, 523–26<br />

in simulation, 533–39<br />

Probability node, 495<br />

Productive hour rate, 399<br />

Profitability index, 237, 332<br />

Profitability ratios, 563<br />

Profit-and-loss statement, 562<br />

Project net-investment, 187–90<br />

Property class, 426<br />

Property of independent random variables, 533<br />

Public-private partnerships, 234–35<br />

Public sector projects, 230–35<br />

and annual worth, 157–58<br />

B/C analysis, 235–38<br />

capitalized cost, 138–42<br />

characteristics, 231–32<br />

design-build contracts, 234–35<br />

profitability index, 237<br />

public-private partnerships, 234–35<br />

Purchasing power, 367, 374, 376<br />

PV function, 28, 44, 556<br />

versus NPV function, 556<br />

and present worth, 41, 44<br />

and single payment, 41<br />

and uniform series present worth, 44<br />

PW vs. i graph, 177–79, 182–84, 210–13, 460–62<br />

R<br />

RAND function, 523–25, 556<br />

Random numbers, 523–25, 556<br />

generation, 538, 556<br />

Random samples, 523–26, 535–39<br />

Random variable<br />

continuous, 519–20, 525<br />

cumulative distribution, 519–22, 525–26<br />

defined, 518<br />

discrete, 519–20, 523<br />

expected value, 526, 528, 530<br />

probability distribution of, 519<br />

standard deviation, 527–28, 530<br />

Range, 15, 530<br />

Rank and rate technique, 282<br />

Ranking inconsistency, 213, 216, 462<br />

RATE function, 61–62, 177, 557<br />

Rate of depreciation<br />

declining balance, 419<br />

defined, 416<br />

MACRS, 422–23, 435–38<br />

straight line, 418, 420<br />

sum-of-years digits, 430<br />

Rate of return. See also Incremental rate of return<br />

after-tax, 458–62<br />

and annual worth, 175, 213–14, 218<br />

of bonds, 190–92, 272<br />

breakeven, 210–12, 460<br />

in capital budgeting, 332–34<br />

cautions, 179–80<br />

on debt capital, 272–73, 276<br />

defined, 12, 173, 175<br />

on equity capital, 273–74, 276<br />

evaluation method, 261–62<br />

external, 185–90<br />

on extra investment, 206<br />

incremental, 206, 207–10<br />

and independent projects, 207<br />

and inflation, 12–13, 368, 374–75


Index 619<br />

installment financing, 175<br />

internal, 173, 175, 185<br />

minimum attractive ( see Minimum attractive rate of return)<br />

modified ROR approach, 185–87<br />

multiple, 180–90<br />

and mutually exclusive alternatives, 206, 207–18<br />

and present worth, 175, 177–79, 207–09, 218<br />

ranking inconsistency, 213, 216, 462<br />

return on invested capital (ROIC) approach, 185, 187–90<br />

spreadsheet solution, 177, 179, 211–13, 216–19<br />

Ratios, accounting, 563–65<br />

Real interest rate, 368, 370, 374–75<br />

Real options, 498–503<br />

and decision trees, 500<br />

definition, 499<br />

Real property, 416, 423–24, 426<br />

Recovery period<br />

defined, 416<br />

effect on taxes, 452–53<br />

MACRS, 423–24, 426–27<br />

straight line option, 426<br />

Recovery rate. See Rate of depreciation<br />

Recurring cash flows, 139<br />

Reinvestment, assumption in capital budgeting,<br />

324, 328–29<br />

Reinvestment rate. See Investment rate<br />

Repayment of loans, 24–25<br />

Replacement analysis, 292–313, 462–65<br />

after-tax, 462–65<br />

annual worth, 295, 302–06<br />

and capital losses, 454, 462<br />

cash flow approach, 306<br />

depreciation recapture, 462–64<br />

and economic service life, 296–99, 305<br />

first costs, 294–96<br />

and marginal costs, 300–01<br />

market value, 294, 301<br />

need for, 294<br />

one-additional year, 302–05<br />

opportunity cost approach, 306<br />

overview, 302<br />

and study periods, 307–12<br />

sunk costs, 295<br />

terminology, 294<br />

viewpoint, 295<br />

Replacement life. See Economic service life<br />

Replacement value, 312<br />

Retained earnings, 27, 267, 273<br />

Retirement life. See Economic service life<br />

Return on assets ratio, 564<br />

Return on invested capital, 187–90<br />

Return on investment (ROI), 12, 173. See also Rate of return<br />

Return on sales ratio, 564<br />

Revenue alternatives, 131, 204, 214, 237, 460<br />

Risk<br />

and debt-equity mix, 275–77<br />

and decision making, 517, 526–30, 533–39<br />

and decision trees, 494–98<br />

description, 515<br />

and MARR, 267–69<br />

and payback analysis, 349<br />

and random sampling, 523<br />

and real options, 498–502<br />

Risk-free investment, 26, 274<br />

ROI. See Return on investment<br />

Root mean square deviation, 528<br />

ROR. See Rate of return<br />

Rule of signs, 181<br />

S<br />

Safe investment, 26, 190, 274<br />

Sales, return on, 564<br />

Salvage value. See also Market value<br />

and capital recovery, 153, 297<br />

defined, 6, 153<br />

and depreciation, 416, 418, 420, 423, 430<br />

and market value, 294, 297–98<br />

and public projects, 235<br />

in PW analysis, 132, 134–37<br />

in replacement analysis, 294, 297, 303–06, 464<br />

and trade-in value, 294, 464<br />

Sampling, 523–26<br />

Savings, tax, 449, 463–65<br />

Scatter charts. See xy Excel charts<br />

Screening projects, 349, 351<br />

Section 179 deduction, 417<br />

Section 1231 transactions, 454<br />

Security, defined, 274<br />

Sensitivity analysis. See also Breakeven analysis<br />

description, 485<br />

and Excel cell referencing, 29, 547<br />

of one parameter, 485–87<br />

spider graph, 488<br />

with three estimates, 490–91<br />

two alternatives, 488–90<br />

Service alternative. See Cost alternative<br />

Service sector projects<br />

analysis, 246–50<br />

definition, 246<br />

dominance, 248–49<br />

Shifted gradients, 80–86<br />

Shifted series, 73–80<br />

Sign changes, number of, 181–84<br />

Simple cash flow series, 180<br />

Simple interest, 21–23<br />

Simulation, Monte Carlo, 517, 533–40<br />

Single payment compound amount (F/P) factor, 40<br />

Single payment factors, 39–42<br />

Single payment present worth (P/F) factor, 40<br />

Sinking fund (A/F) factor, 46<br />

SLN function, 418, 557<br />

Social discount rate, 232<br />

Solvency ratios, 563<br />

Solver, 330–31, 559–60<br />

Spreadsheet, usage in examples. See also Excel<br />

annual worth, 155–56, 159, 218, 299, 301, 304,<br />

305, 353, 458, 467<br />

B/C analysis, 245<br />

breakeven analysis, 352, 353–54<br />

cash flow after tax (CFAT), 450, 456, 458, 461, 462<br />

compound interest, 29–30<br />

depreciation, 422, 425, 434<br />

EVA, 467<br />

and factor values, 49<br />

independent projects, 328, 331<br />

inflation, 372<br />

layout, 549–50<br />

multiple attributes, 283<br />

nominal and effective interest, 103–04


620 Index<br />

Spreadsheet (continued)<br />

present worth, 136, 209, 212, 218, 311, 354, 458,<br />

461, 489, 501, 539<br />

rate of return, 63, 182, 184, 189, 192, 209, 212,<br />

217, 218, 461, 462, 501<br />

replacement analysis, 299, 301, 304, 305, 311, 465<br />

replacement value, 312<br />

sensitivity analysis, 487, 489<br />

simulation, 538–39<br />

Staged funding, 494–503<br />

Standard deviation<br />

for continuous variable, 530<br />

definition, 527–28<br />

for discrete variable, 528–29<br />

Standard normal distribution, 531–33<br />

Stocks<br />

CAPM model, 274<br />

common, 267, 274<br />

in equity financing, 273–75, 276<br />

preferred, 267, 273<br />

Straight line alternative, in MACRS, 426–27<br />

Straight line depreciation, 418–19<br />

Straight line rate, 418<br />

Study period<br />

and AW evaluation, 155<br />

and equal service, 133<br />

and FW analysis, 137<br />

and PW evaluation, 133–34<br />

and replacement analysis, 302, 307–11<br />

and salvage value, 153<br />

spreadsheet example, 136, 310–11<br />

Sum-of-years digits depreciation, 430<br />

Sunk costs, 295<br />

SYD function, 430, 557<br />

System, phases of, 160–61<br />

T<br />

Tax depreciation, 415–16, 417<br />

Taxable income, 446–70<br />

and CFAT, 448–50, 456<br />

and depreciation, 446, 450–53<br />

negative, 449, 463<br />

and taxes, 446–48, 463–65<br />

Taxes. See After-tax; Income tax; Taxable income<br />

Time, 13<br />

Time value of money<br />

defined, 4<br />

and equivalence, 19<br />

factors to account for, 39–61<br />

and no-return payback, 349–50<br />

Total cost relation, 342–45. See also Breakeven analysis<br />

Trade-in value, 6, 294, 416. See also Market value; Salvage value<br />

Treasury securities, 26, 190<br />

Triangular distribution, 520, 522<br />

U<br />

Unadjusted basis, 416<br />

Uncertainty, 515, 517<br />

Uniform distribution, 520–21, 535, 538<br />

Uniform gradient. See Gradient, arithmetic<br />

Uniform percentage method. See Declining balance depreciation<br />

Uniform series<br />

compound amount (F/A) factor, 46<br />

compounding period greater than payment period, 112–14<br />

compounding period less than payment period, 109–12<br />

description, 13<br />

present worth (P/A) factor, 43<br />

shifted, 73–80<br />

Unit method, 390–91<br />

Unit-of-production depreciation, 431<br />

Unknown interest rate, 61–63<br />

Unknown years (life), 61, 63–64<br />

Unrecovered balance, 173–74<br />

V<br />

Value, resale, 6. See also Salvage value; Trade-in value<br />

Value added analysis, after tax. See Economic value added<br />

Value-added tax, 470–72<br />

Variable. See Random variable<br />

Variable costs, 341<br />

Variance<br />

in cost allocation, 399<br />

description, 528<br />

formula for, 528, 530<br />

and normal distribution, 531<br />

VDB function, 424, 433–34, 557–58<br />

W<br />

WACC. See Weighted average cost of capital<br />

Websites, 580<br />

Weighted attribute method, 282–83<br />

Weighted average cost of capital, 27, 270–71, 275–77<br />

Working capital, 563<br />

Worth, measures of, 4, 6, 129, 534<br />

X<br />

xy Excel charts, 182, 184, 212, 352, 461, 487, 548–49<br />

Y<br />

Year(s)<br />

and end-of-period convention, 15–16<br />

fiscal versus calendar, 561<br />

half-year convention, 416, 424, 427<br />

symbols, 13<br />

unknown, 61, 63–64


Format for Spreadsheet Functions on Excel ©<br />

Present worth: Contents of ( )<br />

PV(i%, n, A, F)<br />

NPV(i%,second_cell:last_cell) first_cell<br />

Future worth:<br />

FV(i%, n, A, P)<br />

Annual worth:<br />

PMT(i%, n, P, F)<br />

PMT(i%, n, NPV)<br />

Number of periods (years):<br />

NPER(i%, A, P, F)<br />

for constant A series; single F value<br />

for varying cash flow series<br />

for constant A series; single P value<br />

for single amounts with no A series<br />

to find AW from NPV; embed NPV function<br />

for constant A series; single P and F<br />

(Note: The PV, FV, and PMT functions change the sense of the sign. Place a minus in front of the<br />

function to retain the same sign.)<br />

Rate of return:<br />

RATE(n, A, P, F)<br />

IRR(first_cell:last_cell)<br />

Interest rate:<br />

EFFECT(r%, m)<br />

NOMINAL(i%, m)<br />

Depreciation:<br />

SLN(P, S, n)<br />

DDB(P, S, n, t, d)<br />

DB(P,S,n,t)<br />

for constant A series; single P and F<br />

for varying cash flow series<br />

for nominal r, compounded m times per period<br />

for effective annual i, compounded m times<br />

per year<br />

straight line depreciation for each period<br />

double declining balance depreciation for<br />

period t at rate d (optional)<br />

declining balance, rate determined by the<br />

function<br />

Relations for Discrete Cash Flows with End-of-Period Compounding<br />

Factor Notation<br />

Sample Cash Flow<br />

Type Find/Given and Formula Relation Diagram<br />

Single<br />

Amount<br />

Uniform<br />

Series<br />

Arithmetic<br />

Gradient<br />

Geometric<br />

Gradient<br />

FP<br />

Compound<br />

amount<br />

PF<br />

Present<br />

worth<br />

PA<br />

Present<br />

worth<br />

AP<br />

Capital<br />

recovery<br />

FA<br />

Compound<br />

amount<br />

AF<br />

Sinking<br />

fund<br />

P G G<br />

Present<br />

worth<br />

A G G<br />

Uniform<br />

series<br />

P g A 1 and g<br />

Present<br />

worth<br />

(FP,i,n) (1 i) n<br />

(PF,i,n) <br />

1<br />

(1 i)<br />

n<br />

F P(FP,i,n)<br />

P F(PF,i,n)<br />

(Sec. 2.1)<br />

(PA,i,n) (1 i)n 1<br />

i(1 i)<br />

n<br />

P A(PA,i,n)<br />

(AP,i,n) <br />

i(1 i) n<br />

<br />

(1 i) n 1<br />

(FA,i,n) (1 i)n 1<br />

i<br />

(AF,i,n) <br />

i<br />

<br />

(1 i)<br />

n<br />

1<br />

A P(AP,i,n)<br />

(Sec. 2.2)<br />

F A(FA,i,n)<br />

A F(AF,i,n)<br />

(Sec. 2.3)<br />

(PG,i,n) (1 i)n in 1<br />

<br />

i 2 (1 i) n P G G(PG,i,n)<br />

(AG,i,n) 1 n<br />

<br />

i (1 i)<br />

n<br />

1<br />

A G G(AG,i,n)<br />

(Gradient only) (Sec. 2.5)<br />

<br />

A 1 [ 1 ( 1 g n<br />

———<br />

1 i )<br />

———————<br />

P g i g<br />

A<br />

n<br />

1 ———<br />

1 i<br />

]<br />

g i<br />

g i<br />

(Gradient and base A 1 ) (Sec. 2.6)<br />

P G<br />

0<br />

P<br />

0<br />

P<br />

1 2 n–1<br />

…<br />

A<br />

A<br />

1 2<br />

…<br />

…<br />

0 1 2 n–1<br />

…<br />

A<br />

A … A<br />

…<br />

0 1 2 3<br />

G<br />

2G<br />

P g<br />

A<br />

n–1<br />

F<br />

A<br />

F<br />

n<br />

A<br />

n<br />

n<br />

A G A G A G<br />

… A G A G<br />

…<br />

n<br />

(n–1) G<br />

A 1<br />

A 1 (1+g) A 1 (1+g)n–1<br />

0 1 2<br />

…<br />

…<br />

n–1<br />

n<br />

VBD(P,0, n,MAX(0, t1.5),<br />

MIN(n, t0.5), d)<br />

MACRS depreciation for year t at rate d for<br />

DDB or DB method<br />

Logical IF function:<br />

IF(logical_test,value_if_true,value_if_false)<br />

for logical two-branch operations

Hooray! Your file is uploaded and ready to be published.

Saved successfully!

Ooh no, something went wrong!